Anda di halaman 1dari 544

ASIR SURGICAL RESIDENCY PROGRAM

Presents:

FIRST EDITION
1422-2002

This project was raised after an idea by:


Dr. Gharama Al-Shehri (surgical consultant)

Developed and typed by:


Dr. Ghazi Al-Shumrani (intern)
INTRODUCTION
In The Name of Allah, The Most Gracious, The Most Merciful

The surgical field is a live specialty that has a good outcome. Unfortunately, many
people may believe that surgery is a branch that does not need any effort more than
training and practicing operations. This is not enough, because theoretical knowledge is
essential for any medical field including surgery. We, as doctors, should always keep in
mind that famous word saying: “to study the phenomena of disease without books is to
sail an uncharted sea, while to study books without patients is not to go to sea at all”.
For the aim of consolidation of the theoretical knowledge in an easier way than
reading the text books, and for preparing for surgical written examinations; this MCQs
bank was prepared. The idea was raised by Dr. Gharama Al-Shehri, the surgical
residency program director in Asir Central Hospital. The idea is that any surgical
resident reads the major, most famous five or six surgical MCQs books should pass the
written examinations especially the first part or first step examination.
The beginning of any thing is the most difficult part of its entire course. During
developing of this project, I have faced many difficulties. The most difficult difficulties
were the technical ones. My initial impression about MCQs books that are created on
CD-ROM is that the all questions with their answers in a certain topic are going to be in
one page. What I was faced with is totally the opposite. I found each question is located
in one page with its answer is in another page. That is why I spent about seven weeks to
develop this project, while I was expecting it not to take more than seven days!
As just a beginning of this project, almost all the MCQs in two major surgical
MCQs books have been included; those are Sabiston and Greenfield. Other books were
about to be included but I was faced with technical difficulties, that is why those books
need to be typed (manually). I organized this book in a way similar to the surgical book
so called “RUSH”, as I find it very famous and widely accepted by surgical residents.
Because of the short time before a coming written surgical examination, I believed that
this MCQs bank should be distributed at this time.
I think the most important success gained at this time, is that we developed a
backbone for this project. It is going to be much easier in the future to add to and modify
this MCQs bank.
I will be happy to receive any suggestions to improve this bank or to add to it in
the coming editions.
I like to say (at the end) that this project in this shape “is not the end, nor the
beginning of the end, but, probably, is the end of the beginning”.
Finally, I pray to Allah to bless our efforts and to let them aiming only for sake of
Him.

Dr. Ghazi Adlan Al-Shumrani (intern)


Abha- 4/1/1422* 18.March.2002
Mobile No. 055782749
Email: <muslimdoctor@muslimdoctor.net>
CONTENTS
000-introduction …………………………………………..…………………….. 1

001-basic surgical background ………………………………………………… 4

002-trauma & burns …………………………………………………………….. 105

003-abdominal wall & acute abdomen ……………………………………….. 110

004-esophagus, stomach & duodenum ……………………………………... 157

005-small intestine ……………………………………………………………… 203

006-large intestine ……………………………………………………………… 221

007-hepatobiliary & pancreatic ……………………………………………….. 247

008-spleen ………………………………………………………………………..304

009-endocrine …………………………………………………………………… 311

010-breast ……………………………………………………………………... 339

011-thoracic surgery ……………………………………………………………. 353

012-cardiac surgery ……………………………………………………………... 381

013-vascular surgery ……………………………………………………………. 422

014-transplantation ……………………………………………………………… 460

015-oncology ……………………………………………………………………. 481

016-pediatric surgery …………………………………………………………… 496

017-urology …………………………………………………………………… 519

018-orthopedics ………………………………………………………………. 529

019-neurosurgery ……………………………………………………………….. 545

1. Skeletal muscle breakdown produces predominantly liberation of which two amino acids?
A. Lysine.
B. Tyrosine.
C. Alanine.
D. Glutamine.
E. Arginine.
Answer: CD

DISCUSSION: Alanine is released from skeletal muscle and extracted by the liver, where it is converted to new glucose.
Glutamine is also released from muscle and participates in renal acid-base homeostasis and serves as fuel for rapidly
growing cells such as enterocytes, stimulated macrophages, and fibroblasts. Together, these two amino acids account for
approximately two thirds of the nitrogen released from skeletal muscle.

2. In “catabolic” surgical patients, which of the following changes in body composition do not occur?
A. Lean body mass increases.
B. Total body water increases.
C. Adipose tissue decreases.
D. Body weight decreases.
Answer: A

DISCUSSION: Lean body mass represents the body compartment that contains protein. Because critical illness stimulates
proteolysis and increased excretion of body nitrogen, this compartment is consistently reduced, not increased. The change in
body composition is associated with a loss of body weight, an increase in total body water, and a decrease in body fat.

3. The hormonal alterations that follow operation and injury favor accelerated gluconeogenesis. This new glucose is
consumed by which of the following tissues?
A. Central nervous system.
B. Skeletal muscle.
C. Bone.
D. Kidney.
E. Tissue in the healing wound.
Answer: ADE

DISCUSSION: Glucose is produced in increased amounts to satisfy the fuel requirements of the healing wound. In addition,
nerve tissue and the renal medulla also utilize this substrate. Skeletal muscle primarily utilizes fatty acids, and bone utilizes
mineral substrate.

4. Cytokines are endogenous signals that stimulate:


A. Local cell proliferation within the wound.
B. The central nervous system to initiate fever.
C. The production of “acute-phase proteins.”
D. Hypoferremia.
E. Septic shock.
Answer:ABCD

DISCUSSION: Although cytokines exert primarily autocrine and paracrine effects, they may also cause systemic effects.

5. The characteristic changes that follow a major operation or moderate to severe injury do not include the following:
A. Hypermetabolism.
B. Fever.
C. Tachypnea.
D. Hyperphagia.
E. Negative nitrogen balance.
Answer: D

DISCUSSION: The characteristic metabolic response to injury includes hypermetabolism, fever, accelerated
gluconeogenesis, and increased proteolysis (creating a negative nitrogen balance). Food intake is generally impossible
because of abdominal injury or ileus. With time, food intake increases, but the patient generally experiences anorexia, not
hyperphagia.
6. Shock can best be defined as:
A. Hypotension.
B. Hypoperfusion of tissues.
C. Hypoxemia.
D. All of the above.
Answer: B

DISCUSSION: Shock, no matter what the cause, is a syndrome associated with tissue hypoperfusion. Tissue hypoperfusion
leads to tissue hypoxia, which may or may not be due to hypoxemia. Hypotension is a late sign of shock and, therefore, is
not a good clinical indicator of the presence of tissue hypoperfusion.

7. Which of the following statements about continuous cardiac output monitoring are true?
A. Continuous cardiac output monitoring may unmask events not detected by intermittent cardiac output measurements.
B. Continuous cardiac output monitoring by the thermodilution method requires continuous infusion of fluid injectate at a
constant rate and temperature.
C. The major advantage of the Fick method over the thermodilution method of calculating cardiac output is that it is
noninvasive, requiring only the determination of oxygen consumption by respiratory gas analysis.
D. The technique of thoracic electrical bioimpedance utilizes sensors to determine stroke volume by detecting changes in
resistance to a small, applied alternating current.
Answer: AD

DISCUSSION: Various techniques are available to measure cardiac output continuously. The advantages of continuous
cardiac output monitoring, as compared with intermittent methods, are (1) previously undetected events may be unmasked;
(2) more prompt recognition of adverse events may be achieved; and (3) earlier therapeutic intervention may be possible.
Continuous cardiac output monitoring using the thermodilution method appears to be as accurate as the “standard”
intermittent bolus method, but it does not require fluid injectates. In this method, a modified pulmonary artery catheter
incorporating a thermal filament heats blood in the right ventricle at pulsed intervals, and a distal thermistor detects the
temperature change, which can be related mathematically to cardiac output. The Fick method combines respiratory gas
analysis with oximetery to determine oxygen consumption (V(overdot)O 2) and to estimate mixed venous and arterial
oxygen content differences, respectively. Cardiac output (CO) is then determined from the formula: CO = V(overdot)O 2/
{C(a-v)O 2 × 10} @ V(overdot)O 2/ {SaO 2 - SvO 2) × (Hb) × (1.39) × 10}. Thoracic electrical bioimpedance is a
technique by which the resistance to a small-amplitude alternating current (i.e., the impedance) is measured using various
electrodes. The impedance change induced by each cardiac ejection is a function of the stroke volume, which then can be
used to calculate the cardiac output.

8. Which of the following statements regarding cytokines is incorrect?


A. Cytokines act directly on target cells and may potentiate the actions of one another.
B. Interleukin 1 (IL-1) is a major proinflammatory mediator with multiple effects, including regulation of skeletal muscle
proteolysis in patients with sepsis or significant injury.
C. Platelet-activating factor (PAF) is a major cytokine that results in platelet aggregation, bronchoconstriction, and
increased vascular permeability.
D. Tumor necrosis factor alpha (TNF-a), despite its short plasma half-life, appears to be a principal mediator in the
evolution of sepsis and the multiple organ dysfunction syndrome because of its multiple actions and the secondary cascades
that it stimulates.
Answer: C

DISCUSSION: Cytokines are soluble peptide molecules that are synthesized and secreted by a number of cell types in
response to injury, inflammation, and infection. Cytokines, which include the interleukins, tumor necrosis factor, colony-
stimulating factors, and the interferons, comprise only one category of inflammatory mediators involved in the host
response. Endotoxin, complement fragments, eicosanoids, kinins, nitric oxide, oxidants, and PAF are noncytokine mediators
that also have important roles in the systemic inflammatory response. IL-1 and TNF-a, like other cytokines, have multiple
effects on target cells and potentiate the actions of other mediators to produce an amplified inflammatory response. TNF-a is
thought to play a central role in the stress response, particularly in response to endotoxemia.

9. True statements concerning hypoadrenal shock include which of the following?


A. Adrenocortical insufficiency may manifest itself as severe shock refractory to volume and pressor therapy.
B. The presence of hyperglycemia and hypotension may suggest the diagnosis of shock due to adrenocortical insufficiency.
C. Hydrocortisone does not interfere with the serum cortisol assay and should be given to hemodynamically unstable
patients suspected of having hypoadrenal shock.
D. The rapid adrenocorticotropic hormone (ACTH) stimulation test should be performed to help establish the diagnosis of
acute adrenocortical insufficiency.
Answer: AD

DISCUSSION: Shock due to acute adrenocortical insufficiency is relatively uncommon but must be considered when shock
refractory to volume replacement and pressor therapy is present. Hypoglycemia may be present. Patients with high
metabolic stress may exhibit adrenal insufficiency only under conditions of severe stress; thus, a history of adrenal
insufficiency or steroid dependency need not be elicited. When adrenocortical insufficiency is suspected, the rapid ACTH
(cosyntropin) stimulation test should be performed. Serum cortisol levels should be drawn before intravenous administration
of 250 mg. of cosyntropin, and 30 and 60 minutes afterward. A peak cortisol level of less than 20 mg./100 ml. suggests
abnormal adrenal function. In a hemodynamically unstable patient therapy should be instituted before the test results
become available. Dexamethasone does not interfere with the cortisol assay, and it is the corticosteroid of choice while the
ACTH stimulation test is being performed.

10. All of the following are true about neurogenic shock except:
A. There is a decrease in systemic vascular resistance and an increase in venous capacitance.
B. Tachycardia or bradycardia may be observed, along with hypotension.
C. The use of an alpha agonist such as phenylephrine is the mainstay of treatment.
D. Severe head injury, spinal cord injury, and high spinal anesthesia may all cause neurogenic shock.
Answer: C

DISCUSSION: Neurogenic shock occurs when severe head injury, spinal cord injury, or pharmacologic sympathetic
blockade leads to sympathetic denervation and loss of vasomotor tone. Both arteriolar and venous vessels dilate, causing
reduced systemic vascular resistance and a great increase in venous capacitance. The patient's extremities appear warm and
dry, in contrast to those of a patient in cardiogenic or hypovolemic shock. Tachycardia is frequently observed, though the
classic description of neurogenic shock includes bradycardia and hypotension. Volume administration to fill the expanded
intravascular compartment is the mainstay of treatment. The use of alpha-adrenergic agonist is infrequently necessary to
treat neurogenic shock.

11. True statements regarding eicosanoids include which of the following?


A. Prostaglandins and thromboxanes are synthesized via the cyclo-oxygenase pathway.
B. The vasoconstricting, platelet-aggregating, and bronchoconstricting effects of thromboxane A 2 are balanced by the
actions of prostacyclin, which produces the opposite effects.
C. Leukotriene synthesis is inhibited by the action of nonsteroidal anti-inflammatory drugs (NSAIDs).
D. The principal prostaglandins have a short circulation half-life and exert most of their effects locally.
Answer: ABD

DISCUSSION: The eicosanoids are a group of compounds arising from the metabolism of arachidonic acid. The
prostaglandins and thromboxanes are synthesized via the cyclo-oxygenase pathway; thus, their synthesis is blocked by
NSAIDs. Leukotrienes, on the other hand, are synthesized via the lipoxygenase pathway. Prostacyclin, produced largely by
vascular endothelium, inhibits platelet aggregation and causes vasodilatation as well as bronchodilatation. Its effects are
balanced by those of thromboxane A 2, which is produced by platelets and also local actions, including platelet aggregation,
vasoconstriction, and bronchoconstriction. The leukotrienes also have pulmonary and hemodynamic effects and may be
involved in the physiologic responses associated with anaphylactic and septic shock.

12. Which of the following statements about delivery-dependent oxygen consumption are true?
A. Below the critical oxygen delivery (D(overdot)O 2crit), one would expect to see a decrease in the lactate-pyruvate ratio.
B. D(overdot)O 2crit may be increased in patients with sepsis.
C. A desirable goal in the treatment of shock is to achieve delivery-independent oxygen consumption.
D. The oxygen extraction ratio remains constant as long as oxygen delivery remains above D(overdot)O 2crit.
Answer: BC
DISCUSSION: Oxygen consumption is said to be delivery dependent below a critical point, D(overdot)O 2crit, at which
anaerobic metabolism supervenes. Above this point, oxygen consumption is relatively independent of oxygen delivery
because the body's cells can compensate for falls in oxygen delivery by extracting more oxygen. In the delivery-dependent
region, if cellular hypoxia is present, the lactate-pyruvate ratio rises, owing to the switch to anaerobic metabolism.
Generally, it is desirable to achieve delivery-independent oxygen consumption, to avoid ongoing tissue hypoxia. There is
considerable debate, however, about the nature of the oxygen consumption–oxygen delivery relationship in cases of
established sepsis or multiple organ dysfunction syndrome. In such cases, D(overdot)O 2crit may be increased, although the
therapeutic benefit of trying to achieve “supranormal” oxygen delivery has not been firmly established.

13. All of the following may be useful in the treatment of cardiogenic shock except:
A. Dobutamine.
B. Sodium nitroprusside.
C. Pneumatic antishock garment.
D. Intra-aortic balloon pump.
Answer: C

DISCUSSION: Cardiogenic shock occurs when the heart fails to generate adequate cardiac output to maintain tissue
perfusion. Intrinsic causes such as myocardial dysfunction secondary to coronary artery disease, or extrinsic causes such as
pulmonary embolism, tension pneumothorax, and pericardial tamponade, may produce cardiogenic shock. Principles of
treatment of cardiogenic shock are aimed at optimizing preload, cardiac contractility, and afterload. Preload is usually
adequate or high in cardiogenic shock. Dobutamine is a useful inotropic agent, particularly when filling pressures are high,
because of its mild vasodilatory effect, as well as its effect to enhance cardiac contractility. Afterload-reducing agents, such
as sodium nitroprusside, may be beneficial in cardiogenic shock in the setting of elevated filling pressures, low cardiac
output, and elevated systemic vascular resistance. Cardiac output may improve with use of afterload-reducing agents by
decreasing myocardial wall tension and optimizing the myocardial oxygen supply-demand ratio. The intra-aortic balloon
pump (IABP), by providing diastolic augmentation, reducing left ventricular afterload, and reducing myocardial oxygen
consumption, is sometimes useful in the treatment of cardiogenic shock. The IABP is especially useful in low–cardiac
output postcardiotomy patients, in patients awaiting revascularization, and in patients with acute myocardial infarction
complicated by mitral insufficiency or ventricular septal defect. The pneumatic antishock garment (PASG), which causes an
increase in systemic vascular resistance, is contraindicated in cardiogenic shock.

14. Which of the following statements concerning monitoring techniques in the intensive care unit are true?
A. Pulmonary artery and pulmonary capillary wedge pressure readings should be made at end inspiration, to minimize
ventilatory artifacts.
B. Continuous SvO 2 monitoring based on the technique of reflectance spectrophotometry has been shown to be accurate
and reliable.
C. Direct measurement of gastric intramucosal pH can be provided by gastrointestinal tonometry.
D. Hyperlactatemia may be seen in a number of clinical conditions not associated with tissue hypoxia, including liver
disease and hypermetabolic states.
Answer: BD

DISCUSSION: Many different monitoring techniques may be used to assess the adequacy of therapy for shock. The
pulmonary artery catheter can provide important hemodynamic and oxygen transport data that are very useful in directing
therapy aimed at optimizing cardiac function and oxygen delivery. Pulmonary artery and pulmonary capillary wedge
pressure readings should be made at end-expiration to minimize ventilatory artifacts. Continuous SvO 2 monitoring, an
accurate, reliable method that combines pulmonary artery catheterization with the technique of reflectance
spectrophotometry, may provide early warning signs of hemodynamic compromise or inadequate oxygen delivery.
Gastrointestinal tonometry provides information that allows one to infer the adequacy of splanchnic tissue perfusion. In this
technique, intramucosal pH is calculated using the Henderson-Hasselbalch equation and measurements of gut intraluminal
PCO 2 and arterial bicarbonate concentration. Serum lactate concentration may be monitored in shock to detect metabolic
acidosis associated with anaerobic metabolism; however, mild to moderate hyperlactatemia may also be seen with liver
disease, toxin ingestion, and hypermetabolic states not associated with shock.

15. An 18-year-old man shot once in the left chest has a blood pressure of 80/50 mm. Hg, a heart rate of 130 beats per
minute, and distended neck veins. Immediate treatment might include:
A. Administration of one liter of Ringer's lactate solution.
B. Subxiphoid pericardiotomy.
C. Needle decompression of the left chest in the second intercostal space.
D. Emergency thoracotomy to cross-clamp the aorta.
Answer: AC

DISCUSSION: The finding of distended neck veins in conjunction with hypotension should suggest tension pneumothorax
or pericardial tamponade. Absent ipsilateral breath sounds and a trachea deviated to the contralateral side may provide
additional evidence for a tension pneumothorax, the immediate treatment of which is needle decompression of the chest in
the second or third intercostal space in the midclavicular line. Pericardial tamponade may initially respond to volume
administration by enhancing preload. Pericardiocentesis may need to be performed emergently if hemodynamic instability
persists after an initial fluid bolus when signs of compressive cardiogenic shock are present. Subxiphoid pericardiotomy
should be performed only in the operating room by experienced persons who are trained to deal with penetrating cardiac
injuries. There is no role for aortic cross-clamping in this scenario of cardiogenic shock.

16. Which of the following statements are true of the multiple organ dysfunction syndrome (MODS)?
A. The “two-hit” model proposes that secondary MODS may be produced when even a relatively minor second insult
reactivates, in a more amplified form, the systemic inflammatory response that was primed by an initial insult to the host.
B. The systemic inflammatory response syndrome (SIRS), shock due to sepsis or SIRS, and MODS may be regarded as a
continuum of illness severity.
C. Prolonged stimulation or activation of Kupffer cells in the liver is thought to be a critical factor in the sustained,
uncontrolled release of inflammatory mediators.
D. The incidence of MODS in intensive care units has decreased owing to increased awareness, prevention, and treatment
of the syndrome.
Answer: ABC

DISCUSSION: MODS is part of a clinical continuum that begins with the systemic inflammatory response syndrome,
which is the host's stress response to any major insult such as injury or infection. MODS may develop as a result of the
initial insult, but more commonly, it develops following a second or subsequent insult to the host. The two-hit theory holds
that the systemic inflammatory response is amplified following the second hit, such as nosocomial pneumonia, leading to
exaggerated, persistent release of inflammatory mediators that contribute to the pathogenesis of MODS. The liver appears to
be a pivotal organ in the progression and outcome of MODS, partly because of the activation and prolonged stimulation of
the Kupffer cells, which comprise the majority of the body's macrophage population. Macrophages are known to play a
critical role in the elaboration of numerous inflammatory mediators. Despite advances in critical care and in the
understanding of the pathogenesis of MODS, the incidence of MODS continues to increase without a significant
improvement in outcome.

17. All of the following statements about hemorrhagic shock are true except:
A. Following hemorrhagic shock, there is an initial interstitial fluid volume contraction.
B. Dopamine, or a similar inotropic agent, should be given immediately for resuscitation from hemorrhagic shock, to
increase cardiac output and improve oxygen delivery to hypoperfused tissues.
C. The use of colloid solutions or hypertonic saline solutions is contraindicated for treatment of hemorrhagic shock.
D. In hemorrhagic shock, a narrowed pulse pressure is commonly seen before a fall in systolic blood pressure.
Answer: BC

DISCUSSION: Hemorrhagic shock is associated with a contraction of the interstitial fluid compartment because of
precapillary vasoconstriction and reabsorption of interstitial fluid into the vascular compartment along hydrostatic pressure
gradients. Systolic hypotension may not be evident in hemorrhagic shock until at least 30% or more of blood volume is
exsanguinated. A decrease in the pulse pressure (the difference between systolic and diastolic pressures) may be observed
with losses of 15% to 30% of blood volume. Treatment of hemorrhagic shock includes intravascular fluid administration
and definitive control of the source of the hemorrhage. Inotropic agents should not be started before volume resuscitation
but may be added to improve oxygen delivery to hypoxic tissues if volume administration alone does not produce
resuscitative goals. Colloid or hypertonic saline solutions are not contraindicated in the treatment of hemorrhagic shock;
however, definitive evidence that such solutions are better than standard crystalloid solutions is lacking.

18. Which of the following statements about septic shock are true?
A. A circulating myocardial depressant factor may account for the cardiac dysfunction sometimes seen with shock due to
sepsis or SIRS.
B. A cardiac index (CI) of 6 liters per minute per square meter of body surface, a pulmonary capillary wedge pressure of 15
mm. Hg, and a systemic vascular resistance index (SVRI) of 800 dynes-sec/(cm 5-m 2) is a hemodynamic profile consistent
with septic shock.
C. An increase in SvO 2 in septic patients may be explained by the finding of anatomic arteriovenous shunts.
D. Results of human trials employing antimediator therapy, such as antiendotoxin antibodies, IL-1 receptor antagonist, and
tumor necrosis factor (TNF) antibodies, have confirmed animal studies that demonstrate a significant improvement in
survival with the use of such agents.
Answer: AB

DISCUSSION: Shock due to sepsis or SIRS frequently manifests as a hyperdynamic cardiovascular response, consisting of
an elevated CI and a decreased SVR or SVRI. Occasionally, myocardial depression may be seen, characterized by increased
ventricular volumes and decreased ejection fractions. A circulating myocardial depressant factor, possibly TNF, may be
responsible for the cardiac dysfunction in such instances. The cause of the increased SvO 2 frequently observed in septic
patients is unclear, but it may be secondary to bioenergetic failure, metabolic downregulation, or microcirculatory
maldistribution leading to physiologic shunting. True anatomic arteriovenous shunting has not been demonstrated in humans
in septic shock. Treatment of septic shock consists of appropriate antibiotic use and supportive therapy. Experimental
antimediator therapies have not been encouraging thus far in human clinical trials, despite the promising results from many
animal studies.

19. Which of the following statements are true of oxidants?


A. In addition to their pathophysiologic roles in inflammation, injury, and infection, oxidants also have physiologic roles.
B. Oxidants may be generated from activated neutrophils and during reperfusion following a period of ischemia.
C. The deleterious effects of oxidants include lipid peroxidation and cell membrane damage, oxidative damage to DNA,
and inhibition of adenosine triphosphate (ATP) synthesis.
D. The mechanism of ischemia-reperfusion injury involved the catalytic production of superoxide anion (O 2•) by the
enzyme xanthine oxidase.
Answer: ABCD

DISCUSSION: Oxidants are reactive oxygen metabolites that have both physiologic and pathophysiologic roles. As potent
oxidizing agents, oxidants are involved in cytochrome P 450–mediated oxidations, for example. In pathophysiologic
processes associated with inflammation, injury, and infection, oxidants may be generated by activated neutrophils and in
ischemia-reperfusion injury. During ischemia, the enzyme xanthine oxidase accumulates. When oxygen availability
increases during reperfusion, O 2• is formed in a reaction catalyzed by xanthine oxidase. Further oxidant formation ensues,
causing the production of H 2O 2 and the extremely reactive hydroxyl ion (OH•). Oxidants may cause direct cell damage by
the mechanisms of lipid peroxidation and cell membrane disruption, inhibition of ATP synthesis, reduction of cellular
nicotinamide adenine dinucleotide (NAD), and oxidative damage to DNA and amino acids. In addition, oxidants may have a
chemotactic role, leading to leukocyte infiltration and activation, causing further tissue damage by the release of cytotoxic
proteases.

20. Which of the following statements about the role of the gut in shock and sepsis are true?
A. Selective decontamination of the digestive tract with the use of oral antibiotics has been shown to reduce nosocomial
pneumonias and to improve mortality rates.
B. Enteral nutrition, as compared with parenteral nutrition, preserves the villus architecture of the gut.
C. Gut dysfunction may be an effect of shock, but it may also contribute to the development of MODS by the mechanism
of bacterial translocation.
D. As compared with parenteral nutrition, enteral nutrition is associated with a reduction in septic morbidity.
Answer: BCD

DISCUSSION: The gut has a vital role in the pathophysiology of shock. The splanchnic circulation is very vulnerable to the
circulatory redistribution that occurs in shock, thus, gut ischemia may occur early in the various shock syndromes. Gut
injury, as a result of ischemia or reperfusion injury, leads to disruption in the intestinal mucosal barrier and increased gut
permeability. Translocation of enteric flora or bacterial toxins across the gut wall may then occur, resulting in amplification
of the systemic inflammatory response and the development of multiple organ dysfunction. Gut dysfunction, therefore, may
perpetuate the inflammatory process. Various methods have been tried to modulate the deleterious effects of gut
dysfunction. Selective decontamination of the digestive tract by oral antibiotics has been shown to reduce the incidence of
nosocomial pneumonias, but no improvement in mortality has been demonstrated thus far with this controversial technique.
Early enteral nutrition probably has the biggest impact on the preservation of gut architecture and function. When compared
to parenteral nutrition, enteral feeding is more cost effective and is associated with a lower rate of septic morbidity.

21. Which of the following statements about head injury and concomitant hyponatremia are true?
A. There are no primary alterations in cardiovascular signs.
B. Signs of increased intracranial pressure may be masked by the hyponatremia.
C. Oliguric renal failure is an unlikely complication.
D. Rapid correction of the hyponatremia may prevent central pontine injury.
E. This patient is best treated by restriction of water intake.
Answer: A

DISCUSSION: Acute symptomatic hyponatremia is characterized by central nervous system signs of increased intracranial
pressure. Changes in blood pressure and pulse are secondary to increased intracranial pressure. In the absence of
hypovolemia, asymptomatic patients may be treated by restriction of water intake; however, in such patients, hyponatremia
should be partially corrected by parenteral sodium administration. Rapid correction, particularly to hypernatremia, may lead
to central pontine myelinolysis. Oliguric renal failure may rapidly develop in severe hyponatremia.

22. Which of the following statements about total body water composition are correct?
A. Females and obese persons have an increased percentage of body water.
B. Increased muscle mass is associated with decreased total body water.
C. Newborn infants have the greatest proportion of total body water.
D. Total body water decreases steadily with age.
E. Any person's percentage of body water is subject to wide physiologic variation.
Answer: CD

DISCUSSION: Since fat contains little water, lean persons with a proportionately greater muscle mass have a greater than
expected volume of total body water. Likewise, the female body habitus and obesity contribute to decreased total body
water percentage. The highest proportion of total body water is found in newborn infants, and total body water decreases
steadily and significantly with age. The actual figure for a healthy person is remarkably constant.

23. Which of the following statements about extracellular fluid are true?
A. The total extracellular fluid volume represents 40% of the body weight.
B. The plasma volume constitutes one fourth of the total extracellular fluid volume.
C. Potassium is the principal cation in extracellular fluid.
D. The protein content of the plasma produces a lower concentration of cations than in the interstitial fluid.
E. The interstitial fluid equilibrates slowly with the other body compartments.
Answer: B

DISCUSSION: The total extracellular fluid volume represents 20% of body weight. The plasma volume is approximately
5% of body weight. Sodium is the principal cation. The Gibbs-Donan equilibrium equation explains the higher total
concentration of cations in plasma. Except for joint fluid and cerebrospinal fluid, the majority of the interstitial fluid exists
as a rapidly equilibrating component.

24. Which of the following statements are true of a patient with hyperglycemia and hyponatremia?
A. The sodium concentration must be corrected by 5 mEq. per 100 mg. per 100 ml. elevation in blood glucose.
B. With normal renal function, this patient is likely to be volume overloaded.
C. Proper fluid therapy would be unlikely to include potassium administration.
D. Insulin administration will increase the potassium content of cells.
E. Early in treatment adequate urine output is a reliable measure of adequate volume resuscitation.
Answer: D

DISCUSSION: Each 100-mg. per 100 ml. elevation in blood glucose causes a fall in serum sodium concentration of
approximately 2 mEq. per liter. Excess serum glucose acts as an osmotic diuretic, producing increased urine flow, which can
lead to volume depletion. Insulin therapy and the correction of the patient's associated acidosis produce movement of
potassium ions into the intracellular compartment.

25. Which of the following statements about respiratory acidosis are true?
A. Compensation occurs by a shift of chloride out of the red blood cells.
B. Renal compensation occurs rapidly.
C. Retention of bicarbonate and increased ammonia formation are normal compensatory mechanisms.
D. Narcotic administration is an unusual cause of respiratory acidosis.
E. The ratio of bicarbonate to carbonic acid is less than 20:1.
Answer: CE

DISCUSSION: Renal compensation for acute hypoventilation is relatively slow. Depression of the respiratory center by
morphine can lead to respiratory acidosis. Renal retention of bicarbonate, ammonia formation, and shift of chloride into red
cells combine to increase the ratio of bicarbonate to carbonic acid to 20:1.

26. Which of the following statements are true of elevated–anion gap metabolic acidosis?
A. Hypoperfusion from the shock state rarely produces an elevated anion gap.
B. Retention of sulfuric and phosphoric acids may lead to this form of acidosis.
C. Copious diarrhea does not produce this condition.
D. Rapid volume expansion may produce this form of acidosis.
E. Use of lactated Ringer's solution is inappropriate in the treatment of lactic acidosis.
Answer: BC

DISCUSSION: An elevated anion gap may be produced by lactic acidosis from shock or by retention of inorganic acids
from uremia. Lactated Ringer's solution rapidly corrects the lactic acidosis from hypovolemia, as lactate is converted to
bicarbonate with hepatic reperfusion. Bicarbonate loss from diarrhea and “dilutional acidosis” are non–anion gap types of
metabolic acidosis.

27. Which of the following is true of loss of gastrointestinal secretions?


A. Gastric losses are best replaced with a balanced salt solution.
B. Potassium supplementation is unnecessary in replacement of gastric secretions.
C. Bicarbonate wasting is an unusual complication of a high-volume pancreatic fistula.
D. Balanced salt solution is a reasonable replacement fluid for a small bowel fistula.
E. A patient with persistent vomiting usually requires hyperchloremic replacement fluids.
Answer: DE

DISCUSSION: Gastric secretions are relatively high in chloride and potassium. Other than an isolated pancreatic fistula,
gastrointestinal tract losses below the pylorus are best replaced by a balanced salt solution. Although potassium
concentrations are low, copious losses require potassium supplementation to prevent hypokalemia.

28. Which of the following statements regarding hypercalcemia are true?


A. The symptoms of hypercalcemia may mimic some symptoms of hyperglycemia.
B. Metastatic breast cancer is an unusual cause of hypercalcemia.
C. Calcitonin is a satisfactory long-term therapy for hypercalcemia.
D. Severely hypercalcemic patients exhibit the signs of extracellular fluid volume deficit.
E. Urinary calcium excretion may be increased by vigorous volume repletion.
Answer: ADE

DISCUSSION: Markedly elevated serum calcium levels produce polydipsia, polyuria, and thirst. Vigorous volume repletion
and saline diuresis correct the extracellular fluid volume deficit and promote the urinary excretion of calcium. Metastatic
breast cancer is the most common cause of hypercalcemia, from bony metastasis. The calcitonin effect on calcium is
diminished with repeat administrations.

29. Which of the following statements about normal salt and water balance are true?
A. The products of catabolism may be excreted by as little as 300 ml. of urine per day.
B. The lungs represent the primary source of insensible water loss.
C. The normal daily insensible water loss is 600 to 900 ml.
D. Excessive cell catabolism causes significant loss of total body water.
E. In normal humans, urine represents the greatest source of daily water loss.
Answer: CE

DISCUSSION: The skin is the primary source of insensible water loss. Including losses from the lungs, this averages 600 to
900 ml. per day. Catabolism liberates “water of solution.” In normal humans, urine represents the greatest source of water
loss. The patient deprived of external access to water must still excrete a minimum of 500 to 800 ml. of urine per day to
expel the products of catabolism.

30. Which of the following is/are not associated with increased likelihood of infection after major elective surgery?
A. Age over 70 years.
B. Chronic malnutrition.
C. Controlled diabetes mellitus.
D. Long-term steroid use.
E. Infection at a remote body site.
Answer: C

DISCUSSION: Controlled diabetes mellitus has been shown repeatedly not to be associated with increased likelihood of
incisional infection provided one avoids operations on body parts that may be ischemic or neuropathic. Uncontrolled
diabetes mellitus, such as ketoacidosis, is associated with a dramatic increase in surgical infection. The other parameters
noted—age over 70, chronic malnutrition, regular steroid use, and an infection at a remote body site—are well-recognized
adverse predictive factors and are identified in tables within the chapter.

31. Which of the following are not determinants of a postoperative cardiac complication?
A. Myocardial infarct 4 months previously.
B. Clinical evidence of congestive heart failure in a patient with 8.5 gm. per dl. hemoglobin.
C. Premature atrial or ventricular contractions on electrocardiogram.
D. A harsh aortic systolic murmur.
E. Age over 70 years.
Answer: B

DISCUSSION: Clinical evidence of congestive heart failure in a patient with 8.5 gm. per dl. hemoglobin concentration is a
misleading sign. Evidence of congestive failure is ordinarily a major risk factor, but in this particular patient the anemia
lends itself to correction by preoperative transfusion with packed red blood cells, and often it is found that congestive failure
and the associated increased risks disappear when the hemoglobin concentration is returned to the 12 gm. per dl. or higher
ratio. All other factors are overt signs of increased likelihood of a postoperative cardiac event, the most ominous being a
myocardial infarction 4 months preoperatively or the presence of a harsh aortic systolic murmur suggesting the presence of
aortic stenosis. Age over 70 years and the presence of premature atrial or ventricular contractions on the electrocardiogram
are less strong determinants of a postoperative cardiac complication.

32. Rank the clinical scenarios in order of greatest likelihood of serious postoperative pulmonary complications.
A. Transabdominal hysterectomy in an obese woman that requires 3 hours of anesthesia time.
B. Right middle lobectomy for bronchogenic cancer in a 65-year-old smoker.
C. Vagotomy and pyloroplasty for chronic duodenal ulcer disease in a 50-year-old who had chest film findings of old,
healed tuberculosis.
D. Right hemicolectomy in an obese 60-year-old smoker.
E. Modified radical mastectomy in a 58-year-old woman who is obese.
Answer: BDCAE

DISCUSSION: If one considers the constellation of risk factors for pulmonary complications that is provided in tabular
form in the accompanying chapter, one should readily recognize B, right middle lobectomy for bronchogenic cancer in a 65-
year-old smoker, as the highest risk of a clinical situation for the likelihood of serious pulmonary complications. The next in
rank may be properly debated between answer D and answer C. D, right hemicolectomy, is judged to have somewhat
greater likelihood of complications since the patient is older, smokes, and is obese, although the procedure may be done
through a transverse or lower abdominal incision. C, vagotomy and pyloroplasty, is viewed as being somewhat less serious
since it is an upper abdominal operation on an elective basis in a 50-year-old whose only abnormalities include old, healed
tuberculosis on a chest film. A very low risk of pulmonary complication should follow a transabdominal hysterectomy done
through a lower abdominal incision in a woman whose only risk factors are obesity and a 3-hour anesthesia time. The
lowest risk probably resides with the younger patient undergoing modified radical mastectomy, whose only risk factor is
obesity. This is particularly true since this operation is conducted on the surface of the body, is associated with relatively
little postoperative pain, and provides free and unrestricted respiratory function.

33. Rank the following laboratory tests and procedures in terms of their relative value to a 65-year-old woman who is
to undergo elective resection of a sigmoid cancer.
A. Carcinoembryonic antigen (CEA).
B. Blood urea nitrogen (BUN).
C. Electrocardiogram (ECG).
D. Hemoglobin concentration (Hgb).
E. Serum creatinine (Cr).
F. Arterial blood oxygen tension (PaO 2) on room air.
G. Serum sodium concentration (Na+).
Answer: CDFEBAG

DISCUSSION: The most important test by far is the electrocardiogram, with its capacity to indicate signs of occult heart
disease. The second most important evaluation is the hemoglobin concentration, which in this patient may show an anemia
related to chronic alimentary tract blood loss that would require correction prior to safe induction of a general anesthetic.
Arterial blood gases vary from individual to individual depending primarily on smoking habits and age. Accordingly, each
older person should have a resting baseline determination prior to operation. Serum creatinine may show evidence of occult
renal disease and is substantially more useful than blood urea nitrogen, which is more vulnerable to transient volume
changes. Carcinoembryonic antigen is important to know in many patients with cancer with respect to postoperative follow-
up since in some cases it may be an early herald of recurrent disease. However, it has little to do with the patient's
preoperative assessment in terms of risk and preparation for an elective operation. The presence of liver metastases, for
example, can be discovered with significant accuracy by palpation at the time of operation, and an elevated
carcinoembryonic antigen in no set of circumstances would lead one to withhold colon resection with its relief of potential
obstruction and bleeding. Finally, serum sodium concentration in a 65-year-old woman who is admitted electively for
resection of the colon is always normal and would be of least value among these tests.

34. Which of the following statements regarding whole blood transfusion is/are correct?
A. Whole blood is the most commonly used red cell preparation for transfusion in the United States.
B. Whole blood is effective in the replacement of acute blood loss.
C. Most blood banks in the United States have large supplies of whole blood available.
D. The use of whole blood produces higher rates of disease transmission than the use of individual component therapies.
Answer: B

DISCUSSION: Whole blood is effective as a replacement fluid for acute blood loss because it provides both volume and
oxygen-carrying capacity (red blood cells). It is rarely used in the United States nowadays, and most blood banks do not
provide whole blood transfusions. It is significantly more efficient to separate donated blood into its components. In this
manner, the red blood cell mass can be used to provide oxygen-carrying capacity, the plasma can be used for factor
replacement, and the platelets and white cells can be used for patients deficient in these components. The use of whole
blood to replace acute blood loss is associated with lower disease transmission rates than the use of packed red blood cells,
fresh frozen plasma, and platelets, each from a different donor.

35. Which of the following statements about the preparation and storage of blood components is/are true?
A. Solutions containing citrate prevent coagulation by binding calcium.
B. The shelf life of packed red blood cells preserved with CPDA-1 is approximately 35 days at 1‫ ؛‬to 6‫ ؛‬C.
C. There are normal numbers of platelets in packed red blood cells stored at 1‫ ؛‬to 6‫ ؛‬C for more than 2 days.
D. The storage lesion affecting refrigerated packed red blood cells includes development of acidosis, hyperkalemia, and
decreased intracellular 2,3DPG (diphosphoglycerate).
Answer: ABD
DISCUSSION: After blood has been collected from a donor, it is anticoagulated with a solution containing citrate, which
acts by binding calcium. Blood is then separated into its components. Packed red blood cells stored at 1 ‫ ؛‬to 6‫ ؛‬C using
CPDA-1 preservative have a shelf life of 35 days. There are essentially no functional platelets in refrigerated blood stored at
1‫ ؛‬to 6‫ ؛‬C after approximately 48 hours in storage. Refrigerated packed red blood cells undergo progressive changes termed
a storage lesion. Such changes include acidosis, hyperkalemia, and decreased levels of 2,3-DPG, which are reversed after
transfusion or produce effects other than those predicted based on the content of the unit of blood.

36. Which of the following is/are acceptable reasons for the transfusion of red blood cells based on currently available
data?
A. Rapid, acute blood loss with unstable vital signs but no available hematocrit or hemoglobin determination.
B. Symptomatic anemia: orthostatic hypotension, severe tachycardia (greater than 120 beats per minute), evidence of
myocardial ischemia, including angina.
C. To increase wound healing.
D. A hematocrit of 26% in an otherwise stable, asymptomatic patient.
Answer: AB

DISCUSSION: Currently accepted guidelines for the transfusion of packed red blood cells include acute ongoing blood
loss, as might occur in an injured patient, and the development of symptomatic anemia with manifestations of decreased
tissue perfusion related to decreased oxygen-carrying capacity of the blood. This includes situations in which the patient is
unable to compensate for a decreased oxygen-carrying capacity by the usual mechanisms, such as increased cardiac output.
Such patients develop myocardial dysfunction if an excessive demand is placed on the heart. The patient should be
transfused with packed red blood cells, which afford added oxygen-carrying capacity. This decreases the workload on the
myocardium while providing the necessary oxygen-delivery capability. The use of packed red blood cells to improve wound
healing or to improve the patient's sense of well-being is highly questionable. No data support such a practice. In general,
the use of a transfusion trigger such as a hematocrit of 30% or hemoglobin of 10 gm. per dl. constitutes a questionable
indication for transfusion. If a patient is asymptomatic and stable and has no risk of myocardial ischemia, packed red blood
cell transfusion should not be given based solely or predominantly on a numerical value such as a hematocrit of 28%.

37. The transfusion of fresh frozen plasma (FFP) is indicated for which of the following reasons?
A. Volume replacement.
B. As a nutritional supplement.
C. Specific coagulation factor deficiency with an abnormal prothrombin time (PT) and/or an abnormal activated partial
thromboplastin time (APTT).
D. For the correction of abnormal PT secondary to warfarin therapy, vitamin K deficiency, or liver disease.
Answer: CD

DISCUSSION: The use of FFP as a volume expander is not indicated. There are currently several preparations (both
crystalloid and colloid) that are equally effective and do not carry the infectious and other risks associated with the use of
FFP. The use of FFP as a “nutritional” supplement is to be condemned. Patients with specific deficiencies of coagulation
factors generally benefit greatly from the infusion of FFP. In cases of specific factor deficiency, other preparations may be
more appropriate, but FFP is generally immediately available and is effective in most patients. Patients receiving warfarin
therapy, those who have vitamin K deficiency, and those with liver dysfunction have abnormalities of the vitamin K–
dependent factors II, VII, IX, and X, as well as protein C and protein S.

38. In patients receiving massive blood transfusion for acute blood loss, which of the following is/are correct?
A. Packed red blood cells and crystalloid solution should be infused to restore oxygen-carrying capacity and intravascular
volume.
B. Two units of FFP should be given with every 5 units of packed red blood cells in most cases.
C. A “six pack” of platelets should be administered with every 10 units of packed red blood cells in most cases.
D. One to two ampules of sodium bicarbonate should be administered with every 5 units of packed red blood cells to avoid
acidosis.
E. One ampule of calcium chloride should be administered with every 5 units of packed red blood cells to avoid
hypocalcemia.
Answer: A
DISCUSSION: Patients who are suffering from acute blood loss require crystalloid resuscitation as the initial maneuver to
restore intravascular volume and re-establish vital signs. If 2 to 3 liters of crystalloid solution is inadequate to restore
intravascular volume status, packed red blood cells should be infused as soon as possible. There is no role for “prophylactic
infusion” of FFP, platelets, bicarbonate, or calcium to patients receiving massive blood transfusion. If specific indications
exist patients should receive these supplemental components. In particular, patients who have abnormal coagulation tests
and have ongoing bleeding should receive FFP. Patients who have depressed platelet counts along with clinical evidence of
oozing (microvascular bleeding) benefit from platelet infusion. Sodium bicarbonate is not necessary, since most patients
who receive blood transfusion ultimately develop alkalosis from the citrate contained in stored red blood cells. The use of
calcium chloride is usually unnecessary unless the patient has depressed liver function, ongoing prolonged shock associated
with hypothermia, or, rarely, when the infusion of blood proceeds at a rate exceeding 1 to 2 units every 5 minutes.

39. Hemostasis and the cessation of bleeding require which of the following processes?
A. Adherence of platelets to exposed subendothelial glycoproteins and collagen with subsequent aggregation of platelets
and formation of a hemostatic plug.
B. Interaction of tissue factor with factor VII circulating in the plasma.
C. The production of thrombin via the coagulation cascade with conversion of fibrinogen to fibrin.
D. Cross-linking of fibrin by factor XIII.
Answer: ABCD

DISCUSSION: Hemostasis requires the interaction of platelets with the exposed subendothelial structures at the site of
injury followed by aggregation of more platelets in that area. Interactions between endothelial cell and subendothelial tissue
factor with factor VII activate the coagulation cascade. The end product is large amounts of thrombin that catalyze the
conversion of fibrinogen into fibrin. Fibrin thus formed is cross-linked by factor XIII to form a stable clot that incorporates
the platelet plug and fibrin thrombus into a stable clot.

40. Which of the statements listed below about bleeding disorders is/are correct?
A. Acquired bleeding disorders are more common than congenital defects.
B. Deficiencies of vitamin K decrease production of factors II, VII, IX, and X, protein C, and protein S.
C. Hypothermia below 32‫؛‬C rarely causes a bleeding disorder.
D. Von Willebrand's disease is a very uncommon congenital bleeding disorder.
Answer: AB

DISCUSSION: Acquired bleeding disorders are significantly more common than congenital bleeding defects. Vitamin K
deficiency may be related to malnutrition or competitive inhibition of the production of the vitamin K–dependent factors II,
VII, IX, X, protein C, and protein S by warfarin (Coumadin). Hypothermia causes significant platelet dysfunction with a
significant bleeding disorder in many patients. It is among the least recognized causes of altered coagulation in surgical
patients. Von Willebrand's disease is a relatively common disorder of bleeding and is generally undetectable by routine
screening methods.

41. The evaluation of a patient scheduled for elective surgery should always include the following as tests of
hemostasis and coagulation:
A. History and physical examination.
B. Complete blood count (CBC), including platelet count.
C. Prothrombin time (PT) and activated partial thromboplastin time (APTT).
D. Studies of platelet aggregation with adenosine diphosphate (ADP) and epinephrine.
Answer: A

DISCUSSION: The evaluation of most patients scheduled for elective surgery who do not have a history of significant
bleeding disorders is somewhat controversial. An adequate history and physical examination screen out most patients with
bleeding problems. For patients who are scheduled to undergo a major surgical procedure, it is advisable to obtain a CBC
and platelet count, as well as a PT and APTT level. This detects a large number of bleeding disorders but does not rule out
all possible causes of perioperative bleeding. Studies of platelet aggregation are indicated only for patients who are
suspected of having qualitative defects of platelet function (e.g., von Willebrand's disease).
42. Which of the following statements regarding the transmission of infectious agents through blood transfusions is/are
true?
A. The transmission rates for human immunodeficiency virus (HIV) have been decreasing progressively since the early
1980s.
B. The transmission rates of hepatitis have been decreasing steadily since the 1980s.
C. Cytomegalovirus (CMV) is the infectious agent most commonly transmitted in blood.
D. Severely immunocompromised patients (such as patients undergoing transplantation) should receive specially screened
blood products.
Answer: ABCD

DISCUSSION: The incidence of both HIV and hepatitis transmitted via blood transfusions has been steadily decreasing
since the 1980s. This is related to improved methods for detection and increased awareness of surrogate markers of disease.
The currently available techniques for the detection of HIV are highly effective, provided the donor is not in the “window”
before the formation of specific antibody. The surrogate markers for hepatitis C, as well as the specific assays for the
organism, are now sufficiently refined to allow the detection of a large percentage of hepatitis C infection in donated blood.
Screening for hepatitis B surface antigen has effectively eliminated the transmission of hepatitis B through blood products
in most cases. CMV is the most commonly transmitted infectious agent in blood. Since a large percentage of the population
carry the virus, routine screening is not performed for this organism; however, severely compromised patients such as those
undergoing transplantation should receive CMV-negative blood products.

43. The most common cause of fatal transfusion reactions is:


A. An allergic reaction.
B. An anaphylactoid reaction.
C. A clerical error.
D. An acute bacterial infection transmitted in blood.
Answer: C

DISCUSSION: The most common cause of fatalities related to transfusion reactions result from ABO-incompatible
transfusion related to clerical error. Most such reactions occur if a type O person receives type A red cells owing to a clerical
error that occurs either at the time the blood sample was drawn, during processing in the laboratory, or at the time a unit is
administered. The importance of extremely careful labeling, transfer, and handling of specimens and of cross-matched blood
products cannot be overemphasized. Allergic and other reactions are common but rarely fatal. The transmission of bacterial
organisms (e.g., Staphylococcus aureus) has been reported especially with platelet concentrates maintained at or near room
temperature. Fortunately, such reactions are rare.

44. Which of the following statements about the coagulation cascade is/are true?
A. The intrinsic pathway of coagulation is the predominant pathway in vivo for hemostasis and coagulation.
B. The intrinsic pathway beginning with the activation of factor XII is the predominant in vivo mechanism for activation of
the coagulation cascade.
C. Deficiencies of factor VIII and IX cause highly significant coagulation abnormalities.
D. Deficiencies of factor XII cause severe clinical bleeding syndromes.
Answer: AC

DISCUSSION: Although it was previously held that two somewhat distinct pathways existed for the activation of the
coagulation cascade, it is now recognized that the predominant mechanism for coagulation in vivo is the “extrinsic
pathway.” Tissue factor is exposed in the subendothelial tissues when endothelial cell injury occurs. Tissue factor then
tightly binds factor VII circulating in the plasma and activates the coagulation cascade. Factor VIII and factor IX deficiency
cause the clinical syndromes of hemophilia A and hemophilia B, respectively. Both of these disorders involve very severe
clinical bleeding disorders, whereas deficiencies of factor XII do not generally cause clinically significant bleeding. This
further emphasizes the secondary role that the “intrinsic pathway” plays in coagulation.

45. A major problem in nutritional support is identifying patients at risk. Recent studies suggest that these patients can be
identified. Which of the following findings identify the patient at risk?
A. Weight loss of greater than 10% over 2 to 4 months.
B. Serum albumin of less than 3 gm. per 100 ml. in the hydrated state.
C. Malnutrition as identified by global assessment.
D. Serum transferrin of less than 220 mg. per 100 ml.
E. Functional impairment by history.
Answer: ABCDE

DISCUSSION: All of these are at least partially correct. It is not clear whether weight loss of 10% or 15% is the required
threshold, but it certainly is close. Serum albumin of less than 3 gm per 100 ml. remains the most constant identifier of
patients at risk in the literature and has been so for years. Global assessment in the hands of an experienced investigator is
quite efficacious at identifying persons at risk. Serum transferrin is certainly a confirmatory identifier of patients with
malnutrition—and may be even a primary one. Graham Hill and his co-workers have pioneered the concept of global
assessment using functional parameters, and in the hands of an experienced observer is quite a reasonable way of
approaching and identifying patients at risk.

46. Essential fatty acid deficiency may complicate total parenteral nutrition (TPN). Which of the following statements are
true?
A. Essential fatty acid deficiency may be prevented by the administration of 1% to 2% of total calories as fat emulsion.
B. Fat-free parenteral nutrition results in the appearance of plasma abnormalities, indicating essential fatty acid deficiency,
within 7 to 10 days of initiation.
C. An abnormal plasma eicosatrienoic-arachidonic acid ratio is always associated with essential fatty acid deficiency.
D. Following initiation of fat-free parenteral nutrition, dry, scaly skin associated with a maculopapular rash indicates
essential fatty acid deficiency.
Answer: BD

DISCUSSION: Biochemical evidence of essential fatty acid deficiency may occur as early as 7 to 10 days following
initiation of fat-free parenteral nutrition. The decrease in arachidonic acid in plasma and the appearance of the abnormal
eicosatrienoic acid may yield the earliest indication of prostaglandin deficiency; it is not absolute. Decreased intraocular
pressure, another early indication of prostaglandin deficiency, may appear as soon as 7 days following initiation of fat-free
parenteral nutrition. While my current practice is to give at least 500 ml. of 10% lipid emulsion daily to provide 20% to 25%
of total calories to support hepatic protein synthesis, as little as 4% to 6% of total daily calories as fat prevents essential fatty
acid deficiency. Practically, this may be undertaken by the administration of 500 ml. of 10% lipid three times weekly. The
appearance of eicosatrienoic acid and a decrease in arachidonic acid, and a change in ratio, is not essential to the diagnosis
of essential fatty acid deficiency, but this plasma abnormality is often present.

47. It is stated that enteral nutrition is safer than parenteral nutrition. Which of the following may be complications of
enteral nutrition?
A. Hyperosmolar, nonketotic coma.
B. Vomiting and aspiration.
C. Pneumatosis cystoides intestinalis.
D. Perforation and peritonitis.
Answer: ABCD

DISCUSSION: It is not necessarily true that enteral nutrition is safer than parenteral nutrition, and it may in fact be
associated with a higher risk of death than parenteral nutrition. Specifically, a well-run parenteral nutrition service should
not be associated with significant mortality, except for the occasional death due to undetected yeast infection. On the other
hand, enteral nutrition, especially if not carried out safely, can result in significant mortality. The most common of the
severe complications of enteral nutrition result from the gastrostomy, or tube feedings into the stomach. Sudden changes in
gastric motility, such as those associated with sepsis, may result in aspiration. Nasoenteric or nasoduodenal tubes help
prevent this complication, as does shutting off enteral feedings between the hours of 11 P.M. and 7 A.M. It is also essential
to keep the patient's head elevated 30 degrees. Also necessary is the use of extreme care when initiating enteral nutrition. If
hypertonic material is given into the stomach, one can increase osmolality followed by an increase in volume. If, however,
the material is given into the small bowel, volume must be increased first and then tonicity, with the expectation that
osmolality greater than 400 or 500 mOsm per liter may never be achieved without provoking severe diarrhea. If care is not
taken with the initiation of enteral nutrition, massive diarrhea may result, including fluid loss, the absorption of enormous
amounts of carbohydrate into the circulation with inadequate fluid to support it, and the development of hyperosmolar,
nonketotic coma. Alternatively, severe unremitting diarrhea may result in necrosis of the intestinal wall, the appearance of
pneumatosis cystoides intestinalis, and, finally, perforation and death. All of these complications may be prevented by
judicious use of enteral nutrition with the same care one uses for parenteral nutrition.
48. It has been suggested that enterocyte-specific fuels be utilized for all patients receiving parenteral nutrition.
Theoretically, the benefits of such fuels include:
A. Glutamine increases gut mucosal protein content and wall thickness.
B. Butyrate increases jejunal mucosal protein content and wall thickness.
C. The short-chain fatty acids—butyrate, propionate, and acetate—are useful in supporting ileal mucosal protein content
and thickness.
D. The use of glutamine-enriched solutions for parenteral nutrition for patients with chemotherapy toxicity or radiation
enteritis is without hazards.
Answer: NONE IS ENTIRELY TRUE

DISCUSSION: The use of enterocyte-specific fuels is part of a new and potentially exciting phase of “nutritional
pharmacology” in parenteral nutrition; however, exciting as the research may be, the use of such fuels is by no means
acceptable for indiscriminate use at present. Though some studies have shown that the provision of glutamine in amounts up
to 2% in standard parenteral nutrition solutions increases both jejunal and ileal mucosal protein content, cell wall thickness,
and DNA content, this has not been the case in all studies, and this reported effect seems very dependent on experimental
design. In many of the studies that have shown such an effect, 2% glutamine has been used to replace virtually all
nonessential amino acids, probably initiating a deficiency state. The beneficial effects seen with glutamine are far less
impressive than those seen with epidermal growth factor, for example, and disappear entirely when a different experimental
design is used in which 2% glutamine is added to an adequate amino acid formulation in which glutamine does not replace
nonessential amino acids but is added to them. Nonetheless, the use of enterocyte-specific fuels, specifically glutamine, is
potentially exciting and should be carefully investigated. More striking are the results that follow massive bowel resection,
radiation enteritis, and chemotherapy toxicity. Glutamine may help the small bowel regenerate more quickly, enabling more
rapid use of the small bowel for nutrition. It should be pointed out, however, that glutamine is a fuel utilized by many
tumors and, thus, one runs the risk of stimulating the growth of the tumor with excessive glutamine. The short-chain fatty
acids, produced from bacterial fermentation of soluble pectin, may be useful in both the maintenance of colonocyte-specific
nutrition and, in the case of butyrate, ileal enterocyte nutrition.

49. Essential amino acids have been advocated as standard therapy for renal failure. Which of the following statements are
true?
A. Increased survival from acute renal failure has been reported with both essential and nonessential amino acid therapy of
patients in renal failure.
B. Essential amino acids retard the rise of blood urea nitrogen (BUN) secondary to decreased urea appearance.
C. Essential amino acids and hypertonic dextrose are a convenient form of therapy for hyperkalemia.
D. Essential amino acids decrease BUN and creatinine to the same degree as solutions containing excessive nonessential
amino acids.
Answer: BC

DISCUSSION: Essential amino acids and hypertonic dextrose, as opposed to hypertonic dextrose alone, was reported by
Abel and co-workers to be associated with a decreased mortality rate in mostly surgical patients with acute tubular necrosis.
The most significant improvement in mortality, as compared with the control group receiving hypertonic dextrose, was
among patients who required dialysis (i.e., the more severely affected patients). Another group responding favorably to
treatment includes patients with nonoliguric renal failure whose need for dialysis is not clearly established. The effect of
essential amino acids in preventing a rise in BUN, as well as its beneficial effect in preventing hyperkalemia, may obviate
dialysis in such patients. With increasing amounts of nonessential amino acids, BUN increases, and thus, dialysis is
required. Prospective randomized studies comparing the use of essential versus nonessential amino acids in patients with
acute renal failure have not been carried out in sufficient numbers to yield answers to this question.

50. A modified amino acid solution with increased equimolar branched-chain amino acids and decreased aromatic amino
acids has been proposed for patients with hepatic insufficiency. Which of the following statements is/are true?
A. This formulation is proposed for the use of patients with fulminant hepatitis.
B. Nitrogen balance is achieved in such patients with amounts of 40 gm. of amino acids per 24 hours.
C. The use of 80 to 100 gm. of such solutions is associated with hepatic encephalopathy.
D. In some studies of surgical patients, improvements in mortality have been reported.
Answer: D
DISCUSSION: The use of modified amino acid solutions is based on the false neurotransmitter hypothesis of the cause of
hepatic coma. According to this hypothesis, the imbalance between aromatic and branched-chain amino acids in the plasma
results in abnormally high levels of the toxic aromatic amino acids in the brain, thus provoking hepatic encephalopathy. The
use of modified amino acid mixtures, with glucose as the calorie base, has been associated in a number of studies with
improvement in encephalopathy. Meta-analysis has concluded that the use of such solutions is indicated as therapy for
hepatic encephalopathy but has been proposed only for hepatic encephalopathy complicating acute exacerbation of chronic
liver disease. Although there are a few anecdotal reports of beneficial effects on hepatic encephalopathy of acute fulminant
hepatitis, the use of such a solution has not been advocated, but such a modified solution is tolerated better than standard
amino acid mixtures in patients requiring TPN. In some studies, particularly in complicated surgical cases, the use of a
high–branched-chain, low–aromatic amino acid solution has been associated with lower mortality. These statements are true
only for studies in which the modified solutions are given with hypertonic glucose as a calorie base. Studies in which lipid
was the principal calorie source have not revealed such improvements in mortality. In recent studies, giving an aromatic
amino acid–deficient, branched-chain amino acid–enriched solution to patients about to undergo resection of the liver has
proved particularly efficacious in a group of patients with cirrhosis, decreasing morbidity and showing a trend toward
decreased mortality.

51. In the nutritional support of patients with cancer, which of the following statements is/are true?
A. Nutritional support benefits the patient's lean body mass but does not enable the tumor to grow.
B. In experimental animals, the growth of implanted tumors is directly proportional to the amount of calories and protein
supplied.
C. Prospective randomized trials of nutritional support utilizing chemotherapy and radiation therapy have revealed benefits
to patients receiving total parenteral nutrition.
D. Studies of nutritional support for patients with cancer about to undergo surgery revealed decreased morbidity and
mortality, especially morbidity from sepsis.
Answer: B

DISCUSSION: The problem with the patient with cancer is a very vexing one. Clearly, one of the metabolic effects of
cancer, cachexia, affects patients in the last quartile of their disease and makes such patients intolerant of chemotherapy,
radiation therapy, and, in many cases, operative procedures. Total parenteral nutrition (TPN) has been proposed as a means
of reversing cachexia and enabling patients to better tolerate surgery, chemotherapy, and radiation therapy. In experimental
animals, it is clear that the provision of calories and protein, especially in excessive amounts, is associated with the more
rapid growth of tumors and decreased survival, especially in the group that is overfed in the extreme. There is also evidence
suggesting that overfeeding, or at least TPN, may result in increased growth (or at least change cell kinetics) in patients who
are overnourished with TPN. Of the randomized prospective trials that have been carried out, no trial utilizing
chemotherapy or radiation therapy has revealed a survival advantage for patients receiving TPN. Indeed, in Shamberger's
study, there is a suggestion that the tumor-free interval following treatment of lymphoma may be shorter in patients
receiving TPN. In patients undergoing surgery, however, especially those who are severely malnourished (as recently
revealed in the VA study) or in patients with major procedures such as esophagogastrectomy (as in Muller's study), evidence
suggests that TPN is beneficial. In a late follow-up in Muller's study, there was no apparent increase in recurrence, and the
survival rate was the same, despite much higher mortality in the non-TPN group. This suggests that any improved survival
following operation may have been offset by an increased late recurrence rate, although it is difficult to reach this
conclusion. In summary, for patients with cancer TPN probably nourishes the tumor as well as the host. Nonetheless, in
severely malnourished patients provision of TPN from 5 to 10 days preoperatively may increase survival and decrease
morbidity. Overfeeding must be avoided. Future studies will undoubtedly reveal that there are certain nutrients that tumors
require, which probably should be best avoided.

52. Glucose overload results in increased CO 2 production. Which of the following statements are true?
A. In patients with respiratory insufficiency, administration of glucose as a principal calorie source is contraindicated.
B. In patients with pulmonary infection and sepsis, calorie support should consist of 95% fat and 5% glucose.
C. In Askanazi's study, increased CO 2 production and difficulty in weaning was associated only with pronounced
overfeeding.
D. CO 2 production should be measured in most patients who are supported by respirators in intensive care units and are
receiving nutritional support.
Answer: C

DISCUSSION: Few papers have excited as much interest as that by Askanazi, Kinney, and co-workers, which demonstrated
that glucose calories given to patients with severe respiratory impairment may result in difficulty in weaning from a
respirator. Subsequent research has suggested, however, that this occurs only with severe overfeeding, when the respiratory
quotient is greater than 1 and calories are excessive. If one examines the conditions under which Askanazi's patients were
studied, these were a group of septic, depleted patients who were taken from almost no nutritional support to a caloric
supply of 2.25 times their caloric requirement, most of the calories consisting of glucose. Suffice it to say that, in patients
with impaired respiratory function, one should measure VCO2 and, when VCO2 is significantly elevated and appears to
interfere with weaning, decrease the amount of glucose calories and increase the amount of fat. If one measures or estimates
calorie requirements and does not overfeed, lipid can be utilized for 25% of the caloric requirement and glucose for the
remainder, without much fear of excessive CO 2 production.

53. Hepatic abnormalities have been noted in adults since the beginning of hyperalimentation. Which of the following
statements are true?
A. Hepatic steatosis appears to be associated with an overload of glucose.
B. Hepatic steatosis is usually associated with abnormalities in hepatic enzymes.
C. Hyperbilirubinemia is inevitably associated with hepatic steatosis.
D. Abnormalities in the portal insulin-glucagon ratio are thought to be causative of hepatic steatosis in experimental
animals.
Answer: AD

DISCUSSION: The most common metabolic complication of TPN in adults is hepatic steatosis. Unlike the hepatic
abnormalities in children, which may progress to cholestasis, liver damage, and in some cases death, hepatic steatosis, or
fatty infiltration of the liver with triglycerides, appears to be a rather benign complication. It may be, but is not necessarily,
associated with hepatic enzymatic abnormalities, which usually occur in the first week, peak at the third week, and generally
disappear by the sixth week of parenteral nutrition. Abnormalities in the transaminases are most common, with alkaline
phosphatase also being elevated, but there is no correlation between the degree of fatty infiltration and enzymatic
abnormalities. Fatty infiltration appears to be largely vacuolization with increased storage of triglycerides. Hepatic steatosis
is almost always associated with an overload of glucose. Recent studies in experimental animals have suggested that the
portal insulin-glucagon ratio, which is elevated under these circumstances, may be causally related to hepatic steatosis.
Insulin is the leading storage enzyme and is responsible for lipogenesis. The presence of insulin inhibits lipolysis. Glucagon,
on the other hand, results in the mobilization of hepatic lipid. The liver “sees” the portal vein insulin-glucagon ratio.
Excesses of insulin elicited by hypertonic dextrose increase lipid deposition in the liver, whereas glucagon, which is elicited
by certain amino acids, results in the mobilization of hepatic lipid.

54. Which of the following statements about the presence of gallstones in diabetes patients is/are correct?
A. Gallstones occur with the same frequency in diabetes patients as in the healthy population.
B. The presence of gallstones, regardless of the presence of symptoms, is an indication for cholecystectomy in a diabetes
patient.
C. Diabetes patients with gallstones and chronic biliary pain should be managed nonoperatively with chemical dissolution
and/or lithotripsy because of severe complicating medical conditions and a high operative risk.
D. The presence of diabetes and gallstones places the patient at high risk for pancreatic cancer.
E. Diabetes patients with symptomatic gallstones should have prompt elective cholecystectomy, to avoid the complications
of acute cholecystitis and gallbladder necrosis.
Answer: E

DISCUSSION: Gallstones have been found to be very prevalent in patients with type II (non–insulin-dependent) diabetes
mellitus, perhaps related to the dyslipoproteinemia in such patients. Although the complications of acute cholecystitis
(infection, sepsis, gangrene of the gallbladder) are more common in diabetics, a decision-analysis study has shown that
prophylactic cholecystectomy cannot be justified since the risk of morbidity and/or mortality from the cholecystectomy
procedure is as great as that of complications or death from acute cholecystitis. Patients who become symptomatic should be
promptly prepared and should undergo elective cholecystectomy, because an emergency operation in these patients with
comorbid conditions, especially coronary artery disease, has substantial added mortality associated with it. There is no
causal relationship between diabetes and pancreatic cancer.

55. Intensive insulin therapy:


A. Prevents the aggressive development of atherosclerosis in diabetic patients.
B. Is not associated with unawareness of hypoglycemia.
C. Improves peripheral neuropathy.
D. Improves established retinopathy and nephropathy.
E. Is indicated in all patients with non–insulin-dependent diabetes mellitus (NIDDM).
Answer: C

DISCUSSION: Intensive insulin therapy is indicated in patients with IDDM who can actively participate in their own
management and the attainment of the goals set for their blood glucose and glycosylated hemoglobin (HgA1 c) levels.
Because the main complication of intensive therapy is iatrogenic hypoglycemia, this mode of treatment is not indicated for
patients with NIDDM, who often have coexisting medical conditions such as coronary artery disease and who tolerate
hypoglycemia poorly. There is little or no evidence that macrovascular disease is affected by intensive insulin therapy, and
the added weight gain and hyperinsulinemia associated with the therapy may worsen atherosclerosis. Unawareness of
hypoglycemia is directly related to a recent hypoglycemia episode, so patients treated intensively are often unaware of the
problem. Intensive therapy does not improve established retinopathy or nephropathy but slows or prevents progression of
these complications; however, better glucose control may improve peripheral neuropathy.

56. Which of the following statements about hypertension in diabetes patients is/are correct?
A. Hypertension worsens the macrovascular disease of diabetes patients.
B. Hypertension accelerates the progression of diabetic nephropathy.
C. Hypertension is associated with sodium retention in diabetes patients.
D. Angiotensin-converting enzyme (ACE) inhibitors should be used in all patients with chronic hyperglycemia, regardless
of the presence of hypertension.
E. Diuretics, as single-drug therapy, are not indicated in the treatment of hypertension in diabetes patients.
Answer: ABCDE

DISCUSSION: All of the answers listed are correct. By damaging endothelial cells, hypertension worsens macrovascular
disease in all patients but especially in diabetics. Hypertension dramatically accelerates the onset and progression of diabetic
renal disease and proteinuria, and this phenomenon can be slowed or prevented by a combination of treatment modalities,
including ACE inhibitors, which dilate efferent glomerular vessels and lower intraglomerular pressure. Despite sodium
retention in diabetes patients, single-drug therapy with a diuretic is not indicated because the chronic state of dehydration in
such patients may become worse.

57. What is the major determinant in an individual patient's risk for perioperative complications?
A. The surgical procedure.
B. The length of the surgical procedure.
C. The anesthetic technique (e.g., general, regional).
D. The length of anesthesia.
E. All of the above.
Answer: A

DISCUSSION: The planned surgical procedure is the major determining factor in assessing an individual patient's risk for
perioperative complications and in deciding which anesthetic technique will be most appropriate. Good communication
between the surgeon and the anesthesiologist is vital, as the surgeon knows better than anyone else the extent of the
operation and the length of time it will require.

58. Which of the following are considered routine intraoperative monitors?


A. Temperature probe.
B. Electrocardiogram.
C. Capnograph.
D. Blood pressure cuff.
E. Foley catheter.
Answer: ABD

DISCUSSION: The American Society of Anesthesiologists requires that the patient's ventilation, circulation, oxygenation,
and temperature be continually monitored during all anesthetics. Routine monitors are considered to be a temperature probe,
electrocardiogram, pulse oximetry, and blood pressure cuff.
59. Muscle relaxants can be used for which of the following?
A. To facilitate intubation.
B. To provide optimal surgical conditions.
C. To optimize ventilator support.
D. To provide sedation.
Answer: ABC

DISCUSSION: Muscle relaxants are administered to facilitate endotracheal intubation, to provide the surgeon with optimal
working conditions during anesthesia, and to optimize mechanical ventilator support in some patients. They do not produce
analgesia, sedation, or amnesia. Therefore, muscle paralysis should not be performed without sedation or general anesthesia.

60. Local anesthetics:


A. Inhibit transmission of nerve impulses by increasing sodium membrane permeability and the displacement of ionized
calcium.
B. Are classified as amides or esters.
C. Produce peripheral vasodilation.
D. Are weak acids.
Answer: BC

DISCUSSION: Local anesthetics act within the nerve membrane, where they inhibit transmission of nerve impulses by
reducing sodium membrane permeability and the displacement of ionized calcium. All local anesthetics consist of a
hydrophilic region and a hydrophobic region separated by an alkyl chain. The bond of the alkyl chain is either an ester or an
amide, and these drugs are classified based on this bond. All local anesthetics except cocaine produce vasodilatation and are
weak bases.

61. Absolute indications for a double-lumen endotracheal tube during thoracic surgery are:
A. Massive hemorrhage from one lung.
B. Unilateral lung infection.
C. Facilitation of surgical exposure.
D. Unilateral bronchopulmonary lavage.
E. All of the above.
Answer: ABD

DISCUSSION: The absolute indications for a double-lumen tube are for the purposes of protecting one lung from the other.
These indications include ventilation with a bronchopleural fistula, massive hemorrhage from one lung, pulmonary air cyst
resection, unilateral lung infection, and unilateral bronchopulmonary lavage. Relative indications include facilitation of
surgical exposure, for pneumonectomy, upper lobectomy, and thoracic aneurysm repair.

62. Determinants of cerebral blood flow include:


A. Preoperative neurologic dysfunction.
B. Arterial CO 2 tension.
C. Arterial O 2 tension.
D. Systemic arterial pressure.
E. All of the above.
Answer: BCD

DISCUSSION: Determinants of cerebral blood flow include arterial CO 2 and O 2 tensions, systemic arterial pressure, and
temperature. Other factors that may affect cerebral blood flow and intracranial pressure are head position, jugular venous
obstruction, and positive end-expiratory pressure.

63. Discharge criteria following ambulatory surgery include:


A. Ability to eat solid food.
B. Stable vital signs.
C. Ability to ambulate.
D. Ability to have protective airway reflexes.
Answer: BCD

DISCUSSION: Discharge criteria following ambulatory surgery include the patient's being fully awake and oriented, the
ability to have protective airway reflexes, stable vital signs, adequate hydration with the ability to hold down oral intake, the
ability to ambulate, and adequate pain control. All patients must have a competent person with them to transport them—and
ideally to stay with them on the first postoperative night.

64. Advantages of patient-controlled analgesia (PCA) include:


A. Immediate medication delivery.
B. Less contact with nursing staff.
C. Rapid onset of analgesia.
D. Patient control over pain medication.
E. All of the above.
Answer: ACD

DISCUSSION: Advantages of PCA are immediate medication delivery, rapid onset of analgesia, and patient control over
pain medication. Disadvantages of PCA are less contact with nursing staff and patients' fears that they could inadvertently
administer an overdose or possibly become addicted to the opioid.

65. Advantages of epidural analgesia include:


A. Earlier mobilization after surgery.
B. Earlier return of bowel function.
C. Shorter hospitalizations.
D. Decreased stress response to surgery.
E. All of the above.
Answer: E

DISCUSSION: Epidural analgesia include excellent pain relief, decreased sedation with more rapid recovery to presurgical
levels of consciousness, earlier mobilization after surgery with increased ability to co-operate with respiratory therapy and
physical therapy. Following vascular surgery epidural analgesia may also improve graft flow through mild sympathetic
blockade. Earlier return of bowel function, decreased stress response, shorter hospitalizations, and decreased morbidity have
all been associated with epidural analgesia.

66. Ketorolac:
A. Is a nonsteroidal anti-inflammatory drug (NSAID) approved for intravenous, intramuscular, and oral administration.
B. Can be used indefinitely for postoperative analgesia.
C. Can cause renal dysfunction.
D. May decrease surgical blood loss.
Answer: AC

DISCUSSION: Ketorolac tromethamine, an NSAID, is approved by the FDA for intravenous, intramuscular, and oral
administration. The agent is an effective analgesic with minimal side effects; however, ketorolac, like all NSAIDs, can
enhance surgical bleeding and cause renal and platelet dysfunction. Additionally, it is recommended that ketorolac should
not be used for more than 5 consecutive days.

67. Factors that decrease collagen synthesis include all of the following except:
A. Protein depletion.
B. Infection.
C. Anemia.
D. Advanced age.
E. Hypoxia.
Answer: C

DISCUSSION: Collagen synthesis, an integral part of wound healing, is affected by many local and systemic factors.
Protein depletion impairs fibroplasia. Hypoproteinemia leads to diminution of fibroblast proliferation, proteoglycan and
collagen synthesis, angiogenesis, and wound remodeling. Although anemia was once believed to be a significant cause of
wound disruption, studies have shown that, in the absence of malnutrition or hypovolemia, anemia with a hematocrit greater
than 15% does not interfere with wound healing. In contrast, molecular oxygen is critical for collagen synthesis because it is
one of the factors required for the hydroxylation of lysine and proline. Also, hypoxia favors wound infection. The role of
age in collagen synthesis is not clear, but the incidence of wound failure and incisional hernias is greater in patients older
than 60. Fibroplasia occurs at a slower rate in older animals. Perhaps more than any other factor, wound infection is
associated with the risk of wound failure.

68. Wound contraction and ultimate contracture may be controlled by which of the following drugs?
A. Colchicine.
B. D-Penicillamine.
C. Thiphenamil (Trocinate).
D. Glucocorticoids.
E. Ibuprofen (Motrin).
Answer: AC

DISCUSSION: Wound contraction is carried out by highly specialized cells called myofibroblasts, which, as their name
implies, have histologic characteristics of fibroblasts and smooth muscle cells. The activity of these cells, and therefore
wound contraction, can be influenced by topical application of smooth muscle inhibitors such as thiphenamil. Inhibitors of
microtubule formation in myofibroblasts, such as colchicine and vinblastine, also inhibit wound contraction under
experimental conditions. Glucocorticoids and NSAIDs do not affect the wound contraction process.

69. Which of the following is/are true of the actions of transforming growth factor beta (TGF-b) during wound repair?
A. Increased matrix and proteoglycan synthesis.
B. Inhibition of proteases.
C. Stimulation of plasminogen inhibitor.
D. Chemotaxis for fibroblasts and macrophages.
E. Autoinduction of TGF-b.
Answer: ABDE

DISCUSSION: Through autocrine and paracrine mechanisms TGF-b stimulates the deposition of collagen and other matrix
components by fibroblasts, inhibits proteases, blocks plasminogen inhibitor, enhances angiogenesis, and is chemotactic for
fibroblasts, monocytes, and macrophages. TGF-b modulates the expression of cell-surface integrins in a manner that
enhances cell-matrix interaction and matrix assembly. TGF-b also induces cell production by cells, thus amplifying its
biologic effects. The sustained production of TGF-b at the wound site leads to tissue fibrosis.

70. In contrast to adult wound healing with scar formation, which of the following are characteristic of scarless fetal skin
repair?
A. Matrix rich in hyaluronic acid.
B. Increased inflammatory response.
C. Increased production of TGF-b.
D. No collagen deposition.
E. Minimal angiogenesis.
Answer: AE

DISCUSSION: The ability of a fetus to heal without scar formation depends on its gestational age at the time of injury and
the size of the wound defect. In general, linear incisions heal without scar until late in gestation, whereas excisional wounds
heal with scar at an earlier gestational age. The profiles of fetal proteoglycans, collagens, and growth factors are different
from those in adult wounds. The less differentiated state of fetal skin is probably an important characteristic responsible for
scarless repair. There is minimal inflammation and angiogenesis in fetal wounds. Fetal wounds are characterized by high
levels of hyaluronic acid and its stimulator(s) with more rapid, highly organized collagen deposition. The roles of peptide
growth factors such as TGF-b and basic fibroblast growth factor are less prominent in fetal than in adult wound healing. An
understanding of scarless tissue repair has possible clinical applications in the modulation of adult fibrotic diseases and
abnormal scar-forming conditions.
71. Which of the following cell types are not crucial for healing a clean, incisional wound?
A. Macrophage.
B. Platelet.
C. Fibroblast.
D. Polymorphonuclear leukocyte.
E. Myofibroblast.
Answer: DE

DISCUSSION: Experimental studies have shown that healing may progress normally in the absence of polymorphonuclear
leukocytes in an uninfected wound. In contrast, depletion of monocytes and macrophages causes a severe alteration in
wound healing with poor débridement, delayed fibroblast proliferation, and inadequate angiogenesis. Platelets carry a cadre
of biologically active substances that are important for wound repair, including peptide growth factors like platelet-derived
growth factor (PDGF) and TGF-b. Fibroblasts are the principal cell for matrix synthesis and deposition. Myofibroblasts are
important for wound contraction in open defects but have little if any role in clean, incisional wounds.

72. Which of the following is/are not a substrate or cofactor for prolyl hydroxylase?
A. Alpha-ketoglutarate.
B. Ascorbate.
C. Biotin.
D. Oxygen.
E. Copper.
Answer: CE

DISCUSSION: Prolyl hydroxylase is one of the rate-limiting enzymes in collagen synthesis. Substrates and cofactors such
as iron, alpha-ketoglutarate, ascorbate, and oxygen are important participants in this process. If insufficient prolines are
hydroxylated, then the alpha-peptide collagen chains cannot assume a stable triple helix, the collagen cannot be exported
from the fibroblasts, and the incomplete, unassociated alpha chains are broken down. Thus, ascorbate deficiency (scurvy)
and hypoxia have similar effects on collagen synthesis.

73. Which of the following is an adhesion glycoprotein?


A. Fibronectin.
B. Tenascin.
C. Laminin.
D. Hyaluronic acid.
E. Collagen type IV.
Answer: ABC

DISCUSSION: Cell adhesion glycoproteins such as fibronectin, vitronectin, laminin, and tenascin provide a “railroad track”
to facilitate epithelial and mesenchymal cell migration over the wound matrix. Hyaluronic acid is a glycosaminoglycan, and
collagen type IV is a protein that is a crucial component of basement membrane.

74. Which of the following is/are true concerning wound fibroblasts?


A. Fibroblasts synthesize and secrete collagen molecules.
B. Wound fibroblasts are derived from blood-borne precursor cells.
C. Fibroblasts migrate to a wound along fibrin strands, which are used as a scaffold.
D. Large amounts of fibrin or blood clot can act as a physical barrier to fibroblast penetration, which delays normal wound
healing.
Answer: ACD

DISCUSSION: Fibroblasts appear in the wound on about the third day of healing and begin to synthesize and secrete
collagen molecules. Wound fibroblasts arrive from cells surrounding the wound (e.g., the adventitia of blood vessels),
change their phenotype(s), and become mobile during the process of replication. Fibroblasts migrate into a wound using the
provisional fibronectin and fibrin matrix as a scaffold. Fibroblasts do not have fibrinolytic enzymes, and large amounts of
fibrin and blood clot prevent fibroblasts from entering the wound.
75. Which of the following is/are true?
A. Because of its thickness, the tensile strength of a healing wound on the eyelid is much less than one on the thick skin of
the back.
B. By 2 days, the experimental burst strength of skin is minimal since collagen has been formed in the wound but has not
yet cross-linked.
C. Wound strength reaches a plateau by 3 weeks.
D. Wounds rarely, if ever, regain the strength of intact tissues.
Answer: BD

DISCUSSION: Tensile strength measures load per cross-section area at rupture, whereas burst strength measures load
required to break a wound, regardless of dimension. Therefore, skin wounds have comparable tensile strength, regardless of
thickness. Collagen appears in the wound by 3 to 4 days. Minimal wound strength on day 2 is due to fibrin polymerization
and adhesion of globular proteins. Wounds rapidly gain strength for about 4 months and then continue to gain strength at a
slower rate for more than a year. Wounds do not regain the strength of normal tissue.

76. Which of the following interfere with normal collagen formation or cross-linking?
A. Beta-aminopropionitrile.
B. Iron chelators.
C. Vitamin C depletion.
D. Proline analogs (e.g., cis-hydroxyproline).
E. D-Penicillamine.
Answer: ABCDE

DISCUSSION: Intramolecular and intermolecular cross-links are crucial for collagen structural stability. Formation of
cross-links can be inhibited by two pharmacologic agents: beta-aminopropionitrile inhibits the enzyme lysyl oxidase, and D-
penicillamine binds to collagen substrate directly to prevent collagen cross-link formation. Iron is a cofactor for prolyl
hydroxylase, which is important for collagen synthesis. In high enough concentration, proline analogs prevent collagen
formation with minimal effects on noncollagenase protein synthesis.

77 Which of the following statement(s) is/are true concerning the cell plasma membrane?
a.The plasma membrane is composed of amphipathic molecules
b.The hydrophobic core of the lipid bilayer of the cell membrane contains specialized transport proteins which
maintain the intracellular ionic milieu different from the extracellular fluid
c.Plasma membrane proteins extend externally and bear phospholipid moieties which contribute to the cell coat
d.The membrane proteins of nerve cells are highly voltage-dependent
Answer: a, b, d

The plasma membrane defines the boundary of the cell and serves to contain and concentrate enzymes and other
macromolecule constituents. The plasma membrane is composed of amphipathic molecules, mainly phospholipids and
proteins that contain distinct regions that are either insoluble in water (hydrophobic) or soluble in water (hydrophilic). The
plasma membrane forms a continuous barrier between the aqueous extracellular and intracellular fluids. Transport proteins
in the membrane act as regulated channels or transporters to maintain the intracellular ionic milieu that is clearly different
from the extracellular milieu. In some cells, membrane proteins are diversified such as in nerve cells where the ion channels
are highly voltage-dependent, providing the basis for information transmission in the form of electrical impulses. Most
plasma membrane proteins extend externally and bear carbohydrate moieties primarily as oligosaccharide chains that
contribute to the cell coat or glycocalyx.

78 Which of the following statement(s) is/are true concerning water movement across cell membranes?
a.Water moves only actively through cell membrane transport proteins
b.For most cells of the body, the transmembrane hydrostatic pressure is 0
c.Water distribution is determined entirely by solute distribution
d.Specialized cells such as the glomerulus of the kidney actively transport water to maintain hydrostatic pressure
Answer: b, c

The energetics of water transport across cell membranes is simplified by the fact that water moves only passively due to
gradients of hydrostatic pressure or water concentration. Hydrostatic pressure is an important driving force only for certain
specialized cells—the capillary endothelium and the glomerulus of the kidney. For most cells of the body, the
transmembrane hydrostatic pressure is 0 and water moves only in response to water concentration gradients. Because the
concentration of water is determined by the amount of dissolved solute, the difference in water concentration is typically
expressed as a function of the difference in solute concentration or osmotic pressure difference. Because there are no
specialized, energy-converting transport mechanisms for water, water is distributed at equilibrium. Water distribution is
determined entirely by solute to solute distribution.

79 The transport of proteins out of the cell is termed exocytosis. Which of the following statement(s) is/are true
concerning this process?
a.Secretory vesicles fuse with the plasma membrane
b.The process can occur in either a constitutive or regulated process
c.A regulated secretion is triggered by a stimulus, most likely a hormone or a neurotransmitter
d.A decrease in cytoplasmic calcium occurs as part of the secretion process
Answer: a, b, c

Transport vesicles that bud off the Golgi network carry both material to be secreted from the cell and protein destined to
become components of the plasma membrane. These vesicles can fuse with the plasma membrane in a process termed
exocytosis. Vesicular transport to the cell surface can be divided into two components, constitutive and regulated secretion.
Regulated secretion occurs in cells secreting digestive enzymes, hormones and other regulatory molecules, and
neurotransmitters. In regulated secretion, the material to be secreted is sorted in a storage vesicle or granule; fusion with the
plasma membrane in exocytosis then takes place in response to external stimulation. Regulated secretion is triggered in
most cases by a hormone or neurotransmitter. The ensuing process is termed stimulus-secretion coupling. In most cases the
coupling involves an increase in cytoplasmic concentration of Ca++, but may also involve generation of diacylglycerol or
production of cyclic AMP which activate kinases or phosphatases.

80 Which of the following statement(s) is/are true concerning the cell function of phagocytosis?
a.Phagocytosis is a mechanistically distinct process of endocytosis performed by special cells to take up larger
particles such as bacteria or erythrocytes
b.Lymphocytes are the primary blood cell involved with this process
c.The process involves a coating of the cytoplasmic surface known as clathrin
d.Phagocytosis is performed only by white blood cells and tissue macrophages
Answer: a

Phagocytosis is a specialized form of endocytosis by which large particles are internalized by specialized cells primarily
macrophages and neutrophils. To be phagocytosed, particles must bind to the surface of the phagocytic cell, usually as the
result of specific antibody coating the particle. The phagocytic cell then extends pseudopods which engulf the particle. This
event is followed by membrane fusion and a pinching off. As opposed to endocytosis, this process does not involve the
membrane protein, clathrin, but rather actin. A physiologically relevant site of phagocytosis is the thyroid gland, where
thyroid follicular cells phagocytose and digest thyroglobulin from the lumen of the thyroid follicle, thereby releasing the
thyroid hormones, thyroxine triiodothyronine.

81 A striking feature of living cells is a marked difference between the composition of the cytosol and the extracellular
milieu. Which of the following statement(s) concerning the mechanisms of maintenance of these differences is/are true?
a.The cell membrane is able to maintain a 10,000 fold gradient between the extracellular concentration of ionized
calcium and the intracellular concentration
b.The key to these differences is the fact that the plasma membrane is normally impermeable to sodium, potassium
and calcium
c.The selectivity of biologic membranes is highly consistent and seldom changes
d.The selectivity of cell membranes relates only to ions and not organic compounds

Answer: a

The survival of the cell requires that cytosolic composition be maintained within narrow limits, despite the constant influx
of nutrients and the simultaneous outflow of waste. A familiar example of the distribution of ions across the cell membrane
is that of sodium and potassium. Cells are typically rich in potassium and contain very little sodium. Despite the fact that
they are constantly bathed by fluid that is precisely the opposite composition. Even more impressive is the distribution of
ionized calcium. The extracellular concentration of this ion is typically of the order of 10–3M, whereas that of cytosol is
typically 10–7M, a 10,000-fold gradient. Such nonequilibrium ion distributions are even more remarkable in light of the fact
that the plasma membrane is, to varying degrees, leaky to ions such as sodium, potassium and calcium. The plasma
membrane is leaky to a variety of substances, but it exhibits an astonishing ability to discriminate or select one substance
over another. This selectivity relates to not only ions but also for organic compounds such as glucose. Finally, the selectivity
of biologic membranes can be altered drastically as a result of regulatory or signaling processes that occur within the cell.

82 Which of the following statement(s) is/are true concerning DNA?


a.DNA is contained only in the nucleus of the cell
b.DNA strands are encoded by the sequence of four bases—adenine, guanine, cytosine and uridine
c.The basic unit of information of DNA is the intron, a sequence of three bases
d.There are an infinite number of possible codons
Answer: a

The genetic blueprint of an organism is carried in the nucleus of every cell, encoded by the sequence of four bases—
adenine, guanine, cytosine and thymine, which together make up two long chains bound together by hydrogen bonds to
form a DNA double helix. A gene is a segment of DNA that is transcribed into a corresponding RNA molecule that either
codes for a protein or forms a structural RNA molecule. Genes are commonly between 10,000 and 100,000 base pairs in
length and include, in addition to the coding sequence, flanking regions and intervening sequences, termed introns. Introns
are removed from the primary RNA transcript by a process called splicing. The basic unit of information is the codon, a
sequence of three bases or triplet. The four nucleotide bases arranged as triplets lead to 64 possible codons. Sixty-one of
these code for amino acids and three are termination signals called stop codons.

83 Which of the following statement(s) is/are true concerning cell membrane receptors?
a.The largest family of cell surface receptors are the G-protein-linked receptors
b.Activities of the G-protein involve binding and hydrolysis of ATP
c.The G protein receptor generates an intracellular messenger commonly through the use adenylate cyclase
d.Tyrosine kinase receptors are considered G-protein-linked receptors
Answer: a, c

All water-soluble regulatory molecules bind to the cell surface receptor proteins. Binding of the appropriate ligand evokes
an intracellular signal which usually regulates enzyme activity, membrane transport, or in some cases gene expression. Most
cell surface receptors belong to one of three functional classes—these are ion channel receptors, catalytic receptors, and G-
protein-linked receptors. Ion channel receptors are multisubunit assemblies which, with each subunit, have a multiple
membrane spanning segment. Together these subunits form an ion-selected pore that can be gated by a change in
transmembrane electrical potential or binding of a ligand to one of the subunits. Catalytic receptors are membrane proteins
that possess enzymatic activity. The best understood receptors of this class are the tyrosine kinases. The largest family of
cell surface receptors are the G-protein-linked receptors. G-proteins are a family of proteins that bind and hydrolyze GTP.
The final component of single transduction by G-protein-linked cell surface receptors is the effector that generates the
intracellular messenger. The two best understood effectors are adenylate cyclase, which converts ATP to cAMP, and the
polyphosphoinositide-specific phospholipase C.

84 Which of the following statement(s) is/are true concerning cellular ion channels?
a.Ion channels are transmembrane proteins that form pores that can conduct ions across the plasma membrane
b.Ion channels are formed by membrane-spanning peptides that are arranged so that polar moieties line a central core
c.Ion channel proteins undergo conformational changes between open states and closed states
d.Ion channels can be blocked
Answer: a, b, c, d

Ion channels are transmembrane proteins that form pores that can conduct ions across the plasma membrane. Ion channels
are formed by membrane-spanning peptides that are arranged so that polar moieties line a central pore. These polar groups
take the place of the water of hydration, which stabilizes an ion in an aqueous solution creating, in essence, a water-like
environment into which the ion can partition and move in the presence of the appropriate driving force. Ion channels are
permissive transport elements. Ions flow through a channel only through the presence of an appropriate driving force. Ion
channels do not conduct all the time, rather the channel protein undergoes conformational changes between a conducting
(open) state and nonconducting (closed) state. These conformational changes are collectively referred to as gating. The
conduction process can also be blocked by ions or organic compounds that enter the channel, bind there, and occlude the
pore.

85 Which of the following statement(s) is/are true concerning carrier proteins?


a.Carrier proteins are distinguished by three types of mechanisms: carrier-type, channel-type, and conduction-type
b.Conformational changes in the membrane protein occur between the conducting and the nonconducting states
c.A channel-type carrier protein has two states—closed and open
d.Carrier-type transport proteins are equally accessible from either side of the membrane
Answer: b, c

Most transport proteins appear to function as carriers, rather than channels. Important distinctions can be made between
types of carrier proteins on the basis of transport kinetics. Two primary types can be distinctly identified based on carrier-
type and channel-type mechanisms. The most important difference between the channel mechanism and the carrier
mechanism is the role in the transport event played by conformational changes in the membrane protein. The channel is
depicted as having two states, closed and open, so that it operates like a switch. In contrast, carrier transport is envisioned as
requiring a cycle of conformational changes. The transport of one molecule of substrate requires one complete cycle of the
protein. In a channel mechanism, binding sites within the open pore are equally accessible from either side of the
membrane, whereas in a carrier mechanism, the binding site is available only one side of the membrane at any instant.

86 Which of the following statement(s) is/are true concerning translation of the mRNA message to protein synthesis?
a.An adaptor molecule, tRNA, recognizes specific nucleic acid bases and unites them with specific amino acids
b.Covalent attachment of tRNA to amino acids is energy dependent
c.The formation of a peptide bond between the growing peptide chain and the free amino acid occurs in the free
cytoplasm
d.Complete protein synthesis takes hours
Answer: a, b

The synthesis of protein involves conversion from a four-letter nucleotide language to one of 20 chemically distinct amino
acids. This process is referred to as translation. There is no mechanism for direct chemical recognition between specific
nucleic acid bases and specific amino acids. Instead, an adaptor molecule, tRNA, is used. Each tRNA carries only one
amino acid and must be recognized by a distinct enzyme which catalyzes the covalent attachment of the carboxyl end of the
amino acid to the end of the tRNA in a process using ATP. Protein synthesis occurs by the formation of a peptide bond
between the carboxyl terminal of the growing peptide chain and the free amino acid of deactivated amino acid tRNA. This
event does not occur in free solution, but within ribosomes. Ribosomes are protein synthesizing machines that bring all of
the necessary components together in the correct sequence and spacial orientation. Protein synthesis consumes a great deal
of energy because four high-energy phosphate bonds must be split to make each peptide bond. Complete synthesis of a
single protein takes 30 seconds to a few minutes, but multiple ribosomes can initiate translation and be moving down the
mRNA molecules simultaneously, thus increasing the rate of protein synthesis.

87 Cell regulation can be thought of as the effector side of cell communication. Most commonly cell regulation occurs
by means of extracellular chemical messengers. Which of the following statement(s) is/are true concerning these
messengers?
a.Paracrine regulation involves a messenger which is produced and acts systemically
b.The extracellular signal or stimulus is received by a receptor on or in the target cell
c.Neurocrine regulation depends on a physical connection between the neuron and the target cell
d.Most hormones, local mediators, and neurotransmitters readily cross the cell plasma membrane
Answer: b, c

Depending on how the extracellular messenger arrives, cell regulation can be classified as paracrine, endocrine, or
neurocrine. In paracrine regulation, a chemical messenger or mediator is produced and acts locally. In endocrine regulation,
the extracellular messengers (hormones) are released into the blood and act on target cells anywhere on the body that has
appropriate receptors. In neurocrine regulation, neurons secrete transmitters into highly localized regions, the synaptic cleft,
so that the regulation depends on a physical connection between the neuron and the target cell as well as the presence of a
specific receptor. In almost all cases of cell regulation, the extracellular signal or stimulus is restricted to being an
informational molecule. This information is received by receptor on or in the target cell, which generally has an affinity for
the signal molecule. Most hormones, local mediators, and neurotransmitters are water-soluble and cannot readily cross the
plasma membrane.

88 Proteins that are destined to be secreted from the cells must pass through a series of organelles. These organelles
include:
a.Endoplasmic reticulum
b.Golgi apparatus
c.Mitochondria
d.Lysosomes
Answer: a, b, d

Proteins targeted for the secretory pathway most commonly begin with translocation from the cytoplasm across the lipid
bilayer into the lumen of the endoplasmic reticulum. It must then pass through a number of compartments including the
Golgi apparatus where they are further processed and sorted and end up in a secretory vesicle or lysosome.

89 The best understood intracellular messenger is cyclic AMP (cAMP). Which of the following statement(s)
concerning this intracellular messenger is/are correct?
a.Intracellular cyclic AMP is constantly degraded by a specific enzyme, cAMP phosphodiesterase
b.Most of the actions of cAMP are mediated by activation of protein kinase A
c.Intracellular levels of cAMP are relatively stable and change solely in response to activation of adenylate cyclase
d.cAMP is the only cyclic nucleotide active as an intracellular messenger
Answer: a, b

The prototypic intracellular messenger is cAMP. To function as a mediator, the concentration of cAMP must change rapidly.
In resting cells, cAMP is continuously being degraded by a specific enzyme, cAMP phosphodiesterase. cAMP levels can
increase 10-fold or more within seconds of receptor binding through activation of adenylate cyclase. cAMP acts as an
allosteric regulator, and most, if not all, of its actions are mediated by activation of cAMP-dependent protein kinase A.
cAMP is not the only cyclic nucleotide active as an intracellular messenger. Most animal cells also produce cGMP.
Intracellular calcium ions also serve as second messengers in a large number of cells.

90 The activities of the cytoskeleton is dependent on which of the following types of filaments?
a.Microtubules
b.Intermediate filaments
c.Actin filaments
d.None of the above
Answer: a, b, c

The cytoskeleton is a collection of filamentous protein structures that allow cells to assume and maintain a variety of
shapes, to produce directed movement of organelles within the cell, and to affect movement of the entire cell relative to
other cells. These multiple activities depend upon three main types of filaments: actin filaments, intermediate filaments, and
microtubules.

91 Intracellular organelles involved with protein synthesis include:


a.Mitochondria
b.Endoplasmic reticulum
c.Golgi complex
d.Lysosomes
Answer: b, c

Mitochondria are the major source of energy production in eukaryotic cells. The endoplasmic reticulum is the network of
interconnected membranes forming closed vesicles, tubules, and saccules. The endoplasmic reticulum has a number of
functions and is primarily involved in the synthesis of proteins and lipids. Adjacent to the rough endoplasmic reticulum and
functionally involved in the sorting and package of secreted protein is the Golgi complex. Lysosomes are membrane-limited
organelles containing acid hydrolytic enzymes that degrade polymers such as proteins, carbohydrates, and nucleic acids.

92 An important step in protein synthesis is transcription. Which of the following statement(s) is/are true concerning
this process?
a.The first step in gene transcription involves separating the double helix of DNA by an enzyme known as DNA
polymerase
b.The initial product of DNA transcription is called heterogeneous nuclear RNA which codes directly for proteins
c.After processing is complete, the mRNA is exported from the nucleus to the cytoplasm
d.Only one protein can be produced from an initial mRNA strand
Answer: c

Transcription of a gene begins at an initiation site associated with a specific DNA sequence, termed a promoter region. After
binding to DNA, the RNA polymerase opens up a short region of the double helix to expose the nucleotides. Once the two
strands of DNA are separated, the strand containing the promoter acts as a template to which ribonucleoside triphosphates
base pair by hydrogen bonds. The initial products of transcription are known as heterogeneous nuclear RNA because of their
large size variation. These primary transcripts are then processed to form mRNA. RNA splicing accounts for mature RNA
being much shorter than nuclear RNA. Moreover, alternative splicing can lead to the production of different mRNA
molecules and in some cases different proteins from the same gene. mRNA is exported from the nucleus only after
processing is complete.

93 There are two properties of the cell necessary to maintain nonequilibrium cellular composition; the first is
selectivity and the second is energy conversion. Which of the following statement(s) is/are true concerning energy
converting transport?
a.The site of energy conversion and transport in the plasma membrane involves the phospholipid component
b.The Na+-K++-ATPase derives energy from hydrolysis of extracellular ATP
c.In some systems, energy inherent in the transmembrane ion gradient can be used to drive transport of a second
species
d.Examples of species transported via secondary active transport include hydrogen ions, calcium, amino acids and
glucose
Answer: c, d

The selectivity of the plasma membrane, although impressive, cannot account for the nonequilibrium composition of living
cells. A cell can be maintained in a nonequilibrium state only by continual expenditure of energy. The maintenance of a
steady-state, nonequilibrium cellular composition is possible because the plasma membrane is the site of energy converters,
membrane proteins that function as biologic transport machines using energy derived from metabolic processes to perform
transport work. The archetype for the biologic transport machine is the Na+-K+-ATPase, a membrane protein that
hydrolyses cytosolic ATP and couples the resulting free energy to transport of Na+ and K+. A second equally important type
of energy-converting transporter is one in which the energy inherent in a transmembrane ion gradient, usually that of Na+
can be used to drive the transport of a second species such as protons, calcium, amino acids, or glucose.

94 Which of the following statement(s) is/are correct concerning cell junctions?


a. The major occluding junction is the tight junction or zonula occludens
b. Tight junctions are usually located near the basal pole of the cell
c. Desmosomes are button-like points of attachment which serve to weld together adjacent cells
d. Gap junctions are a type of cell junction specialized for cell communication
Answer: a, c, d

Cell junctions are classified as occluding, anchoring, and communicating. The major occluding junction is the tight junction
or zonula occludens which connects cells in epithelia and thereby allows epithelia to serve as selective permeability barriers.
Tight junctions are normally located near the apical pool of the cell and form a belt that completely encircles the cell.
Anchoring junctions connect the cytoskeleton of the cell to the extracellular matrix or neighboring cells. Morphologically
these are adherens junctions or desmosomes. Desmosomes are button-like points of attachment with a prominent
intracellular plaque that serves to weld together adjacent cells by serving as anchoring sites for intermediate filaments
within the cell. The third functional type of cell junction is a gap junction which is specialized for communication. This
junction mediates both electrical and chemical coupling.

95 Examples of ion channel blockers include:


a. Tetrodotoxin
b. Amiloride
c. Xylocaine
d. None of the above
Answer: a, b, c

Channel blockade is an important mechanism of action for toxins and some therapeutic agents. The deadly toxin of the
puffer fish, tetrodotoxin, acts by blocking the Na+ channels that are responsible for the conduction of nerve impulse. The
diuretic, amiloride, acts by blocking the Na+ channels that inhabit the apical membrane of the epithelial cells of the distal
nephron. Local anesthetics such xylocaine also act by blocking ion channels.

96 Most hormone receptors are localized on the cell membrane and transduce hormone binding into altered levels of
intracellular messengers. A limited number of intracellular receptors do exist. Which of the following statement(s) is/are
true concerning intracellular receptors?
a. The messengers or hormones must by lipophilic
b. These intracellular receptors generally regulate protein synthesis
c. The intracellular receptors are located entirely in the nucleus of the cell
d. A heat-shock protein serves as an inhibitor protein blocking the DNA-binding domain of the steroid receptor
Answer: a, d

Although most hormone and other messenger receptors are extracellular, intracellular receptors have been identified. The
hormone messengers involved for these receptors are primarily steroid and thyroid hormones and are lipophilic. By virtue of
their hydrophobic nature, they are able to readily penetrate the lipid portion of the cell membrane. Receptors for these
hormones exist intracellularly in the cytoplasm or in the nucleus and generally act as regulators of gene expression. These
hydrophobic signaling molecules exist in plasma bound to protein, so that the concentration of this class of regulators does
not fluctuate rapidly in plasma and their actions are generally slower in onset and more prolonged than those of water-
soluble class. Some types of steroid receptors, particularly for glucocorticoids, are located in the cytosol in the inactive
state. Once the ligand binds, the receptor undergoes a conformational change, termed activation. This allows cytoplasmic
receptors to move into the nucleus and bind to DNA. Receptors already in the nucleus increase their affinity for DNA. In the
case of glucocorticoid receptors and probably others of this class, the inactive receptor is associated with another protein,
the heat-shock protein. They block the DNA-binding domain of the receptor. Activation involves the dissociation of the
inhibitor protein.

97 Altering the amino acid profile in total parenteral nutrition solutions can be of benefit in certain conditions. Which
of the following conditions are associated with a benefit by supplementation with the amino acid type listed?
a. Acute renal failure and essential amino acids
b. Hepatic failure and aromatic amino acids
c. Short gut syndrome and glutamine
d. Chronic renal failure and essential amino acids
Answer: a, c

In a number of conditions, altering the amino acid profile of the total parenteral nutrition solution can be of benefit. TPN
with amino acids of high biologic value may decrease the mortality in patients with acute renal failure. These solutions,
containing high quality amino acids, can improve nitrogen balance and diminish urea nitrogen. Provision of essential amino
acids only allows the body to maximally utilize nitrogen for the synthesis of non-essential amino acids and thereby helps
prevent rapid rises in blood urea nitrogen. There appears to be no advantages to using essential amino acids if the patient is
already being dialyzed every other day and therefore a balanced standard amino acid solution is recommended. Because of
liver damage and portasystemic shunting, patients with hepatic failure develop derangements in circulating levels of amino
acids. The plasma aromatic/branch chain amino acid ratio is increased favoring the transport of aromatic amino acids across
the blood brain barrier. These amino acids are precursors of false transmitters which contribute to lethargy and
encephalopathy. Treatment of individuals with liver failure with solutions enriched in branch chain amino acids and
deficient in aromatic amino acids results in improved tolerance to administration of protein and clinical improvement in
encephalopathic states. Glutamine-enriched TPN partially attenuates villous atrophy and may be useful in treatment of short
gut syndrome.

98 Under certain circumstances, the gut may become a source of sepsis and serve as the motor of systemic
inflammatory response syndrome. Microbial translocation is the process by which microorganisms migrate across the
mucosal barrier to invade the host. Which of the following mechanisms can promote bacterial translocation?
a. An increased number of gut bacteria
b. Altered intestinal mucosal permeability
c. Decreased host defense mechanisms
d. Lack of enteral feeding

Answer: a, b, c, d

99 Translocation is promoted in three general ways: 1) altered permeability of the intestinal mucosa as caused by
shock, sepsis, distant injury, or cell toxins; 2) decreased host defense (secondary to glucocorticoid administration,
immunosuppression, or protein depletion; and 3) an increased number of bacteria within the intestine. Because many
factors that facilitate bacteria translocation occur simultaneously in surgical patients, these effects may be either additive or
cumulative. In addition, many patients in Surgical Intensive Care Units do not generally receive enteral feedings and
therefore current parenteral therapy results in gut atrophy which further promotes translocation.
Which of the following statement(s) is/are true concerning nutritional support of the injured patient?
a. The goal of nutritional support is maintenance of body cell mass and limitation of weight loss to less than 25% of
preinjury weight
b. Under-nutrition may compromise the patient’s available defense mechanisms
c. Nutritional support is an immediate priority for the trauma patient
d. Fifty percent of non-nitrogen caloric requirements should be provided in the form of fat
Answer: b

Metabolic response to injury results in increased energy expenditure. If energy intake is less than expenditure, oxidation of
body fat stores and erosion of lean body mass will occur with resultant loss of weight. When weight loss exceeds 10–15%
of body weight, the complications of malnutrition interact with disease processes, with increased morbidity and mortality
rates. The goal of nutritional support is maintenance of body cell mass and limitation of weight loss to less than 10%
preinjury. The major impact of nutritional support in the trauma patient is to aid host defense. Under-nutrition may
compromise the available host defense mechanism and may thus increase the likelihood of invasive sepsis, multiple organ
system failure, and death. Resuscitation, oxygenation and arrest of hemorrhage are immediate priorities for survival.
Nutritional support is an essential part of the metabolic care of the critically ill patient and should be instituted after
resuscitation before significant weight loss occurs. The nutritional requirements of a trauma patient can be determined by
determining basal metabolic rate with appropriate increases based on extent of injury and hospital activity. After initial
determination of nitrogen requirements, caloric requirements should be distributed at a ratio of 70% as glucose and 30% as
fat.

100 Which of the following statement(s) is/are true concerning body fuel reserves?
a. The largest fuel reserve in the body is skeletal muscle
b. Fat provides about 9 calories/gram
c. Free glucose and glycogen stores are a trivial fuel reserve
d. Body protein is a valuable storage form of energy
Answer: b, c

The body contains fuel reserves which it can mobilize and utilize during times of starvation or stress. By far the greatest
energy component is fat, which is calorically dense since it provides about 9 calories/gram. Body protein comprises the next
largest mass of utilizable energy, but amino acids yield only about 4 kcal/gram. Unlike fat reserves, body protein is not a
storage form of energy but rather serves as a structural functional component of the body; loss of body protein, if severe, is
associated with functional consequences. Glycogen stored in muscle and liver and free glucose have a trivial caloric value
of less than 1000 kcal for a 70 kg male.
101 Which of the following statement(s) is/are true concerning the indications and administration of nutritional support
to cancer patients?
a. Preoperative nutritional support should be provided to all patients with cancer
b. To be effective, preoperative nutrition must be given for at least two weeks preoperatively
c. Parenteral nutrition is the preferred route of feeding for all cancer patients
d. Standard total parenteral nutrition solutions maintain integrity of the small bowel
e. None of the above
Answer: e

The role of nutritional support in the cancer patient remains an important component of overall therapy. Preoperative
nutritional support should be given only to those patients who do not require an emergency operation and who have severe
weight loss (> 15% of pre-illness body weight) and a serum albumen < 2.9 mg%. Preoperative nutrition (enteral or
parenteral) should not be given for longer than 7 to 10 days. Enteral nutrition is always the preferred route of feeding cancer
patients if the GI tract is functional. There are several benefits of using the bowel lumen for nutrient delivery. The trophic
effects of enteral feeding on small bowel mucosa have been well described. The integrity of the mucosal lining is
maintained and it may provide an effective barrier to intraluminal enteric organisms which might otherwise translocate into
the systemic circulation. Atrophic changes may be seen in the intestinal epithelium after several days of bowel rest; this
atrophy is not reversed by currently available total parenteral nutrition solutions.

102 Which of the following hormones can be expected to be released as part of the stress response?
a. Antidiuretic hormone (ADH)
b. Aldosterone
c. Insulin
d. Epinephrine
nswer: a, b, d

Several important responses occur in response to stress. The body immediately attempts to compensate for a reduction in
circulating blood volume in order to maintain adequate organ perfusion. Afferent nerve signals are also initiated which
stimulate the release of both antidiuretic hormone (ADH) and aldosterone. The pain and fear associated with the stress
response lead to excessive production to catecholamines which also increase metabolic rate, stimulate lipolysis, hepatic
glycolysis, and gluconeogenesis. Glucagon, which has a potent glycogenolytic and gluconeogenic effect in the liver, is also
released. This hormone has the exact opposite effect of insulin, which promotes glucose storage and uptake by the cells.

103 Cytokines which play an important role in the metabolic response to injury include:
a. Tumor necrosis factor—a (TNF)
b. Interleukin-1 (IL-1)
c. Interleukin-6 (IL-6)
d. Interferon-g
Answer: a, b, c, d

TNF or cachetin is considered the primary mediator of the systemic effects of endotoxin, producing anorexia, fever,
tachypnea, and tachycardia at low doses and hypotension, organ failure, and death at higher doses. TNF is produced
primarily by macrophages, but lymphocytes, Kupffer cells, and a number of other cell types have been identified as sources
of TNF. IL-1, like TNF, has a variety of pro-inflammatory activities. IL-6 is now recognized as a primary mediator of
altered hepatic protein synthesis known as acute-phase protein synthetic response. Glucocorticoid hormones augment the
cytokine effects on acute phase protein synthesis. Interferons are a family of proteins which are readily identified for their
ability to inhibit viral replication in infected sells. IFN-g has the ability to upregulate the number of TNF receptors on
various cell types.

104 A 16-year-old boy suffers a mid-gut volvulus with massive loss of small intestine. Which of the following
statement(s) is/are true concerning his nutritional requirements and management?
a. If at least 18 inches of residual small intestine survives, the patient may tolerate some form of enteral nutrition
b. A nutritional regimen consisting of supplemental glutamine, growth hormone, and a modified high carbohydrate,
low fat diet may be beneficial in this patient
c. The regimen described above may decrease the cost of care
d. TPN needs will increase after discontinuation of growth hormone
Answer: a, b, c

Prior to the availability of TPN, most patients developing short bowel syndrome from either surgery or catastrophic event
died. In selected patients, however, with residual small intestine (at least 18 inches), post-resectional hyperplasia may
develop with time such that they can tolerate enteral feeds. Recent studies have demonstrated the requirement for TPN
could be decreased or even eliminated in patients with short-gut syndrome by providing a nutritional regimen consisting of
supplemental glutamine, growth hormone, and a modified high carbohydrate, low fat diet. There was a marked
improvement in absorption of nutrients with this combination of therapy and a decrease in stool output. In addition, TPN
requirements were reduced by 50% as were costs associated with the care of these individuals. Discontinuation of the
growth hormone did not increase TPN needs in patients once they had undergone successful gut rehabilitation.

105 A number of changes in trace mineral metabolism are noted during sepsis. Which of the following change(s) may
be observed in a septic or trauma patient?
a. Plasma iron levels are noted to decrease
b. Plasma copper levels are noted to decrease
c. Plasma serum zinc levels may decrease
d. Administration of iron is appropriate
Answer: a, c

Changes in the balance of magnesium, inorganic phosphate, zinc, and potassium generally follow alterations in nitrogen
balance. Although the iron-binding capacity of transferrin is usually unchanged in early infection, iron disappears from the
plasma, especially during severe pyogenic infection; similar alterations are observed in serum zinc levels. The
administration of iron to the infected host, especially early into the disease, is contraindicated, however, because increased
serum iron concentrations may impair resistance. Unlike iron and zinc, copper levels generally rise, and the increased
plasma concentrations can be ascribed almost entirely to the levels of the ceruloplasmin produced by the liver.

106 A 17-year-old patient involved in an automobile accident is paralyzed with multiple peripheral extremity injuries.
Nutritional support is instituted with a transnasal feeding catheter. Which of the following statement(s) is/are true
concerning the patient’s management?
a. Feeding into the stomach results in stimulation of the biliary/pancreatic axis which is probably trophic for small
bowel
b. Gastric secretions will dilute the feedings increasing the risk of diarrhea
c. The major risk in this patient is tracheobronchial aspiration
d. Placement of the feeding catheter through the pylorus into the first portion of the duodenum reduces the risk of
regurgitation and aspiration
Answer: a, c, d

The use of transnasal feeding catheters for intragastric feeding or for duodenal intubation are popular adjuncts for providing
nutritional support by the enteral route. The stomach is easily accessed by passage of a soft flexible feeding tube.
Intragastric feeding provides several advantages for the patient. The stomach has the capacity and reservoir for bolus
feedings. Feeding into the stomach results in stimulation of the biliary/pancreatic axis which is probably trophic for the
small bowel. Gastric secretions will have a dilutional effect on the osmolarity of the feedings, reducing the risk of diarrhea.
The major risk of intragastric feeding is the regurgitation of gastric contents resulting in aspiration into the tracheobronchial
tree. This risk is highest in patients who have an altered sensorium or who are paralyzed. The placement of the feeding tube
through the pylorus into the fourth portion of the duodenum reduces the risk of regurgitation and aspiration of feeding
formulas.

107 Although TPN has major beneficial effects to the patient and specific organ systems, TPN has a downside which is
related to intestinal disuse. Which of the following statement(s) is/are true concerning the effects of TPN on the GI tract?
a. Patients receiving TPN have an accentuated systemic response to endotoxin challenge compared to enterally fed
volunteers
b. TPN can result in disruption of intestinal microflora
c. In experimental models, bacterial translocation from the gut is increased
d. Effects of TPN on the gut may lead to multiple organ failure
nswer: a, b, c, d

A number of studies have examined the effects of TPN on intestinal function and immunity. Although most of these studies
have been done in animal models, TPN has consistently been shown to have some detrimental effects. In rats, TPN results in
significant disruption of the intestinal microflora and bacterial translocation of the gut to the mesenteric lymph nodes. In
addition, when stresses such as a burn injury, chemotherapy, or radiation are introduced into these models, animals on TPN
have a much higher mortality. The body of literature suggests that TPN under certain circumstances may predispose patients
to an increase in gut-derived infectious complications. In a study in human volunteers, individuals receiving TPN had an
accentuated systemic response to endotoxin challenge compared to enterally fed volunteers. This study is consistent with
impairment of gut barrier function during parenteral feedings which may promote the release of bacteria and/or cytokines
leading to pronounced systemic responses and possibly multiple organ failure.

108 Total body mass is composed of an aqueous component and a nonaqueous component. The nonaqueous component
is made up of which of the following?

a. Liver
b. Tendons
c. Skeletal muscle
d. Extracellular fluid
e. Adipose tissue
Answer: b, e

The nonaqueous portion of total body mass is made up of bones, tendons, and mineral mass as well as adipose tissue. The
aqueous component contains the body cell mass which is made up of skeletal muscle, intraabdominal and intrathoracic
organs, skin, and circulating blood cells. Also contributing to the aqueous portion is the interstitial fluid and intravascular
volume.

109 Fatty acids are a major energy source for the body. Which of the following statement(s) is/are true concerning the
use of fatty acids as an energy source?
a. Fatty acids are stored in adipocytes as triglycerides
b. Hormone-sensitive lipase is present only in adipose tissue
c. Fatty acids are released into the circulation traveling freely in plasma
d. Approximately 25% of total nonprotein caloric needs supplied via total parenteral nutrition should be in the form
of fat
Answer: a, b, d
In most tissues, fatty acids are readily oxidized for energy. They are especially important energy sources for the heart, liver
and skeletal muscle. In adipose tissue, fatty acids may be re-esterified with glycerol and stored as triglycerides in
adipocytes. Stored fat is mobilized during starvation and stress. Hormone-sensitive lipase, present only in adipose tissue,
catalyzes the breakdown of stored triglycerides into glycerol and fatty acids. The fatty acids that are produced are released
in the circulation. The major lipids in plasma do not circulate in a free form, thus free fatty acids must be bound to albumin.
During stress, the activity of hormone-sensitive lipase is increased which leads to mobilization of fat stores. However, fat
remains an important fuel source for critically ill patients and as a rule the amount of fat administered to patients receiving
total parenteral nutrition should comprise about 5–30% of total nonprotein caloric needs.

110 Which of the following metabolic effects may be observed in patients with sepsis?
a. Increased gluconeogenesis
b. Accelerated proteolysis
c. Increased lipolysis
d. Impaired gut metabolism of glutamine
Answer: a, b, c, d

A number of metabolic responses to sepsis have been defined. Glucose production is increased in infected patients which
appears to be additive to the augmented gluconeogenesis that occurs following injury. Accelerated proteolysis, increased
nitrogen excretion and prolonged negative nitrogen balance also occur following infection with a response pattern similar to
that described with injury. Severe infection is often associated with a hypercatabolic state that initiates marked changes in
interorgan glutamine metabolism. This process results in accelerated muscle proteolysis and net skeletal muscle glutamine
release. The bulk of glutamine is taken up by the liver at the expense of the gut. It appears that sepsis can impair gut
metabolism of glutamine. Fat is a major fuel oxidized in infected patients, and increased metabolism of lipids from
peripheral fat stores is especially prominent during a period of inadequate nutritional support.

111 Which of the following statement(s) is/are true concerning protein/amino acid metabolism in man?
a. The major source of amino acids is breakdown of circulating proteins
b. The recommended daily allowance for protein may triple in critically ill patients
c. Urinary nitrogen losses will approach 0 in the face of protein starvation
d. Negative nitrogen balance refers to a decrease in nitrogen taken into the body versus the amount of nitrogen lost
Answer: b, d

About 15% of the total body weight is made up of proteins, about half of which are intracellular and half extracellular. In
man and other animals, dietary protein is the source of most amino acids. Intestinal absorption is the only physiological
pathway by which the body obtains exogenous amino acids. Digestion of ingested protein provides free amino acids that are
absorbed by the small intestine and transported to the liver where they can be incorporated into new proteins or other
biosynthetic products. Excess amino acids are degraded and their carbon skeleton is oxidized to produce energy or it is
incorporated into glycogen or into free fatty acids. In addition to the metabolism of dietary amino acids, the existing
proteins in the cell are continuously recycled, such that total protein turnover in the body is about 300 g/day. Vertebrates
cannot reutilize nitrogen with 100% efficiency; therefore, obligatory nitrogen losses occur, mainly in the urine. Urinary
nitrogen losses will diminish when individuals are fed a protein-free diet, but will never become 0 because of the body’s
inability to completely reutilize nitrogen. In stressed patients, this ability to adapt to starvation is compromised such that
proteolysis of body proteins continues at a substantial rate. This increases the amount of obligatory nitrogen losses which
are accentuated by the catabolic disease states. This results in a negative nitrogen balance in which the amount of nitrogen
taken in by the patient is exceeded by the amount of nitrogen lost in the urine, stool, skin, wounds, and fistula drainage.

112 Which of the following statement(s) concerning intravenous nutritional support is/are true?
a. Concentrations of glucose no higher than 5% should be used to avoid peripheral vein sclerosis
b. A major disadvantage of the peripheral technique is limited caloric delivery
c. If total parenteral nutrition is required, access to the superior vena cava via the external jugular vein is the most
suitable site
d. Venous thrombosis is an uncommon complication for long-term central vein catheterization
Answer: b

Although peripheral access can be used for intravenous nutrition, the major disadvantage of this technique is limited caloric
delivery to meet catabolic demands within tolerated fluid limits. Infusion of glucose (up to 10%), amino acid solutions, and
fat emulsions can be administered peripherally but these solutions must be nearly isotonic to avoid peripheral vein sclerosis.
The preferred method of access for total parenteral nutrition is into the superior vena cava by cutaneous cannulation of the
subclavian vein. Alternative sites include the internal and external jugular vein but the catheter exiting from the neck region
makes it more difficult to secure and maintain a sterile dressing. Complications from long-term central venous
catheterization include venous thrombosis and venous catheter-related infection. Thrombosis of central vessels is a
complication which is often overlooked. The clinical suspicion of subclavian vein thrombosis is only about 3%, whereas
studies that use phlebography or radionucleotide venography indicate the incidence is as high as 35%.

113 Sepsis causes a marked metabolic response. Which of the following statement(s) is/are true concerning the
metabolic response to sepsis?
a. Oxygen consumption is increased in the face of infection
b. In a patient with a maximal metabolic rate secondary to trauma, the presence of infection will increase the rate
further
c. Metabolic rate increases at a rate of approximately 10% for each increase of 1°C in central temperature
d. The extent of increase in oxygen consumption relates to the severity of the infection
Answer: a, c, d

Oxygen consumption is usually elevated in the infected patient. The extent of this increase is related to the severity of the
infection, with peak elevations reaching 50% to 60% above normal. If the patient’s metabolic rate is already elevated to a
maximal extent because of severe injury, no further increase will be observed. In patients with only a slightly accelerated
rate of oxygen consumption, the presence of infection will cause a rise in metabolic rate added to the preexisting state. A
portion of the increase in metabolism may be ascribed to increase in reaction rate associated with fever. Calculations suggest
that the metabolic rate increases 10% to 13% for each elevation of 1°C in central temperature.

114 Interleukin-6 is recognized as the cytokine primarily responsible for the alteration in hepatic protein synthesis
recognized as the acute phase response. Which of the following statement(s) is/are true concerning acute phase protein
response to surgical stress?
a. Glucocorticoid hormones inhibit this response
b. Proteins such as albumin and transferrin which serve in serum transport are generally increased in this response
c. Examples of acute phase proteins include fibrinogen and C-reactive protein
d. In general, the physiologic role of acute phase proteins are to reduce the systemic effects of tissue damage
Answer: c, d

IL-6 is now recognized at the cytokine primarily responsible for the alteration in hepatic synthesis recognized as the acute
phase response. Glucocorticoid hormones augment this response. The primary metabolic component of the acute phase
response is a qualitative alteration in hepatic protein synthesis with resulting alteration in plasma protein composition.
Characteristically, proteins which act as serum transport in binding molecules, (albumin, transferrin) are reduced in quantity
and acute phase proteins (fibrinogen, C-reactive proteins) are increased. Acute phase proteins are elaborated for the purpose
of reducing the systemic effects of tissue damage. Many act as anti-proteases, opsonins, or coagulation and wound healing
factors that generally inhibit the tissue destruction that is associated with the local initiation of inflammation.

115 A 59-year-old trauma patient has suffered multiple septic complications including severe pneumonia,
intraabdominal abscess, and major wound infection. He has now developed signs of multisystem organ failure. Which of
the following statement(s) is/are true concerning necessary changes to be made in his nutritional management?
a. Carbohydrate load should be reduced in the face of respiratory failure
b. In patients with renal failure, protein intake should be diminished
c. During hemodialysis protein intake should be limited to the same extent
d. In patients with hepatic failure, carbohydrate load should be increased
Answer: a, b

The most severe complication of sepsis is multiple system organ dysfunction syndrome, which may result in death. The
development of organ failure requires changes in the nutritional requirements and creates special feeding problems. A
problem associated with systemic infection is oxygenation and elimination of carbon dioxide. Most of the enteral and
parenteral formulas used to provide nutritional support for critically ill patients contain large amounts of carbohydrate,
which generate large amounts of carbon dioxide following oxygenation. Such a large CO2 load may worsen pulmonary
function or may delay weaning from the respirator. If this factor becomes a problem, the carbohydrate load should be
reduced to 50% of metabolic requirements and fat emulsion administered to provide additional calories. When renal failure
becomes progressive, the use of hemodialysis minimizes the effect of uremia superimposed on the metabolism of sepsis.
Metabolic studies in patients with acute and chronic renal failure have limited the intake of nonessential amino acids, in an
attempt to lower urea production. Proteins of high biologic value, but in much smaller quantities than usually given, are
administered along with adequate calories, usually in the form of glucose. When enteral feedings are not feasible, a central
venous infusion of an essential amino acid solution and hypertonic dextrose provides calories and a small quantity of
nitrogen to reduce protein catabolism while simultaneously controlling the rise in BUN. During dialysis, protein intake is
liberalized, but the BUN should still be maintained below 100 mg/dl. Hepatic dysfunction is a common manifestation of
septicemia. The carbohydrate load is usually reduced to consist of no more than 5% of metabolic requirements, and the
additional calories should be provided as fat emulsion. If encephalopathy develops, protein load should also be reduced0.

116 Which of the following statement(s) is/are true concerning the role of glutamine in total parenteral nutrition?
a. Glutamine is an essential amino acid
b. Glutamine appears to be of primary benefit in critical illness
c. Glutamine is included in most standard TPN solutions
d. Glutamine is the primary energy source for intestinal mucosal cells of the small bowel and colon
Answer: b

Glutamine is the most studied gut-specific nutrient. Glutamine has been classified as a nonessential or nutritionally
dispensable amino acid since glutamine can be synthesized in adequate quantities from other amino acids and precursors.
Glutamine is not included in most nutritional formulas and has been eliminated from TPN solutions because of its relative
instability and short half life compared to other amino acids. With few exceptions, glutamine is present in oral enteral diets
but only at relatively low levels characteristic of the concentration in most animal and plant stores (about 7% of total amino
acids). Several recent studies, however, have demonstrated that glutamine may be an essential amino acid during critical
illness, particularly as it relates to supporting the metabolic requirements of the intestinal mucosa. These studies
demonstrate that dietary glutamine is not required during states of health but appears to be beneficial when glutamine
depletion is severe and/or when intestinal mucosa is damaged by insults such as chemotherapy or radiation therapy. The
addition of glutamine to enteral diet reduces the incidence of gut translocation but these improvements are dependent upon
the amount of supplemental glutamine and the type of insult studied. Glutamine-enriched TPN partially attenuates villous
atrophy that develops during parenteral nutrition. The use of intravenous glutamine in patients appears to be safe and
effective in its ability to maintain muscle glutamine stores and improve nitrogen balance. In contrast to glutamine, short
chain fatty acids are primary energy source for colonocytes.

117 Which of the following are determinants of the host response to surgical stress?
a. Gender
b. Age
c. Nutritional status
d. Body composition
Answer: a, b, c, d

The pattern of physiologic changes elicited in response to surgical stress results from the specific interaction of an
individual patient with a stressful stimulus. Several factors specific to the patient may determine the nature of the host
response to stress. Body composition is a major determinant of the metabolic responses observed during surgical illness.
Post-traumatic nitrogen excretion is directly related to the size of the body protein mass. A strong relationship between
protein depletion and postoperative complications has been demonstrated in nonseptic, nonimmunocompromised patients
undergoing elective major gastrointestinal surgery. Protein-depleted patients have significantly lower preoperative
respiratory muscle strength and vital capacity, increased incidence of postoperative pneumonia, and longer postoperative
hospital stay. Impaired wound healing and respiratory, hepatic, and muscle function in protein-depleted patients awaiting
surgery has also been reported. Many of the changes in the metabolic responses to surgical illnesses that occur with aging
can be attributed to alterations in body composition and to long-standing patterns of physical activity. Fat mass tends to
increase with age and muscle mass tends to decrease. Loss of strength that accompanies immobility, starvation and acute
surgical illness may have marked functional consequences. Furthermore, the prevalence of cardiovascular and pulmonary
diseases increase with age. Thus, the delivery of oxygen to tissues may be impaired in the elderly. Finally, observed
differences in metabolic responses of men and women generally reflect differences in body composition. Lean body mass is
lower in women than in men; and this difference is thought to account for the net loss of nitrogen after major elective
abdominal surgery generally being lower in women than in men.

118 In contrast to a patient undergoing an elective operation, which of the following statement(s) is/are true concerning
a patient who has suffered a multiple trauma?
a. Basal metabolic rates are similar
b. The patient is highly sensitive to insulin
c. Utilization of the amino acids, glutamine and alanine, is similar to their composition in skeletal muscle
d. Fat and protein stores are rapidly depleted
Answer: b, d
The degree of hypermetabolism is generally related to the severity of injury. Patients with long-bone fractures have a 15–
25% increase in metabolic rate, whereas metabolic rates in patients with multiple injuries increases by 50%. These
metabolic rates in trauma patients are contrasted with those in postoperative patients, who rarely increase their BMR by
more than 10–15% following operation. Studies have shown that uninjured volunteers are able to dispose of exogenous
glucose load much more readily than injured patients. Other studies have demonstrated a failure to suppress hepatic glucose
production in trauma patients during glucose loading or insulin infusion. Thus, profound insulin resistance occurs in injured
patients. Skeletal muscle is the major source of nitrogen that is lost in the urine following extensive injury. Although it is
recognized that amino acids are released by muscle in increased quantities following injuries, it has only been recently
appreciated that the composition of amino acid reflux does not reflect the composition of muscle protein. The release is
skewed towards glutamine and alanine, each of which comprise about one-third of the total amino acids released by skeletal
muscle. To support hypermetabolism, stored triglyceride is mobilized at an accelerated rate. Although mobilization and use
of free fatty acids are accelerated in injured subjects, if unfed, severely injured patients rapidly deplete their fat and protein
stores.

119 A 47-year-old patient undergoing a complicated laparotomy for bowel obstruction develops a postoperative
enterocutaneous fistula. Which of the following statement(s) is/are true concerning parenteral nutritional support in the
postoperative period?
a. Oral intake can result in severe dehydration, electrolyte abnormalities, and perifistula skin injury
b. Total parenteral nutrition increases the spontaneous closure rate of intestinal fistula
c. Total parenteral nutrition decreases mortality rate in patients with intestinal fistulas
d. The use of TPN better prepares the patient for surgery if surgical intervention proves necessary
Answer: a, b, d

Patients with gastrointestinal-cutaneous fistulas represent the classical indication for TPN. In such patients, oral intake of
food almost invariably results in increased fistula output with associated metabolic disturbances, dehydration, skin
breakdown, and death. Several comprehensive reviews have concluded that TPN clearly impacts on the treatment course of
the disease in patients with GI fistulas. The following conclusions can be drawn from studies evaluating the use of TPN in
patients with enterocutaneous fistula. First, TPN increases spontaneous closure rate of enterocutaneous fistulas but does not
markedly decrease the mortality rate in patients with fistulas. Second, if spontaneous closure of the fistula does not occur,
patients are better prepared for operative intervention because of the nutritional support they have received. Finally, certain
fistulas are associated with a lower rate of spontaneous closure than others and should be treated more aggressively
surgically after a defined period of nutritional support (unless closure occurs).

120 Appropriate guidelines for the use of TPN in cancer patients include:
a. Long-term TPN in patients with rapid progressive tumor growth unresponsive to other therapy
b. Mildly malnourished patients undergoing surgery for a curable cancer
c. Preoperatively administered TPN prior to surgery or other therapy in patients with severe malnutrition
d. Patients in whom treatment toxicity precludes the use of enteral nutrition
Answer: c, d

As a general rule, the most important factor to consider when making decisions about the use of TPN in patients with cancer
is the response of the tumor to antineoplastic therapy. Appropriate guidelines would include the following: Short-term TPN
is indicated in severely malnourished patients or in those in whom gastrointestinal or other toxicities preclude adequate
enteral intake for seven days or a longer period. TPN is not indicated in well nourished or mildly malnourished patients
undergoing therapy or surgery who would be expected to be able to resume adequate nutrition in approximately seven days.
Long-term TPN is indicated in patients in whom treatment associated toxicities preclude the use of enteral nutrition and
represent the primary impediment to the restoration of performance status. These patients should be expected to be
responding to anti-tumor therapy. Long-term TPN is not indicated with rapidly progressive tumor growth which is
unresponsive to such therapy.

121 Which of the following statements(s) is/are true concerning human energy requirement?
a. In normal subjects, less than 5% of basal energy requirement is spent on cardiac output and the work of breathing
b. Mechanical ventilation can decrease the energy expenditure for normal respiration
c. For a 70 kg male, average resting energy consumption is almost 1500 kcal/day
d. Similar increases in energy expenditures are associated with elective surgery and trauma or thermal injury
Answer: a, c

Basal energy requirements are measured with the subject at rest when no external work is being done; the energy is used
mainly for transport and synthetic work within cells. A surprisingly small percentage (< 5%) of this energy is spent on
cardiac output and the work of breathing in normal subjects. In contrast, the work of breathing in individuals with chronic
obstructive lung disease or in patients on a ventilator may account for 15–20% of caloric expenditure. The average resting
post-absorptive 70 kg male consumes about 1500 kcal/day. Energy needs increase as severity of illness increases. The
expenditure of kcal is only minimally increased after elective surgery. The largest increase in energy expenditure occurs in
patients with severe multiple trauma or major thermal injury. The average-sized adult who sustains a major burn rarely may
require more than 3500 kcal/day for maintenance.

122 Which of the following complications of TPN are appropriately managed with the listed treatment?
a. Air embolism—place patient in reverse Trendelenburg and the left lateral decubitus position and aspirate venous
air
b. Hyperchloremic metabolic acidosis—give sodium and potassium as acetate salts
c. Carbon dioxide retention—decrease glucose calories and replace with fat
d. Line sepsis—intravenous antibiotics
Answer: b, c

A number of complications of TPN can occur which can be divided into three types: mechanical, metabolic, and infectious.

123 A 55-year-old male undergoes a total abdominal colectomy. Which of the following statement(s) is/are true
concerning the hormonal response to the surgical procedure?
a. Adrenocorticotropic hormone (ACTH) is secreted from the anterior pituitary gland
b. ACTH stimulation results in elevation of serum cortisol levels for up to a week after the operation
c. An increased secretion of aldosterone and ADH may contribute to postoperative fluid retention
d. An increase in serum insulin and a fall in glucagon accelerate hepatic glucose production and maintain
gluconeogenesis
Answer: a, c

One of the earliest consequence of a surgical procedure is the rise in levels of circulating cortisol that occur in response to a
sudden outpouring of ACTH from the anterior pituitary. The rise in ACTH stimulates the adrenal cortex to elaborate cortisol
which remains elevated for 24–48 hours after operation. The neuroendocrine responses to operation also modify the various
mechanisms that regulate salt and water excretion. Alterations in serum osmolarity and tonicity of body fluids secondary to
anesthesia and operative stress, stimulate the secretion of aldosterone and ADH. Thus, the ability to excrete a water load
after elective surgical procedures is restricted, and weight gain secondary to salt and water retention is usual following an
operation. Alterations occur in response to the endocrine pancreas following elective operation. Insulin elaboration is
diminished and glucagon concentrations rise. The rise in glucagon and the corresponding fall in insulin are important signals
to accelerate hepatic glucose production, and, with other hormones (epinephrine and glucocorticoids), gluconeogenesis is
maintained.

124 A number of prospective clinical trials have addressed the role of total parenteral nutrition in the cancer patient.
The results have been somewhat conflicting. Which of the following statement(s) have been proven correct by prospective
trials?
a. Preoperative TPN is beneficial in surgical patients with severe preoperative nutrition
b. Postoperative TPN is of value following pancreatic resection
c. Routine use of perioperative (including prior to the procedure) TPN is of benefit in patients undergoing
hepatectomy for hepatoma
d. TPN is of no benefit in patients undergoing bone marrow transplant
Answer: a, c

Numerous clinical trials have failed to yield a consensus with regard to the efficacy of TPN in cancer patients. In 1991, a
multicenter VA cooperative trial demonstrated that preoperative TPN is of benefit in surgical patients (many of whom had
cancer) with severe preoperative malnutrition. Another study examined the use of routine postoperative TPN following
major pancreatic resection. Patients randomized to receive TPN starting on postoperative day 1 were noted to have an
increased incidence of intra-abdominal abscesses as well as a tendency towards increased incidence in peritonitis and bowel
obstruction. These investigators concluded that routine use of postoperative TPN was not indicated and may, in fact, be
harmful following pancreatic resection. In another study, however, perioperative (starting 7 days prior to the planned
procedure) TPN for patients undergoing hepatectomy for hepatocellular carcinoma demonstrated that this regimen
statistically reduced infectious complications compared to patients who did not receive TPN. This was one of the few
studies that demonstrated that routine TPN (without the requirement of severe preoperative malnutrition) was of benefit.
The use of TPN in patients receiving bone marrow transplantation has also been shown to be a valuable component of
overall care.
125 Which of the following statements concerning perioperative nutrition is true concerning the above-described
patient?
a. Since the patient’s weight had been stable with no preoperative nutritional deficit, 5% dextrose intravenous
solutions are adequate for the initial postoperative source of nutrition
b. Preoperative immunologic status should be determined including total peripheral lymphocyte count and delayed
hypersensitivity reaction to determine skin-test response to common antigens
c. Routine postoperative fluid administration with intravenous 5% glucose solutions can provide the calories to meet
basal energy requirements
d. A jejunal feeding catheter should be placed at the time of surgery for postoperative enteral feeding
Answer: a

Most patients undergoing elective operations are adequately nourished. Unless the patient has suffered significant
preoperative malnutrition, characterized by weight loss greater than 10–15%, or has major intraoperative or postoperative
complications, solutions containing 5% dextrose may be administered for five to seven days before initiation of enteral
nutrition, with no detrimental effect on outcome. The usual postoperative surgical patient is given intravenous glucose at
125 cc/hour receives about 500 kcal/day, far less than the actual number of kcal needed to meet energy requirements. The
increased cost of feedings and potential complications associated with intravenous nutrition cannot be justified. Although
the use of jejunal feedings in the postoperative period may be useful in some patients, especially those undergoing extensive
gastrointestinal surgery, this technique would not appear indicated in the patient described above.

126 The neurohormonal arm of the stress response is well defined. Less is known about the inflammatory arm mediated
primarily by cytokines. Which of the following statement(s) is/are true concerning this arm of the surgical stress response?
a. Cytokines primarily work locally via direct cell-to-cell communication
b. Cytokines are never detectable in the systemic bloodstream
c. Cytokines are produced only by immune cells attracted to the site of injury
d. Cytokine release may stimulate the release of other cytokines leading to an important cascade of events
Answer: a, d

Cytokines, which are produced at the site of injury by endothelial cells and by diverse immune cells throughout the body,
also occupy a pivotal position in the stress response. Cytokines differ from classic endocrine hormones in that they are
produced by a variety of cell types and in that they have the capacity to exert their tissue effects locally via direct cell-to-cell
communications in a paracrine and/or autocrine fashion. Cytokines can stimulate the production of other cytokines, leading
to important cascades which both amplify and diversify the effects of the proximal cytokine. Occasionally, when in excess,
cytokines act as hormones and “spill over” into the systemic circulation and become detectable in the bloodstream.

127 Which of the following tissues contain significant collagen useful for placing sutures to allow the prolonged
tension necessary to maintain tissue approximation?
a. Dermis
b. Intestinal submucosa
c. Muscular fascia
d. Blood vessel wall
Answer: a, b, c, d

It takes at least three weeks for collagen to undergo sufficient remodeling and cross linking to attain moderate strength.
Since most skin sutures are removed at one to two weeks, the wound has only a small fraction of its eventual strength and
may therefore disrupt with even modest stress. Therefore, deep sutures are placed in collagen containing structures to
maintain the prolonged tension necessary. Dermis, intestinal submucosa, muscular fascia, tendon, ligament, Scarpa’s fascia,
and blood vessel wall represent a partial list of tissues with high collagen content.

128 Products of platelet degranulation include:


a. Tumor necrosis factor
b. Interleukin-1
c. Transforming growth factor b
d. Platelet-derived growth factor
Answer: c, d

The initial response to injury and disruption of a blood vessel is bleeding. The hemostatic response to this is clot formation
to stop hemorrhage. Platelet plug formation initiates the hemostatic process along with clotting factors activated by collagen
and the basement membrane proteins exposed by the injury. Platelets then degranulate, releasing the contents of their alpha
granules and dense granules, most notably platelet derived growth factor and transforming growth factor b. These
substances initiate chemotaxis and proliferation of inflammatory cells, beginning the inflammatory response that will
ultimately heal the wound. Tumor necrosis factor and interleukin-1 also stimulate fibroblast proliferation, however are
produced by macrophages.

129 A patient with gross fecal contamination and peritonitis from a ruptured sigmoid diverticulum has his midline
wound left open to heal by secondary intention. Which of the following statement(s) describes this healing process?
a. Wounds healing in this fashion have an altered sequence of healing compared to a primarily closed wound
b. A bed of granulation tissue forms over exposed subcutaneous tissue
c. Epithelialization is enhanced in the face of bacterial colonization
d. The ability of a wound to form granulation tissue is dependent on the blood supply of the tissue
Answer: b, d

Open wounds, whether they be ulcers or open surgical incisions closing by secondary intention, heal with the same
sequence of inflammation, matrix deposition, epithelialization, and scar maturation as in all wounds. The major difference is
in the healing incisional wound, the healing process progresses in an orderly temporal sequence. In an open wound, the
healing events are spatially separated. In the healing wound, a bed of granulation tissue forms over the exposed
subcutaneous tissue. Granulation tissue is composed of new capillaries, proliferating fibroblasts, an immature matrix of
collagen, proteoglycans, substrate adhesion molecules, and acute and chronic inflammatory cells. Granulation tissue is the
cobblestone pink surface of the healthy new tissue in an open wound. The ability of an open wound to form granulation
tissue is governed by the blood supply to the tissue and the relative absence of devitalized tissue and bacteria.
Epithelialization is therefore enhanced by limiting bacterial growth which presumably interferes via bacterial and
phagocytic cell products such as proteases, collagenases, elastases, and other enzymes.

130 Which of the following factors can be associated with impaired wound healing?
a. Chemotherapy
b. Chronic steroid use
c. Peripheral vascular disease
d. Radiation therapy
e. Diabetes mellitus
Answer: a, b, c, d, e

Bone marrow suppression, a common consequence of chemotherapy, is detrimental to wound healing. Quantitative and
qualitative lymphocyte and monocyte deficiency impairs cellular proliferation in the inflammatory phase of wound healing.
Any chemotherapeutic agent that suppresses the bone marrow will impair healing. Glucocorticoids inhibit wound healing
based on their anti-inflammatory and immunosuppressive effects. The anti-inflammatory effect of steroids is, in part, the
result of inhibiting arachidonic acid metabolism by impairing macrophage migration, and by altering neutrophil function.
Glucocorticoids also inhibit the synthesis of procollagen by fibroblasts, thus delaying wound contraction. Radiation injury
leads to arteriolar fibrosis and impaired oxygen delivery. In addition, there is progressive obliteration of blood vessels in the
radiated area over time. Radiation also causes intranuclear and cytoplasmic damage to fibroblasts, and this appears to limit
their proliferative potential. Diabetes mellitus is often associated with decreased healing of open wounds and increased
susceptibility of infection. Many factors contribute to poor healing in diabetic patients and most of them reflect local wound
ischemia. However, healing is not impaired in a normally perfused area in a well-controlled diabetic. Peripheral arterial
occlusive disease secondary to atherosclerosis can be a primary cause of impaired healing, and may be also a cofactor with
other conditions.

131 Which of the following cells or blood elements play a role in the initial phases of wound healing?
a. Polymorphonuclear leukocytes (PMNs)
b. Platelets
c. Monocytes
d. Lymphocytes
Answer: a, b, c, d

Shortly after the initial injury, the wound is full of debris which is cleared over the next several days by recruited and
activated phagocytic cells. PMNs begin to arrive immediately, reaching large numbers within 24 hours. The PMNs are
followed by macrophages which appear in wounds in significant numbers within two to three days. Macrophages are
mononuclear phagocytic cells derived from circulating monocytes or resident tissue macrophages. They complete the
process of removing all material not necessary for the ensuing steps of wound healing. Lymphocytes also appear in wounds
in small numbers during the inflammatory response. The role of lymphocytes in the wound healing process remains to be
clarified, but they are thought to be more related to the chronic inflammatory processes than the initial response to
wounding. Platelets are anuclear discoid blood elements derived from bone marrow megakarocytes which play a role in the
initial hemostatic process as well as releasing chemotactic factors and factors leading to fibroblast proliferation.

132 Which of the following surgical techniques lead to improved wound healing?
a. Atraumatic handling of tissue
b. Approximation of underlying fatty tissue to obliterate dead space
c. Protecting the wound from water for at least one week
d. Meticulous hemostasis
Answer: a, d

There are numerous practical implications for the care of wounds and surgical incisions. Meticulous hemostasis reduces the
inflammation of phagocytosis necessary to clear the wound of blood. Atraumatic handling of tissue decreases the load of
necrotic or nonviable cells at the wound margin. Deep sutures are best placed only into collagen laden structures that will
hold tension, i.e., fascia and dermis. These tissues have a tensile strength to hold sutures under tension. Fat does not contain
collagen and will not hold tension. Therefore, fatty tissue should not be sutured as a separate layer. Given that
epithelialization of an incision is normally complete within 24–48 hours, there is no reason to protect the incision from
water beyond this time period. Allowing the patient to wash or shower one or two days after surgery actually serves useful
purpose in debriding the wound.

133 Which of the following statement(s) is/are true concerning the clinical management of an open wound?
a. A wet-to-dry dressing is the most optimal form of wound management
b. A moist occlusive dressing promotes epithelialization and reduces pain
c. The protein rich plasma exudate covering the open wound facilitates healing
d. Irrigation of the wound disrupts epithelialization therefore inhibiting the healing process
Answer: b

Epithelialization is more rapid under moist conditions than dry conditions. Without dressings, a superficial wound, or one
with minimal devitalized tissue forms a scab or crust, meaning that the blood and serum will coagulate, dry, and form a
protective moisture barrier over the open wound. If a wound is kept moist with an occlusive dressing, then epithelial
migration is optimized. In addition, the pain of an open wound is dramatically reduced under an occlusive dressing. The
traditional wet-to-dry dressing if truly left to dry, simply produces desiccation and necrosis of the surface layer of the wound
which delays epithelialization. Although wet-to-dry dressings can be effective for debridement of wound exudate, they are
generally less desirable than a moist healing environment combined with effective cleaning of the wound (i.e. water
irrigation). Any open wound will leak plasma. With more inflammation, the plasma capillary permeability is further
increased. This exudate of serum proteins and inflammatory cells serves as a rich culture medium. This, in turn, will
continue to cycle bacterial proliferation and lead to further exudate formation. The net result of this cycle is delayed or
absent wound healing. In addition, the edema that results from capillary dysfunction, increases the distance for diffusion
from oxygen and nutrient sources to their metabolic targets.

134 Which of the following statement(s) is/are correct concerning the management of an open wound?
a. Frequent surgical debridement is usually necessary
b. Water irrigation can effectively debride most wounds
c. Hydrogen peroxide is particularly useful in the management of open wounds
d. A number of the newer dressing products have clearly been shown to promote wound healing compared to simple
moist occlusive dressing
Answer: b

Although there are numerous dressing products commercially available at present, no treatment has been demonstrated to
improve healing beyond that of standard treatment which adheres to basic principles. In the absence of large amounts of
necrotic tissue, wound debridement does not need to be accomplished surgically. Simple water irrigation either with
whirlpool or by water from a hand held shower spray can generate enough power to effectively debride most wounds.
Frequent moist dressing changes can accomplish this as well, and in some cases, occlusive absorptive dressings can
generate enough tissue proteases to effectively degrade proteins which the absorptive dressings remove. Deeper portions of
a wound may accumulate exudate and bacteria. In such cases, water irrigation may be particularly useful. Commonly used
agents such as hydrogen peroxide actually may be harmful to normal tissue and are weak oxidants and do a poor job of
debriding. Enzymatic debriding agents can be effective when used properly. Most of the newer dressing products have been
designed to be more absorptive and achieve moist healing without infection from excess exudate. However, it must be
emphasized that as long as moist healing is achieved, there has been no evidence that one product is better than another.

135 Which of the following statement(s) is/are true concerning the proliferative phase of wound healing?
a. The macrophage is the predominant cell type
b. The pink or purple-red appearance of a wound is due to ingrowth and proliferation of endothelial cells
c. Collagen, the dominant structural molecule of the wound matrix, contains two unique amino acids, hydroxyproline
and hydroxylysine
d. The predominant collagen type in a scar is type 3
Answer: b, c

The proliferative phase of wound healing begins with the formation of a provisional matrix of fibrin and fibronectin as part
of the initial clot formation. Initially, the provisional matrix is populated by macrophages; however, by day three fibroblasts
appear in the fibronectin-fibrin framework and initiate collagen synthesis. Fibroblasts proliferate in response to growth
factors become the dominant cell type during this phase. Growth factors produced by macrophages simultaneously induce
angiogenesis which results in the ingrowth and proliferation of endothelial cells, forming new capillaries. This
neovascularity is visible through the epithelium and gives the wound a pink or purple-red appearance.
Collagen is the dominant structural molecule in the wound matrix and in the final scar. Collagen is synthesized into an
organized cable-like network in a multi-step process with both intra- and intercellular components. The collagen molecule
has quantities of two unique amino acids, hydroxyproline and hydroxylysine. The hydroxylization processes which form
these amino acids require ascorbic acid (vitamin C) and is necessary for the subsequent stabilization and cross linkage of
collagen. The principal collagen type scar is type 1, with lesser amounts of type 3 collagen also present.

136 Which of the following statement(s) is/are true about the role of macrophages in the wound healing process?
a. Macrophages are the dominant cell type during the inflammatory phase of wound healing
b. Macrophages are not essential for wound healing
c. The macrophage role in wound healing is limited to phagocytosis
d. Macrophages are a source of a number of humoral factors essential for wound healing
Answer: a, d

Within three or four days after injury, macrophages become the dominant cell type in the inflammatory phase of wound
healing. The role of macrophages is not limited only to phagocytosis. In addition, macrophages are the source of more than
30 different growth factors and cytokines. These growth factors induce fibroblast proliferation, endothelial cell proliferation
(angiogenesis), extracellular matrix production, and recruit and activate additional macrophages. The result is the induction
of a wound healing amplification cycle as growth factors recruit macrophages and elicit additional growth factor release.
Experimental studies in which antibodies, which either destroy PMNs or block certain aspects of their function, have shown
that wounds heal normally, but that healing is significantly impaired without functional macrophages. These studies confirm
the dominant role of the macrophage and the inflammatory phase of wound healing.

137 Which of the following statement(s) is/are true concerning the role of antibiotics in wound care?
a. Systemic antibiotics are indicated for all open wounds
b. Bacterial resistance can occur with systemic but not topical antibiotics
c. An indication for systemic antibiotic administration is a granulation tissue bacterial count in excess of greater than
105 organisms/gram of tissue on quantitative analysis
d. Silver sulfadiazine is useful only for the management of burns
Answer: c

The role of antibiotics in wound care is controversial. All open wounds are colonized with bacteria. Only when surrounding
tissue is invaded (cellulitis) are systemic antibiotics clearly indicated. Antibiotics may also be useful in other situations such
as when granulation tissue has a high bacterial count (> 105 organisms/gram tissue), or in the case of reduced resistance to
bacteria such as in a diabetic foot ulcer. The routine use of systemic antibiotics for chronic wounds should be avoided to
reduce the development of resistant bacterial strains within the wound. Topical ointments are frequently used and can be
useful. The topical vehicle may help keep the wound moist and the bacterial count in the wound may be lowered as the
result. However, as with most antibiotics, resistant organisms quickly emerge. Silver sulfadiazine, frequently used for burn
care, is also useful for chronic wounds. Its broad spectrum of activity, lack of relevant drug-resistant plasmids in bacteria,
and its low cost make it a good choice.

138 Which of the following statement(s) is/are true concerning wound contraction?
a. Wound contraction accounts for similar rates of reduction of wound size regardless of their location
b. The fibroblast, at the cellular level, is the primary force driving wound contraction
c. Excessive wound contraction, when occurring over a joint, may lead to disability
d. Actin microfillaments are found in fibroblasts and may play a role in wound contracture
Answer: b, c, d

Wound contraction is an important event which contrasts healing open wounds and closed incisions. When open wounds
contract, the surrounding skin is pulled over the open wound to reduce its size. This can occur much faster than
epithelialization. As opposed to other animals, human skin does not have a significant degree of mobility in most sites and
specifically on the lower leg, the skin is tightly adherent and less elastic. Therefore, although contraction may account for
90% of reduction of wound size on the perineum, it accounts for, at most, 30–40% of healing of a lower leg ulcer. All
healing wounds generate a strong contractile force. When this force is exerted across a joint, it may result in scar contracture
which may limit the functional range of motion. At the cellular level, the force which drives wound contraction comes from
fibroblasts. Fibroblasts, like muscle cells, contain actin microfilaments. When these filaments increase in number, the cells
take a morphologic appearance of myofibroblasts. Myofibroblasts are seen in an increased number in contracting wounds
and are felt to play an active role in the process of wound contraction.

139 There are a multitude of various dressings available. Which of the following statement(s) is/are true concerning
options for surgical dressings?
a. Hydrocolloids, such as karaya compounds, offer the primary advantage of increased absorptive ability
b. Films, such as Op-site, provide a water impermeable environment to achieve a dry wound
c. Impregnates are fine gauze impregnated with a variety of substances such as antibiotics or moisturizing agents that
adhere tightly to the wound and do not require a secondary dressing
d. Absorptive powders and paste are highly useful in debriding necrotic and fibrous material from wounds and
absorbing wound serum
Answer: a, d

Although the simplest dressing of gauze and tape combined with the use of antibacterial ointment can achieve moist wound
healing in most patients. A multitude of other products are available. These can be classified into films, foams,
hydrocolloids, hydrogels, and absorptive powders. Films are semipermeable to water, generally made of polyurethane, and
are nonabsorptive. They are useful to achieve a moist wound healing environment over a minimally exudative wound such
as split thickness skin graft donor sites. The hydrocolloids deserve special mention because they have achieved widespread
use. These agents contain hydrophilic materials such as karaya or carboxymethyl cellulose with an adhesive material and are
covered by a semipermeable polyurethane film. The material adheres to the skin surrounding the wound, is highly
absorptive, and achieves a moist healing environment. Impregnants are generally fine mesh gauze impregnated with either
moisturizing, antibacterial, or bactericidal compounds. They are generally not adherent and require a secondary dressing.
They do promote reepithelialization and have a antiinfective effect when combined with antibacterial or bactericidal agents.
A variety of absorptive powders and pastes are available which consist of starch copolymers or colloidal hydrophilic
particles. These agents have high absorbency for tissue wound fluid and debride necrotic and fibrous material from the
wound.

140 Which of the following statement(s) is/are true concerning the remodeling phase of wound healing?
a. Total collagen content increases steadily through this phase
b. The normal adult ratio of collagen is approximately 4:1 of type I to type III collagen.
c. Eventually a scar will achieve the strength of unwounded skin
d. The proteoglycans are responsible for the ground substance of the extracellular matrix
Answer: b, d

The transition from the proliferative phase to the remodeling phase of wound healing is defined by reaching collagen
equilibrium. Collagen accumulation within the wound becomes maximal by two to three weeks after wounding. Although
supramaximal rates of synthesis and degradation continue throughout remodeling, there is no further change in total
collagen content. During the initial phase of wound healing, there is a relative abundance of type III collagen in the wound.
With remodeling, the normal adult ratio of 4:1 (type I to type III) collagen is restored. The other important component of the
extracellular matrix is the ground substance or proteoglycans. These substances are composed of a protein background with
long hydrophilic carbohydrate side chains. The hydrophilic nature of these molecules accounts for much of the water
content of scar.
Scars never achieve the degree of order advanced by collagen in normal skin or tendons, but they do increase in strength for
six months or more, eventually reaching 70% of the strength of unwounded skin.
141 Which of the following statement(s) is/are true concerning pharmacologic agents used to accelerate wound
healing?
a. A number of these agents are now currently approved for use in this country
b. PDGF (platelet-derived growth factor) promotes fibroblast proliferation, chemotaxis, and collagenase synthesis
c. PDGF has been demonstrated in a number of clinical trials to promote healing in chronic wounds
d. Growth hormone functions by promoting fibroblast proliferation and collagen synthesis
Answer: b, c

Currently there are no approved clinical agents that accelerate normal healing. Although a number of clinical trials are in
progress, no agents are currently approved. PDGF (platelet-derived growth factor) accelerates wound healing by promoting
fibroblast proliferation and chemotaxis and collagenase synthesis. Clinical trials have demonstrated that PDGF has
accelerated healing in patients with chronic wounds such as pressure sores and diabetic ulcers. Growth hormone has been
successfully used in some situations to reverse the catabolic effect of severe injuries. Wound healing is fundamentally an
anabolic event, and in the setting of a severe burn, growth hormone administration significantly accelerates donor site
healing, presumably due to its effects in minimizing catabolism.

142 Which of the following statement(s) describe the effects of aging on wound healing?
a. A finer, more cosmetic scar might be expected
b. In vitro studies demonstrate decreased proliferative potential of fibroblasts and epithelial cells
c. Skin sutures should be left in for a longer period of time
d. Wound infection occurs more frequently in elderly patients due to diminished ability to fight infection
Answer: a, b, c

There are important age-dependent aspects of wound healing. The elderly heal more slowly and with less scarring. There is
a gradual attenuation of the inflammatory response with age, and decreased wound healing is one of the consequences. In
vitro studies have documented an age-dependent decrease in proliferative potential of fibroblasts and epithelial cells.
Clinically this will account for the formation of finer scars and improved cosmetic appearance in the elderly. Sutures should
be left in place longer to allow for the slow regain of tensile strength in the aged. This can also be done without concern for
formation of suture marks as slower epithelialization occurs along the sutures. There is no evidence to suggest that wound
infections occur more commonly in elderly patients.

143 Reconstitution of the epithelial barrier (epithelialization) begins within hours of the initial injury. Which of the
following statement(s) is/are true concerning the process of epithelialization?
a. Bacteria, protein exudate, and necrotic tissue all will compromise this process
b. Epithelial cells exhibit contact proliferation
c. Epithelialization occurs only from the margins of the wound
d. Visible scarring can occur only when the injury extends deeper than the superficial dermis
Answer: a, d

The initial step of epithelialization involves epithelial cells from the basal layer of the wound edge flattening and migrating
across the wound, completing wound coverage within 24–48 hours in a co-opted surgical wound. Epithelial cells exhibit
contact inhibition. That is, they will continue to migrate across an appropriate bed until a single continuous layer is formed.
Epithelial cell migration occurs by a process in which the epithelial cells send out pseudopods, attaching to the underlying
extracellular matrix by integrin receptors. Bacteria, large amounts of protein exudate from leaky capillaries, and necrotic
tissue all compromise this process delaying epithelialization. In the case of open wounds, epithelialization results from
migration of epithelial cells from remaining dermal appendages, sweat glands, and hair follicles, if the dermis is not
completely destroyed. In a full thickness injury, the entire dermis is destroyed or removed. Epithelialization therefore occurs
only at the margins of a wound, at a dermal rate of 1–2 mm/day.
Visible scarring occurs only when the injury extends deeper than the superficial dermis. Superficial abrasions and burns
usually heal without scar, while deeper abrasions and burns may scar significantly. Whenever the dermis is incised, a scar
will form.

144 Scar formation is part of the normal healing process following injury. Which of the following tissues has the ability
to heal without scar formation?
a. Liver
b. Skin
c. Bone
d. Muscle
Answer: c
Every tissue in the body undergoes reparative processes after injury. Bone has the unique ability to heal without scar and
liver has the potential to regenerate parenchyma, the only organ that has maintained that ability in the adult human.
Although liver does regenerate, it often heals with scar (cirrhosis) as well. With these exceptions, all other mature human
tissues heal with scar.

145 Which of the following factors have been demonstrated to promote wound healing in normal individuals?
a. Vitamin A supplementation
b. Vitamin C supplementation
c. Vitamin E application to the wound
d. Zinc supplementation
e. None of the above
Answer: e

Several important systemic factors or conditions influence wound healing. Interestingly, there are no known systemic
conditions that lead to enhanced or more rapid wound healing. Overall nutrition as well as adequate vitamins play an
important role in wound healing. Vitamin A is involved in the stimulation of fibroplasia, collagen cross-linking, and
epithelialization. Although there is no conclusive evidence in humans, vitamin A may be useful clinically for steroid-
dependent patients who have problematic wounds or who are undergoing extensive surgical procedures. Vitamin C is a
necessary cofactor in hydroxylization of lysine and proline in collagen synthesis and cross-linkage. The utility of vitamin C
supplementation in patients who otherwise take in a normal diet has not been established. Vitamin E is applied to wounds
and incisions empirically by many patients. The evidence to support this practice is entirely anecdotal. In fact, large doses of
vitamin E have been found to inhibit wound healing. Zinc and copper are also important cofactors for many enzyme
systems that are important to wound healing. Deficiency states are seen with parenteral nutrition but are rare and readily
recognized and treated with supplements. Overall, vitamin and mineral deficiency states are extremely rare in the absence of
parenteral nutrition or other extreme dietary restrictions. There is no evidence to support the concept that supranormal
provision of these factors enhance wound healing in normal patients.

146 Which of the following statement(s) is/are true concerning excessive scarring processes?
a. Keloids occur randomly regardless of gender or race
b. Hypertrophic scars and keloid are histologically different
c. Keloids tend to develop early and hypertrophic scars late after the surgical injury
d. Simple reexcision and closure of a hypertrophic scar can be useful in certain situations such as a wound closed by
secondary intention
Answer: d

True keloids are uncommon and occur predominantly in dark skinned people with a genetic predisposition for keloid
formation. In most cases, the gene appears to be transmitted as an autosomal dominant pattern. The primary difference
between a keloid and a hypertrophic scar is that a keloid extends beyond the boundary of the original tissue injury. It
behaves as a tumor and extends into or invades the normal surrounding tissue creating a scar that is larger than the original
wound. Histologically, keloids and hypertrophic scars are similar. Both contain an overabundance of collagen. Although the
absolute number of fibroblasts is not increased, the production of collagen continually out paces the activity of collagenase,
resulting in a scar of ever increasing dimensions. Hypertrophic scars respect the boundaries of the original injury and do not
extend into normal unwounded tissue. There is less of a genetic predisposition, but hypertrophic scars also occur more
frequently in Orientals and the Black population. They are often seen on the upper torso and across flexor surfaces. Some
improvement in a keloid can be obtained with excision followed by intra-lesional steroid injection. However, the resulting
scar is unpredictable and potentially worse. Reexcision and closure should, however, be considered for hypertrophic scars, if
the condition of closure can be improved. This is especially pertinent for wounds that originally healed by secondary
intention or that are complicated by infection. Keloids typically develop several months after the injury and rarely, if ever,
subside. Hypertrophic scars usually develop within the first month after wounding and often subside gradually.

147 Which of the following statement(s) is/are true concerning the vascular response to injury?
a. Vasoconstriction is an early event in the response to injury
b. Vasodilatation is a detrimental response to injury with normal body processes working to avoid this process
c. Vascular permeability is maintained to prevent further cellular injury
d. Histamine, prostaglandin E2 (PGE2) and prostacyclin (PGI2) are important mediators of local vasoconstriction
Answer: a
After wounding, there is transient vasoconstriction mediated by catecholamines, thromboxane, and prostaglandin F2
(PGF2a). This period of vasoconstriction lasts for only five to ten minutes. Once a clot has been formed and active bleeding
has stopped, vasodilatation occurs in an around the wound. Vasodilatation increases local blood flow to the wounded area,
supplying the cells and substrate necessary for further wound repair. The vascular endothelial cells also deform, increasing
vascular permeability. The vasodilatation and increased endothelial permeability is mediated by histamine, PGE2, and
prostacyclin as well as growth factor VEGF (vascular endothelial cell growth factor). These vasodilatory substances are
released by injured endothelial cells and mast cells and enhance the egress of cells and substrate into the wound and tissue.

148 Which of the following statement(s) concerning laboratory studies used in monitoring a patient with intravenous
heparinization is/are correct?
a. The platelet count should be followed because of the risk of heparin-associated thrombocytopenia
b. The prothrombin time should be observed if prolonged treatment is necessary
c. The activated partial thromboplastin time (aPTT) should be maintained at approximately 1.5 times normal
d. The serum creatinine should be measured daily to allow adjustments in dose based on renal function
Answer: a, c

In monitoring the effect of heparin, an activated partial thromboplastin time (aPTT) of 1.5 control or a thrombin clotting
time (TCT) of 2 times control reflects adequate anticoagulation. The prothrombin time remains normal. Heparin-associated
thrombocytopenia from an immune mechanism is a potential complication of the use of this anticoagulant. Therefore any
patient undergoing heparin therapy should have a platelet count determined every other day after the fourth day of therapy
or earlier if he or she is known to have been exposed to heparin in the past. Heparin is not excreted through the kidneys or
the liver but is cleared through the reticuloendothelial system. Therefore the dose of heparin need not be adjusted in cases of
liver or renal dysfunction.

149 Which of the following statement(s) is/are true concerning heparin-associated thrombocytopenia?
a. Heparin-associated thrombocytopenia occurs only in the face of over anticoagulation with heparin
b. Severe thrombocytopenia (platelet count less than 100,000) is seen in less than 10% of patients treated with
heparin
c. Heparin-associated thrombocytopenia is due to the aggregation of platelets and may result in thrombosis or
embolic episodes
d. Heparin-associated thrombocytopenia may be seen within hours of initiation of heparin therapy
Answer: b, c

Heparin-associated thrombocytopenia occurs in 0.6% to 30% of patients who receive heparin, although severe
thrombocytopenia (platelet counts less than 100,000) is seen in fewer than 10% of patients treated with heparin. It is caused
by a plasma factor, most likely a heparin-dependent platelet antibody, that causes aggregation of platelets when exposed to
heparin. Activation of platelets in this setting results in thrombocytopenia, thrombosis and embolic episodes, which can lead
to death. Both bovine and porcine heparin have been associated with this syndrome, which usually begins 5 to 15 days after
initiating heparin therapy. Even trivial exposure with heparin such as coating on pulmonary artery catheters or low rate
infusion into arterial catheters may cause this syndrome.

150 Antithrombin III deficiency is a commonly observed hypercoaguable state. Which of the following statement(s)
is/are true concerning this condition?
a. A patient with this deficiency usually presents with thrombosis while on heparin or exhibits an inability to become
adequately anticoagulated with heparin
b. This deficiency may be either congenital or acquired
c. Thrombotic episodes are related to predisposing events such as operations, childbirth, and infections
d. Treatment involves acutely the administration of fresh frozen plasma followed by long-term treatment with
Coumadin
Answer: a, b, c, d

Antithrombin III deficiency accounts for about 2% of venous thrombotic event. This deficiency has been described in
patients with pulmonary embolism, mesenteric venous thrombosis, lower extremity venous thrombosis, arterial thrombosis,
and dialysis fistula failure. Antithrombin III is a serine protease inhibitor of thrombin and factors Xa, IXa and XIa. Because
one of the main actions of heparin is to potentiate the anticoagulant effects of antithrombin III, a patient with this deficiency
usually presents with thrombosis while on heparin or exhibits the inability to become adequately anticoagulated with
heparin. This deficiency may be either congenital (1n2000–5000 births) or acquired. Acquired defects occur with inadequate
production, as in liver disease, malignancy, nephrotic syndrome, disseminated intervascular coagulation, malnutrition, or
increased protein catabolism. Thrombotic episodes are related to predisposing events such as operations, childbirth, and
infections. Once the diagnosis of antithrombin III deficiency is established, fresh frozen plasma should be administered
followed by long-term treatment with Coumadin.

151 Mini-dose heparin has been shown to be useful in the prophylaxis of postoperative venous thrombosis.
Mechanism(s) by which low-dose heparin is/are thought to protect against venous thrombosis include:
a. Enhancement of antithrombin III activity
b. A decrease in thrombin availability
c. Inhibition of platelet aggregation and subsequent platelet release action
d. A mild prolongation of activated partial thromboplastin time
Answer: a, b, c

Low-dose heparin is thought to protect against venous thrombosis through three different mechanisms. First, antithrombin
III activity with its inhibition of activated Factor X is enhanced by only trace amounts of heparin; second, there is a decrease
in thrombin availability that prevents its activation and thus its fibrin-stabilizing effect; and third, small doses of heparin
may inhibit the second wave of platelet aggregation and subsequent platelet release reaction. The standard doses of heparin
administered (5000 units bid) does not affect aPTT.

152 Tests of coagulation are used to monitor anticoagulation treatment and detect intrinsic abnormalities in coagulation.
Which of the following statement(s) is/are true concerning coagulation tests?
a. Prothrombin time (PT) measures both the intrinsic and extrinsic clotting pathways and fibrinogen
b. Activated partial thromboplastin time (aPTT) can be used to monitor both oral anticoagulation with Warfarin and
intravenous anticoagulation with heparin
c. Thrombin clotting time (TCT) is a measurement of the time it takes for exogenously administered thrombin to turn
plasma fibrinogen into fibrin clot
d. Whole blood activated clotting time (ACT) is a measurement of the ability of whole blood to clot and is used to
monitor heparin levels intraoperatively during cardiovascular and peripheral vascular operations
Answer: a, c, d

Coagulation tests include prothrombin time (PT), which measures the intrinsic and extrinsic pathways of fibrinogen
production and is the most common method for measuring a level of oral anticoagulant therapy. The activated partial
thromboplastin time (aPTT) identifies the abnormalities of the contact and intrinsic phases of coagulation. Values of aPTT
have variably been shown to correlate with heparin dosages and serum heparin levels and are therefore most commonly
used in monitoring heparin therapy. It is of no value in long-term management of patients on oral Warfarin therapy.
Thrombin clotting time (TCT) is the measure of the time it takes for exogenously administered thrombin to turn plasma
fibrinogen into fibrin clot. It is extremely sensitive to levels of heparin and is an excellent measure of measuring the level of
heparin-induced anticoagulation. The beauty of the TCT is that it is not specific for any disease condition; thus it may be
used to differentiate factor deficiencies from the presence of heparin, or to separate lupus anticoagulant from abnormalities
in fibrinogen levels. The whole blood activated clotting time (ACT) is a measurement of the ability of whole blood to clot,
and as such, is an available technique for monitoring heparin levels intraoperatively. The ACT responds in a linear fashion
to increasing heparin dosage and correlates well with the observed clinical anticoagulation. Adequate anticoagulation for
extracorporeal circulation is defined as an ACT of 480 seconds or more while for peripheral vascular applications, values of
250 seconds or greater are considered appropriate.

153 Thrombolytic therapy has become a useful adjunct in the management of peripheral arterial occlusion. In this
setting, direct intraarterial administration rather than intravenous has been advocated to decrease the risk of systemic
bleeding. Which of the following true statement(s) concerning the use of thrombolytic agents for arterial occlusion is/are
true?
a. A standard technique involves infusing high-dose urokinase, 4000 units per minute for 1–2 hours, directly into the
clot by a catheter embedded in the thrombus
b. If progress is made, further fibrinolytic therapy is given at 1000 to 2000 units per minute until clot lysis has
occurred
c. The usual infusion time by the above-stated technique is usually in excess of 24 hours
d. Successful clot lysis occurs more frequently in arterial graft occlusions than native arterial occlusions
e. The use of intraoperative thrombolytic therapy may be indicated for situations where complete clot evacuation
cannot be accomplished surgically
Answer: a, b, e

The most popular method for intraarterial thrombolytic therapy for arterial occlusion involves passing a guidewire through
the thrombus with arteriographic guidance and then infusing high-dose urokinase, 4000 units per minute for 1–2 hours,
directly into the clot. If progress is made, further fibrinolytic therapy is given at 1000 to 2000 units per minute for a 6–12
hour period or until clot lysis has occurred. Using this technique, mean infusion time in a recent study was found to be 18
hours and the incidence of bleeding complications was significantly lessened. Selective intraarterial infusion of urokinase
was associated with complete clot resolution in 77% of native arterial occlusions versus only 41% with arterial graft
occlusion. After thrombolytic therapy has reopened an occluded vessel or graft, radiologic or surgical correction of the
lesion responsible for the thrombosis in the first place must be addressed for any hope of long-term success. The use of
intraoperative thrombolytic therapy is advocated in those situations where complete clot resolution cannot be accomplished
(such as following balloon embolectomy for acute arterial occlusion) or when distal vasculative is occluded and precludes
appropriate inflow patency.

154 Which of the following statement(s) is/are true concerning hemophilia A?


a. Hemophilia A is inherited as a sex-linked recessive deficiency of factor VIII
b. A positive family history for bleeding disorders present in all patients
c. Laboratory tests reveal a prolongation of aPTT, prothrombin time (PT), thrombin clotting time and platelet
aggregation
d. Spontaneous bleeding is unusual with factor VIII levels greater than 10% of normal
Answer: a, d

Hemophilia A is inherited as a sex-linked recessive deficiency of factor VIII although 0% of cases are secondary to
spontaneous mutation. The incidence of this abnormality is approximately 1/10,000 births. Laboratory screening tests
usually reveal a prolongation of an aPTT but normal prothrombin time (PT), thrombin clotting time (TCT) and platelet
aggregation testing. The minimum level of VIII required for hemostasis is 30% for minor bleeding, whereas spontaneous
bleeding is unusual with factor levels greater than 5 to 10% of normal. In severe genetic deficiency states however, factor
levels as low as 1% have been noted and patients are at risk for spontaneous bleeding.

155 Fibrinolytic therapy is based on activation of plasminogen, the inactive proteolytic enzyme of plasma that binds to
fibrin during the formation of thrombosis. Activation of plasminogen to plasmin results in selective thrombolysis at the
fibrin clot surface. Which of the following statement(s) is/are true concerning agents used in thrombolytic therapy?
a. Streptokinase is a bacterial protein which is antigenic in humans, resulting in allergic reactions in up to l5% of
cases
b. Tissue plasminogen activator acts directly on plasmin without an intermediate drug–plasmin complex
c. The half-life of urokinase, streptokinase, and TPA all exceed 30 minutes
d. Streptokinase is significantly cheaper than urokinase or TPA
Answer: a, b, d

Streptokinase is a bacterial protein produced by group C b-hemolytic streptococci. It is therefore antigenic in humans and
can be associated with allergic reaction in between 2 and 18% of cases. In addition an unusual serum sickness has been
reported with streptokinase. Neither urokinase or TPA which is now manufactured with recombinant DNA technology are
either associated with allergic side effects or antigenicity. Streptokinase acts through a streptokinase-plasmin complex,
whereas urokinase and TPA act directly on plasmin without intermediate drug plasmin complex. The level of the lytic state
is greatest with streptokinase, intermediate with urokinase, and least with TPA with the half-lives ranging all less than 1/2
hour in duration. Although the relative efficacy of the three agents has been compared in a number of studies, there appears
to be no significant benefit of one agent over the other. Streptokinase however, is markedly less expensive than either
urokinase or TPA.

156 Von Willebrand’s disease is a common, congenital bleeding disorder. Which of the following statement(s) is/are
true concerning Von Willebrand’s disease?
a. As in hemophilia, it is much more common in men
b. A history of spontaneous bleeding is common
c. Screening laboratory tests will include a prolonged aPTT with a normal prothrombin time
d. Pre-treatment for elective surgery require administration of cryoprecipitate to achieve levels of 23–50% of normal
Answer: c, d

Von Willebrand’s factor is an adhesive protein that mediates platelet adhesion to collagen. In addition, it protects and
prevents the rapid removal of factor VIII from blood. The classical deficiency state, Von Willebrand’s disease, is caused by
reduction of factor VIII activity (although not as great as Hemophilia A) and the Von Willebrand factor. Clinical
manifestations include epistaxis, gingival bleeding, menorrhagia, rare joint or muscle bleeding, and subcutaneous bleeding.
Spontaneous bleeding is not as common as in classic Hemophilia A. The syndrome is transmitted as both autosomal
dominant (heterozygous) and autosomal recessive disease (homozygous) traits. Therefore there is no sex predilection.
Screening laboratory tests include a prolonged aPTT with a normal prothrombin time. In addition, because of the
importance of this factor in platelet adhesion, patients display a prolonged bleeding time and have decreased level of factor
VIII activity, decreased immunoreactive levels of Von Willebrand’s antigen, and abnormal platelet aggregation responses to
ristocetin. The most reliable source of Von Willebrand’s factor is cryoprecipitate.

157 External pneumatic compression has been advocated for the prevention of deep venous thrombosis during
operative procedures. Which of the following statement(s) concerning the use of external pneumatic compression devices
is/are true?
a. Intermittent pneumatic compression is as effective as low-dose heparin in prevention of venous thrombosis
b. These devices function by compressing the lower extremities therefore augmenting venous return
c. Pneumatic compression devices may also exhibit their antithrombotic effect through stimulating local and systemic
fibrinolysis
d. The length of time that intermittent pneumatic compression should be used includes through the operation and for
at least several days in the postoperative period
Answer: b, c, d

In many well-controlled studies of venous prophylaxis, intermittent pneumatic compression has been found to be as
effective as low-dose heparin therapy. In addition to augmentation of venous return with these devices, local and systemic
fibrinolysis appears to be stimulated. Fibrinolytic activities are usually reduced for a 7–10 day period after an operation.
Studies have demonstrated that the pneumatic-compression devices may exhibit their antithrombotic effect through
prevention of this fibrinolytic shutdown even when applied to the upper extremity. The length of time that intermittent
pneumatic compression should be used has not been adequately determined but most data suggest that devices should be
used through the operation and for at least five days in the face or prolonged immobilization.

158 The standard management oral anticoagulant therapy for chronic treatment of venous thromboembolism is with the
drug warfarin. Which of the following statement(s) is/are true concerning the administration of warfarin?
a. An important complication of warfarin therapy is skin necrosis in patients with protein C deficiency
b. Warfarin interferes with vitamin K dependent clotting factors II, VII, IX, X
c. For effective anticoagulation the prothrombin time (PT) should be kept at 2 control
d. It is recommended that warfarin be continued for at least one year after initial episode of deep venous thrombosis
Answer: a, b

Warfarin interferes with the vitamin K dependent clotting factors II, VII, IX and X, protein C, and protein S. An important
complication of warfarin is skin necrosis with patients both with and without protein C deficiency. This syndrome usually
involves full thickness skin slough over fatty areas such as the breasts and buttocks. Warfarin therapy should be monitored
using the one stage prothrombin time (PT). The PT should be kept at 1.3 to 1.4 control for effective anticoagulation. At
higher levels, there is a five-fold increase in the frequency of bleeding complications. Two major complications of Warfarin
therapy include recurrent thrombosis and bleeding. It is recommended that Warfarin be continued four months after an
initial episode of deep venous thrombosis. Between ten weeks and four to six months after deep vein thrombosis, there is a
recurrent thrombosis rate of 8.3 episodes per 1000 patient months. Between four months and three years, recurrences fall to
four episodes per 1000 patient-months. At four months, the risks of bleeding complications matches and exceeds the benefit
from anticoagulant therapy and thus is the basis for discontinuing warfarin administration at this time.

159 Which of the following statement(s) is/are true concerning the management of a patient with hemophilia A
undergoing an elective surgical operation?
a. Concentrates of factor VIII should be given several days prior to elective surgery
b. The half-life of factor VIII concentrates is less than 24 hours
c. A dose of 40–50 IU/kg of factor VIII concentrate should be given prior to the planned surgical procedure
d. Factor VIII concentration administration should be given for the first 24 hours after surgery but may then be
stopped if no abnormal bleeding has been observed
e. A new recombinant preparation of factor VIII offers the advantage of being virus-free
Answer: b, c, e

Although the half-life of factor VIII is 2.9 days in normal individuals, the half-life of factor VIII concentrates is 9 to l8
hours. Levels of 80% to 100% of normal should be obtained for surgical bleeding or life-threatening hemorrhage. A dose of
40 to 50 IU/kg of factor VIII should be given with half of this dose then administered every twelve hours. After surgery,
transfusion of factor VIII concentrates should be continued for at least ten days. Unfortunately, past use of concentrates of
factor VIII obtained from donors has led to a high incidence of HIV infection in the hemophilia population. A new
recombinant preparation of factor VIII offers the advantage of being virus-free.
160 Transfusions of blood products can be associated with a number of complications including immediate and delayed
hemolytic reactions; nonhemolytic reactions; infectious disease transmission; and complications of massive transfusions.
Which of the following statements are true concerning complications of blood transfusions?
a. Immediate hemolytic transfusion reactions are caused by major ABO blood group incompatibility
b. Nonhemolytic transfusion reactions are usually due to RH incompatibility and are therefore more common in
women of childbearing age
c. The most common complication of massive blood transfusion is dilutional thrombocytopenia
d. Routine impaired calcium supplementation is necessary during most massive transfusion episodes
Answer: a, c

Immediate hemolytic reactions are usually caused by blood group ABO incompatibility although they may be caused by
antigens of other blood group systems on the transfused red blood cells. The clinical manifestations revolve around the
antigen on the red blood cell stroma and the antibody in the patient’s serum, and include production of bradykinin,
compliment activation, release of vasoactive agents from platelets, and initiation of systemic clotting. Chills and fevers,
chest pain and lumbar pain, tachycardia and hypotension in the conscious patient, and often diffuse bleeding in the
anesthetized, unconscious patient constitute this syndrome. Although reaction occurs immediately, death related to the
syndrome is uncommon, unless associated with a transfusion of more than 100 ml of blood. Death usually occurs from acute
renal failure or hemorrhage due to DIC. Nonhemolytic reactions occur with the frequency of 1 to 2% of all transfusions and
consist primarily of chills and fevers during the transfusion or in the first 2 to 3 hours after the transfusion is complete.
Mechanism of these reactions includes the presence of antibodies to white blood cell antigens in the transfused blood,
especially in the multitransfused or multiparous patient. Massive transfusion complications relate to the rate and volume of
blood transfused. The most common complication is dilutional thrombocytopenia. Factor deficiency of the labile factors V
and VIII rarely is of sufficient magnitude to result in problems with hemostasis. For hypocalcemia to occur with massive
transfusion, citrated blood must be administered, one unit every five minutes. Routine empiric calcium supplementation is
unnecessary during most massive transfusion episodes. Conversely, hypothermia is clearly a problem, especially when
associated with massive transfusion during complex intraoperative procedures such as thoracoabdominal aneurysm
resection.

161 A 67-year-old male with advanced cholangiocarcinoma develops gram-negative sepsis. Excessive bleeding is noted
around vascular catheters and from needle puncture sites. The diagnosis of disseminated intervascular coagulation (DIC) is
considered. Which of the following laboratory test(s) is/are indicative of DIC?
a. Decreased platelet count
b. Decreased fibrinogen level
c. Normal prothrombin time
d. Elevated fibrin split products
Answer: a, b, d

Disseminated intravascular coagulation (DIC) is the primary form of acute thrombosis. Causes of this syndrome include
abruptio placenta, gram-positive and gram-negative sepsis, endotoxemia, malignant tumors, pelvic operations, certain snake
bites, hematologic malignancies, and hepatic failure. Blood coagulation is activated by the release of tissue factor into the
circulation, which activates factor VII of the extrinsic pathway to VIIa, leading to massive thrombin production and fibrin
generation. This in turn activates the fibrinolytic system, leading to bleeding in the later stages of the syndrome due to
consumption of coagulation factors, depletion of fibrinogen, and unchecked plasma activities. Laboratory values in DIC
usually include a decline in the platelet count and fibrinogen level, along with an elevation of fibrin split products.

162 Which of the following substances, not normally present in the circulation, trigger the initiating events in the
hemostatic process?
a. Thrombin
b. Platelet factor 3
c. Tissue factor
d. Collagen
Answer: c, d

The initiating agents for hemostasis involve two substances that are not normally present in the circulation—collagen and
tissue factor. Tissue factor is released from injured cells, beginning the activation of the extrinsic pathway of coagulation,
while disruption of the protective endothelial barrier of blood vessels exposes the underlying collagen to the activation of
platelets. In the bloodstream, tissue factor complexes with factor VII which then activates factor X to factor Xa. At the same
time, activated platelets change from their discoid shape with their procoagulant phospholipid (termed platelet factor 3)
buried on the inner side of the surface membrane to a spreading shape to allow for the externalization of platelet factor 3
activity. Activated factor X, activated factor V, ionized calcium and factor II (prothrombin) then assemble on the platelet
phospholipid surface to form the so-called prothrombinase complex which catalyzes the formation of thrombin.

163 Bleeding complications are frequently associated with fibrinolytic therapy. Which of the following statement(s)
concerning complications of fibrinolytic therapy is/are true?
a. Careful monitoring of prothrombin time and aPTT time are necessary to avoid bleeding complications
b. A level of serum fibrinogen less than 100 mg/dl is associated with an increased risk of bleeding
c. Recent (less than 10 days) major surgery is a contraindication to systemic but not regional fibrinolytic therapy
d. A patient with a cerebrovascular event occurring less than two months ago can be treated with fibrinolytic therapy
if head CT scan is normal
Answer: b

Fibrinolytic therapy induces a hemostatic defect through a combination of factors. Hypofibrinogenemia and fibrin
degradation products inhibit fibrin polymerization and, in combination with a decrease in the clotting factors V and VIII,
prolong the ability of blood to clot. However, coagulation tests in general do not correlate well with bleeding complications.
A level of fibrinogen less than 100 mg/dl is associated with an increased risk of bleeding. Absolute contraindications to
thrombolytic therapy include active internal bleeding, recent (less than 2 months) cerebral vascular accident, and
documented left heart thrombosis. Recent (less than 10 days) major surgery, obstetric delivery, organ biopsy, or major
trauma is considered a major relative contraindication to either regional or systemic thrombolytic therapy.

164 Which of the following statement(s) is/are true concerning the results of a National Institute of Health Consensus
Conference on venous thrombosis and low-dose heparin prophylaxis?
a. The odds of developing deep venous thrombosis with low-dose heparin prophylaxis decreases by 67%
b. The risk of pulmonary embolism is decreased by almost 50%
c. There is no increase in mortality from other causes found in patients treated with low-dose heparin
d. There was no difference in the incidence of bleeding complications
Answer: a, b, c

In a metaanalysis of 70 randomized trials in 16,000 patients comparing low-dose heparin prophylaxis with standard therapy,
the odds of developing deep venous thrombosis with low-dose heparin prophylaxis decreased 67%, whereas for pulmonary
embolism (both fatal and non-fatal), the odds decreased by 47%. For fatal pulmonary embolism, the odds reduction was
even greater (64%). No increase in mortality from other causes was found in those patients treated with low-dose heparin.
Bleeding complications were more frequent in the heparin-treated patients, with no difference between 5000 units twice
daily and 5000 units three times daily. Similarly, the effectiveness of prophylaxis was not influenced by either two or three
times daily dosage.

165 Laboratory monitoring of coagulation and anticoagulation includes testing of platelet function. Which of the
following statements is/are true concerning tests of platelet function?
a. A platelet count of 50,000/µL or more usually ensures hemostasis
b. Bleeding time assays assessibility of platelets to perform hemostatic plugs and is determined from a sample of
blood drawn in an EDTA coated test tube
c. Aspirin therapy can be associated with a bleeding time in the range of 8–15 minutes
d. Tests of platelet aggregation should be part of the standard preoperative evaluation of patients using aspirin
Answer: a, c

Tests of platelet function include peripheral platelet counts, bleeding times, and platelet aggregation. Usually, a platelet
count of 50,000/mL or more ensures adequate hemostasis, whereas counts less than 10,000/mL are dangerous and may lead
to spontaneous bleeding. Bleeding time performed by observing the clotting of blood induced with a small needle stick,
assesses the ability of platelets to perform hemostatic plugs and are usually shorter than eight minutes. A bleeding time
between 8 and 15 minutes most often reflects a low plasma level of Von Willebrand’s Factor or the use of antiplatelet drugs.
A bleeding time greater than 15 minutes is clearly prolonged and indicates severe platelet functional impairment. Platelet
aggregation studies involve the use of a number of different agonists. Although a relatively straightforward technique,
platelet aggregation is not available in most laboratories, probably because of the observer-dependent nature of the test.

166 As thrombin generation proceeds, the body has natural anticoagulant systems opposing further thrombus
formation. Natural anticoagulants include:
a. Tissue plasminogen activator (TPA)
b. Antithrombin III
c. Activated protein C
d. Heparin cofactor II
Answer: b, c, d

Just as thrombin generation is the key to coagulation, antithrombin III is the most central anticoagulant proteins. This
glycoprotein binds to thrombin, preventing its removal of fibrinoprotein A and B from fibrinogen, prevents the activation of
factor V and VIII and the activation and aggregation of platelets. The second line of defense is the activated protein C,
which inactivates factors Va and VIIIa. This inactivation reduces the ability of the prothrombinase complex to accelerate the
rate of thrombin formation. A third natural anticoagulant is heparin cofactor II. Its concentration in plasma is estimated to be
some four-fold lower than antithrombin III, and its action is primarily implicated in the regulation of thrombin formation in
extravascular tissues. Tissue plasminogen activator (TPA) is a natural catalyst for the activation of plasminogen to plasmin,
the main fibrinolytic enzyme in the body. Therefore, TPA is part of the fibrinolytic system rather than a natural
anticoagulant.

167 Infectious disease transmission during blood transfusions is of clinical significance to surgeons and of major
importance to patients contemplating surgery potentially associated with the need for blood administration. Which of the
following statement(s) is/are true concerning the transmission of infectious disease during blood transfusions?
a. Post-transfusion hepatitis is usually due to hepatitis B
b. Hepatitis and HIV transmission is greatest with the administration of pooled plasma products such as serum
albumin
c. The most important cause of post-transfusion disease in immunosuppressed patients is CMV infection
d. The risk of HIV transmission in blood transfusions is significantly less than the risk of hepatitis transmission
Answer: c, d

The most common infectious diseases transmitted during blood transfusions include viral hepatitis, CMV, and HIV
infection. Post-transfusion hepatitis in 90% of cases consists of non-A, non-B hepatitis known as hepatitis C. All blood
products except for immune serum globulin and albumin can carry and transmit this form of hepatitis. Because heat
treatment eliminates the risk of viral transmission, products from pooled plasma that are heat treated such as albumin are not
at risk for HIV or hepatitis transmission. CMV transmission exists in three forms—primary, reinfection, and reactivation.
Primary exposure results in an IgM response to the virus. Reactivation is most commonly related to pregnancy,
transplantation, and immunosuppression, and is the most important cause of post-transfusion disease accompanying
immunosuppression of patients. Although the risk of the public concern for transmission of HIV disease associated with
blood transfusions has significantly outweighed other infectious disease transmission, the risks of HIV transmission is
markedly less than that of hepatitis.

168 There are a number of hypercoaguable states which can be associated with arterial or venous thrombosis and
embolic phenomenon. These include:
a. Heparin-associated thrombocytopenia
b. Antithrombin III deficiency
c. Von Willebrand disease
d. Vitamin C deficiency
Answer: a, b

A number of hypercoaguable states are present. These include heparin-associated thrombocytopenia in which a heparin-
dependent platelet antibody causes aggregation of platelets when the patient is exposed to heparin. Activation of platelets in
this setting results in thrombocytopenia, thrombosis, and embolic episodes. Antithrombin III deficiency accounts for about
2% of venous thrombotic events and has been described in pulmonary embolism, mesenteric venous thrombosis, lower
extremity venous thrombosis, arterial thrombosis, and dialysis fistula failure. Von Willebrand’s disease is a hereditary
complex coagulation factor deficiency which is manifested by a reduction of factor VIII activity, and the Von Willebrand
factor which is an adhesive protein that mediates platelet adhesion to collagen. Severe vitamin C deficiency results in a
disorder in soft tissue increasing vascular permeability and fragility resulting in the potential for bleeding disorders.

169 Cytokines with clearly defined actions in acute inflammation and early tissue injury include which of the
following?
a. Cysteine-X-Cysteine (C-X-C) chemokines
b. Tumor Necrosis Factor (TNFa)
c. Transforming Growth Factor-b (TGF-b)
d. Interleukin-6 (IL-6)
e. Platelet Derived Growth Factor (PDGF)
Answer: a, b, c, d, e

Polypeptide mediators, such as TNFa and IL-1, are considered “early response” cytokines and are actively involved in the
initiation of the cascade of events which precipitate acute inflammation. In addition to being important triggers for the
induction of other cytokines important inflammatory network, TNFa and IL-1 appear to be key mediators in promoting the
adherence of inflammatory cells to the endothelium. IL-1 is a complex, multifunctional molecule that shares many
overlapping biological properties with TNFa. In addition, both IL-1 and TNFa potentiate the effects of one another. The
most important function of IL-6 appears to be the regulation of the hepatic acute phase response. Following injury, a number
of physiologic changes develop within several hours. IL-6 is one of the primary stimuli for the production of acute phase
proteins from the liver. Endotoxin, IL-1, TNFa and PDGF are capable of causing significant induction of IL-6 synthesis.
Over the last decade, at least 12 different C-X-C chemokines have been identified. These include IL-8, one of the most
potent mediators of chemotaxis known. TNFa and IL-1 are key molecules for the induction of IL-8, which in turn is
important for the induction of neutrophil recruitment and activation.
Similar properties are apparent for other members of this chemokine family.
Platelet activation and degranulation occur during coagulation following injury, leading to the deposition of a number of
cytokines into the provisional matrix. These cytokines include transforming growth factor-a, (TGFa), transforming growth
factor b (TGF-b), platelet-derived growth factor (PDGF), and neutrophil activating peptide-2 (NAP-2). These cytokines are
either important growth factors or chemotaxis for leukocytes, endothelial cells, fibroblasts, and keratinocytes which are key
components in the process of tissue repair. Thus, coagulation and platelet activation provide the initial foundation for
subsequent cellular recruitment.

170 Which of the following statements regarding transforming growth factor b (TGF-b) are true?
a. TGF-b expression is autoregulated
b. TGF-b enhances collagen synthesis
c. TGF-b inhibits extracellular matrix production
d. TGF-b may inhibit or promote cellular proliferation
Answer: a, b, d

TGF-b appears to be one of the key cytokines in control of tissue repair. TGF-b is strongly chemotactic for neutrophils, T
cells, monocytes, and fibroblasts. TGF-b activates inflammatory cells to elaborate fibroblast growth factor, TNFa, IL-1 and
increase their synthesis of extracellular matrix proteins. TGF-b also induces both the infiltrating cells and resident cells to
produce more TGF-b. This auto-induction amplifies its biological effects at the site of injury and may play an important role
in the development of chronic fibrosis in a variety of pathologic states. TGF-b enhances collagen synthesis as well. Lastly,
TGF-b may function as a mitogen or growth inhibitor for a wide variety of cell types, including selected cell types of
mesenchymal origin. Whether TGF-b stimulates or inhibits proliferation depends on the presence of other growth factors,
the concentration of TGF-b, and the cell density. Thus, at low doses, TGF-b stimulates the proliferation of densely plated
human marrow fibroblasts, but is inhibitory at high concentrations.

171 Leukocyte activation and adhesion to vascular endothelial cells is a critical step in the inflammatory process. This
process is regulated by which of the following molecules?
a. The selectins
b. The b5 integrins
c. The immunoglobulin supergene family
d. Nitric oxide
e. IL-8
Answer: a, c, d, e

The temporal events that initiate and propagate neutrophil recruitment and inflammation include endothelial cell activation
and expression of endothelial-derived neutrophil adhesion molecules, neutrophil-endothelial cell adherence, and neutrophil
transendothelial migration via established neutrophil chemotactic gradients. There are three major families of adhesion
molecules which are expressed on the surface of leukocytes and endothelial cells and are important for leukocyte-
endothelial cell interactions. These include the immunoglobin supergene family (ICAM-1, VCAM-1, and PECAM-1), the
selectins (E-selectin, P-selectin and L-selectin), and the integrins. The leukocyte b2 integrin adhesion molecule family
consists of three members with heterodimeric glycoproteins displayed as a variable alpha and a constant beta chain. Nitric
oxide regulates the adhesion process both by direct influence on leukocyte binding as well as by regulation of regional
blood flow. IL-8 is one of the most potent mediators of chemotaxis in the C-X-C chemokine family. It serves an important
role in neutrophil recruitment and activation, and the continued propagation of the inflammatory response.
172 A 65-year old patient has colon carcinoma metastatic to the liver and lungs. He has had a weight loss of 10 kg.
Cytokine-dependent tumor cachexia is attributable to which of the following?
a. Increased glucose uptake and increased glycogen breakdown occur in this circumstance.
b. Suppressed activity of lipoprotein lipase results from TNFa
c. TNFa stimulates lipolysis
d. The differentiation process of pre-adipocytes is impaired
e. Partial reversal of differentiated adipocytes to pre-adipocyte morphology and gene expression occurs
Answer: a, b, c, d, e

Tumor cachexia appears to be mediated by TNFa. Lipopolysaccharide (LPS), as well as other cytokines, activate a variety
of inflammatory cells, most importantly macrophages, to produce TNFa. Both the chronic administration of TNFa to rats
and implantation of tumors secreting TNFa in mice induce a syndrome of cachexia. In vitro, higher TNFa concentrations
alter glucose metabolism in cultured myotubules by increasing glucose uptake and glycogen breakdown. It has also been
demonstrated that purified TNFa suppresses lipoprotein lipase activity and stimulates lipolysis in cultured adipocytes.
Further, TNFa not only inhibits the differentiation process of preadipocytes, but partially reverses differentiated adipocytes
to a preadipocyte morphology and pattern of gene expression. All of these metabolic effects at least partially explain the
chronic syndromes of anorexia, weight loss, and cachexia that are associated with both chronic infection and malignancy.

173 Which of the following statements regarding fibroblasts and their function in wound healing are true?
a. IL-1 has both inhibitory and promotional effects on fibroblast growth
b. TNFa stimulates fibroblast collagen synthesis
c. IL-1 and TNFa have opposite effects on the healing of bone
d. In human clinical trials, EGF (epithelial growth factor) has been demonstrated to accelerate epidermal regeneration
in cutaneous wounds
Answer: a, d

IL-1 appears to be important in the process of normal wound repair. IL-1 has been shown to stimulate skin fibroblast and
keratinocyte growth, as well as fibroblast collagen synthesis and keratinocyte chemotaxis. IL-1 also promotes increased
transcription of the matrix degradative enzymes collagenase and stromelysin. These are important and potent tissue
degrading proteinases. Other studies have demonstrated that IL-1 inhibits fibroblast growth and matrix synthesis, and
stimulates collagenase production. These actions are at least partly due to the ability of IL-1 to upregulate prostaglandin E2
production which results in the down regulation of matrix synthesis. IL-1 has both promoting and inhibiting effects on
fibroblast collagen synthesis, therefore, the overall activity in this area is somewhat unclear in comparison to other well-
defined fibroblast growth-promoting cytokines. TNFa inhibits fibroblast collagen synthesis, however it also has potent
mitogenic effects. The mitogenic response correlates well with an increased stimulation of tyrosine phosphorylation. Both
IL-1 and TNFa have similar effects upon bone. Both stimulate cartilage resorption, the release of proteoglycans from
cartilage by limited proteolytic degradation, and both inhibit proteoglycan synthesis. Recent studies have also demonstrated
that TNFa inhibits fracture healing in experimental animals. This is due to the inhibition of cartilage formation and new
bone synthesis, and the inhibition of mesenchymal cell differentiation into chondroblasts. The family of epithelial growth
factor (EGF)-like molecules induce mitogenesis and play a role in wound healing. In human clinical trials, EGF has been
demonstrated to accelerate epidermal regeneration in cutaneous wounds. In vitro data show that recombinant EGF enhances
keratinocyte migration. EGF is also a potent chemoattractant for granulation tissue fibroblasts.

174 Neutrophil chemotaxis is a fundamental aspect of inflammatory injury in conditions such as the Adult Respiratory
Distress Syndrome (ARDS). Neutrophil chemotaxis is directly attributable to which of the following molecules?
a. C5a
b. TNFa
c. LPS
d. IL-1
e. ENA-78 (Epithelial Neutrophil Activating Protein)
Answer: a, e

There is a large collection of peptide, polypeptide and lipid mediators which have chemotactic properties. Although TNF a,
IL-1 and LPS were initially reported to have direct neutrophil chemotactic activity, recent studies have demonstrated that
these molecules are not directly chemotactic for neutrophils. This finding suggests that cytokine networks may be operative
in vivo and depend on the initial expression of early response cytokines. This initial interaction is followed by the
generation of more distal inflammatory mediators that directly influence neutrophil chemotaxis and activation. There is a
particularly important group of novel chemotactic cytokines which share significant homology with the presence of four
conserved cysteine amino acid residues. These cytokines in their monomeric forms are all less than 10 kD, are
characteristically basic heparin-binding proteins, have specific neutrophil chemotactic activity and display four highly
conserved cysteine amino acid residues, with the first two cysteines separated by one non-conserved amino acid residue.
Because of their chemotactic properties and the presence of C-X-C cysteine motif, these have been designated the C-X-C
chemokine family. Twelve different chemokines have been identified in the last decade. These include IL-8, epithelial
neutrophil activating protein (ENA-78), and others. Among the other endogenous chemoattractants are several complement-
derived peptides. Perhaps, the most potent of these is the short-lived C5a peptide.

175 Which of the following statements regarding angiogenesis are true?


a. Angiogenesis is a seminal biologic event with clinical relevance limited to its effect upon tumor growth
b. C-X-C chemokines regulate angiogenesis
c. PF-4 has angiogenic properties
d. Sites of atherosclerosis demonstrate chronic angiogenic activity
Answer: b, d

An important component of tissue repair and wound healing is the process of angiogenesis. This normal, physiologic
process is a local, transient event which is regulated strictly. A biological imbalance in the production of angiogenic and
angiostatic factors contributes to the pathogenesis of several angiogenesis-dependent disorders. These include both
malignant and nonmalignant disorders such as rheumatoid arthritis, scleroderma, psoriasis, atherosclerosis, and idiopathic
pulmonary fibrosis. Persistent neovascularization in these benign disorders is a prerequisite for the perpetuation of
fibroproliferation. IL-8 and potentially other C-X-C chemokines are involved with the angiogenesis process. IL-8 is a potent
angiogenic factor. In contrast, another member of the C-X-C chemokine family, PF-4 has angiostatic properties. This
suggests that the C-X-C chemokines may function as either angiostatic or angiogenic factors, and the biologic balance that
is maintained between these factors may govern overall angiogenic potential in a variety of physiological and
pathophysiological states.

176 Which of the following statements regarding IL-1 are correct?


a. While IL-1 and TNFa share many biologic effects, IL-1 appears to be more potent
b. IL-1 expression is in part autoregulated
c. IL-1 inhibits prostaglandin production
d. The ability of IL-1 to upregulate endothelial cell-neutrophil adhesion molecules is relatively limited
Answer: b

IL-1 and TNFa share many biologic properties. In addition, each potentiates the effects of the other one when given
concurrently. Overall, IL-1 alone probably has weaker effects than TNFa with respect to the induction of shock; its role is
likely to be important with respect to its marked potentiating abilities as it relates to TNFa. IL-1 expression is regulated by a
host of factors including IL-2, granulocyte macrophage colony stimulating factor (GM-CSF), transforming growth factor b
(TGF-b), TNFa, all of the interferons, and IL-1 itself. Other endogenous stimuli for IL-1 production include antigen-
antibody complex, the Fc region of IgG, and C5a; other nonspecific exogenous stimuli include silica particles and UV
irradiation.
One of the key proinflammatory features of IL-1-induced inflammation is the stimulation of arachadonic acid metabolism.
IL-1 stimulates the release of pituitary stress hormones and increases the synthesis of collagenases, resulting in the
destruction of cartilage, bone and other collagen-rich structures. IL-1 stimulates prostaglandin production.
One of the most important properties of IL-1 involves its interaction with the vascular endothelium. This includes the
adherence of neutrophils, basophils, eosinophils, monocytes, and lymphocytes to the vascular endothelium via interaction
between adhesion molecules on leukocytes and adhesion-receptor complex on the endothelial cells. By inducing the
expression of ICAM-1, E-selectin, and VCAM-1 on endothelial cells, IL-1 provides a key step in the extravasation of
leukocytes to sites of local inflammation and injury.

177 Which of the following statements regarding TNFa are true?


a. TNFa has a marked procoagulant effect
b. Passive immunization of patients with neutralizing antibodies to TNFa improves survival from multi-organ system
failure
c. TNFa upregulates E-selectin expression
d. The most potent known stimulus for TNFa production and release is IL-1
Answer: a, c

TNFa has a marked procoagulant effect on endothelial cells, precipitating intravascular thrombosis. TNFa causes
endothelial cells to release procoagulant activity (tissue factor), platelet activating factor, and von Willebrand factor, all of
which favor thrombosis. TNFa also down regulates the expression of thrombomodulin, which has the potential to block the
assembly of protein C and protein S complexes, further decreasing the anticoagulant properties of the endothelial cell
surfaces.
Administration of recombinant TNFa to experimental animals produces a clinical syndrome similar to that seen in septic
shock and multi-organ system failure in humans. Passive immunization of animals with neutralizing antibodies against
TNFa, prior to the infusion of TNFa or endotoxin, has been shown to prevent the development of this syndrome. No such
evidence exists in human patients.
TNFa upregulates a variety of leukocytic adhesion molecules including ICAM-1, PECAM-1, VCAM-1, E-selectin and P-
selectin. A variety of exogenous and endogenous factors (including IL-1) are capable of inducing cells to produce TNFa,
however the most potent stimulus for TNFa production and release is endotoxin.

178 Which of the following belong to the family of C-X-C chemokines?


a. IL-8
b. IL-10
c. Growth Related Oncogene-a
d. Leukotreine B4
e. b Thromboglobulin
Answer: a, c, e

A particularly important group of novel chemotactic cytokines has been elucidated over the last decade. Twelve are known
and are listed below.
C-X-C Chemokines
Connective Tissue Activating Protein III
b-Thromboglobulin
Growth Related Oncogene-a
Growth Related Oncogene-b
Related Oncogene-g
Interleukin-8
Epithelial Neutrophil Activating Protein
Granulocyte Chemotactic Protein-2
Platelet Factor-4
g-Interferon-inducible Protein
Monokine-induced by g-Interferon
Each has unique biologic functions. There appear to be important in vivo cytokine networks involving these molecules
which regulate chemotaxis, and other fundamental aspects of inflammation.

179 Which of the following statements regarding the complement system are true?
a. Complement activation yields products which are directly cytotoxic as well as products which act indirectly via
activated leukocytes
b. Complement products referred to as anaphylatoxins include C1, C3a, C4a, and C5a
c. The principal role of C5a is in bacterial opsonization
d. The alternative and classical pathways converge proximal to generating the membrane attack complex (C5b-9)
Answer: a, d

The complement system is composed of two different but linked sequences, the classic and alternative pathways. The
pathways involve serum proteins that act to amplify the inflammatory-immune response as well as to directly mediate tissue
injury. Complement activation by either pathway has been associated with a cascade of events, some of which are mediated
directly at a physiologic level by complement products and some of which occur indirectly via activated leukocytes. The
direct physiologic effects mediated by C3a and C5a, and to a lesser extent C4a, include increased vascular permeability and
contraction of smooth muscle. These are key elements of anaphylaxis. C1 is not an anaphylatoxin as it is the initial
complement component which binds to antigen-antibody complex to initiate classical pathway activation. C5a acts
principally to alter the behavioral characteristics of leukocytes. Effects include enhanced adherence, enhanced chemotactic
activity, release of proteinases, and production of toxic metabolites of oxygen. C3, on the other hand, plays a key role in
bacterial opsonization, resulting in enhanced phagocytosis of invading microorganisms. The alternative and classical
complement pathways converge at the C5 level proximal to generating the membrane attack complex (C5b-9) (Figure 6-3).

180 Which of the following statements regarding neutrophils are true?


a. The neutrophil undergoes final maturation after release into the circulation
b. Patients with chronic granulomatous disease have a defective neutrophil H-oxidase system
c. Neutrophil killing of bacteria is achieved by oxidants, proteinases and cationic proteins
d. The normal human neutrophil circulates in the blood for 7–10 days
Answer: b, c

The neutrophil is a migratory phagocytic cell that defends the host against bacteria and eliminates necrotic tissue. The
neutrophil matures in the bone marrow and is released into the circulation as a fully differentiated cell. It is loaded with
granules containing a variety of proteinases, hydrolases, antimicrobial agents and cationic proteins. The cell phagocytoses
material and the granules fuse with the phagocytic vacuoles to degrade the foreign material. When the cells are challenged
with a large amount of material, the granule contents may be released into the extracellular space where damage to
surrounding tissue occurs. The neutrophil normally circulates in the human bloodstream for 7 to 10 hours. Thereafter,
neutrophils are thought to exist for 1 to 2 days in the tissues before being cleared from the system. Granule constituents are
formed during differentiation and replenishment of spent granules does not occur once the cells are in the circulation.
Hence, the neutrophil is a fully differentiated end-cell poised to respond rapidly to stimuli, but it is rapidly spent in the
process. Neutrophils have a NADPH-oxidase enzyme system on the plasma membrane which can be activated to produce
toxic oxygen species including the superoxide anion (02–). Patients with chronic granulomatous disease (CGD) have a
defective NADPH-oxidase system in their neutrophils, and are thus unable to generate 02–. Although neutrophils from
patients with CGD are able to phagocytose bacteria, they are unable to kill the intracellular microbes and chronic,
unresolved infections result.

181 Which of the following statements regarding the alternative complement pathway are true?
a. C1, C4 and C2 are involved
b. NH3 apparently activates complement via this pathway
c. Factors B and D are involved
d. Endotoxin activates complement via the alternative pathway
Answer: b, c, d

The alternative pathway differs from the classic pathway in that the first steps involving C1, C4 and C2 are bypassed. (See
Figure 6-3 previously reproduced.) This pathway can be directly activated by agents other than antigen–antibody complex
(e.g., complex polysaccharides like endotoxin and zymosan). Other serum protein factors (e.g., factors B and D) are
involved in the activation sequence. Ammonia can attack the thiol-ester, producing amidated C3 and activate the alternative
pathway. This leads to membrane attack complex formation (C5b-9) and activation of a number of phagocytic cell functions
including toxic oxidant production. This phenomenon may have relevance to several in vivo disease states. In animal
models of renal failure, elevated levels of renal vein NH3 have been correlated with impaired renal function and the
presence of complement components at the sites of renal injury.

182 Platelet activating factor is:


a. Generated by the action of phospholipase A2 on membrane phospholipids
b. Antiinflammatory in most of its actions
c. Synthesized by endothelial and other cells
d. Exerts a variety of biologic effects which are platelet-independent
Answer: a, c, d

Like the eicosanoids, platelet-activating factor (PAF) is not stored in cells but is rapidly produced during inflammation. PAF
exerts a variety of biologic effects that are platelet-independent. The synthesis of PAF is initiated by the activation of
phospholipase A2. Activation of phospholipase A2 releases arachidonic acid in addition to lyso-PAF. Hence, PAF synthesis
and eicosanoid production are coordinately regulated. PAF is synthesized on activation of a variety of inflammatory cells
including platelets, neutrophils, basophils, mast cells, mononuclear phagocytes, eosinophils and vascular endothelium. PAF
is a stimulatory agonist for many inflammatory cells, as well as for smooth muscle cells, vascular endothelium and others.
PAF enhances the ability of neutrophils to respond to challenge with N-formylpeptides and LTB4. There is considerable
overlap and redundancy in the effects produced by PAF and eicosanoids.

183 Platelets have a wide array of functions in inflammation. Which of the following are among these?
a. Synthesis and release of vasoactive eicosanoids
b. Release of chemotactic factors
c. Adherence to and coating of bacterial and tumor cells
d. Increase of vascular permeability
e. Phagocytosis of bacteria
Answer: a, b, c, d
Platelets are anucleated cells derived from megakaryocytes in the bone marrow. Their central role in hemostasis is well
known. Platelets possess a wide array of functions in inflammation, including the following:
Synthesis and release of vasoactive eicosanoids
Release of chemotactic factors
Interaction with other inflammatory cells
Interaction with endothelial cells
Adherence to and coating of bacterial and tumor cells
Platelets are not capable of phagocytosis.
Few of the factors released or the functions carried by platelets during inflammation are unique to this cell type. Other
inflammatory cells often have the same or similar capabilities. Indeed, some platelet functions may reflect vestigial
functions inherited from a primitive precursor inflammatory cell. Platelets serve primarily as an amplifier or modulator of
the inflammatory response.

184 Eicosanoids mediate inflammation in a variety of ways. Of the following statements, which are true with regard to
this?
a. Eicosanoids are stored in cytoplasmic granules for release after receptor mediated signaling
b. Eicosanoids include prostaglandins, thromboxanes, leukotrienes and lipoxins
c. Eicosanoids generally have a plasma half-life measured in hours
d. Physiologic responses to eicosanoids include vasodilatation, vasoconstriction, increased vascular permeability and
both chemotaxis and chemoattractant inhibition
Answer: b, d

The eicosanoids are derived from arachidonic acid (eicosatetraenoic acid) and consist of prostaglandins, thromboxanes,
leukotrienes and lipoxins. The eicosanoids are not stored in cells but are rapidly synthesized by cells in response to a variety
of stimuli. They have potent effects on vascular and bronchial smooth muscle including vasodilatation, vasoconstriction,
bronchodilation and bronchoconstriction. In addition, they directly regulate vascular permeability. LTB4 is a potent,
neutrophil chemoattractant whereas lipoxin A4 inhibits other chemoattractants. It appears that eicosanoids are important
regulators of the endogenous inflammatory response. The rapid destruction of eicosanoids in the circulation limits their role
primarily to that of mediators of local inflammatory changes. The local effects can be substantial. In general, the
eicosanoids are rapidly metabolized or are so chemically unstable that they primarily exert their effects near the site of
synthesis. Arachidonic acid does not exist in cells but is esterified to membrane phospholipids. Thus, the first step in the
production of eicosanoids is phospholipase action, which liberates arachidonic acid. (Figure 6-6)

185 Which of the following statements are true?


a. Eosinophils are the major, if not sole, source of histamine in the blood
b. Basophils are effector cells in allergic reactions by virtue of IgE receptors
c. Mast cells are the major source of tissue histamine except in the stomach and central nervous system
d. Mononuclear phagocytes release a variety of proinflammatory cytokines and growth factors
Answer: b, c, d

Eosinophils constitute 1% to 3% of the leukocyte population of the bloodstream. They also reside in tissues and they exhibit
phagocytic capabilities. They are less effective as bactericidal cells than neutrophils, but play a major role in defense against
parasites. Eosinophils are primary effectors in allergic reactions by virtue of IgE receptors (which are not found on
neutrophils).
Basophils are fully differentiated cells released into the circulation from bone marrow. Basophils are the major, if not sole,
source of histamine in the blood. Histamine is a vasoactive amine and the major mediator of the IgE-mediated immediate
hypersensitivity response. Histamine release from basophils is induced by complement products as well as by IgE receptors.
Mast cells are formed from bone marrow precursors that differentiate and proliferate in connective tissue. Mast cell granules
contain histamine and proteoglycans. They represent the major source of histamine in most tissues except the stomach and
central nervous system.
The monocyte–macrophage system consists of phagocytic cells scattered throughout the body. During acute inflammation,
monocytes respond to chemoattractants released and are recruited to the site of inflammation. Mononuclear phagocytes
respond to inflammatory stimuli by releasing M-CSF, GM-CSF, IL-1, and TNF, in addition to a variety of growth factors.
These factors increase the production of mononuclear phagocytes and several of these factors enhance the ability of effector
cells to respond to chemotactic stimuli released at the site of injury. Thus, the mononuclear phagocytes are important in
initiating and augmenting the cycle of events that result in recruitment and activation of inflammatory cells at sites of
inflammation.

186 Cellular injury from oxidants may be manifest by which of the following?
a. Cell membrane lipid peroxidation
b. DNA strand breaks
c. Cytoskeletal disassembly
d. ATP depletion
Answer: a, b, c, d

Free oxygen radicals are chemical species that are intermediates in the normal process of cellular respiration. Oxidants that
are free radicals have been implicated as initiators of reactions which lead to a variety of cellular injuries. Oxidants are
derived from several sources, notably phagocytes. Among the effects of oxygen free radicals are membrane lipid
peroxidation, DNA strand breaks, cytoskeletal disassembly and inhibition of glucose metabolism leading to decreased
cellular ATP concentrations. (Figure 6-16)

187 Which of the following acute-phase protein levels are increased in human plasma following acute inflammation?
a. C-reactive protein
b. Serum amyloid
c. a -Proteinase inhibitor
d. Fibrinogen
e. Albumin
Answer: a, b, c, d

The acute-phase response is a series of homeostatic responses of the organism to tissue injury in infection and inflammation.
After an inflammatory stimulus occurs, a number of events occur within hours. These reflect altered set-points for various
physiologic parameters including thermoregulation (fever), nitrogen balance (negative), and levels of various plasma
proteins (increased or decreased). The erythrocyte sedimentation rate, which increases with inflammatory states, is an
example of this phenomenon. The increased sedimentation rate is due to increased levels of fibrinogen and some of the
other acute-phase reactants in plasma. Some proteins show a large increase (about 1000-fold), some a 4-to 5-fold increase,
and others about a 50% increase over resting nonstressed levels.
Note that albumin is an acute-phase reactant. Levels of albumin drop after an inflammatory stimulus, usually 30% to 50% of
the level before injury. The reason for the decrease in production is poorly understood.

188 Which of the following statements regarding endothelial cells in acute inflammation are true?
a. Endothelial cells are characterized by phenotypic homogeneity
b. Specific patterns of receptor expression regulate leukocyte adherence
c. Endothelial cell nitric oxide generation regulates regional blood flow and leukocyte adhesion
d. Endothelial cells may be capable of phagocytosis
Answer: b, c, d

Endothelial cells are increasingly recognized to be phenotypically heterogeneous. Specific receptor molecules are expressed
at various sites where they help to direct lymphocytes and other leukocytes to their appropriate target organ. In the high
endothelial venues, these receptor molecules are known as vascular addressing. Endothelial cells play a major role in
regulating vascular tone. This is the result of angiotensin-converting enzyme on the cell surface as well as the production of
both endothelia (a potent vasoconstrictor) and nitric oxide (a potent vasodilator). Both play important physiologic roles in
determining the distribution of blood flow. In addition, recent evidence suggests that NO may have direct effects upon the
expression of a variety of leukocyte adhesion molecules. Under unusual circumstances, endothelial cells can exhibit
macrophage-like properties in that they can act as antigen-presenting cells and also phagocytose particles. They may also be
a significant source of oxidants in inflammatory reactions after ischemic injury. Endothelial cells are not passive participants
in inflammatory processes; rather, they possess the ability to direct and focus many aspects of an inflammatory event.

189 The first line of host defense is the barrier presented to the external environment. Which of the following
statement(s) is/are true concerning host barriers?
a. Sebaceous glands secrete chemical compounds that maintain a relatively high pH, providing effective bacterial
stasis
b. Within the respiratory tract, ciliary function serves to extrude microorganisms trapped within the mucus secretion
layer
c. The low pH within the stomach markedly decreases bacterial content of the upper gastrointestinal tract
d. Gut peristalsis serves to prevent microbial adherence and invasion
Answer: b, c, d
The skin, mucus membranes, and epithelial layers of various organs of the body constitute effective physical barriers against
microbial invasion. In certain portions of the body, these barriers have developed ancillary adaptations to increase the
effectiveness of the barrier functions. Skin structures such as sebaceous glands secrete chemical compounds that serve to
maintain a relatively low pH, providing effective bacterial stasis. Mucus secretion by specialized glands within the bronchi
and gut provide a mucus layer that represents a physical and chemical barrier to microbial invasion. Within the respiratory
tract, ciliary function serves to extrude microorganisms trapped within this mucus layer. In the alimentary track, the very
low pH within the stomach and gut peristalsis both serve to prevent microbial adherence and invasion.

190 Which of the following statement(s) is/are true concerning the antibody response to an invading antigen?
a. All antibodies are composed of one type of heavy and one type of light protein chain
b. The carboxyl terminus of the heavy chain is the antigen binding site
c. Antibody of the immunoglobulin G class is the initial antibody produced in response to an antigenic stimulus
d. Immunoglobulins A, D, and E play an active role in the circulating humoral response
Answer: a

Humoral defenses consist of antibody (immunoglobulin; Ig) and complement. All Ig classes (IgM, IgG, IgA, IgE, IgD) and
IgG subclasses are composed of one type (M, G, A, E, D) of heavy and one type (K and g ) of light protein chains that
consist of several domains both structurally and functionally. Each Ig molecule contains one or more units that consist of
two heavy and two light chains linked by disulfide bonds. The amino terminus of both heavy and light chains contain
several hypervariable regions that fold in three dimensions to produce the antigen-binding site. The carboxyl terminus of the
heavy chains contain regions that activate complement and bind Fc receptors, by which direct adherence to
polymorphonuclear leukocytes and macrophages take place after antigen binding occurs.
Initially, antibody of the IgM class is produced in response to an antigenic stimulus. A second exposure to the same antigen,
or a cross-reactive antigen, leads to the so-called second set response, in which antibody of the IgG class with two binding
sites is produced more rapidly and in larger quantity compared to the initial IgM primary response. Immunoglobulin of the
IgA class is secreted by gut-associated lymphoid tissue and is combined with secretory components of protein to form a
dimer termed secretory IgA. This antibody acts at a variety of epithelial sites to prevent microbial adherence and invasion.
IgD and IgE exist in smaller amounts in the circulation and do not appear to play a major role as host defense components.

191 Increasing evidence has implicated gram-negative bacterial lipopolysaccharide (LPS endotoxin) as the portion of
the gram-negative bacterial cell membrane responsible for many, if not all the toxic effects that occur during gram-negative
bacterial sepsis. The following statement(s) is/are true concerning LPS and the host response.
a. The LPS molecule can in itself cause physiologic responses similar to that seen during gram-negative bacterial
sepsis
b. LPS triggers host macrophages to release a variety of cytokines including TNF-a, IL-1a, and IL-1b, IL-6, and IFN-
a
c. Excessive cytokine production is not associated with detrimental consequences
d. TNFa and IL-1b appear to be the primary mediators within the host, exerting deleterious effects on the host when
excessive amounts reach the systemic circulation
Answer: a, b, d

The LPS molecule exerts diverse effects on the mammalian host. Immunologic responses to LPS include nonspecific
polyclonal B-cell proliferation, macrophage activation and cytokine secretion, tolerance to subsequent LPS or bacterial
challenge, and production of antibody directed against various portions of the LPS molecule after repeated challenge.
Physiologic responses similar to those seen during gram-negative bacterial sepsis occur during LPS administration alone
and include hypotension, hypoxemia, acidosis, bacterial translocation across the gut, complement and coagulation cascade
activation, white blood cell and platelet margination, and death. Indirect effects result from LPS-triggering of host
macrophages. Activated macrophages secrete a wide array of cytokines that include TNF-a, IL-1a, and IL-1b, IL-6 and
interferon-a (IFNa). Excessive secretion of cytokines produce substantial systemic effects in the mammalian host. TNFa and
IL-1b appear to be the primary mediators within the local host milieu, exerting deleterious effects on the host only after
large amounts are secreted and reach the systemic circulation.

192 Which of the following statement(s) concerning the gut microflora is/are correct?
a. Gut microflora evolves constantly throughout development
b. The gut microflora can contribute to the physical and chemical barriers at the mucus membrane level
c. Most of the microorganisms found in the oropharynx eventually pass into the intestine
d. In the colon, anaerobic organisms outnumber aerobic organisms in a ratio in excess of 100:1
Answer: b, d

The composition of the gut microflora is established in neonates after ingestion of microbes that are acquired during
contamination from the birth canal and during initial feeding, and remain relatively constant thereafter. Although this flora
acts to promote development of the immune system, the specific interactions that produce this effect have not been fully
elucidated. The microflora also contributes to physical and chemical barriers at the mucus membrane level, in that many
autochthonous microbes possess adhesion proteins by which they can bind to certain areas of the mucosal cell or to specific
types of bacteria, occupying potential binding sites for pathogenic organisms and producing a substantial physical
mucobacterial layer. The oropharynx contains a number of aerobic and anaerobic microorganisms, however, these microbial
inhabitants do not usually pass into the intestine, because the stomach itself represents a significant barrier to invading
microorganisms by virtue of its low pH, which kills most microbes. The upper small intestine contains few organisms,
mainly gram-positive aerobes and lactobacilli. Conversely, the lower small intestine contains a large number of aerobes and
anaerobic forms, especially in patients in whom the ileocecal valve allows free backwash of cecal contents into the terminal
ileum. Within the colon, a wide diversity and a large number of facultative and strict anaerobic isolates are present. Only a
small number of aerobes are present, these microbes being outnumbered 100–300 to 1 by anaerobes.

193 The use of antibiotics can be based on either the clinical course of a patient without the benefit of well-defined
microbiologic data (empiric therapy), or targeted at specific identified pathogens once sensitivity reports are available
(directed therapy). The following statement(s) is/are true concerning these therapies.
a. The issue of toxic side effects of antibiotics is only important in dealing with emperic therapy
b. Single agent therapy is generally inferior to specific multi-drug therapy (aminoglycoside plus an antianaerobic
agent) for the treatment of secondary bacterial peritonitis due to appendicitis, diverticulitis, penetrating gastrointestinal
injury, or anastomotic leak
c. With the empiric use of antibiotics, a diligent search for the septic source should be undertaken and continued until
identified
d. In clinical situations in which polymicrobial infection is identified, specifically-directed treatment for the
predominant organism is satisfactory
Answer: c

The use of empiric therapy without the benefit of well-defined microbiologic data is appropriate when there is sufficient
clinical evidence to support the diagnosis such that it would be imprudent to withhold antimicrobial therapy. In this setting,
however, a diligent search for the septic focus source should be undertaken and continued (cultures, radiographic
procedures, etc.), and initial limits should be placed in the course of empiric therapy with continued reevaluation based on
the clinical course of the patient. The choice of antibiotic agents should be based on the clinical situation and known activity
patterns within the given institution. Single broad-spectrum agents, although suffering slightly from a lack of individual
pathogen specificity, are useful in this setting in that they provide a broad coverage against several groups of pathogens and
may avoid some of the toxic effects with specific combined modality regimens. Similarly, for directed therapy, single-agent
therapy has been demonstrated to be equivalent to combined therapy and should be chosen in an attempt to select agents
with appropriate sensitivities which retain suitable clinical efficacy but exhibit minimal toxicity. After review of cultural
reports, many patients have demonstrated polymicrobial infection. Because experimental clinical evidence supports the
concept of aerobic-anaerobic synergy, therapy should be directed against all potential components of the infection if the
body site is such that these microorganisms may be present.

194 The following statement(s) is/are true concerning newer detection methods of systemic infection.
a. Enzyme-linked immunosorbent assay (ELISA) is a rapid immunologic assay used for both antigen and antibody
detection
b. Southern, northern, and western immunoblot techniques are used to detect DNA, RNA, or proteins, respectively
c. Polymerase chain reaction (PCR) is a sensitive assay used to detect small amounts of microbial DNA, thus
detecting infection at its early stages
d. Infectious agents currently detected using advanced molecular techniques include cytomegalovirus (CMV) and
human immunodeficiency virus (HIV)
Answer: a, b, c, d

Although the classic detection of infection based on clinical signs of infection and bacterial culture remain the most
common clinical tools, increasing reliance has been placed on assays that do not employ cultural data. Specifically, the
antibody and cytokine host responses are being intensely examined and extremely sensitive amplified assays that rely on
antigen, antibody or microbial DNA detection are employed in the clinical setting. Enzyme-linked immunosorbent assay
(ELISA) is a rapid, antigen-based, immunologic assay that can be used for both antigen and antibody detection, for
determination of antibody titer, as well as for screening for monoclonal antibody production. Transblot techniques are being
used increasingly in the clinical setting. These include southern, northern, and western immunotransblot techniques used to
detect DNA, RNA, or proteins, respectively. The polymerase chain reaction (PCR) is being used in some centers as a
sensitive assay to detect small amounts of microbial DNA. This technique involves extraction of the DNA from the test
sample with in vitro amplification through repeated nucleic acid denaturing and polymerization so that the gene copy
number increases exponentially. This marked amplification of the gene copy number results in extremely sensitive tests
which can detect infection at its early stages.
Clinically, these detection methods are being used to detect a wide variety of infectious agents including CMV and HIV.
Furthermore, preliminary investigations into possible detection of fungal pathogens are underway.

195 Cytokines are low-molecular-weight polypeptides exerting a wide variety of biologic effects at both local and
systemic levels. Which of the following statement(s) is/are true concerning the production and actions of cytokines?
a. Cytokines are produced solely by macrophages
b. Cytokines act only on other cells within the same local environment
c. Cytokines may have both protective and deleterious effects on the host
d. Each specific cytokine is produced by a single cell type
Answer: c

Macrophages, endothelial cells, lymphocytes, and other cells secrete a large number of different compounds, termed
cytokines, that are most probably evolved for the purpose of local intercellular and intracellular signaling. Cytokines
frequently are secreted after initial lymphocyte or macrophage activation, and may act on the secreting cell itself (autocrine
activation) or on other cells within the same local environment (paracrine activation) to cause increased secretion of the
same cytokine or other cytokines, respectively. Some cytokines are produced by several cell types, and most produce a wide
array of effects. The duality of the effects of the cytokine component of host defenses, exerting both salutatory and
deleterious effects on the host, has become increasingly evident.

196 The following statement(s) is/are true concerning cellular defense mechanisms.
a. Macrophages function solely as antigen processing cells in the initial reaction to exposure to an antigen
b. Macrophages may become activated and secrete cytokines
c. Macrophages serve as phagocytic cells in the tissues but not within the bloodstream
d. Polymorphonucleocytes (PMNS) are normally present in only small numbers within the tissue and enter an area of
infection through diapedesis
Answer: b, d

A wide variety of cell types serve to provide host defense at several levels. Macrophages act as the initial antigen processing
cell that serves to present antigen to help T cells, thus initiating the immune response. Macrophages, however, are
pluripotent cells that, in the process of engulfing and processing antigen, may become activated. Activated macrophages
secrete a variety of cytokines. Macrophages also act as phagocytic cells in the tissues and within the bloodstream, and
because of their resident nature in many tissues, also represent the first line of host defenses in many areas of the body, even
before activation. PMNS are present within the bloodstream, but only in small numbers within the tissue, and enter an area
of infection through diapedesis after chemotactic stimuli are excluded by macrophages, bacterial breakdown products, and
complement activation.

197 A diabetic develops a severe perineal infection with skin necrosis, subcutaneous crepitance, and drainage of a thin,
watery, grayish and foul-smelling fluid. Management should consist of:
a. Gram stain of the fluid, which will likely demonstrate multiple bacteria including predominantly gram-positive
rods
b. A CT scan is indicated in a stable patient to define the extent of the disease
c. Broad spectrum antibiotics followed with prompt extensive debridement is indicated
d. A safe guideline is to resect infected necrotic tissue so that a several centimeter margin of grossly normal, healthy
tissue can be achieved
e. A colostomy is of little benefit in this situation
Answer: a, b, c, d
The presence of severe perineal infection (referred to as Fournier gangrene when this process involves the perineum and
scrotum in males) is associated with a continued high mortality despite aggressive and appropriate therapy. The clinical
description provided would suggest an underlying soft tissue necrosis. In a stable patient radiologic studies including a CT
scan to define the extent of the disease and the presence of pelvic infection is indicated. Gram stain will likely show
evidence of polymicrobial organisms but the presence of Clostridia marked by gram-positive rods would suggest
involvement with this organism. Prompt, aggressive and extensive debridement to remove all devitalized and affected tissue
and the addition of broad spectrum antibiotics, fluid resuscitation, hemodynamic monitoring, and nutritional support would
appear to afford the patient the best chance of survival. The clearest guidelines to determine the limits of resection involve
removal of clearly infected, necrotic tissue so that margins several centimeters into grossly normal, healthy tissue are
achieved. Because the entire perineal region and buttocks are frequently involved in these patients, performance of a fecal
stream diversion by means of a colostomy often provides improved wound care and patient management, although it is not
invariably a positive outcome.

198 The use of prophylactic antibiotics has become commonplace. Which of the following statement(s) is/are true
concerning the prophylactic use of antibiotics?
a. The appropriate use of prophylactic antibiotics must include the initiation of the agent prior to the surgical
procedure
b. Continuing the antibiotic into the postoperative period has led to improved results in antibiotic prophylaxis
c. The prophylactic administration of broad-spectrum agents (third-generation cephalosporins) has been shown to be
particularly advantageous
d. The topical use of antimicrobial agents is of no advantage in the prophylactic setting
Answer: a

Intravenous administration of an antibiotic is clearly indicated for patients undergoing clean contaminated operations. These
antibiotics should be administered prior to surgery to obtain adequate tissue levels at the time of potential contamination.
However, there has been no added benefit demonstrated for the postoperative use of antibiotics with regard to prophylaxis.
The choice of antibiotic is a complex issue which remains unresolved largely because both superficial and deep wound
infections can occur as a result of either or both skin (superficial wound) flora (e.g., Staphylococcus aureus) and body site
(deep wound) infection. For this reason, the administration of agents which possess activity directed against these expected
agents is reasonable. Although administration of a first-generation cephalosporin is acceptable, second-generation
cephalosporins or extended-spectrum penicillins with gram-positive and gram-negative activity and biliary tract excretion
may be more suitable for patients undergoing gastrointestinal or biliary tract procedures. Similarly, the use of agents with
additional anaerobic activity for patients undergoing gastrointestinal procedures involving the small bowel or colon should
be considered. The administration of broad-spectrum agents such as third-generation cephalosporins for prophylaxis does
not seem to provide additional benefit in comparison to the above-mentioned type antibiotics and may foster the
development of resistant organisms within a given institution or superinfection within a given patient. There is evidence that
in some cases the topical use of antimicrobial agents is equivalent to the administration of intravenous antimicrobial agent
antibiotics.

199 If a necrotizing soft tissue infection is considered, therapy mandates:


a. Empiric administration of antibiotics active against gram-positive, gram-negative, and anaerobic bacteria
b. Due to usually resistant species, penicillin is not indicated
c. Immediate operative intervention and aggressive resection of all involved tissues is mandatory
d. The use of hyperbaric oxygen has been demonstrated to be clearly advantageous
Answer: a, c

Identification of a necrotizing, soft tissue infection mandates immediate operative intervention with aggressive resection of
all involved tissues and empiric administration of antibiotics active against gram-positive, gram-negative, and anaerobic
bacteria. In most cases, this involves the use of several antimicrobial antibiotics in combination. Because of concern in all
cases for the presence of Clostridia infection, high doses of aqueous penicillin G are administered. Gram-positive organisms
are treated with vancomycin or a semisynthetic penicillin and gram-negative organisms are treated with an aminoglycoside
or a monobactam. Anaerobic coverage is typically achieved by use of metronidazole of clindamycin. The use of hyperbaric
oxygen therapy is controversial and unfortunately due to the rarity of the disease, prospective randomized data is not
available so that the literature remains without controlled trials demonstrating any additional benefits derived from
hyperbaric oxygen therapy.
200 Wounds are classified according to the likelihood of bacterial contamination. Which of the following statement(s)
is/are true concerning wound classifications?
a. A clean-contaminated wound would be that associated with an elective colon resection with adequate mechanical
and antibiotic bowel preparation
b. A contaminated wound would include a resection of obstructed bowel with gross spillage of intestinal contents
c. In a clean wound, no viscus is entered
d. Antibiotic prophylaxis should be administered for all clean-contaminated and contaminated wounds and selectively
in patients involving a clean wound
Answer: a, b, c, d

Wounds are classified under three classes according to the likelihood of bacterial contamination: 1) clean (no viscus is
entered; e.g., herniorrhaphy); 2) clean-contaminated (minimal contamination; e.g., elective colon resection with adequate
mechanical and antibiotic bowel preparation, and 3) contaminated (heavily contaminated surgery; e.g., resection of
unprepared, obstructed bowel with gross spillage of intestinal contents or stool, drainage of abscesses, debridement of
traumatic neglected wounds). Antibiotic prophylaxis generally should be administered for class 2 and 3 types of wounds,
but patients undergoing clean surgery do not always require antimicrobial antibiotic prophylaxis. An exception to this tenet
involves cases in which a prosthetic material may be used (artificial joint, heart valve, tissue patch).

201 The following statement(s) is/are true concerning HIV infection.


a. Initial screening with ELISA is highly sensitive but can be associated with a false positive rate of 25%
b. Treatment with azidothymidine (AZD) appears to prolong survival when administered early in the disease
c. Predisposition to infection in HIV infection is primarily due to reduction in the number of helper T cells
d. Common infections in patients with AIDS are Pneumocystis, carinii pneumonia, CMV pneumonitis, Cryptococcus
meningitis, and disseminated infection due to atypical mycobacteria
Answer: b, c, d

Acquired immunodeficiency syndrome (AIDS) is a syndrome caused by the human retrovirus (HIV-1) that infects T
lymphocytes and causes severe immunosuppression. Individuals who become infected with HIV are prone to a variety of
infections and different types of malignancy. A spectrum exists in which patients regress from asymptomatic infection, to
development of AIDS-related complex (ARC) of diseases to AIDS itself. Common infections occurring in patients with
AIDS are Pneumocystis carinii pneumonia; CMV pneumonitis; gastritis, hepatitis and meningitis due to Cryptococcus
neoformans; and pneumonia and disseminated infection due to atypical mycobacteria. Predisposition to these infections is
due, in part, to the lymphotrophic nature of HIV, which markedly reduces the number of helper T cells as well as the
absolute number of T cells.
HIV detection typically consists of initial ELISA screening, but this test has about a 1–3% false-positive rate, thus
mandating all positive tests be confirmed by the western immunoblot analysis.
Treatment of ARC and AIDS consists of aggressive antiinfective therapy once a specific infection occurs and the use of
AZT. AZT has been shown to prolong survival when administered early in the course of disease and is considered routine
therapy.

202 The following statement(s) is/are true concerning initial microbiologic diagnostic techniques.
a. Appropriate expeditious transport of specimens to the microbiology laboratory is essential for obtaining accurate
clinical information
b. The use of potassium hydroxide in preparing a specimen slide for light microscopy will be useful in identification
of anaerobic bacteria
c. Antibiotic sensitivity is determined by exposing the specific microorganism to varying amounts of antibiotic with
the concentration of the antibiotic inhibiting growth referred to as the MIC (minimal inhibitory concentration)
d. Serum levels of antimicrobial agents should achieve in excess of a 4-to 8-fold increase over the MIC to be
considered clinically efficacious.
Answer: a, c, d

Because most surgical infections are polymicrobial, specimens should be cultured for aerobic and anaerobic bacteria, as
well as fungi. Although aerobic and aerotolerant microorganisms often do not require special transport media, a delay in
specimen processing may markedly reduce the yield, and anaerobic transport media have been demonstrated to markedly
increase the cultural yield of this type of organism. The initial piece of information gained concerning potential infection
may come from simple staining of a specimen. Gram stain, which will identify the staining characteristics of the organisms,
as well as their number should be performed on all specimens. Potassium hydroxide is useful in that it will lyse bacteria and
other cellular elements within a preparation and allow observation of yeast or mycelial elements.
Initial culture results may solely indicate that microorganisms are growing and full characterization may take two to three
days. Once a specific microorganism is identified, a sample is inoculated during the log phase into broth containing varying
amounts of an antibiotic. After an 18-to 24-hour period, the tube or well that exhibits no visible growth is then noted, and
the reciprocal of this dilution is termed the minimal inhibitory concentration (MIC). This value may be compared to either
measured or known achievable serum levels for a particular antibiotic. In general, antimicrobial agents that achieve in
excess of a 4-to 8-fold increase over MIC during the peak serum level have been demonstrated to be clinically efficacious.

203 The complement system consists of a series of serum proteins that exist in a quiescent or very low-level state of
activation in the uninfected host. Which of the following statement(s) is/are true concerning complement activation?
a. The alternate (properdin) pathway of complement activation can occur directly through contact with fungal or
bacterial cell wall compounds
b. Complement component fragments may serve to decrease vascular permeability
c. Excessive complement activation can produce deleterious effects
d. Fragments of certain complement components serve as chemoattractants to additional cellular components of the
host defense mechanism
Answer: a, c, d

Complement activation can occur through either classic or alternate (properdin) pathways, both of which eventuate in
deposition of terminal complement pathway components on the antigenic cell surface. The classic pathway of complement
activation usually begins with immunoglobulin G-binding which has also bound the antigen. The alternate pathway
activation occurs in response to activation of direct binding of the antigen or directly through contact with fungal and
bacterial cell wall compounds such as zymosan and gram-negative bacterial lipopolysaccharide (LPS endotoxin). Several
complement components represent important host defenses acting to recruit or augment cellular host defenses or to directly
inactivate invading microbes through lytic activity. The production of complement component fractions C3a and C5a during
activation of this cascade serve primarily to markedly increase vascular permeability, and C5a functions as a PMN and
macrophage chemoattractant. This process leads to the recruitment of additional humoral and cellular defenses to the
specific area of infection. Excessive complement activation can produce deleterious effects in some instances. Complement
activation causes enhanced PMN adhesion, margination, and release of lysosomal enzymes that can directly damage certain
target tissues, such as the lung.

204 A 55-year-old renal transplant patient has been hospitalized in a Surgical Intensive Care Unit, receiving a
prolonged course of antibiotics following an attack of acute cholecystitis. The following statement(s) is/are true concerning
his management.
a. Due to the potential risk of Candida infection, prophylaxis with oral nystatin should be instituted early in the
patient’s course
b. A Candida urinary tract infection should be treated with systemic amphotericin B
c. Changes of Candida retinitis are of little significance
d. The presence of a virulent Candida bacteremia should suggest a dosage reduction in immunosuppressive agents
until the infection can be adequately controlled
Answer: a, d

Infections due to fungal pathogens have become increasingly common during the past decade, frequently occurring in
patients undergoing prolonged hospitalization in the Surgical Intensive Care Unit and in immunocompromised individuals.
Prophylaxis with oral antifungal agents (nystatin) is warranted, especially during periods of maximal immunosuppression in
transplant patients, in patients with uncontrolled diabetes, or during some cases of prolonged antibacterial microbial therapy.
In general, local, apparently noninvasive Candida infections involving the integument and mucus membranes are treated
with oral decontamination and topical antifungal therapy using topical agents such as nystatin. Candida urinary tract
infections can be treated with either an oral antifungal agent or with topical amphotericin B as a continuous bladder
irrigation. Several studies have demonstrated that those patients with three positive sites of Candida infection, or with
peritoneal or blood cultures positive for Candida exhibit higher survival rates when amphotericin B therapy is instituted
earlier in the course of infection. The presence of retinal changes compatible with Candida retinitis or Candida present
within the peritoneal cavity are considered indications for a limited course of amphotericin B therapy (300% to 500 mg).
Patients receiving exogenous immunosuppressive agents should undergo a marked dose reduction, and some agents should
be discontinued until evidence of infection is absolutely controlled or is eradicated.
205 The initiation of a humoral immune response involves a complex interaction of the antigen, cells and intercellular
messengers. Which of the following statement(s) concerning the initiation of the humoral immune response is/are correct?
a. Helper T lymphocytes stimulate B lymphocytes through secretion of cytokines such as interleukin 4 and 6
b. A number of cells can aid in presenting the antigen to the helper T cell including B lymphocytes and macrophages
c. All antigens require coordinated efforts of the various cellular components of the immune system
d. An antigen must be a living microorganism
Answer: a, b

Stimulation of the immune system occurs after a variety of antigen-presenting cells (B lymphocytes, macrophages, dendritic
cells, and Langerhans cells) act to engulf, process, and present antigen to T lymphocytes of helper lineage. These T
lymphocytes, in turn, act to stimulate B lymphocytes to become mature plasmacytes (through secretion of cytokines such as
interleukin 4 and 6) dedicated to the production of antibody directed against the specific antigen. An antigen may be defined
as any substance that stimulates the host immune response; that is, that the host immune system recognizes is foreign. Thus,
an antigen may be an invading microorganism, an inert particle, or any type of chemical compound that triggers the host
immune system. Although some antigens are able to directly stimulate B lymphocytes in and of themselves to produce
antibody (many polysaccharides), most antigens require coordinated efforts of the various cellular components of the
immune system.

206 The following statement(s) is/are true concerning viral infections.


a. The most common post-transplantation viral infections are caused by herpes viruses and include CMV and herpes
simplex virus
b. Viral infections occur at equal frequency anytime during the post-transplantation period
c. CMV infection in the post-transplant patient is most likely a pulmonary process
d. Herpes simplex virus (HSV) infection primarily presents with a mononucleosis-type syndrome with fever, lethargy,
and cough
Answer: a, c,

Solid organ transplant patients are prone to develop viral infection by virtue of exogenous immunosuppression. The most
common post-transplantation viral infections are those caused by herpes viruses (CMV, herpes simplex virus [HSV],
Epstein-Barr virus [EBV], and Varicella-Zoster virus [VZV]). All are most common during periods of maximal host
immunosuppression that occur immediately post-transplantation and during periods of allograft rejection. CMV is a
common cause of fever after solid organ transplantation, and evidence of CMV infection occurs in approximately 30% of
patients. The most common presentation for CMV infection is that of a febrile, leukopenic patient with a cough, diffuse
interstitial infiltrates on chest x-ray, and hypoxia.
HSV infection causes primarily oral pharyngeal ulcerations in most cases, although sporadic cases of disseminated disease
have been reported. EBV causes an occasional case of mononucleosis-type syndrome but has also been clearly indicated in
the pathogenesis of post-transplantation lymphomas. VZV infection can present as disseminated and occasionally life-
threatening infections in the nonimmune transplant patient or as painful herpes zoster in patients who have previously
developed chicken pox.

207 The following statement(s) is/are true concerning necrotizing fascitis.


a. Mortality rates as high as 40% can be expected
b. The infection involves only the superficial fascia, sparing the deep muscular fascia
c. An impaired immune system is a common factor predisposing to this condition
d. The infection is usually polymicrobial
e. Necrotizing fascitis is most likely to develop in the face of impaired fascial blood supply
Answer: a, c, d, e

Necrotizing fascitis is an uncommon infection of the deep and superficial fascia that is associated with mortality as high as
40% in many series. Although many underlying disease processes predispose patients to necrotizing fascitis, three common
factors are almost invariably present: 1) impairment of the immune system; 2) compromise of fascial blood supply, and 3)
the presence of microorganisms that are able to proliferate within this environment. Infections of this type are usually
polymicrobial in nature, with gram-positive organisms such as staphylococci and streptococci, gram-negative enteric
bacteria, and gram-negative anaerobic being frequently identified. These polymicrobial cultural results are assuredly
indicative of the occurrence of a synergistic process, perhaps in large part accounting for the severity of these infections.
Some microorganisms possess virulence factors that, in conjunction with an underlying host predisposition, allow this
disease process to occur without dependence on other bacteria. Examples of such bacteria include Clostridium,
Pseudomonas, and Aeromonas. In these patients, the process is often fulminant and is frequently associated with cellulitis,
myositis, fascitis, and bacteremia with attendant high mortality.

208 New treatment modalities designed to modulate host defense mechanisms that have been demonstrated
conclusively to be of benefit include:
a. Gut decontamination
b. Anti-LPS antibody
c. Anti-TNF antibody
d. Thymopentin
e. None of the above
Answer: e

Selective gut decontamination involves the use of orally administered antibiotics that achieve a high intraluminal level
directed against gram-negative aerobes and yeast, leaving the host anaerobic intestinal microflora relatively undisrupted.
Although a reduction and alteration of the microorganisms responsible for infectious episodes have been demonstrated in
certain groups of patients, a clear-cut impact on host mortality has not been shown. Because LPS may be responsible for
toxicity both directly and through host mediator systems, the availability of agents to bind against this portion of the gram-
negative bacteria to reduce mortality has been intensively examined. Unfortunately, large multicenter randomized trials
provide no evidence of benefit for this treatment. Similarly, since many of the systemic manifestations of gram-negative
bacteremia are mediated by cytokines, the effect of an anti-TNF antibody preparation is currently in clinical trial. No proven
benefits have yet been identified. Finally, the use of immunostimulants to enhance the state of activation of host defenses
has been proposed. Thymopentin is a peptide that contains active thymopoetin, a thymic molecule that acts to stimulate T-
lymphocyte activity. Preliminary trials indicate that this agent ameliorates host septic response after major operations and
trauma but conclusive evidence that concurrent reduction of infection-related mortality occurs is not available.

209 Antibacterial agents can be classified with regard to their structure, mechanism of action, and activity pattern
against various types of bacterial pathogens. Which of the following statement(s) is/are true concerning antimicrobial
classes?
a. Penicillins and cephalosporins share the compound structure of a b-lactam ring which binds to bacterial division
plate proteins
b. Tetracyclines and macrolides such as erythromycin inhibit bacterial ribosomal activity and therefore protein
synthesis
c. Aminoglycosides act in a similar fashion to tetracyclines and therefore are both bacteriostatic
d. Sulfonamides and trimethoprim act synergistically to inhibit purine synthesis
Answer: a, b, d

Penicillins, cephalosporins, and monobactams possess a b-lactam ring of some type and act to bind bacterial division plate
proteins, thus inhibiting cell wall peptidoglycan synthesis and either causing or inducing autolytic bacteriolysis. Because
gram-positive and gram-negative bacteria possess different types of division plate proteins, many of these agents exhibit
differential activity between these two types of microorganisms. Tetracyclines, chloramphenicol, and macrolides inhibit
bacterial ribosomal activity, and thus overall protein synthesis by a variety of different mechanisms. Aminoglycosides act to
inhibit protein synthesis and also presumably act on a different target site, a supposition based on the fact that
aminoglycosides are bacteriolytic and the other agents are bacteriostatic. Vancomycin inhibits assembly of peptidoglycan
polymers, whereas quinolones bind to DNA helicase proteins and inhibit bacterial DNA synthesis. Sulfonamides and
trimethoprim act in different mechanisms to inhibit protein synthesis, therefore two agents in combination act
synergistically.

210 The treatment of the following patient should include:


a. Initial empiric therapy directed against both aerobes and anaerobes
b. The addition of anti-fungal therapy in an elderly patient
c. A minimum of two weeks of antibiotic therapy is indicated
d. The addition of appropriate antibiotic therapy has made surgical therapy unnecessary in such cases
e. Either a single agent or combination therapy is appropriate if the agents selected possess activity against both
aerobic and anaerobic bacteria
Answer: a, e
The primary treatment for a perforated viscus is surgical, however antimicrobial therapy is an extremely important adjunct.
Empiric antibiotic therapy for secondary bacterial peritonitis and intraabdominal abscess should be directed against both
aerobes and anaerobes. Administration of an agent directed against only one component of the infection or the other is
inferior to combined therapy. Several studies indicate that the results of using several agents in combination is equivalent to
the use of a single agent therapy as long as the agents selected possess activity against both components of the infection.
The addition of antientercoccal or antifungal agents as initial therapy has not been substantiated. The most beneficial
duration of antibiotic therapy must be based on the setting for the specific patient. Minimal peritoneal contamination with
adequate surgical treatment may be treated with a three-to five-day course of antibiotics, whereas longer periods are
indicated for immunosuppressed patients and with patients with extensive contamination.

211 The following statement(s) is/are true concerning host defense mechanisms to intraabdominal infection.
a. Bacterial clearance can occur via translymphatic absorption
b. Phagocytic activity and bacterial killing can occur via resident phagocytic cells and an influx of PMNs
c. A fibrinogen-rich inflammatory exudate is released into the peritoneal cavity, trapping large numbers of bacteria
and other particulate matter
d. Perforations of a bowel may be walled off but are seldom sealed by the omentum and other mobile viscera
Answer: a, b, c

The introduction of microorganisms into the normally sterile peritoneal environment invoke several potent specialized host
antimicrobial defense mechanisms. Bacterial clearance, also termed translymphatic absorption, occurs through specialized
structures found only on the peritoneal mesothelium on the underside of the diaphragm that act as conduits for both fluid
and particulate matter. Lymphatic channels eventually form which drain into the venous circulation via the thoracic duct.
Bacteria not cleared via translymphatic absorption are rapidly engulfed by resident and recruited phagocytic cells including
resident macrophages on the peritoneal surface and omentum and attracted PMNs. The final primitive host defense
mechanism is sequestration by which a fibrinogen-rich exudate containing plasma oposonins appears during peritoneal
infection and fibrin polymerization occurs. Fibrin has the capacity to trap large numbers of bacteria and other particulate
matter. Acting in conjunction with omentum and other mobile viscera, perforations are sealed and the contaminated enteric
contents walled off, preventing continued soilage of the peritoneal cavity.

212 A 67-year-old male presents with an intraabdominal abscess secondary to perforated sigmoid diverticulitis. The
following statement(s) is/are true concerning his intraabdominal abscess.
a. Culture will likely reveal a solitary organism
b. Both aerobic and anaerobic islets are encountered in 50% of specimens
c. The most common aerobic islet will be likely E. coli and other gram-negative enteric bacilli
d. The most common anaerobic islet will be a Bacteroides species
Answer: b, c, d

Typically an intraabdominal infection results in perforation of a hollow viscus and the ensuing contamination of a normally
sterile peritoneal cavity. The normal bacterial flora found in that particular location of the alimentary tract thus determines
the initial inoculum. In parallel with the overall quantity of microorganisms, (both aerobes but predominantly anaerobes)
perforations of the lower small bowel and colon produce a high frequency of infections that contain anaerobic
microorganisms. Certain predictable patterns of bacterial islets are found, but on average four to five islets occur in patients
with established intraabdominal infection, more than half of which are anaerobes. Both aerobes and anaerobes are
encountered in 80% to 90% of specimens. Commonly encountered aerobes isolated are E. coli and other gram-negative
enteric bacilli such as Enterobacter, Klebsiella. Among the anaerobes, Bacteroides species (especially B. fragilis,
Clostridium), and anaerobic cocci are most consistently isolated.

213 The following statement(s) is/are true concerning gram-negative bacterial sepsis.
a. Mortality due to this condition has almost been eliminated due to therapeutic intervention with antibiotics,
aggressive hemodynamic monitoring and fluid resuscitation
b. Recent series have noted a decrease in the incidence of this condition
c. Predisposing factors include old age, malnutrition, and immunosuppression
d. Pseudomonas bacteremia is the most common cause of gram-negative sepsis
e. Polymicrobial sepsis is generally considered a more serious problem than sepsis due to a single organism
Answer: c, e
Gram-negative bacterial sepsis is a serious disease process that produces substantial morbidity and mortality in both normal
and immunocompromised patients (10% to 20% and 30% lethality, respectively), despite therapeutic intervention with
antimicrobial agents, aggressive hemodynamic monitoring, fluid resuscitation, and metabolic support. During the past
several decades, nosocomial infections due to gram-negative pathogens have increased in frequency with resultant increase
in the incidence of gram-negative bacteremia to between 3 and 13 cases per 1000 hospital admissions. Factors that
predispose to these infections include: 1) underlying host disease processes such as malignancy, diabetes; 2) old age and
disability; 3) malnutrition; 4) previous or concurrent antimicrobial antibiotic therapy; 5) major operations; 6) respiratory or
urinary manipulation or intubation; and 7) immunosuppression.
Although many different organisms cause this form of sepsis, E. coli predominates in overall frequency. Also common are
isolates of Klebsiella, Enterobacter and Serratia; Pseudomonas bacteremia is less common. Some studies, however, have
suggested that Pseudomonas sepsis is associated with the highest lethality. In several series, 10% to 20% of patients have
had polymicrobial series, and most investigators agree that polymicrobial sepsis is more lethal than infection with a single
organism.

214 Which of the following statement(s) is/are true concerning the various types of shock?
a. Traumatic shock is more commonly associated with subsequent organ injury and multiorgan failure syndrome than
hemorrhagic shock
b. Cardiogenic shock can be of either an intrinsic or compressive nature
c. Hypodynamic septic shock is associated with a decreased mortality risk when compared with hyperdynamic septic
shock
d. Hypoadrenal shock usually responds quickly to resuscitation
e. Neurogenic shock occurs with the absence of sympathetic activity
Answer: a, b, d, e

Classification schemes of shock based on cause have been developed for the seemingly dissimilar processes leading to
circulatory collapse and the shock state. Hypovolemic shock, the most common, is the result of intravascular volume
depletion through loss of red blood cell mass or plasma volume. Microvascular hypotension results from a combination of
low intravascular blood volume, diminished cardiac output, and compensatory sympathetic peripheral vasoconstriction.
Shock associated with trauma (traumatic shock) arises from the consequences of hypovolemia due to hemorrhage in
conjunction with direct soft tissue injury and bone fracture. Hypovolemia caused by blood loss and fluid extravasation into
injured tissues is compounded by activation of maladaptive inflammatory cascades initiated by the tissue injury. In contrast
to pure hemorragic shock, subsequent organ injury and multiorgan failure syndrome (MOFS) occurs much more frequently
following traumatic shock due to the over-expression of these immuno-inflammatory cascades. Cardiogenic shock is the
result of failure of the heart as an effective pump, resulting in inadequate cardiac output, tissue perfusion and oxygen
delivery. Intrinsic causes include myocardial infarction, cardiomyopathy, valvular heart disease, or rhythm disturbances.
Compressive cardiogenic shock is a discrete entity that results when extrinsic compression of the heart limits diastolic
filling and thus systolic ejection and cardiac output. Septic shock refers to hypotension and circulatory insufficiency
developing as a consequence of infection and the systemic response to that infection. In its hyperdynamic form, septic shock
is marked by diminished peripheral vascular resistance and generalized vasodilatation causing relative hypovolemia. In
contrast, hypodynamic septic shock occurs in situations of inadequate resuscitation or preterminal cardiovascular
decompensation, and is associated with vasoconstriction and a greatly increased mortality risk. Sympathetic denervation
through spinal cord injury, spinal anesthesia, or severe head injury produces generalized arterial vasodilatation and
venodilation. Shock occurs when the normal blood volume fails to fill the available intravascular space and severe relative
hypovolemia exists. Despite hypotension, there is a noteworthy absence of sympathetic activity, as occurs in hypovolemia
or cardiogenic shock. Profound shock can occur in surgical patients following stress due to the loss of the homeostatic
corticosteroid response. Hemodynamic instability may develop after an operative procedure or coincident with an unrelated
illness. The profound circulatory collapse is often refractory to vigorous resuscitation with fluids and pressor agents. The
response to exogenous corticosteroids is usually dramatic and potentially life-saving.

215 Which of the following statement(s) is/are true concerning metabolic derangements in sepsis and the systemic
inflammatory response syndrome which may follow progressive shock?
a. Alterations in glucose metabolism lead to the development of efficient substrate utilization
b. A progressive rise in serum triglyceride levels result from less efficient clearance and increased hepatic lipogenesis
c. A net negative nitrogen balance occurs due to the oxidative metabolism of proteins to meet energy needs
d. The serum aromatic amino acids fall rapidly as they are actively used in oxidative metabolism
Answer: b, c
A broad spectrum of metabolic abnormalities become apparent in sepsis and the systemic inflammatory response syndrome
following shock. Disruption of the normal cycles of carbohydrate, lipid, protein, and oxygen metabolism occur as
hypermetabolism develops. Through the Cori cycle, lactate from the periphery is shuttled back to the liver, where it is used
in the production of glucose. Because pyruvate is converted to alanine in the periphery, flux of alanine also contributes to
hepatic gluconeogenesis. The glycolytic oxidation of glucose to pyruvate and its subsequent glugoneogenic regeneration
from lactate is an inefficient cycling of substrate. There is no net energy production, but heat is released in significant
quantities. Alterations in lipid metabolism cause a progressive rise in the serum triglyceride level as a result of less efficient
clearance of exogenous triglycerides coupled with increased hepatic lipogenesis. Profound alterations in protein and amino
acid metabolism develop with characteristic changes in amino acid levels, nitrogen balance, and skeletal muscle mass.
Initially levels of the branch chain amino acids are reduced, whereas those of the aromatic amino acids are elevated. There
is an increase in the oxidative metabolism of protein to meet energy needs and a tremendous mobilization of nitrogen with
net negative nitrogen balance. The branch chain amino acids are preferentially utilized in the TCA cycle to maintain an
activity that otherwise would be lost from the diminished entry of carbohydrate-and fatty acid-generated acetyl coenzyme A.
This results in reduced serum level of leucine, isoleucine and valine.

216 Which of the following statement(s) is/are true concerning the microvascular and cellular response to shock?
a. Osmodically induced mobilization of intracellular fluid is the initial response to restore intravascular volume
b. With larger volume hemorrhagic shock deterioration of normal cellular transmembrane potential occurs resulting in
an increase in extracellular sodium and water
c. The accumulation of anaerobic metabolites override normal homeostatic vasomotor tone and contribute to the
maladaptive vasodilatation
d. Abnormal intracellular calcium homeostasis may contribute to the cellular dysfunction of shock
Answer: c, d

Moderate hypovolemia results in a relatively rapid spontaneous restitution of intravascular volume through expansion of the
plasma space. This plasma expansion by erythrocyte free fluid occurs within one hour as a result of alterations in pressure
and osmolarity and produces an associated hemodilution. Sympathetic discharge, associated arteriolar constriction, and
induced metabolic changes in osmolarity initiate the compensatory events at the microcirculatory level. The initial pressure-
related phase of restitution of blood volume in shock is overlapped by a second phase involving osmotically induced
mobilization of intracellular fluid. Osmotic mechanisms contributing to the restitution of blood volume after moderate
hemorrhage are not adequate in hemorrhage of greater magnitude. In larger hemorrhages (over 25% blood volume), there is
also deterioration of the normal cellular transmembrane potential, an increase in intracellular sodium and water, and a
concomitant decrease in extracellular fluid volume. Tissue hypoxia results, anaerobic metabolites accumulate, and the cell
cannot maintain the normal cell membrane potential. Accumulation of hydrogen ion, lactate, and other products of
anaerobic metabolism override homeostatic vasomotor tone and contribute to a maladaptive vasodilatation, further
augmenting hypotension and hypoperfusion. The uptake of fluid by the “failing” cell is a major source of food sequestration
following shock. Loss of membrane function is proportional to both the extent and duration of shock or degrees of sepsis.
The etiology of membrane failure is unclear but appears multifactorial. Loss of intracellular ATP energy stores during
hypoperfusion or direct toxicity during sepsis may inhibit the membrane sodium-potassium pump. Cellular dysfunction also
appears to be related to abnormal intracellular calcium homeostasis.

217 Which of the following statement(s) is/are true concerning the pulmonary response to shock?
a. The acute pulmonary vascular response to shock differs markedly from that of systemic vasculature
b. The pulmonary edema of ARDS occurs in the face of elevated left heart pressures
c. The initial physiologic changes of ARDS involve the capillary endothelial cells and the type I pneumocyte
d. Mechanisms proposed in the pathogenesis of ARDS include injury from mediators of inflammation elsewhere and
from activated cellular elements
e. A decrease in lung compliance may result from the loss of type I pneumocytes
Answer: c, d, e

Contributing pathophysiologic processes to the pulmonary manifestations of shock include the pulmonary component of the
cardiovascular response, disruption of the normal lung mechanics, and acute lung injury or ARDS due to sepsis. Pulmonary
function may be further compromised by pathology intrinsic to the lung itself, including pulmonary contusion, aspiration,
airway obstruction, pneumonia, pneumothorax, hemothorax, and atelectasis. The acute pulmonary vascular response to
shock largely parallels that of the systemic vasculature. The increase in pulmonary vascular resistance, which may
proportionally exceed that of the systemic circulation, transiently accompanies the systemic adrenergic response. ARDS is a
syndrome of progressive lung injury that may arise as a direct consequence of shock or other disease processes. The
characteristic findings of ARDS are the presence of pulmonary edema, hypoxemia, and significantly decreased lung
compliance. The pulmonary edema is noncardiac in origin and occurs in the face of normal left heart pressures. The
hypoxemia results from the development of intrapulmonary shunting and perfusion of under and nonventilated alveoli. The
decrease in lung compliance results from the loss of surfactant and lung volume in combination with the presence of
interstitial fluid and alveolar edema. Progressive histologic changes of ARDS become apparent in pulmonary capillaries,
interstitium, and alveoli. Initially, interstitial edema develops with swelling of the capillary endothelial cells and the type I
pneumocytes. The type I pneumocytes subsequently slough, and alveolar edema ensues. Functional surfactant is lost with a
significant increase in alveoli opening pressure and decrease in alveolar surface tension. Mechanisms proposed in the
pathogenesis of ARDS include injury from mediators of inflammation elaborated elsewhere, and from activated cellular
blood elements.

218 Which of the following statement(s) is/are true concerning the diagnosis and management of hypovolemic shock?
a. A fall in hematocrit or hemoglobin always accompanies hemorrhagic shock
b. The treatment of shock is generic regardless of the etiology
c. Pharmacologic intervention to increase myocardial contractility in hypovolemic shock is an important part the
early management
d. Complications are less frequent after treatment of hemorrhagic shock than septic or traumatic shock
Answer: d

Hypovolemic shock is readily diagnosed when there is an obvious source of volume loss and overt signs of hemodynamic
instability and increased adrenergic output are present. After acute hemorrhage, hemoglobin and hematocrit values do not
change until compensatory fluid shifts have occurred or exogenous fluid is administered. These values decrease once
transcapillary refill, osmotic-induced shifts, or non-RBC volume resuscitation expands the blood volume. It is imperative
that the distinction be made between hypovolemic and cardiogenic forms of shock, because appropriate therapy differs
dramatically. Restoration of perfusion in hypovolemic shock requires reexpansion of circulating blood volume in
conjunction with necessary interventions to control ongoing volume loss. Continued hemodynamic instability after fluid
resuscitation implies that shock has not been reversed or that there is ongoing blood or volume loss. In severe, prolonged
hypovolemia, ventricular contractile function may itself become depressed and require inotropic support to maintain
ventricular performance, but in general, pharmacologic interventions directed toward increased contractility in situations of
inadequate preload are ineffective, further complicate metabolic derangements, and are not indicated until adequate volume
replacement has been completed. Complications are less frequent after treatment of hemorrhagic shock than in situations of
septic or traumatic shock. In the later circumstances, the massive activation of inflammatory mediator response systems and
consequences of their disseminated, indiscriminate cellular injury can be quite profound.

219 Which of the following statement(s) is/are true concerning the neuroendocrine responses to shock?
a. Sympathetic nerve endings release epinephrine which is responsible for greater than 80% of systemic vascular
resistance
b. Endogenous epinephrine is the primary contributor to systemic vascular resistance
c. Increased pancreatic secretion of glucagon contributes to glucose intolerance associated with injury and sepsis
d. The renin-angiotensin axis further augments the sympathetic-mediated vasoconstriction
Answer: c, d

The neuroendocrine response to shock attempts to achieve restoration of effective blood volume, mobilization of metabolic
substrates, and maintenance of central profusion. Both peripheral and central afferent stimuli to the central nervous system
are involved in inducing this response. Hypotension, associated with a decrease in impulses from the aortic and carotid
baroreceptors, disinhibits the vasomotor center. This disinhibition results in increased adrenergic output and decreased vagal
activity. Sympathetic nerve endings release norepinephrine, inducing peripheral and splanchnic vasoconstriction which is
responsible for greater than 80% of systemic vascular resistance and is a major contributor to maintenance of central organ
perfusion and venous return. Plasma levels of both epinephrine and norepinephrine are elevated with injury, and the degree
of the catecholamine elevation corresponds to the magnitude of injury. In shock the effects of endogenous epinephrine are
largely metabolic. In addition to initiating autonomic nervous activity, the hypothalamus secretes releasing hormones, which
induce the stress hormone release of the pituitary. As part of this response, adrenocorticotropic hormone (ACTH) secretion
by the anterior pituitary is increased stimulating cortisol secretion by the adrenal cortex. In conjunction with elevated
plasma levels of cortisol and epinephrine, increased pancreatic secretion of glucagon accelerates hepatic gluconeogenesis
and further aggravates the glucose intolerance that follows injury and sepsis. The secretion of renin is increased in responses
to adrenergic discharge and decreased perfusion of the juxtaglomerular apparatus in the kidney. Renin allows formation of
angiotensin I in the liver, which is then converted to angiotensin II in the lungs. Angiotensin II is an extremely effective
vasoconstrictor that further augments sympathetic-mediated vasoconstriction.

220 A 22-year-old man sustains a single stab wound to the left chest and presents to the emergency room with
hypotension. Which of the following statement(s) is/are true concerning his diagnosis and management?
a. The patient likely is suffering from hypovolemic shock and should respond quickly to fluid resuscitation
b. Beck’s triad will likely be an obvious indication of compressive cardiogenic shock due to pericardial tamponade
c. Echocardiography is the most sensitive noninvasive approach for diagnosis of pericardial tamponade
d. The placement of bilateral chest tubes will likely resolve the problem
Answer: c

Shock from cardiac compression occurs when external pressure on the heart impairs ventricular filling. Because ventricular
filling is a function of venous return and myocardial compliance, any process that places pressure on the heart can cause
compressive cardiogenic shock. Included among these are pericardial tamponade, tension pneumothorax, mediastinal
hematoma, and positive pressure from mechanical ventilation. Any patient with hypotension after a wound in proximity of
the heart should be considered to have compressive cardiogenic shock until proven otherwise. The classical clinical findings
of pericardial tamponade include Beck’s triad of hypotension, neck vein distention and muffled heart sounds. Pulses
paradoxus may be noted (this involves a decrease rather than the normal increase of systolic blood pressure with inspiration;
values 10mmHg are significant). These findings, however, may be obscured in a noisy emergency room environment by
positive pressure ventilation or by associated injuries. Placement of a CVP catheter confirms the elevation of right-sided
filling pressure. If a pulmonary artery catheter has been placed, findings consistent with tamponade or other forms of
cardiac compression are a trend toward equalization of chamber pressures as hypotension progresses. In the patient at risk,
echocardiography is an extremely sensitive and noninvasive approach to demonstrate pericardial fluid and the need for
operation. Pericardial tamponade must be relieved urgently and cardiac injuries require emergent sternotomy. Chest tube
placement would not be appropriate as the sole treatment in this patient.

221 A 32-year-old man suffers a spinal cord injury with a resultant paraplegia in a motorcycle accident. He presents to
the emergency room with hypotension. Which of the following statement(s) is/are true concerning his diagnosis and
management?
a. The low blood pressure can be assumed to be due to neurogenic shock
b. The sole cause of hypotension is the loss of sympathetic input to the venous system
c. Despite significant hypotension, secondary organ injury will be uncommon
d. There is no role for pharmacologic intervention to maintain blood pressure
Answer: c

Neurogenic shock results from interruption of sympathetic vasomotor input and develops after spinal cord injury, spinal
anesthesia, and severe head injury. Under normal conditions, baseline sympathetic activity establishes a degree of arteriolar
and venous constriction. Ablation of this tone results in decreased systemic vascular resistance and a dramatic increase in
venous capacity, causing hypotension due to relative hypovolemia. Arteriolar dilatation not only lowers the systemic
vascular resistance but also allows previously unopened vascular beds to be perfused, greatly expanding venous capacity.
Removal of sympathetic inputs to innervated portions of the venous system allows further venodilatation. Restoration of an
effective, albeit expanded, intravascular volume may require extremely large volumes of resuscitation fluid to restore
normal cardiac filling pressures. This will restore cardiac output and reverse hypotension. However, pharmacologic
intervention with vasoactive drugs may be necessary and is preferable to excessive volume resuscitation. Post-shock
sequelae are infrequent. Although there is significant hypotension with neurogenic shock, there is usually little if any
hypoperfusion. Thus, activation of inflammatory cascade and subsequent organ injury rarely occur.
A major pitfall in the management of neurogenic shock arises when there is coexistent hemorrhage or ongoing volume loss
that is not appreciated. This is not an unusual situation because cervical spine trauma causing paraplegia or severe head
injury is frequently associated with multiple injuries. Thus, in trauma the initial response to neurogenic shock is large
volume resuscitation regardless of the presumed etiology. If hemodynamic instability persists after initial trauma
resuscitation, one must assume that the cause is not neurogenic and search for occult blood loss or cardiogenic causes of
shock.

222 Which of the following statement(s) is/are true concerning septic shock?
a. The clinical picture of gram negative septic shock is specifically different than shock associated with other
infectious agents
b. The circulatory derangements of septic shock precede the development of metabolic abnormalities
c. Splanchnic vascular resistance falls in similar fashion to overall systemic vascular resistance
d. Despite normal mechanisms of intrinsic expansion of the circulating blood volume, exogenous volume
resuscitation is necessary
Answer: d

The clinical findings in sepsis and septic shock represent the host response to infection. Gram-positive and gram-negative
bacteria, viruses, fungi, rickettsiae, and protozoa have all been reported to produce a clinical picture of septic shock, but the
overall response is independent of the specific type of invading organism. Septic shock develops as a consequence of the
combination metabolic and circulatory derangements accompanying the systemic infection. It appears that the circulatory
deficits are preceded by the metabolic abnormalities induced by infection. In fact, the circulatory changes in hyperdynamic
sepsis appear to be an adaptive response to the underlying metabolic dysfunction. Cardiac output is high and systemic
vascular resistance low in hyperdynamic septic shock. However, splanchnic vasoconstriction is pronounced even in the
absence of systemic hypotension and even though systemic vascular resistance is reduced. Expansion of circulating blood
volume can occur through either transcapillary refill or fluid resuscitation. Due to the ongoing inflammatory mediator-
induced increases in capillary permeability and continued loss of intravascular volume, exogenous volume resuscitation
must be provided to restore venous return and ventricular filling.

223 Which of the following statement(s) is/are true concerning tumor necrosis factor (TNF)?
a. TNF is a product of activated macrophages secreted in response to contact with endotoxin or lipopolysaccharide,
antibody complexes, or inflammatory stimuli
b. The liver and gut appear to be a major source of TNF following hypoperfusion
c. Circulating levels of TNF correlate well with severity of tissue injury in shock
d. Recently completed clinical trials of anti-TNF antibody in septic patients shows a marked improvement in survival
Answer: a, b

Tumor necrosis factor (TNF), a protein product of activated macrophages, is secreted in response to contact with endotoxin
or lipopolysaccharide, antibody complexes, or other inflammatory stimuli. Elevation of serum levels of TNF have been
reported shortly after experimental trauma and shock, however, documentation of elevated circulating levels of TNF in
human shock is less clear. Furthermore, circulating levels of TNF cannot be correlated with severity of tissue injury or
shock. This variability is thought to be due to rapid clearance and uptake by membrane receptors and by soluble membrane
receptors that are released from multiple cells following stress and injury. Following hypoperfusion the liver and gut appear
to be the major source of TNF that is rapidly cleared but responsible for inducing hepatocyte changes following shock. The
release of breakdown products and escape of bacterial and endotoxin through the damaged mucosal barrier of the gut
following shock allows or induces activation of tissue-fixed macrophage (Kupffer cell) of the liver which then produces
secondary inflammatory mediators contributing to the post-resuscitation clinical response and inflammatory mediator
activation seen in the systemic inflammatory response syndrome.
TNF is central to inflammatory response, particularly in sepsis and following endotoxemia or bacteremia. TNF also induces
secondary inflammatory responses through direct interaction with specific membrane receptors, TNF-r. Treatment with anti-
TNF antibody in the experimental setting protects animals from the deleterious effects of lethal bacteremia and
endotoxemia. However, recently completed clinical trials in septic patients utilizing infusion of monoclonal antibodies to
the TNF molecule have shown no overall survival benefit.

224 Which of the following statement(s) is/are correct concerning the immunoinflammatory response to shock?
a. The anaphylactoxins, C3a and C5a, are products of activation of only the classical pathway of the compliment
cascade
b. Eicosanoids, such as prostaglandins are stored in platelets and endothelial cells and released in response to
inflammatory stimuli
c. Thromboxane and PGI2 have similar effects
d. Platelet-activating factor can be released by both circulating and fixed tissue cells
Answer: d

Inflammatory mediators have recently been recognized as playing a significant role in the clinical manifestations and
progression of shock and the development of subsequent complications. These mediator systems function primarily as
parcrine and autocrine agents in the local environment and are not usually detectable systemically. The over-expression and
systemic dissemination of these mediators produces the toxic autodestructive processes underlying multiorgan failure
syndrome with attendant high mortality. The compliment cascade is activated in shock and tissue injury through both the
classical and alternative pathways. Activation of either pathway results in generation of the anaphylatoxin, C3a and C5a,
soluble products with potent systemic hemodynamic effects. The eicosanoids, which include the prostaglandins and
leukotrienes are formed acutely from arachidonic acid released from the membrane phospholipid by phospholipase A2.
Eicosanoids are not stored in any measurable level and are generated as needed from readily available arachidonic acid in
response to various inflammatory phenomena. Platelets, white cells, and endothelial cells are a rich source of these
compounds. Thromboxane (TXA2) is the major arachidonic acid metabolite elaborated by platelets. TXA2 induces intense
vasoconstriction, platelet aggregation and degranulation, neutrophil margination in the microcirculation and bronchial
constriction. PGI2, the major arachidonic acid metabolite formed by endothelial cells, serves a check against actions of
TXA2. PGI2 is a vasodilator and a potent inhibitor of platelet aggregation. Platelet aggravating factor is a potent
phospholipid mediator released by neutrophils, platelets, macrophages and endothelial cells in response to ischemia, tissue
injury and sepsis. Its effects include decreased cardiac function, increased pulmonary vascular resistance,
bronchoconstriction, peripheral vasodilatation, and increased vascular permeability.

225 Which of the following physical findings are associated with the various classes of hemorrhagic shock?
a. Mild shock (< 20% blood volume): Pallor, cool extremities, diminished capillary refill and diaphoresis
b. Moderate shock (20%–40% blood volume): All of the above plus tachycardia and hypotension
c. Severe shock (> 40% blood volume): Systemic hypotension, changes in mental status, tachycardia, oliguria
d. All of the above
Answer: a, c
PHYSICAL FINDINGS IN HEMORRHAGIC SHOCK*

Moderate
Mild (<20% (20%-40% Severe(>40%
Blood Volume) Blood Volume) Blood Volume)
Pallor Pallor Pallor
Cool extremities Cool extremities Cool extremities
Diminished capillary Diminished capillary Diminished capillary
refill refill refill
Diaphoresis Diaphoresis Diaphoresis
Collapsed Collapsed Collapsed
subcutaneous subcutaneous subcutaneous
veins veins veins
Tachycardia Tachycardia
Oliguria Oliguria
Postural Hypotension
hypotension Mental status
changes

* Alcohol or drug intoxication may alter physical findings.

226 A 68-year-old male who underwent a repair of an abdominal aortic aneurysm 5 days ago, develops tachycardia,
tachypnea, hypotension with cool, pale, mottled cyanotic extremities. He is agitated and complains of shortness of breath.
Which of the following statement(s) is/ are correct concerning his diagnosis and management?
a. Myocardial ischemia secondary to preexisting coronary artery disease is most likely the underlying cause of this
problem
b. Invasive hemodynamic monitoring with a Swan-Gantz catheter will demonstrate a low cardiac output, a high
systemic vascular resistance, and elevated cardiac filling pressures
c. The use of morphine sulphate and nitrates should be part of the initial management
d. The primary pharmacologic treatment involves the use of moderate doses of inotropic agents
e. Afterload reduction with nitroprusside is absolutely contraindicated
Answer: a, b, d

Intrinsic cardiogenic shock results from failure of the heart as an effective pump. Coronary artery disease is the most
common cause of myocardial insufficiency, but contractile dysfunction may also rise as a consequence of cardiomyopathy,
myocarditis, or metabolic abnormalities. Invasive hemodynamic monitoring often establishes a specific nature of shock and
allows appropriate treatment to be delivered in an effective and expedient manner. Hemodynamic findings consistent with
cardiogenic shock include a low cardiac output and high systemic vascular resistance, with elevated cardiac filling
pressures. The initial measures in the management of cardiogenic shock include the administration of supplemental oxygen,
mechanical ventilation (as needed), and appropriate treatment of dysrhythmias. Hypotension usually precludes the use of
morphine sulfate and nitrates, drugs typically used in simple congestive heart failure to alleviate cardiac pain and ameliorate
pulmonary vascular congestion. The use of beta-adrenergic agonists such as dopamine and dobutamine, in moderate doses,
offers positive inotropic support without excessive alpha-adrenergic activity. Increasing the inotropic state of the heart shifts
the entire Starling curve upward, resulting in increased cardiac output for each level of cardiac filling. Afterload reduction
may prompt increases in cardiac output through decreases in resistance to flow. The use of nitroprusside or other dilators
requires relative blood pressure stability and close hemodynamic monitoring. Infusion of afterload-reducing agents can be
administered in conjunction with inotropic support.

227 Which of the following statement(s) is/are true concerning ischemia reperfusion injury?
a. During ischemia, ATP degradation results in increased plasma and intracellular levels of hypoxanthine and
xanthene
b. Oxygen free radicals such as the superoxide radical are involved in the expression of the proinflammatory
phenotype of endothelial cells, macrophages and neutrophils
c. The intracellular adhesion molecule-1 (ICAN-1) contributes to injury and disruption of the endothelial lining, with
extensive capillary leak and resultant interstitial edema
d. Animal models have demonstrated that passive immunization with antibodies to neutrophil adhesive complex
lessen the ischemic/reperfusion microvascular injury
Answer: a, b, c, d

During the ischemia and hypoperfusion phase, degradation of ATP stores essential to maintain cell integrity and significant
loss of diffusible intracellular adenine neuclotides occurs. As ATP further degrades there is an elevation in plasma and
intracellular levels of hypoxanthine and xanthene which upon restoration of perfusion and reoxygenation are catalyzed by
xanthine oxidase resulting in the formation of superoxide radicals. These radicals plus others such as hydrogen peroxide and
hydroxyl radical are generated and lead to endothelial and parenchymal cell injury through membrane lipid peroxidation
and activation of critical enzymes. These radicals have also been shown to be involved in the expression of proinflammatory
phenotype endothelial cells and on macraphages and neutrophils. The proinflammatory phenotype of the endothelium
includes procoagulant activity and the expression of adhesion molecules on the membrane surface, including the
intercellular adhesion molecule-1 (ICAN-1) and the selectins. The subsequent adhesion of activated neutrophils to the
endothelial leads to an explosive oxidative burst producing additional radicals and extensive release of proteolytic enzymes
leading to injury and disruption of the endothelial lining, extensive capillary leak, and massive interstitial edema. Passive
immunization of animals with monoclonal antibodies to either the neutrophil adhesive complex or the endothelial selectins
dramatically lessens ischemia/reperfusion microvascular injury.

228 Which of the following statement(s) is/are true concerning the physiology of the microvascular system?
a. Filtration of capillary fluid into the interstitial and the subsequent reabsorption is influenced by Starling’s law of
ultrafiltration
b. The most important variable controlling blood to a capillary bed is the length of the vessel
c. Most of the resistance to systemic blood flow occurs at the arteriolar level
d. Adrenergic vasoconstriction can arrest blood flow to an entire capillary bed
Answer: a, c, d

Exchange of material between the vascular space and the cell of various tissues via the interstitial space is essential for
organ viability and occurs at the capillary level. The filtration of capillary fluid into the interstitium and its subsequent
reabsorption into the post capillary venule is governed by microvascular permeability in conjunction with the balance
between hydrostatic and oncotic pressures. The relation of these forces to one another (and their net effects) are illustrated
by what is termed Starling’s law of ultrafiltration. In normal circumstances, a net filtration from capillary to interstitium is
effected by a relatively higher capillary hydrostatic pressure, whereas net reabsorption from the interstitium back into the
post capillary venule occurs as hydrostatic pressure falls and oncotic forces predominate. Although the mechanisms
controlling blood flow to the capillary bed are complicated and vary among the different tissues, certain concepts are useful.
Poiseuille’s law describes the relation between flow of fluid through a tube and the tube length and radius, the fluid
viscosity, and the pressure gradient between ends of the tube. The radius of the tube (or vessel) is the single most important
variable, because flow is proportional to the radius to the fourth power. Vasoconstrictive and vasodilatory influences directly
impact local blood flow, as well as flow to other tissues through secondary effects on the systemic pressure. This secondary
effect of peripheral vasoconstriction maintains the pressure gradient for central perfusion of the heart and brain. Systemic
blood flow meets most of its resistance at the arteriolar level. While the individual capillary radius is significantly smaller,
the vast number of capillaries offers less total resistance. The vascular smooth muscle in arterioles has both a-and b-
adrenergic receptors. Alpha stimulation affects vasoconstriction where beta stimulation affects vasodilatation. The efferent
sympathetic fibers innervating the precapillary resistance vessels and the venous capacitance vessels release norepinephrine
on stimulation, which induces smooth muscle contraction and narrowing of the caliber of the vessels. These contractions are
potent enough that blood flow to entire capillary beds can be arrested by adrenergic vasoconstriction.

229 Which of the following statement(s) is/are true concerning the effects of MOFS?
a. Pulmonary dysfunction tends to arise early and may resolve within 7 to 10 days
b. Unless the precipitating insult has prompted oliguric acute tubular necrosis, renal function tends to be maintained
early in the course of MOFS
c. Although hepatic dysfunction is common with MOFS, the GI tract plays little role in this process
d. Intercurrent nosocomial infection, most commonly pulmonary, is a common complication providing a “second hit”
to the patient
Answer: a, b, d

Pulmonary dysfunction typically arises early in the development of systemic inflammation and may represent mild
relatively localized acute lung injury or it may be a prelude to fulminant ARDS. The lung injury, and associated dysfunction,
may resolve over the initial 7 to 10 days or persist, depending on the ongoing pathologic process. Many times a “second hit”
such as a nosocomial infection, which is most commonly pulmonary, is a complication which can frequently worsen the
pulmonary condition. Renal function tends to be maintained early in the course unless the precipitating insult has been
prompted by a sudden oliguric acute tubular necrosis. With persistent activation and inflammatory mediators, glomerular
filtration falls and the development of oliguric or polyuric renal failure marks the gradual transition into MOFS.
Gastrointestinal abnormalities include ileus, stress ulceration, diarrhea, and mucosal atrophy. Breakdown of the mucosal
barrier allows translocation of bacteria and endotoxin. Hepatic dysfunction is marked by progressive rise in serum bilirubin
levels after a latent period of several days.

230 Which of the following statement(s) is/are true concerning hypoadrenal shock?
a. In the United States, idiopathic adrenal atrophy (Addison’s disease) is the most common cause
b. Laboratory abnormalities include hyponatremia, hypochloremia, and hyperkalemia
c. Fever may be seen with hypoadrenal shock
d. ACTH stimulation test is the diagnostic test of choice to confirm hypoadrenal shock
Answer: b, c, d

Shock of a dramatic nature, poorly responsive to resuscitation, may develop as a consequence of adrenal insufficiency. In
this country, adrenal insufficiency most commonly arises as a consequence of the chronic therapeutic administration of high
doses exogenous corticosteroids causing adrenal suppression. Other causes include idiopathic adrenal atrophy (Addison’s
disease), tuberculosis, metastatic disease, bilateral hemorrhage, and amyloidosis. The stress of illness, operation, or trauma
typically requires that the adrenal glands secrete cortisol in excess of that required in the nonstressed state (approximately
3–4 fold). Insufficiency not otherwise apparent may manifest itself only after major physical stress. Findings associated
with adrenal insufficiency include weakness, fatigue, anorexia, abdominal pain, nausea, vomiting, and weight loss. Surgical
patients with significant adrenal insufficiency need not present with the above findings. More typical is the development of
refractory shock, frequently with hyperthermia, in the course of injury or illness. Hypotension may be dramatic despite
massive volume resuscitation and pressor support. Laboratory findings suggesting hypoadrenalism include hyponatremia,
hypochloremia and hyperkalemia. The diagnosis of adrenal insufficiency may be confirmed or excluded by means of an
ACTH stimulation test. A significant major cortisol response should be elicited by ACTH administration.

231 Which of the following statement(s) is/are correct concerning the cardiovascular response to shock?
a. Changes in cardiac contractile function shift the Frank Starling curve up and down
b. Venoconstriction from skeletal muscle is a significant contributor to the restoration of blood volume with shock
c. Arterial vasoconstriction affects all vascular beds equally
d. The total circulating blood volume is equally split between the arterial and venous system
Answer: a

Central in the general cardiovascular response to shock is the action of the heart itself. The principle determinants of cardiac
function in the normal heart are the volume of blood available for the heart to pump (preload), the systolic contractile
capability, and the diastolic filling of the ventricles. In hypovolemia, the two dynamic variables of cardiac function,
ventricular filling and myocardial contractility remain paramount and determine the stroke volume. The product of stroke
volume and heart rate in turn determines the cardiac output. Increases in ventricular end-diastolic volume, reflecting venous
return, cause ventricular distention. Ventricular distention in turn produces increased volume output with each stroke, the
Frank Starling mechanism. Contractile function may vary independent of volume status. Changes in the contractile function
shift the Starling curve up and down, producing increases or decreases in stroke volume for any given end-diastolic volume.
A fundamental requirement for cardiovascular function is adequate cardiac filling, and cardiac output cannot exceed venous
return. The venous system contains nearly two-thirds of the total circulating blood volume, including 20% to 30% within the
splanchnic venous system. Most of this volume resides in small veins, which comprise the bulk of venous capacitance. The
venous system, especially that of the splanchnic circulation, becomes important in the physiologic compensation to
hypoperfusion because it serves as a dynamic reservoir for the autoinfusion of blood volume involving both active and
passive mechanisms. The splanchnic circulation makes major contributions to the maintenance of venous return, therefore,
it is likely that sympathetic venoconstriction is responsible for a portion of the blood mobilized from the splanchnic venous
circulation. Sympathetic mediated venoconstriction in skin and skeletal muscle is probably not as significant as a source of
blood volume. Selective vasoconstriction occurs in response to alpha adrenergic receptor stimulation with increased
sympathetic activity in shock. Sympathetic stimulation does not cause significant vasoconstriction of either cerebral or
coronary vessels, with normal blood flow maintained in these circulations. Blood flow to the skin is sacrificed early,
followed by that to the kidneys and splanchnic viscera.

232 Which of the following statement(s) is/are true concerning pharmacologic agents used in the treatment of shock?
a. The primary difference between dopamine and dobutamine is the absence of significant a adrenergic activity
b. The renal and mesenteric vasoconstrictive effects of norepinephrine complicate and sometimes restrict its use
c. The apparent paradoxical use of vasodilators, such as nitroprusside, in shock is indicated as a means to augment
cardiac function
d. Isoproterenol with its potent b-adrenergic effect, is a particularly useful agent in the treatment of all forms of shock
Answer: a, b, c

Therapeutic adjustments of intravascular volume (preload) and systemic vascular resistance (afterload) form the basis of the
treatment strategies for all forms of shock. Optimal volume resuscitation should precede measures to augment to contractile
function of the heart. Inotropic agents are used in shock when there is inadequate cardiac output despite adequate circulating
blood volume. Dopamine and dobutamine are often times first line agents in the pharmacologic treatment of shock.
Dopamine, at low doses, stimulates dopaminergic receptors producing renal arteriolar vasodilatation with associated
increases in renal blood flow, urine output, and sodium excretion. At moderate doses, stimulation of cardiac b-receptors
produces increases in contractility and cardiac output with little effect on heart rate or blood pressure. At higher doses,
peripheral vasoconstriction from increasing a activity becomes more pronounced, prompting significant increases in
vascular resistance and blood pressure. Dobutamine’s predominant effect is an increasing cardiac contractility with lesser
increases in heart rate. Some reduction of peripheral vascular resistance may also occur. When compared to dopamine,
dobutamine produces less peripheral vasoconstriction and less chronotropic response. Norepinephrine exerts both a and b-
adrenergic effects, with a effects being evident at lower infusion rates and a effects more prominent at high doses. The major
use of norepinephrine in current practice is in the patient with hypotension that persists despite appropriate volume
resuscitation and the use of inotropic agents. Renal and mesenteric vasoconstrictive effects of norephinepherine complicate
its use, especially when support is needed for significant periods of time. Isoproterenol is a potent b-adrenergic agent. With
isoproterenol, myocardial oxygen demand is increased and diastolic coronary feeling is limited by tachycardia or
diminished diastolic pressure. Indications for the use of isoproterenol are fairly limited, because agents with fewer adverse
effects have become available.
Vasodilators are used to augment cardiac function through optimization of ventricular filling pressures (preload) and
systemic vascular resistance (afterload) both of which reduce demands on the myocardium. Decreases in afterload prompt
increases in cardiac output and venodilatation contributes to decreases in pulmonary venous pressure and central venous
pressure. Hypotension, however, may develop therefore patients must have careful constant monitoring of arterial pressure
and repeated hemodynamic measurements with a pulmonary artery catheter.

233 Which of the following statement(s) is/are true concerning the treatment of MOFS?
a. Prevention and therapy of MOFS requires control of the infectious or inflammatory source
b. Restoration of normal clinical parameters such as blood pressure, pulse rate, and urine output ensures optimal
resuscitation in most patients
c. Branch chain amino acids play and important role in the nutritional support of the patient
d. Because of the nature of gut injury, total parenteral nutrition is preferred for most patients with MOFS
Answer: a, c
The therapy of MOFS is directed towards interrupting the involving pathophysiologic process and providing an optimal
physiologic environment for healing and recovery. Fundamental concerns are control of the source of infection,
inflammation or instability; restoration of microcirculatory blood flow and oxygen transfer, and the institution of optimal
supportive care. Both the prevention and therapy of MOFS, therefore, requires source control and restoration of adequate
profusion. Resuscitation efforts are directed toward restoration of adequate microcirculatory blood flow in all organ
systems. Restoration of normal clinical parameters such as blood pressure, pulse rate, urine output, and acid-base balance
does not ensure optimal resuscitation. The physiologic endpoint that most closely corresponds with adequate
microcirculatory flow is the level of cardiac output and the oxygen delivery at which oxygen consumption and lactate
production remain independent of flow.
The importance of metabolic support in the patient with MOFS cannot be overemphasized. The malnutrition of MOFS is
markedly different than that of starvation and the nutritional requirements also differ. If optimal quantities of appropriately
formulated amino acid solutions are given, protein synthetic rates can approach catabolic rates and the goal of nitrogen
balance can be achieved. Formulas rich in branch chain amino acids appear to be more efficient in promoting nitrogen
retention and minimizing urea production. Whenever feasible, enteral feeding is preferred over TPN because evidence
suggests that bacterial translocation from the gut can be limited through the use of enteral feeds. Enteral absorption and
processing of nutrients appears superior to TPN and lessens overall complications.

234 Which of the following statement(s) is/are true concerning the multiorgan failure syndrome (MOFS)?
a. Changes in the splanchnic and pulmonary microcirculation are critical to the development of MOFS
b. Tissue fixed microphages, including the liver Kupffer cell, have little role in the development of MOFS
c. MOFS represents systemic consequences of loss of homeostatic control of local inflammation and microcirculatory
hypoperfusion
d. The nature of MOFS is highly dependent upon the etiology of the underlying problem
Answer: a, c

The nature of multiorgan failure syndrome (MOFS) is that of a diffuse cellular injury, developing systemically as a
consequence of losing homeostatic control of local inflammation and microcirculatory hypoperfusion. Endothelial injury,
platelet aggregation and activation of macrophages and neutrophils occur, and the clotting, fibrinolytic, kinin, and
complement cascades are activated, along with the release of potent inflammatory cytokines. The effects of shock,
resuscitation, and reperfusion, and the subsequent development of MOFS appear to be critically dependent on changes in
the splanchnic and pulmonary microcirculations. These vascular beds appear to be major sites of activation of subsequent
inflammatory mediator production that underlies the diffuse systemic inflammatory response. Extensive activation of the
liver Kupffer cell and release of inflammatory mediators coupled with the ongoing release of activated neutrophils and by-
products of activated gut macrophages is responsible for the injury to the pulmonary microcirculation and secondary
induction of alveolar macrophage and additional inflammatory mediator systems. Excessive and persistent macrophage
activation plays an essential role in MOFS and is hypothesized to represent the penultimate step in a series of continuous
immuno-inflammatory stimulatory events, including local hypoxia, exposure to bacteria and toxins, and mediator release
from localized areas of inflammation. When infection is the underlying or major contributing process, the diffuse
inflammatory response develops independently of the specific type of microorganism. In noninfectious cases, the response
also appears independent of the specific underlying cause.

235 Invasive hemodynamic monitoring using a Swan-Gantz catheter is essential in the optimal management of patients
in shock or those suffering post-shock sequelae. Which of the following physiologic characteristics are associated with the
various forms of shock?
a. Hypovolemic shock: Decreased pulmonary capillary wedge pressure (PCWP), decreased cardiac output, increased
systemic vascular resistance (SVR)
b. Cardiogenic shock: Increased PCWP, decreased cardiac output, decreased SVR
c. Septic shock (hypodynamic): Decreased cardiac output, increased SVR
d. Neurogenic shock: Decreased PCWP, increased cardiac output, decreased SVR
Answer: a, c

236 Which of the following statement(s) is/are true concerning the relationship between cardiac function and effective
blood volume?

a. A pulmonary capillary wedge pressure of 5–10 rules out fluid overload as a cause of pulmonary edema
b. A shift to the right in the Frank-Starling curve is associated with compromised cardiac function
c. Dilutional anemia may contribute to tachycardia even though blood volume and filling pressures are normal
d. The sole purpose of a pulmonary artery catheter is to measure pulmonary artery pressure and cardiac output
Answer: b, c

Although physical findings are often adequate to establish a diagnosis and institute management of cardiac failure, direct
measurement of filling pressures of the right heart (central venous pressure) or the left heart (pulmonary artery pressure)
may be required. Placement of a pulmonary artery catheter allows us to measure cardiac output by thermodilution and, more
importantly, to sample mixed venous blood for saturation measurements which tell us the ratio between systemic oxygen
delivery and oxygen consumption. From all of these measurements we can determine if cardiac output is normal for the
level of filling pressure of the left ventricle, or if contractility is decreased. In the latter case, cardiac output will be lower
than predicted for a given level of filling pressure. In the Frank-Starling curve, if the patient is to the right of the normal
range, then cardiac function is compromised either because of valvular disease, extrinsic pressure such as pericardial
tamponade, or more commonly, a decrease in contractility. If cardiac function and anatomy are normal, then blood volume,
filling pressure and cardiac function are related to the Starling curve. The intake and output of fluid and salt is autoregulated
to maintain the filling pressure of the left ventricle around 10 mm Hg. Extracellular fluid expansion is usually associated
with normal blood volume. Gross expansion of extracelluar space results in deleterious effects if tissue edema can and often
do exist with perfectly normal blood volume. In other words, a pulmonary capillary wedge pressure of 5–10 does not rule
out fluid overload as a cause of pulmonary or GI dysfunction. In critically ill patients, the fear of hypotension and effect of
perfusion usually results in infusion of intravenous salt and water in quantities which exceed losses. Consequently, most
patients in the Intensive Care Unit have anemia, dilutional hypoproteinemia, and a compensatory increase in cardiac output.
In response to anemia, these patients are tachycardic, even though blood volume is normal, filling pressures are normal, and
total body extracellular fluid is excessive.

237 Which of the following statement(s) is/are true concerning methods of nutritional support?

a. Optimal results for enteral feedings are achieved with approximately half of calories supplied as carbohydrate and
half as fat
b. Diarrhea is the most common complication of enteral feeds and is due to the high osmolarity of the carbohydrate
components
c. The hyperosmolar nature of parenteral fat solutions requires central venous administration
d. Approximately 25–50% of calories should be provided as fat emulsion in patients receiving total parenteral
nutrition
Answer: a, b, d

Most formulas for enteral feeding range from 1.0 to 2 cal/ml and include 3 to 7% protein. Most of the calories are supplied
as glucose or sucrose, so that the solutions have a very high osmolarity. Cramps or diarrhea can result when these high
osmolar solutions are placed into the stomach or intestine. Diarrhea is the major complication with most tube feeding
formulas. Diarrhea can be minimized by the use of starch or fat as an energy source in tube feedings. This can be supplied
as part of the commercial preparation or added in the form of medium chain triglycerides or other oils. The best results are
usually achieved by supplying approximately half the calories as carbohydrate and half as fat. In patients receiving total
parenteral nutrition, energy source is provided as carbohydrate, fat, and amino acid solutions. Parenteral feeding with
carbohydrate is limited by the sclerotic effect of hyperosmolar solutions on veins. Fat is a more efficient energy source and
can be given through peripheral veins in concentrations of either 10 or 20%. Most intensivists favor supplementing standard
total parenteral nutrition solution with intravenous fat to provide at least 100 grams of fat emulsion each week to preclude
fatty acid deficiency. Giving up to 25 to 50% of calories each day as fat emulsion may optimize the delivery of this caloric
delivery.

238 Which of the following statement(s) is/are true concerning the autoregulation necessary to maintain oxygen
consumption and oxygen delivery?

a. A change in oxygen consumption is followed by a proportionate change in oxygen delivery


b. A change in oxygen delivery is followed by a change in oxygen consumption
c. Increases in oxygen delivery are due solely to an increase in cardiac output
d. The normal ratio of oxygen delivery to consumption is 2:1
Answer: a

The relationships between oxygen consumption and oxygen delivery represent one of the most interesting regulation
systems in homeostasis. First of all, if one of the three components of oxygen delivery is abnormal, endogenous
mechanisms regulate the other two until normal oxygen delivery has been restored. The various combinations of
compensatory mechanisms supply adequate oxygen for systemic metabolism through a wide range of variations in oxygen
delivery. When there is a change in oxygen consumption, there is a proportionate change in oxygen delivery, which occurs
almost immediately, mediated completely by a change in cardiac output. Conversely, a primary change in oxygen delivery is
not followed by any change in oxygen consumption. The normal ratio of oxygen delivery to consumption is approximately
5:1.

239 Which of the following statement(s) is/are true concerning O2 venous saturation monitoring?

a. The normal saturation of mixed venous blood is 50%


b. Mixed venous blood obtained for saturation monitoring can be obtained from any peripheral vein
c. If arterial blood is fully saturated, the saturation of mixed venous blood is 80%
d. In less than fully saturated blood, the difference between arterial and venous saturation corresponds to oxygen
extraction
Answer: c, d

The relationship between oxygen delivery and oxygen consumption is reflected in the amount of oxygen in venous blood.
Under normal circumstances, oxygen delivery is 1000 cc/min and oxygen consumption is 200 cc/min. The amount of
oxygen extracted is 20% of that delivered, and 80% of oxygen is still present in venous blood returning to the heart. Usually
arterial blood is fully saturated, and under normal circumstances, the saturation of mixed venous blood (SVO2) will be 80%.
This measurement must be made in mixed venous blood since the relative extraction of organs served by the superior and
inferior vena cava and coronary sinus are quite different. As long as arterial blood is fully saturated, this observation holds
true regardless of the absolute level of oxygen consumption or oxygen delivery. If the arterial blood is less than fully
saturated, the difference between arterial and venous saturation corresponds to the oxygen extraction, hence the oxygen
delivery/oxygen consumption ratio.

240 Which of the following statement(s) is/are true concerning oxygen kinetics in a critically ill, febrile patient?

a. Oxygen consumption will likely exceed three times normal


b. The high cardiac output and pulse rate are designed to increase oxygen delivery
c. The hyperdynamic response may actually increase oxygen delivery to exceed the increase in oxygen consumption
d. The patient can maintain adequate compensation as long as the oxygen delivery/oxygen consumption rate is greater
than 2:1
Answer: b, d

In critically ill patients oxygen consumption may be elevated or depressed, but slight to moderate elevations in oxygen
consumption is the most common abnormality in critically ill patients. Oxygen consumption will be elevated in proportion
to the amount of inflammation. A febrile patient with significant signs of septic toxicity will typically have an oxygen
delivery at 1.5 to 2 times normal. It is very unusual for a critically ill patient to experience oxygen consumption greater than
twice normal. This occurs only in situations of severe muscular exercise such as seizures or tetanus. During
hypermetabolism, a change in oxygen consumption is followed promptly by a proportionate change in oxygen delivery.
Hence, it is “normal” for a hypermetabolic patient to have a high cardiac output and pulse rate. Rarely the hyperdynamic
response exceeds the increase in oxygen consumption, reflected in a ratio higher than 5:1 and venous saturation greater than
80%. Some patients cannot mount an increased oxygen delivery in response to increased oxygen consumption because of
the combination of hypoxemia, anemia, and myocardial failure. If this occurs, then the oxygen delivery/oxygen
consumption ratio will be less than 5:1. The patient will compensate for this by increased oxygen extraction, however, and
the patient will remain stable as long as the ratio is greater than 2:1.

241 Which of the following statement(s) is/are true concerning the treatment of pulmonary interstitial edema?

a. Diuresis and blood transfusion is a valuable step


b. Salt-poor albumin leaks through the capillaries and worsens the condition
c. Mannitol is contraindicated as a diuretic in this clinical situation
d. Isoproterenol is a poor choice as an ionotropic agent
Answer: a
Treatment of pulmonary edema has two important goals, the first is to improve oxygenation if it is impaired, and the second
is to minimize fibrosis and bacterial infection, which often accompany pulmonary edema caused by capillary injury. The
treatment of interstitial edema is to maintain the hydrostatic pressure as low as compatible with adequate cardiac output and
to raise the oncotic pressure selectively in the vascular space. These measures, combined with fluid restriction and diuresis,
will decrease the amount of pulmonary edema. Since it is desirable to maintain filling pressures of the left ventricle as low
as possible while maintaining a good cardiac output, inotropic drugs to improve left ventricular contractility are helpful.
Isoproterenol or dopamine should be used, with serial cardiac output and filling pressure measurements. The first step in
decreasing pulmonary edema is to decrease the pulmonary capillary hydrostatic pressure as low as is compatible with an
adequate cardiac output. This is done by diuresis and fluid restriction. As the patient falls behind in blood volume, signs of
hypovolemia may appear. Blood volume is then replenished with a fluid that stays in the vascular space. Packed red cells
are ideal for this application. When the hematocrit is normal, concentrated salt-poor albumin should be used. This
hyperoncotic fluid replenishes the blood volume by attracting interstitial fluid from throughout the body into the vascular
space and supplements diuresis. This technique is useful even in the septic patient who may have increased capillary
permeability and may loose albumin from the vascular space at a rapid rate. Even if albumin “leaks out”, the short term
effects of expanding blood volume and decrease in edema will appear.

242 Which of the following statement(s) is/are true concerning the pathophysiology of gas exchange?

a. Hypoventilation in relation to perfusion can result in an oxygen saturation of less than 100%
b. Diffusion block and / mismatch can almost completely be overcome by breathing 100% oxygen
c. Transpulmonary shunting does not occur under normal circumstances
d. The normal arterial oxygen saturation should be 100%
Answer: a, b

Under normal conditions, red blood cells in the pulmonary capillaries become fully saturated and oxygen dissolves in
plasma resulting in blood PO2 of 100 and O2 saturation of 100%. This equilibration may be disturbed by hypoventilation in
relationship to the perfusion (/ mismatch), diffusion block caused by interstitial fibrosis, or perfusion of nonventilated
alveoli. Diffusion block and / mismatch can almost be completely overcome by breathing 100% oxygen, hence hypoxemia
during exposure to high alveolar PO2 is caused by total / mismatch, so-called transpulmonary shunting or venous ad
mixture. Under normal circumstances, about 5% of the blood entering the left atrium has been shunted away from the
pulmonary capillaries, either as the result of bronchial nutritive blood flow or through thebesian veins opening directly into
the left side of the heart. This phenomenon, combined with a normal minor / mismatch associated with breathing at rest and
positional changes in pulmonary blood flow, result in the fact that normal arterial PO2 is 90–100 mm Hg and the normal O2
saturation is 98%.

243 Which of the following statement(s) is/are true concerning CO2 transfer in the lung?

a. Carbon dioxide excretion is a direct function of alveolar ventilation


b. Normally end tidal CO2 should be identical to PaCO2
c. The gradient between end tidal and arterial CO2 can be an indirect measure of nonperfused alveoli
d. Positive pressure ventilation under normal airway pressures creates a significant end tidal PaCO2 gradient
Answer: a, b, c

The amount of carbon dioxide excretion is directly related to alveolar ventilation. While oxygenation is a function of
matching blood flow to alveoli, carbon dioxide excretion is a direct function of ventilation or hyperventilation of alveoli
with some blood flow. Normally the end tidal CO2 represents mixed alveolar gas which is at equilibrium with pulmonary
capillary blood, hence with arterial blood. Therefore, the end tidal CO2 and the PaCO2 should be identical. End tidal CO2
measurement is a very useful continuous measurement of PaCO2 which can be used as a monitor when the lung is normal,
as in ventilator weaning. Furthermore, the gradient between end tidal and arterial CO2, when it is large, acts as an indirect
measure of nonperfused alveoli and/or compression volume. In patients who are ventilated with positive pressure
ventilation, a significant end tidal PaCO2 gradient occurs only when peak airway pressures are very high (over 30 cm H2O)
and the compression volume is a significant component of each exhaled breath.

244 Which of the following statement(s) is/are true concerning the assessment of protein reserve?

a. Conventional serum proteins such as albumin and globulin are early indicators of malnutrition
b. The total lymphocyte count reflects immune status and not nutrition
c. Antigen skin testing reflects patient immunity and not nutrition
d. Measurement of urea excretion in urine can be used as a measurement of protein breakdown
Answer: d

Since protein is the functional and structural chemical of the body, most nutritional assessment techniques are estimates of
protein reserves. The actual nitrogen balance can be measured by measuring the amount of nitrogen excreted. This is most
conveniently done by measuring the amount of urea excreted in the urine, assuming that urea constitutes 85% of the total
nitrogen excretion. Knowing nitrogen excretion, the amount of protein catabolized can be estimated and compared with the
amount of protein ingested by the patient. Indirect assessments of protein reserves are based on single measurements of
body substances that are dependent on rapid protein synthesis for maintenance of normal levels. Conventional serum
proteins such as albumin and globulin are not affected by malnutrition until it is very severe. Proteins such as prealbumen
and transferrin, which turn over more rapidly, are better indicators of protein status. Lymphocytes are rapidly destroyed and
protein is required for the formation of new cells. Consequently, the absolute lymphocyte count is a useful measure of the
status of protein reserves. The lymphocyte count is considered by some the best single “static” measurement characterizing
nutritional status. Protein is also required for synthesizing the cells and mediators involved in skin test reactivity. Although
skin test reactivity is a manifestation of lymphocyte-mediated immunity, its usefulness in patient assessment is probably that
of assessment of the inflammatory response than lymphocyte activity per se. Some chronically and acutely malnourished
patients convert from reactive to anergic, and reactivity can be restored by nutritional repletion.

245 Useful steps to optimize systemic oxygen delivery include:

a. Maintaining mean arterial blood pressure between 50 and 90 mm Hg


b. Optimizing PEEP levels by monitoring mixed venous saturation
c. Turning the patient prone
d. Sedation or paralysis
Answer: a, b, c, d

Optimizing systemic oxygen delivery in relationship to oxygen requirement is the primary goal of management. Improving
oxygenation of the blood itself by improving alveolar inflation is only one of the steps in optimizing oxygen delivery.
Equally or more important are treating anemia and optimizing cardiac output. Cardiac output should be optimized to
maintain delivery of four to five times consumption. In general, this means avoiding those factors which decrease cardiac
output, rather than actively trying to increase cardiac output. Blood pressure should be maintained high enough to provide
coronary perfusion (over 50 mm Hg mean pressure) but not so high as to limit left ventricular function (over 90 mm Hg
mean arterial pressure). Alveolar collapse is treated by cleaning the airways, avoiding 100% oxygen, and moving fluid from
the lung or chest, and finally by the use of positive end exploratory pressure to hold open those alveoli which have been
opened by other measures. The optimal level of PEEP is that level that maintains arterial oxygenation but does not decrease
venous return or cardiac output. This optimal level is best determined by monitoring mixed venous saturation. Another step
in optimizing lung function is to take advantage of the gravitational effects on pulmonary blood flow by turning the patient
prone or to a full lateral position to direct blood flow to areas of optimal alveolar function. This step will often result in an
opening in the closed posterior alveoli which have been compressed by the weight of the fluid in the lungs. At the same time
that oxygen delivery is optimized, oxygen consumption should be decreased to normal or even below normal if necessary.
Treating infection, providing adequate sedation, and establishing muscular paralysis decrease oxygen consumption, and
decrease the need for oxygen delivery.

246 Phases of multiorgan failure will include:

a. Generalized increased capillary permeability


b. A hypermetabolic state
c. Organ malfunction
d. All of the above
Answer: d

Clinically the multiple-organ failure patient progresses through well-defined phases. These phases include: Phase 1—a
generalized increased capillary permeability resulting in edema, weight gain, and intravenous volume replacement,
increased protein concentration in urine and lymph. Although the pulmonary microvasculature has been most thoroughly
studied, it is apparent that the lung is simply the most obvious end organ in a generalized permeability defect. Phase 2—A
hypermetabolic state, with increased oxygen consumption and a compensatory increase in oxygen delivery characterized by
tachycardia and high cardiac output. This condition following systemic ischemic and reperfusion is similar to
hypermetabolism following endotoxemia, localized sterile inflammation, and infusion of stress hormones, suggesting a
common mechanism. Phase 3—Organ malfunction due to localized edema and cellular injury, particularly in the kidney,
liver, brain, and host defense system. Hemorrhagic shock predisposes to bacterial translocation and endotoxin absorption
from the intestine. Phase 4—In the absence of systemic sepsis, organs may recover to normalcy or may be irreversibly
damaged, leading to a need for chronic support. If the organ failure phases lead to systemic infection or irreversible tissue
damage in the lung or brain, the death of the entire organ is likely.

247 Which of the following statement(s) is/are true concerning oxygen consumption (O2)?

a. O2 is normally 100–120 cc2/m2/min


b. Resting O2 is controlled by the level of thyroid and catecholamine hormones
c. Under steady state conditions the amount of oxygen consumed exceeds the amount of oxygen taken up by the
pulmonary capillaries
d. O2 is dependent on the status of pulmonary function
Answer: a, b

Oxygen consumed in the process of metabolism is expressed as the volume of oxygen per minute (O2). O2 is normally 100–
120 cc2/m2/min, or 200 cc/min for a typical adult. Resting O2 is a function of metabolizing body cell mass, with fine tuning
control provided by the level of thyroid and catecholamine hormones. O2 decreases under conditions of hypothermia,
paralysis, and hypothyroidism. O2 increases during exercise or muscular activity, hyperthermia, profound hypothalamic
injury, hyperthyroidism, catecholamines, and inflammatory mediators, particularly the interleukin cytokines. Under steady
state conditions, the amount of oxygen consumed in systemic metabolism is exactly equal to the amount of oxygen taken up
by the pulmonary capillaries via the airway. This is true regardless of the status of pulmonary function or dysfunction, so we
measure O2 across the lung and assume that it is exactly the amount consumed in the systemic metabolism.

248 Which of the following statement(s) is/are true concerning the outcome in patients with acute renal failure?

a. Mortality for ischemic acute tubular necrosis without other organ failure is approximately 6%
b. Multiple organ failure complicated with acute renal failure is associated with mortality ranging from 50% to 90%
c. Recovery of renal function after six weeks is unlikely
d. There is no difference in survival between oliguric and nonoliguric renal failure
Answer: a, b, c

Survival of patients with acute renal failure is a function of the successful treatment of the primary disease from which the
renal failure was derived. The mortality for ischemic acute tubular necrosis without organ failure has been reported at
approximately 6%. By contrast, mortality of multiorgan failure complicated by acute renal failure ranges from 50% to 90%.
In patients who survive the acute phase of illness, recovery of renal function after acute renal failure is dependent on the
type and extent of injuries to the renal parenchyma. If renal function is not returned after six weeks, recovery is unlikely.
Nonoliguric renal failure is usually limited in its extent and is almost always reversible.

249 Which of the following statement(s) is/are true concerning oxygen delivery?

a. The amount of oxygen delivered to peripheral tissues is dependent upon the oxygen content in arterial blood and
cardiac output
b. Oxygen content is commonly measured in arterial blood
c. The normal arterial-venous difference is 4 cc O2/dL
d. Normal systemic oxygen delivery for a typical adult is approximately 1000 cc/min
Answer: a, c, d

The amount of oxygen that is delivered to peripheral tissues is the product of the oxygen content in arterial blood times the
cardiac output. Normally, oxygen content of arterial blood is approximately
20 cc/dL, and the normal cardiac index is 5 L/min. Therefore, the normal systemic oxygen delivery is approximately 1000
cc/min. Although oxygen content is the most important measure of oxygen in the blood, PO2 and oxyhemoglobin saturation
is more commonly measured in the Intensive Care Unit, hence it is necessary to convert between these measurements. The
normal oxygen content of venous blood is 16 cc/dL. Hence, the normal arterial-venous difference is 4 cc O2/dL.

250 Which of the following statement(s) is/are true concerning carbon dioxide kinetics?

a. The amount of carbon dioxide produced is equivalent to the amount of oxygen consumed
b. Carbon dioxide levels in blood, present mostly as a bicarbonate ion, can quickly change
c. Normally the amount of carbon dioxide excreted through the lung is exactly equal to the amount of carbon dioxide
produced in peripheral tissues
d. The amount of carbon dioxide excreted is a function of ventilation of perfused alveoli
Answer: a, c, d

The total amount of carbon dioxide produced by systemic metabolism is roughly equivalent to the amount of oxygen
consumed (100–120 cc/m2/min, 200 cc/min in a typical adult). CO2 production is increased or decreased by each of the
factors that causes an increase or decrease in oxygen consumption. Most of the carbon dioxide in blood is present as
bicarbonate ion which cannot be changed quickly. However, the metabolically produced CO2 is mostly present as dissolved
carbon dioxide, added to the blood in the peripheral tissues and excreted in the lung. In a steady state, the amount of carbon
dioxide excreted through the lung is exactly equal to the amount of carbon dioxide produced in peripheral tissues. The
amount of carbon dioxide excreted is a function of ventilation of perfused alveoli (i.e. the alveolar ventilation/min).

251 Which of the following result in a decrease in functional residual capacity?

a. Shallow breathing
b. Partial airway occlusion
c. Absorption atelectasis
d. Hemothorax
Answer: a, b, c, d

A decrease in functional residual capacity is caused by incomplete alveolar inflation related to 1) shallow breathing; 2)
partial or complete airway occlusion, which may be generalized (as in bronchospasm) or localized (as in gastric aspiration);
3) absorption atelectasis, which occurs when oxygen is substituted for nitrogen in the inspired gas; or 4) conditions in which
air or fluid is occupying a potential alveolar space in the chest such as pneumothorax, hemothorax, or pulmonary edema.

252 Which of the following statement(s) meet the criteria for organ failure?

a. Bilirubin greater than 5 mg/dl


b. Creatinine greater than 3 mg/dl
c. Alveoloarterial O2 gradient greater than 300 mm Hg
d. Glasgow Coma score less than 10
Answer: a, b, c, d

Multiple organ failure is defined by dysfunction of two or more of the six vital organ systems: cardiovascular, respiratory,
nervous system, renal, liver, and host defenses.

253 Which of the following statement(s) is/are true concerning pulmonary edema?

a. Pulmonary edema effectively narrows bronchi and increases pulmonary vascular resistance
b. Ventilation and perfusion are decreased equally
c. Positive pressure ventilation improves gas exchange by decreasing lung edema
d. The condition is frequently caused by decreased plasma protein levels
Answer: a

The causes of pulmonary edema are: 1) increased hydrostatic pressure; 2) increased capillary permeability and 3) decreased
plasma oncotic pressure. The latter, however, is rarely a problem unless the concentration of plasma protein is very low.
When fluid begins to collect in the lung interstitium, it migrates to the loose areolar portion of the lung microanatomy that
surround the small bronchioles and pulmonary arteries. The edema in these areas has the effect of narrowing bronchi and
increasing resistance in the pulmonary vasculature. This will decrease both ventilation and perfusion in the edematous area,
but ventilation is often affected more than blood flow, resulting in a decreased / ratio, with all of its attendant effects on gas
exchange. Ventilator treatment of pulmonary edema which increases airway pressure tends to hold the alveoli open,
spreading out the space available for water accumulation and overcomes the effect of small bronchial occlusion. Positive
pressure ventilation does not, therefore, affect the amount of edema in the lung, only its manifestations.

254 Which of the following statement(s) is/are true concerning various causes of acute renal failure?

a. Acute tubular necrosis is the most common pathologic finding of acute renal failure
b. Drug-induced renal failure is compounded in situations of hypovolemia
c. Myoglobin-induced renal failure can be prevented using diuretics and alkalization of urine
d. The incidence of radiographic contrast dye-induced renal failure occurs independent of preexisting conditions
Myoglobin is a direct nephrotoxin
Answer: a, b, c

Acute tubular necrosis results from ischemia to the renal parenchyma and is the most common pathologic finding of acute
renal failure. In conditions of diminishing renal blood flow, perfusion to the kidneys is first maintained by vasomotor
responses which dilate the afferent arteriole and constrict the efferent arteriole. As continued hypotension occurs, the renin-
angiotensin system is activated and vasoconstriction of the afferent arteriole occurs which exacerbates
corticohypoperfusion. Pigment nephropathy is a common cause of acute renal failure occurring after trauma, burns,
operations, or hemodynamic catastrophe. With ischemia or blunt injury to large muscles, myoglobin is released into the
circulation. In the kidney, it is filtered from blood and reabsorbed by the tubule. Although myoglobin is not a direct
nephrotoxin, in the presence of aciduria, myoglobin is converted to ferrihemate, which is toxic to renal cells. Prevention of
myoglobin-induced renal failure may include the use of diuretics and alkalinization of urine. Drug-induced acute renal
failure is responsible for approximately 5% of all cases of acute renal failure. Through normal reabsorption and secretion,
the kidney is exposed to high concentrations of drugs and solutes, which may be toxic. This problem is compounded by
hypovolemia, which causes increased reabsorption of water and solutes and exposes the lumen to even higher
concentrations of toxins. The incidence of radiographic contrast dye induced nephropathy is approximately 1 to 10% and
may be predicted according to a number of risk factors which include contrast load, age, preexisting renal insufficiency, and
diabetes. The incidence in patients with normal renal function is significantly lower at 1% to 2%.

255 The patient requires renal replacement therapy. Which of the following statement(s) is/are true concerning the
differences between hemodialysis and continuous arteriovenous hemodialysis (CAVHD)?

a. Anticoagulation is not required for CAVHD


b. Hemodynamic instability will be a particular problem with both techniques
c. Both techniques will decrease serum urea ni+62trogen levels
d. CAVHD will likely result in better removal of excessive volume
Answer: c, d

256 Which of the following statement(s) is/are true concerning continuous arteriovenous hemofiltration (CAVH)?

a. The technique runs continuously


b. It is not associated with the hemodynamic instability
c. Systemic heparin anticoagulation is necessary
d. Fluid balance and correcting electrolyte abnormalities takes several days
Answer: a, b

Continuous arteriovenous hemofiltration (CAVH) is an extracorporial filtration technique that removes extracellular fluid
across a synthetic membrane via hydrostatic pressure gradient created between the indwelling arterial and venous catheters.
Arteriovenous access is accomplished by percutaneous cannulation of femoral artery and vein with a low incidence of
complications. Although full systemic anticoagulation is not necessary for CAVH, heparinization of the extracorporial
circuit is required. CAVH is run continuously for as many days as renal replacement is required. Experience with CAVH has
demonstrated very little or no hemodynamic instability with treatment of critically ill renal failure patients. The stable nature
of this therapy is attributed to a slow and continuous fluid and solute removal and to the fact that the membrane does not
induce compliment activation when in contact with blood. Fluid balance and serum electrolyte concentrations can be titrated
to any level in a matter of hours by manipulating the composition and rate of replacement solution. Solute clearance with
CAVH is limited by the ultrafiltration and replacement fluid exchange rate. In patients with high urea generation rates,
solute removal with CAVH may be inadequate and variations of the technique may be used to enhance clearance.

257 A 64-year-old diabetic patient develops acute renal failure following an aortic aneurysm repair. Which of the
following statement(s) is/are true concerning his diagnosis and management?

a. Resting energy expenditure will likely be less than would be expected for a patient with normal renal function
b. Maintenance of positive energy balance reduces protein catabolism and makes the management of renal failure
easier
c. Expected metabolic abnormalities include hyperkalemia, hypercalcemia, and metabolic alkalosis
d. A nonoliguric renal failure is usually associated with a better outcome
Answer: b, d

In patients with nonoliguric renal failure, treatment may differ little from that required for identical patients with normal
renal function. Management of fluids, solutes, and nutrition is usually unaffected by nonoliguric renal failure, although
BUN may be elevated. The extent of renal dysfunction is limited and almost always reversible. The use of renal replacement
therapies is rarely necessary. Acute renal failure can result in severe derangements in electrolyte and acid-based physiology.
Of all electrolyte abnormalities that might occur, hyperkalemia is the most serious. Other electrolyte abnormalities such as
hyponatremia, hyperphosphatemia, hypocalcemia, and metabolic acidosis are common and must be monitored carefully.
The metabolic requirements of a patient with acute renal failure are those of a critically ill hospitalized patient. The actual
measurements of resting energy expenditure has shown that caloric requirements for multiorgan failure patients with renal
failure are often 50% above normal, healthy individuals. Although acute renal failure may require fluid restriction,
providing adequate nutrition is an important aspect of their treatment. Positive energy balance may make management of
uremia and hyperkalemia less difficult. By providing adequate calories, endogenous protein catabolism with resultant
generation of urea and release of potassium can be avoided. Maintenance of positive energy balance with glucose and lipids
should reduce protein catabolism, urea generation, and hyperkalemia.

258 Which of the following statement(s) is/are true concerning various energy sources?

a. Carbohydrate is the most efficient source of energy


b. Endogenous fat is the major source of energy during starvation
c. The respiratory quotient of carbohydrate is greater than either fat or protein
d. Ketones can be used as a source of energy during starvation
Answer: b, c, d

The major sources of energy are carbohydrates and fats. Carbohydrates are a major source of energy during normal, non-
starving existence. The brain, the red cells, and some other organs are obligate glucose users. The brain and red cells can
develop the capacity to use ketones as an energy source, a process known as starvation adaptation. Fat is the most efficient
source of energy. Fat produces 9 calories of energy per gram of substrate metabolized while carbohydrate produces only 4
calories. The respiratory quotient represents the number of molecules of carbon dioxide for each molecule of oxygen
consumed. For carbohydrates it is 1.0, whereas for fat, this respiratory quotient is 0.7. Endogenous fat is the major source of
energy during starvation. The glycogen storage is basically depleted after a day of fasting and fat becomes a major energy
source with protein breakdown supplying glucose through the process of gluconeogenesis.

259 Which of the following statement(s) is/are true concerning the treatment of multisystem organ failure?

a. Forced diuresis with negative fluid balance may improve survival and acute respiratory failure
b. The titration of ionotropic drugs based on desired blood pressure optimizes the results.
c. Nutritional support should be withheld for several days until the patient’s condition stabilizes
d. Continuous arteriovenous hemofiltration is preferred to intermittent hemodialysis for most critically ill patients
e. Hepatic failure should be treated specifically with pharmacologic manipulation
Answer: a, d

The important principles in the management of multiple organ failure are to avoid further episodes of local or systemic
ischemia and to keep the brain viable by pharmacologic or mechanical support of the failing organs until organ recovery
occurs. Respiratory failure is treated by mechanical assistance for lung inflation and ventilation and by decreasing lung
edema as much as possible. Airway intubation is usually required. There is now good evidence that forced diuresis and
negative fluid balance is associated with improved survival and acute respiratory failure. Cardiac failure is treated with
inotropic drugs. Although ionotropic drugs are usually titrated to achieve a desired arterial blood pressure, it is more
sensible to titrate ionotropes to achieve a normal oxygen delivery/oxygen consumption ratio. Pulmonary artery pressure and
mixed venous saturation monitoring are essential for intelligent management of the patient with severe respiratory or
cardiac failure. Adequate nutrition is also important for recovery from organ failure. Renal failure is treated by mechanical
substitution of renal function. Although hemodialysis and peritoneal dialysis can serve this purpose, each has a significant
drawback in the critically ill, multiple organ failure patient. Continuous arteriovenous hemofiltration (CAVH) and
continuous arteriovenous hemodialysis (CAVHD) are the methods of choice for renal replacement therapy. Hepatic failure
often occurs as part of the multiple organ failure syndrome but unfortunately there is no specific treatment.

260 Which of the following statement(s) is/are true concerning pulmonary mechanics?

a. The standard compliance or volume pressure curve is measured during lung inflation
b. The decreased compliance in acute respiratory failure occurs because the lung is smaller not stiffer
c. In acute respiratory failure, higher pressures are required to achieve the same level of inflation
d. Areas of normal lungs are more vulnerable to overdistention which may lead to progressive lung dysfunction
Answer: b, c, d

The standard compliance or volume pressure curve is drawn by measuring volume and pressure at stages of lung deflation
after total inflation. The decreased compliance in acute respiratory distress syndrome occurs because the lung is smaller, not
stiffer. In acute respiratory failure, the cause of decreased compliance is almost always associated with a decrease in
functional residual capacity (FRC). The decreased FRC represents lost alveoli which are either collapsed or filled with fluid
but still perfused with blood. Because the lung is smaller, the compliance curve has shifted to the right and much higher
pressures are required to achieve the same level of inflation. Lung damage can be caused by high airway pressure, so that
overdistension is not merely inefficient but actually detrimental. Since the most normal areas of lung have the best
compliance, they are most vulnerable to overdistension, contributing to the steady progression of lung dysfunction in
patients ventilated at high peak pressure.

261 Which of the following statement(s) is/are true concerning the use of a ventilator in the treatment of respiratory
failure?

a. The assist-control mode is appropriate in the paralyzed patient


b. Peak inspiratory pressure should be optimized at a level in excess of 40 cm H2O
c. A patient receiving excessive carbohydrate as a nutritional support may have an elevated minute ventilation and
may tire with spontaneous breathing
d. In general, weaning requires an adequate inspiratory force, vital capacity, and a minute ventilation less than 10
L/min
Answer: c, d

Most intensivists favor setting the ventilator on the assist-control mode at a low sensitivity. In this fashion, the patient
breathes at a rate that regulates the PaCO2 to normal, but each breath is mechanically assisted, providing maximal inflation.
The volume of each breath is set by limiting the maximal pressure or maximal volume of each breath. Whichever method is
used, the peak inspiratory pressure should not generally exceed 40 cm H2O. If the patient is comatose or paralyzed, the
assist mode cannot be used and the rate is set in addition to the volume.
Adequate weaning indices are: inspiratory force greater than 20 cm H2O, vital capacity twice the tidal volume, adequate gas
exchange at assisted ventilation at FiO2 of 0.3 and 5 cm H2O of PEEP, and minute ventilation less than 10 L/min. If the
patient is hypermetabolic or is receiving excessive carbohydrate as nutritional support, the minute ventilation will be
elevated, even during assisted mechanical ventilation. If this is the case, the patient will tire rapidly on spontaneous
breathing.

262 Which of the following statement(s) is/are true concerning the estimation and measurement of energy requirements
in the critically ill patient?

a. One can only estimate energy expenditure with actual measurement not technically possible
b. The amount of oxygen absorbed through the lungs is equal to the amount of oxygen consumed by metabolic
processes
c. Metabolic rate, normalized to body surface area, may underestimate metabolism in a fat person
d. To convert cc’s of oxygen per minute to calories per day, a conversion factor of 10 kcal of energy per liter of
oxygen should be used
Answer: b, c

The actual metabolic rate of any patient can be estimated from the predicted basal rate according to the clinical situation.
The amount of energy is most conveniently expressed in calories/day. The metabolic rate is normalized to body surface area;
however, the actively metabolizing tissue is the lean body cell mass. Consequently, reporting “per square meter”
underestimates metabolism in a fat person and overestimates in a very lean person. Although most of studies on nutrition in
critical illness have been based on estimated energy expenditure, actual measurement is much more accurate and has
become an important aspect of critical care management. The most commonly used method of measurement is indirect
calorimetry. In this method, the amount of oxygen absorbed across the lungs into the pulmonary blood is measured over a
given period of time. Assuming the patient is at a metabolic steady state during this time, the amount of oxygen absorbed
across the lungs is equal to the amount of oxygen consumed in the metabolic process. The metabolic rate, measured in cubic
centimeters of oxygen/minute, can be converted to calories/hour or /day if the oxygenated substrates are known. For
practical purposes, a conversion factor of 5 kcal of energy/liter of oxygen consumed is a reasonable approximation.

263 Which of the following statement(s) is/are true concerning the response to a decrease in functional residual
capacity percent (FRC)?

a. Supplying supplemental oxygen will always improve the situation


b. Respiratory alkalosis may occur
c. Decreasing compliance is a common occurrence
d. Respiratory rate and depth of breathing generally decrease
Answer: b, c

Pulmonary arterial spasm in response to local hypoxia autoregulates pulmonary blood flow and maintains adequate gas
exchange during alveolar collapse—up to a point. However, when the loss in ventilation exceeds the decrease in perfusion,
a ventilation-perfusion mismatch occurs, which results in incomplete oxygenation of blood perfusing that area of the lung.
The resultant hypoxemia stimulates an increased rate and depth of breathing which may serve to reexpand the person’s
inflated area of lung. If it does not, hypoxemia will continue but increased ventilation in other areas of the lung will result in
excess CO2 excretion, hypocapnea and respiratory alkalosis. The blood gas picture, hypoxemia with respiratory alklalosis,
is the most common abnormality of gas exchange in surgical patients and it is a hallmark of ventilation-perfusion
imbalance. Oxygenation of blood in the poorly ventilated area of lung can be improved by increasing concentration of
oxygen in the inspired gas. The use of supplemental oxygen, however, treats the symptom rather than the basic cause and
may actually make the problem worse by adding to absorption atelectasis, depriving the poorly ventilated area of nitrogen to
hold alveoli open. This may result in total alveolar collapse. In this circumstance, blood perfusing the nonventilated area
will mix with blood from other areas of the lung, resulting in hypoxemia that does not improve significantly in response of
administration of oxygen. Aside from the effects on gas exchange, loss of alveolar space results in changes in the volume-
pressure relationships in the lung. A decrease in functional residual capacity always results in a shift in the volume-pressure
relationship toward a condition of decreasing compliance.

264 Which of the following statement(s) is/are correct concerning the body fluid compartments?

a. Both the extracellular and intracellular components of total body water can be directly measured
b. The intravascular space accounts for the majority of extracellular fluid
c. All water in the interstitial space is freely exchangeable
d. Transcellular fluid, separated from other compartments by both endothelial and epithelial barriers, constitute about
4% of total body water
Answer: d

Total body water (TBW) is distributed within the intracellular and extracellular compartments. Intracellular fluid cannot be
measured directly but is calculated as the difference between TBW and the measured extracellular water. Extracellular fluid
can be measured directly. The extracellular fluid compartment can be further simplified into the intravascular and interstitial
spaces. Intravascular space, which accounts for 20% of the extracellular fluid, contains the plasma volume which is
approximately 8% of TBW or 5% of body weight. The interstitial space extends from the blood vessels to the cells
themselves and includes the complex ground substance making up the acellular matrix of tissue. Although the water within
the space is thought to be freely exchangeable, this water exists in two phases. The free phase contains water that is
generally freely exchangeable and in a constant state of flux. The bound or gel phase is composed of water that is closely
associated with glycosaminoglycans, mucopolysaccharides, and other matrix components. This water is much less freely
exchangeable. An additional extracellular fluid compartment, the transcellular compartment, consists of water that is poorly
exchangeable under normal circumstances. This fluid is separated from other compartments by both endothelial and
epithelial barriers and includes cerebrospinal fluid, synovial fluid, water within cartilage and bone, fluids of the eye, and the
lubricating fluids of the serous membranes. Together, these fluids constitute about 4% of TBW.

265 Which of the following statements(s) is/are true concerning metabolic alkalosis?

a. Either increased extracellular bicarbonate concentration or inhibited renal excretion of bicarbonate can cause
metabolic alkalosis
b. In metabolic alkalosis secondary to prolonged gastric outlet obstruction, the urine pH is usually acidic
c. Hypokalemia can lead to metabolic alkalosis
d. The respiration compensatory mechanisms for metabolic alkalosis are quite ineffective
Answer: b, c, d

Sustained metabolic alkalosis occurs only if extracellular bicarbonate concentration is increased and renal excretion of
excess bicarbonate is inhibited. Alone, neither is sufficient to result in metabolic alkalosis. Extracellular bicarbonate
concentration is increased by numerous mechanisms. Loss of HCl is the leading cause of metabolic alkalosis in surgical
patients. External loss of gastric acid results in net gain in bicarbonate, which causes metabolic alkalosis. Although the
kidney can excrete excess bicarbonate, this must be accompanied by excretion of sodium. Renal excretion of sodium is
limited in the face of volume depletion, which also occurs with external losses of gastric secretion. As volume depletion
progresses, sodium is conserved in exchange for hydrogen. Thus, in metabolic alkalosis secondary to prolonged gastric
outlet obstruction, the urine, although initially alkalotic, becomes paradoxically acidotic in prolonged or uncorrected cases.
Hypokalemia and cellular exchange of potassium for hydrogen can also lead to metabolic alkalosis. Hypokalemia results in
enhanced proximal tubular bicarbonate reabsorption and distal tubular acid secretion. The major compensatory mechanism
in metabolic alkalosis is respiratory, since the presence of metabolic alkalosis implies renal dysfunction in either generating
or failing to excrete increased amounts of bicarbonate. Hypoventilation is limited by the development of hypoxemia, which
stimulates ventilation. Among the four major types of acid-base disorders, this compensatory mechanism is the least
effective.

266 Which of the following statement(s) is/are true concerning respiratory acidosis?

a. Respiratory acidosis is associated with chronic pulmonary disease far more commonly than is hypoxemia
b. The initial buffering effect occurs at the cellular level
c. Renal compensation occurs within 24 hours
d. Correction of hypoxemia in patients with chronic lung disease may worsen respiratory acidosis
Answer: b, d

Respiratory acidosis, the decrease in extracellular pH from a primary increase in PCO2, is due to inadequate ventilation.
Although pulmonary disease commonly causes hypoxemia, respiratory acidosis is far less common, since defusion of O2 is
more readily impaired than diffusion of CO2. Increased PCO2 results in increased H2CO3 which disassociates into H+ and
HCO3–. Cellular exchange of Na+ and K+ for H+ allows the reaction to continue in this direction with increased
extracellular bicarbonate. This tissue buffering is accomplished within minutes. Persistently elevated PCO2 also stimulates
increased renal acid excretion. Full renal compensation occurs over 3 to 5 days. The treatment of chronic compensated
respiratory acidosis may be complicated by accompanying hypoxemia. In chronic hypercapnia, the chemical
chemoreceptors may be insensitive and the accompanying hypoxemia may supply the main respiratory drive through the
stimulation of peripheral chemoreceptors. In such patients, complete correction of hypoxemia may further depress
respiration and worsen the respiratory acidosis.

267 Which of the following(s) is/are true concerning the control of the volume of body water?

a. Osmoreceptors and baroreceptors work equally to control fluid balance during normal conditions
b. The cardiac atrium regulates volume only by means of its sympathetic and parasympathetic connections
c. The kidney is the primary effector organ in controlling water balance
d. The conversion of angiotensin I to angiotensin II is dependent on the amount of the enzyme, renin, available
e. Nitric oxide plays a number of important roles in regulation of renal hemodynamics
Answer: c, d, e

Changes in volume are detected both by osmoreceptors, which detect changes in plasma osmolality and baroreceptors,
which are sensitive to changes in pressure. The osmoreceptors are responsible for day-to-day fine-tuning of volume whereas
the baroreceptors contribute relatively little to the control of fluid balance under normal conditions. Changes in effective
circulating volume are sensed by the volume receptors of the intrathoracic capacitance vessels and atria, the pressure
receptors of the aortic arch and carotid arteries, the intrarenal baroreceptors, and, to a lesser extent, by the hepatic and
cerebrospinal volume receptors. These baroreceptors control volume by means of sympathetic and parasympathetic
connections. The atria also appear to serve as endocrine organs capable of directing responses to volume changes with the
elaboration of the hormone, atrial natriuretic peptide. The major hormonal mediator of baroreceptor modulation of volume
control is the renin-angiotensin system. The end result of this complex system of receptors or messengers is a change in
sodium and water balance mediated by the kidneys. It is through changes in sodium and water reabsorption that volume and
pressure ultimately normalize. Renin is a proteolytic enzyme that is released in response to changes in arterial pressure,
changes in delivery of sodium to the macula densa of the distal convoluted renal tubule, increases in beta adrenergic activity
and increases in cellular cAMP. Renin cleaves angiotensin I from circulating angiotensinogen. Angiotensinogen is abundant,
so this reaction is enzyme dependent rather than substrate dependent. Angiotensin I is further cleaved to angiotensin II
which acts with locally and systemically to increase vascular tone. Angiotensin II affects sodium reabsorption by decreasing
renal plasma flow and the glomerular filtration coefficient. Finally, angiotensin II increases sodium reabsorption by direct
tubular action as well as by stimulation of aldosterone release from the adrenal cortex.
The importance of nitric oxide and its many biologic functions has recently been recognized. Nitric oxide participates in the
regulation of renal hemodynamics and renal handling of water and electrolytes.

268 Which of the following statement(s) is/are true concerning maintenance intravenous fluid therapy?

a. The total daily water requirement for a 70-kg man is about 2500 mL/day
b. Normal maintenance IV therapy requires administration of sodium, potassium, calcium, phosphate, and magnesium
c. Fluid volume calculations for elderly patients generally are decreased compared to their younger counterparts
d. A child requires a lesser amount of maintenance fluid per kilogram than a larger individual
Answer: a, c

Maintenance fluid replacement is aimed at replacing fluids normally lost during the course of a day. Calculation of
maintenance fluid replacement does not include replacement of preexisting deficits or ongoing additional losses. Formulas
exist for calculating maintenance fluid requirements which adjust for differences in body weight and for changes in TBW
content. A smaller (or younger) individual who has a high percentage of TBW in relation to body weight requires a greater
amount of maintenance fluid per kilogram than a larger individual. The total daily water requirement for a 70-kg man is
about 2500 mL/day. Because hypervolemia is poorly tolerated in older individuals and in patients with cardiac disease, the
volume calculated is generally diminished in this age group. Normal maintenance therapy requires administration of sodium
and potassium. Replacement of calcium, phosphate or magnesium are generally not necessary in patients requiring short-
term therapy. In critically ill patients, however, critical deficits in these electrolytes may occur and must be replaced.

269 Which of the following statement(s) is/are true concerning abnormalities in calcium concentration?

a. Parathyroid hormone affects calcium homeostasis only at the exchange of calcium between bone and extracellular
fluid
b. About 45% of total plasma calcium is in the ionized state and is responsible for most physiologic actions
c. Changes in plasma protein levels or pH can alter the proportion of calcium in the ionized state
d. Intravenous normal saline administration is the first step in treatment of hypercalcemia
e. Classic signs of hypocalcemia include hyperactive deep tendon refluxes, Cvostek’s sign and Trousseau’s sign
Answer: b, c, d, e

Calcium is a divalent cation found in abundance in the human body. About 99% of total body calcium is located in bone in
the form of hydroxyapatite crystals. Calcium homeostasis depends on the exchange of calcium between bone and
extracellular fluid, renal excretion, and intestinal absorption. These three processes are controlled to a great extent by
parathyroid hormone. In extracellular fluid, calcium exists in three forms: ionized calcium, non-ionized calcium, and
protein-bound calcium. Ionized calcium, which comprises about 45% of total calcium is responsible for most physiologic
actions of calcium in the body, and its level is tightly controlled by a regulatory mechanisms. Some nonionized calcium is
complexed with non-protein anions, including phosphate and citrate, and does not easily disassociate. These molecular
forms make up only about 15% of total calcium present in plasma. About 40% of extracellular nonionized calcium is bound
to proteins, with most being bound to albumin. Changes in either plasma protein levels or pH can alter the proportion of
calcium in the ionized state. The most common cause of hypercalcemia is primary hyperparathyroidism. Hypercalcemia can
also occur secondary to malignant disease, caused either by a metastasis to bone or by autonomous tumor secretion of
hormone-like substances that alter calcium homeostasis. Neuromuscular effects may be the earliest manifestations and
include muscle fatigue, weakness, personality disorders, psychosis, confusion, and coma. Elevation of total serum calcium
concentrations to greater than 14mg/dL requires prompt treatment to prevent any serious and potentially lethal
complications. Immediate measures are directed toward maximizing renal excretion of calcium. Vigorous hydration with
0.9% saline solution to prompt diuresis should be the initial step in treatment. The addition of potassium to the resuscitation
fluid as well as the use of furosemide can also be used for treatment.
Serum calcium levels below 8 mg/dL may be associated with symptoms and signs that are primary manifestations of
neuromuscular abnormalities. These include muscle cramps, perioral tingling, parastesias, laryngeal stridor, tetany, seizures
and psychotic behavior. Classic signs of hypocalcemia include hyperactive, deep tendon reflexes, Cvostek’s sign, and
Trousseau’s sign. Symptomatic hypocalcemia is best treated with intravenous infusion of calcium in the form of calcium
gluconate or calcium chloride.

270 Which of the following statement(s) is/are true concerning total body water?

a. Total body water in men represents a higher percent body weight than in women
b. In infants, water comprises up to 80% of body weight
c. Total body water content decreases with increasing age
d. Total body water is equally distributed within the intra-and extracellular compartments
Answer: a, b, c

The total volume of water within the body is termed total body water. The relationship between total body water (TBW) and
body weight is relatively consistent for any given individual and depends on the amount of fat within the body. Because fat
contains little water, TBW as a percentage of body weight decreases with increasing body fat. The estimated TBW in men is
60% of body weight, whereas in women, who typically have more adipose tissue, the average TBW is 50% of body weight.
The percentage of body weight accounted for by water also varies with age. In infants, water comprises about 80% of body
weight. Throughout adult life, a gradual decrease occurs in TBW content because of the amount of fat within the body
usually increases with age. In obese patients, estimates of TBW should be decreased by 10% to 20% whereas in lean
patients, estimates should be increased by about 10%.
TBW is distributed within the intra and extracellular compartments. Intracellular fluid makes up about 2/3 of the TBW, or
40% of body weight.

271 Which of the following statement(s) is/are true concerning the clinical presentation and treatment of severe
metabolic alkalosis?

a. In most cases clinical signs are obvious


b. Correction of potassium and volume depletion corrects most cases of metabolic alkalosis
c. Acetazolamide can enhance renal excretion of bicarbonate
d. Acid replacement should be provided at a molar equivalent basis for excess serum bicarbonate
Answer: b, c

Clinical signs of metabolic alkalosis may not be prominent, since the condition usually develops relatively slowly.
Correction of the underlying cause is the mainstay of treatment in this disorder. In general, correction of potassium and
volume depletion corrects the metabolic alkalosis. In patients without intravascular volume deficits, renal excretion of
bicarbonate can be enhanced by administration of the carbonic acid anhydrase inhibitor acetazolamide. If renal excretion of
bicarbonate cannot be increased because of underlying renal insufficiency or if the metabolic alkalosis is severe, acid may
be administered to directly titrate the excess extracellular bicarbonate. Acids that can be used include ammonium chloride,
arginine hydrochloride, or dilute hydrochloric acid. Partial correction of alkalosis is the initial goal. A general guide is that
2.2 mEq/kg decreases serum bicarbonate by about 5 mEq/L.

272 Which of the following statement(s) is/are true concerning abnormalities in serum sodium?
a. The most common cause of hyponatremia is a deficit in total body sodium
b. Hyponatremia can occur in situations of excessive solute
c. Most surgical patients with hyponatremia are best treated by free water restriction
d. Central nervous system effects are the predominant symptom of hypernatremia
e. Hypernatremia should be rapidly corrected with free water administration
Answer: b, c, d

The most common cause of hyponatremia is an excess of free water rather than a deficit of total body sodium.
Hyponatremia is frequently seen in the postoperative or post-trauma patients because increased ADH secretion acts on the
collecting tubules of the kidney to increase free water reabsorption. Although hyponatremia most often results from excess
free water, it can occur in the presence of excess solute. In this situation, TBW content is either normal or diminished but
the plasma osmolality is increased. An example of this hyperosmolar-hyponatremic state is untreated hyperglycemia. Excess
solute may also be due to exogenous administration or ingestion of mannitol, ethanol, methanol, or ethylene glycol. Most
surgical patients with hyponatremia are euvolemic or hypervolemic. Such patients, if asymptomatic, are best treated by free
water restriction, since free water overload is the cause of the condition. Hypernatremia is a less common problem in
surgical patients than hyponatremia and is usually the result of excess free water loss associated with hypovolemia.
Hypernatremia may also be secondary to increased total body content of sodium, which is usually related to exogenous
administration of sodium. The symptoms of hypernatremia are related to the hyperosmolar state. CNS effects predominate
because of cellular dehydration as water passes into the extracellular space. Once hypernatremia becomes symptomatic, it is
associated with significant morbidity and mortality. Prompt treatment of hypernatremia is essential. Rapid correction,
however, of hypernatremia is associated with significant risk of cerebral edema and herniation. Because chronic
hypernatremia is relatively well tolerated, there are few advantages to rapidly correcting the free water deficit. Moderate
degrees of hypernatremia are tolerated well, and symptoms rarely develop unless serum sodium levels exceed 160
mEq/liter. The development of symptoms also depends on the rapidity at which hypernatremia develops.

273 Which of the following statement(s) is/are true concerning abnormalities in serum potassium?

a. Hyperkalemia can occur in the otherwise normal surgical patient due to excessive intravenous potassium
administration
b. The primary EKG change associated with severe hyperkalemia is peaked T-waves
c. Temporary treatment of hyperkalemia includes administration of calcium, sodium bicarbonate, or glucose and
insulin
d. Alterations in membrane potentials reflected in cardiac and skeletal muscle are common results of both hypo-and
hyperkalemia
e. A reduction in serum potassium of 1mEq/liter requires replacement of 40mEq of potassium
Answer: c, d

Potassium is the major intracellular cation and is a major determinant of intracellular osmolality. Because of the large
differences between intracellular and extracellular potassium concentrations, a transmembrane potential is generated.
Alterations in potassium concentration gradient (both hyper- and hypokalemia) have profound effects on transmembrane
potential and consequently on cellular function. This is especially true for cardiac, skeletal, and smooth muscle.
Extracellular potassium concentration is primarily determined by renal excretion. About 90% of ingested potassium is
secreted by the urine. Hyperkalemia therefore rarely develops from excessive potassium intake in the absence of renal
insufficiency, since the capacity for renal potassium excretion is large. In the surgical patient, diminished renal function is
perhaps the most common problem leading to hyperkalemia. Both chronic and acute renal failure result in the deficit in
potassium excretion. Hyperkalemia can also be associated with cellular disruption, such as with crush injuries or lysed
erythrocytes in large hematomas or after massive blood transfusion. The clinical manifestations of hyperkalemia are
primarily related to membrane depolarization. The most life-threatening manifestations are related to the cardiac effects of
membrane depolarization. Mild hyperkalemia results in peaked T-waves on the EKG and may cause parethesia and
weakness. More severe forms of hyperkalemia cause flattened P-waves, prolongation of the QRS complex, and deep S-
waves on EKG. Ventricular fibrillation and cardiac arrest may follow. Severe hyperkalemia with EKG abnormalities
requires urgent treatment. Rapid infusion of 10% to 20% calcium gluconate may reduce the effects of hyperkalemia on
membrane potentials. Administration of sodium bicarbonate is another temporary measure. The increase in serum sodium
antagonizes the effects of hyperkalemia on the membrane potential, whereas the increase in extracellular pH shifts
potassium into the cells. Movement of potassium into the intracellular compartment can also be achieved by giving insulin
and glucose.
Hypokalemia is usually caused by total body potassium depletion secondary to the decreased potassium intake, increased
extra-renal potassium losses, or increased renal potassium losses. Decreased serum potassium levels may also be secondary
to redistribution of potassium into the intracellular space. Symptoms of hypokalemia, like those of hyperkalemia, are
manifested by disturbances in membrane potentials. As potassium levels fall below 2.5mEq/L, muscle weakness is common.
The primary treatment of hypokalemia is potassium replacement. The route and rate of potassium replacement depends on
the presence and severity of symptoms. A reduction in serum potassium of 1mEq/L represents a total body potassium
deficiency of 100 to 200 mEq.

274 Which of the following statement(s) is/are true concerning the derangement of metabolic acidosis?

a. A major source of acid production of the body is sulfuric acid


b. Excessive loss of bicarbonate can occur with intestinal or pancreatic fistulas
c. Ketoacidosis can occur in conditions of either hyper-or hypoglycemia
d. Lactic acidosis is present when serum lactate concentration is > 2 mEq/L
e. Lactic acidosis can be associated with ethanol toxicity
Answer: a, b, c, e

Most clinically significant metabolic acidosis is related to the net loss of bicarbonate, which occurs when consumption due
to either loss or titration is greater than bicarbonate generation. Under normal circumstances of ingestion of the average
amount of protein in an American diet, about 70 mEq acid is generated daily. The major source of acid production is sulfuric
acid from the metabolism of sulphur-containing amino acids. Increased protein intake and tissue catabolism resulting in
greater metabolism of sulphur containing amino acids may lead to a generation of increased amounts of sulfuric acid. This
excess acid utilizes excess bicarbonate for neutralization. Diarrhea, intestinal or pancreatic fistulas, and burns can cause
metabolic acidosis secondary to loss of bicarbonate. The two most common types of organic acidosis are ketoacidosis and
lactic acidosis. The abnormality primarily responsible for ketoacidosis is deficiency of insulin whether primary, as in
diabetic ketoacidosis, or secondary to hypoglycemia. Under normal conditions a small amount of ketoacids is produced.
During prolonged starvation, production of ketoacids increase to modest levels, providing an important source of energy to
nonhepatic tissues, particularly the brain. In ketoacidosis, the ketoacid production is excessive because of insulin deficiency.
In diabetic acidosis, insulin deficiency also contributes to hyperglycemia by decreasing the metabolism of glucose by
extrahepatic tissue and increasing hepatic production of glucose.
Lactic acidosis can be divided into type A, caused by tissue hypoxia, and type B, caused by other mechanisms. Hypoxia, the
most common cause of lactic acidosis, impairs the mitochondrial oxidation of NADH to NAD that is necessary for
glycolysis. Normal serum lactate concentration is below 2 mEq/L. Lactate acidosis is secondary to hypoxemia, usually due
to an increased production of lactate as well as decreased use, and serum lactate concentrations greater than 6 mEq/L. The
most common cause of type B lactate acidosis is ethanol intoxication.

275 Which of the following statement(s) is/are true concerning the osmotic activity of body fluids?

a. Urea contributes to the osmolality of a solution but not its tonicity


b. The osmolality of the body remains fairly constant at approximately 289 mOsm/kg H2O
c. The two primary regulators of water balance are antidiuretic hormone and aldosterone
d. Serum sodium is the most valuable laboratory indicator of abnormal total body water content
Answer: a, b, d

Body fluids are aqueous solutions composed primarily of water and contained in different compartments of the body. The
movement of water from these compartments depends on a number of physical properties, the most important of which is
osmosis. According to the principles of osmosis, if two solutions are separated by semipermeable membrane, water moves
across the membrane to equalize the concentration of the osmotically active particles. The osmotic activity across a
semipermeable membrane is determined by the concentration of solutes on each side of the membrane. The body is capable
of fine regulation of solute and water concentrations, so that osmolality remains fairly constant at an average of 289
mOsm/kg H2O. In response to small changes in cell volume, osmoreceptors in the paraventricular and supraoptic nuclei of
the hypothalamus send signals to the neuronal centers that control the two primary regulators of water balance, thirst and
antidiuretic hormone secretion. Changes in TBW are reflected by changes in extracellular solute concentration. Because
sodium is the primary extracellular cation and potassium is the predominant intracellular cation, the serum sodium
approximates the sum of the exchangeable total body sodium and exchangeable total body potassium divided by the TBW.
Because total body solute content remains relatively stable over time, changes in TBW content result in inversely
proportional changes in serum sodium. Thus, abnormalities in serum sodium are the indication of abnormal TBW content.
In contrast to impermeable solutes that are excluded from the intracellular space, such as sodium, permeable solutes such as
urea can freely cross the cell membranes. Although urea contributes to the osmolality of a solution, it has no effect on
tonicity because it distributes equally across membranes, and as such does not contribute to the osmols that affects cell
volume.

276 Which of the following statement(s) is/are true concerning the compensatory mechanisms and treatment of
metabolic acidosis?

a. Maximal renal compensation for metabolic acidosis occurs before full respiratory compensation can occur
b. All patients with lactic acidosis should receive prompt treatment with bicarbonate
c. Potassium replacement is essential even in the face of normal or high serum potassium when treating diabetic
ketoacidosis
d. Sodium bicarbonate administration should begin simultaneous with volume resuscitation in patients with hypoxia
secondary to shock
Answer: c

The kidney is extremely sensitive to changes in serum bicarbonate concentration and responds by increasing net acid
excretion primarily by increasing ammonia excretion. Maximal renal compensation requires 2 to 4 days. Delay in achieving
maximal renal response to an increased acid load causes blood pH to decline, which stimulates hyperventilation. Although
effective in promptly raising blood pH, ventilatory compensation is only partial, and full respiration compensation requires
12 to 24 hours. The major principal of treatment for mild to moderate acute metabolic acidosis is correction of the
underlying cause. In surgical and trauma patients, metabolic acidosis is often the result of hypoxia secondary to inadequate
tissue perfusion and subsequent lactic acidosis. Volume and/or blood resuscitation alone may be enough to correct the
acidosis. Attempts to correct acidosis with exogenous bicarbonate before correction of inadequate tissue perfusion are
usually unsuccessful. The use of bicarbonate for the treatment of lactic acidosis is controversial at best. In several studies
the use of bicarbonate in patients with lactic acidosis does not improve clinical parameters or outcome. The correction of
both acidosis and hypoglycemia of diabetic ketoacidosis is best achieved by the administration of insulin. Volume
resuscitation is also required. Potassium replacement is essential, even in the face of normal or high serum potassium, and
as hypokalemia develops as acidosis in hyperglycemia are corrected.

277 Which of the following statement(s) is/are true concern renal tubular acidosis?

a. Renal tubular acidosis is primarily caused by reduction in ammonia excretion


b. The renal tubular defect in renal tubular acidosis can either be at the distal or proximal renal tubule
c. In distal renal tubular acidosis associated with hyperkalemia, the defect involves increased tubular permeability
with backleak of secreted sodium and potassium into the tubular cell
d. Uremic acidosis occurs independently of protein intake
Answer: a, b

The impaired ability of the kidney to excrete acid and hence generate bicarbonate may be secondary to a decrease in the
number of functioning nephrons and is termed uremic acidosis or renal tubular acidosis. Renal tubular acidosis, which can
occur both in acute and chronic renal failure, is primarily caused by reduction in ammonia excretion secondary to a
reduction in the number of functioning proximal tubular cells. In addition, decreased proximal tubular bicarbonate
reabsorption contributes to the development of acidosis. Although the onset of uremic acidosis is related to declining renal
function, its appearance may be influenced by diet-dependent protein and organic anion ingestion. Renal tubular acidosis
may be classified as distal or proximal, depending on the primary site of the renal tubular defect leading to acidosis. In renal
tubular acidosis with hyperkalemia, the mechanism is decreased luminal negativity secondary to impaired sodium
reabsorption. In distal renal tubular acidosis with hypokalemia, mechanisms including increased tubular permeability with
backleak of secreted H+ into the tubular cell and reduced H+ pump activity are proposed mechanisms.

278 Which of the following statement(s) is/are true concerning the postoperative fluid management in a surgical
patient?

a. Standard formulas are available that essentially can direct the therapy for all patients
b. Isotonic solutions containing potassium should be used throughout the entire postoperative period
c. Urine output should be maintained at a level greater than 0.5 ml/kg/h
d. A urine specific gravity of greater than 1.012 may indicate that the patient is dehydrated
Answer: c, d

Fluid therapy during the postoperative period should be tailored to each patient and depends on the adequacy of patient’s
volume status at the completion of the operative procedure, as well as ongoing fluid losses. Maintenance fluid should be
supplemented by replacement of the additional fluids needed to replace the ongoing third space loss as well as losses from
various tubes and drains. In general, isotonic solution should be used for volume resuscitation during the early postoperative
period. It is best not to give potassium supplements during this period unless they are specifically required as indicated by
serum electrolyte measurements. Monitoring fluid status during the postoperative period is best accomplished by careful
monitoring of vital signs, urine output, and central venous pressure, if necessary. Urine output is maintained at a level
greater than 0.5 mL/kg/h. A urine specific gravity of greater than 1.010 to 1.012 indicates that urine is being concentrated
and the patient may not be receiving adequate hydration.

279 Which of the following statement(s) is/are true concerning parenternal electrolyte solutions?

a. Lactated Ringer’s solution contains physiologic concentrations of all important electrolytes


b. Glucose is added to hypotonic saline solutions to increase their tonicity
c. About 1/2 of all exogenously administered albumin ends up in the extravascular space
d. Normal saline solution provides excessive sodium and chloride which may lead to body sodium overload
Answer: b, c, d

A number of electrolytes solutions are available for parenteral administration. Lactated Ringer’s solution is a physiologic
solution containing many of the electrolytes found in plasma. The disadvantage of this solution is the relatively low sodium
content (130 mEq/L) as compared to plasma. Hyponatremia can occur with extended use of lactated Ringer’s solution.
Isotonic saline (0.9% or normal saline) contains 154 mEq of both sodium and chloride. The excess of both sodium and
chloride can lead to electrolyte and acid-base disturbances. Infusion of large volumes of 0.9% saline can lead to total body
sodium overload and hyperchloremia. The less-concentrated saline solutions are hypo-osmotic and have excess free water.
In addition, 0.2% saline solution is hypotonic with respect to plasma and can result in red blood cell lysis if rapidly infused.
For this reason, 5% dextrose is added to these solutions to increase the tonicity. Plasma expanders are commonly used in
surgical patients. Plasma protein solutions such as 5% and 25% albumin act initially by increasing plasma oncotic pressures.
Abnormalities in microvascular permeability such as those found in the pulmonary circulation in adult respiratory distress
syndrome, in regional circulatory bed burns or infections, and in the systemic circulation in sepsis, may result in
extravasation of these proteins into the interstitial space. About half of all exogenously administered albumin eventually
ends up in the extravascular space. The half life of exogenously administered albumin is about 11 days.

280 An 11-year-old boy has experienced severe diarrhea for 10 days. He presents with decreased skin tungor, sunken
eyes, orthostatic hypotension, and tachycardia. Which of the following statement(s) may be true concerning his diagnosis
and treatment?

a. His hematocrit will likely be elevated


b. His BUN may be elevated out of proportion to serum creatinine
c. His serum sodium will be elevated
d. Fluid resuscitation should begin with D5/.2 normal saline because of the expected high serum sodium associated
with excessive fluid loss
Answer: a, b

Chronic volume deficits may be manifested by decreased skin turgor, weight loss, sunken eyes, hypothermia, oliguria,
orthostatic hypotension and tachycardia. Serum BUN and creatinine may be elevated, with a high BUN/creatinine ratio. The
hematocrit may be elevated as well. Plasma sodium is not an indicator of intravascular volume, and if the loses have been
isotonic, plasma sodium concentration remains normal. Fluid resuscitation for hypovolemia is initiated with an isotonic
solution such as lactated Ringer’s solution. Urine flow in critically ill patients is monitored with an indwelling Foley
catheter, with the goal of a urine output 0.5mL/kg/h desirable.

281 Clinical manifestations of acute metabolic acidosis include:

a. Decreased cardiac contractility


b. Decreased catecholamine secretion
c. Peripheral arteriolar dilitation
d. Shift of the oxygen-hemoglobin disassociation curve to the left
Answer: a, b, c

The major cardiovascular effects of acute metabolic acidosis are peripheral arteriolar dilitation, decreased cardiac
contractility, and central venous constriction. These may lead to cardiovascular collapse and pulmonary edema.
Catecholamine secretion is stimulated by metabolic acidosis and in mild cases, heart rate may be increased. In addition to
these cardiovascular effects, metabolic acidosis may also affect oxygen delivery by shifting the oxygen-hemoglobin
disassociation curve to the right.

282 Which of the following statement(s) is/are true concerning respiratory alkalosis?

a. Exposure to high altitudes can result in respiratory alkalosis


b. Renal compensation for respiratory alkalosis is obtained by increasing excretion of bicarbonate
c. Symptoms of respiratory alkalosis may mimic those of hypocalcemia
d. The treatment of acute respiratory alkalosis may involve a brown paper bag
Answer: a, c, d

A primary decrease in PCO2 resulting in an increase extracellular pH is referred to as respiratory alkalosis. Hyperventilation
and the ensuing fall in PCO2 may be secondary to hypoxia, reflux simulation from decreased pulmonary compliance, drugs,
mechanical ventilation, and other causes. The two most common causes of hypoxia resulting in respiratory alkalosis are
pulmonary disease and exposure to high altitudes. Renal compensation for respiratory alkalosis is not achieved by
increasing excretion of bicarbonate but by decreasing net acid excretion, primarily through the reduction in ammonia
excretion and increases in organic anion excretion. Chronic respiratory alkalosis is generally asymptomatic. Acute
respiratory alkalosis may cause sensations of breathlessness, dizziness, and nervousness and can result in circumoral and
extremity parathesias, altered levels of consciousness, and tetany. These signs are related to decreased cerebral blood flow
secondary to decreased PCO2 and decreased ionized calcium concentration secondary to increased blood pH. In acute
symptomatic respiratory alkalosis rebreathing, by breathing in and out of a paper bag, can temporarily relieve the
symptoms.

283 Which of the following statement(s) is/are true concerning alterations in serum magnesium?

a. Renal failure is the primary cause of hypermagnesemia


b. Hypomagnesemia may occur during prolonged periods of intravenous fluid replacement
c. Symptoms of hypomagnesemia may mimic symptoms of hypocalcemia
d. Intravenous administration of magnesium sulfate is usually the most efficient method of correction of magnesium
deficiency
Answer: a, b, c, d

Renal failure is the primary cause of hypermagnesemia. Because of the kidneys ability to excrete large magnesium loads,
hypermagnesemia rarely occurs if renal function remains normal. Because the kidneys are able to conserve magnesium well
in states of magnesium depletion, hypomagnesemia rarely occurs from poor intake alone. The combination of low intake
and increased gastrointestinal loss may lead to hypomagnesemia. Prolonged periods of intravenous fluid replacement
without magnesium replacement and the chronic use of loop diuretics or other medications such as cyclosporine or
aminoglycosides can also result in hypomagnesemia. Deficiencies of magnesium may present signs and symptoms similar
to hypocalcemia. Hypomagnesemia may be treated by the oral administration of magnesium however large doses frequently
leads to diarrhea. Correction of major deficits is therefore best managed by intravenous administration of magnesium sulfate
at a dose of 50 to 100 mEq/d.

284 Muscle relaxants are a class of anesthetic agents used to prevent movement and facilitate surgical exposure. Which
of the following statement(s) is/are true concerning the use of muscle relaxants in surgical procedures.

a. Succinylcholine produces rapid obvious muscle fasciculations


b. Pancuronium can be reversed by increasing the acetylcholine concentration using an anticholinesterase inhibitor
(neostigmine)
c. Prolonged periods of muscle relaxation in patients requiring prolonged ventilation should be used in conjunction
with analgesics and amnesic agents
d. The best clinical test for complete reversal of neuromuscular blockade is the ability of the patient to produce a
large negative inspiratory force
Answer: a, b, c

Neuromuscular blocking agents can be classified as depolarizing or nondepolarizing inhibitors of the neurotransmitter,
acetylcholine at the neuromuscular junction. The only noncompetitive inhibitor employed clinically is succinylcholine. This
drug rapidly binds to the neuromuscular junction and produces depolarization, clinically obvious as fine muscle
fasciculations occurring approximately 60 seconds after injection. All other clinically useful muscle relaxants are termed
competitive inhibitors and do cause depolarization when they attach to the neuromuscular junction. Since these agents
compete with acetylcholine, the block produced is in direct proportion to the concentration of the agent relative to the
concentration of acetylcholine. If the concentration ratio is low enough, competitive relaxants can be “reversed” if the
concentration of acetylcholine is artificially elevated. Increase of acetylcholine concentration can be achieved by giving a
drug which blocks metabolism of anticholinesterase (neostigmine).
Nondepolarizing relaxants are frequently used in critically ill patients who are difficult to manage otherwise during
prolonged periods of mechanical ventilation. It is imperative that these drugs be given in conjunction with analgesics and
amnesic agents, since neuromuscular blocking agents have no analgesic or amnestic properties and only prevent motion of
voluntary muscles. Patients may therefore be totally aware and in pain and unable to communicate. All muscles of the body
do not have equal sensitivity in muscle relaxants. The diaphragm is both resistant to neuromuscular blockade while the neck
and pharyngeal muscles that support the airway are most sensitive. It is possible for an intubated patient to spontaneously
ventilate and even to produce a large negative inspiratory force and yet have complete airway obstruction when extubated
due to effects of residual muscle relaxants on upper airway muscles. The definitive clinical test for complete reversal of
neuromuscular blockade is the ability of the patient to sustain a head lift from the bed for five seconds.

285 Local anesthetics are essential agents used in current surgical practice. Which of the following statement(s) is/are
true concerning the use of local anesthetic agents.

a. Complications due to excessive plasma concentration can result only from inadvertent intravascular injection of the
agent
b. Bupivacaine is noted for a slow onset but long duration
c. The addition of epinephrine to a local anesthetic agent will both lower the toxicity and increase the duration of
local anesthesia
d. Hypotension observed when a local anesthetic is administered in the form of a spinal epidural block, is the result of
myocardial depression
Answer: b, c

Local anesthetics constitute a class of drugs which produce temporary blockage of nerve conduction by binding to neuronal
sodium channels. Adverse consequences associated with the use of local anesthetics fall into three categories: acute central
nervous system toxicity due to excessive plasma concentration, hemodynamic and respiratory consequences due to
excessive conduction block of the sympathetic or motor nerves, and allergic reactions. Whenever a local anesthetic has been
injected, there may be inadvertent intravascular injection or an overdose of the drug due to rapid uptake from the tissues. All
may produce seizures. Complications can be minimized by aspirating prior to injection to avoid intravascular injection and
limiting the doses to the safe range. When local anesthetics are administered for a spinal or epidural block, there will be a
progressive blockade of the sympathetic nervous system which will produce systemic vasodilatation. If the block travels
along the thoracolumbar region, a sympathetic blockade will result in profound systemic vasodilatation and bradycardia
with resultant hypotension.
Local anesthetics are divided into two groups: esters and amides. Most commonly used agents, the amides, include
lidocaine and bupivacaine. Lidocaine is noted for a fast onset of action but a short duration whereas bupivacaine has a
slower onset with the duration lasting for four to 12 hours. The addition of epinephrine (100 µg) will lower the toxicity and
increase the duration of the local anesthetic.

286 A 65-year-old gentleman with a history of coronary artery disease and a recent myocardial infarction requires an
elective colon resection for a nonobstructing neoplasm. Which of the following statement(s) is/are true concerning the risks
of general anesthetic in this patient?

a. The age of the previous infarct has no effect on the perioperative reinfarction risk
b. The incidence of reinfarction appears to stabilize after six months
c. Invasive hemodynamic monitoring has no effect on perioperative reinfarction rates
d. Reinfarction has minimal effect on mortality
e. Perioperative infarction most frequently occurs after the first 72 hours from surgery
Answer: b, d

The history of myocardial infarction is an important risk factor for general anesthesia. Large retrospective studies have
found that the incidence of reinfarction is related to the time elapsed since the previous myocardial infarction. The incidence
of reinfarction appears to stabilize at approximately 1% after six months, with the highest rate of reinfarction occurring in
the first three months after the infarct. Mortality from reinfarction, for patients undergoing non-cardiac surgery, has been
reported to be between 20–50% and usually occurs within the first 48 hours after surgery. Invasive hemodynamic
monitoring with pulmonary artery catheters and aggressive pharmacologic intervention has been demonstrated to reduce
reinfarction rates.

287 General anesthesia is not without risks. Which of the following statement(s) is/are true concerning the risk
associated with general anesthesia.

a. Current estimates for mortality due to anesthesia alone are 1:10,000


b. Human error accounts for between 50 and 75% of anesthetic-related deaths
c. Most anesthetic-related deaths are associated with overdose of analgesic agents
d. The most common problems associated with adverse anesthetic outcomes are related to the airway
Answer: b, d

Anesthetic agents effectively obtund or completely block nearly all physiologic protective mechanisms, therefore, there is
an associated risk even without a surgical procedure. Fortunately, with the advent of newer agents and monitoring
techniques, it is estimated the mortality due to anesthesia alone has decreased from approximately 1:10,000 in the 1950s to
as low as 1:100,000 or less for healthy patients today. It has been estimated that between 50–75% of anesthetic-related
deaths are due to human error and are preventable. The most common problems associated with adverse outcomes are
related to the airway: inadequate ventilation, unrecognized esophageal intubation, unrecognized extubation, and
unrecognized disconnection from the ventilator.

288 Over the last decade, the routine use of both invasive and noninvasive monitoring devices has been instituted for
the administration of most anesthetics. The following statement(s) is/are true concerning monitoring of the surgical patient.

a. A pulse oximeter reading will reflect changes in PaO2 only below 80 mm Hg


b. Monitoring of end tidal CO2 will reflect changes in ventilation but not cardiac output
c. Intermittent, noninvasive systemic blood pressure monitoring using an oscillometric blood pressure cuff has
essentially replaced clinical measurement by auscultation
d. Pulmonary arterial catheter monitoring is generally reserved for critically ill patients with significant left
ventricular dysfunction
Answer: a, c, d

Pulse oximetry continuously, noninvasively and inexpensively provides arterial hemoglobin saturation and peripheral pulse
determination. It must be remembered, however, that a pulse oximeter measures oxygen saturation and not arterial oxygen
tension (PaO2). The PaO2 must drop below 80 mm Hg before any significant change in oxygen saturation will occur. End
tidal CO2 monitoring reflects metabolism (the production of CO2), circulation (blood flow to the lungs), and ventilation
(respiratory rate in an intact ventilatory circuit). It can be used as a surveillance monitor for both the respiratory circuit and
the cardiovascular system. Any acute decrease in cardiac output will decrease output to the lung and increase alveolar dead
space, causing an acute drop in end tidal CO2.
Hemodynamic stability can be monitored in a variety of methods, the most basic of which is systemic arterial blood
pressure measure. Intermittent, noninvasive measure of systemic blood pressure with an oscillometric blood pressure cuff
has become the standard in the operating room with an accuracy equal to that of clinical measurement by auscultation.
When tighter control is required in patients with significant hypertension, serious heart disease, or in patients who may
suffer acute blood loss, invasive arterial monitoring is employed. In patients with left ventricular dysfunction who are
undergoing extended surgical procedures with significant fluid shifts and potential blood loss, central venous pressure
monitoring is frequently used, with pulmonary arterial catheter monitoring reserved for more critically ill patients and for
those with significant left ventricular dysfunction.
289 Correct statement(s) concerning complications occurring in the post-anesthetic care unit include which of the
following?

a. The use of nitrous oxide has been well documented to increase the incidence of postoperative nausea
b. Perioperative myocardial ischemia is usually easily diagnosed in the early postoperative period
c. Hypothermia results in a deleterious effect on drug metabolism therefore delaying recovery from anesthesia
d. The serotonin antagonist, odansetron, holds promise as the superior antiemetic agent in the perioperative period
Answer: c, d

Twenty-four percent of patients experience a post-anesthetic care unit complication. Nausea, vomiting and airway support
comprise 70% of these complications. The need to maintain airway support is by far the most common respiratory
complication. Hypothermia has a deleterious effect on altering drug metabolism and delaying recovery. Nausea and
vomiting are rarely unifactorial and cause considerable discomfort to the patient. There is little evidence to favor one
anesthetic or anesthetic technique over another. Nitrous oxide does not appear to increase incidence of nausea in well
documented studies. The new serotonin antagonist, odansetron, has been shown in several studies to be superior to other
agents as a perioperative antiemetic agent.
Perioperative myocardial ischemia is an extremely important complication but difficult to recognize. Diagnosis is
complicated by the fact that only 10—30% of patients suffering documented myocardial infarction will have pain and that
postoperative EKG changes are often nonspecific. One must therefore look for secondary indications of on-going ischemia
such as hypotension, arrhythmias, elevated filling pressures, or postoperative oliguria.

290 Patient-controlled analgesia ( PCA) is a commonly used technique for postoperative analgesia. The following
statement(s) is/are true for the use of PCA.

a. Satisfactory pain relief is provided by the administration of higher narcotic doses


b. The technique is not applicable in the semiconscious or uncooperative patient
c. PCA is as safe as conventional intramuscular administration of pain medication
d. Excessive administration of narcotic medication can be limited by a lockout duration which controls administration
of the narcotic
Answer: b, c, d

The technique of patient-controlled analgesia is based on investigations that small intravenous bolus doses of narcotic on
demand can provide patients with improved pain relief at the same or less total narcotic dose. The system requires some
degree of sophistication and a conscious patient who has been instructed in the technique. Numerous studies have
demonstrated that PCA is as safe as conventional IM medication. The patient can be restricted from receiving excessive
agents via setting a lockout interval duration of several minutes during which time a dose of narcotic cannot be successfully
administered. In addition, limits to the total hourly dose can be set.

291 Narcotics are commonly used in the administration of general anesthesia. Which of the following statement(s)
is/are true concerning this class of agents.

a. Narcotics have both profound analgesic and amnestic properties


b. Narcotics can cause hypotension by direct myocardial depressive effects
c. Naloxone should be used routinely for the reversal of narcotic analgesia
d. Acutely injured hypovolemic patients are at significant risk for decreased blood pressure with the use of narcotic
analgesics
e. Propofol is a new intravenous short-acting narcotic used frequently in the outpatient setting
Answer: d

Narcotics and synthetic analogues belong in the class of drugs called opioids. Narcotics produce profound analgesia and
respiratory depression. They have no amnesic properties, no myocardial depressive effects, and no muscle relaxant
properties. Narcotics may produce significant hemodynamic effects indirectly through the release of histamine and/or
blunting of the patient’s sympathetic vascular tone due to analgesic properties. Acutely injured patients may be hypovolemic
and in pain, with high sympathetic tone and peripheral resistance. Therefore, such patients can experience a dramatic drop
in systemic blood pressure with minimal doses of opioids. All opioids can be reversed with naloxone. Naloxone reversal,
however, can be dangerous because the agent acutely reverses not only the analgesic effects of the opioid but also analgesics
effects of native opioids. Naloxone treatment has been associated with acute pulmonary edema and myocardial ischemia
and should not be used electively to reverse the effects of narcotic. Propofol is a lipid-soluble substitute isopropyl phenol
non-narcotic agent that produces rapid induction of anesthesia followed by awakening in four to eight minutes.

292 Anesthetic techniques used in the management of patients with significant pulmonary disease include:

a. Intubation at a deep level of anesthesia


b. Choice of an anesthetic agent which produces bronchodilatation
c. The use of epidural analgesia for postoperative pain control
d. Perioperative use of intermittent positive pressure breathing
Answer: a, b, c

Patients with significant pulmonary diseases require special anesthetic techniques. Obstructive pulmonary disease can either
be chronic (COPD) or acute (asthma). In either case, the reversible component of obstruction should be reversed prior to
elective surgery. In patients with reactive airway disease, the endotracheal tube may induce severe bronchospasm. Even in
patients who are well treated preoperatively, reactive bronchospasm may complicate anesthetic induction and emergence
from anesthesia. The principal method used to prevent or diminish this “foreign body” induced bronchospasm is intubation
of the patient at a deep level of anesthesia when reflexes are blunted. The classic way of managing a patient with severe
asthma is to induce with an agent that produces bronchodilatation and to ventilate the patient with an inhalation agent until
deeply anesthetized prior to laryngoscopy and intubation. The patient should be extubated while spontaneously ventilating,
but with the inhalation agent still in effect, bringing the patient to consciousness while ventilating by mask.
Because of the potential adverse effects of systemic narcotics on respiratory drive, the use of epidural narcotics and local
anesthetics for postoperative pain control has become very popular. These techniques allow the patient to be extubated
earlier, and patients with intrathoracic and upper abdominal surgery, help restore pulmonary function toward preoperative
values. Preoperative use of intermittent positive pressure breathing has not been demonstrated to decrease the incidence of
postoperative pulmonary complications.

293 Which of the factors listed below will adversely affect the risk of perioperative cardiac complications and
reinfarction in the patient described above?

a. Greater than five premature ventricular beats per minute on EKG rhythm strip
b. The anesthetic technique used
c. Withdrawal of medical therapy with beta blockers and topical nitrates
d. Length of surgical procedure less than three hours
e. Known three vessel coronary artery disease
Answer: a, c, e

The incidence of reinfarction is increased in patients undergoing intrathoracic or intra-abdominal procedures lasting longer
than three hours. The site of surgery or anesthetic technique have not been shown to change the incidence of reinfarction if
the procedure is less than three hours in duration. Patients with known three-vessel or left main coronary artery disease are
at increased risk, while those who have undergone prior coronary artery bypass grafting are of substantially decreased risk
of reinfarction. Prophylactic therapy with beta blockers, calcium channel agents, and nitrates has not been proven beneficial;
however, withdrawal of these agents has been associated with perioperative ischemia, myocardial infarction, and death.
CHF is the single most important factor predicting postoperative cardiac morbidity. Rhythm disturbances, particularly
frequent premature ventricular beats, more than five beats/minute, are also independently associated with an increased risk
of perioperative cardiac complications.

1. Nasotracheal intubation:
A. Is preferred for the unconscious patient without cervical spine injury.
B. Is preferred for patients with suspected cervical spine injury.
C. Maximizes neck manipulation.
D. Is contraindicated in the patient who is breathing spontaneously.
Answer: B

DISCUSSION: The first principle in the management of any injured patient is to secure an adequate airway. This can be
particularly difficult in the presence of facial or laryngeal trauma, or in the unconscious patient with a suspected cervical
spine injury. The mechanical removal of oral debris followed by the “chin lift” or “jaw thrust” maneuvers to relieve soft
tissue obstruction of the pharynx are the first steps. However, when there is any question regarding the adequacy of the
airway, or in the presence of severe head injury, or when the patient is in profound shock, more definitive airway control is
required. In most patients this involves oral endotracheal intubation. However, the insertion of an oral endotracheal tube
often involves hyperextension of the neck with the potential for aggravating cervical spine ligamentous or bony injury.
Nasotracheal intubation is the preferred option for the patient with suspected cervical spine ligamentous or bony injury since
the head and neck can be maintained in the neutral position with minimal manipulation. This technique requires a breathing
patient, as the passage of air must be heard through the nasotracheal tube prior to its insertion through the larynx into the
trachea. Nasotracheal intubation is contraindicated in the presence of mid-face fractures. In this situation, a surgical airway
(cricothyroidotomy, tracheostomy, or needle cricothyroidotomy) is the preferred option.

2. Cardiac contusions caused by blunt chest trauma:


A. Are fairly easy to diagnose.
B. Occur in up to 20% to 40% of patients with major blunt thoracic trauma.
C. Do not usually cause right ventricular dysfunction.
D. Demonstrate arrhythmia as the most common complication.
Answer: BD

DISCUSSION: Cardiac contusions are often difficult to diagnose, but have been estimated to occur in 5% of major trauma
patients, and up to 20% to 40% of patients with severe blunt chest injury. The difficulty in diagnosing cardiac contusions is
that they remain a pathologic diagnosis, confirmed only at autopsy or on direct cardiac examination. The injury may vary
from superficial epicardial petechiae to complete transmural damage. Although significant myocardial injuries, such as
ventricular rupture, coronary vessel thrombosis, and valvular disruption, have been reported, the most common clinically
significant result of cardiac contusion is the occurrence of arrhythmias. Hence, an initial electrocardiogram (ECG) and
subsequent continuous cardiac monitoring for at least 24 hours is generally recommended. Alternative methods of
diagnosing myocardial contusion include creatine phosphokinase cardiac isoenzymes (CPK-MB), two-dimensional
echocardiography, gated ventricular scintigraphic angiography (GVA), radioactive thallous chloride ( 201Tl) uptake, and
right ventricular monitoring. Unfortunately, none of these tests is adequately sensitive or specific in the diagnosis of cardiac
contusion, and their correlation with the presence of arrhythmias or ECG changes is also imprecise.

3. According to the recommendations of the American College of Surgeons Committee on Trauma, which of the following
patients should be transported to a trauma center?
A. Fifty-year-old female who fell 8 feet from a step ladder, with isolated hip fracture and normal vital signs.
B. Fifteen-year-old bicyclist with closed head injury and Glasgow Coma Scale score of 12.
C. Twenty-three-year-old male assault victim with stab wound to the back, normal vital signs, and respiratory distress.
D. Three-year-old infant passenger (restrained) in motor vehicle accident with normal vital signs and no apparent injuries
except abdominal wall contusion.
Answer: BCD

DISCUSSION: The American College of Surgeons Committee on Trauma has developed a field triage decision scheme to
help identify trauma victims with a significant risk of dying as a result of their injuries. This classification is based on four
factors: (1) abnormal physiologic signs, (2) anatomic area of injury, (3) mechanism of injury, and (4) concurrent or co-
morbid disease states. Major physiologic abnormalities include a Glasgow Coma Scale score of less than 13, systolic blood
pressure less than 90 mm. Hg, respiratory rate less than 10 or greater than 29 per minute, or a Revised Trauma Score of less
than 11 or a Pediatric Trauma Score of less than 9. Significant anatomic considerations include penetrating injuries to the
torso, head and neck, and proximal extremities, flail chest, combination of trauma with burns to greater than 10% of body
surface area, two or more proximal long bone fractures, pelvic fractures, paralysis, or traumatic amputation above the wrist
or ankle. Significant mechanisms of injury include a death in the same passenger compartment or ejection from the
automobile, high-impact (greater than 5 miles per hour) auto-pedestrian injuries, or a pedestrian thrown or run over. The co-
morbid factors include pediatric or elderly (<5 or >55) patients or known history of insulin-dependent diabetes or cardiac,
respiratory, or psychotic disorders. These criteria should serve as guidelines for medical control and the pre-hospital care
providers. Such triage guidelines have been shown to produce the triage of only a small fraction (5% to 10%) of all injured
patients to Level I or Level II trauma centers.
4. Which of the following statements about head injuries is/are false?
A. The majority of deaths from auto accidents are due to head injuries.
B. Head injury alone often produces shock.
C. A rapid and complete neurologic examination is part of the initial evaluation of the trauma patient.
D. Optimizing arterial oxygenation is part of initial therapy.
Answer: B

DISCUSSION: Head injuries cause the majority of deaths following automobile accidents, with rupture of the thoracic aorta
the second most common cause of fatality. Head injury itself rarely produces hypotensive shock. It is only in the terminal
phases of brain death that hypotension may be attributable to head injury alone. Therefore, hypotension in trauma patients
must be assumed to be secondary to volume depletion or ongoing hemorrhage. An occult site of hemorrhage (chest,
abdomen, pelvis, retroperitoneum, or extremities) must be strongly suspected and dealt with accordingly. A rapid and
complete neurologic assessment is a crucial part of the initial assessment of all trauma patients. This initial exam gives an
excellent indication of injury severity and prognosis. Since the ultimate outcome of a brain injury is dependent on adequate
cerebral perfusion and oxygenation, adequate airway control, ventilation, hemorrhage control, volume restitution, and
arterial oxygenation are crucial factors in the early management of head injuries.

5. Which of the following statements about maxillofacial trauma is/are false?


A. Asphyxia due to upper airway obstruction is the major cause of death from facial injuries.
B. The mandible is the most common site of facial fracture.
C. The Le Fort II fracture includes a horizontal fracture of the maxilla along with nasal bone fracture.
D. Loss of upward gaze may indicate either an orbital floor or orbital roof fracture.
Answer: B

DISCUSSION: Maxillofacial injuries generally do not cause life-threatening injuries, with the exception of those that
occlude the airway. Therefore, the first priority in assessing and managing the patient with maxillofacial trauma is to assess
and assure the adequacy of the airway. The face is typically divided into thirds when defining injuries. Injuries to the upper
third of the face are often accompanied by ocular or central nervous system complications as well as facial deformities.
Fractures of the orbital roof are frequently associated with frontal sinus and nasal ethmoid fractures, and are accompanied
by a loss of upward gaze due to involvement of the superior rectus muscle. However, the most common cause of loss of
upward gaze is orbital floor injury and associated entrapment of the globe or injury to the inferior rectus muscle. Middle
third of facial structures include the maxilla, zygoma, orbits, and nose. The Le Fort classifications of facial fractures are
commonly employed to describe these complex fracture lines. In a Le Fort II fracture, the superior fracture line is transverse
through the nasal bones or through the articulation of the maxillary and nasal bones with the frontal bones. This is also
known as the “pyramidal” fracture of the mid-face. The diagnosis is established by digital manipulation of the anterior
maxilla and observation for mobility of the central triangle (the maxilla and nose). The lower third of the face contains a
single facial bone, the mandible. After the nasal bones, the mandible is the second most commonly fractured facial bone.

6. What percentage of patients with thoracic trauma require thoracotomy?


A. 10%–15%.
B. 20%–25%.
C. 30%–40%.
D. 45%–50%.
Answer: A

DISCUSSION: Twenty-five per cent of civilian trauma deaths are caused by thoracic trauma, and two thirds of these deaths
occur after the patient reaches the hospital. Mortality of hospitalized patients with isolated chest injury ranges from 4% to
8% and increases to 35% when multiple additional organ systems are involved. Despite high mortality, only 10% to 15% of
thoracic injuries require thoracotomy. Most injuries are successfully managed by the rather simple life-saving maneuvers of
airway control and tube thoracotomy. Unrelenting hemorrhage following either penetrating or blunt thoracic trauma is a
primary indication for immediate thoracotomy. An initial thoracic blood loss of greater than 1500 ml. (30% of blood
volume) or an ongoing loss of 250 ml. for 3 consecutive hours serves only as a practical guideline. The patient's
hemodynamic status and overall condition should be the most influential factors.
7. The radiographic findings indicating a torn thoracic aorta include:
A. Widened mediastinum.
B. Presence of an apical “pleural cap.”
C. First rib fractures.
D. Tracheal deviation to the right.
E. Left hemothorax.
Answer: ABCDE

DISCUSSION: All of the listed radiographic findings should arouse suspicion of a possible torn thoracic aorta. The most
common abnormality noted is a widening of the mediastinal shadow, although only 20% to 40% of patients with a wide
mediastinum have aortic injury. In addition to the radiographic signs listed, other findings that may alert the physician to the
possibility of an aortic tear include loss of aortic contour, elevation of the left mainstem bronchus, depression of the right
mainstem bronchus, shift of the nasogastric tube to the left, and the presence of retrocardiac density. Aortography remains
the “gold standard” diagnostic modality and is indicated if aortic injury is suspected on the basis of mechanism of injury and
any of these suggested findings.

8. Which of the following statements about diagnostic peritoneal lavage (DPL) is/are false?
A. DPL is the diagnostic procedure of choice for gunshot wounds to the abdomen with no obvious intra-abdominal injuries.
B. The average reported incidence of false-positive DPL in patients with significant pelvic fractures is 20% to 30%.
C. Accuracy rates for DPL have generally been reported between 95% and 97%.
D. DPL has been entirely replaced by computed tomography as the diagnostic procedure of choice following blunt
abdominal trauma.
Answer: AD

DISCUSSION: DPL remains the most sensitive and specific indicator of intra-abdominal injury in the trauma patient. The
accuracy rates for DPL in several large collective series reveal an overall sensitivity of 95%, specificity of 98% to 99%, and
overall accuracy of 97%. As result, DPL remains the mainstay for diagnosis of intraperitoneal injury in the trauma patient;
however, not every trauma patient requires DPL. In the awake, alert, and responsive patient with isolated abdominal
injuries, the physical examination and history are very helpful in predicting the presence of significant injury. In the patient
with lower torso (nipples to pubis) or back or flank gunshot wounds, the incidence of intra-abdominal injury is so high that
exploratory laparotomy without further diagnostic modalities is generally advocated. In addition, DPL is generally
inaccurate in the diagnosis of retroperitoneal injuries (duodenum, renal, pancreas), and significant retroperitoneal
hemorrhage in association with pelvic fractures produces a false-positive DPL rate of up to 30%. Computed tomography
(CT) scans have proved extremely valuable in these situations. General recommendations for the use of abdominal CT scans
in trauma victims include patients who are hemodynamically stable (normal) with (1) equivocal abdominal examination, (2)
closed head injury, (3) spinal cord injury, (4) hematuria, and (5) pelvic fractures with significant bleeding. These five
indications are appropriate if the patient is truly hemodynamically stable and the time required to perform CT does not delay
any surgical procedures.

9. A 28-year-old male was injured in a motorcycle accident in which he was not wearing a helmet. On admission to the
emergency room he was in severe respiratory distress and hypotensive (blood pressure 80/40 mm. Hg), and appeared
cyanotic. He was bleeding profusely from the nose and had an obviously open femur fracture with exposed bone. Breath
sounds were decreased on the right side of the chest. The initial management priority should be:
A. Control of hemorrhage with anterior and posterior nasal packing.
B. Tube thoracostomy in the right hemithorax.
C. Endotracheal intubation with in-line cervical traction.
D. Obtain intravenous access and begin emergency type O blood transfusions.
E. Obtain cross-table cervical spine film and chest film.
Answer: C

DISCUSSION: Airway remains the first priority in the management of any patient with multiple injuries. Control of the
airway in a patient with head, face, and neck injury can be extremely challenging. In the patient presented, the best option
given for control of the airway is endotracheal intubation with in-line cervical traction. This requires at least two persons,
one to maintain the head in the neutral position and one to insert the endotracheal tube under direct vision. An alternative in
this case would be emergency cricothyroidotomy, tracheostomy, or needle-jet insufflation. Nasotracheal intubation is not an
option in the presence of a mid-face fracture and a nasal hemorrhage. Clearly, attention must also be directed at assuring
adequacy of ventilation (potential right pneumothorax), assessing and treating obvious hemorrhage, determining if there is
occult intra-abdominal or thoracic hemorrhage, and determining the patient's neurologic status. While management of these
other issues can occur simultaneously, they do not take priority over securing an adequate airway. In this patient the airway
is so tenuous that time should not be spent obtaining a cross-table cervical spine film and chest film prior to definitive
control of the airway.

10. True or False?


A. Trauma is second only to congenital heart disease as the leading cause of death in children.
B. Each year in the United States, approximately 50,000 people die from injuries.
C. Motor vehicle accidents (MVAs) involving intoxicated drivers are responsible for 50% of all MVA fatalities.
D. Active prevention strategies (e.g., seat belts, helmets) have not proved effective in reducing injuries and fatalities.
E. Falls and diving accidents comprise approximately 30% to 40% of cervical spine injuries.
Answer: TRUE C; FALSE ABD

DISCUSSION: The statistics on injuries highlight trauma as “the principal public health problem in America today.”
Trauma remains the leading cause of death in children and adults up to the age of 44 years, and injuries kill more Americans
age 1 to 34 years than all diseases combined. Each year more than 140,000 Americans die of injuries, 50,000 due to motor
vehicle accidents. Just over 50% of motor vehicle injuries involve intoxicated drivers. Injury prevention would be the most
cost-effective method of dealing with this major social and economic burden. Active injury prevention strategies are those
that require active continued cooperation on the part of the individual, such as wearing a helmet when driving a motorcycle
or wearing seat belts in automobiles. Passive approaches such as fitting all motor vehicles with driver air bags require little
or no individual cooperation and have clearly proved the more effective option, but active prevention strategies have
repeatedly been demonstrated to reduce injury fatalities. States with seat belt and child restraining laws show an increase in
seat belt use of more than 60% with a concurrent 9% to 12% reduction in occupant fatalities. Motor vehicle accidents are
responsible for approximately 60% of spinal cord injuries, falls for 20% to 30%, and diving accidents for an addition 5% to
10%. Spinal cord injury acute care and rehabilitation represent some of the most expensive medical treatment, with an
average hospital charge of $50,000 in 1988 for a quadriplegic survivor.

11. Regarding the diagnosis and treatment of cardiac tamponade, which of the following statements is/are true?
A. Accumulation of greater than 250 ml. of blood in the pericardial sac is necessary to impair cardiac output.
B. Beck's classic triad of signs of cardiac tamponade include distended neck veins, pulsus paradoxicus, and hypotension.
C. Approximately 15% of needle pericardiocenteses give a false-negative result.
D. Cardiopulmonary bypass is required to repair most penetrating cardiac injuries.
Answer: C

DISCUSSION: Cardiac tamponade is most frequently caused by penetrating thoracic injury, but may occasionally be
observed following blunt thoracic trauma from cardiac chamber rupture, coronary artery laceration, or ascending dissection
of an aortic tear. Accumulation of as little as 150 ml. of blood in the pericardium will sufficiently decrease diastolic filling to
produce distended neck veins, cyanosis, and decreased cardiac output. Beck's classic triad of distended neck veins, muffled
heart sounds, and hypotension is present in only one third of patients with tamponade. Pulsus paradoxicus is even less
frequently discernible. Immediate temporary treatment consists of pericardiocentesis, which also provides a diagnosis.
However, approximately 15% of pericardiocenteses give false-negative results because of a clotted hemopericardium.
Therefore, echocardiography prior to needle aspiration is generally advisable if promptly available. In the patient in
extremis, emergency thoracotomy with pericardiotomy and cardiac repair should be performed. Most patients with
penetrating cardiac wounds do not require cardiopulmonary bypass to repair their injuries.

12. Which of the following statements or descriptions typically characterizes the syndrome of overwhelming
postsplenectomy sepsis?
A. A syndrome of fulminant gram-negative bacteremia and septicemia in asplenic individuals, characterized by the
presence of as many as 10 6 bacterial organisms per cu. mm. circulating in the bloodstream.
B. A syndrome caused primarily by impaired host ability to mount an effective humoral (immunoglobulin) response to
infection.
C. A syndrome that occurs in 5% to 7% of patients following traumatic splenectomy.
D. A syndrome of rapidly appearing septic shock unresponsive to antibiotic therapy, with an average mortality of 50%.
E. The syndrome may be prevented by preserving as little as 15% of splenic mass in adult trauma victims.
Answer: D
DISCUSSION: In 1952 King and Schumaker suggested that children who had undergone splenectomy were at risk for the
development of bacterial infections, and the syndrome of overwhelming postsplenectomy sepsis (OPSS) was suggested by
Diamond in 1969. The syndrome is unlike fulminating bacteremias and septicemia in individuals with normal splenic
function. The onset is sudden, with nausea, vomiting, headache, and confusion leading to coma. The new infecting organism
is a gram-positive organism in over half the cases, primarily Streptoccoccus pneumoniae. Blood cultures may occasionally
demonstrate up to as many as 10 6 bacterial organisms per cu. mm. circulating in the bloodstream. Disseminated
intravascular coagulation is common along with hypoglycemia, electrolyte imbalance, and shock unresponsive to antibiotics
and fluid or pharmacologic support. Mortality has generally been reported as high as 50% and even up to 80% for
pneumococcal infections. The true incidence of overwhelming postsplenectomy sepsis following a splenectomy from
trauma is not well defined. Green and colleagues suggested that the risk of OPSS is 166 times the rate expected for the
general population. Eraklis and Filler suggested that the incident rate of mortality from sepsis and OPSS is 78 times greater
than that expected for the general population. Despite this increased frequency, overwhelming postsplenectomy sepsis
remains a rare event. Singer's large review of 688 children who had undergone splenectomy for trauma demonstrated only a
1.45% incidence of postsplenectomy sepsis, but a 40% mortality. The occurrence of OPSS appears to be less following
splenectomy for trauma when compared with splenectomy for congenital hematologic disorders. Nonetheless, the
recognition of the severe nature of this entity has prompted many trauma surgeons to more aggressively attempt splenic
salvage. Animal laboratory evidence suggests that at least 50% of the splenic tissue mass must be preserved to prevent
overwhelming postsplenectomy sepsis. The immunologic function of the spleen that appears to be most beneficial in
preventing OPSS is the spleen's capacity for clearance of blood-borne particles and the provision of circulating opsins,
which assist in cell-mediated immunologic functions.

13. Trauma deaths most commonly occur at three distinct time periods after injury. Which of the following statement(s)
is/are true concerning the time pattern of trauma mortality?

a. Only 10% of trauma deaths occur within seconds or minutes of the injury
b. A second mortality peak occurs within hours of injury with deaths in this time period being markedly reduced with
the development of trauma and rapid transport systems
c. Death one day to weeks after the injury are almost entirely due to infection and multiple organ failure
d. Late mortality in trauma patients, occurring days to weeks after the injury, has not been affected by better trauma
delivery systems
Answer: b

Trauma deaths occur at three traditionally recognized times after injury. About half of all trauma-related deaths occur within
seconds or minutes of injury and are related to lacerations of the aorta, heart, brain stem, brain, and spinal cord. Few of
these patients are saved by health care systems, regardless of efficiency. The second mortality peak occurs within hours of
injury and accounts for about 30% of deaths, half of which are due to hemorrhage and half due to central nervous system
injuries. Important reductions in mortality during this period have resulted from the development of trauma and rapid
transport systems. Overall, trauma mortality rates have been reduced from about 30% to 2% to 9% where well-organized
trauma care systems exist. The third mortality peak includes deaths that occur one day after trauma to weeks later. This
mortality rate is usually attributed to infection and multiple organ failure. Ten to 20% of trauma deaths occur during this
period. The development of efficient trauma systems, however, has changed the epidemiology of these deaths. During the
first week after trauma, refractory intracranial hypertension after severe head injury now accounts for a significant number
of these deaths. The incidence of sepsis and multiple organ failure has diminished as the result of aggressive and better early
resuscitation and care. Sepsis and multiple organ failure now account for about 5% of overall mortality and only 30% of late
mortality where organized trauma systems exist.

14. Which of the following statement(s) is/are true concerning the epidemiology of trauma?

a. Trauma is the leading cause of death of individuals less than 44 years of age
b. Trauma follows only cancer and heart disease as leading causes of productive life lost
c. Motor vehicle accidents are the most common cause of traumatic death in young males of all ethnic groups
d. Young males are the population at highest risk for trauma death
Answer: a, d

Although injury affects all age groups, it is epidemic within the younger population of our society. In the United States,
injury is the leading cause of death in individuals less than 44 years and results in 70% of the total hospital admissions.
Young males are the highest risk group, not because of physiologic distinctions, but because of their propensity to engage in
high-risk activities. Although the three leading causes of traumatic death in all ethnic groups are motor vehicle accidents,
homicide, and suicide, for individuals under 35 years of age, the order in which these occur differs. In the African-American
population, the leading cause of death in this age group is homicide, while in all other groups it is motor vehicle accidents.
Although morbidity and mortality figures are important, another important method of analyzing the toll injury places on a
society is in years of productive life lost. Years of productive life lost is used to reflect the amount of productive working
time lost due to premature death. Since injury is so prevalent in the younger population, a traumatic death in this age group
will result in a large number of years of productive life lost, more so than deaths in the older age groups due to chronic
diseases. In fact, years of productive life lost due to injury are approximately 40% higher than those found in cancer or heart
disease patients, the second and third leading causes of productive life lost.

15. Which of the following statement(s) is/are true concerning the biomechanics of blunt trauma?

a. A small child and a large adult have a markedly different level of energy transfer in a high speed vehicular collision
b. Shear strain injuries result from rapid acceleration or deceleration
c. Tensile strain results from direct compression of tissues
d. The tolerance of biologic tissue to trauma injury is directly proportional to the elasticity of the organ
Answer: b, c, d

The severity of any injury is directly proportionate to the amount of kinetic energy transferred to the tissues and the
properties of that tissue which accept and dissipate the energy. Kinetic energy (KE) is a function of the mass (M) of an
object and its
velocity (V):

KE = M x V^2 /2

It is clear from this relationship that changes in velocity alter the kinetic energy transferred more significantly than changes
in mass. Therefore, a small child and a large adult, though significantly different in size and weight, are subjected to similar
levels of energy transfer in a high-speed vehicular collision, the primary determinant being velocity rather than mass. The
tolerance of a biologic tissue to traumatic injury is directly proportional to the elasticity of the organ—that is, its ability to
return to its original shape and position. Elasticity is directly affected by the rate of loading, or the rate at which strain is
applied to the tissues. Applying the force more rapidly increases the likelihood of exceeding tolerance. Blunt trauma results
in two types of forces during impact. First, changes in speed (acceleration or deceleration) create shear strain, and second,
deformity changes (stretch or compression) creates tensile strain.

16. The patient described above has also suffered major facial trauma. Which of the following statement(s) is/are true?

a. A frontal bone fracture and injury to the frontal sinus is a common facial injury in a young adult
b. The optic nerve can be injured in a LeFort type II fracture
c. A facial nerve injury may occur with the fracture of the temporal bone
d. Coronal CT scan images can be a useful adjunct to the evaluation of the patient with facial and head injuries
Answer: c, d

A major cause of maxillofacial trauma are motor vehicle accidents. Facial skeletal fractures and soft tissue damage in the
frontal, orbital, nasal, zygomatic, maxillary and mandibular regions are included. The frontal bone, which houses the frontal
sinuses, is particularly strong due to its arched configuration as well as thick, hard bone. The amount of force necessary to
fracture the frontal sinus is two to three times greater than that necessary for other facial bone fractures. Consistent fracture
patterns from blows to the maxilla have been classified by LeFort and occur within and along the maxilla at its junction with
weaker and aerated bone of the paranasal sinuses and nasal cavity. The classic LeFort fractures are classified as LeFort I,
LeFort II and LeFort III and are of increasing complexity and morbidity. The cribriform plate, ethmoidal arteries, optic
nerve and internal maxillary artery are all vulnerable to injury with a LeFort III fracture.
Soft tissue injuries of the face are encountered even more often than facial fractures. The facial nerve is the most important
underlying structure at risk since blunt or penetrating trauma to the nerve or branches can cause complete or partial
ipsilateral facial paralysis. The most common cause of facial nerve injury is fracture of the temporal bone, but injury can
occur anywhere from the intracranial to the extracranial facial course of the nerve.
After securing the airway and controlling life-threatening hemorrhage, the secondary survey including the facial area is
carried out. The nose is inspected for deformity, pain, mobility, septal hematoma and obstruction. Bleeding should be
managed immediately. Leakage of cerebral spinal fluid suggests a cribriform plate or ethmoidal fracture and a presence
should warn against insertion of any nasal tubes or packing. Since CT scan is part of the standard management of the head-
injured patient, sections of the facial skeleton can be obtained simultaneously, providing information on the extent of facial
fractures in addition to the status of the brain. Axial and coronal sections (obtained with the patient’s head hanging with the
neck extended) are complimentary and are especially helpful in delineating the cribriform plate and ethmoid roof region, the
orbital rims, and the overall vertical facial height.

17. There are a number of options for resuscitative fluids. Which of the following statement(s) is/are true concerning fluids
used for resuscitation of shock?

a. Resuscitation with crystalloid requires volume replacement in a ratio of 1:1 to volume lost
b. The literature strongly supports the use of colloid as being superior to crystalloid in the resuscitation of shock
c. Risks of autotransfused blood include disseminated intravascular coagulation and activation of fibrinolysis
d. Hypertonic saline solution results in volume expansion, an increase in left ventricular performance, decreased
peripheral resistance, and redistribution of cardiac output to kidneys and viscera
e. The use of perfluorocarbons as an experimental resuscitative fluid has been demonstrated to stimulate the immune
system
Answer: c, d

Balanced salt solutions are the most commonly used resuscitative fluids, and their use to restore extracellular volume
significantly decreases the transfusion requirement after hemorrhagic shock. Lactated Ringers and normal saline are the
most effective crystalloid solutions in common use. Resuscitation with crystalloid require a volume administration ratio of
3:1 to 4:1 over volume lost. Although colloids do not replete the interstitial space, they have a volume-expanding effect
somewhat greater than the amount used. Colloids commonly used for volume expansion in hypovolemia include albumen,
dextran 70, dextran 40, and hydroxyethyl starch (hetastarch). Significant controversy exists concerning the use of crystalloid
versus colloid resuscitation. Although the question has not been resolved, several recent studies have indicated an advantage
to crystalloid in resuscitation. A meta-analysis of colloid versus crystalloid resuscitation after hemorrhagic shock has
demonstrated a higher mortality rate in the colloid resuscitated patients, partly due to pulmonary complications. Patients
who lose more than 25 to 30% of total blood volume will need blood for resuscitation. Type O, Rh-negative (universal
donor blood) is immediately available without a cross match. Type-specific blood is available within most blood banks
within five to ten minutes of receipt of the blood specimen, while the patient is being resuscitated with balanced salt
solutions. Although not cross matched, this blood can be administered safely, and therefore its rapid availability and safety
make type-specific blood the blood of choice for resuscitation in trauma. Autotransfusion involves the collection of shed
blood and its reinfusion through a filter back into the patient. Autotransfused blood may produce disseminated intravascular
coagulation (DIC) and activation of fibrinolysis. In addition, blood collected from the peritoneal cavity after hollow viscus
injury, even with cell washing, may lead to bacterial contamination of the autotransfused blood. Hypertonic solutions have
been used in the resuscitation of patients after burn, shock, elective vascular surgery and trauma. In addition to volume
expansion, hypertonic saline solutions have been shown to increase left ventricular performance, decrease peripheral
resistance from arteriolar dilatation, and redistribute cardiac output to the kidneys and viscera. Perfluorocarbons are an
experimental resuscitation fluid comprised of large, branched or cyclic aliphatic compounds which have the ability to
dissolve and carry oxygen. Although effective in volume resuscitation with improved oxygen delivery and oxygen-carrying
capacity, perfluorocarbon infusion has been shown to depress platelet counts, plasma immune globulin levels and depress
other aspects of immune function.

18. Hemorrhage initiates a series of compensatory responses. Which of the following statement(s) is/are true concerning
the physiologic responses to hemorrhagic shock?

a. An immediate response is an increased sympathetic discharge with resultant reflex tachycardia and
vasoconstriction
b. Transcapillary refill is a response serving to restore circulating volume
c. Extracellular fluid becomes increasingly hyperosmolar
d. Adrenergically mediated vasoconstriction is well maintained at the arteriolar and precapillary sphincters
Answer: a, b, c

Hemorrhage initiates both rapid and slower, more sustained compensatory responses. The body responds to maintain
hemostasis almost immediately after the onset of hemorrhage. Decreased activation of the arterial baroreceptors, though a
decrease in blood pressure or even more subtly, a decrease in pulse pressure, causes an increased sympathetic discharge,
resulting in reflex tachycardia and vasoconstriction. Increased adrenergic output with increased secretion of catecholamines
also leads to vasoconstriction, increased heart rate, and increased myocardial contractility. Sustained compensatory
responses include the release of vasoactive hormones and fluid shifts from the interstitium and the intracellular space.
Adrenergically mediated vasoconstriction affects arterial precapillary and postcapillary sphincters and small veins and
venules. The decrease in intravascular hydrostatic pressure distal to the precapillary sphincter leads to reabsorption of
interstitial fluid into the vascular space and thereby functions to restore circulating volume. This is known as transcapillary
refill. The increased release of stress hormones coupled with relative insulin resistance after shock leads to high
extracellular glucose concentrations. In addition, products of anaerobic metabolism from hypoperfused cells accumulate in
the extracellular compartment, inducing hyperosmolarity. This extracellular hyperosmolarity draws water from the
intracellular space, increasing interstitial osmotic pressure, which in turn drives water, sodium and chloride across the
capillary endothelium into the vascular space. If the shock state continues, however, the postcapillary sphincter remains in
spasm, but the arteriolar and precapillary sphincters cannot maintain the tension, and they become relaxed. As sphincters
relax, the capillary hydrostatic pressure increases and sodium, chloride and water move into the interstitium leading to
further depletion of intravascular volume.

19. Which of the following steps is/are part of the primary survey in a trauma patient?

a. Insuring adequate ventilatory support


b. Measurement of blood pressure and pulse
c. Neurologic evaluation with the Glasgow Coma Scale
d. Examination of the cervical spine
Answer: a, b, c

The resuscitation team’s first priority is to simultaneously assess the airway, blood pressure and level of consciousness of
the patient. The first priority is assessment of the airway. After establishment of an airway, the next priority is to insure
adequate ventilatory exchange by rapid auscultation of both lung fields and assessment for mechanical factors that may
interfere with breathing. After establishment of an airway, ventilation and appropriate pleural drainage, if necessary, the next
priority is the assessment of the patient’s circulatory status. This includes an estimation of blood volume and cardiac
function. The initial survey evaluates blood pressure, pulse, and skin perfusion. It is important to emphasize that effective
resuscitation from hemorrhagic shock requires both restoration of intravascular volume and control of hemorrhage. The
final priority of the primary survey is a brief neurological evaluation using the components of the Glasgow Coma Scale.
Although maintaining axial immobilization of the cervical spine is an important early component of all assessments and
resuscitation protocols, examination of the cervical spine regardless of injury is part of the secondary survey.

20. Immediate life-threatening injuries that preclude air exchange which can be treated in the field include which of the
following?

a. Tension pneumothorax
b. Massive open chest wounds
c. Sucking chest wounds
d. Tracheal disruption
Answer: a, b, c

After establishment of a patent and controlled airway, the next priority is to insure that air exchange is taking place.
Immediate life-threatening injuries that preclude air exchange include: tension pneumothorax, massive open chest wounds,
sucking chest wounds, and tracheal disruption. There are no maneuvers likely to correct tracheal disruption in the field.
Both open chest wounds and sucking chest wounds will respond to endotracheal intubation and positive pressure
ventilation. Tension pneumothorax may require field decompression in the rare patient. Field techniques to deal with tension
pneumothorax include needle thoracostomy and chest tube thoracostomy.

21. Which of the following statement(s) is/are true concerning the diagnosis of a peripheral vascular injury?

a. The presence of a Doppler signal over an artery in an extremity essentially rules out an arterial injury
b. Doppler examination is a valuable tool in the diagnosis of venous injuries
c. A gunshot wound in the proximity of a major vessel is an absolute indication for arteriography
d. Both the sensitivity and specificity of arteriography of the injured extremity approaches 100%
Answer: d
Segmental arterial pressure determination by Doppler technique is a valuable adjunct to the physical examination of
extremity vascular trauma. The presence of audible Doppler signals over an artery in the extremity does not rule out an
arterial injury or indicate adequate perfusion. In the healthy and normovolemic person, the normal ankle-brachial index is
1:1. A ratio less than 0.9 or a 20-mm Hg difference between extremities should arouse the suspicion of significant arterial
trauma. Doppler examination has not been widely used to screen for significant venous injuries and is of unproven value.
The selective use of arteriography is fundamental to the evaluation of patients with suspected vascular trauma. The
indications for arteriography generally have included a history of moderate hemorrhage at the penetrating injury site, injury
in proximity to major arterial structures, diminished pulses, and peripheral nerve injury in the distribution of a nerve that is
in proximity to a major vessel. Proximity as the sole indication for arteriography in the absence of diminished ankle-brachial
ratio or other signs of major trauma, has proven to be an unreliable indicator of the need for arteriography. In the absence of
classical signs of major vascular injury, patients with penetrating wounds in proximity to major vessels may be observed
closely without arteriography. The use of arteriography can significantly reduce the rate of unnecessary exploration for
suspected vascular trauma. If routine surgical exploration is performed whenever vascular injury is suspected, a negative
exploration rate of about 60% or more can be expected. Selective use of arteriography reduces the negative exploration rate
to about 35%. Arteriography is an extremely reliable method of excluding vascular trauma. In this context, the sensitivity is
97% to 100% and the specificity is 90% to 98%, with an overall accuracy between 92% and 98%.

22. A 22-year-old male is hospitalized with multiple extremity fractures including a comminuted fracture of the femur and
multiple rib fractures. Which of the following statement(s) is/are true concerning his hospital course?

a. Low-dose heparin should not be employed during his hospital stay


b. Acute respiratory failure associated with petechiae of the head, torso, and sclerae would suggest a pulmonary
embolism
c. Early fracture fixation would decrease the incidence of fat emboli
d. The placement of a Greenfield filter should be avoided due to the risk of lower extremity edema
Answer: c

Perhaps the most catastrophic post-injury complication is pulmonary embolism. All patients with orthopedic injury,
particularly those with fractures of the lower extremities or pelvis, are at high risk for deep venous thrombosis and
subsequent pulmonary embolism. Prophylaxis with sequential compression devices or low dose heparin has reduced the
incidence of deep venous thrombosis in this group. Although concern for the use of heparin is appropriate, prospective
studies demonstrate that low-dose heparin therapy can begin safely within 24 hours in 37% of patients and within 48 hours
in 75%. Trauma patients who are paralyzed or immobilized by head injury, spine injury, or multiple orthopedic injuries
should be considered for placement of a Greenfield filter.
Fat embolism syndrome is a classic triad of acute respiratory failure; altered mental status, and petechiae of the head, torso
and sclerae; and is frequently associated with long-bone and pelvic fractures. Less fulminant presentations, without
petechiae and with lesser degrees of pulmonary dysfunction, are more common. At present, the only therapy for fat emboli
syndrome is supportive care. Therefore, prevention is critical and numerous studies indicate that early fracture fixation
decreases the incidence of this and other pulmonary complications. However, a subset of patients with femoral fractures and
coexisting lung contusion has been recently found to have a higher incidence of ARDS if the fracture is repaired early than
if repaired late.

23. A middle-aged construction worker had a significant fall on the job and presents with obvious high cervical spine
injury. Which of the following statement(s) is/are true concerning his diagnosis and management?

a. A paradoxical breathing pattern in which the abdomen protrudes on inhalation may be observed
b. If the patient appears well compensated on initial evaluation, intubation is unlikely to be necessary
c. The presence of hypotension strongly suggests significant blood loss from associated injury
d. The patient’s extremities are likely to appear warm and well perfused despite the presence of hypotension
e. The use of methylprednisolone beginning 24 hours after the injury will be indicated
Answer: a, d

Fractures to the axial spine, especially in the high cervical spine, can cause varying degrees of respiratory compromise.
Patients with ventilatory failure from acute cord injury typically present with a paradoxical breathing pattern in which the
abdomen protrudes on inhalation, creating a see-saw appearance. This is caused by paralysis of abdominal musculature and
is seen with injuries as low as T-10 to T-11. Early endotracheal intubation and mechanical ventilation must be considered,
even in patients who appear compensated on initial evaluation. There is a strong tendency for such patients to tire and
develop respiratory failure a few hours after the injury.
In addition to ventilatory compromise, high axial spinal lesions can cause significant hypotension, confusing the initial
evaluation of the patient. Most CNS control of arterial tone is mediated through the sympathetic nervous system. In high
thoracic and cervical spinal cord injuries, these controlling pathways may be interrupted, with subsequent loss of vasomotor
tone. This results in hypotension even without significant blood loss. Unlike hypovolemic shock, the patient’s extremities
are warm and well perfused.
A prospective, randomized trial has suggested that high doses of methylprednisolone given within 8 hours of injury have
improved neurologic recovery. Starting treatment with steroids more than 8 hours after injury results in worse recovery than
the placebo and is not recommended.

24. Which of the following statement(s) is/are true concerning Emergency Room thoracotomy?

a. Overall survival rates approach 25%


b. Blunt trauma patients without signs of life upon arrival in the Emergency Room are candidates for Emergency
Room thoracotomy
c. All patients with penetrating trauma to the chest and the absence of vital signs are candidates for ER thoracotomy
d. None of the above
Answer: d

A recent meta-analysis of 24 reports concerning the outcome of Emergency Room thoracotomy found that the overall
survival rate was 11%. There were no survivors among patients with no signs of life (supraventricular electrical activity,
pupillary reaction, and agonal respirations) at the scene. In addition, there were no neurologically intact survivors among
blunt trauma patients without signs of life upon arrival in the Emergency Department. Considering these findings, an
appropriate algorithm would indicate that Emergency Room thoracotomy for penetrating trauma is indicated only if patients
had signs of life at the scene and had lost signs of life less than five minutes prior to arrival in the Emergency Room. Blunt
trauma patients would be allowed Emergency Room thoracotomy only if the patient had signs of life upon arrival at the
Emergency Room. If patients meet these criteria and lose cardiac function, airway placement and fluid resuscitation is
initiated simultaneously with or immediately followed by left anterior thoracotomy, pericardiotomy, and internal cardiac
massage.

25. An untreated or an unrecognized compartment syndrome produces nerve and muscle damage and prevents good
functional recovery despite the patency of vascular repair. Which of the following factors suggests the need for a
fasciotomy?

a. A period of 6 hours or more between injury and restoration of perfusion


b. Combined arterial and venous injuries
c. Postoperative signs of muscle pain or pain on passive stretch
d. Elevated compartment pressures
answer: a, b, c, d

Factors that suggest the need for fasciotomy are as follows:

1. Prolonged period (6 hours or more) between injury and restoration of perfusion


2. Associated crush injury
3. Preoperative calf swelling
4. Combined arterial and venous injuries
5. Extensive venous ligation
6. Postoperative signs or disproportionate muscle pain, pain on passive stretch, or tender and firm muscles
7. Elevated compartment pressures

26. Which of the following statement(s) is/are true concerning the consequences of vascular injuries?

a. Outcome is time-dependent
b. Further injury can take place after restoration of blood flow
c. Acute acidosis, hyperkalemia and myoglobin-induced renal failure can be consequences of severe extremity
ischemia
d. Ischemia to peripheral nerves and muscles can be tolerated to up to four hours without permanent injury
Answer: a, b, c, d

Local consequences of vascular injuries are generally related to acute arterial occlusion from thrombosis after injury. The
results of ischemia distal to the injury sites may lead to limb or organ loss. The degree of tissue loss is related to the
adequacy of collateral flow, the sensitivity of distal tissue to ischemia, and the delay involved in repairing the injury and
restoring blood flow. With regard to these latter issues, the variability is great. The brain is more sensitive to ischemia
because of high basal energy requirements in the absence of glycogen stores. Brain ischemia for longer than 4 minutes
results in irreversible injury. The nerves and muscles are much more resilient, tolerating periods of ischemia up to 4 hours
without permanent injury. An important principle of vascular repairs, however, is that the outcome is time-dependent,
necessitating an aggressive approach and a high priority.
The mechanism of injury from acute arterial ischemia includes both the initial anoxic phase when blood flow is ceased and
reperfusion phase after restoration of blood flow. Termed the reperfusion injury, this phase includes the production of toxic
metabolites and an inflammatory response which causes significant endothelial damage. The events associated with
restoration of arterial blood flow after complete ischemia extend the magnitude and severity of the original insult in skeletal
muscle and peripheral nerves. If the severity of ischemia is significant enough to cause skeletal muscle necrosis,
rhabdomyalysis with the release of potassium and myoglobin into the systemic circulation follows. Acute acidosis,
hyperkalemia, and myoglobin-induced renal failure can occur.

27. Which of the following statement(s) is/are true concerning the surgical management of vascular injuries?

a. A direct approach through the site of injury is often effective as the initial step
b. Systemic heparinization must be avoided in patients with multiple injuries
c. Reversed saphenous vein from the same extremity is the first choice as an interposition graft for extensive arterial
injuries
d. Venous repair should not be attempted in a hemodynamically unstable patient
Answer: b, d

The goal of operative management of vascular injuries is the rapid control of hemorrhage and the restoration of perfusion,
with salvage of extremity or organ in jeopardy. In isolated-extremity vascular injury with arterial occlusion, systemic
heparin should be administered to avoid propagation of thrombus in vessels distal to the occlusion. In multiple-injury
patients, especially those with central nervous system trauma, heparin is inappropriate. The initial steps in the surgical
management of vascular injuries is to obtain proximal and distal control of the injured vessel. This is most easily
accomplished through uninjured areas adjacent to the injury using incisions normally employed for elective exposure of
these vessels. Direct approach to the site of injury is fraught with the hazards of severe hemorrhage and iatrogenic trauma to
the vessel itself or adjacent nerves. The management of the arterial injury is determined by the extent of injury. In a repair of
more extensive arterial injuries, reversed saphenous vein from an uninjured lower extremity is the first choice for an
interposition graft. The repair of concomitant venous injuries is a controversial injury. Proximal extremity veins and the
great veins are repaired whenever technically possible to avoid the sequela of venous occlusion. Venous repair should not be
attempted, in a hemodynamically unstable patient; rather, ligation should be performed to expedite the operation.

28. Penetrating injuries to the pancreas and duodenum are uncommon occurring in 4% and 6% of patients, respectively.
Which of the following statement(s) is/are true concerning the management of pancreaticoduodenal injuries?

a. The Kocher maneuver is essential for providing exposure for the duodenum
b. A large injury of the duodenum which cannot be closed primarily will always require a pancreaticoduodenectomy
c. Pyloric exclusion involves suture or staple closure of the pylorus, gastrojejunostomy, tube decompression of the
duodenum, and placement of a T-tube in the common bile duct
d. Class III injuries of the head of the pancreas should be treated with simple external drainage rather than resection
Answer: a, d

Because of the retroperitoneal location of the duodenum and pancreas and the close proximity to a number of viscera and
major structures, isolated penetrating injuries to the duodenum and pancreas are rare. Diagnosis of pancreaticoduodenal
injuries depends on adequate exposure. A Kocher maneuver whereby the duodenum and head of the pancreas are mobilized
from the retroperitoneal position by excising the lateral peritoneal reflection of the duodenum is essential for this exposure.
Most penetrating injuries of the duodenum are simple lacerations that can be repaired primarily. Large injuries to the
duodenum are more difficult to repair. Injuries of greater than 50% can lead to luminal compromise if repaired primarily.
Treatment with a jejunal patch or duodenojejunostomy with a defunctionalized Roux-en-Y limb of jejunum can avoid the
need for pancreaticoduodenectomy and its associated substantial mortality. Since many duodenal repairs are tenuous
especially in combination with pancreatic injury and the concern about the digestive action of activated pancreatic enzymes
on the repair, the technique of pyloric exclusion has been devised and is advocated by some. Pyloric exclusion involves
suture or staple closure of the pylorus and restoration of gastrointestinal continuity by performing a gastrojejunostomy. Tube
decompression of the duodenum should be performed in severe duodenal injuries but the biliary tract does not require
decompression unless there has been an associated biliary tract injury. The management of pancreatic injuries depends on
the location with respect to the head, body, and tail of the gland. Class III injuries of the head of the pancreas involve a
relatively severe injury. In almost all situations, these injuries should simply be drained without attempts at resection or
emergency internal drainage. If a patient develops a pancreatic fistula, the fistula can be controlled by the drain. If the fistula
does not resolve with time, the pancreas can be drained internally at a later date.

29. A CT scan is performed on this patient. Which of the following statement(s) is/are true concerning the findings on CT
scan and the patient’s management?

a. The CT finding that correlates most significantly with intracranial hypertension is compression or obliteration of
the basilar cisterns
b. Intracranial pressure monitoring is indicated immediately in any patient with cisternal compression.
c. A brain contusion appears as a very homogeneous high density area in the cerebral cortex
d. Intracerebral hematomas are routinely treated with craniotomy
Answer: a, b

The CT finding which correlates most significantly with intracranial hypertension is compression or obliteration of the
basilar cisterns. Not only does this finding portend a stormy intracranial pressure course, but the primary predictor of
outcome in patients with this CT picture is the peak level intracranial hypertension occurring during the first 72 hours.
Intracranial pressure monitoring should be immediately initiated in any patient with cisternal compression and the
intracranial hypertension should be vigorously treated. Intracerebral hemorrhage and cerebral contusion are common after
trauma and are readily visualized on CT scan. Brain contusion appears as a focal, heterogeneous density with hemorrhage
interspersed with injured tissue. Intracerebral hematomas are generally more homogeneous in their high density appearance.
These lesions tend to “blossom” over time due to continued hemorrhage and the development of edema. Therefore, it is
important to closely observe and monitor the ICP of such patients because a significant and hazardous mass effect may
evolve, requiring surgical extirpation. Cerebral contusions and intracerebral hematomas are treated operatively only when a
mass effect results in intracranial hypertension or signs of herniation.

30. Which of the following statement(s) is/are true concerning the management of chest trauma?

a. The majority of injuries to the chest require surgical intervention


b. The posterior lateral thoracotomy is the optimal approach for emergency thoracotomy
c. Either computed tomography or angiography are suitable methods for detecting aortic disruption in a patient with
an abnormal chest x-ray
d. Persistent bleeding associated with a penetrating injury to the chest is often due to injury to an artery of the
systemic circulation
Answer: d

The chest radiograph is by far the most important diagnostic study in patients with chest trauma and should be obtained
early in all patients. Angiography is the best study to rule out major injury to the great vessels in the chest, and angiography
remains mandatory in the majority of patients at risk for aortic disruption who have an abnormal chest radiograph. At the
present time, CT scan of the chest appears to have a higher rate of missed injury than angiography for assessment of the
aorta and should probably be avoided in patients with abnormal chest films.
The majority of injuries to the chest can be successfully managed without surgical intervention. The routine use of a tube
thoracostomy for treatment of hemothorax and pneumothorax is the cornerstone of therapy. Thoracotomy is most often
needed for the control of massive bleeding, or bleeding which persists despite tube thoracostomy. About 80% to 85% of
hemorrhages within the chest can be treated by tube thoracotomy alone. Even larger and deep lacerations of the lung
parenchyma, which bleed with relatively low pressure from the pulmonary circulation, will be controlled by the reinflated
lung parenchyma as well as edema in the tissue from the injury. Persistent bleeding is most commonly due to injuries to
major proximal branches of the pulmonary circulation or injuries to systemic arteries including intercostal arteries and
internal mammary arteries. The choice of position and surgical approach for thoracotomy for thoracic injury is dictated by
the nature of the patient’s injuries, the certainty of diagnosis, and the potential for associated injuries involving other body
sites. Although the standard postero-lateral thoracotomy provides optimal exposure to the contents of a particular
hemithorax, the lateral position of the patient makes access to the other side of the chest or abdomen difficult if not
impossible. Therefore, though postero-lateral thoracotomy provides the best access, it can be used only in patients who have
injuries isolated to a given hemithorax. In most patients undergoing emergency thoracotomy for chest trauma, an antero-
lateral approach must be used in patients supine to allow access to the abdomen and contralateral chest cavity. Although
exposure through this incision is considerably more difficult, it is adequate with proper technique. Median sternotomy
incision provides excellent exposure to the heart and the great vessels in the anterior mediastinum, but it provides very
difficult exposure for repair of injuries to the lungs, descending aorta, chest wall, diaphragm, or esophagus. Therefore, like
the postero-lateral thoracotomy, it can be used only when the patient’s injuries can be determined with relative certainty.

31. The anterior neck is divided into three zones defined by horizontal planes. Which of the following statement(s) is/are
true concerning penetrating injuries to the anterior neck?

a. Penetrating injuries to Zone I carry the highest mortality


b. Injuries to Zone II are the most common and the mortality rate is second only to those of Zone I
c. Exposure of Zone III for detection of injuries to the distal carotid artery and pharynx can be quite difficult
d. All hemodynamically stable patients with penetrating injuries to Zone I should have angiography
e. Most vascular lesions in Zone III are best treated by surgical exploration
Answer: a, c, d

The anterior neck is divided into three zones defined by horizontal planes. Zone I represents the base of the neck and it
invariably extends from the sternal notch to the top of the clavicles or the cricoid cartilage. Injuries here carry the highest
mortality because of the risk of major vascular and intrathoracic injury. Zone II is the mid-body and largest portion of the
neck. It extends from the top of Zone I to the angle of the mandible. Zone II injuries are most common but carry a lower
mortality rate than either Zone I or Zone III injuries, since the injury is generally apparent and exposure of the vital
structures is readily accomplished. Zone III is that part of the neck above the angle of the mandible. The risk of injury to the
distal carotid artery, salivary glands and pharynx is greatest in this zone. Exposure in this region can be particularly difficult.
Most surgical groups advocate exploration in the majority of penetrating neck wounds that penetrate the platysma in Zone II
and in all patients with clinical signs of tracheal, esophageal, or major vascular injury. Preoperative angiography is generally
not required for Zone II injuries because of the relative ease of exposure and control of critical vascular structures. Zone I
and III penetrating injuries are selectively managed based on clinical presentation and the result of diagnostic studies.
Hemodynamically unstable patients are immediately explored with operative incision based on the most likely source of
vascular injury. Zone I injuries are essentially managed similar to mediastinal traversing wounds. Angiography is performed
in all hemodynamically stable patients with penetrating wounds to Zone I to identify potential injuries to the thoracic outlet
vessels or to plan better operative approach. Angiography is also performed for Zone III injuries, because of the possible
inaccessibility of the internal carotid artery lesions or to demonstrate a need for systemic anticoagulation. Furthermore, most
of the vascular lesions identified at the base of the skull are best managed by interventional angiography techniques.

32. Which of the following statement(s) is/are true concerning the definitive management of neck injuries?
a. Patients with evidence of an acute stroke following penetrating injury involving the carotid artery should be
managed with arterial ligation
b. Unilateral vertebral artery occlusion usually results in a clear neurologic deficit and therefore revascularization is
indicated
c. The combination of esophography and endoscopy improves the accuracy of detecting esophageal injury with
penetrating trauma
d. External drainage is an important aspect of the surgical management of an esophageal injury
e. Arterial dissection secondary to blunt trauma is best managed by operative exploration and resection of the
dissection
Answer: c, d

Blood vessels are the most commonly injured structures in the neck. Major arterial injuries occur in 18% of penetrating
neck wounds and major venous injuries in 26%. Blunt vascular injures account for a very small percentage of carotid
injuries, however their management is somewhat controversial with treatment highly variable dependent upon the vascular
lesion as well as concomitant injuries. When anatomically feasible, pseudoaneurysms are probably best managed by
resection. The best treatment for arterial dissection, however, although not completely resolved, would appear to be
systemic anticoagulation if possible to prevent propagation, embolization or thrombosis. Resection may not be required in
the majority of patients. Penetrating carotid injury most commonly presents with exsanguinating hemorrhage. The
indication for repair versus ligation of a carotid injury depends, in part, on the neurologic presentation. Patients without a
neurologic deficit and a carotid injury should have restoration of vascular continuity with good neurologic outcome
anticipated. Also, patients with all grades of neurological deficits short of coma should have primary vascular repair.
Although experience with revascularization of patients suffering acute stroke from arteriosclerotic occlusive disease
suggests that hemorrhagic infarction and death may result from revascularization, several reviews of acute revascularization
in the trauma patient note that combined morbidity and mortality are significantly less in those patients repaired primarily
compared to those managed with arterial ligation. Traumatic injury to the vertebral arteries are now more commonly
identified due to the more liberal application of neck angiography. Unilateral vertebral artery occlusion seldom results in a
neurologic deficit. Treatment of blunt vertebral artery injury with thrombosis generally is nonoperative: systemic
anticoagulation is recommended to avoid further propagation of existing thrombus.
The diagnosis of esophageal injury can be difficult. The sensitivity of esophography in detecting esophageal injuries varies
from 50% to 90%; the sensitivity of endoscopy ranges from 29% to 100%. These modalities should be considered
complimentary, and when combined have an accuracy of nearly 100%. Since virtually all reported deaths from cervical
esophageal injuries are the result of delayed or misdiagnosis, a particularly high index of suspicion is warranted. When
injured, the esophagus should be meticulously debrided and repaired primarily in one or two layers. It is important to drain
all such wounds, because infection or salivary fistula is not an infrequent complication.

33. A 25-year-old male is involved in a motor vehicle accident with a significant head injury. Which of the following
statement(s) is/are true concerning his injury and management?

a. A single episode of systolic blood pressure < 90 mm Hg occurring during the early period after injury significantly
increases the chances of mortality and morbidity
b. Systemic hypertension should be avoided to reduce the risk of intracranial hemorrhage
c. The patient should be vigorously hyperventilated to reduce PaCO2
d. The patient should be heavily sedated and pharmacologically paralyzed after the initial neurologic examination
Answer: a

Brain injury is the most common cause of death in trauma victims, accounting for about half of deaths at the accident site.
The injuries are generally the result of blunt trauma, and motor vehicle accidents are the most frequent cause. Head injuries
involve not only the primary injury but secondary injuries which can result from the events occurring after the primary
insult, due to either the direct consequences of a process initiated by the primary injury or to deleterious outside influences.
The occurrence and magnitude of secondary insults is often the determining factor in outcome from brain injury. Since
secondary insults, in contrast to primary injuries, are amenable to medical therapy, they are the focus toward which the
medical treatment of brain injury is directed. The primary external secondary injury processes occurring following brain
injury are hypotension and hypoxia. Hypotension is the number one treatable determinant of severe head injury. A single
episode of systolic blood pressure less than 90 mm Hg occurring during the period from injury through resuscitation
doubles the mortality and significantly increases the morbidity of any given brain injury. Intracranial hypertension may be
considered as being deleterious via two somewhat separate mechanisms—herniation and ischemia. Herniation occurs when
a pressure gradient exists across an incomplete barrier such as the tentorium or the falx cerebri. It is deleterious because of
the tissue damage that results when herniation occurs. The second aspect of the intracranial hypertension that is deleterious
is elevated resistance to cerebral blood flow, resulting in or exacerbating ischemia. Treatment of systemic hypertension is
rarely indicated in the head injured patient. There is no evidence that hypertension promotes continued intracranial
hemorrhage, and hypertension related to brain injury generally resolves when the intracranial hypertension is controlled.
The treatment of intracranial hypertension involves elevating the head of the bed (reversed Trendelenburg position) but
should only be performed after complete resuscitation has been accomplished. The confusion and agitation often attendant
to head injury renders sedation desirable, therefore, patients with suspected head injury should generally be sedated.
Pharmacologic relaxation, however, has the notable effect of limiting the neurologic examination to the pupils and, upon
arrival to the hospital, the computed tomography scan. Therefore, its use in the absence of evidence of herniation should be
limited to situations which sedation alone is not sufficient to optimize safe and efficient patient transport and resuscitation.
When used, short acting agents are strongly preferred. Prophylactic administration of mannitol is not recommended due to
volume depleting diuretic effect. In addition, although it is desirable to approximate the lower end of the normal range of
PaCO2 during transport of a patient with suspected brain injury, the risk of exacerbating early ischemia by vigorous
hyperventilation outweighs the questionable benefit in the patient without evidence of herniation. Therefore, ventilatory
parameters consistent with optimal oxygenation and “normal” ventilation are recommended.

34. Which of the following statement(s) is/are true concerning the biomechanics of penetrating injuries?
a. Stab wounds are associated with significant cavitation
b. A hollow point bullet is associated with an enlarged area of injury
c. A high velocity gunshot wound creates a vacuum pulling clothing, bacteria, and other debris into the wound
d. The frontal area of impact of a bullet is determined by the caliber of the bullet
Answer: b, c

Penetrating trauma involves the transfer of energy to a relative small tissue area. The kinetic energy of a bullet disrupts and
fragments cells and tissues, moving them away from the path of the bullet. The actual size of the frontal area of impact is
determined by three factors—profile, tumble (spin and yaw), and fragmentation. A knife or jacketed bullet does not deform
significantly during impact, whereas a hollow-point bullet flattens, spreads, and fragments on impact and therefore enlarges
the area of injury. Low energy missiles including knives and other objects produce damage only by sharp cutting edges.
Cavitation is minimal, and injury can be predicted simply by tracing the pathway of the weapon within the body. Low,
medium and high velocity gunshot wounds, however, produce damage not only to tissue directly in the path of the missile
but also produce cavitation injury to tissues in close proximity to the impact. The size of the cavitation injury is directly
proportional to the bullet’s velocity. The essential difference between high velocity weapons and low and medium velocity
weapons is that the higher velocity weapons have a much larger cavity or pressure cone than low-and medium-velocity
missiles. The temporary cavity extends well beyond the actual bullet tract, producing a wider injury. The vacuum created by
the cavitation pulls clothing, bacteria, and other debris from the surrounding areas into the wound, creating the additional
risk of contamination.

35. In which of the following clinical situations is peritoneal lavage indicated?

a. A patient with suspected intraabdominal injury who will undergo prolonged general anesthesia for another injury
outside the abdomen
b. A patient with a high velocity abdominal gunshot wound
c. A patient with an abdominal knife wound
d. A hemodynamically unstable patient with a high suspicion of intraabdominal hemorrhage
e. A patient with major noncontiguous injuries (i.e., chest and lower extremity)
Answer: a, c, e

Peritoneal lavage is a standard technique to detect significant intraabdominal hemorrhage after blunt trauma. Its
applicability after low-velocity gunshot or stab wounds is less clear, but it has no place in the evaluation of high-velocity
gunshot wounds. Abdominal paracentesis can be used in place of peritoneal lavage when the suspicion of intraabdominal
hemorrhage is high and time is critical. Specific indications for peritoneal lavage and blunt trauma include a number of
conditions such as a patient with major noncontiguous injuries, a patient with suspected intraabdominal injury in whom
physical examination is unreliable or impossible due to the need for prolonged general anesthesia for another injury.
Peritoneal lavage is not useful for patients with abdominal gunshot wounds; all of these patients require immediate
laparotomy. When local examination of a stab wound suggests penetration to the anterior fascia and peritoneum, diagnostic
peritoneal lavage may help discriminate between those with significant and insignificant injuries.

36. Physiologic responses to hypothermia include:

a. Tachycardia regardless of core temperature


b. Tachypnea regardless of core temperature
c. Pupillary dilatation and loss of cerebral autoregulation at temperatures below 26°C
d. A cardiac rhythm contraindicates cardiopulmonary resuscitation even in the absence of a palpable pulse
Answer: c, d

The physiologic response to hypothermia is one of transitional changes, with few exact temperature-dependent responses.
Broadly speaking, the transition from a “safe zone” of hypothermia (where physiologic adaptations to heat loss are working)
to a “danger zone” of hypothermia occurs between 33°C and 30°C. The cardiovascular response includes tachycardia
followed by progressive bradycardia which starts at about 34°C , and which results in a 50% heart rate decrease at 28°C.
Asystole occurs below 25°. Due to difficulty in palpating weak, bradycardic pulses in cold, stiff hypothermic patients, the
presence of an organized rhythm should be taken as a sign of life that contraindicates CPR, despite the absence of a palpable
pulse. Respiratory drive is increased during the early stages of hypothermia, but below 30°C progressive respiratory
depression occurs, resulting in a decrease in minute ventilation.
The neurologic response to hypothermia is heralded by progressive loss of lucidity and deep tendon reflexes, and eventually
flaccid muscular tone. Pupillary dilatation and loss of cerebral autoregulation occur at temperatures below 26°, and
electroencephalography becomes silent at 19–20°. It is important, however, to remember that patients have been revived
with core temperatures as low as 17°C, and therefore the saying “No one is dead until warm and dead”.

37. Which of the following statement(s) is/are true concerning the injury pattern in patients with blunt versus penetrating
injuries?

a. Solid organs are most frequently injured following blunt trauma


b. The liver is the most frequently injured organ in both penetrating and blunt trauma
c. Major vascular injuries occur much more commonly in penetrating trauma than with blunt abdominal trauma
d. Injury patterns for blunt abdominal trauma in children are different than adults whereas with penetrating trauma no
such difference exists
Answer: a, c, d

Most series list the spleen as the most commonly injured intraabdominal organ after blunt trauma. However, the means of
diagnosis may affect this finding since small liver injuries, often detected only on CT scan of the abdomen, may go
unreported while splenic injuries are likely to be clinically significant and require surgical intervention. Solid organs are
most frequently injured from blunt trauma since the sudden application of pressure to the abdomen is more likely to rupture
a solid organ than a hollow viscus, and this accounts for the greater incidence of solid organ injury. More elastic tissues of
young people tolerate trauma better than those of older people, and this accounts, in part, for the differences in injuries
between children and adults with blunt abdominal trauma. Major vascular injuries occur in over 10% of patients sustaining
penetrating trauma but occur in only approximately 2% of patients with blunt trauma.

38. An 18-year-old male suffers a gunshot wound to the abdomen, resulting in multiple injuries to the small bowel and
colon. Which of the following statement(s) is/are true concerning this patient’s perioperative management?

a. A multi-agent antibiotic regimen is indicated


b. Antibiotics should be continued postoperatively for at least 7 days
c. Laparotomy, as a diagnostic test for postoperative sepsis, should be considered
d. The incidence of postoperative wound or intraabdominal infection would be increased in association with a colon
injury
Answer: d

Post-traumatic intraabdominal infection is almost always the result of gastrointestinal tract contamination. Penetrating
trauma accounts for the largest proportion of these infections. Because of the higher bacterial counts, the colon is
consistently associated with a higher incidence of infectious complications than isolated gastric, duodenal, or small bowel
injuries. The precise incidence of intraabdominal or incisional wound infection after colonic injuries depends on factors
present at the time of injury (blood loss, degree of contamination, and other associated injuries) and on whether the wound
is closed or left open. The use of perioperative antibiotics for trauma has been investigated extensively. Most studies have
demonstrated that single-agent cephalosporins are at least as effective as multi-agent regimens in retarding intraabdominal
abscess or wound infections resulting from a variety of contaminated traumatic wounds.
Fever, leukocytosis, tachycardia, the development of a paralytic ileus, increased fluid requirements, and failure to wean
from a mechanical ventilation may all represent warning signs of the development of intraabdominal infection. CT is the
single most useful diagnostic tool in this clinical setting because it yields considerable information with regard to organ
injury in the presence of intraabdominal abscesses or fluid collections. Laparotomy, as a diagnostic tool for unexplained
sepsis, has a low-yield in critically ill trauma patients and should not be used routinely.

39. A middle-aged man is undergoing laparotomy for blunt abdominal trauma. The spleen and liver are both found to be
injured. Which of the following statement(s) is/are true concerning the management of these injuries?

a. If the patient has multiple other abdominal injuries and hypotension, splenic salvage should not be attempted
b. The incidence of life-threatening sepsis in the adult following splenectomy is no greater than in the normal
population
c. All liver injuries regardless of their depth require external drainage
d. The Pringle maneuver should control all bleeding from hepatic parenchymal vessels
e. If concern for a biliary fistula from the liver parenchyma exists, a T-tube should be placed even if the common bile
duct is otherwise normal
Answer: a

Solid abdominal organs such as the liver and spleen, are most commonly injured during blunt abdominal trauma. The
management of splenic trauma has been the subject of major reexamination in the last few decades. Historically, splenic
injuries are routinely treated with splenectomy. With increased appreciation of the danger of post-splenectomy sepsis,
splenic salvage procedures and nonoperative management of these injuries have become well accepted. This is particularly
true in children. The incidence of post-splenectomy sepsis varies from 0.5% to as much as 12% to 15%, depending on the
age and underlying disease. The incidence is inversely related to age and is higher with underlying hematologic disorders
such as lymphoma or thalassemia. The incidence of life-threatening sepsis in adult trauma patients is low, but higher than in
the normal population. Splenic salvage should not be attempted if the patient has protracted hypotension or other severe
injuries or if undue delays are encountered in an attempt to repair the spleen.
Simple lacerations of the liver found at the time of surgery do not require drainage unless they are deep into the liver
parenchyma, in which case they have a high probability of postoperative bile leakage. Biliary fistulas usually will close
spontaneously, and major extrahepatic ductal injuries are rare. A T-tube placed in an otherwise normal common bile duct is
inappropriate unless the extrahepatic biliary tree is injured. In the event that bleeding continues despite segmental ligation of
parenchymal vessels, the structures of the porta hepatis should be compressed as a diagnostic maneuver (Pringle maneuver).
If the bleeding stops, it is assumed to originate from the portal veins or hepatic artery. If the bleeding continues, it is
presumed to arise principally from the hepatic veins or inferior vena cava.

40. Which of the following conclusions can be drawn from prospective randomized studies involving restoration of
circulation in the field?

a. Pneumatic anti-shock garment is of benefit only in patients with a field blood pressure less than 50
b. Patients with major vascular injury should not receive intravenous fluid infusion until bleeding can be controlled in
the operating room
c. Hypertonic saline, used as resuscitation fluid, provides no benefit to patients
d. Hypertonic saline has been shown to exacerbate bleeding and precipitate coagulopathy
Answer: a, b

The most common cause of death during the first hour after injury is hemorrhage. Therefore, after establishment of patent
airway and adequate air exchange, the next priority is to support the circulation. The standard of care in the pre-hospital
setting for hypotensive patients has been volume replacement and application of pneumatic anti-shock garment. In a recent
large prospective randomized study, pneumatic anti-shock garments offered no survival advantage and actually increased
mortality when used in patients with thoracic injuries. On the other hand, there was the suggestion that patients with a field
blood pressure less than 50 mm Hg may benefit from this treatment. A second prospective study confirmed this result,
indicating that the pneumatic anti-shock garment is of value to selected patients with field blood pressure less than 50 mm
Hg. A recent clinical study has also demonstrated that internal hemorrhage from major vascular injuries should not be
treated with intravenous fluid infusion until bleeding can be controlled in the operating room. In the hypotensive state, such
major vascular injuries have a chance to clot and temporarily stop bleeding. But if intravenous volume restores blood
pressure, the clot may dislodge and the rate of bleeding significantly increases. This may lead to both loss of oxygen
carrying capacity and clotting factors, and ultimately exsanguination. Hypertonic saline restores intravascular volume and
blood pressure to near normal very rapidly. The prospective randomized trial of normal saline versus hypertonic saline
demonstrated a significant improvement in survival when the data were normalized to a select group of patients. There was
no evidence that nontamponaded bleeding was exacerbated by the use of hypertonic saline despite the fact that blood
pressure and intravascular volume increased.

41. Which of the following statement(s) is/are true concerning hypothermia following traumatic injury?

a. The majority of patients presented to a level I trauma center are hypothermic at some time
b. The initial temperature for trauma-associated hypothermia is associated with no seasonal variation
c. Moderate levels of hypothermia (34°–32°C) has no effect on mortality in the trauma patient
d. The coagulation system is most affected in hypothermic patients who have sustained major trauma
Answer: a, b, d

Mild hypothermia is very common following traumatic injury and is considered a form of secondary accidental
hypothermia. It has been reported that 57% of trauma patients admitted to a level I trauma center are hypothermic at some
time, with temperature loss most significant in the Emergency Room. This effect appears to have no seasonal variation.
Although the mortality rate for moderate (28°–32°C) degrees of primary accidental hypothermia is only approximately
21%, the same level of hypothermia in surgical patients who are victims of trauma can be associated with mortality rates
approaching 100%. Hypothermia affects multiple systems, however, the system most affected in patients sustaining major
trauma are those involved in clotting. This effect seems to involve both abnormalities in platelet and clotting function.

42. Which of the following statement(s) is/are true concerning injuries to the chest wall?

a. The mortality rate currently associated with sternal fractures is as high as 25–30%
b. The severe ventilatory insufficiency associated with a flail chest is due to the paradoxical motion of the involved
segment of chest wall
c. In most cases of an open pneumothorax, or sucking chest wound, surgical closure is necessary
d. Persistent chest tube bleeding at a rate greater than 200 ml/hour for four hours, or greater than 100 ml/hour for
eight hours is an indication for thoracotomy for control of hemorrhage
e. A 20% incidence of splenic injury is associated with fractures of ribs 9, 10 and 11 on the left
Answer: c, d, e

Rib fracture is the most common injury associated with blunt chest trauma and may occur directly at the site of force or
laterally as the result of significant antero-posterior compression of the chest. The location area of the rib fracture may be
indicative of associated injuries. A 20% incidence of splenic injury is associated with fracture of ribs 9, 10, and 11 on the
left with a similar association with right lower rib fractures and hepatic parenchymal injuries. The mortality rate associated
with sternal fractures in older series was as high as 25–30%, mainly because of other injuries to the chest, such as aortic
transection, cardiac contusion, tamponade or tracheo-bronchial rupture. More recent studies have suggested a change in the
pattern and severity of injuries associated with sternal fracture. Widespread improvements in automobile safety have likely
contributed to this change such that isolated sternal fractures may result from shoulder belt use and may not necessitate
hospital admission in the stable patient. A flail chest occurs when consecutive ribs are fractured in more than one place,
creating a free-floating segment of the chest wall. The creation of a free-floating segment may result in paradoxical chest
wall motion with respiration. The intact chest wall expands during inspiration, but the negative intrathoracic pressure
generated causes the flail segment to move inappropriately inward. Historically it was believed that the paradoxical motion
was the cause of severe ventilatory insufficiency associated with the flail chest. Gradually, understanding of the
pathophysiology of the flail chest has evolved. The ventilatory impairment is not simply due to paradoxical motion of the
chest wall, but rather due to underlying pulmonary parenchymal injury in combination with the hypoventilation and
splinting that results from the pain of multiple contiguous rib fractures. The open pneumothorax, or sucking chest wound, is
an uncommon injury usually caused by impalement, high-speed motor vehicle accident, or shotgun blast, which causes a
large chest wall defect. The diagnosis of a sucking chest wound can be made on simple inspection of the chest wall and
hearing the flow of air through the wound. The defect should be occluded immediately with an impermeable dressing,
essentially converting the situation to a closed pneumothorax. Tube thoracostomy is then performed to re-expand the lung.
The chest wall defect usually requires operative debridement and formal chest wall closure. A hemothorax is the
accumulation of blood in the pleural space and it occurs in 50–75% of patients with severe blunt or penetrating chest
trauma. Massive hemothorax (i.e., larger than 1000–1500 ml) may require thoracotomy. Persistent bleeding, at a rate of >
200 ml/hour for four hours, or > 100 ml/hour for eight hours, is also an indication for thoracotomy. If the patient manifests
any hemodynamic instability during the period of observation, urgent thoracotomy is mandatory.

43. A 22-year-old male driving a car at a high speed and not wearing a seatbelt, leaves a road and crashes with a full frontal
impact into a tree. Which of the following injury patterns may be predictable from this type of motor vehicle accident?

a. Orthopedic injuries involving the knees, femurs, or hips


b. Laceration to the aorta
c. Hyperextension of the neck with cervical spine injury
d. Diaphragmatic rupture due to marked increase in intraabdominal pressure
Answer: a, b, c

With frontal impact, when the vehicle stops abruptly, unrestrained front-seat occupants move in one of two predictable
pathways—down and under the dashboard or up and over the steering wheel. With the former movement, the knees strike
the dashboard, and the upper legs absorb the primary energy transfer. Dislocated knees, fractured femurs, and posterior
fracture dislocation of the hips are expected injuries. After the knees impact, the upper body flexes forward and up and over
the steering wheel. The chest or abdomen impacts the steering wheel and the head impacts the wind shield.
Predictable injury patterns following the up-and-over component of a frontal impact include the following: 1) anterior chest
wall compression; 2) compression injuries to both hollow and solid abdominal viscera; 3) shear injuries such as lacerations
to the aorta or liver, kidneys or other solid viscera; 4) injury to the brain from direct compression with scalp lacerations,
skull fractures and cerebral contusions or from deceleration or shear forces; 5) acute neck flexion, hyperextension or both
resulting in cervical spine injury.
Three-point passenger restraints and air bags, although overall very effective in reducing injury, can cause specific related
injuries. Common injuries when lap belts are incorrectly strapped above the anterior iliac spine include compression injuries
of intraabdominal organs (liver, pancreas, spleen, small bowel, large bowel), increased intraabdominal pressure and
diaphragmatic rupture.

44. Which of the following statement(s) is/are correct concerning the pathophysiology of frostbite?

a. Frostbite injury may have two components: initial freeze injury and a reperfusion injury that follows during
rewarming
b. The formation of extracellular ice crystals in the tissue begins to occur at -10°C
c. The release of oxygen free radicals and arachidonic acid metabolites aggravates vasoconstriction and platelet and
leukocyte aggregation
d. Experimental evidence suggests that a substantial component of severe cold injury may be mediated due to platelet
aggregation
Answer: a, c

Recent evidence suggests that frostbite injury may have two components: the initial freeze injury, and a reperfusion injury
that occurs during rewarming. The initial response to tissue cooling is vasoconstriction and arterio-venous shunting,
intermittently relieved by vasodilatation. With prolonged exposure, this response fails, and the temperature of the freezing
tissues will approximate ambient temperature until -2°C. At this point, extracellular ice crystals form, and as these crystals
enlarge, the osmotic pressure of the interstitium increases resulting in movement of intracellular water into the interstitium.
Cells begin to shrink and become hyperosmolar, disrupting cellular enzyme function.
During rewarming, red cell, platelet and leukocyte aggregation is known to occur and results in patchy thrombosis of the
microcirculation. These accumulated blood elements are thought to release, among other products, the toxic oxygen-free
radicals and the arachidonic acid metabolites which further aggravate vasoconstriction and platelet and leukocyte
aggregation. Recent experimental evidence suggests that a substantial component of severe cold injury may be neutrophil-
mediated in that a monoclonal antibody to neutrophil-endothelial and neutrophil-neutrophil adherence can markedly
ameliorate the pathology of severe injury.

45. The management of a patient with frostbite includes:

a. Gradual spontaneous warming


b. Emersion of the tissue in a large water bath with a temperature of 40–42°C
c. Immediate initiation of prophylactic antibiotics
d. Systemic anticoagulation with heparin
e. Immediate debridement of necrotic tissue
Answer: b

The treatment of frostbite with rewarming should begin in the Emergency Room and not in the field. Gradual, spontaneous
warming is generally inadequate and delayed thawing, or rubbing the injured part in ice or snow often results in marked
tissue loss. Rapid rewarming should be achieved by immersing the tissue in a large bath of 40–42°C. The water should feel
warm, but not hot to the normal hand. The skin should be gently but meticulously cleansed, air dried, and affected area
elevated to minimize edema. Infection develops in only about 13% of urban frostbite victims, but half of these infections are
present at the time of admission. Therefore, most clinicians reserve antibiotics for identified infections. Following
rewarming, the treatment goals are to prevent further injury while awaiting demarcation of the irreversible tissue
destruction. The use of sympathetic blockade, surgical sympathectomy, and intraarterial vasodilating drugs has generally
been ineffective. Heparin, thrombolytic agents, and hyperbaric oxygen have also failed to demonstrate any substantial
treatment benefit. The difficulty in determining the depth of tissue injury and cold injury has led to a conservative approach
to the care of frostbite injuries. As a general rule, amputation and surgical debridement are delayed for 2–3 months unless
infection with sepsis intervenes. The natural history of full thickness frostbite is gradual demarcation of the injured area
with dry gangrene and mummification clearly delineating a nonviable tissue.
46. There are a number of injuries associated with common orthopedic injuries. Which of the following diagnosed
orthopedic injuries is associated with the injury listed?

a. Sternal fracture—cardiac contusion


b. Posterior dislocation of the knee—popliteal artery thrombosis
c. Pelvic fracture—ruptured bladder or urethral transection
d. Posterior dislocation of hip—-sciatic nerve injury
Answer: a, b, c, d
PATTERNS OF INJURY TO THE HEAD, NECK, TRUNK, AND EXTREMITIES ASSOCIATED WITH ORTHOPEDIC
INJURIES

Diagnosed Injury Associated Injury


Fracture—temporal, parietal bone Epidural hematoma
Maxillofacial fracture Cervical spine fracture
Sternal fracture Cardiac contusion
First and second rib fracture Descending thoracic aorta, intraabdominal bleeding
Fractured scapula Pulmonary contusion
Fractured ribs 8–12, right Lacerated liver
Fractured ribs 8–12, left Lacerated spleen
Fractured pelvis Ruptured bladder, urethral transection
Fractured humerus Radial nerve injury
Supracondylar humerus Brachial artery injury
Distal radius fracture Median nerve compression
Supracondylar femur fracture Thrombosis popliteal artery
Anterior dislocation shoulder Axillary nerve injury
Posterior dislocation of hip Sciatic nerve injury
Posterior dislocation of knee Popliteal artery thrombosis

47. Correct statement(s) concerning cold injury include:

a. Chilblain is a form of local cold injury characterized by pruritic papules, macules, or plaques on the skin associated
with repeated exposure to cold temperatures
b. Trenchfoot is a freeze injury of the hands or feet due to chronic exposure to cold, wet conditions below freezing
c. Frost nip is reversible with warming of the tissue and will result in the return of sensation and function with no
tissue loss
d. Characteristic large blisters can be seen with all degrees of frostbite
Answer: a, c

Cold injuries limited to digits, extremities, or exposed surfaces are the result of either direct tissue freezing (frostbite) or
more chronic exposure to an environment just above freezing (Chilblain or pernio; trenchfoot). Chilblain or pernio are
descriptive forms of local cold injury characterized by pruritic, red-purple papules, macules, plaques or nodules in the skin.
This pathology appears to be provoked by repeated exposure to cold but not freezing temperatures. Trenchfoot or cold
emersion foot describes a non-freezing injury of the hands or feet, typically in sailors, fishermen, or soldiers resulting from
chronic exposure to wet conditions and temperatures just above freezing. Frost nip is the mildest form of cold injury
characterized by initial pain, pallor, and subsequent numbness of the affected body part. The injury is reversible and
warming of the cold tissue results in return of sensation and function with no tissue loss. Frostbite is more severe and
common form of cold injury and essentially describes local freezing of tissues. The mildest form (first degree injury) is
associated with hyperemia and edema but without blistering. Second, third and fourth degree frostbite have progressive
degrees of tissue injury and are noted by either characteristic clear blisters (second degree) or more hemorrhagic vesicles
which are generally smaller than second degree blisters (third degree frostbite). In fourth degree frostbite, tissue necrosis,
gangrene and full thickness tissue loss can be seen.

48. A 37-year-old man driving an automobile travelling at a rapid speed hits a tree. At arrival to the Trauma Center, aortic
disruption is suspected. Which of the following statement(s) is/are true concerning the patient’s diagnosis and
management?
a. If undiagnosed, a thoracic aortic disruption is associated with a 50% mortality within the first 24 hours
b. Transesophageal echocardiography is a promising new modality for the diagnosis of aortic injury
c. Repair of aortic disruption is best completed with cardiopulmonary bypass
d. Pharmacologic control of blood pressure with sodium nitroprusside should be used routinely in the preoperative
management
Answer: a, b

Blunt injuries to the thoracic aorta occur in as many as 20% fatalities due to motor vehicle accidents. About half of these
patients die at the scene. It is estimated that of the 50% who survive the initial injury, half will die within the first 24 hours
and 90% will die within 10 weeks without surgical treatment. Blunt aortic disruption is associated with the mechanism of
abrupt deceleration. Therefore, this mechanism of injury should lead to high index of suspicion. A chest radiograph is a
useful screening procedure. Abnormal findings on chest film, or suspicion of the injury, must be aggressively investigated.
Due to the very high morbidity of missed injuries, angiography is the diagnostic study of choice in patients at significant
risk. Transesophageal echocardiography is a promising modality for the diagnosis of aortic injury, especially in patients who
cannot be transported to the angiography suite. Early experience has shown transesophageal echo to be a very sensitive
method, with very few missed injuries in experienced hands.
Injuries to the aorta require surgical repair. The technique of aortic repair has been the subject of some controversy primarily
due to the risk of spinal cord ischemia with cross clamping of the thoracic aorta. The complete use of cardiopulmonary
bypass with full heparinization, however, has been shown to increase the mortality of patients who have other cerebral and
vascular injuries, and is probably contraindicated in the blunt trauma patient. Most surgeons favor cross clamping of the
aorta with expeditious repair of the injury. Rapid surgical repair is vital to survival of the patients. Preoperative management
of patients with aortic disruption involves careful control of blood pressure and avoidance of hypertension. Pharmacologic
control of blood pressure is indicated to avoid possible rupture before surgical repair. The use of sodium nitroprusside,
however, should be avoided in patients with head injuries because the vasodilatory effect of this drug may cause an increase
in intracranial pressure. A short-acting beta agonist such as esmolol or labetolol is probably the best choice if blood pressure
control is needed.

49. Which of the following statement(s) is/are true concerning endotracheal intubation at the site of injury?

a. Bag valve mask systems are equally as efficient as endotracheal intubation for early management of the trauma
patient
b. Paramedic intubation in the field is successful in over 90% of cases
c. Indications for intubation in the field include respiratory distress, significant head injury, severe chest injury and
hypovolemic shock
d. If patients clench their teeth violently, endotracheal intubation is impossible without the use of paralytic agents
Answer: b, c

The most immediately life-threatening problem to the injured patient is loss of airway patency and therefore this is the first
priority of the first response team upon arrival at the injury site. Basic life support skills such as suctioning, placement of
oropharyngeal airways, the use of a bag mask device are usually sufficient at least to temporarily restore oxygenation at the
injury site. On the other hand, approximately 10% of patients require endotracheal intubation and up to 20% would benefit
from field intubation. Endotracheal intubation is the best procedure for airway control in patients who are in shock, have
abnormal breathing patterns, or who are unable to protect their airways due to unconsciousness. Endotracheal intubation is
far superior than that of bag valve mask systems because it provides larger total volumes and less risk of aspiration.
Indications for endotracheal intubation in the field should include respiratory distress, hypovolemic shock, unconsciousness,
significant head injury, and severe chest injury. Reported paramedic intubation success rates range between 90 and 98% in
the literature, and complications are rare. On the other hand, there are problems with intubation at pre-hospital sites. Patients
with head injuries may have C-spine injuries so in-line mobilization techniques are necessary to insure intubation without
further injury to the cervical spine or cord. Patients often clench their teeth in which case either nasotracheal intubation or
the use of paralytic agents such as succinylcholine may be necessary for successful intubation.

50. Which of the following statement(s) concerning the operative approach to abdominal trauma is/are correct?

a. Pelvic hematomas associated with pelvic fractures should be explored


b. Central retroperitoneal hematomas should be explored after control of other injuries within the peritoneal cavity
c. Stable hematomas in the perinephric space lateral to the midline should be explored to rule out renal injury
d. The initial approach is control of hemorrhage by packing and controlling ongoing contamination from enteric
injuries
Answer: b, d

Once the abdomen is opened at laparotomy for trauma, obvious blood and clot is sequentially removed, first from the lower
abdomen and then from the upper abdomen by packing all four quadrants of the abdomen. Any areas found to be a source of
hemorrhage can be repacked. Obvious hollow viscus wounds should be rapidly sutured or controlled with noncrushing
clamps. Once hemorrhage is controlled by packing and ongoing contamination is stopped, time is then taken to allow
resuscitation of the patient’s circulating blood volume. Retroperitoneal hematomas may be the source of exsanguinated
hemorrhage if rupture into the free peritoneal cavity has occurred. If not, these can be left for investigation at a later time,
depending on the location. Hematomas of the pelvis that are associated with pelvic fractures should not be disturbed.
Similarly, stable hematomas of the perinephric space lateral to the midline are also best left undisturbed. Central hematomas
that may involve injuries to the major vascular structures, pancreas or duodenum are noted and explored after control of
injuries within the peritoneal cavity.

51. Which of the following statement(s) is/are true concerning trauma involving children?

a. The greater head/body ratio in children compared to adults leads to a higher frequency of head injuries in children
b. Unfused cranial sutures and open fontanels serve as a protective mechanism against intracranial hemorrhage
c. A greater propensity to hypothermia is seen in children
d. A propensity to single organ system injury is seen in the child
Answer: a, c

The smaller size of pediatric patients results in an increased likelihood of multiple system trauma because of the force of
impact is dissipated over a relatively small area. A higher frequency of head injuries in children is partially explained by the
proportionately greater head/body ratio, the thin skull, and the weaker supporting cervical musculature. In infants with
unfused cranial sutures and open fontanels, intracranial hemorrhage can be perfuse and result in shock. The protuberant
abdomen of the child obtains little protection from either the thoracic cage or pelvis, accounting for a higher incidence of
intraabdominal injuries.
The physiologic response to hypovolemia after pediatric trauma is characterized by the immediate constriction of small and
medium-sized arteries, thus maintaining normal blood pressure. Decompensation generally occurs with a blood volume
deficit of 20% to 25%. Tachycardia, tachypnea, diminished peripheral perfusion, and change in the level of consciousness
are better potential indicators of early shock than blood pressure. The thin skin, lack of subcutaneous fat, and large surface
area/body weight ratio all contribute to the propensity of the young child for hypothermia.

52. Indications for Cesarean section during laparotomy for trauma include:

a. Maternal shock after 28 weeks gestation


b. Unstable thoracolumbar spinal injury
c. Mechanical limitation for maternal repair
d. Maternal death if estimated gestational age is at least 28 weeks
nswer: b, c, d

The indications for exploratory laparotomy in a pregnant patient are the same as in all other trauma patients. However,
Cesarean section should not be added unless indicated due to the prolongation of operative time and the increase in blood
loss (approximately 1 liter). Vaginal delivery is always encouraged even in the postoperative period. During laparotomy for
trauma, indications for Cesarean section are as follows:

1. Maternal shock, pregnancy near term


2. Threat to life from exsanguination
3. Mechanical limitation for maternal repair
4. Risk of fetal distress exceeding risk of prematurity
5. Unstable thoracolumbar spinal injury

The outcome of postmortem C-section depends on the duration of the gestation and the time interval between maternal
death and delivery. Under optimal conditions, at 26 to 28 weeks gestation, estimated fetal survival is about 50%. Post-
mortem C-section is justified if the estimated age is about 26—28 weeks. If the time interval between maternal death and
delivery is less than 5 minutes, the fetal prognosis is considered excellent. If the time interval since maternal death is
prolonged to about 20 minutes, fetal prognosis is poor.

53. A 75-year-old man is involved in a motor vehicle accident. Which of the following statement(s) is/are true concerning
this patient’s injury and management?

a. Acceptable vital sign parameters are similar across all age groups
b. Hypertonic solutions should not be used for resuscitation due to concerns for fluid overload
c. The patient would be more prone to a subdural hematoma than a younger patient
d. There is no role for inotropic agents in the management of this patient
Answer: c

Although most principles of management of the elderly trauma patient are similar to their younger counterpart, some
important differences must be noted. Evaluating the circulatory system following injury in the elderly, it must be
remembered that elderly patients most likely are accustomed to a higher than normal blood pressure. Thus, while a systolic
blood pressure of 100 mm Hg is not alarming in a 25 year old, in a 75 year old, this may very well represent hypotensive
shock if the “normal” pressure is 150 mm Hg systolic. Recent reports have suggested that pulmonary arterial catheters can
be useful in the monitoring of patients with evidence of shock or hypoperfusion or history of intercurrent disease. In patients
with a low pulmonary capillary wedge pressure, volume replacement can be provided as needed, however, in the face of an
elevated pulmonary capillary wedge pressure, inotropic support may be of benefit. Lactated Ringer’s solution remains the
resuscitation fluid of choice in the elderly patient. However, the initial experience with hypertonic solutions have been very
favorable. Hypertonic fluids can reduce elevated blood pressures and improve cardiac performance with much smaller
volumes when compared to isotonic solutions.
Cerebral atrophy accompanies aging. In addition, the cerebral vasculature is fragile, particularly the veins. The combination
of these factors make the elderly more prone to develop subdural hematomas, which may initially be subtle.

54. Important physiologic alterations of pregnancy which may alter the injury response include:

a. Increased cardiac output


b. Expanded plasma volume
c. Decreased fibrinogen and clotting factors
d. Partial obstruction of the inferior vena cava
Answer: a, b, d

55. A number of systems have been developed in an effort to allow comparison of trauma injuries and trauma patients
among institutions. Which of the following statement(s) is/are true concerning trauma scoring systems?

a. The Revised Trauma Score uses the physiologic parameters of blood pressure, heart rate, and head injury to
mathematically assess injury severity
b. The Abbreviated Injury Scale (AIS) is a specific anatomic index
c. The Injury Severity Score (ISS) correlates not only the severity of the injury but adjusts for patient age and
comorbid risk factors
d. The Triss System is the most complete system in combining trauma score and anatomic component as well as
patient age
Answer: a, b, d

Many systems have been developed in an effort to allow comparison of trauma injuries and trauma patients among
institutions. The impetus for injury severity scoring system is provided by the need to identify and classify severely injured
patients in the pre-hospital phase, to predict mortality, to assess results, and to improve communication. The Revised
Trauma Score has been the most widely applied as well as the most useful scoring system for the initial evaluation of
trauma victims. It assumes that the physiologic parameters of blood pressure, respiratory rate, and head injury (assessed by
the Glasgow Coma Score) can be used mathematically to assess injury severity and predict the most timely and
sophisticated medical care. The Abbreviated Injury Scale (AIS), initially devised for blunt trauma and subsequently updated
to include penetrating trauma, assesses the severity of nonfatal injuries determined in six different body areas. Thus, it is a
specific anatomic index. The Injury Severity Score (ISS) is calculated by assigning the AIS values to each injury in six body
parts and then mathematically squaring the three most severely injured areas and adding the total. Unfortunately, this system
does not adjust for patient age or patient-related comorbid risk factors. The Triss methodology is of great importance
because it attempts to combine the trauma score, or physiologic component, and the ISS, or anatomic component. It also
incorporates the patient’s age. The Triss method yields a specific probability of survival, and is recommended for use by the
American College of Surgeons Committee on Trauma to be used to maintain a trauma registry and quality assurance
program.

56. Alterations in the immunologic response after a major trauma include:

a. Decreased CD3 and CD4 population


b. Depression of neutrophil antimicrobial functions including chemotaxis and phagocytosis
c. Decreased levels of pro-inflammatory cytokines including tumor necrosis factor, interleukin-1, and interleukin-6
d. Impaired macrophage receptor expression and antigen presentation
Answer: a, b, d

Major perturbations in the immune system occur in patients after injury contributing to the late septic mortality in trauma
patients. The changes in the immune system are significant and global, affecting both humoral and cellular components of
the system. Macrophage receptor expression and subsequent antigen presentation are impaired with similar defects in
lymphocyte function including shifts in T-cell populations with decreased CD3 and CD4 subpopulations, depression of B-
cell and immunoglobulin production, and a loss of antigen recall. Multiple neutrophil antimicrobial functions are suppressed
following trauma including chemotaxis, phagocytosis, respiratory bursts, and intracellular killing.
There are significant changes in humoral mediators following trauma with increased levels of pro-inflammatory cytokines
including tumor necrosis factor, interleukin-1 and interleukin-6 along with decreased levels of interleukin-2, interleukin-3,
and interferon g.

57. Which of the following statement(s) is/are true concerning penetrating injuries to the colon and rectum?

a. A patient with 2 or more additional organs injured, significant fecal spillage, preoperative hypotension, or
intraperitoneal hemorrhage exceeding 1 liter should not have a primary repair of a colon injury
b. If rectal injury is documented, a loop colostomy provides adequate decompression.
c. Irrigation of the rectal stump should be avoided to prevent contamination via the site of injury
d. The rectal wall should be repaired in all cases
Answer: a

The central issue in the operative management of colonic injuries is the controversy between primary repair of low-risk
colonic injuries and repair or resection with exteriorization. Primary repair may be selected when additional risk factors
have been excluded. Complications increase with primary repair when there is preoperative hypotension, intraperitoneal
hemorrhage exceeding 1 liter, more than two additional organs injured, significant fecal spillage, or when more than six
hours have elapsed since injury. Many patients with low-risk penetrating colon injuries can be treated with primary closure
in the absence of these risk factors. High-risk colon injuries are those associated with severe injuries, as indicated above,
and should be treated with resection and colostomy.
Rectal injuries should be suspected when there is any penetrating injury or a significant pelvic fracture. Sigmoidoscopic
examination is essential. The principles of operative management include wide debridement of all dead and devitalized
tissue and repair of rectal wall when possible. A totally defunctioning colostomy (not a simple loop colostomy) is necessary.
Retrorectal drainage is indicated only in selected severe injuries. The distal stump should be washed out to evacuate the
fecal contents. Broad-spectrum intravenous antibiotics, nutritional support, and serial debridements are also indicated.

58. Genitourinary injuries are common with both blunt and penetrating trauma. Which of the following statement(s) is/are
true concerning genitourinary trauma injuries?

a. All patients with microscopic hematuria and blunt trauma should be evaluated with an intravenous pyelogram
b. The indications for radiographic assessment of renal injury in the face of blunt trauma is more liberal than
penetrating trauma
c. CT scan is the current imaging technique of choice for suspected renal trauma
d. Perinephric hematomas occurring after either penetrating or blunt trauma should not be explored
e. Extraperitoneal bladder ruptures can often be treated nonoperatively using urethral catheter drainage alone
Answer: c, e
Renal injuries constitute the greatest proportion of genitourinary tract trauma. The presence of hematuria remains the most
sensitive clinical indicator of renal trauma. The specificity of hematuria is low, however, and the practice of performing an
IVP in all patients with blunt trauma and microscopic hematuria is both time-consuming and unnecessary. In several studies
examining clinical features associated with significant renal trauma, three factors have been identified—shock, gross
hematuria, and major associated injuries. The incidence of renal trauma requiring operation in the absence of any of these
factors was 0 in several series. The indications for radiographic assessment of renal injury in the face of penetrating trauma
should be far more liberal, since there are conflicting reports on the degree of correlation between the injury’s severity and
the degree of hematuria. Radiographic studies for the diagnosis of renal trauma include single-or multiple-film IVP, formal
nephrotomography, and CT scan. Single-film (“one-shot”) IVP is useful primarily for documenting the presence of two
functioning kidneys and has limited use as a screening examination for renal trauma. CT scan, however, has emerged as the
imaging technique of choice for most renal trauma. Renal injuries can be staged with respect to those likely to require an
operation or to develop complications. CT scan also allows more precise assessment of the degree of perinephric
hemorrhage and the degree of collecting system disruption than operative inspection.
A number of major renal injuries are diagnosed at the time of laparotomy. Most commonly, a perinephric hematoma is
encountered in association with blunt hepatic or splenic trauma. Indications for renal exploration at laparotomy following
blunt trauma include an expanding or pulsatile perinephric hematoma or suspected renal vascular injury. In a patient with
blunt injuries, it is preferable to defer exploration of nonexpanding, nonpulsatile perinephric hematomas to complete
treatment of intraabdominal and other associated life-threatening injuries. Postoperative CT scan may be useful for formal
staging of these injuries. A perinephric hematoma that is found during laparotomy for penetrating trauma should generally
be explored carefully. Unlike blunt injuries, continued or recurrent hemorrhage is more often a problem.
Most bladder injuries (over 95%) occur in association with pelvic fractures. Bladder ruptures are classified into those that
rupture into the free peritoneal cavity and those with extravasation limited to the retroperitoneum. Intraperitoneal bladder
ruptures are characteristically large and require early operative repair. Extraperitoneal bladder ruptures in most cases,
however, can be treated nonoperatively using simple urethral catheter drainage alone.

59. In children who sustain multiple trauma, 25% have serious intraabdominal injuries. Which of the following
statement(s) is/are true concerning blunt abdominal trauma in children?

a. Peritoneal lavage plays an important role in the evaluation of the patient


b. Most pediatric trauma patients will be hemodynamically unstable at the time of admission
c. Splenic salvage can be achieved in 90% to 100% of patients
d. The indications for laparotomy for splenic injury include refractory hypotension or transfusion requirement in
excess of 50% of blood volume within the first 24 hours
e. Unlike splenic injury, hepatic injury will frequently require exploratory laparotomy
Answer: c, d

Diagnostic peritoneal lavage is a rapid and sensitive test for the presence of intraabdominal hemorrhage. In general,
however, peritoneal lavage has a limited role in the care of pediatric patients as it provides confirmation of a finding,
hemoperitoneum, that does not mandate operation. Unlike adults, 95% of pediatric trauma patients are hemodynamically
stable on admission. The single most useful diagnostic maneuver is a CT scan, which delineates solid visceral injuries with
a high degree of both sensitivity and specificity. The spleen and liver are injured with about equal frequency in children
sustaining blunt trauma, and together these two target organs account for about 75% of childhood abdominal injuries.
Experience at virtually every major children’s trauma center in the world supports the safety and efficacy of nonoperative
management of children with splenic ruptures. Most series report splenic salvage in 90% to 100% of children. Although
therapy must be individualized, the general guidelines are that operation is not indicated until there is refractory hypotension
or a transfusion requirement in excess of 50% of blood volume in the first 24 hours. In reality, few patients approach this,
and transfusion practices are such that only 10% to 20% of patients with isolated splenic injuries require blood transfusion
at all. The management of liver injuries in pediatric patients has also changed since the advent of routine abdominal CT scan
for blunt trauma. Several reports describe successful nonoperative treatment of liver injuries detected radiologically in
children. This approach is applicable in most children, with a success rate of 90% and transfusion requirements similar to
those patients with ruptured spleens.

60. Which of the following statement(s) is/are true concerning the diagnosis and management of pelvic fractures secondary
to blunt trauma?

a. Most pelvic fractures are apparent on the basis of physical examination


b. An infra-umbilical approach to peritoneal lavage in a patient with a major pelvic fracture may yield a false-positive
rate approaching 50%
c. If a large expanding pelvic hematoma is found at surgery, the intraabdominal injury should be dealt with, and the
hematoma explored
d. The application of pelvic external fixation may be used as the initial step in control of hemorrhage from pelvic
fractures
e. A urethral catheter should be placed immediately in patients with suspected pelvic fracture to allow early
peritoneal lavage
Answer: b, d

The spectrum of pelvic fracture injuries range from minor isolated non-displaced fractures of the pubic rami to severe
injuries with multiple fractures that can be rapidly lethal. Unlike most long bone fractures, only 25% of pelvic fractures are
apparent on physical examination. Hemorrhage caused by laceration of the sacral venous plexus, multiple arterial branches
of the hypogastric vessels, or bleeding from fractured cancellous bone presents a formidable challenge to the trauma
surgeon. Massive hemorrhage is the principle cause of early death in patients with pelvic fracture, and survival depends
principally on rapid identification and control. The presence of hemorrhage from associated intraperitoneal injuries should
be considered first, therefore diagnostic peritoneal lavage is indicated for most patients with pelvic fractures. A
supraumbilical lavage is preferable under these circumstances because the possibility of catheter penetration of a large
retroperitoneal hematoma dissecting into the preperitoneal space. Peritoneal lavage performed incorrectly in the
infraumbilical site with a major pelvic fracture may yield an incidence of false-positive results as high as 45%. When
performed properly in the supraumbilical position, false-positive lavage rates have been reported to be as low as 1%. If
laparotomy is indicated once a thorough abdominal exploration is performed and injury is repaired, the size of the pelvic
hematoma may be assessed. If a rapidly expanding pelvic hematoma is seen, rapid closure of the abdominal wound is
indicated followed immediately by pelvic angiography and embolization of active arterial bleeding. In selected patients with
unstable fractures involving the sacrum or pubic diastasis injuries, the application of pelvic external fixation may reduce
hemorrhage from cancellous bone and sacral venous plexus. In many centers, pelvic fixation is preferred to arteriography
and embolization for the initial control of bleeding.
A urethral tear should be suspected in any male with a pelvic fracture. These patients should be examined carefully for signs
of urethral injury including scrotal or perineal hematoma, blood at the urethral meatus, or anterior displacement of the
prostate gland on rectal examination. The presence of any of these clinical findings constitutes a contraindication to
immediate placement of a urethral catheter. A retrograde urethrogram should be obtained in these cases by the placement of
a small balloon catheter in the fossa navicularis and gravity infusion of 10–15 ml of contrast medium.

61. Which of the following statement(s) is/are true concerning the Advanced Trauma Life Support (ATLS) classification
system of hemorrhagic shock?

a. Class I shock is equivalent to voluntary blood donation


b. In Class II shock there will be evidence of change in vital signs with tachycardia, tachypnea and a significant
decrease in systolic blood pressure
c. Class III hemorrhage can usually be managed by simple administration of crystalloid solution
d. Class IV hemorrhage involves loss of over 40% of blood volume loss and can be classified as life-threatening
Answer: a, d

The classification of hemorrhagic shock as defined by the ATLS classification system of the American College of Surgeons
is useful in comprehending the manifestations and physiologic changes associated with shock. Mild hemorrhage, up to 15%
of total blood volume, is exemplified by voluntary blood donation. In the supine position, there are no measurable changes
in heart or respiratory rates, blood pressure or pulse pressure. Class II hemorrhage involves loss of 15% to 30% of volume
loss. Clinical signs include tachycardia and tachypnea. The systolic blood pressure may be only slightly decreased,
especially in the supine position, but the pulse pressure is narrowed. Patients with Class II hemorrhage can generally be
resuscitated with crystalloid solutions but some may require blood transfusions. With Class III hemorrhage, 30% to 40% of
total body volume is lost. Patients with Class III hemorrhage present with obviously inadequate perfusion; marked
tachycardia and tachypnea, cool, clammy extremities with significant delayed capillary refill; hypotension; and significant
changes in mental status. Class III hemorrhage represents the smallest volume of blood loss that consistently produces a
decrease in systolic blood pressure. The resuscitation of these patients requires blood transfusion in addition to crystalloid
administration. Class IV hemorrhage involves loss of greater than 40% of blood volume. This represents life-threatening
hemorrhage. These patients require immediate transfusion for resuscitation and frequently require immediate surgical
intervention.
62. Which of the following statement(s) is/are true concerning traumatic pericardial tamponade?

a. The condition only develops in cases of penetrating trauma


b. Beck’s triad, consisting of muffled heart sounds, decreased pulse pressure, and jugular venous distention can be
seen in most patients
c. Two-dimensional echocardiography has replaced diagnostic pericardiocentesis in most hemodynamically stable
patients
d. The majority of patients with a small injury to a single chamber of the heart arriving with vital signs at the hospital
will die of their injuries
Answer: c

Injuries to the heart resulting in cardiac tamponade can occur from either blunt or penetrating trauma, though penetrating
injuries are much more common. Pericardial tamponade occurring after blunt trauma usually results from rupture of a
chamber of the heart, with many associated with death at the scene. Penetrating trauma is the usual cause of pericardial
tamponade and the outcome is directly related to the character of the weapon. Reported survival rates for small injuries to a
single chamber are between 60 and 87%, although patients who arrive moribund do poorly regardless of care. The diagnosis
of pericardial tamponade should be considered in any patient with penetrating chest trauma, particularly to the central
portion of the chest. The classic Beck’s triad, consisting of muffled heart sounds, decreased pulse pressure, and jugular
venous distention, occurs in only the minority of patients. The diagnosis of pericardial tamponade can be somewhat difficult
but should be suspected in patients with trauma who remain hypotensive and have no evidence of external blood loss or
hemorrhage into the thorax, abdomen, or pelvis. Unfortunately, CVP measurements are neither sensitive or specific for the
diagnosis of pericardial tamponade, and are dependent upon the patient’s volume status and the level of agitation. Two-
dimensional echocardiography is very sensitive to the presence of pericardial fluid and wall motion abnormalities. If
available in a timely fashion, cardiac echo is a very good diagnostic test to rule out cardiac tamponade in a stable patient.
Under most circumstances, there is no role for diagnostic pericardiocentesis.

63. The intravenous fluid that a 60 kg., 30-year-old woman with an 80% burn should be given in the first 24 hours
following burn injury is:
A. 19.2 liters of 5% glucose in lactated Ringer's.
B. 14.4 liters of lactated Ringer's.
C. 9.6 liters of hypertonic salt solution (sodium concentration 200 mEq. per liter).
D. 7.2 liters of 5% albumin solution.
E. 5.5 liters of the pentafraction component of hydroxyethyl starch.
Answer: B

DISCUSSION: The consensus range for estimating fluid needs of burn patients in the first 24 hours is 2 to 4 ml. of a
physiologic crystalloid solution per kilogram body weight per percent of body surface burned. In this patient that would be
9.6 to 19.2 liters of lactated Ringer's solution. The early elevation of circulating levels of catecholamines and
glucocorticoids following burn injury induces glycogenolysis, which results in elevated circulating blood glucose levels.
Glucose should not be administered in the resuscitation fluids, since the resulting exaggeration of hyperglycemia could
induce osmotic diuresis. In the first 24 hours colloid-containing solution is not commonly used, but if it is used even the
Evans formula recommends only 1 ml. per kg. body weight per percent of body surface burned. Moreover, patients in one
study who received colloid-containing fluids continued to gain weight during the first 3 postinjury days, retained more
sodium, and had less urine output than patients who received only crystalloid fluids in the first 24 hours. Hypertonic salt
solution is also not commonly used for burn patient resuscitation because of the recently described and surprisingly frequent
occurrence of acute renal failure and increased mortality associated with its use. If hypertonic salt is used, the amount
infused should be less than 9 liters, to avoid excessive elevation of the serum sodium concentration (i.e., above 160 mEq.
per liter). The recommended limit of hydroxyethyl starch infusion is currently 1500 ml. per day. Although a 10%
pentastarch form of hydroxyethyl starch has been used to expand the plasma volume of burn patients at the end of the first
24 hours, even as little as 500 ml. of that solution has been reported to prolong both prothrombin and plasma thromboplastin
time.

64. Indications for escharotomy of a circumferentially burned right lower limb include all of the following except:
A. Progressively severe deep tissue pain.
B. Coolness of the unburned skin of the toes of the right foot.
C. A pressure of 40 mm. Hg in the anterior compartment of the distal right leg.
D. Edema of the unburned skin of the right foot.
E. Absence of pulsatile flow in the posterior tibial artery.
Answer: BD

DISCUSSION: The blood flow to distal and underlying unburned tissue in a limb can be compromised by vascular
compression due to edema formation beneath the unyielding eschar of a full-thickness circumferential burn. The most
reliably noninvasive means of monitoring adequacy of the circulation in a circumferentially burned limb is serial
examination using the ultrasonic flowmeter. The absence or progressive diminution of pulsatile flow in the posterior tibial
artery in the lower limb or the palmar arch arteries in the upper limb indicates a need for escharotomy. Delayed capillary
refilling, cyanosis of the digits, and progressively severe paresthesias, particularly deep tissue pain, are all clinical signs that
may indicate vascular compromise and should be monitored if an ultrasonic flowmeter is not available. Persistent deep
tissue pain and progressively severe paresthesias are the most reliable of the nonspecific clinical signs. A muscle
compartment pressure that exceeds 30 mm. Hg, which is greater than normal capillary pressure, has also been used as an
indication for escharotomy in burn patients. Edema and coolness to the touch of distal unburned tissue commonly
accompany thermal injury and are not useful in assessing the need for escharotomy.

65. Which of the following is/are true about inhalation injury in burn patients?
A. A chest x-ray obtained within 24 hours of injury is an accurate means of diagnosis.
B. Its presence characteristically necessitates administration of resuscitation fluids in excess of estimated volume.
C. When moderate or severe, it exerts a comorbid effect that is related to both extent of burn and the age of the patient.
D. It increases the prevalence of bronchopneumonia.
E. Prophylactic high-frequency ventilation reduces the occurrence of pneumonia and the mortality in burn patients with
inhalation injury.
Answer: BCDE

DISCUSSION: Extensive inflammatory changes are evoked in the airway following the inhalation of smoke and other
irritating products of incomplete combustion. Clinical signs are nonspecific and may be delayed. Chest x-rays are also
unreliable in detecting even severe inhalation injury. Chest x-rays taken within 24 hours of injury were found to be falsely
negative in 92% of 106 patients with inhalation injury. Fiberoptic bronchoscopic examination is the most reliable single
means of diagnosing inhalation injury of the large airways, but in patients who inhaled finely particulate smoke the large
airways may show little if any inflammatory change. The distance smoke particles travel before deposition in the airways is
inversely related to particle size. When the smoke particle mass median diameter is less than 0.5 mm., deposition occurs in
the terminal bronchioles and alveoli. In such patients, inhalation injury is best identified by prolonged retention of xenon
133 as assessed by a ventilation perfusion lung scan.
Historically, fluid restriction was recommended for patients with inhalation injury, but in recent years it has become obvious
that such patients typically require more resuscitation fluid than the volume estimated by commonly used formulas. Edema
of the small airways and occlusion due to endobronchial sloughing and inspissation predispose burn patients to develop
pneumonia. In one study, 46% of burn patients with inhalation injury developed pneumonia and 69% of the pneumonias
occurred in the first postburn week. The comorbid effect of moderate to severe inhalation injury is related to both age and
burn size and increases mortality by a maximum of 20% above that predicted on the basis of age and burn size in patients
whose burn injury alone would be associated with a 75% likelihood of death. In patients with only mild inhalation injury
there is little if any increase in mortality above that predicted on the basis of age and burn size alone. Prophylactic use of
high-frequency percussive ventilation minimizes airway collapse and atelectatic changes, as a consequence of which the
incidence of pneumonia is reduced and survival is significantly increased.

66. Adequacy of fluid resuscitation in burn patients is indicated by which of the following?
A. Urine output of 45 ml. per hr. in a 70-kg. 30-year-old man with flame burns involving 55% of the total body surface.
B. Hourly urine output of 7 ml. in a 7-kg. 15-month-old child with burns involving 40% of the total body surface.
C. A pulmonary capillary wedge pressure of 17 to 20 mm. Hg.
D. Hourly output of 40 ml. of port wine–colored urine in an 80-kg. male who has severe high-voltage electric injury of the
right arm and left leg.
E. A urinary sodium concentration of 4 mEq. per liter.
Answer: ABC

DISCUSSION: The goal of burn patient resuscitation is the maintenance of vital organ function at the least immediate or
delayed physiologic cost. Fluid resuscitation of burn patients does not need to be a test of maximum renal function.
Adequacy of volume replacement and of renal blood flow are indicated by an hourly urine output of 30 to 50 ml. in adults
and 1 ml. per kg. per hr. in children weighing less than 30 kg. In patients with extensive muscle damage caused by high-
voltage electric injury, heavy loads of hemochromagens give the urine the appearance of port wine. Such patients are prone
to develop acute renal failure unless brisk urine output is maintained until the pigment concentration is reduced to
insignificant levels. Fluid should be infused into such patients at the rate needed to achieve an hourly urine output of 75 to
100 ml., but if the patient does not respond to increased fluid input with an increase in urine volume and clearing of the
heme pigments, a diuretic should be given. A pulmonary capillary wedge pressure of 17 to 20 mm. Hg is indicative of an
adequate circulating blood volume, but a urinary sodium concentration of less than 20 mEq. per liter is consistent with an
intravascular volume deficit.

67. Common electrolyte changes during and after resuscitation in a patient with a burn of 65% of the total body surface
include:
A. A serum sodium concentration of 128 mEq. per liter following 48 hours of resuscitation fluid therapy.
B. A serum sodium concentration of 152 mEq. per liter on the fifth postburn day in a 75-kg. male with a 75% burn who has
received only calculated maintenance fluids each day following successful resuscitation.
C. A serum potassium concentration of 5.7 mEq. per liter as a consequence of the destruction of red cells and other tissues
in a patient with high-voltage electrical injury.
D. Hypokalemia due to the kaliuretic effect of 0.5% silver nitrate soaks.
E. Hypocalcemia with a low ionized calcium level on the third postburn day as a consequence of dilution and
hypoalbuminemia.
Answer: ABC

DISCUSSION: At the end of the first 48 hours of resuscitation, when lactated Ringer's solution is used in the first 24 hours
and colloid-containing fluid and electrolyte-free fluid in the second 24 hours, modest hyponatremia (serum sodium
concentration of 128 to 130 mEq. per liter) is commonly observed but requires no treatment. The total body salt load is
actually increased, and appropriate fluid management permits the increased evaporative water loss to correct that imbalance.
The most common postresuscitation fluid and electrolyte disturbance is hypernatremia associated with dehydration due to
inadequate replacement of insensible water loss. The hourly insensible water loss, which far exceeds maintenance fluid
requirements in uninjured patients, can be calculated thus:
Insensible water loss (in ml./hr.)=(25 + % of body surface burned) × total body surface area (sq. m.)
The release of potassium from red cells and other tissues injured by the burn or by electrical current can cause usually
modest hyperkalemia. If acidosis occurs, the hyperkalemia may be exaggerated to symptomatic levels that require
treatment. Hypokalemia can be induced following resuscitation by the kaliuretic effect of sulfamylon burn cream, but the
hypokalemia associated with 0.5% silver nitrate soak treatment is due to transeschar leaching of potassium. Hypocalcemia
is frequently associated with hypoalbuminemia as a consequence of hemodilution by the resuscitation fluid and the
cytokine-induced reprogramming of hepatic protein synthesis. In such cases ionized calcium levels are commonly normal.

68. The clinical and histologic signs of invasive burn wound infection include which of the following?
A. Focal dark red or dark brown discoloration of the eschar.
B. Delayed separation of the eschar.
C. Conversion of an area of partial-thickness burn to full-thickness necrosis.
D. The presence of micro-organisms in the unburned subcutaneous tissue in a burn wound biopsy specimen.
E. Perineural and perivascular microbial migration through the eschar with proliferation of micro-organisms in the
subeschar space.
Answer: ACD

DISCUSSION: It is essential to examine the entire burn wound at the time of the daily cleansing to identify invasive burn
wound infection at the earliest possible time. The appearance of focal areas of dark red or dark brown discoloration are the
most common changes indicative of burn wound infection, but similar changes may be caused by hemorrhage due to local
trauma or maceration. Accelerated separation of the eschar is often produced by burn wound infections, but delayed
separation of the eschar is indicative of effective control of the microbial population in the burn wound. Conversion of an
area of partial-thickness burn to full-thickness necrosis is the most reliable clinical sign of invasive burn wound infection.
Identification of such a change mandates histologic examination of a burn wound biopsy, which is the only reliable means
of differentiating the colonization of nonviable tissue from the invasion of viable tissue. Identification of micro-organisms
in the unburned viable tissue of a burn wound biopsy confirms the diagnosis of invasive burn wound infection. Microbial
migration along the skin appendages, terminal nerve radicles, and thrombosed capillaries in the eschar and heavy growth of
micro-organisms in the subeschar space are manifestations of the colonization of nonviable tissue and represent the
mechanisms by which eschar separation occurs.
69. The treatment of invasive burn wound infection may include which of the following?
A. Subeschar infusion of half the daily dose of a broad-spectrum penicillin suspended in 1 liter of normal saline.
B. Use of 0.5% silver nitrate soaks for topical therapy.
C. Specific systemic antibiotic therapy.
D. Excision and immediate autografting.
E. Amputation when the infection has extended to involve underlying muscle.
Answer: ACE

DISCUSSION: The Pseudomonas organisms that most commonly cause invasive bacterial burn wound infection are
typically sensitive to high concentrations of broad-spectrum penicillins. When the diagnosis of invasive Pseudomonas burn
wound infection has been made, one half of the daily dose of a broad-spectrum penicillin, typically piperacillin, suspended
in 1 liter of normal saline, should be infused into the subeschar tissues beneath the infected wound. A number 20 spinal
needle should be used for the infusion, to minimize the number of injection sites. Following a second subeschar infusion of
the broad-spectrum penicillin just prior to operation, the infected tissue should be excised. The excised wounds should not
be autografted but covered with a biologic dressing or a dressing soaked with an antimicrobial solution such as 5%
mafenide acetate. The patient is returned to the operating room in 24 to 48 hours to examine the excised wound and assess
the adequacy of the débridement. The frequency of perilymphatic and perivascular proliferation of invading Pseudomonas
organisms is associated with a risk of metastatic spread to remote organs or tissues. Consequently, systemic antimicrobial
therapy should be instituted based on the sensitivity patterns of the resident microbial flora and adjusted as necessary on the
basis of the patient's culture and sensitivity results. Amputation is frequently necessary to control invasive burn wound
infection when a phycomycotic infection on a limb has traversed the investing fascia and involves significant amounts of
the subfascial tissue.

70. The treatment of patients with high-voltage electric injury differs from that of patients with conventional thermal injury
with respect to the need for:
A. Fasciotomy.
B. Hemodialysis.
C. Amputation.
D. Pulse oximetry.
E. Prehospital cardiopulmonary resuscitation.
Answer: ABCE

DISCUSSION: Both lightning injury and contact with electric current can induce cardiopulmonary arrest due to either
asystole or fibrillation. Cardiopulmonary resuscitation must be initiated at the site of injury if cardiac arrest is present.
Cardiac arrhythmias may also occur following resuscitation, necessitating electrocardiographic (ECG) monitoring for at
least 48 hours following injury in patients with a history of loss of consciousness or an abnormal ECG. Tissue damage and
tissue destruction beneath the investing fascia can result in the formation of edema that increases muscle compartment
pressure to a level that necessitates fasciotomy (> 30 mm. Hg). The current flow in a limb in contact with high-voltage
current can be so great as to damage even the periosseous muscles and make amputation necessary. Liberation of
hemochromogens as a consequence of deep tissue injury is associated with an increased incidence of acute renal failure
necessitating hemodialysis. Electric injury does not influence the need for monitoring by pulse oximetry, and in fact the
destruction of deep tissue in a limb may preclude application of the pulse oximeter to that extremity.

71. Therapeutic interventions needed for specific chemical agents include which of the following?
A. Prolonged saline irrigation of eyes injured by concentrated sodium hydroxide using a scleral lens with an irrigating
sidearm.
B. Administration of an emetic agent as immediate treatment following lye ingestion.
C. Intra-arterial infusion of calcium gluconate for relief of refractory deep tissue pain due to hydrofluoric acid injury.
D. Use of propylene glycol to remove residual phenol following water lavage.
E. Application of 5% copper sulfate solution soaks to areas of embedded particles of white phosphorus.
Answer: ACD

DISCUSSION: An eye injured by a strong chemical agent must be irrigated immediately at the site of the accident to
minimize corneal damage. Prolonged irrigation for 12 to 72 hours is recommended for eyes injured by a strong alkali
solution. Irrigation is difficult because of blepharospasm unless a modified scleral contact lens with an irrigating sidearm is
used. Emetics should be avoided in the early treatment of patients following chemical ingestion since additional injury of
the esophagus, oropharynx, and upper airway may be caused as the chemical is regurgitated. The intra-arterial infusion of
calcium gluconate has been reported to limit tissue damage and relieve pain, but local excision of the involved tissue may be
necessary for definitive control of pain and removal of the injured tissue.
Even though phenol is only slightly soluble in water, initial water lavage of burns caused by phenol should be carried out.
Following that initial lavage, the involved area should be washed with a lipophilic solvent such as polyethylene glycol,
propylene glycol, or glycerol to remove the residual phenol. A dilute 0.5% to 1% solution of copper sulfate can be used as a
wash to facilitate identification and impede the ignition of embedded phosphorus particles. If excessive amounts of copper
sulfate are absorbed through the injured tissues, intravascular hemolysis can occur and may cause renal failure.
Consequently, one should avoid the use of more concentrated solutions of copper sulfate and should never apply the copper
sulfate solution as a soak. The important principle of treatment is to prevent ignition of the particles by preventing their
exposure to air, and that can be done most safely by applying an occlusive dressing soaked with saline or water.

72. Characteristics of the hypermetabolic response to burn injury include:


A. Elevation of core temperature, skin temperature, and core-to-skin heat transfer.
B. Ambient temperature dependency of metabolic rate.
C. A marked increase of blood flow to the burn wound.
D. A curvilinear relationship to the extent of burn.
E. Oxidation of stored lipid as the major source of metabolic energy.
Answer: ACE

DISCUSSION: At thermal neutral and higher temperatures, the core temperature, skin temperature, and core-to-skin heat
transfer in burn patients remain elevated, but metabolic rate can be diminished in patients with burns of more than 50% of
the body surface by maintaining the ambient temperature above 30‫ ؛‬C. Blocking evaporative water loss by application of an
impermeable membrane is not attended by a consistent diminution in metabolic rate, indicating that the burn patient is not
externally cold but is internally warm. The hypermetabolism in burn patients is temperature sensitive but not temperature
dependent. Even though earlier measurements described a curvilinear relationship between metabolic rate and extent of
burn, recent measurements have shown that metabolic rate increases in linear fashion and rises to levels of twice normal in
patients with burns of 75% and more of the total body surface. Lipid stores are the major source of metabolic fuel that is
oxidized for energy, and not lean body mass, which undergoes proteolysis to provide the amino acids necessary for protein
synthesis and wound healing as well as gluconeogenic processes that provide fuel for tissues requiring glucose. Blood flow
to a burned limb is markedly increased as compared with flow in an unburned limb of the same patient, and the flow
increase is directed to the wound per se, not the underlying muscles.

73. A 32-year-old mountain climber who struck his head in a fall lay in the snow overnight before he could be rescued and
brought to the hospital. Upon admission he is semicomatose and not shivering, with a pulse rate of 48 beats per minute and
a blood pressure of 80/50 mm. Hg. His rectal temperature as measured by a standard thermometer is reported as 34‫ ؛‬C. All
the digits on both feet appear to be frozen. Treatment for this patient should include:
A. Administration of inotropic and chronotropic vasoactive agents.
B. Intra-arterial infusion of vasodilating agents.
C. Infusion of lactated Ringer's solution warmed to 40‫ ؛‬C.
D. Immersion in a circulating water bath heated to 40‫ ؛‬C.
E. Excision of damaged tissue within 48 hours after thawing.
Answer: CD

DISCUSSION: A standard clinical thermometer will not measure body temperatures below 34‫ ؛‬C. This patient's clinical
condition—depressed mental status, bradycardia, and hypotension—indicates that the patient is likely suffering from severe
hypothermia and, so, requires prompt active rewarming by immersion in a water bath at 40‫ ؛‬C. or the use of partial
cardiopulmonary bypass, if available, and the administration of resuscitation fluid warmed to 40‫ ؛‬C. Vasoactive agents do
not treat the basic pathology in hypothermic patients and are typically ineffective. Vasodilating agents will be of little if any
value in the treatment of the frozen tissue in the feet since histologic studies have indicated that the vasculature in freeze-
injured tissue is dilated, not constricted. Since assessment of tissue viability immediately after thawing is difficult and often
erroneous, one should await clear demarcation of dead tissue before undertaking surgical excision of damaged tissue.

74. Valid points in the management of burns on special areas include:

a. The large majority of genital burns are best managed by immediate excision and autografting
b. All digits with deep dermal and full-thickness burns should be immobilized with six weeks of axial Kirschner wire
fixation
c. Deep thermal burns of the central face are best managed with immediate excision and autografting
d. Burns of the external ear are commonly complicated by acute suppurative chondritis if topical mafenide acetate is
not applied
Answer: d

Because of the thickness and deep appendages of the skin of the central face, relatively deep burns of these areas frequently
heal. This is fortunate, because it is difficult to achieve a favorable result with primary excision and grafting of the central
face. Management of the burned hand is dictated by the depth of injury. Superficial burns are managed with elevation,
topical antimicrobials, and full passive range of motion for each joint twice daily. Deep, partial and full-thickness injuries
are best managed by excision and sheet grafting as soon as practical. Hands are immobilized in a functional position for
seven days after surgery before passive and active therapy is resumed. Fourth degree hand burns, which involve the
underlying extensor mechanism, joint capsules or bone are significantly more difficult management problems and are
managed by staged sheet autografting and often benefit from temporary axial Kirschner wire fixation of open and unstable
interphalangeal or metacarpophalangeal joints. Burns of the external ear are treated with twice daily cleansing and
application of mafenide acetate. Deep burns of the external ear are commonly complicated by acute suppurative chondritis
if topical mafenide acetate is not applied. In general, the practice for deep genital burns is to manage these limited surface
area injuries with topical therapy for a period of two to three weeks unless the wounds are remarkably deep. Unhealed
injuries are debrided and grafted with sheet autograft at this time, with generally excellent cosmetic and functional results.

75. The hypermetabolic response seen in patients with large burns, who are successfully resuscitated, is thought to be
driven by which of the following factors?

a. Deficient gastrointestinal barrier function


b. Bacterial contamination of the burn wound
c. Evaporative heat loss
d. Changes in hypothalamic function
Answer: a, b, c, d

The physiologic challenge of a burn in excess of 20% of the body surface frequently results in an initial decrease in cardiac
output and metabolic rate. Subsequently, effected by a complex cascade of mediators, a hypermetabolic response is seen
with a near doubling of cardiac output and resting energy expenditure over the next 24 to 48 hours in those who are
successfully resuscitated. The magnitude of this response peaks in those with injuries of 60% or more of the body surface at
as high as twice the normal basal metabolic rate. The etiology of the hypermetabolic response is not entirely understood but
is assumed to involve a combination of factors including a change in hypothalamic function with coincident increases in
glucagon, cortisol and catecholamine secretion, deficient gastrointestinal barrier function with translocation of bacteria and
their byproducts, bacterial contamination of the burn wound with systemic release of similar products from this source, and
some element of enhanced heat loss via transeschar evaporation of fluid. An important element of successful management
of patients who have sustained large injuries is support of this response through the provision of adequate quantity and
quality of substrate.

76. Which of the following statement(s) is/are true concerning inhalation injury?

a. The physiology of these injuries include upper airway obstruction secondary to progressive edema, reactive
bronchospasm from aerosolized irritants, and microatelectasis from loss of surfactant and alveolar edema
b. Endotracheal intubation is indicated immediately in all patients with suspected inhalation injury
c. Distal airway injuries are usually caused by heat injury
d. Peak inspiratory pressures of > 40 cm of water are indicated to maintain functional residual capacity
Answer: a

The pathophysiology of inhalation injury is complex and varies with the aerosolized toxins particular to the circumstances
of individual injuries. However, these injuries routinely demonstrate the following: 1) upper airway obstruction secondary
to progressive edema; 2) reactive bronchospasm from aerosolized irritants; 3) small airway occlusion initially from edema
and subsequently from sloughed endotracheal debris and loss of ciliary clearance mechanisms; 4) microatelectasis from the
loss of surfactant and alveolar edema; and 5) interstitial and alveolar edema secondary to loss of capillary integrity. The
physiologic consequences of these aberrations are upper and lower airway obstruction, increased airway resistance,
decreased compliance, and an increase in the dead space to tidal volume ratio and intrapulmonary shunting.
Upper airway obstruction is best managed with prompt endotracheal intubation which is maintained for 48 to 72 hours and
elevation the head. In equivocal cases, bronchoscopy is performed and patients with significant airway edema are intubated
using the bronchoscope as a stylet. Although severe steam inhalation can result in direct heat injury to the distal
tracheobronchial tree, more distal airway injuries are usually caused by aerosolized toxins rather than thermal injury, as the
upper airway is a highly effective heat sink. Although moderate inflating pressures will help expand recruitable segments,
peak inspiratory pressures in excess of 40 cm H2O should be avoided because they are associated with both overt
barotrauma as well as more subtle overpressure injuries to the pulmonary microvasculature and alveoli which themselves
exacerbate respiratory failure. High inflating pressures are also ineffective in recruiting additional lung, because the
compliance decrements are not homogeneous and high pressures simply over distended more compliant segments.

77. Which of the following statement(s) is/are true concerning the initial fluid resuscitation of a burn patient?

a. Rigid adherence to the Modified Brooke formula is advised


b. In general, children require less fluid than that predicted by standard formulae
c. Patients with inhalation injuries require less fluid than predicted by standard formulae
d. Dextrose should not be given as the primary resuscitative fluid for any age group
e. Most resuscitative formulae withhold colloid solutions until 24 hours post-injury
Answer: e

The large number of fluid resuscitation formulae in common use is attributed to the fact that no formula accurately predicts
fluid requirements in every patient. No formula can replace a physician at the bedside repeatedly evaluating the patient’s
physiology through the resuscitative period. A reasonable consensus formula is the Modified Brooke formula, however,
regardless of the formula chosen to initiate resuscitation, subsequent fluid administration is best guided by regular
assessment of the resuscitation end points, rather than prediction of any formula. Vasoactive mediators released from the
injured tissue result in diffuse capillary leaks seen shortly after major burn injury with resulting extravasation of both
crystalloid and colloid for the first 18 to 24 hours after burn. The pathophysiology explains the enormous volume
requirements seen in such patients and is the reason that most resuscitative formulae withhold colloid until 24 hours post-
injury. Children have been found to commonly require fluid in excess of that predicted by several formulae. These
requirements are generated if one uses a urine output of 1–2 cc/kg/hour as a resuscitation end point. These needs are real in
infants and very young children whose renal concentrating abilities are not completely mature. However, in toddlers and
older children whose concentrating abilities are more mature, targeting urine flow of 0.5–1 cc/kg/hour results in overall
fluid requirements closer to that of an adult and less overall edema. Patients with inhalation injury have demonstrated to
have overall volume requirements greater than that predicted by standard formulae, possibly secondary to release of
vasoactive mediators from injured burned parenchyma. During the first 24 hours, Ringer’s lactate is the primary
resuscitative fluid. Because children less than 10 kg can develop hypoglycemia if glucose is not administered, Ringer’s
lactate or half normal saline with 5% dextrose at a maintenance rate is given along with the reduced amount of Ringer’s
lactate. Dextrose containing fluids should not be given as a primary resuscitative fluid in adults, as hyperglycemia and
osmotic diuresis will result.

78. Which of the following statement(s) is/are true concerning techniques of burn excision, and temporary and definitive
wound closure?

a. Techniques to conserve blood include subeschar injection of dilute epinephrine solution, exsanguination of the
extremity and inflation of a pneumatic tourniquet
b. Fresh or cryopreserved human allograft is usually rejected within 2 to 4 weeks
c. A common use for human allograft is as a physiologic cover for selected clean superficial wounds as they
epithelialize
d. A donor site can only serve as a single source for autograft
Answer: a, b

A common argument against the policy of early burn wound excision is the prodigious blood loss which has been associated
with these procedures. However, modern blood conserving practices as well as earlier excision of wounds have diminished
this concern. Tangential excision of the torso, neck and head are done after subeschar injection of dilute epinephrine
solutions. Tangential excisions of the extremities are done after exsanguination and inflation of a pneumatic tourniquet.
Once necrotic eschar is excised to a bed of viable tissue, immediate biologic closure is mandatory. Ideally, immediate
autografting is performed. When donor sites are insufficient for this purpose, a temporary biologic cover must be chosen
while awaiting healing of donor sites for further use. Such covers should prevent desiccation and provide a vapor and
bacterial barrier over the excised wound. Fresh or cryopreserved human allograft is most appropriate for this use. Once
placed on a viable wound bed, it will vascularize and provide physiologic wound closure until rejected 2 to 4 weeks later at
which time or before, it is replaced with reharvested autograft. A second common use for biologic dressings is a physiologic
cover for selected clean superficial wounds as they epithelialize, which minimizes the pain associated with open partial
thickness burns. Allograft, screened for malignant and infectious diseases, a precarious resource, is however, not commonly
used as a biologic dressing in these circumstances. For this purpose, reconstituted porcine xenograft should be used.

79. Which of the following statement(s) is/are true concerning topical antimicrobials in common use in the United States
today?

a. Of the common topical antimicrobials, only mafenide acetate is painful upon application
b. The use of 0.5% silver nitrate is associated with trans-eschar leeching of sodium and potassium from the wound
c. Silver sulfadiazine has the best eschar penetration
d. Silver sulfadiazine, mafenide acetate, and 0.5% silver nitrate all have a broad spectrum activity, however, only
silver nitrate has anti-fungal activity
Answer: a, b, d
THREE COMMON TOPICAL MEDICATIONS USED IN THE UNITED STATES

Agent Characteristics
Silver sulfadiazine Painless on application
Fair to poor eschar penetration
No metabolic side effects
Broad antibacterial spectrum
Mafenide acetate Painful on application
Excellent eschar penetration
Carbonic anhydrase inhibitor
Broad-spectrum antibacterial
0.5% Silver nitrate Painless on application
Poor eschar penetration
Leeches electrolytes
Broad-spectrum antibacterial and antifungal

80. The anthropometric changes observed as a patient progresses from infancy to adulthood include which of the following
statement(s)?

a. The major anthropometric changes involve the head and torso


b. A decrease in the relative size of the head from 18% to 9% of the body’s surface area occurs
c. The total surface area of the legs increases from 14 to 18%
d. The upper extremities increase to 12% of the body surface area
Answer: b, c

An accurate assessment of burn size can be made early and is important to the initial management as resuscitative fluid
administration is primarily determined by overall burn size. Burn size in children is best estimated with an age-specific
chart, because the child’s body proportions change with growth. The major anthropometric change involves the head and
legs. The infant’s head represents 18% of the total body surface and legs 14%. In older adolescents and adults, the head
represents 9% of the body surface and the legs 18%. Each upper extremity in the adult is usually considered to represent 9%
of the total body surface area.

81. Arguments in favor of early wound excision include which of the following statement(s)?

a. Enhanced survival is seen in patients with large burn injuries


b. Hospital stays can be shortened with this technique
c. Early burn excision results in fewer painful dressing changes
d. A decrease in duration and intensity of the hypermetabolic response is observed
Answer: a, b, c, d

Early removal of extensive areas of devitalized tissue with immediate biologic closure of the resulting wounds is the core
surgical objective of the first burn week. The policy of early excision is now widely practiced in the United States and is
carried out as an excision of the entire wound coincident with fluid resuscitation, or more commonly, by staged excision of
all deep partial and full thickness components of the wound (less the face, palms, soles, and genitals) over the first three to
seven days after injury. The increasing popularity of early excision is based upon several documented and perceived
advantages over the traditional approach of allowing eschar to be liquefied by bacterial proteases until separation occurs,
leaving a bed of granulation tissue which is subsequently autografted. Documented advantages include improved survival in
patients with injuries involving more than 30–40% of the body surface, truncated hospital stays, lowered cost, and fewer
painful dressing changes. Although not proven, conventional wisdom suggests that a decrease in the duration and intensity
of the hypermetabolic response, improved immunologic function and less hypertrophic scarring result from early excision.

82. Which of the following are accepted adjuncts in the management of hypertrophic scar?

a. Local steroid injection


b. Compression garments
c. Topical silicone
d. Release or excision with autografting
e. Topical platelet-derived growth factor
Answer: a, b, c, d

Hypertrophic scar formation is a major source of long-term morbidity after burns. All healed and grafted burns become
hypervascular shortly after successful epithelialization. Wounds destined to become hypertrophic develop a second surge of
neovascularization between 9 and 13 weeks. Wounds that are most commonly associated with hypertrophy are deep dermal
burns that heal in three or more weeks and full thickness wounds that heal by contraction and epithelial spread from wound
edges. Current tools used in the prevention of hypertrophic scars include compression garments, topical silicone sheets,
steroid injections, and release or excision and autografting.

83. Which of the following statement(s) is/are true concerning carbon monoxide and cyanide exposure?

a. A normal oxygen saturation by standard transmission pulse oximetry precludes the possibility of significant
carboxyhemoglobinemia
b. Most patients with cyanide exposure require administration of sodium thiosulfate
c. The half-life of carbon monoxide is reduced by a factor of 5 with ventilation with 100% oxygen
d. Even if fire victims are well ventilated with high concentrations of oxygen by emergency response personnel from
the time of extrication, carboxyhemoglobin values are frequently greater than 10% on initial evaluation
Answer: c

Both carbon monoxide and cyanide are commonly inhaled by victims of closed space fires. Patients with significant
amounts of carboxyhemoglobin suffer from a marked reduction in their ability to deliver oxygen to peripheral tissues
despite a normal arterial partial pressure of oxygen. Its 2.5 hour half-life is reduced by a factor of 5 by ventilation with
100% oxygen. Fire victims who are well ventilated with high concentrations of oxygen by emergency response personnel
from the time of extrication commonly have normal carboxyhemoglobin values (< 5%) on initial evaluation despite
significant exposure to carbon monoxide at the time of injury. Carboxyhemoglobin is not sensed by standard transmission
pulse oximetry, so a normal oxygen saturation on such a monitor does not preclude the possibility of significant carboxy-
hemoglobinemia.
Hydrogen cyanide, which is commonly present in the smoke of structural fires, interferes with oxidative metabolism at the
cellular level resulting in lactic acidosis. With proper ventilation and fluid resuscitation, the cyanide-induced acidosis
corrects in most cases and specific treatment with sodium thiosulfate is not generally required.

84. Valid points concerning the initial physical examination in a burn patient include which of the following statement(s)?

a. Patients should be examined in a warm environment to prevent hypothermia


b. All corneal injuries are obvious on initial physical examination
c. Inhalation injury is suggested by the presence of singed facial hair and nasal vibrissae, carbonaceous sputum, and a
hoarse voice
d. Blistering in or around the mouth may suggest hot liquid aspiration in small children
Answer: a, c, d

An organized approach to serious burn injuries facilitates achieving the optimal outcome and begins with a systematic initial
evaluation that includes a primary survey, effective vascular and airway access, and a systematic secondary survey. Many
burn patients have sustained concurrent injuries and the initial evaluation should therefore be approached as any multiple
trauma patient. After evaluating and securing the airway, while maintaining control of the cervical spine, breathing
mechanics are assessed, a rough estimate is made of circulating volume, the level of consciousness is documented, and the
patient is completely exposed. This should be done in a warm environment to avoid hypothermia. The burn specific
secondary survey includes a complete history, vital signs, a detailed physical examination and laboratory and radiologic
studies appropriate for the mechanism of injury. The patient’s neurologic status should be carefully documented early during
the evaluation, because many patients will become progressively obtunded secondary to the administration of analgesics
and sedatives and from intravascular volume depletion. The corneal epithelium and globes should be examined prior to the
development of adnexal edema which will render adequate examination more difficult. Major corneal epithelial burns are
obvious by the opaque appearance that results. More subtle defects are apparent only after staining with topical fluorescein.
Upper airway injuries are suspected by the presence of a hoarse voice, burns to the lips or tongue, singed facial hair and
nasal vibrissae, or carbonaceous sputum. Hot liquid aspiration may complicate facial scald burns with small children, and
should be suspected if there is blistering in or around the mouth.

85. The systemic response to a significant burn includes accelerated fluid losses. Which of the following statement(s) is/are
true concerning tissue edema following a burn?

a. Edema in tissue immediately surrounding the burn is secondary to local disruption of the capillary bed
b. Edema in tissue immediately surrounding the burn is secondary to the local release of vasoactive mediators such as
prostaglandins, thromboxane A2, and reactive oxygen radicals
c. Tissue edema following a burn occurs only in the tissues at or immediately adjacent to the burn
d. Pulmonary changes following a burn occur only secondary to excessive fluid administration
Answer: b

The systemic response to burning is driven by the loss of the skin’s barrier functions with accelerated fluid losses and
decreased host resistance to infection, release of mediators from the injured tissue with secondary interstitial edema and
organ dysfunction and from bacterial overgrowth within the eschar with the resulting systemic sequelae. Edema in tissue
immediately surrounding the burn occurs secondary to local release of vasoactive mediators, such as prostaglandins,
thromboxane A2 and reactive oxygen radicals. When burn size exceeds 20% to 30% of the body surface, clinically
significant interstitial edema is seen in distant soft tissue secondary to a combination of wound released mediators and
hypoproteinemia. These distant microvascular effects also have the ability to interfere with the function of organ systems
not directly injured by the burning process, explaining the frequent occurrence of pulmonary and other organ dysfunction in
patients with large burns.

86. A patient sustains a high voltage electrical injury to the upper extremity. Which of the following statement(s) is/are true
concerning peripheral perfusion to the injured arm?

a. Evidence of peripheral ischemia would be evident within the first few hours after injury
b. Physical signs of diminished blood flow include a progressive increase in the extremity’s consistency and a
decrease in distal temperature
c. A bedside escharotomy is an appropriate treatment
d. An immediate fasciotomy performed in the operating room may be necessary
Answer: b, d

Regular assessment and documentation of peripheral perfusion is crucial during the first post-injury days. Blood flow can be
compromised by constricting circumferential eschar as subeschar tissues become progressively edematous or by progressive
intracompartmental edema in patients with electrical or deep thermal burns. Both are detected by the development of a
progressive increase in the extremity’s consistency and a decrease in its distal temperature. Pulsatile doppler signals in the
lower pressure distal vasculature, such as the plantar arch and digital vessels, should be documented hourly. The loss of
pulsatile doppler signals is consistent with an increase in tissue pressure if intravascular volume is adequate. Although lesser
voltages may cause local destructive injuries without systemic sequelae, patients exposed to higher voltage (> 1000 volts),
present with a combination of deep tissue injuries secondary to the passage of current, locally destructive entrance and exit
wounds, and other local and systemic effects. Compartmental pressure elevation, secondary to edema of injured muscle, can
result in additional ischemic injury if compartments are not promptly released by fasciotomy.

1. The most common hernia in females is:


A. Femoral hernia.
B. Direct inguinal hernia.
C. Indirect inguinal hernia.
D. Obturator hernia.
E. Umbilical hernia.
Answer: C

DISCUSSION: Indirect inguinal hernias are the most common hernia in both females and males. Femoral hernias are more
common in females than in males.

2. Which of the following statements regarding unusual hernias is incorrect?


A. An obturator hernia may produce nerve compression diagnosed by a positive Howship-Romberg sign.
B. Grynfeltt's hernia appears through the superior lumbar triangle, whereas Petit's hernia occurs through the inferior lumbar
triangle.
C. Sciatic hernias usually present with a painful groin mass below the inguinal ligament.
D. Littre's hernia is defined by a Meckel's diverticulum presenting as the sole component of the hernia sac.
E. Richter's hernia involves the antimesenteric surface of the intestine within the hernia sac and may present with partial
intestinal obstruction.
Answer: C

DISCUSSION: Sciatic hernias usually present with intestinal obstruction or a mass in the gluteal or infragluteal region.

3. Staples may safely be placed during laparoscopic hernia repair in each of the following structures except:
A. Cooper's ligament.
B. Tissues superior to the lateral iliopubic tract.
C. The transversus abdominis aponeurotic arch.
D. Tissues inferior to the lateral iliopubic tract.
E. The iliopubic tract at its insertion onto Cooper's ligament.
Answer: D

DISCUSSION: Placement of staples inferior to (below) the lateral iliopubic tract may result in injury to the lateral femoral
cutaneous nerve or the genitofemoral nerve. Staples should also not be placed within the triangle of doom, owing to the risk
of major vascular injury.

4. The following Nyhus classification of hernias is correct except for:


A. Recurrent direct inguinal hernia—Type IVa.
B. Indirect inguinal hernia with a normal internal inguinal ring—Type I.
C. Femoral hernia—Type IIIc.
D. Direct inguinal hernia—Type IIIa.
E. Indirect inguinal hernia with destruction of the transversalis fascia of Hesselbach's triangle—Type II.
Answer: E

DISCUSSION: An indirect inguinal hernia with destruction of the transversalis fascia of Hesselbach's triangle is classified
as a Type IIIb hernia. Also classified as Type IIIb hernias are sliding, pantaloon, and massive scrotal hernias. Type II hernia
is an indirect inguinal hernia with a dilated internal ring but without displacement of the inferior deep epigastric vessels or
destruction of the transversalis fascia of Hesselbach's triangle.

5. Which of the following statements about the causes of inguinal hernia is correct?
A. Excessive hydroxyproline has been demonstrated in the aponeuroses of hernia patients.
B. Obliteration of the processus vaginalis is a contributing factor for the development of an indirect inguinal hernia.
C. Physical activity and athletics have been shown to have a protective effect toward the development of inguinal hernias.
D. Elevated levels of circulating serum elastalytic activity have been demonstrated in patients with direct herniation who
smoke.
E. The majority of inguinal hernias are acquired.
Answer: D

DISCUSSION: A correlation between cigarette smoking and an inguinal hernia formation has been demonstrated. Elevated
circulating serum elastalytic activity and free active unbound neutrophil elastase has been detected in smokers.

6. The following statements about the repair of inguinal hernias are true except:
A. The conjoined tendon is sutured to Cooper's ligament in the Bassini hernia repair.
B. The McVay repair is a suitable option for the repair of femoral hernias.
C. The Shouldice repair involves a multilayer, imbricated repair of the floor of the inguinal canal.
D. The Lichtenstein repair is accomplished by prosthetic mesh repair of the inguinal canal floor in a tension-free manner.
E. The laparoscopic transabdominal preperitoneal (TAPP) and totally extraperitoneal approach (TEPA) repairs are based on
the preperitoneal repairs of Cheattle, Henry, Nyhus, and Stoppa.
Answer: A

DISCUSSION: The Bassini repair is accomplished by high ligation of the hernia sac followed by suturing the conjoined
tendon and the internal oblique muscle to the inguinal ligament.

7. Which of the following statements concerning the abdominal wall layers are correct?
A. Scarpa's fascia affords little strength in wound closure.
B. The internal abdominal oblique muscles have fibers that continue into the scrotum as cremasteric muscles.
C. The transversalis fascia is the most important layer of the abdominal wall in preventing hernias.
D. The lymphatics of the abdominal wall drain into the ipsilateral axillary lymph nodes above the umbilicus and into the
ipsilateral superficial inguinal lymph nodes below the umbilicus.
Answer: ABCD

DISCUSSION: The integrity of the abdominal wall is maintained principally by the transversalis fascia. Scarpia's fascia
affords little strength in wound closure, but its approximation contributes considerably to the creation of an aesthetically
acceptable scar. The cremasteric muscles of the spermatic cord are a continuation of muscle fibers from the internal
abdominal oblique musculature. The lymphatic supply of the abdominal wall follows a simple pattern. These superficial
lymphatics run parallel to the superficial veins, which above the umbilicus drain into the ipsilateral axillary vein and below
it into the ipsilateral femoral vein.

8. Which of the following congenital abnormalities are correctly defined?


A. Omphalocele represents a defect in the abdominal wall lateral to the umbilical cord.
B. The herniated viscera associated with omphaloceles are usually covered with a membranous sac.
C. An umbilical polyp is a small excrescence of omphalomesenteric duct mucosa that is retained in the umbilicus.
D. Meckel's diverticulum results when the intestinal end of the omphalomesenteric duct persists and represents a true
diverticulum.
Answer: BCD

DISCUSSION: Omphalocele may be seen in newborns and represents a defect in the closure of the umbilical ring. The
herniated viscera are usually covered with a sac. Gastroschisis, a defect of the abdominal wall lateral to the umbilical cord,
is caused by failure of closure of the body wall. The intestines protrude through the defect, and no sac is present to cover the
herniated intestine. In the fetus, the omphalomesenteric duct may present as abnormalities related to the abdominal wall
when the duct fails to obliterate. Meckel's diverticulum is the result of the failure of obliteration of the intestinal end of the
omphalomesenteric duct. This is a true diverticulum with all layers of the intestinal wall represented. An umbilical polyp is
a small excrescence of omphalomesenteric duct mucosa retained in the umbilicus. Such polyps resemble umbilical
granulomas except that they do not disappear after silver nitrate cauterization. Appropriate treatment is excision of the
mucosal remnant.

9. The following statement(s) is/are true concerning the indications for treatment of an inguinal hernia.

a. Most adult hernias will remain stable in size, therefore delay seldom affects the technical aspects of a surgical
repair
b. There is a direct correlation between the length of time that a hernia is present and the risk of major complications
c. The morbidity and mortality associated with emergent operation due to hernia complications is significantly greater
than for elective repair of the identical hernia
d. A truss maintains a hernia in the reduced state, therefore, minimizing the risk of incarceration and strangulation
Answer: b, c

The indications for hernia repair must be individualized for each patient and the particular situation. In general, the presence
of a hernia may be considered an adequate indication for hernia repair. Certainly the presence of complications due to hernia
necessitates the correction of those complications and usually the repair of the hernia. As with any treatment, the benefits of
operative repair must be weighed against the natural history of the disease, the extent to which the treatment can correct the
problem, the possibility of treatment-related injury, and the interference of concomitant disease with the treatment results.
With a few exceptions, the natural history of an abdominal wall hernia is that the size of the defect and the sac enlarges over
time, and this enlargement increases the difficulty of adequate repair and the chances of recurrence of the hernia. The risk of
major complications is greater in an individual patient, the longer the exposure to a hernia and the larger the sac relative to
the hernia defect. In addition, major complications necessitate an emergent operation with attended high mortality and
morbidity relative to that experienced with an elective repair. The use of a truss, an external support device using a system
of straps to exert regional pressure over the hernia defect, should generally be avoided. Trusses do not consistently maintain
a hernia in the reduced state, and they may put an unreduced hernia in greater jeopardy of strangulation. The pressure
exerted induces edema by decreasing lymphatic and venous flow out of the herniated bowel. Trusses may also lead to injury
to the skin overlying the hernia.

10. Which of the following statement(s) is/are true concerning the diagnosis and management of epigastric hernias?

a. A large peritoneal sac containing abdominal viscera is common


b. At the time of surgical repair, a careful search for other defects should be performed
c. Recurrent epigastric hernias after simple closure is uncommon
d. Patients with symptoms of a painful midline abdominal mass frequently will contain incarcerated small bowel
Answer: b

Epigastric hernias are usually small but they vary considerably in size. Most of these defects occur in the midline. The small
defects contain only preperitoneal fat with no sac. With increasing size, fat in the falciform ligament and eventually a
peritoneal sac and abdominal viscera may be contained within the hernia. The preperitoneal fat in the small defect is usually
incarcerated. Multiple defects may be present in up to 20% of patients. Surgical treatment is recommended in all adult
patients with symptoms or with a hernia defect greater than 1.5 to 2 cm. in diameter. Methods of repair depend upon the size
of the defect. For small defects, simple closure with obliquely placed sutures after reduction or removal of the preperitoneal
fat from the defect has been recommended. However recurrent epigastric hernias in up to 10% of the cases have been
reported with this method, most likely as a result of additional undetected or unrepaired weaknesses in the epigastric
midline.

11. The following statement(s) is/are true concerning neurovascular structures in the inguinal region.

a. The inferior epigastric artery and vein run upward in the preperitoneal fat posterior to the transversalis fascia close
to the lateral margin of the internal inguinal ring
b. The iliohypogastric and ilioinguinal are motor and sensory nerves in the inguinal region which lie beneath the
external oblique aponeurosis
c. The ilioinguinal nerve runs anterior to the spermatic cord in the inguinal canal and at the superficial inguinal ring,
branches into the sensory supply to the pubic region and the upper scrotum or labium majoris
d. The genital branch of the genitofemoral nerve is a sensory nerve only to the upper thigh and genital area
Answer: b, c

Arising anteriorly from the external iliac artery, the inferior epigastric artery with its accompanying vein runs obliquely
medially and upward in the preperitoneal fat, posterior to the transversalis fascia and close to the inferior margin of the
internal inguinal ring. Inguinal hernias arising superior to the inferior epigastric vessels are indirect inguinal hernias,
whereas those arising inferior to the vessels are direct inguinal hernias. The iliohypogastric and ilioinguinal nerves are
motor and sensory nerves to the muscles and skin of the inguinal region. The nerves penetrate the transversus abdominis
muscle at the point above the middle of the iliac crest, lie below the internal oblique muscle up to the point just medial and
superior to the anterior superior iliac spine, and then penetrate the internal oblique muscle and lie below the external oblique
aponeurosis. The ilioinguinal nerve runs anterior to the spermatic cord in the inguinal canal and at the superficial inguinal
ligament, branches into sensory supply to the pubic region and the upper scrotum or labium majoris. The genital branch of
the genitofemoral nerve perforates the transversalis fascia usually just inferior to the internal ring. It courses along the
posterior surface of the spermatic cord and supplies motor fibers to the cremaster muscle. At the superficial inguinal ring, it
divides to provide sensory innervation to the scrotum and medial aspect of the upper thigh.

12. In advising a patient preoperatively of potential complications of operative treatment of an inguinal hernia, which
of the following statement(s) is/are true?

a. Severe symptoms due to sensory nerve entrapment or injury can occur


b. The most common vascular structure injured during the course of a groin hernia repair is the femoral artery
c. Recurrent hernia after primary groin repair should occur in less than 10% of cases
d. Wound infection increases the risk of recurrent hernia
Answer: a, c, d

Many complications can occur with operations to repair an inguinal hernia. Sensory nerve injury may lead to disabling
symptoms from neuromas or nerve entrapment during inguinal hernia repair. Although vascular injuries are uncommon in
inguinal repair, the proximity of the femoral vein to the structures used in the hernia repair makes injury of this vessel the
most frequent vascular injury observed. Hernia recurrence after primary groin hernia repairs should be infrequent and varies
in several large series from less than one percent to almost nine percent. The prevalence of recurrent hernia may be higher
after repair of recurrent groin hernia. Factors responsible for hernia recurrence include closure under excessive tension,
failure to identify and use an adequately strong musculoaponeurotic tissue, and wound infection.

13. Chylous ascites is the accumulation of chyle within the peritoneal cavity. Which of the following statement(s)
is/are true concerning chylous ascites?

a. The cisterna chyli lies at the anterior surface of the first and second lumbar vertebrae and receives lymphatic fluid
from the mesenteric lymphatics
b. Chylous ascites is most commonly associated with abdominal lymphoma
c. Paracentesis and analysis of chylous fluid typically reveals elevated triglycerides, protein, and leukocyte levels
with cytologic analysis reflecting the underlying presence of malignancy
d. Treatment of chylous ascites with dietary manipulation will be successful in most cases
e. The mortality rate in adults with chylous ascites is in excess of 50%
Answer: a, b, e

Chylous ascites is accumulation within the peritoneal cavity of chyle, a lymphatic fluid with a high lipid content. Access of
intestinal lipids to the circulation is via mesenteric lymphatics that enter the cisterna chyle, which in turn becomes the
thoracic duct which eventually enters the venous system at the junction of the left subclavian and internal jugular veins. The
cisterna chyli lies at the anterior surface of the first and second lumbar vertebrae slightly to the right of the aorta. Chylous
ascites may result from injury to major lymphatic duct or the cisterna. However for lymphatic leakage to persist, widespread
occlusion of lymphaticovenous collaterals within the abdomen must be present. Malignancy is the predominant cause (88%)
of spontaneous chylous ascites in adults, with lymphoma the most common malignancy. Diagnostic studies must include not
only documentation of lymphatic origin of the abdominal fluid but also an attempt to delineate the cause of chylous ascites.
Paracentesis and analysis of chylous fluid typically reveals elevated triglycerides, protein, and leukocyte levels, with a
predominance of lymphocytes. Unfortunately, cytology is seldom positive despite the presence of malignancy.
Lymphangiography may define the site of lymphatic leak for patients in whom the leak is from the cisterna or
retroperitoneal lymphatics but not when from the mesenteric or hepatic lymphatics. Of noninvasive studies, CT is the test of
choice, with a high diagnostic yield in nontraumatic chylous ascites in adults. Frequently, laparotomy with node biopsy is
required for histology and typing in cases suspected to be cancer, particularly for lymphoma. Treatments for chylous ascites
have been directed toward decreasing lymph and triglyceride accumulation. Successful resolution of chylous ascites has
been achieved using a fat-restricted diet with added medium-chain triglycerides in an attempt to reduce lymphatic transport
of triglycerides and perhaps intestinal lymph flow. Although there have been reports of success using such dietary
manipulation, many failures have been reported. Therefore, in most patients with chylous ascites, treatment is likely to be
successful only when directed toward the underlying cause. For patients with lymphoma, therapy effective against
lymphoma is likely to eliminate chylous ascites.
The prognosis for patients with chylous ascites is much better in infants and children than in adults, principally because of
the differences in causes of the condition. A mortality of 21% is reported in infants and children whereas a mortality of 88%
has been noted in adults. Patients with chylous ascites with associated neoplasms typically have the gravest prognosis.

14. Which of the statement(s) is/are true concerning laparoscopic hernia repair?

a. General anesthesia is required


b. Either an abdominal or preperitoneal approach is possible
c. The use of prosthetic mesh is required in all variations
d. Long-term results suggest that the laparoscopic approach is equal or better than traditional repairs
Answer: a, b, c

The laparoscopic approach to the repair of groin hernias has been recently developed. Either a transabdominal approach,
wherein the peritoneum in the inguinal area is opened, and the repair is performed in the preperitoneum or an entirely
preperitoneal approach can be used. In either technique, which are both performed under general anesthesia, after reducing
the visceral contents out of the hernia, the repair is performed by placing a sheet of prosthetic mesh over the internal aspect
of the inguinal floor and internal ring. Although early results and short-term benefits appear promising, long-term follow-up
data is still not available to compare these techniques with traditional repairs.

15. A 28-year-old woman with a history of an appendectomy presents with a nontender palpable mass in the right
lower quadrant abdominal incision. The following statement(s) is/are true concerning the diagnosis and management of this
patient.

a. The best diagnostic test involves imaging of the abdominal wall by either CT or MRI
b. Resection of the mass with a 2 cm margin is usually adequate
c. Low dose radiation is a suitable alternative to surgery for primary treatment
d. Re-resection for recurrence will likely have a higher rate of recurrence than for primary resection
Answer: a

Desmoid tumors are fibromatous tumors that may resemble low-grade fibrosarcoma but never metastasize. The tumor often
infiltrates adjacent muscle and has a high incidence of recurrence despite seemingly adequate gross resection. The highest
frequency is in women of childbearing age of which over 90% of tumors are abdominal in location. For abdominal wall
desmoid tumors, approximately one-third are associated with a previous operation at the tumor site. The most frequent
presenting symptom is a nontender, palpable abdominal wall mass. Diagnostic imaging is best carried out by CT or MRI,
which delineate the extent of involvement of the layers of the abdominal wall and potential intraperitoneal extension. Initial
treatment of abdominal wall desmoid tumors is surgical. Because the margins of the tumor are not easily determined and
because the tumor often infiltrates muscle and periosteum, limited margins around the gross tumor frequently result in
microscopic tumor at the margin. Recurrence rates for abdominal desmoid tumors vary from 9% to 40%, and recurrence is
frequent with inadequate margins. A 5-cm margin of resection is considered adequate with mono bloc resection of rib cage,
pubic or iliac bone or involved portions of organs such as bladder to achieve these margins. Reconstruction of the
abdominal wall with polypropylene mesh is necessary in most cases. In patients in whom adequate margins of resection are
achieved, there is no benefit from adjuvant radiotherapy. Second and third resections after recurrence have been associated
with no higher rate of recurrence than primary resection. Radiotherapy alone has achieved local control in desmoid tumor in
as many as 100% of tumors treated primarily and 75% of recurrent tumors. Radiation doses at least 60 Gy are considered
necessary for consistent control. The large radiation dose risks major damage to adjacent bowel and therefore primary
radiation treatment of abdominal wall desmoid tumors has a limited role.

16. Which of the following statement(s) is/are true concerning repair of inguinal hernias?
a. The Bassini repair approximates the transversus abdominis aponeurosis and transversalis fascia and the shelving
edge of the inguinal ligament.
b. The Bassini repair is an adequate repair for a femoral hernia
c. A relaxing incision is important for repairs of direct and large indirect inguinal hernias to prevent excessive tension
in the closure
d. An advantage to the use of prosthetic material is the mesh incites formation of scar tissue to further increase tensile
strength provided by the mesh alone
Answer: a, c, d

The Bassini repair is an inguinal hernia repair used world-wide and has been the standard against which other repairs are
judged. The repair involves approximation of the transversus abdominis aponeurosis and transversalis fascia and the lateral
edge of the rectus sheath to the shelving edge of the inguinal ligament. A femoral hernia cannot be repaired by the Bassini
repair because the orifice to the femoral canal lies deep to the inguinal ligament. A Cooper’s ligament repair does
approximate the structures to the transversalis fascia of the pectineal (Cooper’s) ligament between the pubic tubercle and the
femoral vein and therefore is appropriate for repair of a femoral hernia. A relaxing incision for repairs of direct and large
indirect inguinal hernias prevents excessive tension in the closure. There are an increasing number of proponents for the use
of prosthetic material for the routine repair of inguinal hernias. Prosthetic material, such as polypropylene mesh, have been
used for years for repair of large or recurrent inguinal and femoral hernias. The prosthetic mesh provides a low-tension
repair for such large defects which otherwise could not be closed without excessive tension. In addition, the mesh incites the
formation of scar tissue to further increase tensile strength beyond that provided by mesh alone. Results reported for
inguinal hernia repairs using mesh have been excellent, although there is a slight risk of infection of the prosthetic material
which must be considered.

17. The following statement(s) is/are true concerning the epidemiology of inguinal hernias.

a. Inguinal hernias occur with a male-to-female ratio of about 7:1


b. Femoral and umbilical hernias are more common in women, with a female-to-male ratio of 4:1
c. The frequency of inguinal hernias increases with age
d. Almost all umbilical hernias occur in the pediatric age group
Answer: a, c

Inguinal hernias are the most frequently occurring hernia by a factor of five over other individual types. Umbilical hernias
constitute about 14% of hernias, femoral hernias about 5%, and other types are rare. There is a male prevalence in inguinal
hernias of about 7:1 (male-to-female), whereas there is a female dominance in femoral and umbilical hernias of 8:1 and 7:1
(female-to-male), respectively. For inguinal hernia, which occurs at all age levels, frequency increases with age. Umbilical
hernias have a bimodal distribution, peaking in the pediatric population and then in the 40 to 60 year group, in which the
hernias are principally paraumbilical.

18. A 77-year-old multiparous female presents with a bowel obstruction. She has no previous abdominal operations
and no abdominal wall hernias can be detected. In addition to her abdominal symptoms, she reports pain in her right medial
thigh. The following statement(s) is/are true concerning her diagnosis and management.

a. Expectant management with nasogastric suction and IV fluid replacement is indicated


b. A right groin approach is indicated for exploration and repair of the presumed hernia
c. The use of a polypropylene mesh will likely be necessary for repair
d. A correct diagnosis can usually be made by visualizing an external mass in the upper, medial thigh
Answer: c

An obturator hernia is a hernia that occurs through the obturator canal, accompanied by the obturator vessels and the
obturator nerve. Although rare, most obturator hernias occur in older multiparous women and are predominantly right-sided.
Symptoms are frequently intermittent but tend to be acute and become increasingly severe with incarceration of the hernia.
Intestinal symptoms predominate, but dysesthesia or pain in the medial thigh with occasional radiation to the hip is often
present. Dysesthesia results from compression of either division of the obturator nerve. Although the hernia is never
externally visible, in a small percentage of patients a mass can be palpated in the upper, medial thigh. A correct diagnosis of
obturator hernia is made in only about one-third of patients presenting with intestinal obstruction. Plain radiographs are
seldom helpful, however a CT scan will usually confirm the diagnosis. Treatment is operative. There is no place for
expectant therapy, especially in a patient with pain an parasthesias along the inner aspect of the thigh or with clinical or
radiographic evidence of bowel obstruction. Many surgical approaches have been promoted, but the transabdominal
approach should be used because it has several advantages. It best confirms the diagnosis and exposes the obturator canal,
orifice, vessels, and nerve, also permitting bowel resection when required. The sac is dealt with in a standard fashion. The
hernia defect should be repaired, but repair usually requires a polypropylene mesh patch because the margin of the defect
cannot be approximated primarily.

19. The following statement(s) is/are true concerning umbilical hernias in adults.

a. Most umbilical hernias in adults are the result of a congenital defect carried into adulthood
b. A paraumbilical hernia typically occurs in multiparous females
c. The presence of ascites is a contraindication to elective umbilical hernia repair.
d. Incarceration is uncommon with umbilical hernias
Answer: b

An umbilical hernia in a child is usually considered to be congenital. Only about 10% of umbilical hernias in adults are
thought to be the result of a congenital defect carried into adulthood. Most adult umbilical hernias are acquired and are
called paraumbilical hernias. The paraumbilical hernia typically occurs in a multiparous female. Other patients with
increased intraabdominal pressure, particularly with concomitant chronic abdominal distension as from ascites, are also at
increased risk for the development of paraumbilical hernias. Umbilical and paraumbilical hernias vary from small to
extremely large. Incarceration is frequent in the large hernias, which typically have a small neck.
Indications for umbilical hernia repair in adults include symptoms, incarceration, large hernia relative to the neck, and
trophic changes in the overlying skin. Among adults with associated ascites, repair is advocated to avoid potentially serious
complications. The presence of discoloration or ulceration of overlying skin or a rapid increase in size of the hernia herald
impending rupture. Spontaneous rupture of the hernia in these patients can be catastrophic and is frequently associated with
mortality rates approaching 30%. By comparison, elective umbilical hernia repair can be performed safely in patients with
ascites with acceptable morbidity and mortality.

20. Retroperitoneal fibrosis is a fibrosing condition of retroperitoneum, which is of significance as it generally


encompasses the ureters and eventually causes hydronephrosis and kidney damage. Which of the following statement(s)
is/are true concerning this condition?

a. The majority of cases are idiopathic in nature


b. A history of use of methysergide for treatment of migraine headaches would be significant
c. There is no known association of malignancy with retroperitoneal fibrosis
d. The disease occurs more commonly in women than in men
Answer: a, b

Retroperitoneal fibrosis is a rare condition in which fibrosis develops in the retroperitoneal space. The ureters frequently
will become encompassed by the process eventually causing hydronephrosis and kidney damage. Retroperitoneal fibrosis
occurs most commonly in the fifth and sixth decades with a 2:1 male-female predominance. The pathophysiology of
retroperitoneal fibrosis remains to be delineated. In fully two-thirds of cases, retroperitoneal fibrosis is idiopathic, however,
an autoimmune process has been suggested as a potential cause. About 12% of cases of retroperitoneal fibrosis have been
associated with the use of methysergide, a serotonin agonist used for vascular and migraine headache, and in this subgroup
females outnumber males 2:1. Primary or metastatic malignancy in the retroperitoneum is found in 8% of patients with
retroperitoneal fibrosis. Sarcomas are the most common primary tumors, but non-Hodgkin and Hodgkin lymphomas and
ureteral cancer have also been found. Metastases have originated from cancer of the stomach, breast, colon, carcinoid,
pancreas, prostate, ovary, and cervix. The focus of tumor may be small but may induce desmoplasia that is grossly
indistinguishable from benign variance of retroperitoneal fibrosis.

21. The following statement(s) is/are true concerning the anterior abdominal wall musculature.

a. The lateral musculature of the abdominal wall consists of three muscle layers. These are, from external to internal,
the external oblique, the transversus abdominis, and the internal oblique muscles
b. The transversalis fascia lies on the deep side of the transversus muscle and extends to form an essentially complete
fascial envelope of the abdominal cavity
c. Above the semicircular line, the internal oblique aponeurosis splits into posterior and anterior laminae
d. The rectus abdominis muscles originate on the ribs superiorly and on the pubis inferiorly and are clearly distinct
throughout their entire length
Answer: b, c

The anterior abdominal wall consists of a group of lateral sheet-like muscles and paired, longitudinally-oriented flat muscles
on either side of the midline. The lateral musculature of the abdominal wall consists of three layers, each of which has its
fascicles running in an oblique angle to the others. The most superficial of these lateral muscles is the external oblique
muscle. The internal oblique muscle lies deep to the external oblique muscle while the transversus abdominis muscle is the
innermost of the lateral abdominal wall musculature. The transversalis fascia lies on the deep side of the transversus muscle
and extends to form an essentially complete fascial envelope of the abdominal cavity. The semicircular line is defined by the
lower edge of the posterior sheath about 3 to 6 cm below the level of the umbilicus, and its convexity is directed superiorly.
Above the semicircular line, the internal oblique aponeurosis splits into posterior and anterior laminae. The posterior lamina
joins with the transversus abdominis aponeurosis to form the posterior rectus sheath. The anterior lamina fuses with the
external oblique aponeurosis to form the anterior rectus sheath. Below the semicircular line, the internal oblique end
transversus abdominis aponeurosis fuse to form an internal lamina of the anterior sheath, with the external oblique
aponeurosis forming the external lamina of the anterior sheath. The medial paired rectus abdominis muscles originate on the
ribs superiorly and on the pubis inferiorly. Below the semicircular line, the rectus muscles are nearly fused in the midline
and indistinct, and their posterior surfaces covered only with the transversalis fascia.

22. A 48-year-old woman maintained on Warfarin for a history of cardiac valvular replacement and a history of recent
upper respiratory infection presents with severe abdominal pain exacerbated by movement. Her physical examination
shows tenderness in the right paramedian area with voluntary guarding but no peritoneal signs. The following statement(s)
is/are true concerning the diagnosis and management of this patient.

a. Urgent laparotomy should be performed because of concern for arterial mesenteric embolus
b. The correct diagnosis could likely be made by CT scan and operation avoided
c. The status of her anticoagulation should be checked and if her prothrombin time is excessively prolonged,
correction is necessary
d. If untreated, hemodynamic instability is common
Answer: b, c

Rectus sheath hematoma results from arterial or venous bleeding into the rectus sheath, most commonly from arterial
bleeding. Rectus sheath hematomas predominate in women by a ratio of about 3:1. The mean age of incidence is in the late
fifth decade. Although spontaneous formation of a rectus hematoma is rare, it can occur with vasculitis, arterial venous
malformations, a severe coagulopathy, or with the administration of anticoagulants. The usual cause is trauma. Events as
trivial as sneezing, coughing, or twisting to the side have initiated a rectus hematoma. Abdominal pain is almost always
described at presentation. Pain is often described as severe and usually is exacerbated by movements that require muscular
contraction of the abdominal wall. On examination, there is tenderness over the rectus sheath, voluntary guarding, and often
a diffuse mass sensation in the area of tenderness. Contraction of the rectus muscle exacerbates the pain and tenderness.
Peritoneal signs are absent. Ecchymosis may occur but usually appears several days after the onset of pain. In cases where
the hematoma dissects or originates inferiorly and expands into the prevessicle and preperitoneal space, the hematocrit may
fall significantly; however, hemodynamic instability is distinctly unusual. When the intraabdominal source of pain is
unknown, ultrasound and particularly computed tomography can delineate the hematoma and localize it to the abdominal
wall in almost all cases.
Treatment must take into consideration the cause, if known, and whether the hematoma is stable or progressive.
Coagulopathy should be corrected when possible. For patients in whom the hematoma is stable, pain medication and
avoidance of muscular stress on the abdominal wall are sufficient. For patients with progressive hematoma, the treatment of
choice is evacuation of the hematoma from within the rectus sheath and hemostasis, sometimes requiring ligation of the
epigastric vessels above and below the hematoma.

23. True statements concerning the diagnosis and management of retroperitoneal fibrosis include:

a. Most patients present with dull, non-colicky back, flank, or abdominal pain
b. Evidence of impaired renal function with an elevated blood urea nitrogen is common
c. The diagnosis is most commonly suggested by intravenous pyelography although contrast studies with CT scan or
MRI are useful in further defining the disease
d. Most patients can be managed nonoperatively
e. The prognosis for nonmalignant retroperitoneal fibrosis is grim with progression of disease until death occurring in
most patients
Answer: a, b, c

Ninety percent of patients with retroperitoneal fibrosis present with dull, non-colicky pain in the back, flank, or abdomen.
Other symptoms include weight loss, non-specific gastrointestinal complaints, and uncommonly, lower extremity edema,
malaise, and dysuria. Laboratory studies may be normal in 25% of patients, but 55% of patients will have an elevated blood
urea nitrogen. Diagnosis is most commonly suggested by intravenous pyelography. The combination of medial deviation of
the ureter, hydroureteronephrosis, and extrinsic ureteral compression are highly suggestive of retroperitoneal fibrosis. CT
scanning or MRI can both define the level of ureteral involvement and depict the mass appearance of the fibrotic process.
Exploratory laparotomy with multiple deep biopsies of the retroperitoneal process is an essential part of diagnosis, since
foci of carcinoma may be sparse within the predominately sclerotic reaction.
Treatment for retroperitoneal fibrosis must identify and deal with potential causative agents, relieve the ureteral obstruction,
and reverse the inflammatory-fibrotic process. Renal obstruction may need to be relieved acutely, either by retrograde
ureteral stents or by percutaneous nephrostomy tubes. Long-term resolution of ureteral obstruction most frequently has been
accomplished by operative freeing of the ureters from the fibrosis and displacing them laterally or within the peritoneal
cavity. Although renal function is improved in more than 90% of cases so treated, in as many as one-third of patients,
ureteral obstruction recurs on the ipsilateral or contralateral side. Prognosis for patients with nonmalignant retroperitoneal
fibrosis is good. Survivals of 86–100% for several years have been reported.

24. The following statement(s) is/are true concerning incarceration of an inguinal hernia.

a. All incarcerated hernias are surgical emergencies and require prompt surgical intervention
b. Attempt at reduction of an incarcerated symptomatic hernia is generally considered safe
c. Vigorous attempts at reduction of an incarcerated hernia may result in reduction en masse with continued
entrapment and possible progression to obstruction or strangulation
d. Incarcerated hernias frequently cause both small and large bowel obstruction
Answer: b, c

Hernia incarceration denotes the condition wherein viscera are contained within a hernia sac and cannot be disgorged from
the sac. Patients with an incarcerated hernia may be asymptomatic except for the presence of a bulge. Pain associated with
an incarcerated hernia should be interpreted as indicative of strangulation. Many hernias are of such size that they cannot be
reduced either spontaneously or manually. If the patient is asymptomatic, elective surgery should be planned. In a patient
with pain, attempt at reduction is relatively safe as long as excessive force is not applied. An incarcerated hernia with
discomfort or signs of bowel obstruction is best treated with urgent hernia repair, although gentle attempts at reduction may
be without consequences. Reduction of a symptomatic hernia may result in reduction of gangrenous bowel into the
peritoneal cavity. Reduction of bowel with necrotic areas eventuates in bowel perforation and peritonitis with an associated
10% to 30% mortality and high levels of morbidity. Vigorous attempts at reduction may result in reduction en masse, in
which the viscera remain within the peritoneal sac after reduction with the entire sac and its contained viscera forced
through the abdominal wall defect into the preperitoneal layer. Reduction en masse usually occurs when a small fibrous
neck traps enclosed viscera and is associated with a high risk of continued entrapment and progression to obstruction or
strangulation.
World-wide hernias are the leading cause of intestinal obstruction. The obstruction is almost exclusively small intestinal
with only rarely the colon as the site of obstruction.

25. A careful history is necessary in all patients being considered for inguinal hernia repair. Symptoms which deserve
investigation and appropriate treatment prior to proceeding with inguinal hernia repair include:

a. Chronic cough
b. Urinary hesitancy and straining
c. Change in bowel habit
d. A specific episode of muscular straining with associated discomfort
Answer: a, b, c

The history and physical examination are almost exclusively the diagnostic modalities used for diagnosis and delineation of
hernias. Chronic trauma in the form of overstretching of musculoaponeurotic structures is likely to be the significant factor
in spontaneously occurring hernias. Failure to recognize underlying pathology contributing to symptoms of abdominal
straining may both increase the risk of recurrent hernia as well as miss significant existing pathology. A chronic cough from
chronic obstructive pulmonary disease should be investigated and attempts made to control symptoms. Significant
obstructive uropathy may warrant urologic consultation and treatment prior to hernia repair. Such treatment is important
both to prevent postoperative urinary retention, as well as persistent straining on the newly-completed repair. Change in
bowel habits with constipation or the presence of blood associated with bowel movements may suggest a rectal or left-sided
colon cancer. Patients frequently relate a specific episode of muscular straining during which a sudden discomfort occurs
followed by hernia symptoms of discomfort or a bulge. There is little evidence to suggest that such a specific acute event
can precipitate a hernia. A history of heavy lifting is important, however, in both planning of postoperative disability as well
as consideration for long-term recurrence rates.

26. The following statement(s) is/are true concerning abdominal incisional hernias.

a. Large incisional hernias are associated with a high recurrence rate when closed primarily
b. A large potential space remains anterior to the abdominal wall closure in most patients indicating a need for
postoperative wound drainage
c. The use of prosthetic mesh can often be avoided by employing relaxing incisions in the anterior fascia parallel to
the midline
d. Incisional hernias are frequently associated with a tissue deficit either due to chronic retraction and scarring or the
result of tissue necrosis from either infection or tension at the initial closure
Answer: a, b, c, d

Repair of an incisional hernia can be difficult with several factors making these hernias particularly challenging. First,
incisional hernias are often related to a postoperative wound infection, in which case associated fascitis or muscle necrosis
may result in loss of tissue. Second, a previous abdominal wall closure under tension or with a technique that resulted in
tension on particular sutures may lead to a multifenestrated region of the musculoaponeurotic abdominal wall near or
slightly back from its margin. Third, chronic retraction of the abdominal wall muscles result in a larger defect. Fourth, a
large potential space remains anterior to the abdominal wall closure in the subcutaneous area; postoperative fluid
accumulation in this space contributes to the wound infection rate of 5%. Any such potential space should have operatively
placed drains.
The key to successful repair involves sufficient dissection and exposure of the true musculoaponeurotic edge and exclusion
of adjacent musculoaponeurotic defects and avoidance of closing the wound under tension. Large defects greater than 3 to 4
cm in diameter are seldom able to be closed without excessive tension. The use of relaxing incisions decreases tension and
may be particularly useful in midline hernias and therefore may avoid the need for prosthetic mesh.

27. Which of the following structures are derived from the external oblique muscle and its aponeurosis?

a. The inguinal or Poupart’s ligament


b. The lacunar ligament
c. The superficial inguinal ring
d. The conjoined tendon
Answer: a, b, c

The external oblique muscle and its aponeurosis, with its inferiorly and medially-directed fascicles and the overlying
innominate fascia lie deep to the subcutaneous tissue. The inguinal ligament (Poupart’s ligament) is the inferior edge of the
external oblique aponeurosis and extends from the anterior superior iliac spine to the pubic tubercle, turning under itself
posteriorly and then superiorly to form a shelving edge. Medially, the inguinal ligament turns under even further to form the
lacunar ligament, as part of its insertion on the pubis. The superficial inguinal ring is a triangular opening in the external
aponeurosis, with its apex superiorly in position slightly above and lateral to the pubic tubercle, through which the cord
exits the inguinal canal. The conjoined tendon is commonly alluded to in descriptions of inguinal hernia repairs. The
conjoined tendon is the fusion of the aponeurosis of the internal oblique and transversus abdominis muscles.

28. A number of special circumstances exist in the repair of inguinal hernias. The following statement(s) is/are correct.

a. Simultaneous repair of bilateral direct inguinal hernias can be performed with no significant increased risk of
recurrence
b. The preperitoneal approach may be appropriate for repair of a multiple recurrent hernia
c. A femoral hernia repair can best be accomplished using a Bassini or Shouldice repair
d. Management of an incarcerated inguinal hernia with obstruction is best approached via laparotomy incision
Answer: b

The approach to bilateral groin hernias is based on the extent of the hernia defect. For hernias for which inguinal floor
reconstruction is required (all direct and moderate to large indirect inguinal hernias, all femoral hernias), simultaneous
repair of bilateral hernia results in recurrence of one or both of the hernias twice as frequently as if the hernias were repaired
sequentially. Repair of recurrent inguinal or much less commonly femoral hernias can be repaired via an anterior approach
particularly at the time of first recurrence in most cases. If a deficit of aponeurotic tissue exists, methods such as
polypropylene mesh as an overlay or preferably as an underlay, and tailored around the spermatic cord have proved highly
successful. The preperitoneal approach also has potential benefits especially in cases of multiple recurrence where the
technique allows avoidance of the inevitable scar encountered with the anterior approach, excellent assessment of the
defect, and the ease for placement of synthetic mesh. The Bassini and Shouldice repairs involve approximation of the
medial tissues of the transversus abdominis aponeurosis and transversalis fascia to the inguinal ligament. These techniques
cannot be used to repair a femoral hernia because the femoral canal lies deep to the inguinal ligament. Either the anterior
approach of McVay (Cooper’s ligament repair) or a preperitoneal approach is preferred for femoral hernias. In patients with
bowel obstruction attributed to a hernia, the primary operative approach is on the hernia. Assessment of bowel viability is
possible without laparotomy in most cases, and release of adhesions holding the bowel within the sac is more easily
accomplished through direct entry into the hernia sac. Reduction of the herniated and incarcerated bowel may be difficult
from the intraabdominal approach necessitating a counter incision over the external presentation of the hernia.

29. Which of the following statements concerning intraperitoneal fluid collections are correct?
A. Ascites occurs when either the peritoneal fluid secretion rate increases or the absorption rate decreases.
B. Accumulation of lymph within the peritoneal cavity usually results from trauma as tumor involving the intra-abdominal
lymphatic structures.
C. Choleperitoneum (intraperitoneal bile) generally occurs following biliary surgery, but spontaneous perforation of the bile
duct has been reported.
D. The most common cause of hemoperitoneum is trauma to the liver or spleen.
Answer: ABCD

DISCUSSION: Normally, there is a balance between fluid secretion and absorption in the peritoneal cavity. Ascites occurs
when either the secretion rate increases or the absorption rate decreases disproportionately. Accumulation of lymph in the
peritoneal cavity usually results from trauma or tumor involving lymphatic structures. Proposed treatment regimens range
from salt restriction and diuretics to surgical ligation and peritoneovenous shunting. Uninfected bile is a mild irritant to the
peritoneal cavity and causes increased production of peritoneal fluid, resulting in bile ascities or choleperitoneum. Most
cases of choleperitoneum follow biliary tract surgery, but cases of spontaneous bile duct perforation have been reported in
infants and some adults. The most common cause of hemoperitoneum is trauma to the liver or spleen. Less common causes
include ruptured ectopic pregnancy, ruptured aortic aneurysms, and other intra-abdominal injuries.

30. The following statement about peritonitis are all true except:
A. Peritonitis is defined as inflammation of the peritoneum.
B. Most surgical peritonitis is secondary to bacterial contamination.
C. Primary peritonitis has no documented source of contamination and is more common in adults than in children and in
men than in women.
D. Tuberculous peritonitis can present with or without ascites.
Answer: C

DISCUSSION: Peritonitis is inflammation of the peritoneum and can be septic or aseptic, bacterial or viral, primary or
secondary, acute or chronic. Most surgical peritonitis is secondary to bacterial contamination from the gastrointestinal tract.
Primary peritonitis refers to inflammation of the peritoneal cavity without a documented source of contamination. It is more
common in children than in adults and in women than in men. The female predominance is felt to be explained by entry of
organism into the peritoneal cavity through the fallopian tubes. The clinical manifestations of tuberculous peritonitis are of
two types. The moist form consists of fever, ascites, abdominal pain, and weakness. The dry form presents in a similar
manner but without ascites.

31. True or false?


A. Mesenteric cysts are most often due to congenital lymphatic spaces that gradually fill with lymph.
B. Mesenteric cysts usually present as abdominal masses accompanied by pain, nausea, or vomiting.
C. Mesenteric cysts are best treated by marsupialization.
D. Omental cysts are frequently asymptomatic unless they undergo torsion.
Answer: A-TRUE, B-TRUE, C-FALSE, D-TRUE

DISCUSSION: Mesenteric cysts are most often due to congenital lymphatic spaces that gradually enlarge as they fill with
lymph. They generally present as abdominal masses accompanied by pain, nausea, and vomiting. They usually can be
diagnosed by physical examination and have characteristic lateral mobility. They are best treated by surgical excision, and
intestinal resection may be necessary for complete removal. Omental cysts are frequently asymptomatic but may present
with vague discomfort or as a mobile abdominal mass that can cause torsion of the omentum. Torsion generally presents
with signs and symptoms compatible with acute cholecystitis, appendicitis, or a twisted ovarian cyst. Treatment entails local
resection.

32. Which of the following statements about acute salpingitis are true?
A. The disease rarely occurs after menopause.
B. Gonococcal infection is most common.
C. There is minimal cervical tenderness to palpation.
D. Vaginal discharge occurs rarely.
Answer: AB

33. Acute appendicitis is most commonly associated with which of the following signs?
A. Temperature above 104‫ ؛‬F.
B. Frequent loose stools.
C. Anorexia, abdominal pain, and right lower quadrant tenderness.
D. White blood cell count greater than 20,000 per cu. mm.
Answer: C

34. Which of the following most often initiates the development of acute appendicitis?
A. A viral infection.
B. Acute gastroenteritis.
C. Obstruction of the appendiceal lumen.
D. A primary clostridial infection.
Answer: C

DISCUSSION: The majority of patients with acute appendicitis have an obstructed lumen that is due to either hyperplasia of
the lymph follicles in the wall of the appendix or a fecalith. The obstruction creates a site where the bacteria in the lumen
multiply rapidly, producing exotoxins and endotoxins that then ulcerate the mucosa, allowing pathogenic organisms to enter
the wall of the appendix. An inflammatory process follows that can extend to the serosa, and penetration through the serosal
layer causes generalized peritonitis.

35. The diagnosis of acute appendicitis is most difficult to establish in:


A. Persons aged 60 and older.
B. Women aged 18 to 35.
C. Infants younger than 1 year.
D. Pregnant women.
Answer: C

DISCUSSION: It is very difficult to establish a firm diagnosis of acute appendicitis in an infant of 1 year or younger since
the patient cannot provide a history or be helpful during the physical examination. It is rare to make a definitive diagnosis
preoperatively in such infants, and in such cases the appendix is usually perforated at the time of operation. While
appendicitis is somewhat more difficult to diagnose in the elderly because of the reduced response to inflammation;
nevertheless, it is usually possible to make the diagnosis. With pregnant women it is wise to remember that the enlarging
uterus in the last trimester dislocates the appendix higher in the abdomen and that the signs and symptoms follow this
anatomic shift accordingly.
36. Once a diagnosis of acute appendicitis has been made and appendectomy decided upon, which of the following is/are
true?
A. Prophylactic antibiotics should be administered.
B. Prophylactic antibitics are not necessary unless there is evidence of perforation.
C. If the appendix is not ruptured and not gangrenous, antibiotics may be discontinued after 24 hours.
D. Multiple antibiotics are in all cases preferable to a single agent.
Answer: AC

DISCUSSION: It is generally held that patients with a diagnosis of acute appendicitis should receive antibiotics such as
cefoxitin or cefotetan. Administration can be discontinued after 24 hours if the appendix is not gangrenous or ruptured.
Multiple antibiotics are unnecessary in straightforward cases.

37. The best type of x-ray to locate free abdominal air is:
A. A posteroanterior view of the chest.
B. A flat and upright view of the abdomen.
C. Computed tomograph (CT) of the abdomen.
D. A lateral decubitus x-ray, right side up.
Answer: D

38. The most helpful diagnostic radiographic procedure in small bowel obstruction is:
A. CT of the abdomen.
B. Contrast study of the intestine.
C. Supine and erect x-rays of the abdomen.
D. Ultrasonography of the abdomen.
Answer: C

39. The most commonly used imaging method for diagnosis of acute cholecystitis is:
A. CT of the abdomen.
B. Ultrasonography of the gallbladder.
C. Oral cholecystogram.
D. Radionuclide (HIDA) scan of the gallbladder.
Answer: B

40. Acute salpingitis occurs most often:


A. After menopause.
B. In patients with unilateral lower abdominal pain.
C. During the menstrual cycle.
D. In patients with cervical tenderness and vaginal discharge.
Answer: D

41. Meckel's diverticulitis most often occurs in the:


A. Proximal jejunum.
B. Distal jejunum.
C. Proximal ileum.
D. Distal ileum.
Answer:D

42. A patient is seen in the emergency room with reproducible right lower quadrant tenderness. The approximate incidence
of finding a normal appendix on right lower quadrant exploration in similar nonselected patients is which of the
following:

a. 5%
b. 10%
c. 20%
d. 40%
Answer: c
Appendectomy is the most common surgical procedure performed on an emergency basis in Western medicine. Appendicitis
has a negative appendectomy rate of approximately 22% to 26% in broad based reviews. The perforation rate is as low as
3.6% in a subset of young males, although this rises substantially when the children or the elderly are included. Likewise,
young females represent a group at particularly high risk for other intraabdominal pathology.

43. Of adult patients presenting to the emergency room for evaluation of acute abdominal pain, which one of the following
answers includes the most common diagnoses?

a. Urologic problems, cholelithiasis, pelvic inflammatory disease


b. Mittelschmerz, appendicitis, ureterolithiasis
c. Nonspecific abdominal pain, appendicitis, intestinal obstruction
d. Appendicitis, pelvic inflammatory disease, perforated ulcer
Answer: c

Numerous surgical causes exist for the patient presenting with acute abdominal pain. A recent review of nearly 1200
patients presenting for emergency evaluation of abdominal pain affords some interesting findings. The most common
diagnosis was nonspecific abdominal pain, occurring in 35% of patients. Appendicitis (17%), intestinal obstruction (15%),
urologic problems (6%), and gallstones (5%) were the leading surgical causes. The largest number of admissions occurred
in the age groups 10–29 years old (31%) and 60–79 years old (29%). Surgical procedures were required in 47% of these
patients. Large series of elderly patients presenting with acute abdominal pain have found the leading diagnoses to be
cholelithiasis, nonspecific pain, malignancy, incarcerated hernia, ileus, and gastroduodenal ulcer.

44. Nonsurgical causes of acute abdominal pain may include which of the following?

a. Hyperthyrodism
b. Adrenal insufficiency
c. Pneumonia
d. Diabetic ketoacidosis
Answer: b, c, d
Many nonsurgical problems cause acute abdominal pain. A partial listing is provided above. Of the choices in question, the
only one that is not associated with acute abdominal pain is hyperthyroidism. The remainder cause abdominal pain through
a variety of mechanisms, both direct and indirect.
NONSURGICAL CAUSES OF THE ACUTE ABDOMEN

METABOLIC
Diabetic ketoacidosis
Porphyria
Adrenal insufficiency
Uremia
Hypercalcemia

TOXIC
Insect bites
Venoms (scorpion, snake)
Lead poisoning
Drugs

MISCELLANEOUS
Hemolytic crises
Rectus sheath hematoma

NEUROGENIC
Herpes zoster
Abdominal epilepsy
Spinal cord tumor, infection
Nerve root compression
CARDIOPULMONARY
Pneumonia
Myocardial infarction
Myocarditis
Empyema
Costochondritis

45. Which of the following cause visceral pain from the abdominal organs?

a. Stretching and contraction


b. Traction, compression, torsion
c. Cutting
d. Certain chemicals
Answer: a, b, d

Abdominal pain can be divided into three categories; visceral, somatic, and referred. The intramural sensory receptors of the
abdominal organs are responsible for visceral pain. A diverse group of destructive stimuli to the abdominal viscera are
painless. For example, almost all abdominal organs are insensitive to pinching, burning, stabbing, cutting, and electrical and
thermal stimulation. The same is true for the application of acid and alkali to normal mucosa.
The general classes of visceral stimulation that result in abdominal pain include: (1) stretching and contraction; (2) traction,
compression, and torsion; (3) stretch alone; and (4) certain chemicals. Mediating receptors for these responses are located
intramurally in hollow organs, on serosal structures such as the visceral peritoneum and capsule of solid organs, within the
mesentery and the mucosa. These receptors are polymodal, or responsive to both mechanical and chemical stimuli. Mucosal
receptors respond primarily to chemical stimulation. Visceral pain almost always heralds intra-abdominal disease but may
not indicate the need for surgical therapy. When visceral pain is superceded by somatic pain, the need for surgical
intervation becomes likely.

46. Factors which may influence the clinical presentation of intraabdominal pathology include which of the following?

a. Pregnancy
b. Oral anticoagulants
c. Age
d. HIV infection
Answer: a, b, c, d

A variety of conditions influence the presentation of intraabdominal pathology. Pregnancy is among these, principally
because of displacement of adjacent normal viscera and therefore a shift in the location of the parietal pain. Oral
anticoagulation is associated with the development of spontaneous intramural hematomas of the bowel causing pain but not
requiring surgical resection. This pain may be confused with a variety of other intraabdominal emergencies.
Age is likewise a confounding factor, generally in infancy and in the elderly. In these age groups, the symptoms may be less
pronounced and the presentations occur later in the course of disease.
Immunocompromised patients are a heterogenous group that includes those receiving allografts, chemotherapy,
immunosuppressive drugs for autoimmune disorders, and individuals with the acquired immunodeficiency syndrome
(AIDS). This group has a variety of specific abdominal complications that must be appreciated and suspected by the
evaluating physician.
ACUTE ABDOMINAL PAIN ASSOCIATIONS IN THE
IMMUNOCOMPROMISED PATIENT

CYTOMEGALOVIRUS INFECTION
Interstitial pneumonitis
Mononucleosis
Pancreatitis
Hepatitis
Cholecystitis
Gastrointestinal ulceration

PANCREATITIS
Steroids
Azathioprine
Cytomegalovirus
Pentamidine

HEPATITIS
Hepatitis A, B, and C
Cytomegalovirus
Epstein-Barr virus

CHOLECYSTITIS
Cytomegalovirus
Acalculous cholecystitis
Campylobacter

HEPATOSPLENIC ABSCESS
Fungal
Mycobacterial
Protozoal
Splenic rupture

BOWEL PERFORATION
Lymphoma, leukemia (especially after chemotherapy)
Cytomegalovirus
Colon ulcers
Kaposi sarcoma
Pseudomembranous colitis
Mycobacteria
latrogenic

ACUTE GRAFT-VERSUS-HOST DISEASE

PSEUDOACUTE ABDOMEN

FECAL IMPACTION

STANDARD ABDOMINAL PROCESSES


Appendicitis
Cholecystitis
Diverticulitis
Bowel obstruction
Ulcer disease
Pelvic inflammatory disease
Perirectal abscess
Urinary tract infection
Lymphadenitis

NEUTROPENIC ENTEROCOLITIS

47. Prospective studies have shown incidental appendectomy to be advantageous in which of the following patient groups?

a. Children undergoing staging laparotomy for malignancy who are then to enter chemotherapy
b. HIV infected patients
c. Patients over 50 years of age
d. Patients with spinal cord injuries
e. None of the above
Answer: e
Several studies have looked at incidental appendectomies in a variety of populations. The deficiency in all past studies of
this issue is the lack of prospective long-term trials to assess the true cost and benefit.
Incidental appendectomy is clearly not indicated in the elderly and in patients undergoing laparatomy for staging of
Hodgkin’s disease. These two specific groups have been shown to have increased perioperative risks with incidental
appendectomy. No prospective studies have addressed the issue of HIV infected or spinal cord injured patients. While
incidental appendectomies may be performed safely in general, it is difficult to justify any increase in operative risk without
demonstrable benefit.

48. Visceral pain is typically:

a. Well localized
b. Sharp
c. Mediated via spinal nerves
d. Perceived to be in the midline
Answer: d

Peritoneum is a continuous visceral and parietal layer. The nerve supply to each layer is separate. The visceral layer, i.e., the
layer surrounding all intraabdominal organs, is supplied by autonomic nerves (sympathetic and parasympathetic) and the
parietal peritoneum is supplied by somatic innervation (spinal nerves). The pathways relaying the sensation of pain differ
for each layer and differ in quality as well. Visceral pain is characteristically dull, crampy, deep, aching and may involve
sweating and nausea. Parietal pain is sharp, severe and persistent. Visceral organs have very little pain sensation, but
stretching of the mesentery and stimulation of the parietal peritoneum cause severe pain.
Normal embryologic development of the abdominal viscera proceeds with bilateral midline autonomic innervation that
results in visceral pain usually being perceived as arising from the midline. Epigastric pain is typical of foregut origin.
Periumbilical pain signifies pain emanating from the midgut. Hypogastric or lower abdominal midline pain indicates a
hindgut origin.

49. True statements regarding the pathophysiology of acute appendicitis include which of the following:

a. Fecaliths are responsible for the disease process in approximately 30% of adult patients
b. Lymphoid hyperplasia is a rare cause of appendicitis in young patients
c. Clostridium difficile is implicated as a pathogenic organism
d. Carcinoid tumors account for approximately 5% of all cases of acute appendicitis
Answer: a

The most common cause of appendicitis is obstruction of the appendiceal lumen. In young children and young adults, the
most common cause of lumenal obstruction is lymphoid hyperplasia from the submucosal follicles which are abundant.
Lymphoid hyperplasia accounts for 60% of acute appendicitis in the young. In adults, fecalith formation accounts for
approximately 30% of acute appendicitis. There is no known causative relationship of Clostridium difficile or other specific
organisms with acute appendicitis. The normal flora of the appendix is consistent with that of the adjacent cecum.
Neoplasms of the appendix are rare, occurring in 1% to 1.3% of all appendectomy specimens. Carcinoid tumors are the
most common, followed in frequency by benign and malignant mucoceles.

50. A 26-year old woman in her first trimester of pregnancy presents with a 2-day history of right lower quadrant pain and
fever. Physical examination reveals a tender, palpable, right lower quadrant mass. There is no evidence of peritonitis or
systemic sepsis. Laboratory evaluation is remarkable for mild leukocytosis, and abdominal ultrasound demonstrates an
inflammatory mass but no evidence of abscess. As the surgeon on call, your recommendation would be:
a.
b. Intravenous hydration, antibiotic prophylasis, and urgent appendectomy
c. Intravenous hydration, antibiotics, bowel rest, and interval appendectomy in 4 to 6 weeks
d. Intravenous hydration, antibiotics, and appendectomy if no improvement in 12 to 24 hours
e. Intravenous hydration, antibiotics, and interval appendectomy when fever has subsided, leukocyte count has
returned to normal, and the patient is pain free
f. Emergent obstetrical consultation for evaluation and treatment of possible ectopic pregnancy
Answer: a
The patient presented has a perforated appendix with a phlegmon, but no abscess. One must routinely provide resuscitation
and broad spectrum antiobiotic coverage in this circumstance. As she is not systemically toxic, it would be rational in a
nonpregnant patient to treat this patient nonoperatively initially and follow this with interval appendectomy. However, in
this circumstance, the risk of preterm labor associated with anesthesia and pelvic inflammation increases with more
advanced gestation, so the best decision is to proceed with intravenous hydration, broad spectrum antibiotic coverage and
urgent appendectomy.

51. True statements regarding appendiceal neoplasms include which of the following?

a. Carcinoid tumors of the appendix less than 1.5 cm are adequately treated by simple appendectomy
b. Appendiceal carcinoma is associated with secondary tumors of the GI tract in up to 60% of patients
c. Survival following right colectomy for a Dukes’ stage C appendiceal carcinoma is markedly better than that for a
similarly staged colon cancer at 5 years
d. Mucinous cystadenocarcinoma of the appendix is adequately treated by simple appendectomy, even in patients
with rupture and mucinous ascites
e. Up to 50% of patients with appendiceal carcinoma have metastatic disease, with the liver as the most common site
of spread
Answer: a

Carcinoids represent two-thirds of all appendiceal neoplasms. Nearly half of all GI carcinoids arise in the appendix at a
mean age of 41 years. Two-thirds of the time the carcinoid is only incidentally detected, only 0.5% have evidence of distant
metastatic spread at resection. In one experience, carcinoids between 1.5 and 2.0 cm have had minimal metastatic potential
and those smaller than 1.5 cm never metastasized. In the 1% that are larger than 2 cm however, metastases are frequent and
80% recur even after resection at this size.
Adenocarcinoma of the appendix is exceedingly rare. These tumors occur in elderly patients at the base of the appendix.
Appendicitis often follows and the diagnosis is not made preoperatively and is rarely considered during surgery since the
appearance of the tumor may mimic perforated appendicitis. Up to half the patients have metastatic disease at diagnosis and
the peritoneum is the most common site of spread. Survival is proportional to tumor stage. Dukes’ Stage A disease may be
treated simply with appendectomy if all disease can be removed with reasonable margins. Dukes’ B and C lesions require
formal right hemicolectomy for disease control. Survival is, stage for stage, similar to colon cancer after 5 years.
Appendiceal adenocarcinomas also appear to have an association with secondary tumors, often of the GI tract, in up to 35%
of patients.
Patients with mucinous cystadenocarcinoma of the appendix typically are symptomatic, and wide resection of the primary
disease, together with debulking of peritoneal implants, is indicated. Indolent progression of metastases commonly results in
prolonged survival rates (50% at 5 years) during which patients may require repeated laparatomies for complications of the
disease.

1. Which of the following statements about the anatomic course of the esophagus is correct?
A. The cervical esophagus passes behind and to the right of the trachea.
B. The thoracic esophagus enters the posterior mediastinum anterior to the aortic arch.
C. The thoracic esophagus passes behind the right mainstem bronchus and the pericardium.
D. The esophagus enters the diaphragmatic hiatus at the level of T8.
E. The esophagus deviates anteriorly and to the left as it enters the abdomen.
Answer: E

DISCUSSION: Knowledge of the normal course of the esophagus is important in operative and endoscopic procedures. The
cervical esophagus lies just anterior to the prevertebral fascia and courses through the neck posterior to the trachea and to
the left of the midline. The cervical esophagus is therefore more readily approached surgically through a left neck incision.
The thoracic esophagus enters the posterior mediastinum posterior to the aortic arch. Therefore, when operating on the
upper esophagus, a right thoracotomy is preferred, since the aortic arch overlies the esophagus in the left chest. The
esophagus descends in the posterior mediastinum behind the left mainstem bronchus and pericardium. It is the proximity of
the esophagus and left mainstem bronchus that is responsible for the development of malignant tracheoesophageal fistulas
between these two structures. The esophagus enters the diaphragmatic hiatus, which is located at the level of T11. As it
enters the abdomen, the esophagus deviates anteriorly and to the left. Therefore, when performing rigid esophagoscopy as
the distal esophagus is entered, the patient's head must be turned to the right and the esophagoscope elevated anteriorly to
avoid perforation.

2. Which of the following statements about esophageal anatomy is correct?


A. The esophagus has a poor blood supply, which is segmental in distribution and accounts for the high incidence of
anastomotic leakage.
B. The esophageal serosa consists of a thin layer of fibroareolar tissue.
C. The esophagus has two distinct muscle layers, an outer, longitudinal one and an inner, circular one, which are striated in
the upper third and smooth in the distal two thirds.
D. Injury to the recurrent laryngeal nerve results in vocal cord dysfunction but does not affect swallowing.
E. The lymphatic drainage of the esophagus is relatively sparse, localized primarily to adjacent paraesophageal lymph
nodes.
Answer: C

DISCUSSION: Poor technique, not poor blood supply, explains most esophageal anastomotic leaks. While the major blood
supply of the esophagus is from four to six segmental aortic esophageal arteries, there are extensive submucosal collaterals
from the inferior thyroid, intercostal, bronchial, inferior phrenic, and left gastric arteries. The esophagus lacks serosa and
instead is surrounded by mediastinal connective tissue (adventitia). There are two muscle layers in the esophagus, an outer
longitudinal and an inner circular one. Both layers of the upper third of the esophagus consist of striated muscle, while in
the lower two thirds they are (nonstriated) smooth muscle. The recurrent laryngeal branches of the vagus nerves provide
both parasympathetic innervation to the cervical esophagus and innervation to the upper esophageal sphincter (UES). Injury
to the recurrent laryngeal nerve therefore results in improved UES function with secondary aspiration on swallowing as well
as vocal cord dysfunction and hoarseness. The esophagus has extensive lymphatic drainage, with lymphatic capillaries
coursing longitudinally in the esophageal wall and communicating with paraesophageal, paratracheal and subcarinal, other
mediastinal, perigastric, and internal jugular lymph nodes. This accounts for the biologically aggressive nature of
esophageal carcinoma, which tends to metastasize early in its course.

3. Which of the following statements about the lower esophageal sphincter (LES) mechanism, or high-pressure zone
(HPZ), is true?
A. The LES is a circular smooth muscle ring that is 3 to 5 cm. long.
B. In assessing esophageal manometric data, mean HPZ pressure less than 6 mm. Hg or overall length less than 2 cm. is
more likely to be associated with incompetence of the LES and gastroesophageal reflux.
C. Esophageal manometry and the acid perfusion (Bernstein) test reliably identify the patient with an incompetent LES
mechanism.
D. Distal HPZ relaxation occurs within 5 to 8 seconds of initiating a swallow.
E. Twenty-four–hour distal esophageal pH monitoring is achieved with an intraesophageal pH electrode positioned at the
esophagogastric junction.
Answer: B

DISCUSSION: The LES is not an anatomic muscular sphincter like the anus or pylorus, but rather, is a functional sphincter
that serves as a barrier against abnormal regurgitation of gastric contents into the esophagus and is more appropriately
termed the distal esophageal HPZ. While, in general, no HPZ value absolutely indicates either competence or incompetence
of the LES mechanism, patients with a mean HPZ pressure less than 6 mm. Hg or a sphincter length less than 2 cm. are
likely to have an incompetent LES and gastroesophageal reflux. Esophageal manometry defines the amplitude and length of
the distal HPZ and the character of esophageal peristalsis. It does not determine whether or not the LES is competent. The
Bernstein test identifies the patient with an acid-sensitive esophagus but does not indicate whether or not the patient has
gastroesophageal reflux or esophagitis. Reflex distal HPZ relaxation occurs within 1.5 to 2.5 seconds after a swallow is
initiated and lasts 4 to 6 seconds. Twenty-four–hour distal esophageal pH monitoring is achieved by means of an
intraesophageal pH electrode positioned 5 cm. proximal to the HPZ, as determined by prior manometric evaluation.

4. Which of the following statements about esophageal motility is/are true?


A. The act of swallowing initiates UES relaxation, which persists until the bolus of food passes the LES.
B. The primary peristaltic wave normally propels the swallowed bolus through the esophagus in 4 to 8 seconds.
C. Normally, a progressive peristaltic contraction (primary wave) follows 50% of all swallows, the remainder being
secondary or tertiary contractions.
D. Secondary peristalsis is initiated when the entire swallowed bolus of food fails to empty from the esophagus into the
stomach.
E. Tertiary esophageal contractions are high-amplitude progressive peristaltic contractions that produce the “corkscrew”
appearance of esophageal spasm on barium esophagography.
Answer: BD

DISCUSSION: Swallowing initiates UES relaxation, which lasts only 0.5 to 1.0 second, considerably less than the 4 to 8
seconds required for a swallowed bolus to pass through the esophagus and into the stomach. A progressive peristaltic
contraction normally follows 97% of all swallows. If the swallowed bolus does not empty completely into the stomach,
distention of the esophagus initiates secondary peristalsis, which is progressive and sequential and persists until the retained
intraesophageal contents have passed into the stomach. Tertiary contractions are simultaneous, nonprogressive,
incoordinated contractions of the esophageal smooth muscle that produce the contraction rings responsible for the
corkscrew esophagus of diffuse esophageal spasm on barium esophagography.

5. Which of the following statements about UES dysfunction are correct?


A. This condition is diagnosed by the characteristic manometric findings of UES spasm.
B. Typical symptoms include cervical dysphagia, expectoration of saliva, and hoarseness.
C. The classic finding on barium esophagogram is a posterior cricopharyngeal bar.
D. Medical or surgical therapy of gastroesophageal reflux may be curative.
E. A cervical esophagomyotomy for UES dysfunction should be limited to 2 to 3 cm. in length so that normal muscle is not
damaged.
Answer: BCD

DISCUSSION: The unique anatomic characteristics of the UES and the limitations of existing equipment in recording the
rapid sequence of events associated with swallowing make standard manometric definition of UES motor abnormalities
extremely difficult. Characteristic consistent abnormalities of UES function in patients with cricopharyngeal dysfunction
have not been well documented. UES dysfunction results in cervical dysphagia, expectoration of saliva that is no longer
swallowed freely, and, often, intermittent hoarseness due to alteration of the larynx and vocal cords by the pull of the
abnormal cricopharyngeal sphincter. Barium esophagography in the patient with UES dysfunction frequently shows a
posterior cricopharyngeal “bar,” representing the prominent impression of the sphincter on the esophageal lumen. Patients
with gastroesophageal reflux may present with cervical dysphagia due to secondary UES dysfunction. Successful medical or
surgical treatment of the reflux may eliminate the cervical complaints. Since the UES is normally 3 to 5 cm. long, when a
cervical esophagomyotomy is required for treatment of cricopharyngeal dysfunction, a generous myotomy, 7 to 10 cm. long,
is carried out to ensure complete division of all incoordinated UES muscle fibers.

6. Which of the following statements about achalasia is/are correct?


A. In most cases in North America the cause is a parasitic infestation by Trypanosoma cruzi.
B. Chest pain and regurgitation are the usual symptoms.
C. Distal-third esophageal adenocarcinomas may occur in as many as 20% of patients within 10 years of diagnosis.
D. Manometry demonstrates failure of LES relaxation on swallowing and absent or weak simultaneous contractions in the
esophageal body after swallowing.
E. Endoscopic botulinum toxin injection of the LES, pneumatic dilatation, and esophagomyotomy provide highly effective
curative therapy for achalasia.
Answer: D

DISCUSSION: While in South America achalasia is the result of Chagas' disease caused by parasitic infestation by the
leishmanial forms of T. cruzi, in Europe and North America the cause of achalasia is unknown. The common presenting
symptoms of achalasia are dysphagia, regurgitation, and weight loss. Chest pain is an infrequent symptom in achalasia and
is more characteristic of esophageal spasm. Achalasia is a premalignant esophageal lesion: the retention esophagitis leads to
metaplasia and squamous cell carcinoma, which occurs after 15 to 25 years in the middle third of the thoracic esophagus in
10% of patients. The classic manometric findings of achalasia are failure of relaxation of the LES on swallowing and absent
or weak simultaneous contractions in the esophageal body after swallowing. Achalasia is currently incurable, and, though
the recently described endoscopic botulinum toxin injection of the lower esophageal sphincter, pneumatic dilatation, and
esophagomyotomy effectively relieve dysphagia in the majority of patients, all of these treatments are strictly palliative. The
motility disturbance persists throughout life.
7. Which of the following statements about diffuse esophageal spasm is/are true?
A. Chest pain due to esophageal spasm is readily differentiated from angina pectoris of cardiac origin.
B. Bouts of esophageal obstruction and regurgitation of food are characteristic.
C. Associated psychiatric disorders are common.
D. During manometric assessment, unless the patient is having pain there may be no detectable multiphasic, high-
amplitude, simultaneous esophageal contractions.
E. The treatment of choice is a long esophagomyotomy from the aortic arch to the esophagogastric junction.
Answer: CD

DISCUSSION: The chest pain of diffuse esophageal spasm is often indistinguishable from that of angina pectoris of cardiac
origin. Many patients undergo cardiac catheterization to rule out coronary artery disease. Patients may experience slow
emptying of the esophagus, but obstructive symptoms and regurgitation of food are unusual. Psychiatric disorders
(depression, psychosomatic complaints, anxiety) have been documented in more than 80% of patients with esophageal
contraction abnormalities. The classic manometric criteria of diffuse esophageal spasm are simultaneous, multiphasic,
repetitive, high-amplitude contractions occurring after a swallow and spontaneously. These changes may not be detected if
manometry is performed when the patient is having no pain. As the cause of esophageal spasm is unknown, treatment is far
from ideal. Conservative management—avoidance of “trigger” foods and drinks, psychiatric counseling, treatment of reflux,
esophageal dilatations, use of histamine H 2 blockers, anti-spasmodics, and smooth muscle relaxants—should always be
attempted first. Esophagomyotomy provides no lasting relief of esophageal spasm for as many as 50% of patients and
should be used only in a minority of these patients.

8. Which of the following statements about epiphrenic diverticula of the esophagus is/are correct?
A. They are traction diverticula that arise close to the tracheobronchial tree.
B. They characteristically arise proximal to an esophageal reflux stricture.
C. The degree of dysphagia correlates with the size of the pouch.
D. They are best approached surgically through a right thoracotomy.
E. The operation of choice is a stapled diverticulectomy, long esophagomyotomy, and partial fundoplication.
Answer: E

DISCUSSION: Epiphrenic diverticula are pulsion diverticula that arise in the distal 10 cm. of the esophagus. The cause is
elevated intraesophageal pressure, which forces mucosa and submucosa to herniate through the muscle layers. Though this
may occur with an esophageal reflux stricture or tumor, primary disordered esophageal motility is the most common cause.
Many patients are asymptomatic when their epiphrenic diverticula are diagnosed by barium esophagography, symptoms
being related more to the degree of disordered motility than to the size of the pouch. Distal esophageal diverticulum is best
approached through a left thoracotomy. When surgery is indicated, the preferred approach is transthoracic diverticulectomy
using a surgical stapler, a long esophagomyotomy to relieve the elevated intraesophageal pressure, and a nonobstructing
partial (e.g., Belsey's) fundoplication.

9. Which of the following statements about Schatzki's ring is correct?


A. The ring represents a panmural fibrotic stricture resulting from gastroesophageal reflux.
B. Dysphagia occurs when the ring diameter is 13 mm. or less.
C. The ring occurs within 1 to 2 cm. of the squamocolumnar epithelial junction.
D. Schatzki's ring indicates reflux esophagitis.
E. Schatzki's ring signifies the need for an antireflux operation.
Answer: B

DISCUSSION: Schatzki's ring is seen radiographically as an annular weblike constriction at the esophagogastric junction in
a patient with a sliding hiatalhernia. It represents prominence of the esophagogastric junction with slight submucosal
fibrosis but not true panmural fibrotic reflux stricture. Intermittent dysphagia may occur when the ring size is 20 mm. or
less, but dysphagia is almost invariable when the ring measures 13 mm. or less. Schatzki's ring occurs precisely at the
squamocolumnar epithelial junction. It is indicative of the presence of a hiatal hernia but not of gastroesophageal reflux or
esophagitis. An asymptomatic Schatzki's ring requires no therapy per se. Patients with refractory severe reflux symptoms
after dilation therapy are candidates for antireflux surgery.

10. Choose the distance in centimeters from the upper incisor teeth at which the following radiographically identified
esophageal lesions would be encountered endoscopically:
A. 10 cm. 1. Zenker's diverticulum
B. 15 cm. 2. Traction diverticulum
C. 25 cm. 3. Tumor 10 cm. proximal to the esophagogastric junction
D. 30 cm.
E. 40 cm.
Answer: 1-B, 2-C, 3-D

DISCUSSION: A barium esophagogram should be obtained routinely before performing elective esophagoscopy. The
location of an esophageal lesion seen on the barium swallow study can be related to adjacent anatomic landmarks. This
allows the endoscopist to anticipate the level (as measured from the upper incisor teeth) where he should expect to see the
abnormality at esophagoscopy. For example, the upper esophageal (cricopharyngeal) sphincter is generally seen at the level
of the C7–T1 vertebrae radiographically and at 15 cm. endoscopically. This is the level at which the mouth of a Zenker's
diverticulum is seen. The tracheal bifurcation occurs at the level of the T4 or 25 cm. from the upper incisors endoscopically;
a traction (parabronchial) diverticulum at the level of the carina on a barium esophagogram will be seen at approximately 25
cm. The esophagogastric junction occurs at approximately the level of T11, 40 cm. from the upper incisors; a tumor 10 cm.
proximal to the esophagogastric junction stricture is seen endoscopically at 30 cm.

11. Which of the following statements about pathology encountered at esophagoscopy is/are correct?
A. Reflux esophagitis should be graded as mild, moderate, or severe, to promote consistency among different observers.
B. An esophageal reflux stricture with a 2-mm. lumen is not dilatable and is best treated with resection.
C. A newly diagnosed radiographic distal esophageal stricture warrants dilation and antireflux medical therapy.
D. In patients with Barrett's mucosa, the squamocolumnar epithelial junction occurs 3 cm. or more proximal to the
anatomic esophagogastric junction.
E. After fasting at least 12 hours, a patient with megaesophagus of achalasia can safely undergo flexible fiberoptic
esophagoscopy.
Answer: D

DISCUSSION: The traditional subjective grading of reflux esophagitis as mild, moderate, or severe has inherent wide
variations in meaning among observers. Consistent use of standardized grading systems for endoscopic reflux esophagitis
(e.g., that of Belsey or Savary) provides a more objective description of the changes seen and allows more meaningful
evaluation of patients at different times and by different observers. The size of the lumen does not predict whether or not a
reflux stricture is dilatable. Even a tight 2-mm. lumen can be traversed with a guidewire over which Savary dilators can be
used to achieve an acceptable lumen size. Every newly diagnosed esophageal stricture warrants esophagoscopy with
brushings and biopsies of the stricture (to exclude carcinoma) and an assessment of its “dilatability.” Antireflux medical
therapy is not justified until carcinoma has been ruled out. Because the squamocolumnar epithelial junction may normally
be found within 2 to 3 cm. of the anatomic esophagogastric junction, the diagnosis of Barrett's mucosa requires
identification of the columnar epithelium at least 3 mm. proximal to the junction of the tubular esophagus and the stomach.
In advanced achalasia with megaesophagus, the dilated esophagus may have a capacity of 1 to 2 liters, and simply fasting
overnight does not ensure that the esophagus is empty of food and drink consumed the day before. Life-threatening massive
regurgitation and aspiration may occur as the endoscope is being introduced unless an effort is made to evacuate the
esophagus first by means of a nasogastric tube.

12. Which of the following statements about the diagnosis and treatment of esophageal leiomyomas is/are correct?
A. The majority are diagnosed after they cause dysphagia and chest pain.
B. Biopsy is indicated at the time of esophagoscopy, to rule out carcinoma.
C. Full-thickness elliptical excision of the esophageal wall is the preferred surgical approach.
D. Endoscopic ultrasonography is a reliable means of following leiomyomas conservatively.
E. Recurrence of resected leiomyomas is minimized by wide local excision.
Answer: D

DISCUSSION: Most esophageal leiomyomas are asymptomatic when discovered incidentally on a barium esophagogram or
upper gastrointestinal tract series. When suspected on the basis of its radiographic appearance, biopsy of the mass should
not be performed at the time of esophagoscopy, so that subsequent extramucosal resection will not be complicated by
scarring at the biopsy site. The preferred surgical approach is submucosal enucleation of the mass, not full-thickness
excision. Leiomyomas have a characteristic hypoechogenic homogeneous appearance on esophageal ultrasonography that
allows a noninvasive diagnosis and means of surveillance. Submucosal enucleation of leiomyomas, without wide local
excision, provides excellent long-term results with virtually no local recurrence rate.
13. Which of the following statements regarding the pathology of esophageal carcinoma is/are correct?
A. Worldwide, adenocarcinoma is the most common esophageal malignancy.
B. Squamous cell carcinoma is most common in the distal esophagus, whereas adenocarcinoma predominates in the middle
third.
C. Patients with Barrett's metaplasia are 40 times more likely than the general population to develop adenocarcinoma.
D. Metastases from esophageal carcinoma are characteristically localized to regional mediastinal lymph nodes adjacent to
the tumor.
E. Achalasia, radiation esophagitis, caustic esophageal stricture, Barrett's mucosa, and Plummer-Vinson syndrome are all
premalignant esophageal lesions that predispose to the development of squamous cell carcinoma.
Answer: C

DISCUSSION: Histologically, 95% of esophageal cancers worldwide are squamous cell carcinomas, but the incidence of
adenocarcinoma is increasing dramatically in the United States and Europe. Squamous cell carcinoma predominates in the
upper and middle thirds of the esophagus, whereas adenocarcinoma is the most frequent distal esophageal cancer. A
columnar lined lower esophagus (Barrett's metaplasia) is associated with an incidence of adenocarcinoma approximately 40
times greater than that of the general population. Esophageal cancer is a biologically aggressive tumor that characteristically
metastasizes widely to regional and distant lymph nodes as well as to liver and lungs. Recognized premalignant esophageal
lesions include achalasia, radiation esophagitis, caustic stricture, Plummer-Vinson syndrome, leukoplakia, esophageal
diverticula, and Barrett's metaplasia. All but Barrett's metaplasia are associated with the development of squamous cell
carcinoma.

14. Which of the following statements about the surgical treatment of esophageal carcinoma is/are correct?
A. The finding of severe dysphagia in association with Barrett's mucosa is an indication for an antireflux operation to
prevent subsequent development of carcinoma.
B. Long-term survival is improved by radical en bloc resection of the esophagus with its contained tumor, adjacent
mediastinal tissues, and regional lymph nodes.
C. The morbidity and mortality rates for cervical esophagogastric anastomotic leak are substantially less than those
associated with intrathoracic esophagogastric anastomotic leak.
D. The leading complications of transthoracic esophagectomy and intrathoracic esophagogastric anastomosis are bleeding
and wound infection.
E. Transhiatal esophagectomy without thoracotomy achieves better long-term survival than transthoracic esophagectomy.
Answer: C

DISCUSSION: Severe dysplasia in Barrett's mucosa is indicative of carcinoma in situ and is an indication for resectional
therapy, not an antireflux operation. In the majority of patients, local tumor invasion or distant metastases preclude cure
when esophageal carcinoma is diagnosed, and attempts to improve survival with a more radical local operation performed in
the face of systemic disease have been disappointingly futile. A cervical esophagogastric anastomotic leak causes a
relatively minor cervical salivary fistula that heals in 7 to 10 days in 95% of patients. In contrast, an intrathoracic
esophagogastric anastomotic leak results in mediastinitis, which is fatal in 50%. The leading complications of transthoracic
esophagectomy and an intrathoracic esophagogastric anastomosis are respiratory insufficiency (from combined thoracic and
abdominal incisions) and anastomotic leak resulting in mediastinitis and sepsis. Both complications are minimized by
transhiatal esophagectomy without thoracotomy plus cervical esophagogastric anastomosis. No single operative approach to
the treatment of esophageal cancer has proved superior to others in terms of long-term survival. The biologic behavior of the
tumor (its stage and aggressiveness)—not the number of lymph nodes resected with the tumor—determines survival.

15. The best management for a 48-hour-old distal esophageal perforation is:
A. Antibiotics and drainage.
B. Division of the esophagus and exclusion of the perforation.
C. Primary repair with buttressing.
D. Resection with cervical esophagostomy, gastrostomy, and jejunostomy.
E. T-tube fistula and drainage.
Answer: C

DISCUSSION: When the esophagus is repaired primarily and covered by well-vascularized autologous tissue, the rates of
fistula and death are significantly less than those observed for patients who receive simple repair without any protection.
Primary repair with buttressing is the first choice for treatment. Resection is reserved for esophageal perforations with
extensive damage to the esophageal wall or with advanced mediastinal infection and sepsis. Exclusion of the perforated
esophagus and T-tube drainage of a perforation are alternative approaches that cannot be considered for primary treatment.
Antibiotics and drainage as the sole treatment is reserved for a very small, selected population of patients with well-
contained esophageal perforation.

16. A 50-year-old patient develops sudden left lower chest pain and epigastric pain after vomiting. The patient shows
diaphoresis, breath sounds are decreased on the left, and there is abdominal guarding. The most appropriate diagnostic test
is:
A. Aortography.
B. Esophagoscopy.
C. Electrocardiogram.
D. Film of the chest.
E. White blood count.
Answer: D

DISCUSSION: The history of pain after vomiting efforts suggests esophageal rupture. Pain is often described as
excruciating and frequently masquerades as a dissecting aneurysm, perforated ulcer, or myocardial infarction. Decreased
breath sounds suggest the possibility of hydropneumothorax. The diagnostic procedure is a chest film. More than 90% of
patients with an esophageal perforation show abnormalities suggestive of perforation. The findings are influenced by the
interval between perforation and the examination, by the site of the perforation, and by the integrity of the mediastinal
pleura. Esophagoscopy is not indicated.

17. The following statements about the influence of diet and lifestyle on lower esophageal sphincter (LES) function are true
except one. Identify the incorrect statement.
A. A high-protein diet increases LES pressure.
B. A fat meal results in sustained decrease in LES pressure.
C. Chocolate ingestion causes a decrease in LES pressure.
D. Peppermint produces a transient decrease in LES values.
E. Cigarette smoking produces no significant changes in LES pressures.
Answer: E

DISCUSSION: There is a dramatic decrease in LES pressure following the ingestion of fat. Chocolate has the same effect
on LES resting pressures. Peppermint was shown to produce transient decreases in LES pressures of 20 to 30 seconds,
which occur approximately 10 minutes after ingestion. The ingestion of carbohydrates produces no significant change in
sphincter pressures, but a high-protein meal increases LES pressure. Cigarette smoking has also been shown to produce
significant decreases in LES pressure that persist throughout the duration of active smoking.

18. When a stricture is present in association with gastroesophageal reflux, each of the following is an acceptable repair for
reflux control except one. Identify the poorest repair.
A. Intrathoracic total fundoplication.
B. Lengthening gastroplasty with total fundoplication.
C. Total fundoplication.
D. Lengthening gastroplasty with partial fundoplication.
E. Partial fundoplication.
Answer: E

DISCUSSION: When a stricture is present, periesophagitis and shortening limit the chances of obtaining a sufficient length
of intra-abdominal esophagus. Even extensive mobilization of the esophagus to the aortic arch and freeing of the
esophagogastric junction does not afford a comfortable 4 to 5 cm. of esophagus under the diaphragm. A tension-free repair
is not possible in such circumstances. Partial fundoplication at this stage of the disease is followed by a 45% failure rate.
Excellent results have been reported using a total fundoplication following dilatation of the stricture, and intrathoracic
fundoplication provided good results. The lengthening gastroplasty with a partial fundoplication or with total fundoplication
shows satisfactory control of reflux in a majority of patients.
19. When assessing gastroesophageal reflux disease by manometry each of the following statements is correct except one.
Identify the incorrect one.
A. Absent or extremely low LES pressures have predictive value in identifying more severe reflux.
B. Peristaltic dysfunction increases with increasing severity of esophagitis.
C. With established reflux disease the UES is hypertensive.
D. Esophageal functional changes are worst in patients with a circumferential columnar-lined esophagus.
E. Absence of peristalsis may be associated with more severe forms of reflux disease.
Answer: C

DISCUSSION: In reflux disease when LES pressure is below 10 mm. Hg, manometry is too imprecise to identify a
potential for significant reflux. If the pressure is less than 6 mm. Hg, this shows a reasonable high specificity as compared
with abnormal reflux on pH testing. When LES pressure is extremely low or nonexistent, this identifies a more severe
degree of reflux and a poorer prognosis for long-term medical therapy. In the esophageal body, active reflux esophagitis
causes altered function. Failed peristalsis increases, and the contractions become weaker. Patients with a columnar-lined
esophagus have the worst functional abnormalities. Although distention or acid perfusion in the proximal esophagus can
produce a significant increase in UES resting pressure, there is at present no solid evidence relating UES resting pressures to
active reflux disease.

20. The presence of a nonmalignant mid- or upper esophageal stricture always indicates the presence of:
A. Alkaline reflux esophagitis.
B. Barrett's esophagus.
C. Idiopathic reflux disease.
D. Mediastinal fibrosis.
E. Scleroderma.
Answer: B

DISCUSSION: A stricture at or above the aortic arch is almost certainly situated above an esophagus lined at its lower end
with columnar epithelium. Barrett's esophagus is suggested radiographically when local esophagitis, ulcer, or stricture is at
the limits of a normal-looking segment of esophagus under the stricture but above a herniated stomach. The columnar-lined
esophagus is not always associated with a high stricture; however Messiaen and Halpert documented strictures in 80% of
their Barrett's patients. These high strictures, when seen with alkaline reflux esophagitis, with idiopathic reflux disease, or in
association with scleroderma, always suggest the presence of a columnar-lined esophagus. Mediastinal fibrosis is a rare
condition that can cause multilevel strictures on the esophagus.

21. Which of the following is most reliable for confirming the occurrence of a significant esophageal caustic injury?
A. History of the event.
B. Physical examination of the patient.
C. Barium esophagraphy.
D. Endoscopy.
Answer: D

DISCUSSION: In the absence of physical or radiographic evidence of upper airway obstruction or esophagogastric
perforation, the presence of a significant caustic injury can be defined reliably only by direct visualization at the time of
endoscopy. Although the history may shed light on the possibility of a burn and its severity, all too often the event goes
unwitnessed or the type and amount of ingested substance are not known with certainty. The identification of oropharyngeal
burns clearly indicates the need for endoscopy, but as many as 70% of patients with such lesions escape associated
esophageal injury. Conversely, 10% to 30% of patients with no external evidence of burns have subsequently been
confirmed by esophagoscopy to have sustained damage. In the absence of an identifiable perforation, a barium
esophagogram can rarely be considered unequivocally diagnostic of acute injury, though such an injury may be suggested
when the esophagus appears atonic and dilated, rigid and persistently narrowed, or excessively irritable. Because of the
importance of early confirmation of the presence or absence of a significant esophageal burn as a guide to formulating
appropriate treatment, esophagoscopy should be performed expeditiously as soon as sufficient time has elapsed to allow
gastric emptying and stabilization of the patient, preferably within the first 12 to 48 hours after ingestion.

22. Indications for surgical reconstruction of the esophagus include which of the following?
A. Continuing requirement for frequent dilation of an extensive esophageal stricture for a minimum of 2 years.
B. Failure or refusal of the patient to comply with a treatment regimen of regular dilation.
C. Development of a fistula between the esophagus and tracheobronchial tree.
D. Iatrogenic perforation of the esophagus during attempted dilation.
Answer: BCD

DISCUSSION: Development of a tracheobronchial fistula almost always necessitates some form of esophageal
reconstruction because of the extensive damage usually associated with it. Failure of the patient to cooperate effectively
condemns to failure any attempt at restoring esophageal patency by bougienage, because sporadic attempts at dilation do not
allow progressive lumen enlargement but, rather, invite additional injury because of the necessity for repeated
instrumentation of a recurrent, tight stricture. The need for repeated dilation of extensive or multiple strictures over a period
exceeding 6 months should prompt surgical reconstruction of the esophagus, especially in young children, for whom the
psychological and physical hazards are intensified by prolonged treatment. In such circumstances, a 2-year period of
attempted bougienage is excessive. Although iatrogenic perforation often signals the need for esophageal reconstruction,
this misadventure should not be considered an absolute indication but should be assessed in relation to (1) the extent and
complexity of the stricture, (2) the potential for eventually achieving successful bougienage, and (3) the severity of
complications caused by the secondary injury.

23. First-line therapy for routine peptic duodenal ulcer disease includes:
A. Vagotomy and antrectomy.
B. Upper endoscopy and biopsy to rule out tumor.
C. Evaluation for Helicobacter pylori.
D. Serum gastrin determination.
E. Cream or milk-based “Sippy” diet.
Answer: C

DISCUSSION: Vagotomy and antrectomy is the definitive surgical therapy for peptic ulcer disease but should be applied
only for complications of the disease or after refractory disease has been documented. Biopsy of routine peptic duodenal
ulcer is not indicated to rule out malignancy except in special circumstances, such as an endoscopic appearance typical of
malignancy. H. pylori is found in a large percentage of peptic ulcer patients, and treatment alters the rate of recurrence if
therapy is directed toward reduction of H. pylori in addition to acid. Measurement of serum gastrin is recommended for
patients with resistant or persistent peptic ulcer disease for patients undergoing surgery for peptic ulcer disease. The Sippy
diet has not been recommended clinically for years. Formerly it was recommended as a bland diet that would not exacerbate
peptic ulcer disease. It is now known that these diets are heavily calcium-laden and probably exacerbate peptic ulcer
disease.

24. Appropriate management of severe vomiting associated with gastric outlet obstruction from peptic ulcer disease
includes all of the following except:
A. Nasogastric suction.
B. Intravenous hydration.
C. Nutritional assessment; upper endoscopy to rule out malignancy.
D. Intravenous H 2 antagonist.
E. Oral antacid therapy.
Answer: E

DISCUSSION: All patients should undergo nasogastric suction, rehydration, and control of acid secretion. This control of
acid secretion requires an H 2 antagonist since oral antacids are often inadequate to neutralize the large volume of acid often
present in the obstructed stomach. Patients with a long history of obstruction are often nutritionally compromised and need
careful nutritional assessment before operative planning.

25. All of the following are complications of peptic ulcer surgery except:
A. Duodenal stump blowout.
B. Dumping.
C. Diarrhea.
D. Delayed gastric emptying.
E. Steatorrhea.
Answer: E
DISCUSSION: Duodenal stump blowout occurs after Billroth II operations, where back-pressure in the duodenal stump
results in breakdown of this stump closure, leading to abdominal sepsis. Dumping syndrome and postvagotomy diarrhea are
complications of peptic ulcer surgery. They represent two different syndromes, both of which are predominately related to
the vagotomy portion of the operation. Delayed gastric emptying occurs frequently after peptic ulcer surgery, for a variety of
reasons, but it is most common after elective peptic ulcer surgery for gastric outlet obstruction. Steatorrhea is not necessarily
related to peptic ulcer surgery but is a complication of pancreatic insufficiency.

26. The presentation of Zollinger-Ellison syndrome includes all of the following except:
A. Hyperparathyroidism in patients with multiple endocrine neoplasia type 1 (MEN 1) syndrome.
B. Diarrhea.
C. Migratory rash.
D. Jejunal ulcers.
E. Duodenal ulcers.
Answer: C

DISCUSSION: Zollinger-Ellison syndrome occurs in two settings: sporadically and in association with MEN 1 syndrome.
MEN 1 syndrome includes parathyroid adenomas, and the initial presentation is often related to this parathyroid disease.
Diarrhea is a common presentation for Zollinger-Ellison syndrome, since hyperacidity can result in diarrhea due to the
volume of acid secreted or from a steatorrhea-type diarrhea when the high levels of acids inactivate the pancreatic enzymes.
Migratory rash is commonly associated with glucagonoma but not with Zollinger-Ellison syndrome. Both jejunal and
duodenal ulcers can be found with Zollinger-Ellison syndrome.

27. All are true about the dumping syndrome except:


A. Symptoms can be controlled with a somatostatin analog.
B. Diarrhea is always part of the dumping syndrome.
C. Flushing and tachycardia are common features of the syndrome.
D. Separating solids and liquids in the patient's oral intake alleviates some of the symptoms of the syndrome.
E. Early postoperative dumping after vagotomy often resolves spontaneously.
Answer: B

DISCUSSION: The somatostatin analog octreotide has been used to control the dumping syndrome and is currently the only
known medical therapy for this disease. Other therapies include dietary measures such as six small meals a day and
separation of solids and liquids. Postvagotomy diarrhea is a secondary complication of vagotomy and is not strictly
associated with the dumping syndrome itself. The vast majority of patients with dumping syndrome experience spontaneous
resolution of their symptoms without intervention in the postoperative period.

28. In patients with bleeding duodenal ulcers, the endoscopic finding associated with the highest incidence of
rebleeding is:
A. Visible vessel.
B. Cherry-red spot.
C. Clean ulcer bed.
D. Duodenitis.
E. Shallow, 3-mm. ulcer.
Answer: A

DISCUSSION: A visible vessel in an ulcer bed is associated with a 50% chance of rebleeding and, other than an actively
bleeding vessel, is the worst endoscopic prognostic indicator for rebleeding. Cherry-red spot, adherent clot, and clean small
ulcers all are associated with a lower incidence of rebleeding.

29. All of the following are contraindications for highly selective vagotomy except:
A. Intractable duodenal ulcer disease.
B. Peptic ulcer disease causing gastric outlet obstruction.
C. Fundic peptic ulceration.
D. Cigarette chain smoking.
E. Perforated peptic ulcer disease with more than 24 hours' soilage.
Answer: A

DISCUSSION: Intractable peptic ulcer symptoms are a classic indication for highly selective vagotomy. Patients with
gastric outlet obstruction often do poorly with highly selective vagotomy and develop recurrent ulceration. Highly selective
vagotomy is not indicated for gastric ulceration. Heavy chain smokers often get recurrent peptic ulceration after highly
selective vagotomy; therefore, vagotomy and antrectomy is indicated for them. Patients who experience long periods of
perforation before exploratory laparotomy should receive either patch plus vagotomy or pyloroplasty or patch of the ulcer
alone. Extensive operations, such as highly selective vagotomy, are usually not indicated in this acute setting.

30. All the following are true of omeprazole except:


A. It is the only drug available that has the potential to achieve pharmacologically induced achlorhydria.
B. It works by blocking the hydrogen-potassium ATPase in the parietal cell.
C. It is parietal cell specific.
D. It has a short half-life (about 90 minutes) when taken orally.
E. It has been associated with gastric neoplasm in a rat model.
Answer: D

DISCUSSION: Omeprazole and drugs in this category are the only drugs that can produce achlorhydria. All other antiacid
drugs reduce acid secretion without producing achlorhydria. Omeprazole inhibits acid at the final common pathway by
blocking the hydrogen-potassium ATPase in parietal cells. It is gastric parietal cell specific and has a very long half-life,
allowing once daily dosing when given orally. When it was given to rats in pharmacologic doses the gastric mucosa formed
carcinoid-type tumors. This problem has not been identified in humans.

31. All of the following statements about gastrin-releasing peptide (GRP) are true except:
A. In species other than man and dog GRP is commonly referred to as bombesin.
B. GRP serves as a neurotransmitter.
C. GRP inhibits pancreatic secretion when given intravenously.
D. GRP stimulates gastric acid secretion when given intravenously.
E. GRP is released in response to cholinergic stimulation of the parietal cells to stimulate release of gastrin.
Answer: C

DISCUSSION: Gastrin-releasing peptide and bombesin are homologous peptides of different amino acid lengths. GRP
functions as a neurotransmitter at the cholinergic nerve ending on the parietal cell and releases gastrin after cholinergic
stimulation. It functions to increase gastric acid secretion and also pharmacologically increases pancreatic secretion.

32. Cholecystokinin (CCK) is believed to function in all of the following processes except:
A. It physiologically delays gastric emptying.
B. It appears to have a role in satiety regulation.
C. It contracts the gallbladder.
D. It stimulates pancreatic secretion.
E. It is important in the control of the anal sphincter.
Answer: E

DISCUSSION: CCK has a physiologic role in the regulation of gastric emptying, eating behavior, gallbladder contraction,
and pancreatic secretion. There is experimental evidence that it may serve as a neurotransmitter in the function of the lower
esophageal sphincter. It probably also has a role in augmenting the release of insulin after a meal. It has no known role in
the function of the anal sphincter.

33. All of the following measures have been recommended for control of acid secretion in patients with Zollinger-
Ellison syndrome except:
A. Antrectomy.
B. Highly selective vagotomy.
C. Total gastrectomy.
D. Vagotomy and pyloroplasty.
E. Medical therapy with Prilosec (omeprazole).
Answer: A

DISCUSSION: Patients with MEN 1 syndrome or sporadic-metastatic Zollinger-Ellison syndrome should be palliated with
omeprazole to control their acid secretion. Patients who undergo exploration may have a variety of operations to control
their ulcer diathesis, including total gastrectomy or various vagotomy-type operations. Antrectomy alone is not indicated,
since the gastrin that is contributing to the production of acid is not coming from the antrum but coming from the tumor.

34. All of the following contribute to peptic ulcer disease except:


A. Cigarette smoking.
B. Nonsteroidal anti-inflammatory drugs.
C. Helicobacter pylori.
D. Gastrinoma.
E. Spicy foods.
Answer: E

DISCUSSION: Cigarettes and nonsteroidal anti-inflammatory drugs are common contributors to peptic ulceration. H. pylori
is found in most patients with peptic ulceration, and eradication of this bacterium decreases the recurrence rate for peptic
ulcer disease. Gastrinoma results in much acid secretion and commonly presents with peptic ulcer disease. Dietary factors
such as spicy foods have little or no effect on postprandial acid secretion and do not contribute to peptic ulceration.

35. Which of the following statements about gastric polyps is/are true?
A. Like their colonic counterparts, gastric epithelial polyps are common tumors.
B. They are analogous to colorectal polyps in natural history.
C. Endoscopy can uniformly predict the histology of a polyp based on location and appearance.
D. In a given patient, multiple polyps are generally of a single histologic type.
E. Gastric adenomatous polyps greater than 2 cm. in diameter should be excised because of the risk of malignant
transformation.
Answer: DE

DISCUSSION: As early as 1895 Hauser reported an association between familial adenomatous polyposis of the colon and
multiple gastric polyps. This early association may have given rise not only to the confusing nomenclature of gastric polyps
but also to the mistaken notion that they are analogous to colorectal polyps in microscopic appearance and natural history.
Unlike colonic polyps, gastric epithelial polyps are very uncommon tumors (prevalence 0.4% to 0.8%). Their histologic
appearance cannot be predicted on the basis of location in the stomach, although the endoscopic literature is beginning to
define predictive algorithms based on location and ultrasound. Multiple polyps are almost always of a single histologic type.
Gastric adenomatous polyps have long been associated with adenocarcinoma. This association is directly related to the size
of the polyps. Up to 24% of polyps 2 cm. or greater in diameter are associated with adenocarcinoma. In contrast, only 4% of
polyps with a diameter less than 2 cm. are associated with carcinoma. The risk, if any, of carcinoma in patients with
hyperplastic polyps appears to be associated with the atrophic gastritis that frequently accompanies them rather than with
the polyps themselves.

36. Which of the following statements about gastric leiomyomas is/are true?
A. They are the most common type of gastric tumor of the stomach at autopsy.
B. The leiomyoblastoma cell type reflects malignant transformation of gastric leiomyomas.
C. A conservative surgical approach is indicated for their resection since regional lymphadenectomy has not been proved
reliable even when they turn out to be malignant.
D. Severe hemorrhage may occur from deep ulcerations overlying the intramural tumor.
Answer: ACD

DISCUSSION: Approximately 40% of benign tumors of the stomach are leiomyomas derived from the smooth muscle of
the stomach or its associated blood vessels. Because it is rare for gastric leiomyomas smaller than 3 cm. in diameter to be
symptomatic, considerably fewer than 2% of gastric neoplasms resected surgically are of smooth muscle origin. Gastric
leiomyomas may be smooth or lobulated, but in time a central ulceration occurs in the mucosal bulge of the tumor in
approximately half of submucosal leiomyomas. Ulceration may be present in smaller tumors but absent in very large
tumors. Overlying central mucosal ulceration, which may penetrate deeply into the tumor, results in hematemesis, melena,
or anemia and draws attention to the tumor. Bleeding from the tumor may be massive and/or intermittent.
Gastric leiomyomas are not encapsulated, even though on section they appear to be well-circumscribed. Microscopically,
the tumor cells at the margin may intermingle with cells of the surrounding gastric wall. Along with the presence of
occasional large cells with hyperchromatic nuclei, this has led to confusion in distinguishing benign tumors from malignant
ones. Stout described a reasonably distinct variety of gastric smooth muscle tumor that he called leiomyoblastoma (bizarre
smooth muscle tumor). They were characterized histologically by polyhedral smooth muscle cells with central nuclei and
abundant cytoplasm rather than elongated cells. A clear zone that surrounds the central nucleus may be an artifact of
fixation. Leiomyoblastoma may be benign or malignant. Carney has described a syndrome characterized by the triad of
multiple malignant leiomyoblastoma, pulmonary chondroma, and functioning extra-adrenal paraganglioma.
The principle of surgical treatment of smooth muscle tumors is local excision with a 2- to 3-cm. margin of surrounding
gastric wall. In view of the difficulty in distinguishing between the benign and malignant variants, enucleation is not an
appropriate method of treatment. Regional lymphadenectomy is not of proven value, even if malignancy is strongly
suspected and is not consistent with the known property of these tumors to spread by the hematogenous route.

37. The sine qua non of the histologic diagnosis of a gastric pseudolymphoma is:
A. Extragastric extension of the gastric lesion.
B. Nodal involvement beyond the immediate stomach.
C. A germinal center in the gastric lesion.
D. Extension into esophagus and duodenum.
E. Unresponsive to conservative gastric resection.
Answer: C

DISCUSSION: Pseudolymphoma represents approximately 10% of all gastric lymphomas. These are benign conditions
involving mostly the mucosa without evidence of nodal disease and without extragastric extension. The sine qua non for the
diagnosis is a germinal center within the gastric lesion. These are premalignant lesions but can be cured completely with
conservative resection.

38. All of the following statements about surgical management of gastric lymphomas are true except:
A. Stage I gastric lymphomas (small lesions confined to the stomach wall) can be cured completely with surgical therapy
alone.
B. Extensive gastric lymphomas that initially are treated with radiation and/or chemotherapy occasionally perforate during
treatment and require secondary resection.
C. Patients explored with a presumptive diagnosis of gastric lymphoma should undergo an attempt at curative resection
when this is safe and feasible.
D. Without a preoperative diagnosis resection for gastric mass should not be attempted unless lymphoma can be excluded.
E. Appropriate staging for primary gastric lymphoma includes bone marrow biopsy.
Answer: D

DISCUSSION: Operation alone is adequate treatment for very early-stage lymphoma, although chemotherapy is commonly
added. For more advanced disease, particularly stages III and IV, preoperative radiation chemotherapy is often indicated,
even though some of these patients suffer perforation during therapy and require emergent resection. Patients who undergo
exploration for gastric mass without a preoperative diagnosis can safely be resected with potential for cure even if the
diagnosis includes gastric lymphoma.

39. Which of the following risk factors have been shown to increase significantly the incidence of gastrointestinal
bleeding from stress gastritis in intensive care unit (ICU) patients?
A. Glucocorticoid administration.
B. Respiratory failure.
C. Coagulopathy.
D. Organ transplantation.
E. Jaundice.
Answer: BC

DISCUSSION: Prophylactic measures such as H 2-receptor antagonists and antacid titration effectively reduce the
incidence of gastrointestinal bleeding; however, prophylaxis against stress gastritis is expensive and may have adverse
effects. Therefore, it should be used selectively in patients with high risk factors. In a prospective multicenter study in which
10 potential risk factors were evaluated for stress gastritis bleeding in ICU patients, respiratory failure and coagulopathy are
two independent risk factors for clinically significant bleeding. Therefore, a strong recommendation for prophylaxis of
stress gastritis can be made for ICU patients who have either respiratory failure or coagulopathy.

40. Which of the following measures are effective in preventing stress gastritis bleeding in critically ill patients?
A. Improving systemic circulation by correcting any shocklike state resulting from blood loss or sepsis.
B. Correcting systemic acid-base abnormality.
C. Maintaining adequate nutrition.
D. Reducing intragastric acidity by either antacid titration or H 2 antagonists.
Answer: ABCD

DISCUSSION: Despite the lack of documentation, a strong impression exists among clinicians and clinical investigators
that the incidence and prevalence of stress gastritis have decreased significantly during the past decade, perhaps owing to
improved general care for critically ill patients. The improvement in general care of these critically ill patients includes
vigorous efforts to correct any shocklike state secondary to blood loss or sepsis, better ventilatory support, and maintenance
of adequate nutrition. These prophylactic measures enhance the ability of the gastric mucosa to protect itself against acid
injury. In addition, several prospective, randomized studies have shown that antacid titration and/or H 2-receptor antagonists
are effective in preventing gastrointestinal bleeding in these patients.

41. Which of the following have been used successfully to treat patients with vascular compression of the duodenum?
A. Subtotal gastrectomy and Roux-en-Y gastrojejunostomy.
B. Total parenteral nutrition.
C. Division of the ligament of Treitz and duodenal mobilization.
D. Percutaneous endoscopic gastrostomy.
E. Duodenojejunostomy.
Answer: BCE

DISCUSSION: Vascular compression of the duodenum is best treated initially with supportive care. Of paramount
importance is supplying adequate nutrition, since most patients have significant weight loss with this syndrome. This can
best be done with a nasojejunal feeding tube placed past the ligament of Treitz (and the obstructed area). Gastrostomy alone
does not provide unobstructed enteral access. Parenteral nutrition may be used successfully when enteral access cannot be
established. When operative therapy is needed, duodenojejunostomy has been the most common and successful operation
and is the treatment of choice for adults. In the pediatric population, division of the ligament of Treitz and duodenal
mobilization has also proved successful. Gastrojejunostomy has been used, but with a lower overall success rate. Distal
gastrectomy usually worsens duodenal obstruction by preventing duodenogastric reflux.

42. Which of the following statements about the anatomic basis for the syndrome of vascular compression of the
duodenum are true?
A. The duodenum is obstructed in its distal third as it crosses over the lumbar vertebral column.
B. Structures crossing beneath the superior mesenteric artery include the duodenum, the uncinate process of the pancreas,
and the left renal vein.
C. Hyperextension of the back allows the angle of origin of the superior mesenteric artery to widen, lessening the
obstruction of the duodenum.
D. Patients are at significant risk for vascular compression of the duodenum if the angle between the takeoff of the superior
mesenteric artery and the aorta is less than 45 degrees.
E. Arteriographic studies show a typical area of extrinsic compression and narrowing of the arterial lumen due to duodenal
pressure.
Answer: AB

DISCUSSION: The superior mesenteric artery originates behind the neck of the pancreas at the level of the first lumbar
vertebra. It arises from the aorta at an acute angle, usually about 37 degrees in normal patients, through which passes the left
renal vein, the uncinate process of the pancreas, and the distal third of the duodenum. The duodenum crosses the lumbar
spine from right to left and passes upward. It is at this point of passage of the duodenum upward and over the spine that the
obstruction occurs. Arteriographic studies show that the aortomesenteric angle in patients with the syndrome is only about 8
degrees. There is no narrowing of the superior mesenteric artery or disturbance of arterial flow, but the area of duodenal
obstruction corresponds to the compression of the bowel by the artery. The duodenal compression may often be relieved by
assuming the knee-chest, the left lateral, or even the prone position. Increasing lumbar lordosis, as with hyperextension of
the back, exacerbates the problem.

43. Numerous epidemiologic associations have been made between (1) environmental and dietary factors and (2) the
incidence of gastric cancer, including all except:
A. Dietary nitrites.
B. Dietary salt.
C. Helicobacter pylori infection.
D. Dietary ascorbic acid.
Answer: D

DISCUSSION: Numerous epidemiologic studies support the role of certain foods in the development of gastric cancer. Salt,
which can act as a gastric irritant, and nitrates and nitrites, which can be converted to the active carcinogens N-nitrosamines,
are implicated in the development of gastric cancer. H. pylori infection is associated with atrophic gastritis, a known
precursor to gastric cancer. Important studies of large populations indicate that the majority of patients with gastric cancer
are H. pylori positive. The bacteria produce toxins such as ammonia and acetaldehyde, which could lead to chronic
inflammation and epithelial damage. Dietary ascorbic acid has been associated with overall improvements in diet and is not
associated with the development of gastric cancer.

44. All of the following benign conditions are associated with increased rates of gastric cancer except:
A. Pernicious anemia.
B. Multiple endocrine neoplasia type I (MEN 1).
C. Adenomatous polyps.
D. Chronic atrophic gastritis.
Answer: B

DISCUSSION: Adenomatous polyps are unusual but carry the distinct potential for malignancy. They occur most often
between the fifth and seventh decades of life. The adenocarcinoma sequence in gastric polyps is thought to be analogous to
that of colonic polyps. An adenomatous polyp is a marker for increased risk of carcinoma in the remaining stomach. Both
pernicious anemia and chronic atrophic gastritis are associated with gastric cancer. Many of these patients develop chronic
achlorhydria, a condition also associated with an increased risk of cancer. Neither multiple MEN 1 nor MEN 2, is associated
with gastric cancer.

45. Which of the following statements concerning the pathology of gastric cancer is true?
A. Distal gastric cancers are becoming more common.
B. Intestinal-type gastric tumors resemble colon carcinomas and have a better prognosis than diffuse type.
C. Early gastric cancers are confined to the mucosa and are lymph node negative.
D. Broders' histologic grading system correlates well with survival: patients with grade IV tumors have 5-year survival
rates around 65%.
Answer: B

DISCUSSION: Distal gastric cancers are decreasing in incidence in several populations. Lesions of the gastroesophageal
junction and cardia have increased in incidence over the past two decades. Early gastric cancers are confined to the mucosa
and submucosa of the stomach. Six to 10% of these early lesions are lymph-node positive. The survival rates from early
gastric cancer is related to node positivity, just as in advanced gastric cancer. Broders' histologic grades do correlate well
with survival. Grade I and IV tumors are associated with a 65% and 11% 5-year survival, respectively. The Lauren
classification system is divided into intestinal and diffuse-type tumors. The intestinal-type tumor is more analogous to colon
carcinoma and has a better prognosis than the diffuse type.

46. An 80% distal gastrectomy is performed for a 6-cm. antral cancer with extension to the muscularis propria and
three positive lymph nodes less than 3 cm. from the tumor. The stage of this tumor was:
A. Stage I.
B. Stage II.
C. Stage III A.
D. Stage III B.
Answer: B

DISCUSSION: The American Joint Committee on Cancer Staging system depends on primary tumor, lymph node
involvement, and distant metastasis. The tumor described is a T2N1M0 tumor, which categorizes it as stage II.

47. Which of the following statements about the surgical treatment of gastric cancer is false?
A. Patients with tumors of the middle and proximal thirds should undergo total gastrectomy.
B. Adenocarcinoma of the cardia-gastroesophageal junction may require reconstruction in the abdomen, chest, or neck.
C. Palliative resection yields better results than palliative bypass.
D. Japanese patients who undergo gastric resection are, on average, 10 years younger and much leaner than their Western
counterparts.
Answer: A

DISCUSSION: Depending on the size and extent of the tumor, cancers of the gastroesophageal junction may extend
proximally into the esophagus for a varying distance. Reconstruction may be required in the abdomen, chest, or neck,
depending on extension and whether the operation is to be palliative or curative. Palliative resection yields better results
than palliative bypass, which is unreliable for relieving obstruction. Japanese patients typically are younger and thinner than
their Western counterparts. In addition, they have a higher prevalence (up to 50%) of early gastric cancer. Depending on the
size and particular location of the tumor, patients with small middle-third tumors or small lesions of the cardia may undergo
subtotal proximal gastrectomy and reconstruction with a gastric tube. If 6-cm. margins can be obtained on either side of the
lesion, total gastrectomy is unnecessary and may be associated with a higher risk of morbidity.

48. Which of the following measures of obesity correlates best with mortality?
A. The 1983 Metropolitan Life Insurance Company tables for ideal body weight.
B. Hydroimmersion measurements of body fat composition.
C. Body mass index (BMI).
D. Skinfold thickness.
E. Waist to hip ratios (WHR).
Answer: C

DISCUSSION: The measurement of obesity is still an inexact science. The Metropolitan Life Insurance tables, although
widely used, do not distinguish between lean muscle mass and fat. Accordingly, the BMI (weight in kg./height in meters) 2,
was developed to place greater emphasis on “fatness.” The measure correlates linearly with mortality tables.
Hydroimmersion data are still too sparse to relate to outcome tables. Skinfold thickness and waist-hip ratios have not been
shown to have the accuracy or relevance of the BMI.

49. The most effective therapy for morbid obesity, in terms of weight control, is:
A. Intensive dieting with behavior modification.
B. A multidrug protocol with fenfluramine, phenylpropanolamine, and mazindol.
C. A gastric bypass with a 40-ml. pouch, a 10- to 20-cm. Roux-en-Y gastroenterostomy.
D. A gastric bypass with a 15-ml. pouch, a 40- to 60-cm. Roux-en-Y gastroenterostomy.
E. Daily exercise with strong emphasis on utilizing all four limbs.
Answer: D

DISCUSSION: Although the various nonsurgical measures listed in the question have proved effective for obese persons,
they work only rarely for those who are morbidly obese. None have proved as effective as gastric bypass with a 15-ml.
pouch and a 40- to 60-cm. Roux-en-Y gastroenterostomy.

50. Which of the following statements about intestinal bypass is/are correct?
A. The operation produced weight loss similar to that of the gastric bypass.
B. The operation produced severe metabolic disturbances, including hypocalcemia, increased bile salts and glycine
synthesis.
C. Bacterial overgrowth in the bypassed segment led to liver failure.
D. The operation demonstrated that an adult human could survive with 40 to 50 cm. of small intestine.
Answer: ABCD

DISCUSSION: Unfortunately, all of the answers are true. Even though the intestinal bypass proved initially to be an
effective procedure to induce weight loss, the side effects proved to be so severe that almost all of the operations had to be
reversed or revised to gastric bypass to avert death from liver failure or severe illnesses due to malnutrition.

51. Which of the following is/are contraindications to gastric bypass surgery?


A. Diabetes mellitus.
B. Hypertension.
C. Pickwickian syndrome.
D. Failure to agree to long-term follow-up.
E. Sleep apnea.
Answer: D

DISCUSSION: The gastric bypass represents the best known therapy for diabetes mellitus, hypertension, the Pickwickian
syndrome, and sleep apnea. In fact, no other therapy provides such complete control of hyperglycemia and
hyperinsulinemia, reversal of hypertension, and total correction of the Pickwickian syndrome and most cases of sleep apnea.
The only contraindication to bariatric surgery listed is failure to agree to long-term follow-up. The gastric bypass represents
controlled malnutrition and, therefore, vitamin therapy is especially important. If patients are not followed closely, vitamin
deficiencies, especially of B 6 and B 12, can develop with serious consequences including a Korsakoff-Wernicke syndrome.

52. A 34-year-old morbidly obese diabetic woman underwent a gastric bypass about 12 hours ago. The operation was
technically difficult but finally went well. You are called because she now has a temperature of 99.2‫ ؛‬F, pulse of
134, and some pain in her incision and her back. She looks well; the incision is clean; and her examination is
otherwise negative. A bolus of 500 ml. of dextrose/lactated Ringer's did not change her vital signs, except that her
pulse rose to 140 without an increase in urine output. Your next step should be:
A. Another bolus of crystalloids.
B. Posterioanterior and lateral chest films.
C. Obtain white cell count, differential count, and electrolyte values.
D. Call the operating room and warn them that you need to re-explore for a leak.
E. Increase her pain medication.
Answer: D

DISCUSSION: Morbidly obese patients are malnourished and brittle and have little resistance. Serious life-threatening
infections may soil the peritoneal cavity without producing any sign except a persistently high pulse rate. The usual tests
listed in A, B, and C, should be done, but the most likely explanation for a continued high pulse is soiling in the area of the
surgery due to either a leak or contamination and development of sepsis. Because of the unreliability of clinical evaluation,
the indications for re-exploration are very liberal, and this approach has saved a number of lives. The risk of such an
exploration is small, whereas failure to contain the infection with lavage and drainage may be followed by a surprisingly
rapid death.

53. Metabolic complications of subtotal gastrectomy with Billroth I or Billroth II reconstruction include:
A. Hypothyroidism.
B. Anemia.
C. Reactive hypoglycemia.
D. Dumping syndrome.
E. Metabolic bone disease.
Answer: BCDE

DISCUSSION: Anemia develops in as many as 30% of patients within 15 years of surgery. The cause is multifactorial and
includes malabsorption of iron, folate, and vitamin B 12. A metabolic bone disease occurs in as many as 33% of patients, is
similar to osteomalacia, and is probably a result of malabsorption of calcium and vitamin D. Reactive hypoglycemia occurs
with rapid gastric emptying, resulting in increased glucose absorption immediately after a meal. Initially there is
hyperglycemia, leading to hyperinsulinemia and subsequent rapid glucose clearance and symptomatic hypoglycemia.
Dumping syndrome varies from very mild symptoms to significantly disabling ones. The severe syndrome occurs in fewer
than 5% of patients. Small, frequent, dry meals of low osmolality reduces symptoms, and somatostatin analog has been of
some clinical use.

54. Which of the following statement(s) concerning the surgical options for an anti-reflux operation is/are true?

a.A patient with normal esophageal length and esophageal body motility is best served by laparoscopic Nissen
fundoplication
b.A patient with a low peristaltic amplitude of the distal third of the esophagus is a candidate for an open Nissen
fundoplication
c.A Collis gastroplasty is an additional procedure that can be added in patients with extensive esophageal shortening
d.End-stage reflux disease such as an undilatable stricture or Barrett’s esophagus with high grade dysplagia is best
managed by a colon interposition
Answer: a, c, d

Patients with normal esophageal length and normal esophageal body motility are best served by a transabdominal Nissen
fundoplication. This is now normally done via the laparoscopic route. If the patient is very obese or requires concomitant
surgery on the lung or esophageal body, the transthoracic approach is preferable. The presence of a motility disorder alters
the operative strategy. If the peristaltic amplitude is low (20 mm Hg) in the distal third of the esophagus, a Nissen
fundoplication would create too much resistance and lead to dysphagia. In this situation the Belsey fundoplication is a better
choice. Moreover, it allows the surgeon to mobilize the esophagus to a much greater extent than is possible through the
abdomen. In addition to extensive mobilization, a Collis gastroplasty can be created to produce an extra 5 cm of “neo-
esophagus” around which a Belsey procedure can be added. End-stage reflux disease, for example, when there is an
undilatable stricture or after previous unsuccessful anti-reflux operations or when Barrett’s esophagus leads to high grade
dysplagia, is best served by esophageal replacement. The most durable substitute is the colon, and the functional results are
especially good if the vagus nerves are intact.

55. Factors associated with the development of complications of gastroesophageal reflux disease include:

a.The presence of a defective lower esophageal sphincter


b.Inadequate esophageal clearance
c.The presence of a hiatal hernia
d.The presence of an alkaline component of the reflux material
Answer: a, b, c, d

The status of the lower esophageal sphincter (LES) has emerged as a significant factor in several long-term studies of
gastroesophageal reflux disease, and serves as a predictor of a poor response to medical treatment. Barrett’s esophagus is
almost always associated with a mechanically defective sphincter. Any defects in esophageal clearance which prolongs the
contact time between the refluxate and the mucosa is likely to lead to increased esophageal injury. The presence of a hiatal
hernia is also associated with more complications of gastroesophageal reflux disease. Finally, the composition of the reflux
material also has an effect on the development of complications. In a clinical situation, complications of gastroesophageal
reflux disease are more common when there is an alkaline component to the refluxate. In Barrett’s esophagus, the
development of complications such as stricture and ulceration is strongly associated with increased alkaline exposure.

56. Fundamental to understanding disorders of esophageal function is the measurement of the contractility of the
esophageal body and sphincters. Which of the following statement(s) is/are true concerning esophageal manometry
in the investigation of benign esophageal disease?

a.A defective sphincter is predictive of poor long-term response to medical therapy, but a good response to surgery
b.Esophageal manometry can determine the resting pressure and the overall length of the sphincter but not its
abdominal length
c.The LES pressure normally drops to gastric baseline immediately after a swallow before the peristaltic wave
reaches the lower esophagus
d.A Vector Volume below the fifth percentile of normal is the most sensitive measure of mechanical deficiency of the
LES
e.There is no correlation between defects in LES with the severity of gastroesophageal reflux disease
Answer: a, c, d
Esophageal manometry is an investigative tool in which a catheter containing pressure sensors is inserted into the esophagus
and used to measure pressures in esophageal body and sphincters at rest and in response to swallowing. It is indicated in a
number of clinical situations including nonobstructive dysphasia, noncardiac chest pain, and the assessment of
gastroesophageal reflux disease.
The indications for manometry in patients with suspected gastroesophageal reflux are chiefly to assess the status of the LES
and to identify a motility disorder of the body. A defective sphincter is predictive of a poor long-term response to medical
therapy, but a good response to surgery. The presence of a motility defect profoundly alters the operative strategy in patients
with GERD and should always be excluded by manometry prior to operative therapy. In assessment of the LES, three
components are measured: the resting pressure, the overall length of the sphincter, and the abdominal length. A defect in the
values for each of these components are determined when the lower limits of normal (fifth percentile) are determined. A
defect in one or even two components of the LES may be compensated for by good esophageal body function, but when all
three components are defective, excessive esophageal acid exposure is inevitable. All the pressures measured along the
length of the sphincter and around its circumference during the pull-through may be treated as vectors having both
magnitude and direction and hence integrated into a three-dimensional image, the volume (Vector Volume) of which is a
measure of LES resistance. A Vector Volume below the fifth percentile of normal is the most sensitive measure of
mechanical deficiency of the LES. The prevalence of a defective LES increases with increasing severity of GERD, being the
lowest in patients without evidence of endoscopic injury and highest in patients with stricture or Barrett’s esophagus.

57. Which of the following statement(s) is/are true concerning the diagnosis and management of the patient whose
barium esophogram is shown in Figure 18-29?

a.The condition is due to neuronal generation of the myenteric plexus in the lower esophageal sphincter
b.The patient will report symptoms of vomiting of sour or bitter material
c.Despite the impressive radiologic picture, passage of the endoscope through the area of narrowing will likely be
possible
d.Manometry and 24 hour pH monitoring should be performed for confirmation of the diagnosis
Answer: c

The x-ray demonstrates moderately advanced achalasia with a dilated esophagus with a narrowed tapering “bird’s beak”
appearance of the distal esophagus. Achalasia is the best known primary motility disorder of the esophagus. It is
characterized by failure of the esophageal body peristalsis and incomplete relaxation of the LES. It is generally thought to
be caused by neuronal degeneration of the myenteric plexus of the esophageal wall, causing aperistalsis, and to loss of
activity of the inhibitory neurons in the LES leading to incomplete relaxation. Patients with achalasia all have dysphagia,
and most have regurgitation. Careful questioning is needed to distinguish the regurgitation from vomiting. Generally, it
occurs during or at the end of a meal, and the material tastes bland rather than sour or bitter. Patients often have to leave the
table to regurgitate, and are usually slow eaters.
Endoscopy frequently reveals residual liquid or food in the esophagus. Unlike a stricture, the narrowing of the lower end
permits the passage of the endoscope, usually with a characteristic “popping” sensation. In every patient with presumed
achalasia, it is very important to view the cardia from below with the endoscope retroflexed, as a small infiltrating
gastroesophageal tumor may otherwise be missed. Manometry is required to establish the diagnosis of achalasia. The classic
features on stationary manometry are: 1) Elevated LES pressure; 2) Incomplete LES relaxation; 3) Absence of esophageal
body peristalsis; and 4) Positive intraesophageal body pressure. Although reports concerning the use of 24 hr pH monitoring
appear in the literature, excessive acid exposure is rare.

58. Which of the following statement(s) is/are true concerning other tests available for investigation of esophageal
disease?

a.A 24 hour pH monitoring is currently the principal method in making the diagnosis of gastroesophageal reflux
disease (GERD)
b.Acid reflux episodes are defined as periods when the esophageal pH is less than 2
c.Twenty-four pH monitoring is only useful in the detection of acid reflux disease
d.The Bernstein test continues to be an important tool in the diagnosis of acid reflux disease
e.Delayed gastric emptying may be an important etiologic factor in patients with GERD
Answer: a, e
The development of 24 hr pH monitoring was a major advance in unraveling the pathophysiology of GERD. It is now the
principal method to make the diagnosis of GERD and has effectively replaced all other methods of measuring esophageal
acid exposure. It is indicated in any patient with symptoms suggesting GERD, unless the symptoms are trivial, or
permanently abolished by a short course of acid suppression therapy. Reflux episodes are defined as periods when the
esophageal pH is less than pH 4. Normal (physiologic) reflux occurs in the form of short rapidly cleared postprandial
episodes. A few episodes of long duration are more injurious than many brief episodes, even though total acid exposure time
may be similar. In addition to the measurement of acid exposure, pH monitoring can also be used to detect excessive
alkaline exposure (pH > 7) in the esophagus. The Bernstein test, in which hydrochloric acid is dripped into the esophagus
via a nasogastric tube, is sometimes used to determine if a patient’s symptoms are reproduced by acidic exposure. It is
basically a measure of esophageal mucosal sensitivity. It has been largely superseded by the use of 24 hr pH monitoring.
Gastric emptying is affected by the composition and consistency of the ingested meal. Delayed gastric emptying may be an
important etiologic factor in patients with GERD and a normal LES.

59. The results for anti-reflux surgery are generally good, however, patients who have failed anti-reflux procedures
constitute a particularly challenging group. Which of the following statement(s) is/are true concerning failed anti-
reflux repairs?

a.A Slipped Nissen is usually the result of an operative technical mistake


b.Disruption of a fundoplication is more prone to occur with a Nissen fundoplication because of the use of the gastric
wall in the repair
c.Postoperative dysphagia in a patient with normal preoperative motility is usually due to a secondary motility
disorder
d.Colonic replacement, although technically challenging, usually has superior long-term results when compared to
esophageal replacement with the stomach
Answer: a, d

When patients are correctly selected and the operation performed with conformity with the basic surgical principles, long-
term relief of symptoms is achieved by more than 90% of patients. A number of patterns of failure, however, can occur. The
so-called Slipped Nissen may develop when the upper stomach rides up through the fundoplication, and causes both
dysphagia and heartburn. It is more likely that the condition was created at the time of surgery because the surgeon did not
mobilize the fundus, or because unrecognized esophageal shortening led to inadequate mobilization of the gastroesophageal
junction, causing the surgeon to wrap the stomach around the upper stomach rather than the lower esophagus. Creating too
tight a fundoplication leads to immediate postoperative dysphagia. Manometry shows a high pressure nonrelaxing sphincter
which may be difficult to distinguish from achalasia. Such patients highlight the importance of manometry in all patients
before proceeding with anti-reflux surgery. In a patient with normal preoperative motility, the cause is usually a fault in
technique, and can be prevented by constructing the fundoplication over a 60 F Bougie. Disruption of the fundoplication
that manifests clinically and physiologically by recurrent reflux can be caused by inadequate suture technique, unrecognized
esophageal shortening leading to tension on the wrap, or poor choice of operation. All partial fundoplications, such as the
Toupet procedure are more prone to disruption than a Nissen. This is because the integrity of the repair depends on sutures
to the esophageal wall and not the stomach, and because all these repairs require much more abdominal length of esophagus
than a Nissen, thus placing the repair under tension. Esophagectomy and esophageal replacement are occasionally indicated
in the treatment of advanced GERD. The indications for esophagectomy are Barrett’s esophagus with high grade dysplasia
and what is generally described “burned out esophagus” which includes failure of a third anti-reflux operation, a severe
coexistent motility disorder, or the presence of an undilatable stricture. Either colon or stomach may be used to replace the
esophagus. Colonic replacement is more difficult, requiring three anastomoses rather than one, but it has superior functional
long-term results.

60. A number of diagnostic modalities exist for investigation of structural abnormalities of the esophagus. Which of
the following statement(s) is/are true concerning the use of these investigative studies?

a.Endoscopy should be the first investigation in any patient with foregut symptoms
b.Barrett’s esophagus is suggested when the squamo-columnar junction is more than 2 cm above the
gastroesophageal junction on endoscopic examination
c.There are three areas of esophageal narrowing which can be noted on both barium esophogram and endoscopy
d.The CT appearance of the esophagus is normally a flattened, hollow structure with a thin wall
Answer: b, c, d
Endoscopy is generally the first investigation in patients with foregut symptoms. The exception is when the patient’s chief
complaint is dysphagia, when a “road map” should first be obtained by a barium swallow. The locations of the esophageal
landmarks are measured endoscopically from the incisor teeth. Three landmarks are measured in the region of the cardia:
the level of the crura, the level of the anatomic gastroesophageal junction, and the level of the squamo-columnar junction (Z
line). A hiatal hernia is present when the gastroesophageal junction is more than 2 cm above the crura. Barrett’s esophagus
is suggested when the squamo-columnar junction is more than 2 cm above the gastroesophageal junction but may be
diagnosed if any specialized epithelium is identified above the gastroesophageal junction histologically, regardless of
measured length of the columnar segment. Three areas of esophageal narrowing are frequently noted on both barium
esophogram and endoscopy. The first narrowing is at the site of the cricopharyngeus muscle. The left mainstem bronchus
and aortic arch caused narrowing of the middle third of the esophagus. The most distal narrowing of the esophagus is at the
diaphragmatic hiatus and is caused by the lower esophageal sphincter mechanism. These normal points of narrowing tend to
retard swallowed foreign objects. Also, corrosive liquid ingestion results in prominent mucosal injury at these sites as the
liquid is slowed at passage. CT scan of the esophagus is important in delineating the relationship of esophageal lesions to
adjacent structures, especially the trachea, left main bronchus and aorta. The esophagus normally appears as a flattened
hollow structure with a thin wall. A more circular cross-sectional appearance with a fluid level is evidence of distal
obstruction.

61. Which of the following patient scenarios would be best managed with anti-reflux surgery?

a.A patient with heartburn but normal 24 hour pH monitoring and an intact lower esophageal sphincter
b.A patient with primarily respiratory manifestations of gastroesophageal reflux
c.A patient with increased acid exposure and a mechanically defective sphincter who responds well to medical
therapy but requires continued long-term medication for continued relief
d.A patient with gastroesophageal reflux but excessive complaints of epigastric pain, nausea, vomiting, and loss of
appetite
Answer: b, c

The first requirement for consideration of anti-reflux surgery is the objective demonstration of the presence of GERD by 24-
hour pH monitoring. Secondly, the patient must have either symptoms or complications of the disease. Thirdly, the disease
should be caused by defect appropriate to surgical therapy, i.e., a mechanically defective sphincter. Some patients with
increased acid exposure and a mechanically defective sphincter, and who have no complications of the disease respond well
to medical therapy, but they require long-term medication for continued relief. These patients should be given the option of
surgery as a cost effective alternative.
Atypical symptoms of reflux such as respiratory manifestations often respond well to anti-reflux surgery. When respiratory
symptoms are combined with typical symptoms such as heartburn and regurgitation, the results of anti-reflux surgery are
generally good.
Complaints of epigastric pain, nausea, vomiting, and loss of appetite may be due to excessive duodenogastric reflux which
occurs in about 11% of patients with gastroesophageal reflux disease. This problem is usually, but not invariably, confined
to patients who have previous upper gastrointestinal surgery. The coexistence of these gastric symptoms in a patient who
also has typical symptoms of GERD should prompt a thorough evaluation of the stomach using a bile probe, 24 hour pH
monitoring or radionucleotide scanning. In such patients, the correction of only the incompetent cardia can result in a
disgruntled individual who continues to complain of nausea and epigastric pain on eating.

62. Which of the following statement(s) concerning pharyngoesophageal disorders is/are true?

a.In neuromuscular diseases, dysphagia is often worse for liquids than for solids
b.Cricomyotomy may be indicated for a wide variety of neuromuscular disorders involving the pharyngoesophageal
phase of swallowing
c.Excision of a Zenker’s diverticulum is indicated to prevent malignant change in the sac
d.Complications of all operations on the cervical esophagus include hematoma formation and recurrent nerve
paralysis
Answer: a, b, d

Disorders of the pharyngoesophageal phase of swallowing result from a discoordination of the neuromuscular events
involved in chewing, initiation of swallowing, and propulsion of the material from the oropharynx to the cervical
esophagus. The commonest causes of pharyngoesophageal dysphagia are neuromuscular diseases. The most important are
cerebrovascular disease, myasthenia gravis, Parkinson’s disease, multiple sclerosis and muscular diseases such as myotonic
dystrophy and polymyositis. In neuromuscular diseases, dysphagia is often worse for liquids than for solids. Choking,
repetitive pneumonia, nasal regurgitation and hoarseness are also prominent features. The surgeon’s role in the treatment of
cricopharyngeal disorders is to reduce outflow resistance by performing a cricomyotomy. Initially this was recommended
only for patients with demonstrable failure of the upper esophageal sphincter relaxation. More recently, a number of reports
indicate a wide variety of neuromuscular diseases that may be improved by cricomyotomy. The surgical options in Zenker’s
diverticulum are either excision or suspension. Excision is sometimes recommended on the grounds that malignant change
in the sac is prevented, but there is no evidence that excision carries any greater protective role than suspension, which
effectively prevents stagnation of food material, thus removing the presumed cause of malignant change. Suspension also
removes the risk of contamination of the operative site, the risk of subsequent breakdown of the closure site with fistula
formation, and the risk of narrowing of the esophagus. In either case, recurrence is likely if cricomyotomy is not performed,
because the underlying defect which predisposes to the diverticulum persists. All operations on the cervical esophagus carry
the risk of hematoma formation and recurrent nerve paralysis. The venous pumping action of the lung can cause the
development of a large hematoma in the mediastinum postoperatively, therefore meticulous hemostasis is critical for the
performance of this operation.

63. Barrett’s esophagus is a complication of gastroesophageal reflux disease. Which of the following statement(s)
is/are true concerning this condition?

a.The histologic hallmark is the presence of “specialized” columnar epithelium regardless of how far it extends into
the esophagus
b.Barrett’s epithelium will frequently regress with medical therapy or anti-reflux surgery
c.High grade dysplasia will frequently be associated with foci of invasive carcinoma
d.Patients with adenocarcinoma arising in Barrett’s esophagus have a high incidence of p53 gene mutations
Answer: a, c, d

Barrett’s esophagus is now recognized as a complication of advanced gastroesophageal reflux disease. The histologic
hallmark of Barrett’s esophagus is the presence of “specialized” columnar epithelium, which shows features of intestinal
metaplasia, easily recognized by the presence of goblet cells. The presence of specialized epithelium is now regarded as the
pathonomonic feature of Barrett’s esophagus regardless of how high it extends into the esophagus. Barrett’s esophagus may
exist on its own, or may be itself associated with ulceration, stricture, and malignant change. Once Barrett’s epithelium is
present, medical therapy or anti-reflux surgery rarely causes it to regress. Unless it is actually ablated, for example with
laser therapy, it persists. The most significant feature of Barrett’s esophagus is its malignant potential. The metaplastic
epithelium usually undergoes dysplastic change prior to becoming frankly neoplastic. High grade dysplagia is synonymous
with carcinoma in situ, and if the esophagus is removed for such a condition, up to 50% will demonstrate foci of invasive
carcinoma.
In the past, the pathophysiology of Barrett’s esophagus was associated with alkaline reflux on esophageal pH monitoring.
However, more recently using a bile sensor for monitoring bilirubin, this condition is frequently associated with excessive
bile in the esophagus. Repetitive injury from noxious gastric juice can lead during the repair process to mutations in the p53
gene. Patients with adenocarcinoma arising in Barrett’s esophagus have a high incidence of p53 mutations.

64. Which of the following statement(s) is/are true concerning the blood supply and lymphatic drainage of the
esophagus?

a.The thoracic esophagus receives no direct branches from the aorta therefore allowing the technique of transhiatal
(blunt) esophagectomy
b.Bleeding esophageal varices are most prominent in the mid-esophagus
c.Lymphatic drainage of the lower third of the esophagus goes entirely to the abdominal lymphatic system
d.Nodal involvement in esophageal cancer is quite common even if the tumor is limited to the level of the submucosa
Answer: d

The blood supply and venous drainage of the esophagus are largely segmental. The inferior thyroid artery provides the main
blood supply to the cervical portion of the esophagus. The thoracic portion of the esophagus receives its blood supply from
two sources; branches from two or three bronchial arteries provide the proximal arterial supply and branches directly from
the aorta supply the more distal thoracic esophagus. Intrathoracic mobilization of the esophagus during performance of anti-
reflux procedures often require ligation of these branches. The venous plexus in the submucosa collects capillary blood and
delivers it into a periesophageal venous plexus. The left gastric vein or coronary vein provides the principal collateral in
portal hypertension when esophageal varices develop. The submucosal veins become much more superficial in the most
distal esophagus, 1–2 cm above the gastroesophageal junction, and are consequently the most common site of bleeding in
portal hypertension.
The lymphatics of the esophagus form a rich submucosal network draining into regional lymph nodes in the periesophageal
connective tissue. There is thus little barrier to longitudinal spread of cancer in the esophagus. Lymphatic drainage from the
upper two-thirds of the esophagus is usually cephalad, but drainage from the lower one-third is in both directions. Although
lymphatic metastasis in the esophagus generally involve the regional lymph nodes in proximity, nodal involvement may
occur several centimeters away from the primary lesion because of the rich intramural lymphatic anastomotic channels.
When a carcinoma is limited to the mucosa, the incidence of lymphatic metastases is low, but once into the submucosa, the
incidence rises to 60%.

65. Which of the following statement(s) is/are true concerning the process of swallowing and esophageal transit of
food?

a.Injury to the recurrent laryngeal nerves can cause motility problems of the cervical esophagus and resulting
aspiration
b.Esophageal reflux does not lead to impaired esophageal motility
c.Relaxation of the LES is mediated via inhibitory neurons
d.The overall length of the LES is the only factor influencing the pressure gradient of the sphincter
e.A mechanically defective sphincter is always associated with increased esophageal acid exposure
Answer: a, c

The cricopharyngeus muscle is a continuation of the inferior constrictor of the pharynx and receives its innervation via both
the right and left recurrent laryngeal nerves. Although much attention is given to vocal cord dysfunction that accompanies
recurrent laryngeal nerve damage, it is clear that cricopharyngeal sphincter dysfunction and motility problems of the
cervical esophagus can occur with injury to these nerves. Serious aspiration following recurrent nerve injury is caused not
only by the cricopharyngeal dysfunction, but also by additional morbidity incurred because of the inability to close the
glottis during swallowing and loss of the protection afforded by effective coughing. Clinically, peristaltic defects of the
esophageal body fall into one of to broad categories. One category is characterized by a defect in organization of peristaltic
waves, and is primarily a neural phenomenon. The other notable defect is reduction of the power (amplitude) of peristalsis
and is usually due to muscle damage secondary to severe reflux or replacement with fibrous tissue as happens in
scleroderma and other connective tissue diseases or with severe reflux. The LES provides a pressure barrier between the
esophagus and stomach. The sphincter normally remains actively closed to prevent reflux of gastric contents into the
esophagus. Relaxation of the LES is mediated by inhibitory neurons. It occurs either to allow entry of food, or to allow exit
of air during belching. The ability of the LES to remain closed in the face of a pressure gradient tending to promote reflux of
gastric contents from the positive pressure environment of the stomach into the negative pressure environment of the chest
depends on several features. The most significant is the resting pressure. However, of equal importance is the ability of the
LES to respond to variations in intra-abdominal pressure associated with daily activities. Such elevations would normally be
transmitted to the sphincter, causing it to collapse and remain closed, provided sufficient length of the sphincter remains
exposed to the abdominal pressure and the compressive effect of the crura. The abdominal length is often reduced in hiatal
herniation, because of attenuation of the pharyngoesophageal membrane. The overall length of the LES is also an important
determinant of competence, much as the total resistance of a series of resisters in a circuit is the sum of the individual
resistances. A mechanically-defective sphincter, however, is not always associated with increased esophageal acid exposure
because it may be compensated by the clearance function of the esophageal body. The role of the esophageal body in
limiting acid reflux is related to its ability to clear the esophagus of acid. This clearance has two components: volume
clearance which requires peristalsis, and chemical clearance which requires saliva.

66. Which of the following statement(s) is/are true concerning the management of this patient?

a.The risk of perforation of the esophagus associated with balloon dilatation may be as high as 10%
b.An anti-reflux procedure should be universally performed for any operative myotomy
c.Successful relief of dysphagia can be achieved in up to 90% of patients with a single pneumatic dilatation
d.Thoracoscopic myotomy is associated with significantly poorer results than the open procedure
e.Prospective randomized studies and retrospective data appear to support a surgical approach for achalasia
Answer: a, e

The mainstay of treatment in achalasia is either balloon dilatation or surgery. Balloon dilatation has an advantage that it can
be performed as an outpatient and has minimal recovery time. It is less likely to be effective than surgical treatment, and
frequently needs to be repeated. The risk of perforation of the lower esophagus is higher with this procedure than with any
other form of esophageal instrumentation and varies from 2–10%. The risk of gastroesophageal reflux following dilatation
is not known, but symptomatically the risk appears to be low.
All surgical procedures employ a variant of Heller’s myotomy, in which the circular muscle of the lower esophagus is
divided. In the United States, most myotomies are carried out through the chest, but the abdominal approach is favored in
Europe. Regardless of the route chosen, four key principles are important, namely: 1) adequate myotomy, 2) minimal hiatal
disturbance, 3) anti-reflux protection without creation of obstruction, and 4) prevention of rehealing. The advent of
minimally invasive surgery has led to the development of thorascopic and laparoscopic myotomy, and these are now being
extensively performed with comparable results to open surgery. There is broad agreement that if a myotomy is performed
through the abdomen, an anti-reflux procedure should be added, and that a full Nissen wrap, however floppy, leads to long-
term failure. When approached through the chest, there is controversy about the need for an anti-reflux procedure, as it is
claimed that less hiatal disturbance and more limited myotomy is possible by this route. Thoracoscopic myotomy, with
enhanced view, enables a more precise determination of the distal myotomy and therefore may not require a anti-reflux
procedure.
A single pneumatic dilatation achieves adequate relief of dysphasia and pharyngeal regurgitation in about 60% of patients.
Repetitive dilatations increase this figure to about 70%. Only one controlled randomized study comparing the two modes of
therapy has ever been performed. The results of this study as well as a number of retrospective studies would appear to
support operative myotomy as the initial treatment of choice.

67. Which of the following statement(s) is/are true concerning the surgical anatomy of the esophagus?

a.Surgical exposure of the cervical esophagus is best gained via the right neck
b.Spontaneous esophageal perforation tends to be associated with leakage into the left chest
c.Access to the entire thoracic esophagus can be obtained only via the left chest
d.The lower esophageal sphincter can be recognized distinctly by inspection of the gastroesophageal junction
Answer: b

A detailed knowledge of the relations of the esophagus is essential for the surgeon to be able to identify the site and
significance of lesions seen by indirect studies such as endoscopy, contrast radiography, and CT scanning, as well as the safe
performance of surgical procedures. The cervical esophagus is about 5 cm long. It begins at the level of C6 and extends to
the lower border of T1, curving slightly to the left in its descent. Consequently, although the surgical approach to this
portion of the esophagus may be from either side of the neck through an incision along the anterior border of the
sternocleidomastoid muscle, the left side is chosen if possible. Above the level of the tracheal bifurcation, the esophagus
moves to the right of the descending aorta. It then moves to the left, passes behind the tracheal bifurcation and the left main
bronchus and descends to the diaphragm. In the lower third, the esophagus courses anteriorly and to the left to pass through
the diaphragmatic hiatus. The lower esophagus is covered only by a flimsy mediastinal pleura on the left, and it is this
portion which is most commonly the site of spontaneous perforation in Boerhaave’s syndrome. In general, the lower
esophagus is most easily approached through the left chest, but access to the supra-aortic esophagus is restricted. Thus, a
left thoracotomy is most useful for performing procedures involving the lower esophagus. However, access to the entire
thoracic esophagus can be obtained only from the right chest. This incision, however, limits access to intraabdominal organs
by the position of the liver and therefore normally requires a separate upper abdominal incision. The abdominal esophagus
begins as the esophagus enters the abdomen through the diaphragmatic hiatus. It is surrounded by a fibroelastic membrane,
the phrenoesophageal ligament which arises from the subdiaphragmatic fascia. The lower limit of the pharyngoesophageal
membrane anteriorly is marked by a prominent fat pad, which corresponds to the gastroesophageal junction. The lower
esophageal sphincter (LES) is a zone of high pressure 3–5 cm long at the lower end of the esophagus. Although it does not
correspond to any macroscopic anatomical structure, its function appears to be related to the microscopic architecture of the
muscle fibers.

68. Which of the following statement(s) is/are correct concerning the patient whose barium esophogram is shown
below?

a.The patient’s complaint would be primarily chest pain and to a lesser degree dysphagia
b.The pathognomic feature of manometry is the presence of prolonged high amplitude waves
c.The patient will likely experience nutritional problems
d.The first line of treatment for this patient is surgical myotomy
Answer: a, b
The barium esophogram of these two patients shows diffuse esophageal spasm resulting in a cork screw esophagus with
multiple contractions. See Fig. 18-35. These primary motor disorders are characterized by substernal chest pain. In the
nutcracker variety, as demonstrated in this x-ray, the pain is central crushing pain with no relation to food ingestion and
differs from angina in that it more frequently comes on at rest. Dysphagia or classic heartburn may be present but tend to be
overshadowed by the chest pain. Barium radiography and endoscopy are generally not helpful. The pathognomic feature of
manometry is the presence of prolonged high amplitude waves, with a peak greater than 180 mm Hg. Diffuse esophageal
spasm and nutcracker esophagus are benign conditions which rarely cause nutritional problems and do not lead to life-
threatening complications. For this reason, symptom control is the only significant goal of treatment. Medical treatment for
diffuse esophageal spasm and nutcracker esophagus is focused on abolishing strong simultaneous contractions and generally
employs calcium channel blocking agents or long-acting nitrates. Surgery for these conditions are generally only considered
when medical treatment is ineffective.

69. Which of the following statement(s) is/are true concerning tracheoesophageal fistulas?

a.The majority of acquired tracheoesophageal fistulas are due to malignant disease


b.A water-soluble contrast esophogram should be obtained for diagnosis
c.Malignant tracheoesophageal fistulas represent one of the few indications for an endoesophageal prosthesis
d.A benign tracheoesophageal fistula from an endotracheal intubation injury often requires a thoracotomy for repair
Answer: a, c

Ninety percent of acquired fistulas between the esophagus and tracheobronchial tree in adults are the result of malignant
disease. Tracheoesophageal fistulas complicate the course of disease in about 5% of patients who have esophageal
carcinoma. Nearly 80% of patients with malignant tracheoesophageal fistulas die within three months of the onset of
symptoms and in 85% of these patients, the cause of death is aspiration pneumonia, not distant metastatic disease. For the
most part, malignant tracheoesophageal fistula represents incurable disease for which resection carries significant mortality
and is seldom indicated. Palliative relief of recurrent aspiration is the aim of therapy. Effective occlusion of the fistula may
be achieved by insertion of one of a variety of available endoesophageal endoprostheses. These tubes are placed into the
esophagus with the aid of an esophagoscope and may occlude the esophageal side of the fistula sufficiently to allow
swallowing of liquids without aspiration into the tracheobronchial tree. More recently, expandable metal stents have been
used successfully in the treatment of malignant tracheoesophageal fistulas.
Nonmalignant fistulas result from the erosion by contiguous infected subcarinal mediastinal lymph nodes; trauma; late
sequelae of chronic mid-esophageal traction diverticulum; or erosion by an endotracheal or tracheostomy tube cuff in a
patient requiring prolonged ventilatory support. Small fistulas, such as resulting from an endotracheal intubation injury, are
approached through a cervical collar or oblique incision anterior to the sternocleidomastoid muscle. Although such cuff
injuries usually produce circumferential tracheal damage which necessitates a tracheal resection, this can also be performed
through a cervical collar incision.

70. Esophageal cysts arise as outpouchings of the embryonic foregut. Which of the following statement(s) is/are true
concerning esophageal cysts?

a.The cyst lining will be lined only by stratified squamous epithelium


b.Most esophageal cysts cause symptoms in the first year of life
c.An asymptomatic esophageal cyst can be managed conservatively
d.The diagnosis of an esophageal cyst is usually made radiographically
Answer: b, d

Embryologically, the esophagus is lined by simple columnar ciliated epithelium, which is eventually replaced by stratified
squamous epithelium. The esophageal cyst may therefore contain both of these types of epithelium as well as fat and smooth
muscle. The esophageal duplication cyst is a variation of the foregut cyst, extends along the length of the thoracic
esophagus, and is lined by squamous epithelium. More than 60% of esophageal cysts cause either respiratory or esophageal
symptoms in the first year of life. Those located in the upper third of the esophagus tend to present in infancy, while the
lower-third cyst may be asymptomatic initially and present later in childhood. Adults present with dysphasia, choking,
retrosternal pain when previously asymptomatic cysts enlarge as the result of bleeding or infection. The diagnosis of an
esophageal cyst can usually be made on the basis of atypical radiographic appearance. The PA and lateral chest x-ray,
barium esophogram, and in some cases a CT scan, will confirm the diagnosis in almost all patients. Because esophageal
cysts have a predilection for bleeding, ulceration, perforation, and infection, excision is generally recommended. This can
generally be achieved with low morbidity by an extramucosal resection.
71. Which of the following statement(s) is/are true concerning infectious esophagitis?

a.Candida albicans is not normally found in the mouth but results from the overgrowth of this fungus in patients on
broad spectrum antibiotics
b.Candida esophagitis is usually self-limited and is seldom associated with chronic problems
c.Systemic therapy is seldom indicated
d.Small ulcers on barium esophogram in a transplant patient complaining of dysphagia and odynophagia are likely
due to herpes simplex viral infection
Answer: d

Chronic debilitation, immunosuppression, and prolonged use of antibiotics predisposes the development of infectious
esophagitis with candida albicans being the most common cause. Candida albicans is a fungus that normally is a commensal
inhabitant of the mouth, oral pharynx, and GI tract. This fungus may become pathogenic in patients who are severely
debilitated or immunosuppressed. In recent years, the use of broad spectrum antibiotics, immunosuppression in organ
transplant patients, and the wide use of chemotherapeutic agents have resulted in an increased number of patients with
monilial esophagitis. As the disease progresses, transmural invasion of the esophageal wall occurs. Although the esophagitis
can be controlled with antifungal therapy, if the patient survives the underlying illness, chronic stricture formation may
result after healing. Minimally compromised patients with mild monilial esophagitis should receive oral nystatin suspension
as a primary treatment. More immunosuppressed patients or those with severe cases warrant high doses of fluconazole and
ketoconazole. Intravenous fluconazole or amphotericin B are utilized in granulocytopenic patients.
Viral esophagitis is the second most common cause of infectious esophagitis. Herpes simplex viral infection is the most
common infection in the immunosuppressed transplant patient. Characteristically, viral esophagitis produces mucosal
ulceration with patients presenting with dysphasia and odynophagia. The esophageal ulcers are characteristically small (<
1.5 cm). The diagnosis is established endoscopically by biopsy, brushings, and washings for cytology, histology, and viral
culture. The infection usually responds well to treatment with acyclovir.

72. Which of the following statement(s) is/are true concerning the pathology of squamous cell carcinoma of the
esophagus?

a. Carcinoma in situ will gradually progress to invasive squamous cell carcinoma over a period of two to four years
b. The most common location for squamous cell carcinoma of the esophagus is the upper and mid-thoracic segment
c. Esophageal carcinoma tends to be multifocal
d. Macroscopically, ulcerative lesions with extensive infiltration of the adjacent esophageal wall are most common
e. Lymph node metastases are present in at least 75% of patients at the time of initial diagnosis
Answer: a, b, d, e

Pathologically, esophageal carcinoma occurs over a spectrum that ranges from the early lesion (carcinoma in situ), which is
limited to the mucosa, to the more advanced form, in which the tumor penetrates the muscle layers of the esophagus and
beyond. Carcinoma in situ typically is found in patients between 40 and 50 years of age and gradually progresses to invasive
squamous cell carcinoma over two to four years. Using the arbitrary division of the esophagus, 8% of squamous cell
carcinomas occur in the cervical esophagus, 55% in the upper and mid-thoracic segments, and 37% in the lower thoracic
segment which extends to the GE junction. Macroscopically, 60% of squamous cell carcinomas of the esophagus are
fungating intraluminal growths, 25% of ulcerative lesions are associated with extensive infiltration of the adjacent
esophageal wall, and 15% are infiltrating. Esophageal carcinoma tends to be multi-focal, and a patient who survives
treatment of one carcinoma has at least twice the risk of developing a second primary esophageal neoplasm than the normal
population.
Esophageal carcinoma is notorious for its aggressive biologic behavior. Mediastinal, supraclavicular, or celiac lymph node
metastases are present in at least 75% of patients with esophageal cancer at the time of initial diagnosis. Unfortunately,
when lymph node metastases are present, five-year survival is only 3%, compared with 42% when there is no lymph node
spread.

73. A 54-year-old woman experiences pain in both the anterior and posterior left chest and the epigastrium following
balloon dilatation performed for achalasia. Which of the following statement(s) is/are true concerning this patient’s
diagnosis and management?
a.A normal chest x-ray will rule out an esophageal perforation
b.Barium should never be used in performance of a contrast study with a diagnosis of esophageal perforation
c.Conservative, nonoperative treatment may be indicated
d.If surgical repair is necessary, the patient should undergo esophagomyotomy and a partial gastric fundoplication
Answer: c, d

It is axiomatic that pain or fever after esophageal instrumentation or operation is indicative of an esophageal perforation
until proven otherwise. It is an indication for immediate contrast esophogram. Because the morbidity and mortality rates
associated with esophageal perforation are directly related to the time interval between diagnosis of the injury and its repair
or drainage, an aggressive attitude toward diagnosing the perforation must be adopted. When the diagnosis is considered, a
water-soluble contrast agent should be administered. If this study is negative, dilute barium should be administered. Barium
is relatively inert, and the fear of barium extravasating in the mediastinum through the site of injury and producing a severe
reactive mediastinitis is unfounded. A chest x-ray may help confirm the diagnosis by demonstrating air in the soft tissues of
the neck or mediastinum or a hyrdo-or pneumothorax. A normal chest x-ray, however, does not rule out an esophageal
perforation.
Although most esophageal perforations require operative intervention, selected patients may be managed nonoperatively
with cessation of oral intake, administration of antibiotics, and intravenous hydration until the disruption heals or the small
contained cavity begins to decrease in size. The usual clinical settings for such perforations that are encountered are cervical
esophageal tears caused by esophagoscopy; intramural dissection that has occurred during dilatation of a stricture or
pneumatic dilatation for achalasia, and an asymptomatic esophageal anastomotic disruption discovered on a routine
postoperative contrast study. Perforations complicating pneumatic dilatation for achalasia occur in 4% to 6% of patients,
and most are small and well managed medically with antibiotics and intravenous hyperalimentation. If operation is required
for suture repair of the perforation, an esophagomyotomy to relieve the distal obstruction, and a partial fundoplication to
buttress the tear should be performed if possible.

74. Which of the following statement(s) is/are correct concerning the diagnostic studies for esophageal carcinoma?

a.A chest and upper abdominal CT scan is useful for both staging and predicting resectability
b.A barium swallow is an unnecessary test in a patient with dysphagia
c.Bronchoscopy should be performed in all patients with carcinoma of the upper and middle thirds of the esophagus
d.Bone and brain scans should be obtained routinely to rule out distant metastasis
e.Endoscopic ultrasound is a potentially sensitive examination for the staging of esophageal cancer
Answer: c, e

A barium swallow examination is the first study that should be obtained in a patient who complains of dysphagia. Tumors of
the cervical esophagus are difficult to identify by barium swallow examination and carcinoma of the cardia may be confused
with achalasia or esophageal spasm. Nevertheless, the barium swallow examination localizes obvious esophageal pathology
in preparation for subsequent esophagoscopy and allows the endoscopist to predict the level at which the tumor is located
and the area which requires the most careful examination. The chest and upper abdominal CT scan is now the standard
radiographic technique for staging esophageal carcinoma. Esophageal wall thickness, regional adenopathy or pulmonary,
liver, adrenal or distant nodal metastasis can be identified. Although CT is suggested to have a role in evaluating
resectability of esophageal carcinoma, it is particularly limited in its ability to detect invasion of the gastric cardia or aortic
invasion. Bone scan is not warranted unless the patient has specific complaints suggesting that bone metastases exists.
Similarly, routine brain scans are not indicated as brain metastases from carcinoma of the esophagus are uncommon.
Bronchoscopy should be performed in patients with carcinoma of the upper and middle thirds of the esophagus to exclude
invasion of the posterior membranous trachea or mainstem bronchi, which precludes a safe esophagectomy. Endoscopic
ultrasound is being used with increasing frequency as an adjunct to the standard radiologic and endoscopic assessment of
esophageal disease. It offers the potential for more sensitive staging of esophageal carcinoma by detecting the depth of
invasion and the presence of abnormal mediastinal adenopathy.

75. Which of the following conditions are associated with the development of esophageal carcinoma?

a.Caustic esophageal stricture


b.Achalasia of the esophagus
c.Plummer-Vinson syndrome
d.Esophageal diverticula
Answer: a, b, c, d
Chronic irritation of the esophageal mucosa by a variety of noxious stimuli (alcohol, tobacco, hot foods and liquids)
eventually may lead to the development of esophageal carcinoma. A variety of other esophageal lesions have a recognized
premalignant nature. The patient who survives the initial injury long enough to develop a caustic esophageal stricture has a
1000-fold increased risk of developing carcinoma compared with the normal population. Ten to 12% of patients with
achalasia of the esophagus who are observed 15 years or more develop esophageal carcinoma. This is thought to be related
to the irritating effects of the fermenting intraesophageal contents on the adjacent esophageal mucosa. Plummer-Vinson
syndrome is a premalignant esophageal condition. Patients with this syndrome are typically elderly women who have
cervical dysphasia and iron deficiency anemia. About 10% of patients will develop squamous cell carcinoma of the
hypopharynx, oral cavity or esophagus. Finally, there have been isolated reports of esophageal carcinomas found
incidentally within esophageal diverticula, presumably as the result of the irritating effects on the mucosa of stagnant,
putrefying food within the pouch. Esophageal diverticula are therefore regarded as premalignant esophageal lesions
although this occurrence is extremely rare.

76. Which of the following statement(s) is/are correct concerning the options for resection of esophageal carcinoma?

a. The development of reflux esophagitis seldom occurs following intrathoracic resection due to the limited life
expectancy of these patients
b. Transhiatal esophagectomy, although conceptually sound, is not technically possible in most patients with
esophageal carcinoma
c. Transhiatal resection, although less morbid, has unfavorable survival statistics compared to transthoracic resection
d. Radical transthoracic esophagectomy with en bloc dissection of continuous lymph node bearing tissues has not
been shown to improve survival over transhiatal esophagectomy
Answer: d

For most patients with localized esophageal carcinoma, resection provides the most effective and reliable palliation of
dysphagia. The rational surgical approach to distal esophageal carcinoma has been a left thoracoabdominal incision. Tumors
involving the mid-esophagus have been resected either through a thoracoabdominal or separate thoracic and abdominal
incision, and a high thoracic esophagogastric anastomosis is performed. The major disadvantages of this technique are the
necessity of a thoracotomy in debilitated patients with esophageal obstruction as well as the disastrous complications
following intrathoracic esophageal anastomotic leak. Although recent results have shown improved operative mortality
rates, the operation can still be associated with significant morbidity and mortality. A further disadvantage of the standard
intrathoracic esophagogastric anastomosis is inadequate long-term relief of dysphagia either due to tumor recurrence at the
anastomotic suture line or due to the development of reflux esophagitis above the anastomosis. Although it has been long
taught that the patient with esophageal carcinoma does not live long enough to develop reflux esophagitis after a low
intrathoracic esophagogastric anastomosis, this is clearly not the case, and the development of reflux in these patients can
produce not only severe pyrosis and reflux symptoms, but also dysphagia from benign stenosis.
During the last two decades, the technique of transhiatal esophagectomy without thoracotomy has been popularized as an
operation that minimizes the factors responsible for poor results from traditional transthoracic esophageal resection and
reconstruction. In experienced hands, transhiatal esophagectomy without thoracotomy is possible in over 90% of patients.
The survival data is comparable to those obtained in most series of transthoracic resection with results usually
demonstrating decreased postoperative morbidity and mortality. Although conceptually radical transthoracic esophagectomy
with en bloc dissection of contiguous lymph node bearing tissues would appear to offer a better “cancer operation” than
transhiatal esophagectomy with no formal lymph node dissection, current survival results are not statistically different.
These data suggest that survival after resection for esophageal carcinoma is more a function of the extent and stage of the
tumor rather than the size of the specimen or the number of lymph nodes removed.

77. The incidence of adenocarcinoma of the esophagus is increasing at a very rapid rate, which is largely the result of
the growing prevalence of adenocarcinoma arising in Barrett’s mucosa. Which of the following statement(s) is/are true
concerning adenocarcinoma of the esophagus?

a. Barrett’s mucosa with specialized columnar epithelium characterized by veliform folds, lined by secreting
columnar and goblet cells has the highest association with carcinoma of the esophagus
b. Less than 5% of patients with Barrett’s mucosa will harbor adenocarcinoma
c. Severe dysplasia of Barrett’s mucosa requires frequent reexamination and biopsy
d. Adenocarcinoma of the esophagus has a less aggressive behavior than squamous cell carcinoma
Answer: a
It is estimated that patients with Barrett’s esophagus are 40 times more likely to develop adenocarcinoma than the general
population. The true incidence of Barrett’s esophagus in the general population is unknown, but it is estimated that
adenocarcinoma arises in up to 8% to 15% of patients with columnar epithelium lined esophagus. Of the three characteristic
histologic patterns for Barrett’s mucosa, the specialized or intestinal type of metaplasia which is characterized by veliform
folds lined by a single layer of glycoprotein secreting columnar cell and mucous-secreting goblet cells has the highest
association with carcinoma. Dysplasia occurs to varying degrees in Barrett’s mucosa and is clearly a premalignant lesion.
Severe dysplasia is almost always associated with carcinoma in situ and mandates aggressive therapy. As is true of
squamous cell carcinomas, esophageal adenocarcinoma has an aggressive biologic behavior that is characterized by frequent
transmural invasion and lymphatic spread.

78. Which of the following statement(s) is/are true concerning esophageal diverticula?

a. A Zenker’s diverticulum characteristically occurs in older patients


b. Mediastinal granulomatous disease usually results in a mid-esophageal traction diverticulum which is usually
asymptomatic
c. An epiphrenic diverticulum that presents to the right of the esophagus should be managed via left thoracotomy
d. Minimally symptomatic epiphrenic diverticula should not be operated upon
Answer: b, c, d

An esophageal diverticulum is an epithelial-lined mucosal pouch that protrudes from the esophageal lumen. Most
esophageal diverticula are acquired, and occur predominantly in adults. The pharyngoesophageal (Zenker’s diverticulum) is
the most common esophageal diverticulum and typically occurs in patients between 30 and 50 years of age. Mediastinal
granulomatous disease (e.g., tuberculosis or histoplasmosis) is the most common cause of mid-esophageal traction
diverticulum. This type of diverticulum is much smaller than the pulsion diverticulum and has a characteristic blunt tapered
tip that points toward the adjacent subcarinal or peribronchial lymph nodes to which it adheres. It is typically diagnosed as
an incidental finding on a barium esophogram and is almost always asymptomatic. No specific treatment is indicated.
An epiphrenic or supradiaphragmatic diverticulum occurs within the distal 10 cm of the thoracic esophagus as a pulsion
diverticulum that arises because of abnormally elevated intraluminal esophageal pressure. Although many patients are
asymptomatic at the time of diagnosis on barium esophogram, others have symptoms from the frequently associated
esophageal conditions: hiatal hernia, diffuse esophageal spasm, achalasia, reflux esophagitis and carcinoma. Pouches
smaller than 3 cm and causing little or no symptoms require no treatment. Severe dysphagia, chest pain, or an anatomically
dependent or enlarging pouch are indications for repair. The surgical approach to an epiphrenic diverticula is through a left
6th or 7th interspace posterolateral thoracotomy. This is the case even for diverticula that present to the right of the
esophagus.

79. Which of the following statement(s) is/are true concerning caustic injury to the esophagus?

a. Alkaline injury is more destructive than acid injury


b. Acid ingestion is not injurious to the stomach due to its normal acidic pH
c. Ingested caustic agents rapidly pass through the esophagus and stomach into the small intestine
d. Unless perforation occurs, clinical manifestations resolve quickly with initial clinical improvement noted
e. Children are less likely to form a late esophageal stricture than adults
Answer: a, d, e

Caustic injury occurs in two broad categories of patients, children younger than 5 years of age who accidentally swallow
these agents, and adults who are attempting suicide. The most common agents responsible for caustic esophageal injuries
are alkalis, acids, bleach and detergents. Ingestion of detergents and bleaches virtually always cause only mild esophageal
irritation which heals without significant adverse sequelae. Acids and alkalis may have devastating effects which range from
acute multi-organ necrosis and perforation to chronic esophageal and gastric strictures. Alkalis are more destructive,
producing liquefaction and necrosis which almost insures deep penetration whereas acids usually cause coagulation necrosis
that, in part, limits the depth of injury. Liquid alkali preparations have prolonged contact with the mucosa of the esophagus
and stomach due to its high viscosity. In addition, ingested acids typically pass through the esophagus quickly, producing
major gastric injury with relative sparing of the esophagus. In response to either ingested acid or alkali, reflex pyloric spasm
occurs, with resultant pooling of these agents in the gastric antrum.
The clinical manifestations of caustic ingestion are directly related to the amount and character of the agent ingested. When
esophageal or gastric perforation results from caustic ingestion, patients demonstrate progressive severe sepsis and
hypovolemic shock until appropriate resuscitative measures are instituted. In the absence of gastric or esophageal
perforation, the acute clinical manifestations typically resolve within days, with clinical improvement lasting for weeks.
After this, symptoms due to either esophageal or gastric stricture begin to form. Most adults who ingest liquid alkali will
develop severe esophageal and usually gastric injury that results in stricture formation. Children, with usually more limited
exposure from accidental ingestions, are less likely to have severe injuries.

80. Which of the following statement(s) is/are true concerning nonresectional therapy for esophageal carcinoma?

a. Radiation therapy can be associated with five-year survival rates equal to surgery
b. Esophageal intubation to provide palliation for esophageal cancer is associated with minimal morbidity and
mortality
c. Endoscopic laser fulguration is successful in up to 75% of patients
d. There is little or no role for surgical bypass for unresectable esophageal carcinoma
Answer: c, d

Therapy of esophageal carcinoma is influenced by the knowledge that in most of these patients, local tumor invasion or
distant metastatic disease preclude cure. While squamous cell carcinoma is generally regarded as a radiosensitive and
therefore potentially curable tumor, radiation therapy has not achieved cure in most patients. Although “curative” super
voltage radiation techniques have been employed, the average five-year survival after such treatment is between 6 and 10%
in most series. This is somewhat poorer than five-year survival rates after resection which usually range between 10 and
15%. A variety of endoesophageal tubes have been used for palliation in patients with esophageal carcinoma. Basically,
these tubes are divided into two types: the pulsion tubes, which are pushed through the tumor with the aid of an
esophagoscope; and the traction or pull-through tubes, which are pulled into place by downward traction through a
gastrostomy. As in the case with many conceptually simple procedures, implementation in the clinical setting is problematic.
Transoral esophageal intubation is associated with an overall mortality of 14% and a complication rate of 25%, the latter
due to perforation of the esophagus, migration of the tubes, or obstruction of the tubes by food or tumor overgrowth. More
recently, a variety of expandable intraesophageal metallic stents have been used to achieve palliation in patients with
unresectable esophageal carcinoma. Additional expertise with this technique is being acquired through a multi-institutional
trial currently underway in the United States. Endoscopic laser fulguration of esophageal carcinoma has been used to
achieve temporary relief of the esophageal obstruction in patients with unresectable tumors. Generally, multiple sessions are
required to resect sufficient tumor to achieve adequate lumen, but functional success with restoration of a comfortable
volume can be achieved in 75–80%. Although a variety of surgical procedures such as substernal gastric or colon bypasses
have been developed as palliative internal bypasses of unresectable esophageal carcinoma, the limited survival in these
unresectable patients can be high and the mortality rates, between 15 and 25%, do not currently justify their use.

81. Benign tumors of the esophagus are rare constituting less than 1% of esophageal neoplasms. Which of the
following statement(s) is/are true concerning benign esophageal neoplasms?

a. Most esophageal polyps are located just above the gastroesophageal junction
b. Malignant degeneration of leiomyomas of the esophagus is a frequent occurrence
c. An asymptomatic leiomyoma can be safely observed and followed with periodic barium esophograms and
endoscopic ultrasonography
d. Most leiomyomas of the esophagus require esophagectomy
Answer: c

Leiomyomas represent the most common benign intramural esophageal tumor and characteristically occur in patients
between 20 and 50 years of age. More than 80% of esophageal leiomyomas occur in the middle and lower thirds of the
esophagus. Tumors less than 5 cm in size rarely cause symptoms. When larger than this, dysphagia, retrosternal pressure
and pain are the common complaints. Bleeding more often occurs with the malignant form of the tumor, leiomyosarcoma.
Malignant degeneration of leiomyomas is exceedingly rare. An asymptomatic leiomyoma or one discovered incidentally on
a barium swallow examination can be safely observed and followed with periodic barium esophograms and endoscopic
ultrasonography. Although excision of the esophageal mass provides the only definitive tissue diagnosis, the characteristic
radiographic appearance, slow growth rate, and low risk of malignant degeneration and the ability to follow leiomyomas
with endoscopic ultrasonography justify conservative management. Tumors that are symptomatic or larger than 5 cm in
diameter should be excised. Tumors of the middle third of the esophagus are approached through a right thoracotomy, while
those in the distal third are approached through a left thoracotomy. Once the esophagus is encircled and the tumor located,
the overlying longitudinal muscle is split in the direction of its fibers. The tumor is then gently dissected away from the
contiguous underlying submucosa and adjacent muscle. When enucleation of the tumor is complete, the longitudinal
esophageal muscle is reapproximated, although a large extramucosal defect may be left without complication. Giant
leiomyomas of the cardia and adjacent stomach may require esophageal resection for their removal. When resection is
complete, leiomyomas virtually never recur.
Benign polyps of the esophagus are rare and typically arise in the cervical esophagus. Most are seen in older men and are
frequently attached to the cricoid cartilage. Histologically, they are composed of fibrovascular tissue with varying amounts
of associated fat.

82. In an effort to improve survival following esophageal resection, trials of multi-modality therapy in combination
with surgery have been completed. Which of the following statement(s) is/are true concerning such treatment?

a. Therapy appears to be indicated in squamous cell carcinoma but not adenocarcinoma


b. No residual carcinoma may be found in the resected specimen in up to 20% of patients
c. Nonrandomized trials would suggest improved survival compared to patients receiving surgery alone
d. Perioperative morbidity is increased due to preoperative radiation and chemotherapy
Answer: b, c,

Combined preoperative chemotherapy and radiation therapy before transhiatal esophagectomy for carcinoma has provided
encouraging survival statistics. The treatment consists of preoperative chemotherapy with three weeks of cisplatin,
vinblastine, and 5-fluorouracil, concurrent with 3750 to 4500 cGy of radiation therapy. Although hematologic toxicity and
radiation esophagitis are common and preoperative deaths due to bone marrow suppression can occur, there is no increase in
perioperative morbidity when compared to patients with no preoperative therapy. In one study, 24% of patients had no
residual carcinoma in the resected specimen (T0, N0 status). At mean follow-up of 36 months, the mean survival in this
series was 29 months, a clear improvement over the 12-month median survival time with transhiatal esophagectomy alone
in historical controls.

83. Which of the following statement(s) is/are correct concerning the management of a patient with a caustic
esophageal or gastric injury?

a. Corticosteroids should be administered immediately


b. Complete endoscopic examination of the esophagus and stomach should be completed
c. Patients requiring operative intervention are best explored through the abdomen
d. If organ resection is indicated, restoration of alimentary continuity should be deferred until the patient has
recovered from the acute insult
e. In patients with esophageal stricture following second and third degree burns, dilatation therapy should be
instituted as soon as possible after the injury
Answer: b, c, d

Acute caustic ingestion is indication for hospitalization. Initial management centers on stabilizing the patient and assessing
the severity of injury. Oral intake should be withheld and hypovolemia corrected with intravenous fluids. Careful
observation for evidence of airway obstruction is mandatory. Broad spectrum antibiotics are indicated once the diagnosis of
substantial esophageal injury has been established to diminish the risk of pulmonary infection from aspiration as well as
bacterial invasion through the damaged esophageal wall. Although corticosteroids have been advocated in the acute phase of
caustic ingestion to minimize subsequent stricture formation, their efficacy has not been established. Furthermore, because
steroids may mask signs of sepsis, visceral perforation, and impair healing, their use in caustic esophageal injury is
potentially deleterious and therefore is not recommended.
A contrast esophogram is the best way to make the diagnosis of esophageal perforation and should be performed if the
diagnosis is suspected at the time of admission or in subsequent followup. Esophagoscopy should be performed soon after
admission to establish whether significant esophageal injury has occurred and to permit grading of the severity of injury.
Although in the past it was taught that the endoscope should not be advanced beyond the first burned area, more recently
complete examination of the esophagus and stomach has been recommended, especially if severe burns are not detected
proximally. The use of a pediatric endoscope and adequate sedation can allow this procedure to be accomplished safely.
Patients with caustic liquid ingestion that necessitate operative intervention are generally best explored through the
abdomen. This approach permits assessment of the injury to the intraabdominal organs as well as resection of areas of full
thickness gastric necrosis. Although only the lower esophagus is well visualized through the diaphragmatic hiatus, if an
esophageal resection is required, transhiatal esophagectomy without thoracotomy is readily performed by the addition of a
cervical incision. When esophageal gastric resection for acute caustic injury is required, restoration of alimentary continuity
should be deferred until the patient has recovered from the acute insult and the development of chronic stricture formation
and retained organs can be evaluated. Esophageal stricture formation following second and third degree burns is the rule,
and dilatation therapy has been the traditional therapy for chronic caustic esophageal strictures. It is important that dilatation
not be instituted until at least six to eight weeks after the injury, when reepithelialization is complete, in order to minimize
the risk of perforation.

84. In regard to the arterial blood supply to the stomach, which of the following statement(s) is/are true?

a. The right gastric artery, a branch of the superior mesenteric artery, supplies the gastric antrum
b. Because of rich intramural collaterals, gastric viability may be preserved after ligation of all but one major artery
c. In cases of celiac artery occlusion, gastric viability is maintained collaterally via pancreaticoduodenal arcades
d. The left gastroepiploic artery is a branch of the celiac trunk
Answer: b, c

The stomach is an extremely well-vascularized organ, supplied by 5 major arterial distributions and protected from ischemia
by rich intramural and extramural collaterals. The left gastric artery and right gastric artery, derived from the celiac
distribution, supply the lesser curvature of the stomach. The right gastroepiploic artery, derived from the gastroduodenal
artery, and the left gastroepiploic artery, from the splenic artery, traverse the greater curvature. The area adjacent to the
spleen receives multiple short gastric arterial branches. In instances of celiac arterial occlusion, the superior mesenteric
artery supplies the stomach collaterally via the pancreaticoduodenal arcades which connect with the gastroduodenal artery.
The stomach may be widely mobilized for use in reconstructive procedures, for example, during trans-hiatal
esophagectomy. Advantage is taken of the abundant blood supply and collaterals of the stomach during mobilization; gastric
viability is usually preserved if one major arterial supply is preserved.

85. At a cellular level, the major stimulant(s) of acid secretion by the gastric parietal cell is/are:

a. Histamine
b. Prostaglandin E2
c. Acetylcholine
d. Gastrin
e. Norepinephrine
Answer: a, c, d

The three major stimulants of acid secretion by the parietal cell are acetylcholine, gastrin, and histamine. Acetylcholine is
released from cholinergic nerve endings in close proximity to parietal cells and binds to muscarinic receptors. Cholinergic
stimulation of parietal cells is coupled to hydrolysis of membrane-associated lipids (termed phosphatidylinositides) and
leads to increases in intracellular calcium. Histamine is released from mast cells in the lamina propria and reaches parietal
cells by diffusion. Histamine occupies H2 receptors that may be selectively blocked by agents such as cimetidine. Histamine
stimulation of parietal cell acid secretion is mediated by a cyclic AMP-dependent pathway. Gastrin is delivered to the fundic
mucosa by the systemic circulation from its source in the antrum and duodenum. Like acetylcholine, gastrin causes
increases in membrane phosphoinositol turnover and increases intracellular calcium.
Activation of parietal cells by acetylcholine, gastrin or histamine can be blocked by somatostatin. Local release of
somatostatin is physiologically important in modulating postprandial gastric acid secretion. Prostaglandin E2 and its
synthetic derivatives are potent inhibitors of histamine-stimulated acid secretion.

86. Which of the following statement(s) regarding the vagus nerves is/are true?

a. The right and left vagus nerves derive from a nerve plexus inferior to the tracheal bifurcation
b. The posterior vagus nerve is closely applied to the intrathoracic esophagus
c. The anterior vagus supplies a hepatic division which passes to the right in the lesser omentum
d. Approximately 90% of vagal fibers are afferent, transmitting information from the gastrointestinal tract to the
central nervous system
e. The vagus nerves transmit gastroduodenal pain sensations associated with peptic ulceration
Answer: a, c, d
The left and right vagus nerves are formed from a periesophageal nerve plexus between the tracheal bifurcation and the
diaphragm. As they pass through the esophageal hiatus, the anterior vagus is closely applied to the esophagus; the posterior
vagus lies intermediate in position between the esophagus and the aorta. The anterior vagus supplies the hepatic division
which provides parasympathetic innervation to the liver and biliary tract. The hepatic division is usually easily seen in the
thin gastrohepatic omentum and is constant in location. The hepatic division is a useful anatomic landmark in vagotomy
procedures. The posterior vagus nerve supplies fibers to the celiac division. After giving off hepatic and celiac divisions,
both anterior and posterior vagus nerves supply branches to the gastric wall. Surprisingly, only 10% of vagal fibers are
efferent, secretomotor fibers; almost 90% are afferent. Sensations of gastric pain are carried in sympathetic fibers, and
vagotomy does not alter perception of painful gastric conditions or stimuli.

87. Important stimulants of gastrin release from endocrine cells in the antrum include:

a. Acidification of the antral lumen


b. Small peptide fragments and amino acids from luminal proteolysis
c. Locally released somatostatin
d. Dietary fats
Answer: b

Gastrin is processed to 34-and 17-amino acid forms in endocrine cells in the gastric antrum. In addition to well-recognized
stimulatory actions on gastric acid secretion, gastrin promotes mucosal growth of the gastric fundus and small intestine. The
most important stimulant of gastrin release is a meal. Small peptide fragments and amino acids that result from intragastric
proteolysis are the food components that stimulate gastrin release. Ingested fats and carbohydrates have no significant
effect. In this regard, intraluminal pH strongly affects gastrin secretion. If intragastric pH is maintained above 3 after
ingestion of a meal, gastrin release is strongly potentiated. Pernicious anemia and atrophic gastritis, which produce chronic
achlorhydria, are associated with fasting hypergastrinemia and an exaggerated gastrin meal response. Conversely, antral
acidification strongly inhibits gastrin secretion. Locally-released somatostatin mediates the effects of luminal acidification,
inhibiting gastrin secretion.

88. Which of the following statements regarding human gastric acid secretion is/are true?

a. Fasting acid secretion, normally 2 to 5 mEq/h, is due to ambient vagal tone and histamine secretion
b. Truncal vagotomy decreases basal secretion by 80%
c. Histamine2 receptor antagonist administration can decrease basal acid secretion by 80%
d. Fasting acid secretion, normally 5 to 10 mEq/h, is due to circulating levels of gastrin
Answer: a, b, c

Both vagal tone and locally secreted histamine are presumed to be the determinants of basal acid secretion in humans.
Gastrin does not have a role in basal acid secretion in normal individuals.
Parietal cell activation and the resultant acid secretory response is greater to a combination of agonists than the sum of the
responses to the agents used singly. This increase in responsiveness is termed potentiation. Potentiating interactions are most
apparent when the stimulants use different second messenger systems, for example, acetylcholine and histamine.
Conversely, blockade of receptors to one stimulant also blocks responsiveness to the other agonist. Because of this
interaction, blockade of histamine receptors by agents like cimetidine decreases responsiveness to acetylcholine. Blockage
of acetylcholine release by vagotomy decreases responsiveness to histamine secreted by gastric mast cells.
Both vagotomy and histamine2 receptor antagonists decrease basal acid secretion by approximately 80%.

89. As a meal is emptied from the stomach, gastric acid secretion gradually returns to baseline. Which of the following
statements correctly characterize control of gastric acid secretion?

a. In humans, the most important inhibitory influence on gastrin release is exposure of the gastric mucosa to luminal
acid
b. Acidification of the antral lumen causes reciprocal increases in somatostatin release and decreases in gastrin
secretion
c. Antral distension stimulates gastric acid secretion
d. Acidification of the duodenal bulb inhibits gastric acid secretion
e. Exposure of the duodenum to hyperosmolar solutions inhibits acid secretion
Answer: a, b, d, e

Inhibitory regulation of gastric acid secretion is accomplished by central nervous system, gastric and small intestinal
mechanisms. In humans, the most clearly established gastric inhibitory influence is suppression of gastrin release by
exposure of the antral mucosa to luminal acid. Antral acidification causes release of gastric mucosal somatostatin which is
linked reciprocally to decreases in gastrin secretion. Antral distension inhibits gastric acid secretion.
The inhibitory phase of gastric acid secretion begins with entry of the products of digestion into the proximal duodenum.
Acidification of the duodenal bulb and exposure of the duodenum to hyperosmolar solutions and those containing fat
potently inhibit acid secretion.

90. Which of the following statements correctly characterizes gastric motor activity associated with ingestion of a
meal?

a. Ingested gastric volumes are accommodated with little increase in pressure by reflex relaxation of the proximal
stomach
b. Receptive gastric accommodation is unaffected by proximal gastric vagotomy
c. In humans, liquid emptying occurs more quickly than solid emptying
d. Gastric emptying of liquids is not affected by proximal gastric vagotomy
Answer: a, c

With ingestion of a meal, increasing gastric volumes are accommodated with little increase in intragastric pressure by
relaxation of the proximal stomach. This process, termed receptive relaxation, is mediated by a reflex carried by the vagal
nerve. After the meal has been ingested, the proximal stomach is the predominant determinant of the rate of gastric
emptying of liquids due to the gastroduodenal pressure gradient generated by proximal gastric contractions. Liquid
emptying occurs more rapidly than emptying of solids, in part, because liquids are not subject to the sieving actions of the
pylorus.
Truncal and proximal gastric vagotomy abolish receptive relaxation. Following vagotomy, an increased gastroduodenal
pressure gradient is observed and correlates with accelerated liquid emptying. Emptying of solids is usually not significantly
altered by proximal gastric vagotomy.

91. It is widely agreed that the gastric mucosa secretes bicarbonate in addition to acid. Gastric secretion of bicarbonate
is correctly characterized by which of the following statements?

a. Bicarbonate is secreted by chief cells within gastric crypts


b. Gastric bicarbonate secretion is stimulated by acetylcholine
c. Gastric bicarbonate secretion during fasting results in luminal pH above 6 in normal individuals
d. Prostaglandin E2 is a potent stimulant of gastric bicarbonate secretion
Answer: b, d

The gastric cells responsible for bicarbonate secretion are believed to be surface mucous cells facing the gastric lumen
between crypts. Although the total amount of gastric bicarbonate secreted is only a small fraction of total acid secretion, pH
close to neutrality is maintained near the mucosal surface while bulk luminal pH is highly acidic. Cholinergic agonists,
vagal stimulation, and sham feeding all increase gastric bicarbonate secretion. Prostaglandin E2 and its synthetic derivatives
are potent stimulants of bicarbonate secretion as well. Conversely, indomethacin and other drugs that inhibit prostaglandin
formation decrease mucosal bicarbonate secretion.

92. Gastric mucosal blood flow is regulated by neural, hormonal, and locally active influences. Which of the following
statements correctly characterizes gastric blood flow?

a. Stimulation of sympathetic nerves supplying the stomach is followed by gastric mucosal hyperemia and increased
total gastric blood flow
b. Vagal nerve stimulation is accompanied by decreased gastric mucosal blood flow
c. Stimulants that increase acid secretion increase mucosal blood flow
d. In humans, prostaglandins increase mucosal blood flow at doses that inhibit gastric acid secretion
Answer: c, d
Because the gastric mucosa is metabolically highly active, control of gastric mucosal blood flow is of great physiologic
significance. Almost all stimuli that increase acid secretion also increase gastric blood flow. A large number of
gastrointestinal hormones stimulate gastric blood flow, most because of their ability to increase acid secretion. Thus, gastrin
is a potent stimulant of blood flow, in proportion to its ability to increase acid secretion. Vagal nerve stimulation has the net
effect of increasing mucosal and total gastric blood flow; sympathetic nerve stimulation is accompanied by opposite effects.
Prostaglandins are important endogenous regulators of gastric blood flow. Prostaglandins of the E class increase blood flow
at doses that suppress acid secretion. Inhibition of cyclo-oxygenase activity by indomethacin causes a reduction in resting
gastric blood flow.

93. Which of the following statements regarding intrinsic factor is/are correct?

a. Intrinsic factor is produced in chief cells located in the gastric fundus


b. Total gastrectomy is followed by folate deficiency due to vitamin malabsorption secondary to intrinsic factor
deficiency
c. Intrinsic factor secretion, like that of acid, is stimulated by gastrin, histamine, and acetylcholine
d. Intrinsic factor deficiency accompanies H pylori-caused antral gastritis
Answer: c

The gastric mucosa is the site of production of intrinsic factor, which is a necessary co-factor for the absorption of vitamin
B12 by the ileal mucosa. Total gastrectomy and atrophic gastritis involving the proximal oxyntic mucosa are regularly
followed by vitamin B12 deficiency, manifest as pernicious anemia. Acid-secreting parietal cells are the site of intrinsic
factor synthesis. Like acid secretion, intrinsic factor secretion is stimulated by gastrin, histamine, and acetylcholine

94. A 24-year-old woman develops epigastric pain and has a diagnosis of duodenal ulcer confirmed by
esophagogastroduodenoscopy. The patient is in the third month of a pregnancy. The most appropriate treatment would
be:

a. Proximal gastric vagotomy


b. Misoprostol 400 mg b.i.d.
c. Sucralfate 1 gm q.i.d.
d. Cimetidine 400 mg b.i.d.
Answer: c

Cimetidine, ranitidine, famotidine and newer H2 receptor antagonists bind competitively to parietal cell histamine receptors
to produce reversible inhibition of acid secretion. While the pharmacokinetic profiles of the H2 receptor antagonists differ,
when administered at equipotent doses, the agents produce similar degrees of acid suppression and similar rates of ulcer
healing. Cimetidine interacts with the hepatic microsomal enzyme system and may increase blood levels and
pharmacological effects of drugs that depend upon hepatic metabolism.
Misoprostol, a derivative of prostaglandin E, causes endoscopic healing in 60% of patients at 4 weeks. The major side effect
of misoprostol is diarrhea due to effects upon gut smooth muscle contractility. Uterine bleeding has been reported in some
women using the drug and the agent has potential abortifacient actions. For this reason, misoprostol is contraindicated in
pregnancy.
Sucralfate, the aluminum salt of sulfated sucrose, has virtually no systemic absorption, and for this reason, is the drug of
choice for pregnant patients. When administered at a dose of 1 gm four times daily, 80% of ulcers will heal by 6 weeks.

95. Helicoabacter pylori has been investigated as a possible etiologic agent in duodenal ulceration. Which of the
following statement(s) regarding H pylori infection in humans is/are correct?

a. H pylori may be isolated from antral gastric mucosa in nearly 100% of patients with active duodenal ulceration but
only 1–2% of normal volunteers
b. H pylori possess cell surface receptors that bind to small intestinal mucous cells
c. Therapeutic regimens for duodenal ulcer that eliminate the organism are associated with lower ulcer recurrence
rates than those in which the organism persists
d. The incidence of the organism in the normal population increases with age
e. Antral gastritis is associated with development of duodenal ulcer
Answer: c, d, e
Helicobacter pylori has received enormous investigative attention in recent years as a possible infectious cause of peptic
ulceration. The evidence that H pylori causes ulcers is substantial but largely inferential. Antral gastritis is nearly always
present in patients with duodenal ulceration. H pylori infestation of antral mucosa is believed to cause gastritis. While
normal small intestinal cells do not permit H pylori binding, areas of gastric metaplasia are usually demonstrated in the
duodenal mucosa immediately surrounding the ulcer. Resolution of gastritis follows eradication of the organism and drug
regimens that are bactericidal are associated with lower rates of ulcer recurrence than those that have no anti-bacterial
actions.
However, it is clear that not all patients infested with H pylori develop ulceration. Half of patients with dyspepsia but no
ulceration have evidence of H pylori infestation and 20% of healthy volunteers can be demonstrated to harbor the organism.
The incidence of infestation increases with age in the asymptomatic population.

96. A 40-year-old male undergoes treatment of acute duodenal ulceration with cimetidine 400 mg b.i.d. and has
resolution of symptoms by 6 weeks. The medication is continued as a nocturnal maintenance dose at the end of a three
month treatment course. Recurrent symptoms develop 6 months after initial diagnosis and repeated endoscopic
examination reveals recurrent ulceration. Biopsies of antral mucosa demonstrate moderate gastritis and the presence of
H pylori. Medical management designed to eradicate H pylori and heal ulceration should include which of the
following agents?

a. Cimetidine
b. Bismuth subcitrate
c. Amoxicillin
d. Metronidazole
e. Vancomycin
Answer: a, b, c, d

The observation that H pylori infection has an important role in ulcer pathogenesis has led to development of antimicrobial
therapy for ulceration. Most successful regimens are based on a bismuth compound (colloidal bismuth subsalicylate or
colloidal bismuth subcitrate) plus metronidazole, alone or in combination with amoxicillin or tetracycline. Bismuth
compounds act locally and achieve gastric concentrations above the MIC for 90% of H pylori isolates. Metronidazole is
secreted into the stomach at high concentration, and the in vivo activity of metronidazole is not diminished by gastric
acidity. Triple therapy with bismuth, metronidazole, and tetracycline or amoxicillin eradicates H pylori in 90% of cases,
compared to 0% eradication with ranitidine. Inclusion of an H2 receptor antagonist or omeprazole has been reported to
increase efficacy of antimicrobial therapy. Currently, antimicrobial therapy has been recommended for peptic ulcer disease
resistant to conventional therapy, including patients with ulcer relapse while on maintenance therapy and failure to heal in
spite of H2 receptor antagonist or omeprazole therapy.

97. Development of duodenal ulceration is dependent upon gastric acid secretion. Which of the following statements
correctly characterizes acid secretion in duodenal ulcer patients?

a. Groups of duodenal ulcer patients demonstrate decreased basal acid secretion


b. Maximal acid output to histamine averages 40 mEq/h in duodenal ulcer patients, twice that of normal
c. Tissue gastrin levels, on average, are twice normal in patients with active ulceration
d. Exogenously administered somatostatin is ineffective in suppressing acid secretion in patients with active
ulceration
Answer: b

The formation of duodenal ulcer is dependent on gastric secretion of acid and pepsin. As a group, duodenal ulcer patients
have an increased capacity for gastric acid secretion relative to normal individuals, manifest by increased basal acid
secretion, increased acid response to meal ingestion, and increased responsiveness to histamine stimulation. No definite
evidence links abnormalities in gastrointestinal hormone secretion with increased acid secretion. Tissue gastrin content and
circulating levels of gastrin are normal (excluding Zollinger-Ellison syndrome patients). Secretion of endogenous
somatostatin and responsiveness to exogenously administered somatostatin are likewise normal.

98. A 45-year-old man undergoes proximal gastric vagotomy for treatment of intractable duodenal ulceration. What
physiologic alterations might be anticipated as a consequence of the operation?
a. Reduction of basal acid secretion by approximately 25%
b. Accelerated gastric emptying of liquids
c. Accelerated gastric emptying of solids
d. Fasting hypergastrinemia
e. Postprandial hyperinsulinemia
Answer: b, d

Division of cholinergic vagal fibers directly affects parietal cell acid secretion by reducing stimulatory input. Basal acid
secretion is diminished by approximately 80% and maximal acid output in response to pentagastrin stimulation is reduced
by about 70%. Fasting hypergastrinemia and an exaggerated gastrin response to meal ingestion is observed due to loss of
feedback inhibition of gastrin release and gastrin cell hyperplasia. Release of pancreatic polypeptide, secretin and
cholecystokinin may be decreased.
Proximal gastric vagotomy accelerates gastric emptying of liquids due to a loss of receptive relaxation. In contrast, gastric
emptying of solids is usually not affected by proximal gastric vagotomy.

99. Which of the following statements regarding postoperative rates of recurrent ulcer and dumping is/are correct?

a. Truncal vagotomy and antrectomy is associated with persistent dumping in 10–15% of patients
b. Recurrent ulceration following truncal vagotomy and pyloroplasty is observed in 25% of patients within 10 years
of operation
c. Patients that undergo proximal gastric vagotomy have a risk of recurrent ulcer of 10–15% and a risk of persistent
dumping approximating 1%
d. Recurrent ulceration occurs in 5% of patients that undergo truncal vagotomy and antrectomy
Answer: a, c

Surgical recommendations for treatment of peptic ulceration should be based upon safety, freedom from long-term
postoperative symptoms, and avoidance of recurrent ulceration. Proximal gastric vagotomy has an operative mortality of
less than 1% and a risk of persistent dumping symptoms of approximately 1%. The low incidence of postoperative
symptoms is associated with a relatively high risk of recurrent ulceration, estimated to be 10% to 15% at 5 years
postoperatively. After truncal vagotomy and pyloroplasty, dumping is initially present in 10%, and is persistent or severe in
1%. Recurrent ulceration is observed in 10% of patients that undergo truncal vagotomy and pyloroplasty. Truncal vagotomy
and antrectomy is associated with the lowest risk of recurrent ulceration, 1–2%, but the greatest incidence of postoperative
dumping symptoms, 10–15%.

100.Which of the following statement(s) is/are correct with regard to pyloric obstruction secondary to peptic
ulceration?

a. Pyloric obstruction is suggested by hypochloremic hyponatremic alkalosis


b. Pyloric obstruction is suggested by hypochloremic hypokalemic alkalosis
c. Approximately 80% of patients with benign gastric outlet obstruction obtain permanent relief of symptoms by
endoscopically-directed balloon dilatation
d. The lifetime risk of pyloric obstruction in peptic ulcer patients is 40%
Answer: b

Repeated episodes of ulceration and healing can lead to scarring and pyloric stenosis. The lifetime risk of this complication
approximates 10%. Gastric outlet obstruction is characterized by the development of hypochloremic hypokalemic alkalosis
due to loss of HCl through vomiting and renal compensatory mechanisms that conserve H+ at the expense of secreted K+.
Although 85% of pyloric stenoses are technically amenable to balloon dilatation, fewer than 1 in 3 will achieve permanent
relief of symptoms through this means.

101.A 42-year-old man with a recently diagnosed duodenal ulcer develops melena and near-syncope. After fluid
resuscitation, upper gastrointestinal endoscopy is performed. During the examination, a 1 cm ulcer is noted in the
proximal duodenum. A fresh clot is observed within the ulcer and blood is noted to be oozing around the clot. Optimal
therapy would consist of which of the following?
a. Angiographic embolization of the gastroduodenal artery
b. Irrigation of the clot followed by endoscopic application of a heat probe
c. Transfusion and intravenous cimetidine
d. Angiographic infusion of vasopressin into the gastroduodenal artery
e. Transfusion and oral omeprazole
Answer: b

The ability to visualize bleeding duodenal ulcers endoscopically has led to attempts to treat hemorrhage endoscopically.
Thermal coagulation may be achieved by bipolar electrocoagulation or by direct application of heat through a probe. An
NIH Consensus Development Conference has recommended endoscopic hemostatic therapy in selected patients.
Hematemesis, age over 60, and serious medical co-morbidity are clinical features that mandate endoscopic therapy.
Rebleeding during hospitalization and the endoscopic findings of visible vessel, oozing, or bleeding associated with an
adherent clot are other indications for endoscopic hemostasis. Operative intervention is appropriate for massive hemorrhage
leading to shock or cardiovascular instability, prolonged blood loss requiring continuing transfusion, recurrent bleeding
during medical therapy or after endoscopic therapy, and recurrent hemorrhage requiring hospitalization. Operative therapy
should consist of duodenotomy with direct ligation of the bleeding vessel within the ulcer base followed by a procedure to
permanently reduce acid production.

102.A 50-year-old patient has undergone truncal vagotomy and antrectomy with Billroth II reconstruction two years
ago. The patient now complains of recurrent postprandial pain, nausea, and vomiting. Endoscopic examination reveals
bile in the stomach; endoscopic biopsies demonstrate histologic evidence of moderately severe gastritis. No other
endoscopic abnormalities are noted. Appropriate therapy could include:

a. Octreotide administration
b. Total gastrectomy
c. Conversion of Billroth II gastrojejunostomy to Billroth I gastroduodenostomy
d. Conversion of Billroth II gastrojejunostomy to Roux-en-Y gastrojejunostomy
e. Roux-en-Y hepaticojejunostomy
Answer: d

Symptoms related bile reflux gastritis occur transiently in 10% to 20% of patients after truncal vagotomy and resection or
drainage. Symptoms persist in only 1% to 2%. No completely satisfactory solution to bile reflux gastritis exists. Medicinal
and dietary treatments have not been proven to be beneficial. Operative diversion of biliary secretions away from the gastric
mucosa by construction of a Roux-en-Y gastrojejunostomy with an intestinal limb of 50 to 60 cm has been widely reported.
The procedure eliminates bilious vomiting in nearly 100% of patients, but pain persists in up to 30%, and 20% develop
delayed gastric emptying as a result of the procedure.

103.A 50-year-old male with a 2 year history of duodenal ulceration develops sudden, severe epigastric pain 4 hours
prior to evaluation. Physical examination reveals T 101° F, pulse 80, BP 125/90, diminished bowel sounds, and
abdominal muscular rigidity. An upright chest x-ray reveals pneumoperitoneum. At laparotomy, an anterior perforation
in the first portion of the duodenum is observed. Optimal treatment would include:

a. Omental patch of the perforation followed by truncal vagotomy and antrectomy after 8 weeks
b. Omental patch of the perforation followed by truncal vagotomy and pyloroplasty after 8 weeks
c. Omental patch of the perforation followed by chronic cimetidine administration
d. Omental patch of the perforation plus proximal gastric vagotomy
e. Omental patch of the perforation only
Answer: d

Simple omental patching of a perforation in patients with chronic ulcer disease does not yield satisfactory long-term results.
Up to 80% of patients so treated develop recurrent ulceration and 10% develop secondary complications. A definitive ulcer
operation should be performed during the initial laparotomy if the following circumstances apply: 1) there has been no
preoperative shock, 2) the perforation has been present for less than 48 hours, and 3) no life-threatening medical co-
morbidity exists. Omental patching of the perforation combined with proximal gastric vagotomy is a preferred approach
because it combines safety, freedom from disabling postoperative symptoms, and a low rate of recurrent ulceration.
104.Which of the following clinical circumstances have been identified as predisposing factors for the development of
stress ulceration?

a. Intraperitoneal sepsis
b. Hemorrhagic shock
c. Isolated tibial fracture
d. 50% total surface area second degree burn
e. Adult respiratory distress syndrome
Answer: a, b, d, e

Several risk factors or predisposing clinical conditions have been identified for stress ulceration. Specific risk factors
include adult respiratory distress syndrome, multiple trauma, major burn of over 35% of body surface area, oliguric renal
failure, large transfusion requirements, hepatic dysfunction, hypotension, prolonged surgical procedures, and sepsis from
any source. A direct correlation has been shown between acute upper gastrointestinal hemorrhage and the severity of critical
illness.

105.Type I gastric ulcers are located in the gastric body, usually along the lesser curvature. Which of the following
statements correctly characterize type I gastric ulcers?

a. Normal to low acid secretion


b. Associated duodenal ulceration
c. High frequency of blood group A
d. Associated hypergastrinemia frequent
Answer: a, c

Gastric ulcers are divided into categories based on their location and gastric acid secretory status. A type I gastric ulcer is an
ulcer in the body of the stomach, usually along the lesser curvature, associated with large volumes of secretion with a low to
normal acid output. Type I ulcers are not associated with duodenal, pyloric, or prepyloric mucosal abnormalities. There is a
slight predominance of patients with blood group A in this type of gastric ulcer.
Type II gastric ulcer is located in the body of the stomach in combination with a duodenal ulcer. These patients are usually
acid hypersecretors. About 23% to 25% of gastric ulcers are type II. A type III gastric ulcer is characterized as a prepyloric
ulcer and accounts for about 23% of lesions. Patients with this lesion are typically acid hypersecretors.
Type IV gastric ulcer occur high on the lesser curvature near the gastroesophageal junction. In the United States, the
incidence of type IV gastric ulcer is less than 10%.

106.Which of the following statement(s) is/are correct regarding gastric ulcers greater than 3 cm in size?

a. Giant gastric ulcers occur in 30–40% of cases along the greater curvature
b. The risk of malignancy increases with increasing size of the ulcer
c. The treatment of choice for giant gastric ulcer is resection to include the ulcer
d. Giant gastric ulcer is a complication of intraarterial hepatic chemotherapy
Answer: b, c

A giant gastric ulcer is defined as an ulcer whose diameter is 3 cm or greater. The lesser curvature is the most common site,
with only 3% to 4% occurring along the greater curvature. Gastric ulcers often penetrate into contiguous structures such as
spleen, pancreas, liver, and transverse colon, and can be falsely diagnosed as a nonresectable malignancy, despite normal
biopsy results. Most reports cite an incidence of malignancy ranging from 6% to 30%, increasing with the size of the ulcer.
Because of the high likelihood of complications from giant gastric ulcer, early operation is the treatment of choice. The
operation of choice is resection, including the ulcer. If the ulcer has penetrated adjacent structures and cannot be dissected
free, the stomach wall can be incised circumferentially, leaving the ulcer intact and behind, and the gastric resection
completed.

107.With regard to benign gastric ulceration, the most common location of disease is which of the following?

a. Along the greater curvature


b. Immediately distal to the esophagogastric junction along the lesser curvature
c. In the area of the incisura angularis along the lesser curvature
d. Within the gastric antrum
Answer: c

Gastric ulcers can occur anywhere in the stomach, although they usually present on the lesser curvature near the incisura
angularis. About 60% are located at or slightly above the angularis. Fifteen percent to 23% of gastric ulcers are within the
gastric antrum and 10% are high on the lesser curvature. Only 5% of gastric ulcers are found on the greater curvature. In
addition, 97% of all gastric ulcers occur within 2 cm of the junctional zone between fundic and antral mucosa. Gastric
ulcers appear at different distances from the pyloric sphincter because the antrum extends for variable (2 to 16 cm) distances
from the pylorus. It is interesting that with increasing age this junctional zone moves proximally along the lesser curvature,
as does the incidence of gastric ulcer.

108.Which of the following statement(s) regarding gastric mucosal defense is/are correct?

a. Gastric mucus, produced by the surface epithelial cells, forms an unstirred layer over the gastric surface
b. Gastric parietal cells produce a bicarbonate-rich fluid
c. Production of gastric bicarbonate is stimulated by prostaglandins and inhibited by non-steroidal antiinflammatory
drugs (NSAIDs)
d. Gastric mucus provides substantial buffering capacity that maintains near-neutrality near the epithelial surface
Answer: a, c

Gastric mucus is produced by the surface epithelial cell. In addition to serving as a lubricant, mucus also enhances mucosal
defense by forming an unstirred layer overlying the epithelial surface. In humans, the thickness of this gel layer has been
reported to be about 500 µm. The vacuoles containing mucus in the cytosol of the surface epithelial cell are released by
cholinergic stimulation and prostaglandins. Synthesis and release are inhibited by aspirin-like compounds that inhibit
cyclooxygenase. Gel mucus retards proton mobility by a factor of only three or four, a degree that is inadequate to maintain
a near neutral pH at the apical membrane of the surface epithelial cell.
In addition to producing mucus, the gastric surface epithelial cells secrete a bicarbonate-rich fluid. The amount of
bicarbonate secreted is about 5% to 10% of the amount of acid that the same surface of mucosa can produce. As with
mucus, bicarbonate secretion is stimulated by cholinergic agents and prostaglandins and inhibited by cyclooxygenase
inhibitors.
The release of bicarbonate into the gel mucus provides a significant mucosal defense by maintaining a near neutral acid-
base milieu at the apical membrane of the surface epithelial cells. When the luminal pH is around 3.0, the apical membrane
of the surface epithelial cell may be exposed to a pH of about 5.0.

109.A 35-year-old smoker is involved in a house fire and receives a 45% total surface area burn. One half of the burned
surface appears to be third degree. On the third post-burn day, the patient is noted to have bloody drainage from a
nasogastric tube and a decrease of 5% in his hematocrit. Appropriate management should include which of the
following?

a. Urgent upper gastrointestinal contrast study to delineate site of bleeding


b. Immediate selective arteriography via the left gastric artery to diagnose and treat presumed stress ulceration
c. Urgent esophagogastroduodenoscopy to diagnosis the cause of bleeding
d. Urgent intravenous infusion of vasopressin at 0.2–0.4 IU/min
Answer: c

Patients who have sustained a major thermal burn of 35% or more of their body surface area are at a predictably high risk
for the development of gastric erosions and hemorrhage. Endoscopy has demonstrated that gastric erosions are present in
93% of these patients, whereas the occurrence of severe acute upper gastrointestinal hemorrhage in severely burned patients
ranges between 25% to 50%.
At least 60% of patients at risk develop stress erosions within 1 to 2 days after the precipitating event. Painless upper
gastrointestinal bleeding may be the only clinical sign. The onset of hemorrhage is often delayed, usually occurring 3 to 10
days after the onset of the primary disease.
Esophagogastroduodenoscopy is the diagnostic modality of choice to confirm the diagnosis and to differentiate stress
erosion from other sources of upper gastrointestinal hemorrhage. Correct identification of the bleeding source is made in
greater than 90% of instances. If endoscopy is not diagnostic, visceral angiography through selective catheterization of the
left gastric or splenic vessels may provide information regarding the primary vessel supplying the bleeding site. In contrast,
barium examinations are usually of little value, due to the superficial nature of stress erosions, and in fact may be
detrimental by interfering with the interpretation of subsequent arteriography.

110.Agents demonstrated to have an efficacy of greater than 90% for prophylactic treatment of stress ulceration include
which of the following?

a. Antacids
b. H2 receptor antagonists
c. Sucralfate
d. Misoprostil
Answer: a, b, c

The hourly administration of antacid (30 to 60 mL) by nasogastric tube, maintaining the gastric luminal fluid at pH above
3.5, has proven to be effective prophylaxis. In a study of 100 seriously ill patients who were randomly assigned to receive
placebo or antacid prophylaxis, bleeding was detected in 25% of patients given no prophylaxis, compared to 4% of patients
given antacids through the nasogastric tube. In a review of data derived from 16 prospective trials, when overt bleeding
manifested by melena, hematemesis, or transfusion requirement was used as the minimum criterion, there was no significant
difference in risk of bleeding, comparing antacids and cimetidine.
Continuous infusions of any of the H2-receptor antagonists provides more consistent maintenance of an intraluminal gastric
pH of greater than 3.5 than do the standard intermittent-infusion regimens. Advantages of continuous infusion of these
agents include a potential reduction in toxicity, decreased pharmacy costs and nursing duties, and possible enhancement of
therapeutic benefit. Controlled trials suggest that sucralfate, 1 g every 6 hours, may be as effective as antacids or cimetidine
prophylactically. In 100 critically ill patients, bleeding occurred in 6% of patients receiving antacids or cimetidine, while
none of the 34 patients on sucralfate bled.
Given exogenously, natural or synthetic prostaglandins of the E, F, and I series inhibit gastric acid secretion. One group
compared the efficacy of 15(R)-15 dimethyl PGE2 given at antisecretory doses to antacids and found that stress-related
bleeding occurred in 50% of patients given the synthetic prostaglandin derivative, compared to only 14% of patients
receiving antacids.

111. A 45-year-old male presents with symptoms of epigastric pain, worsened with ingestion of food. Physical
examination is normal. Upper abdominal ultrasonography is unremarkable. Contrast radiography reveals a 2 cm ulcer
in the gastric fundus along the lesser curvature. Therapy with omeprazole 20 mg per day is begun but symptoms persist
3 weeks later. Appropriate management includes which of the following?

a. Increase in omeprazole dose to 40 mg per day


b. Addition of sucralfate 1 gm every 8 hours
c. Addition of cimetidine 200 mg b.i.d.
d. Esophagogastroduodenoscopy with biopsy of ulceration
Answer: d

About 5% of ulcers appearing radiographically benign are malignant} Gastroscopy is the most reliable method of
distinguishing benign and malignant gastric ulcer, with an accuracy of more than 97% if multiple biopsies and brushings for
cytology are performed. Clinical features prompting early endoscopic evaluation include major weight loss, symptoms of
gastric outlet obstruction, a palpable abdominal mass, and stool hemoccult positivity or blood loss anemia. Endoscopic
features that suggest malignancy include an exophytic mass, abnormal or disrupted mucosal folds, necrotic ulcer crater,
bleeding from the edge of the ulcer crater, a stepwise depression of the ulcer edge, heaped-up margins, or small extensions
of the ulcer that blur a portion of the ulcer wall. If initial biopsies do not demonstrate malignant cells but the endoscopic
appearance strongly suggests that underlying the ulcer is a carcinoma, repeat endoscopy with deeper biopsies should be
undertaken.

112.A 52-year-old woman is hospitalized with acute upper gastrointestinal hemorrhage. Endoscopic examination reveal
a 2.5 cm ulcer in the area of the incisura angularis. The remainder of the endoscopic examination is normal. Continued
bleeding requires operative therapy. Optimal therapy consists of which of the following?

a. Gastrotomy with oversewing of the bleeding site


b. Distal gastrectomy including the area of ulceration
c. Proximal gastric vagotomy and oversewing of the bleeding ulcer
d. Truncal vagotomy, pyloroplasty, and oversewing of the bleeding ulcer
Answer: b

A distinction should be made among the different types of gastric ulcer in selecting the most appropriate operative
procedure, because treatment varies according to location, coexistent duodenal ulcer disease, and acid secretory status. The
elective operation of choice for a type I benign gastric ulcer is a distal gastrectomy with gastroduodenal anastomosis.
Gastrojejunostomy is an acceptable alternative. The ulcer should be included in the antrectomy specimen. The operative
mortality rate associated with this procedure is 2% to 3%, the recurrence rate is 3%, and a good to excellent clinical result
can be anticipated in more than 90% of patients. The addition of truncal vagotomy does not appear to diminish the
recurrence rate.
Definitive treatment for hemorrhage is accomplished by a procedure designed to control bleeding in addition to preventing
recurrent ulceration. An antrectomy, which includes the ulcer with gastroduodenostomy, is considered the procedure of
choice for surgical treatment of this complication. The quoted operative mortality rates in this setting range from 10% to
40%.

113.A 25-year-old man is involved in an automobile accident with resultant injuries including bilateral closed femur
fractures, left pulmonary contusion, and closed head injury. On post-injury day 4, significant upper gastrointestinal
hemorrhage begins. Endoscopic examination reveals an area of confluent ulceration with bleeding in the gastric fundus.
Endoscopic hemostasis fails. Appropriate immediate management includes:

a. Lavage of gastric contents with iced saline


b. Urgent total gastrectomy
c. Selective arterial infusion of vasopressin via the left gastric artery
d. Insertion of Sangstaken-Blakemore balloon
Answer: c

Initial efforts to control gastric hemorrhage consist of gastric lavage using warmed saline. Lavage serves to fragment
existing clots and to remove any pooled blood, reducing fibrinolysis at bleeding sites. Over 80% of patients who present
with upper gastrointestinal hemorrhage stop bleeding using this approach. Definitive treatment of ongoing acute active
stress bleeding by antacids is largely unsuccessful. Administration of H2-receptor blocking agents once active
gastrointestinal bleeding has commenced is also usually ineffective as a definitive form of therapy.
The endoscope has become the preferred therapeutic as well as diagnostic instrument with electrocautery and laser
photocoagulation capabilities. If endoscopic therapy fails, angiography offers an additional means for the control of
bleeding by selective infusion of vasopressin into the splanchnic circulation via the left gastric artery. Vasopressin is
administered by continuous infusion through the catheter at a rate of 0.2 to 0.4 IU/min for a maximum of 48 to 72 hours.
About 10% to 20% of patients with acute stress ulcers continue to bleed or have recurrent bleeding despite these measures.
In these patients, total gastrectomy has a mortality ranging from 17% to 100%. In general, operative mortality rates for
acute stress-induced hemorrhage range from 30% to 60% regardless of the surgical procedure undertaken.

114.Which of the following statements regarding gastroplasty and gastric bypass for morbid obesity is/are correct?

a. Horizontal gastroplasty techniques that rely on a single horizontal application of a stapling device are associated
with weight loss “failure” rates of 40% to 70%
b. Gastric bypass is followed by progressive weight loss over a period of 36 months
c. Gastric bypass is associated with a “failure” of weight loss in 10 to 15% of patients
d. With three superimposed applications of a stapling device, gastric bypass staple line dehiscence occurs in less than
2%
Answer: a, c, d

Horizontal gastroplasties include a single application of a 90-mm stapling device without suture reinforcement of the
“stoma’’ between upper and lower gastric pouches or a double application of staples with either a central or lateral prolene-
reinforced stoma. The failure rates (loss of less than 40% excess weight) for horizontal gastroplasty procedures ranges from
40% to 70%. The vertical banded gastroplasty (VBGP) is a procedure in which a stapled opening is made in the stomach
with the stapling device 5 cm from the cardioesophageal junction. Two applications of a 90-mm stapling device are made
between this opening and the angle of His, and a 1.5 5 cm strip of polypropylene mesh is wrapped around the stoma on the
lesser curvature and sutured to itself.
Gastric bypass can be performed with placement of staples in a vertical or horizontal direction; the vertical direction is
preferred because there is less risk of gastric pouch devascularization or splenic injury. With three superimposed
applications of a 90-mm stapler, the incidence of staple line disruption has been less than 2%.
Roux-en-Y gastric bypass has significantly better weight loss than VBGP. Although 10% to 15% of patients fail gastric
bypass, weight loss seems to remain stable in most patients over 5 years or more after surgery.

115.Severe obesity is associated with a large number of associated problems that form the basis of the term morbid
obesity. Documented causes of excess mortality in severely obese patients include:

a. Coronary artery disease


b. Hypertension
c. Adult-onset diabetes mellitus
d. Obesity hypoventilation and sleep apnea
e. Pulmonary embolization
Answer: a, b, c, d, e

Morbid obesity is arbitrarily defined as 100 pounds above ideal body weight as defined by actuarial tables. Premature death
is much more common in the severely obese; morbidly obese men aged 25 to 34 years have a 12-fold increase in mortality
relative to normal. Causes of early mortality include coronary artery disease, hypertension, impaired ventricular function,
diabetes mellitus, sleep apnea and other hypoventilation syndromes, pulmonary embolization, and necrotizing soft tissue
infections.

116.Jejunoileal bypass was formerly performed as a weight reduction procedure. The operation has now been
abandoned due to the development of serious long-term complications associated with the procedure. Which of the
following statements correctly characterize results following jejunoileal bypass?

a. Kidney stones occur with increased frequency due to increased absorption of pyruvate from the colon
b. The most serious complication of jejunoileal bypass is development of cirrhosis due to protein calorie malnutrition
c. Bacterial overgrowth in the bypassed segment can be treated with oral vancomycin
d. Rapid weight loss following jejunoileal bypass is associated with development of gallstones
Answer: b, d

Jejunoileal bypass is associated with a number of early and late complications. Malabsorption of bile salts, coupled with
rapid weight loss, significantly increases risk of gallstone development. Multiple kidney stones result from excessive
absorption of oxylate from the colon where oxylate is ordinarily chelated with calcium. Malabsorption results in severe
diarrhea, electrolyte abnormalities, metabolic acidosis and anemia. Bacterial overgrowth in the bypassed intestinal segment
coupled with protein malabsorption is postulated to be responsible for development of cirrhosis, the most serious
complication of jejunoileal bypass. Bacterial overgrowth can be temporarily suppressed by metronidazole. Development of
hepatic dysfunction is an indication for reversal of the bypass.

117.Which of the following statements is correct with regard to gastric bypass for obesity?

a. Rapid weight loss following successful gastric bypass for obesity is associated with an increased risk of developing
cholelithiasis
b. Marginal ulcer develops in 25% of gastric bypass patients
c. Vitamin B12 deficiency is a potential complication of gastric bypass due to gastric mucosal atrophy
d. Anastamotic leak after gastric bypass is often heralded by bradycardia
Answer: a

The most serious complication after gastric bypass for obesity is anastamotic dehiscence. Leak is presumed to occur
secondary to gastric necrosis due to ischemia from staple line application or short gastric vessel ligation. Affected patients
may have little pain, with tachycardia, tachypnea and fever as the only manifestations. Physical examination in morbidly
obese patients with peritonitis is unreliable. Marginal ulcers occur in only 10% or less of gastric bypass patients and respond
to H2 receptor antagonists. Rarely, polyneuropathy has been noted after gastric bypass, usually in association with
intractable vomiting and protein calorie malnutrition. Vitamin B12 deficiency has been noted after gastric bypass due to
decreased acid digestion of vitamin B12 in food; monthly B12 supplementation should be routine. Cholelithiasis occurs in
about one third of morbidly obese patients and gallstone formation is accelerated in the early postoperative period by the
effects of rapid weight loss.

118.With regard to operative treatment of gastric carcinoma, which of the following statements is/are correct?

a. Resectional margins of 2 cm are necessary to prevent recurrence due to intramural metastases


b. Prophylactic splenectomy has been shown to improve outcome in similarly staged patients
c. Extended lymphadenectomy including nodes along the aorta and esophagus has not been shown to improve
survival in North American trials
d. Long-term survival is rare if adjacent organs must be resected to achieve local control
Answer: c, d

In gastric cancer, microscopic involvement of the resection margin by tumor cells is associated with poor prognosis. In
contrast to colon cancer, gastric cancer frequently demonstrates extensive intramural spread. Retrospective studies suggest
that a line of resection 6 cm from the tumor mass is necessary to ensure a low rate of anastamotic recurrence.
The value of extended lymphadenectomy in the treatment of gastric adenocarcinoma is controversial. The largest favorable
experience has been reported by Japanese surgeons, where retrospective studies have suggested an improvement of
approximately 10%, stage for stage, for patients with advanced disease. The benefits of extensive lymphadenectomy have
not been confirmed in western countries.
Histologically positive lymph nodes are frequently present in the splenic hilum and along the splenic artery, and routine
splenectomy has been practiced in some centers. Prophylactic splenectomy has not been demonstrated to improve outcome
for similarly staged patients. Resection of adjacent organs may be required for local control if direct invasion has occurred.
In this circumstance, operative morbidity is increased, and long-term survival is rare.

119.Which of the following statement(s) characterizing gastric lymphoma is/are correct?

a. More than half of gastrointestinal lymphomas occur in the stomach


b. The peak incidence of gastric lymphoma is in the 2nd and 3rd decade
c. Endoscopic biopsy is positive diagnostically in 90% of cases
d. Gastric perforation occurs in 40% of patients treated with cytolytic agents instead of gastrectomy
Answer: a, c

The stomach is the site of more than half of gastrointestinal lymphomas and is the most common organ involved in
extranodal lymphomas. Gastric lymphoma is distinctly uncommon in children and young adults. The peak incidence is in
the sixth and seventh decades. Radiologic findings are similar to those noted for adenocarcinoma. Endoscopic examination
has become the diagnostic method of choice. Endoscopic biopsy, combined with endoscopic brush cytology, provides
positive diagnosis in some 90% of cases. When gastric lymphoma is first diagnosed by endoscopic means, evidence of
systemic disease should be sought. CT of chest and abdomen (to detect lymphadenopathy), lymphangiography, bone
marrow biopsy, and biopsy of enlarged peripheral lymph nodes may all be appropriate.
A multimodality treatment program is used in most centers for primary gastric lymphomas, with gastrectomy as the first
step in the therapeutic strategy. Increasing numbers of patients are treated with chemoradiation therapy alone. The risk of
hemorrhage or perforation was frequently alluded to in the past and has probably been overstated. The risk of perforation in
primary gastric lymphomas that are treated with cytolytic agents in unresected patients approximates 5%.

120.A patient with gastric adenocarcinoma undergoes subtotal gastrectomy. Pathological examination reveals that the
tumor penetrates to the serosa. Regional lymph nodes are not involved. Distant metastases are not detected. What is the
correct tumor stage and 5-year survival rate?

a. Stage I: 90% 5-year survival


b. Stage II: 45% 5-year survival
c. Stage III: 15% 5-year survival
d. Stage II: 15% 5-year survival
e. Stage III: 2% 5-year survival
Answer: b
For early lesions of the antrum or middle stomach, distal subtotal gastrectomy including 80% of the stomach provides
satisfactory 5-year survival without increases in operative morbidity. Proximal gastric lesions or larger middle stomach
lesions may require total gastrectomy or esophagogastrectomy to encompass the tumor. Regardless of the extent of gastric
resection, patients with more advanced tumors fare poorly because of the increased likelihood of lymphatic and
hematogenous spread.
The TNM system is shown in Table 25-2. Five-year survival for stage I patients (in situ carcinoma) is close to that of the
normal population. In stage II, 5-year survival approximates 45%, while 15% of stage III patients survive 5 years.
Prolonged survival with systemic metastases in negligible.

121.Which of the following conditions is considered to increase the risk of gastric cancer?

a. Pernicious anemia
b. Prior partial gastrectomy
c. Gastric hyperplastic polyps
d. Gastric adenomatous polyps
Answer: a, b, d

The risk of developing gastric cancer is greater in stomachs that harbor adenomatous polyps. The risk of developing cancer
has been estimated at 10% to 20% and is greatest for polyps more than 2 cm in diameter. Hyperplastic polyps, while
common in the normal population, do not have malignant potential. Gastric malignancy is increased in individuals with
chronic gastritis associated with pernicious anemia. When pernicious anemia has been present for 5 years, the risk of gastric
malignancy is twice that of age-matched controls. An increased risk of gastric carcinoids also exists in patients with
pernicious anemia, presumably due to the effects of long-standing hypergastrinemia. A 3-fold increased risk of gastric
cancer also exists in patients that have previously undergone partial gastric resection. Postgastrectomy cancer is a long-term
concern with increased incidence of malignancy not observed until 15 years postoperatively.

122.A 55-year-old male is evaluated because of symptoms of epigastric pain and anorexia. Physical examination is
normal except for guaiac positivity of stool. Upper endoscopic examination reveals a 1.5 cm ulcer along the lesser
curvature of the stomach proximal to the incisura angularis. Optimal management consists of which of the following:

a. Sucralfate 1 gm q.i.d. for 8 weeks


b. Endoscopic biopsy of the ulcer rim
c. Endoscopic cautery of the ulcer base
d. Endoscopic biopsy of the ulcer base
e. Misoprostol 400 mg b.i.d. for 8 weeks
Answer: b

The symptoms produced by gastric cancer and benign gastric ulcer are non-specific and often similar. Pain is present in 70%
of patients with gastric cancer and is usually constant, non-radiating, and not improved by food ingestion. Physical
examination is usually normal in patients with early gastric cancer. Guaiac positivity is noted in one third.
Fiberoptic endoscopy is the definitive diagnostic method. Although the endoscopic appearance of gastric ulcers may suggest
benign or malignant origins, definite distinction can only be made by gastric biopsy. Accurate diagnosis of gastric cancer
can be made in 95% of cases if multiple biopsies are obtained from the ulcer rim. Biopsy of the ulcer base will more
frequently reveal necrotic material.

123.Which of the following statement(s) regarding gastric leiomyosarcoma is/are correct?

a. Leiomyosarcomas occur with peak frequency in the 2nd and 3rd decades
b. The primary histological indicator of aggressive behavior is the number of mitoses per microscopic field
c. Leiomyosarcomas are usually radiosensitive
d. Lymphadectomy is not indicated during resection because metastases are usually hematogenous
Answer: b, d

Leiomyosarcomas occur with equal frequency in both sexes in the sixth and seventh decades of life. The tumor frequently
may have prominent extraluminal growth and attain large size before causing symptoms. Leiomyosarcomas must be
differentiated from their benign counterparts, leiomyomas. Grossly, the tumors are firm, gray-white masses; a
pseudocapsule separating tumor from normal smooth muscle may occasionally be present. When the tumors reach a large
size, central necrosis is common. Leiomyosarcomas are usually graded histologically, with the frequency of mitotic figures
the prime indicator of aggressive behavior. Lesions with more than 5 to 10 mitoses per 10 high-power fields demonstrate
increased metastasis.
Intraperitoneal sarcomatosis is frequent, as is local recurrence after resection. Metastasis occurs by way of the
hematogenous route, and thus hepatic involvement is common. Lymphatic metastasis is observed in less than 10% of
patients. Negative surgical margins must be ensured histologically, but lymphadenectomy is not indicated because of the
low frequency of lymphatic metastasis. Leiomyosarcomas are not radiosensitive, and chemotherapy has not been shown to
improve survival.

1. The most common site of adenocarcinoma of the small intestine is the:


A. Duodenum.
B. Jejunum.
C. Ileum.
Answer: A

2. The most common benign tumor of the small intestine is:


A. Adenoma.
B. Hemangioma.
C. Leiomyoma.
Answer: C

3. Vigorous bleeding from a small bowel lesion is most likely caused by:
A. Adenocarcinoma.
B. Arteriovenous malformation.
C. Leiomyoma.
Answer: C

4. The lamina propria between the intestinal epithelium and the muscularis mucosae contains:
A. Blood and lymph vessels.
B. Undifferentiated epithelial cells.
C. Nerve fibers.
D. Enterochromaffin cells.
E. Macrophages.
F. Connective tissue.
Answer: ACEF

DISCUSSION: The mucosa of the small intestine encompasses the epithelium, the lamina propria, and the muscularis
mucosae. The lamina propria between the epithelium and the muscularis mucosae contains blood and lymph vessels, nerve
fibers, smooth muscle fibers, fibroblasts, macrophages, plasma cells, lymphocytes, eosinophils, and mast cells, as well as
connective tissue elements.

5. The intestinal epithelial cells, 22 to 26 mm. tall, exhibit a striated luminal border (brush border). The brush border
microvilli:
A. Produce the brush border appearance.
B. Contain amylase.
C. Contain dissacharidases.
D. Increase absorptive area.
E. Play an important role in digestion.
F. Contain trypsinogen.
Answer: ACDE

DISCUSSION: The columnar epithelial cells are responsible for absorption. These cells exhibit a striated luminal border or
brush border. The microvilli account for the appearance of the brush border. The microvilli greatly increase the absorption
surface of the epithelial cell. The brush border contains disaccharidase in high concentrations. In addition to increasing
surface area the microvilli perform an important digestive function.

6. Which of the following statements about carbohydrate digestion are true?


A. Amylopectin has 1-4 straight chains and 1-6 side chains.
B. Amylase has 1-4 straight chains and 1-6 side chains.
C. Amylase breaks 1-4 glucose linkages.
D. Amylase breaks 1-6 side chains.
E. An adult may ingest about 350 gm. of carbohydrate daily.
F. Dietary starch contains two glucose polymers, amylopectin and amylase.
Answer: ACEF

DISCUSSION: Amylopectin, the most abundant constituent of starch, is a 1-4–linked straight chain of glucose molecules.
In addition, amylopectin possesses a 1-6 branching side chain at approximately every 25 glucose units along the straight
chain. Amylase has only 1-4 linkages of glucose molecules. Pancreatic and salivary amylase break the interior 1-4 glucose
linkages.

7. Which of these statements about the digestion of fat are true?


A. Micellar solution provides an optimal environment for the action of pancreatic lipase.
B. Decreasing the pH below 5.5 increases the effectiveness of pancreatic lipase in hydrolyzing fat.
C. Co-lipase blocks triglyceride hydrolysis.
D. Lipase catalyzes the hydrolysis of dietary triglyceride into 2-monoglyceride and fatty acids.
E. Fatty acids and 2-monoglyceride are held in micellar solutions.
F. Fatty acid and 2-monoglyceride enter the intestinal cell by active transport.
Answer: ADE

DISCUSSION: Micellar solution provides an optimal environment for the action of pancreatic lipase. Pro-co-lipase is
converted to co-lipase by trypsin. Co-lipase binds to triglyceride, then lipase complexes with co-lipase, and triglyceride
hydrolysis access. Pancreatic lipase hydrolyzes triglyceride into 2-monoglyceride and fatty acids. The 2-monoglyceride and
fatty acid enter the micellae. An alkaline pH allows lipase to function optimally. Micellar fatty acids and 2-monoglyceride
pass into the epithelial cell by diffusion.

8. Complete mechanical small bowel obstruction can cause dehydration by:


A. Interfering with oral intake of water.
B. Inducing vomiting.
C. Decreasing intestinal absorption of water.
D. Causing secretion of water into the intestinal lumen.
E. Causing edema of the intestinal wall.
Answer: ABCDE

DISCUSSION: One of the most important events during simple mechanical small bowel obstruction, loss of water and
electrolytes from the body, is caused mainly by intestinal distention. Distention may produce reflex vomiting. Distention
causes intestinal secretion. Distention causes decreased absorption.

9. History and physical examination permit the diagnosis of intestinal obstruction in most cases. Which of the
following are important for the clinical diagnosis of small bowel obstruction?
A. Crampy abdominal pain.
B. Fever.
C. Vomiting.
D. Abdominal distention.
E. Leukocyte count above 12,000.
F. Abdominal tenderness.
Answer: ABCDF

DISCUSSION: History and physical examination permit the diagnosis of intestinal obstruction. Any patient having crampy
abdominal pain, vomiting, obstipation, abdominal distention, abdominal tenderness, and peristaltic rushes should be
managed for intestinal obstruction until the diagnosis can confidently be excluded.

10. Patients with established, complete, simple, distal small bowel obstruction usually have the following findings on
plain and upright abdominal radiographs:
A. Distended small bowel identifiable by the valvulae conniventes.
B. Multiple air-fluid levels.
C. Modest amount of gas in the pelvis.
D. Peripheral, rather than central, distribution of gas.
E. Prominent haustral markings.
F. Free air.
Answer: AB

DISCUSSION: Abdominal x-ray examination of patients with intestinal obstruction usually reveals abnormally large
quantities of gas in the bowel. One can usually identify distended small intestine or colon. Gas in the small bowel outlines
the valvulae conniventes, which usually occupy the entire transverse diameter of the bowel image. Colonic haustral
markings occupy only a portion of the transverse diameter of the bowel.
Typically, the small bowel pattern occupies the more central portion of the abdomen, whereas the colon shadow is on the
periphery of the abdominal files or in the pelvis. Patients with mechanical small intestinal obstruction usually have minimal
colonic gas, if any.

11. All of the following statements about the embryology of Meckel's diverticulum are true except:
A. Meckel's diverticulum usually arises from the ileum within 90 cm. of the ileocecal valve.
B. Meckel's diverticulum results from the failure of the vitelline duct to obliterate.
C. The incidence of Meckel's diverticulum in the general population is 5%.
D. Meckel's diverticulum is a true diverticulum possessing all layers of the intestinal wall.
E. Gastric mucosa is the most common ectopic tissue found within a Meckel's diverticulum.
Answer: C

DISCUSSION: Meckel's diverticulum is a true diverticulum containing all layers of the intestinal wall, usually arising from
the antimesenteric border of the ileum 45–90 cm. proximal to the ileocecal valve. It is a vestige of the omphalomesenteric or
vitelline duct, which usually undergoes complete obliteration during the seventh week of gestation. Autopsy studies have
estimated the incidence of Meckel's diverticulum to be 1% to 2% with men being more commonly affected than women by
a ratio of 2:1. Gastric mucosa is present in 50% of all Meckel's diverticula, but in over 75% of symptomatic individuals.

12. Meckel's diverticulum most commonly presents as:


A. Gastrointestinal bleeding.
B. Obstruction.
C. Diverticulitis.
D. Intermittent abdominal pain.
Answer: A

DISCUSSION: It is estimated that only 4% of patients who possess a Meckel's diverticulum will become symptomatic
during their lifetimes. The most common clinical presentation is incidental identification during abdominal exploration.
Symptomatic presentations are secondary to hemorrhage, small bowel obstruction, diverticulitis, perforation, associated
umbilical abnormalities, and tumors. Over half of patients presenting with symptoms are under the age of 2. The most
common clinical problem associated with Meckel's diverticulum is gastrointestinal bleeding presenting as bright red blood
per rectum. The usual source of the bleeding is a chronic acid-induced ileal ulcer in the ileum adjacent to a Meckel's
diverticulum that contains gastric mucosa. Another common symptom associated with a Meckel's diverticulum is intestinal
obstruction. The cause of this obstruction may be volvulus of the small bowel around a diverticulum associated with a
fibrotic band attached to the abdominal wall, intussusception, or rarely, incarceration of the diverticulum in an inguinal
hernia (Littre's hernia). Volvulus is usually an acute event and if allowed to progress, may result in strangulation of the
involved bowel. In intussusception, a broad-based diverticulum invaginates and then is carried forward by peristalsis.

13. Which of the following statements about the surgical treatment of carcinoid tumors are true?
A. Carcinoid tumors should be treated by resection, regardless of the presence of metastases.
B. Appendiceal tumors larger than 1.5 cm. should be treated by ileocolectomy.
C. Local excision with margins is adequate for a rectal carcinoid of any size.
D. Carcinoid tumors are associated with a large percentage of other synchronous or metachronous neoplasms.
Answer: ABD

DISCUSSION: Carcinoid tumors should be treated by resection, regardless of the presence of metastases, because growth
of the primary neoplasm is slow and local complications, such as obstruction and intussusception, are frequent. At clinical
discovery a large percentage (as many as 70%) of small-intestinal carcinoids are metastatic to lymph nodes and/or liver. All
tumors should be managed by wide en bloc resection, regardless of the size of the primary lesion or the presence of distant
metastases. Lesions in the distal ileum require ileocolectomy. Appendiceal tumors larger than 1.5 cm. should be treated by
ileocolectomy. The incidence of metastases depends on the size and location of the primary tumor. Appendiceal carcinoid
tumors smaller than 1.5 cm. are rarely malignant and may be treated safely by routine appendectomy. This is not true of
larger tumors. Like carcinoid tumors elsewhere in the gastrointestinal tract, the malignancy potential of rectal carcinoid
tumors is directly proportional to their size. Tumors smaller than 1 cm. have little or no malignant potential and may be
treated by endoscopic excision. Tumors measuring 1 to 2 cm. should be excised operatively with margins, but when they are
larger than 2 cm. rectal carcinoid tumors may require anterior resection. In patients with ileal carcinoid tumors, the evidence
of a second tumor has been reported as high as 40%. Thus, the search for synchronous metachronous and metastatic
neoplasms should be undertaken.

14. Which of the following statements about carcinoid syndrome are true?
A. Carcinoid syndrome occurs only when hepatic metastases are present.
B. Serotonin is thought to be responsible for the diarrhea, cardiac lesions, and flushing in patients with carcinoid syndrome.
C. Foregut carcinoid tumors cause atypical carcinoid syndrome; hindgut tumors are rarely, if ever, associated with the
syndrome.
D. The long-acting somatostatin analog provides the best symptomatic treatment for carcinoid syndrome.
Answer: CD

DISCUSSION: Carcinoid syndrome occurs when venous drainage from the tumor gains access to the systemic circulation,
escaping hepatic degradation. Although hepatic metastases are most often responsible, retroperitoneal metastases and
bronchial, ovarian, and testicular carcinoid tumors can also cause the carcinoid syndrome. Serotonin is thought to be largely
responsible for both the diarrhea and the fibrosing cardiac lesions associated with the carcinoid syndrome. The vasomotor
changes, however, are mediated by kinins and such vasoactive peptides as substance P, neuropeptide K, neurokinin A, and
neurotensin. Other substances, such as histamine, vasoactive intestinal peptide (VIP), and prostaglandins, may also
contribute to systemic manifestations in the carcinoid syndrome. Foregut carcinoid tumors, of which stomach and bronchial
tumors are the most common, can cause atypical carcinoid syndrome. It is thought that these tumors are deficient in the
enzyme dopa-decarboxylase and have impaired conversion of 5-hydroxytryptophan (5-HTP) into 5-hydroxytryptamine (5-
HT), leading to secretion of 5-HTP into the vascular compartment. Some of the 5-HTP is converted into 5-HT and 5-
hydroxyindoleacetic acid (5-HIAA) in extrarenal sites, and some is decarboxylated in the kidney and excreted into the urine
as 5-HT; but some of the 5-HTP is excreted directly into the urine. Thus, in patients with foregut tumors, the urine contains
relatively little 5-HIAA (but more than normal) but large amounts of 5-HTP and 5-HT, in contrast to patients with midgut
carcinoid tumors in which large amounts of 5-HIAA are secreted into the urine but relatively little 5-HTP. Carcinoid tumors
of the hindgut contain no argentaffin or argyrophil cells, they have no secretory products, and therefore they are not
associated with the carcinoid syndrome. The long-acting somatostatin analog provides the best symptomatic therapy,
because somatostatin inhibits both release and action of humoral mediators of the carcinoid syndrome. By contrast,
serotonin antagonists are of little value and the efficacy of interferon therapy has yet to be established.

15. Simple screening tests for malabsorption include:


A. Microscopic examination.
B. D-xylose absorption.
C. A 72-hour stool collection for fats.
D. Small bowel x-ray series.
Answer: ABD

DISCUSSION: The 72-hour stool collection is quite sensitive and detects even mild malabsorption. As it requires careful
stool collection timed by carmen red markers and documented dietary fat intake, it is not useful for screening. Microscopic
examination of the stool can detect muscle fibers if protein malabsorption is present and with Sudan II staining can estimate
fat content. D-Xylose absorption from oral ingestion of 5 gm., detected by a blood sample after 1 hour, is a simple and quite
accurate test to identify carbohydrate malabsorption. Small bowel x-ray series, using barium contrast, can give very useful
information on mucosal abnormalities, enteric fistulas, mechanical obstructions, and very importantly, intestinal motility
and transit time.

16. Extensive resection of the small bowel, leaving only 2 or 3 feet beyond the ligament of Treitz anastomosed to the
transverse colon, can lead to the following metabolic complications:
A. Gastric hyperacidity and hypersecretion.
B. Hyperoxaluria.
C. Hypermetabolic response.
D. Fat-soluble vitamin deficiency.
Answer: ABD

DISCUSSION: Once the stress of the surgical procedure is over, there is no further hypermetabolic response, nor does there
appear to be any reduced energy expenditure from loss of the metabolically active small bowel. Energy needs are unaltered.
Gastric secretion and hyperacidity are directly related to the extent of small bowel resection and is due in part to increased
concentrations of gastrin in the serum. H 2 blockers are effective in reducing acidity and volume of gastric secretions.
Hyperoxaluria develops owing to binding of calcium to fat in the diet with steatorrhea, leaving less to bind with dietary
oxalate. The soluble oxalate is absorbed by the colon and excreted in the urine. If oxalate is excessive, oxalate kidney stones
can form. With fat malabsorption due to bile salt depletion and rapid intestinal transit, absorption of the fat-soluble vitamins
A, E, K, and D is reduced. Even with oral supplementation, deficiencies can develop.

17. Which of the following physical factors of irradiation is/are related to the potential for radiation injury?
A. The dimension of the radiation portals.
B. The number of portals.
C. The number of fractions.
D. The total amount of irradiation.
E. All of the above.
Answer: E

DISCUSSION: These physical factors are interactive. Less energy is delivered through a small portal than through a large
one. Multiple portals permit concentration of the radiation in the area to be treated and spare skin and viscera from damage.
There is less risk of injury from irradiation of a given intensity if more fractions are applied.

18. For which of the following consequences of radiation injury of the intestine is urgent laparotomy required?
A. Small bowel obstruction.
B. Colonic perforation.
C. Rectovaginal fistula.
D. Malabsorption and diarrhea.
E. Rectal stenosis.
Answer: B

DISCUSSION: Patients who have symptoms of vascular compromise or evidence of perforation require urgent laparotomy.
Patients with small bowel obstruction may require a laparotomy if a complete obstruction persists, but gastrointestinal
decompression and hydration are first steps. Patients with radiation-induced rectovaginal fistula may require temporary or
even permanent colostomy, but the first steps are evaluation and control of sepsis. Malabsorption and diarrhea can generally
be controlled pharmacologically. Rectal stenosis can usually be managed without laparotomy.
19. In addition to its absorptive and digestive roles, the small bowel also plays a significant role in the body’s immune
system. Gut-associated lymphoid tissue (GALT) represents a major division of the immune system. Which of the following
statement(s) is/are true concerning the immunologic functions of the small intestine?

a. The B lymphocytes of the small intestine do not produce immunoglobulin A (IgA)


b. Peyer’s patches, an example of an aggregated cellular portion of the gut-associated lymphoid system tissue, are
large collections of lymphoid follicles found on the antimesenteric border of the ileum
c. The major immunoglobulin of the intestinal immune system is IgM
d. IgA produced by the intestinal immune system produces the classic Fc-mediated inflammatory reactions to antigen
stimulus
Answer: b

Gut-associated lymphoid tissue (GALT) represent a major division of the immune system and is made up of aggregated
(Peyer’s patches, lymphoid follicles, mesenteric lymph nodes) and nonaggregated cellular components. The lamina propria
of the small intestine contains a wide array of nonaggregated lymphoid tissue including B cells, T cells, macrophages,
eosinophils, and mast cells. Some 80% to 99% of B cells are active producers of immunoglobulin A (IgA). In comparison,
only 2% to 5% of B cells found in other lymphoid tissues of the body secrete IgA. IgA is the major immunoglobulin of the
intestinal immune system. The functional characteristics of IgA are unlike those of other antibodies. Unlike IgG or IgM,
secretory IgA does not induce Fc-mediated inflammatory reactions. Antigen-IgA complexes do not activate the classic or
alternate complement systems, nor does IgA promote the phagocytosis of bacteria by opsonization. Most of the protective
effect of IgA derives from its ability to bind the threatening antigen efficiently, while resisting enzymatic degradation by gut
enzymes.

20. During the fasting state, a well-defined pattern of small bowel electrical activity occurs which is known as the
interdigestive myoelectric complex or the migrating motor complex (MMC). Which of the following statement(s) is/are
true concerning the MMC?

a. This complex consists of a cyclic pattern of spike bursts and muscular contractions that migrate from the
duodenum to the terminal ileum and can be divided into four phases
b. The major activity during the MMC occurs during phase I
c. In humans the MMC usually lasts less than one hour
d. Blood levels of the GI peptide, motilin, correlate closely with MMC activity and exogenous motilin can induce the
MMC front
Answer: a, d

The migrating motor complex (MMC) is a cyclic pattern of spike bursts and muscular contractions that migrate from the
duodenum to the terminal ileum. The MMC is divided into four phases: phase I-the period of quiescence with no activity;
phase II-accelerating irregular spike activity; phase III-the activity front with a series of high-amplitude, rapid spikes
corresponding to strong, rhythmic gut contractions; and phase IV-subsiding activity. In humans the cycle lasts about 90 to
120 minutes. Each phase passes in sequence along the bowel, and when the terminal ileum is reached, the process resumes
in the proximal gut. This interdigestive cycle is interrupted and replaced by rapid spiking activity (similar to phase II) when
the gut receives a food bolus. The duration of the interruption depends on the volume and nature of the food stuffs with fats
causing the largest duration of rapid spiking. Blood levels of the GI hormone, motilin, correlate closely with MMC activity
and exogenous motilin can induce the MMC front. Other hormones whose serum levels parallel MMC activity are
pancreatic polypeptide and somatostatin. Drugs that can initiate the MMC front include histamine, metoclopramide, and
morphine.

21. Which of the following statement(s) is/are true concerning the anatomy of the small intestine?

a. The second (descending), third (transverse) and fourth (ascending) portions of the duodenum lie in the
retroperitoneum and are mobilized for surgical procedures via the Kocher maneuver
b. The identification of the superior mesenteric vein and artery can be facilitated by an extensive Kocher maneuver
mobilizing the transverse portion of the duodenum and exposing the vessels as they course over the duodenum and under
the neck of the pancreas
c. In only the minority of patients can the accessory pancreatic duct (the duct of Santorini) be seen on endoscopic
exam entering the duodenum
d. The ileum is the widest portion of the small intestine, with the diameter of the small bowel progressively increasing
as the ileocecal valve is approached
Answer: a, b

The duodenum is divided into four parts-the bulb, followed by the second (descending), third (transverse), and fourth
(ascending) portion. The duodenal bulb begins at the pylorus and extends for the next 5 cm as the duodenum assumes a
retroperitoneal position for the second, third, and fourth portion. The third and fourth portion of the duodenum complete the
duodenal sweep. Mobilization of the duodenum from the retroperitoneum for a multitude of abdominal procedures can be
facilitated by the Kocher maneuver where the retroperitoneal attachment is divided and the duodenum and head of the
pancreas can be brought out of its retroperitoneal position. Endoscopically, the major papilla of the duodenum can be seen
entering at the mid-point of the second portion of the duodenum. The papilla (ampulla of Vater) appears anatomically as a
hooded fold, marking the confluence of the common bile duct and the main pancreatic duct (duct of Wirsung) and is
surrounded by the muscular sphincter of Oddi. In some 50% to 60% of patients, an accessory pancreatic duct (the duct of
Santorini) can be seen entering the duodenum proximal to the ampulla of Vater. Endoscopically, this lesser, or minor, papilla
appears as a one-to-three mm sessile polyp. The jejunum is the portion of the small bowel that courses from the ligament of
Treitz to an arbitrary point approximately two-fifths of the distance to the ileocecal valve. The length of the jejunum has
been estimated at 100 cm although this distance can vary dramatically depending on the status of the small intestine. The
jejunum is the widest portion of the small intestine, and the diameter progressively decreases as the ileocecal valve is
approached. The ileum makes up the distal three-fifths of the combined jejunal/ileal length.

22. Historically, the small intestine was presumed to have only digestive and absorptive function. However, in the last
decade the small intestine has become recognized as the body’s largest endocrine organ, producing a number of hormones,
neurotransmitters, and paracrine substances. Which of the following statement(s) is/are true concerning small bowel
hormones?

a. Cholecystokinin (CCK) is produced from cells in the mucosa of the duodenum and jejunum and is released in
response to luminal fats and proteins
b. Secretin is released in response to rising intraduodenal pH, resulting in inhibition of pancreatic secretion
c. Motilin is a 22-amino acid peptide released during the fasting state with increased levels corresponding with the
onset of the migrating motor complex (MMC)
d. Neurotensin is produced primarily in the duodenal mucosa and its release is stimulated primarily by carbohydrates
and proteins
Answer: a, c

The endocrine functions of the small intestine are diverse with an ever increasing number of hormones, peptides,
neurotransmitters, and paracrine substances identified. Cholecystokinin (CCK) is produced by cells located primarily in the
mucosa of the duodenum and jejunum and released in response to luminal fats and proteins. After CCK release from the
duodenum and jejunum, the gallbladder contracts and the sphincter of Oddi relaxes, emptying bile into the duodenum.
Secretin is found in the S cells of the duodenum and jejunum. Secretin, a true hormone, is released in response to acid in the
duodenum when luminal pH falls below 4.5. Intraduodenal secretion of pancreatic bicarbonate neutralizes duodenal pH and
results in diminished release of secretin. CCK acts in a synergistic fashion with secretin to stimulate pancreatic exocrine
function. Motilin is a 22-amino acid peptide localized in the enterochromaffin cells of the mucosa of the upper small
intestine. Motilin likely has a physiologic role in the regulation of the migrating motor complex (MMC). Motilin is released
during the fasting state, and increased levels correspond with the onset of the MMC. Neurotensin is a 13-amino acid
neurotransmitter found in the central nervous system and in the gut. Specific endocrine cells that contain neurotensin are
found in the ileal mucosa with smaller quantities found in the jejunum, stomach, duodenum, and colonic mucosa.
Neurotensin is released by a mixed meal and fats, with carbohydrates and protein releasing much smaller increments. It has
been proposed that neurotensin has a physiologic role in fat-initiated changes in gastric acid secretion, gastric emptying,
pancreatic secretion, and intestinal motility.

23. The enterohepatic circulation refers to the circular flow of bile through the small intestine and liver. Which of the
following statement(s) concerning the absorption of bile salts is/are correct?

a. The enterohepatic circulation is highly efficient with 80% to 90% of secreted bile salts reabsorbed and returned to
the liver through the portal circulation
b. The reabsorption of bile is entirely an active process
c. The small amount of bile escaping in the colon is deconjugated by bacteria, promoting lipid solubility and passive
colonic absorption
d. Ileal resection results in presenting high concentrations of bile salts to the colon which promotes diarrhea by
bacterial overgrowth
Answer: a, c

Some 80% to 90% of bile salts secreted into the small intestine as micelles are reabsorbed and returned to the liver through
the portal circulation. This circular flow of bile is termed the enterohepatic circulation. In the liver, bile salts are resecreted
and stored in the gallbladder in preparation for the next meal. The reabsorption process of bile is both passive and active.
Passive absorption occurs along the entire length of the small bowel and depends on the lipid solubility of the bile salt.
Glycine bile conjugates are more soluble than taurine conjugates. As much as 50% of bile is passively reabsorbed. Active
absorption of bile occurs only in the terminal ileum. A small amount of bile escapes into the colon, where it is deconjugated
by bacteria, promoting lipid solubility and further passive absorption. High colonic concentration of bile salts promote
diarrhea by inhibiting sodium and water absorption. This commonly occurs in patients with ileal resection and can be
treated with the bile-binding resin, cholestyramine.

24. The most obvious function of the GI tract is digestion and absorption of food for continued growth and survival of
the organism. Which of the following statement(s) is/are true concerning small bowel absorption?

a. The jejunum is the site of maximum absorption for most ingested materials with almost all jejunal absorption
performed via active transfer mechanisms
b. Eighty percent of water presented to the gastrointestinal system is reabsorbed by the small bowel
c. The absorption of carbohydrates requires digestion of large starch molecules by salivary and pancreatic amylase,
therefore presenting smaller oligosaccharides to the brush border of the jejunum to complete the digestion and absorptive
process
d. Dietary fiber represents poorly digestible carbohydrates which can absorb organic materials such as bile salts and
lipids
Answer: b, c, d

The jejunum is the site of maximum absorption of all ingested materials expect for vitamin B12. Although its mucosa
contains numerous specific transport processes, the presence of large intercellular pores produces a permeable membrane
and allows for rapid passive transfer or solutes and water. The ileum is less permeable and makes greater use of active-
transport mechanisms. Normally about 1 to 1.5 liters of water is ingested each day with another 5 to 10 liters secreted by the
GI tract in some form. About 80% of this fluid is absorbed by the small bowel. Because of this large bidirectional movement
of water, a small alteration in bowel permeability or transport can rapidly result in net secretion and diarrheal disease states.
A major source of caloric nutrition comes in the form of carbohydrate. In the Western diet, this is made up primarily of
starch (about 60%), sucrose (30%), and lactose (10%). The digestive process for starch begins with digestion of the
polysaccharide first by salivary amylase and continues with pancreatic amylase yielding smaller oligosaccharides which
along with sucrose and lactose are then presented to the brush border of the jejunum to complete the digestion and
absorptive processes. Dietary fiber consists of nondigestible carbohydrate, such as cellulose. Fiber is found commonly in
all-bran cereals, beans, partially cooked vegetables, and raw pulpy fruits. High fiber diets retain water within the bowel
lumen and significantly shorten bowel transit time. Dietary fiber can absorb organic materials such as bile salts and lipids
and inorganic materials such as zinc, calcium, magnesium and iron.

25. Which of the following statement(s) is/are true concerning the pathophysiology of small bowel obstruction?

a. Most of gas seen on plane abdominal radiographs is produced by gas forming microorganisms
b. Elevation of luminal pressure contributes to fluid accumulation in the small bowel in closed loop but not open loop
small bowel obstructions
c. Intestinal blood flow initially increases to the bowel wall in early bowel obstruction
d. In the face of obstruction, myoelectrical activity of the bowel is consistently increased
Answer: b, c

When a loop of bowel is obstructed, intestinal gas and fluid accumulate. Approximately 80% of the gas seen on plane
abdominal radiographs is attributable to swallowed air. In the setting of acute pain and anxiety, patients with intestinal
obstruction may swallow excessive amounts of air. Fluid accumulates intraluminally with open-or closed-loop small
intestinal obstruction due to a number of factors. Experimental studies and clinical investigations demonstrate that elevation
of luminal pressures above 20 cm H2O inhibits absorption and stimulates secretion of salt and water into the lumen
proximal to an obstruction. In closed-loop obstruction, luminal pressures may exceed 50 cm H2O and may account for a
substantial proportion of a luminal fluid accumulation. In simple, open-loop obstruction, distention of the lumen by gas
rarely leads to a luminal pressure higher than 8–12 cm H2O. Thus, in open-loop obstruction, the contributions of high
luminal pressures to hypersecretion may not be important. In response to heightened luminal pressure, total blood flow to
the bowel may initially increase. Subsequently, however, blood flow to the bowel is compromised as luminal pressures
increase, bacteria invade, and inflammation leads to edema within the bowel wall. Accumulation of gas and fluid in the
obstructed lumen also leads to changes in myoelectrical function in the gut, proximal and distal to the obstructed segment.
In response to distension, the obstructed segment itself may dilate, a process known as “receptive relaxation.” At sites
proximal and distal to the obstruction, changes in myoelectrical activity are time-dependent. Initially, there may be intense
periods of activity and peristalsis. Subsequently, myoelectrical activity is diminished and interdigestive migrating
myoelectrical complex (MMC) is replaced by ineffectual and seemingly disorganized clusters of contractions.

26. A 45-year-old man with a history of previous right hemicolectomy for colon cancer presents with colicky
abdominal pain which has become constant over the last few hours. He has marked abdominal distension and has had only
minimal vomiting of a feculent material. His abdomen is diffusely tender. Abdominal x-ray shows multiple air fluid levels
with dilatation of some loops to greater than 3 cm in diameter. The most likely diagnosis is:

a. Proximal small bowel obstruction


b. Distal small bowel obstruction
c. Acute appendicitis
d. Closed-loop small bowel obstruction
Answer: b

Distinguishing the various types of bowel obstruction can be difficult based on history, physical findings, and radiographic
studies. The patient described has intermittent to constant pain with low volume feculent vomiting. Distension is marked
and progressive, and tenderness is diffuse. This scenario most likely fits with an open-loop distal small bowel obstruction.
The feculent vomiting suggests a more distal rather than proximal obstruction. The lack of severe pain and signs of
peritoneal irritation suggests that a closed-loop obstruction is unlikely. A colon obstruction with an incompetent ileocecal
valve would be another alternative to consider if gas in the colon had been seen on x-ray.

27. In the patient described above, the following statement(s) is/are true concerning the possible etiology of bowel
obstruction.

a. Simple obstruction secondary to an adhesion is most likely to resolve nonoperatively


b. It is most likely that the patient’s obstruction is secondary to recurrent malignancy
c. A history of colon cancer makes carcinomatosis the most likely diagnosis
d. Lower abdominal procedures are more likely to result in obstructive adhesions than are upper abdominal
procedures
Answer: a, d

Peritoneal adhesions account for more than half of small bowel obstruction cases. Lower abdominal procedures such as
appendectomy, hysterectomy, and abdominal perineal resection are common precursor operations to account for obstruction
although adhesions may follow any abdominal procedure including cholecystectomy, gastrectomy, and abdominal vascular
procedures. Simple adhesive obstruction is distinguished from other forms of obstruction by the capacity to resolve without
surgical intervention. In recent surveys, as many as 80% of episodes of small bowel obstruction due to adhesions may
resolve nonoperatively. The likelihood that an obstruction is due to recurrent malignancy relates to several factors including
the origin of the primary malignancy, the stage of the primary malignancy, and the designation of original surgery as
curative or palliative. Gastric and pancreatic cancers often present with, or are subsequently complicated by peritoneal
carcinomatosis and subsequent obstruction. With respect to colon and rectal carcinomas, as many as 50% of cases
presenting with obstruction after resection of the primary may be due to adhesions and not recurrent malignancy.

28. Which of the following statement(s) is/are true concerning laboratory tests which might be obtained in the patient
discussed above?

a. The presence of a white blood cell count > 15,000 would be highly suggestive of a closed-loop obstruction
b. Metabolic acidosis mandates emergency exploration
c. An elevation of BUN would suggest underlying renal dysfunction
d. There is no rapidly available test to distinguish tissue necrosis from simple bowel obstruction
Answer: d

There have been multiple attempts to use common clinical laboratory test criteria to identify the likelihood that obstruction
is associated with strangulation. In most cases of simple obstruction, laboratory studies do not play a direct role in diagnosis
but are helpful in understanding the extent of complications such as dehydration and fluid and electrolyte abnormalities. An
elevation of the white blood cell count along with fever, tachycardia, and localized abdominal tenderness is one of the
“cardinal signs” for risk for strangulation. However, such an elevation is nonspecific. Similarly, metabolic acidosis may be
associated with intestinal ischemia as well as evidence of dehydration and fluid loss. Elevation of BUN and other electrolyte
abnormalities also represent fluid loss and dehydration. Therefore, at present there is no non-invasive rapid laboratory tests
that can provide information to suggest that tissue necrosis is eminent.

29. The patient discussed above was admitted to the hospital and after 24 hours remained distended with no evidence
of resolution. Which of the following radiographic studies would be considered appropriate at this time?

a. Contrast enema
b. Enteroclysis study with dilute barium
c. CT scan with dilute barium oral contrast
d. None of the above
Answer: a, b, c, d

Contrast studies such as those listed above may provide specific localization at the point of obstruction and the nature of the
underlying lesion. When obstruction of the small intestine is not progressively resolving, a small bowel follow-through is
indicated to confirm the presence and location of the obstruction. The history of a previous right hemicolectomy in this
patient may also allow reflux through the colon to define the ileocolonic anastomosis and be able to define the site of
obstruction in a retrograde fashion. The potential benefits for a CT scan include not only defining the obstruction and
perhaps the nature of the lesion, but also in defining any other evidence of abdominal pathology such as metastases, ascites,
or parenchymal liver abnormalities which might be present in a patient with a previous neoplasm. Although none of these
tests would be contraindicated, failure of this patient to improve will likely mandate an operation and make contrast studies
unnecessary. There would appear to be no evidence of strangulation or perforation therefore there are no contraindications
to these studies.

30. A 75-year-old woman is hospitalized after a fall in which she has experienced a hip fracture. Several days after her
surgical procedure, progressive painless abdominal distension is noted. Which of the following statement(s) is/are true
concerning her diagnosis and management?

a. Colon distension with a cecal diameter in excess of 12 cm should indicate the need for urgent operation
b. Endoscopic decompression may be attempted but seldom is successful
c. After successful colonoscopic decompression, recurrence is unlikely
d. A rectal tube as the primary treatment is generally not successful
Answer: d

Acute pseudo-obstruction of the colon, known as Ogilvie’s syndrome, is a paralytic ileus of the large bowel characterized by
rapidly progressive abdominal distension often without associated pain. Plane radiographs of the abdomen may reveal air in
the small bowel and distension of discrete segments of the colon (cecum or transverse colon) or the entire abdominal colon.
Distension can become impressive, oftentimes in chronic cases distension in excess of 15 cm can be observed without
evidence of colon perforation or wall ischemia. Major risk factors for the development of Ogilvie’s syndrome include severe
blunt trauma, orthopedic trauma or procedures, acute cardiac events or coronary bypass surgery, acute neurologic events or
neurosurgical procedures, and acute metabolic derangements. Initial management includes resuscitation and correction of
the underlying metabolic and electrolyte abnormalities. A nasogastric tube is indicated if the patient is vomiting and will
prevent swallowed air from passing distally. If distension is painless and the patient shows no signs of toxicity or bowel
ischemia, expectant management can be successful in about 50% of cases. If distension worsens so that the cecal diameter
increases beyond 10–12 cm or if it persists for more than 48 hours, colonoscopy is recommended. Endoscopic
decompression is successful in 60–90% of cases, but colonic distension may recur in up to 40% of cases. Rectal tubes are
ineffective in managing distension of the proximal colon, however, such tubes may be useful after colonoscopy.
31. Which of the following statement(s) is/are true concerning the etiology of intestinal obstruction?

a. In the United States, peritoneal adhesions account for over half of the cases of small bowel obstruction
b. A leading cause of bowel obstruction is early postoperative adhesions
c. Bowel obstruction cannot occur with a Richter’s hernia
d. Ninety percent of adult cases of intussusception are associated with a pathologic process, most commonly a tumor
Answer: a, d

Peritoneal adhesions account for more than half of the cases of small bowel obstruction in the United States. Obstruction in
the immediate postoperative period following abdominal surgery, however, is uncommon, occurring in only 1% of patients
in the four weeks following laparotomy. Hernias of all types are second only to adhesions as the most frequent cause of
obstruction. External hernias such as inguinal or femoral hernias may present with symptoms of obstruction. Femoral
hernias are particularly prone to incarceration and bowel necrosis, due to the small size of the hernia inlet. One important
consideration is the Richter’s hernia. In this variant, only a portion of the bowel wall is incarcerated. These most frequently
occur in association with femoral or inguinal hernias. Complete obstruction can occur if more than half to two-thirds of the
bowel circumference is incarcerated. About 5% of intussusception cases occur as adults. Intussusception occurs when one
segment of bowel telescopes into an adjacent segment, resulting in obstruction and ischemic injury to the intussuscepting
segment. Ninety percent of adult cases are associated with pathological processes. Tumors, benign and malignant, can act as
a lead point against the sussesception in over 65% of adult cases.

32. Which of the following statement(s) is/are true concerning postoperative ileus?

a. The use of intravenous patient-controlled analgesia has no effect on return of small bowel motor activity
b. The presence of peritonitis at the time of the original operation delays the return of normal bowel function
c. The routine use of metoclopramide will hasten the return of small intestinal motor activity
d. Contrast radiographic studies have no role in distinguishing early postoperative bowel obstruction from normal
ileus
Answer: b

The term ileus reflects the underlying alterations in motility of the gastrointestinal tract, leading to functional obstruction.
From a practical standpoint, ileus represents the interval between abdominal exploration and the reappearance of flatus and
bowel movements. Distinguishing a normal postoperative ileus and the prolonged course of a “paralytic” ileus is based
primarily on the time since operation and the clinical circumstances. Besides the location of the previous operation (upper
abdominal, lower abdominal, pelvic., the nature of the previous operation and the findings may also contribute. Peritonitis
or spillage of noxious material leads to an increase in the delay of return of normal bowel function. Distinguishing a
paralytic ileus from mechanical obstruction can oftentimes be difficult. Abdominal x-rays in a postoperative ileus should
reveal gas in segments of both the small and large bowel. Upper GI contrast or CT scan may also be helpful. Early
postoperative obstruction is uncommon and is particularly rare for upper abdominal surgery, with most cases occurring after
surgery of the colon, particularly abdominal perineal resection. There has been little success in the use of prokinetic agents
to shorten recovery times after lower abdominal procedures. The use of intravenous patient controlled analgesia may delay
the recovery of postoperative ileus when compared to the IM route of narcotic administration.

33. The initial management of this patient should consist of:

a. Fluid resuscitation with D5 half normal saline with 40 mEq of potassium chloride/liter
b. Placement of an indwelling urinary catheter
c. Nasogastric decompression with a nasogastric tube
d. Immediate surgery
e. The patient should be begun on broad spectrum antibiotics at the time of admission
Answer: b, c

The principles of management of a patient with small bowel obstruction include initial fluid resuscitation and restricting oral
intake. The optimal fluid for resuscitation in this patient with a distal small bowel obstruction would likely be Ringer’s
lactate or normal saline. Since gastric secretion is a small component of the fluid loss, potassium replacement is likely not
particularly important. An indwelling urinary catheter should be placed to monitor the urine output to reflect the fluid status.
Invasive hemodynamic monitoring with a central line is likely unnecessary unless concerns are raised about cardiac status.
Nasogastric decompression is indicated in all but mild cases. The nasogastric tube serves to prevent distal passage of
swallowed air and minimizes discomfort of reflux of intestinal contents and eliminates vomiting. There appears to be no
clinical evidence suggesting the need for urgent operation and therefore resuscitation prior to surgery is of optimal
importance in this patient.
It has been well established that perioperatively-administered antibiotics reduce wound infection and abdominal sepsis rates
in patients undergoing operation to relieve intestinal obstruction, simple or strangulated. Once the decision has been made to
proceed with surgery, broad spectrum antibiotics, covering gram-negative aerobes and anaerobes should be given. The use
of antibiotics in patients who have not been committed to operation has not been evaluated systematically. Giving
antibiotics to patients who are being observed can obscure the underlying process and, in the end, delay optimal therapy.

34. An 82-year-old female nursing home resident is admitted with massive abdominal distension and constant
abdominal pain with diffuse tenderness. Abdominal x-ray shows a massively distended loop of colon with a characteristic
“bent inner tube” appearance. The management of this patient should include:

a. Urgent laparotomy because of the massive colon distension


b. An attempt at endoscopic decompression with a flexible sigmoidoscope
c. Elective laparotomy and sigmoid resection should follow if endoscopic decompression is successful
d. If at urgent laparotomy resected bowel is present, colon resection with primary anastomosis is in order
Answer: b, c

The most common site of volvulus is the sigmoid colon, accounting for 65% of cases. The preferred method and
management involves endoscopic decompression. This conservative approach resolves the volvulus in 85% to 90% of cases,
and elective resection of the redundant segment can then be planned. Following endoscopic decompression, recurrence of
the volvulus is higher than 60% if sigmoid resection is not performed. If the patient presents with peritoneal findings, sepsis,
and shock, rapid resuscitation followed by urgent resection and colostomy is warranted.

35. A common manifestation of Crohn’s disease is perianal disease, including anal fistulas with extension to adjacent
organs and soft tissue regions, fissures, and perirectal abscesses. Which of the following statement(s) is/are true concerning
perianal disease with Crohn’s disease?

a. Perianal disease is the initial mode of presentation in the majority of patients


b. The prevalence of perianal disease is increased in patients with either ileocolitis or isolated colonic involvement
c. Metronidazole has been shown to be effective in the treatment of perianal disease secondary to Crohn’s
d. An aggressive surgical approach is appropriate in most cases due to the frequent rapid progression of perianal
disease
Answer: b, c

A common manifestation of Crohn’s disease is perianal disease, including anal fistulas with extension into the adjacent
organs and soft tissue regions, fissures, and perirectal abscesses. The prevalence of perianal disease approaches 25% for
patients with ileitis, 50% for ileocolitis, and 40% for those with isolated colonic involvement. Perianal disease is one of the
initial signs of presentation in one-third of patients. Although broad spectrum antibiotics are clearly indicated for septic
complications of Crohn’s disease, their use as a primary treatment has generally been met without success. Metronidazole
has been used effectively in the treatment of perianal disease. In general, a conservative surgical approach to perianal
disease is usually prudent. Many patients who have indolent anal fistulas can live comfortably with their disease for years.
Although the development of an abscess requires conventional drainage depending on the state of Crohn’s involvement of
the rectum, standard surgical procedures can be applied to most forms of perirectal and perianal disease. Proctectomy may
be indicated for patients with advanced perianal disease in direct continuity with active rectal involvement.

36. Nongastrointestinal complications of Crohn’s disease include:

a. Renal calculi
b. Cholelithiasis
c. Arthritis
d. Anemia
Answer: a, b, c, d
Although Crohn’s disease is primarily a disease involving the alimentary tract, involvement of extraintestinal tissues (joints,
skin, and eyes) is common and indicates that Crohn’s disease is a systemic disorder rather than a localized intestinal disease.
In addition to specific processes, secondary consequences of impaired intestinal absorption and resulting malnutrition
include anemia due to specific deficits in vitamins, trace elements, and bile acids and electrolytes. Growth retardation and
delayed bone maturation are present in 10–40% of children and adolescents with this disease. Patients with terminal ileal
disease are also prone to develop renal urate or oxalate stones. Furthermore, as the result of altered bile salt metabolism and
the development of lithogenic bile,
patients with ileal disease and ileal resections are also at risk for cholelithiasis.

37. Which of the following points is/are true concerning the diagnosis of Crohn’s disease?

a. Recurrent disease on contrast radiographs frequently lags behind the development of clinical signs and symptoms
b. In 10% of cases, Crohn’s disease cannot be distinguished from chronic ulcerative colitis based on clinical,
radiologic, and pathologic criteria
c. Although no specific laboratory tests exist for Crohn’s disease, the erythrocyte sedimentation rate has evolved as a
useful measure of disease activity
d. Specific endoscopic features encountered in Crohn’s disease which allow differentiation from ulcerative colitis
include aphthous ulcers, cobblestoning, and skip areas

Answer: b, c, d

A number of laboratory and radiographic studies as well as the role of endoscopy and biopsy are useful in the diagnosis and
assessment of Crohn’s disease. Although no specific laboratory test exists for Crohn’s disease, acute-phase protein levels
and erythrocyte sedimentation rate have evolved as measures of disease activity and severity. Endoscopic examination of
the colon and rectum is often performed early in the diagnostic workup. In the presence of colorectal involvement, specific
endoscopic features encountered which allow differentiation from ulcerative colitis include: aphthous ulcers, linear ulcers,
cobblestoning, and asymmetric and discontinuous involvement. The radiologic examination is essential for differential
diagnosis in delineating the extent or the severity of the disease primarily involving the small bowel. Barium contrast
studies will disclose a number of specific features in patients with Crohn’s disease. A correlation, however, between the
extent of the disease seen radiographically and clinical symptoms does not exist. Recurrent disease after surgical resection is
often apparent radiologically before the development of clinical signs and symptoms.
The most important differential diagnosis is between Crohn’s disease and chronic ulcerative colitis, especially when the
information is limited to the colon and rectum. Despite extensive clinical, radiologic, and pathologic evaluation, 5% to 10%
of patients will be defined as having indeterminant colitis without clear-cut evidence of either condition.

38. The following statement(s) is/are true concerning the surgical management of Crohn’s disease.

a. Strictureplasty, although offering short-term benefits, is associated with a higher rate of recurrence when compared
to resection
b. Frozen section examination of the margin of resection is essential to prevent both recurrent disease and early
anastomotic complications
c. Conservative margins of resection are appropriate, resecting only grossly involved segments of bowel
d. Patients with Crohn’s disease confined to the colon may be treated with total proctocolectomy with construction of
an ileal-anal pouch anastomosis
Answer: c

Surgical therapy for Crohn’s disease is curative not palliative, therefore is reserved for complications of the disease or
failure of or debilitation, secondary to medical therapy. The lines of bowel resection should be chosen conservatively with
only a few centimeters proximally and distally to the site of visible changes of Crohn’s disease. Microscopic evidence of
Crohn’s disease at the resection margins does not compromise safe anastomosis and therefore frozen section examination of
resection margins is not necessary. In patients with multiple strictures of the small bowel, resection may involve excessive
resection of bowel. Therefore, strictureplasty is an appropriate surgical therapy. Long-term results using this approach
indicate that recurrence rates are not substantially increased with strictureplasty, even though inflamed intestinal tissue is
left in situ. In patients with diffuse disease of the colon or rectum, proctocolectomy with ileostomy is the treatment of
choice. Both the risk of ileal involvement and transmural involvement of the rectum precludes the technique of ileal pouch-
anal reconstruction in patients with Crohn’s disease.
39. The etiology of Crohn’s disease is unknown, although two major hypotheses have evolved: an infectious and an
immunologic theory. The following statement(s) is/are true concerning the possible etiology of Crohn’s disease.

a. The leading infectious agent thus far suggested is infection with a Mycobacterium species
b. Strong evidence linking viral pathogens to Crohn’s disease has been developed
c. Although many alterations in cellular and immune functions in patients with Crohn’s disease have been observed,
no primary defect in the immune system has yet been identified
d. The identification of antibodies to enterocytes provides strong support for the theory that Crohn’s disease is an
autoimmune process
Answer: a, c

Investigations for the last 60 years have tried to determine the etiology of Crohn’s disease. Although a number of theories
have evolved and evidence is available supporting numerous theories, there is no conclusive evidence to support any
etiologic theory for its development. Given the characteristic histologic findings of granuloma formation, early
investigations focused on bacterial causes of Crohn’s disease, most notably infection with Mycobacterium species. Several
reports have isolated Mycobacteria from mesenteric lymph nodes and intestine involved in Crohn’s disease but have not
proven the Mycobacterial cause. Similarly, research in viral causes has been inconclusive, and although viral pathogens
have been isolated from tissue extractions with Crohn’s disease, linkage to induction and persistence of the disease has not
been convincing. Similar difficulties exist in theories concerning immunogenetic causes of Crohn’s disease, and although
many alterations in cellular and immune functions associated with Crohn’s disease have been observed, no primary defect,
either systemic or mucosal, humoral or cellular has been identified. A number of reports have described antibodies and
lymphocyte reactivity to enterocytes, however the presence of antibody cannot be correlated with disease activity and
furthermore antibodies have been found in patients with other diseases and in healthy volunteers.

40. Crohn’s disease is an incurable disease, therefore recurrence after surgical resection is likely. Which of the
following statement(s) regarding the recurrence of Crohn’s disease is/are accurate?

a. Endoscopic evidence of recurrence is present in less than 50% of patients at five years
b. Radiographic or endoscopic evidence of recurrence is frequently not accompanied by symptoms
c. Clinical recurrence of Crohn’s disease is seen in 20% of patients at two years, and 40–50% at four years after
surgery
d. Reoperation for Crohn’s disease is necessary in the majority of patients by five years
e. No solid evidence demonstrating prolongation of remission can be seen with corticosteroids, sulfasalazine, or
antibiotics
Answer: b, c, e

The majority of patients with Crohn’s disease will recur. If recurrence is defined as alterations detected endoscopically, then
70% will recur within one year of surgery, and 85% within three years. However, in most of these patients clinical
symptoms will not accompany the endoscopic or radiographic evidence of disease. A clinical recurrence (return of
symptoms) confirmed as Crohn’s disease radiologically, endoscopically, or surgically, affects 20% of patients at two years,
and 40–50% at four years after surgery. Reoperation becomes necessary in about 30% of the patients by five years. These
statistics give impetus to maintain remission and prevent recurrence. Although it is common practice to stem recurrence
with sulfsalazine, 5-ASA preparations, antibiotics, and possibly azathioprine, none of these (possibly excepting
azathioprine) have definitely been proven effective.

41. Which of the following statement(s) is/are true concerning drug therapy for Crohn’s disease?

a. Corticosteroids have been demonstrated to effectively treat acute exacerbations and to prolong remission in
patients with Crohn’s disease
b. Sulfasalazine is indicated primarily for the treatment of patients with acute exacerbations of Crohn’s disease
involving the small bowel
c. Azathioprine, an immunosuppressant, has been shown to be effective in maintaining remission of Crohn’s disease
d. Low dose cyclosporine has significant therapeutic benefit for patients with both low and high disease activity
Answer: c
Systemic corticosteroids have been used to treat Crohn’s disease since the 1940s. Although the exact mechanism of action is
not clear, nonspecific immunosuppression is the likely effect. Several well designed trials have demonstrated that
Prednisone (or its equivalent) is effective in the treatment of acute exacerbations. Patients with quiescent disease, or patients
who have received remission through medical or surgical therapy, however, do not benefit from long-term continued
corticosteroids. Sulfasalazine consisting of a sulfonamide linked to an aspirin analogue (5-ASA) is more effective than
placebo in the treatment of acute disease. This agent, however, is most effective in patients with predominantly colonic
disease and is less effective than corticosteroids in treating patients with small bowel disease. Asymptomatic patients do not
appear to benefit from prophylactic treatment. The immunosuppressive agent azathioprine, which acts to inhibit nucleic acid
metabolism, has been demonstrated to be highly effective in long-term use. The use of azathioprine has a steroid-sparing
effect with reduction of steroid dose or discontinuation of therapy. In chronic treatment, azathioprine is effective in
decreasing disease activity, steroid requirements, and complications leading to surgery, therefore, in contrast to
corticosteroids and sulfasalazine, azathioprine appears effective in maintaining remission. Side-effects, however, can be
significant including bone marrow suppression and acute pancreatitis. Finally, cyclosporine, an immunosuppressant, has
undergone extensive review with the conclusion that low-dose oral cyclosporine treatment confers no therapeutic benefit for
patients with low or high disease activity and in no reduction in the need for other forms of therapy.

42. Which of the following are predominant histologic features of Crohn’s disease?

a. The presence of granulomas involving the bowel wall and mesenteric lymph nodes
b. Transmural inflammation
c. Fissures and ulceration extending into the muscularis propria
d. Chronic fibrotic changes
Answer: a, b, c, d

Crohn’s disease can affect any part of the gastrointestinal tract with the most common site being the ileocecal region. The
acute, active phase is marked by aphthous mucosal ulcerations, lymphoid aggregates, and granulomas present in both the
bowel wall, adjacent lymph nodes, and in other organs. Transmural inflammation is present with characteristic fissures and
ulcers extending deep into the muscularis propria. The acquiescent or healing phase of Crohn’s disease is marked by fibrosis
with late stricture formation and chronic ulceration.

43. The following statement(s) is/are true concerning the epidemiology of Crohn’s disease.

a. Crohn’s disease has an age distribution with peaks between the ages of 15 and 30 years and 65 and 75 years
b. There is a definite female predilection for Crohn’s disease
c. The disease is equally prevalent in industrialized versus underdeveloped countries
d. First and second generation relatives with Crohn’s disease have an increased prevalence when compared to the
general population
Answer: d

Crohn’s disease arises most commonly between the ages of 15 and 30 years, with a second peak at 55 to 60 years. Men and
women are equally affected. The disease is seen more commonly in urban residents than rural dwellers and is associated
with higher levels of education. The disease is almost exclusively encountered in industrialized nations like Western Europe
and the United States which suggests that environmental factors are important in the pathogenesis. Aggregation in families
can occur with first-and second-generation relatives of patients with Crohn’s disease found to have a 10-and 3-fold increase,
respectively, in the prevalence of Crohn’s disease when compared to other non-related individuals.

44. The management of adenocarcinoma of the small intestine depends primarily on tumor location. Which of the
following statements concerning surgical management are true?

a. Radical pancreaticoduodenectomy (Whipple resection) is necessary for resection of most duodenal


adenocarcinomas
b. Adenocarcinomas of the jejunum or ileum are managed by limited segmental resection including resection of the
mesentery down to the first vascular arcade
c. Distal ileal carcinomas are best managed by right hemi-colectomy to include lymph node chains along the ileo-
cecal blood supply
d. Small invasive adenocarcinomas of the ampulla and peri-ampullary duodenum can frequently be managed by local
excision
Answer: a, c

Optimal surgical treatment of adenocarcinoma of the small intestine requires wide, segmental resection, including the
draining nodal system. For most duodenal adenocarcinomas, a radical pancreaticoduodenectomy (Whipple procedure) is
necessary to incorporate pertinent training lymph nodes. Although local excision of villous adenomas of the periampullary
area has been reported, the presence of invasive carcinoma warrants wider resection as a pancreaticoduodenectomy. Jejunal
and ileal carcinomas are removed with segmental resections with adequate margins on the bowel and wide resection of the
mesentery with associated lymph nodes down to the superior mesenteric artery. Distal ileal carcinomas are drained by
lymph nodes along the ileocolic artery and are best managed by right hemicolectomy.

45. The management of carcinoid tumors must be individualized based on the findings at surgery. Which of the
following is/are components of optimal care?

a. Limited segmental resection without lymphadenectomy


b. Careful exploration of the remaining small bowel and colon
c. Non-anatomic resection of small multiple liver metastases
d. Postoperative adjuvant chemotherapy for all carcinoid tumors regardless of size or level of invasion
Answer: b, c

Operative management of a primary small bowel carcinoid tumor involves principals similar to those of small bowel
carcinomas. Wide en bloc excision should include as many lymphatic drainage pathways as possible because of their
frequent metastatic involvement. Because of the increased incidence of both multicentricity and a second unrelated
malignancy, a diligent search for other primary carcinoids of the small bowel and for other synchronous malignancies of
other organs is imperative. When localized hepatic metastasis are amenable to resection, hepatic resection should be
considered to minimize the potential development of Carcinoid Syndrome. Adjuvant postoperative chemotherapy for
patients with metastatic carcinoid tumor is of modest benefit with response rates in the 20–30% range with median duration
response short-lived. At present, adjuvant therapy is confined only to those patients with Carcinoid Syndrome.

46. An increased evidence of adenocarcinoma of the small intestine has been established with which of the following
conditions?

a. Peutz-Jegher Syndrome
b. Crohn’s disease
c. Simple tubular adenomas of the small intestine
d. Colon carcinoma
Answer: b

The incidence of adenocarcinoma of the small bowel is surprisingly low when compared to that of colon carcinoma when
considering the vast length and surface area of the small intestine. The relative infrequency of these tumors has limited our
knowledge of pathogenic factors. It appears, however, that the polyp-to-cancer sequence is not well established for simple
tubular adenomas of the small intestine. Peutz-Jegher Syndrome is an inherited syndrome which is associated with multiple
small intestinal polyps. These polyps are hamartomas with progression of dysplasia to carcinoma felt not to be a major
concern. The chronic inflammatory changes of Crohn’s disease appears to predispose to the development of
adenocarcinoma, thereby increasing the risk to 100 times that of the general population. There is no known association of
carcinoma of the colon and small intestine.

47. A 60-year-old male presents with nonspecific symptoms of fatigue, malaise, weight loss and abdominal pain.
Barium small bowel series shows a limited segment of small intestine with thickened mucosal folds and partial obstruction.
CT scan confirms small intestinal wall thickening and suggests the presence of bulky mesenteric lymph nodes. Which of
the following is/are components of optimal care?

a. Attempts at percutaneous biopsy of the mesenteric mass


b. Surgical exploration with aggressive resection of the localized disease including wide, en bloc lymphadenectomy
c. Liver biopsy and sampling of periaortic and mesenteric lymph nodes outside the field of resection
d. Splenectomy
Answer: b, c
Most patients suspected of having small intestinal lymphoma require operation with the goals of treatment including
diagnosis, staging, relief of obstruction and perforation, and resection or debulking. Because intraoperative staging affects
postoperative management, liver biopsy and sampling of periaortic and mesenteric lymph nodes outside the field of
resection are important aspects of the operative management. In contrast there is no role for splenectomy for primary small
bowel lymphoma. For localized disease, aggressive resection with wide, en bloc lymphadenectomy is important.
Percutaneous biopsy has no role in such cases both because the preoperative diagnosis will not eliminate the need for
surgical intervention, and percutaneous biopsy is frequently inadequate to determine necessary information for the treatment
of lymphoma.

48. Malignant neoplasms of the small bowel tend to have a characteristic anatomic distribution. Which of the
following statements are true?

a. Adenocarcinomas of the small intestine show a distinct polarity with decreasing frequency from duodenum to
ileum
b. Adenocarcinoma of the small intestine associated with Crohn’s disease occurs primarily in the ileum
c. Lymphomas of the small intestine arise primarily in the jejunum
d. The vast majority of carcinoid tumors of the small intestine occur in the ileum
Answer: a, b, d

Anatomically, adenocarcinomas of the small intestine show a distinct polarity with a decreasing frequency from duodenum
to ileum. Given the difference in length between the duodenum, jejunum and ileum, the duodenal epithelium shows a
substantially greater propensity towards malignant transformation. Even within the duodenum, two-thirds of the carcinomas
occur in the periampullary region suggesting that the periampullary mucosa or luminal content (ingested potential
carcinogens) interacts with pancreaticobiliary secretions to induce local neoplastic changes. Crohn’s disease primarily is a
disease involving the terminal ileum and therefore it is not surprising that most adenocarcinomas developing in association
with Crohn’s disease occur also at this site. Small intestinal lymphomas arise from lymphoid tissue within the wall of the
bowel. Therefore lymphomas predominate in the ileum, where the greatest concentration of gut lymphoid tissue occurs.
Next to the appendix, which harbors 85% of all carcinoid tumors, the small intestine is by far the next most common site of
origin. Approximately ninety percent of small intestinal carcinoids are located in the ileum with 40% found within two feet
of the ileocecal junction. Multiple primary tumors may be present in 30% of patients.

49. Small intestinal carcinoids may present in a multitude of fashions. Which of the following may be seen as a
presentation of carcinoid tumors of the small intestine?

a. Intestinal obstruction
b. Gastrointestinal bleeding
c. Small intestinal infarction
d. Asymptomatic
Answer: a, b, d

Many small intestinal carcinoids are small and asymptomatic and are found only incidentally or at autopsy. Clinical
symptoms can arise either from the primary tumor, from sequelae of metastatic disease, or from the Carcinoid Syndrome.
Obstructive symptoms can occur either from intussusception or more commonly as a submucosal tumor infiltrates the bowel
wall and beyond, the bowel mesentery may become shortened, thickened and fixed by an intense desmoplastic reaction
characteristic of carcinoid tumors. This leads to kinking and angulation of intestinal loops and may eventually result in
mechanical obstruction. Intestinal ischemia or even infarction can occur secondary to an unusual type of mesenteric
angiopathy characterized by vascular thickening and sclerosis that accompanies the desmoplastic mesenteric reaction.
Although mucosal ulceration and bleeding can occur, such symptoms are unusual.

50. With regard to benign neoplasms of the small intestine, which of the following are true statements?

a. Many are asymptomatic and only found as incidental findings


b. Leiomyomas are the most common symptomatic benign neoplasm and may present with gastrointestinal bleeding
c. Villous adenomas carry a distinct malignant potential and occur most commonly in the periampullary duodenum
d. Peutz-Jegher Syndrome is associated with multiple adenomatous polyps throughout the small intestine
Answer: a, b, c

The most common benign small intestinal neoplasms are adenomas, leiomyomas, and lipomas. Hamartomas, fibromas,
angiomas, and neurofibromas may also occur at a lesser frequency. Three types of adenomas occur, simple tubular
adenomas, villous adenomas, and Brunner gland adenomas. Tubular adenomas have a very low malignant potential whereas
villous adenomas carry a distinct malignant potential similar to that of colonic villous adenomas. These adenomas occur
most commonly in the duodenum and especially in the periampullary region. Brunner gland adenomas represent hyperplasia
of the exocrine glands within the proximal duodenal mucosa and have little risk of malignant change. The smooth muscle
tumor, leiomyoma, are the most common symptomatic benign neoplasms. Most leiomyomas enlarge with an extraluminal
orientation and, may reach considerable size. The tumors eventually may outgrow their blood supply leading to central
necrosis, ulceration and intraluminal bleeding. Rupture of the tumor may also occur with intraperitoneal bleeding.
Differentiation of larger leiomyomas from their malignant counterpart, leiomyosarcoma, may be difficult on pathologic
review. Peutz-Jegher Syndrome is an inherited syndrome associated with multiple gastrointestinal polyps throughout the
jejunum and ileum. Histologically these polyps are hamartomas and not adenomas and therefore offer little risk of malignant
transformation.

51. Primary gastrointestinal lymphomas involving the small bowel are uncommon accounting for less than 5% of all
lymphomas. Conditions associated with small intestinal lymphomas include which of the following?

a. Acquired immune deficiency syndrome (AIDS)


b. Celiac disease
c. Crohn’s disease
d. Rheumatoid arthritis
Answer: a, b, c, d

Although rare, small intestinal lymphomas are associated with several conditions. The chronic malabsorptive condition,
celiac disease, is recognized to be associated with small intestinal lymphoma. Clinical deterioration in a patient with
previously controlled celiac disease should immediately suggest the diagnosis of lymphoma. There is also an increased
incidence of lymphoma in Crohn’s disease. Disorders of immunologic function have an increased incidence of extranodal
gastrointestinal lymphoma. These disorders include autoimmune diseases such as rheumatoid arthritis, Wegener
granulomatosis, systemic lupus erythematous, and congenital immunodeficiencies. Immunosuppressed patients after organ
transplantation and patients with prolonged, high-dose chemotherapy are also at increased risk. AIDS has been associated
with the development of aggressive, non-Hodgkin’s lymphoma presenting with primary gastrointestinal involvement.
Although this is usually a diffuse systemic disease, extranodal lymphoma of the small bowel has been frequently
recognized.

1. Which answers are true? In contrast to ulcerative colitis, Crohn's disease of the colon:
A. Is not associated with increased risk of colon cancer.
B. Seldom presents with daily hematochezia.
C. Is usually segmental rather than continuous.
D. Has a lower incidence of perianal fistulas.
E. Never develops toxic megacolon.
Answer: BC

DISCUSSION: Crohn's disease of the colon is a patchy, segmental, chronic, transmural inflammatory process that
penetrates the bowel wall to form fistulas but seldom causes rectal bleeding. In contrast, ulcerative colitis is a mucosal
ulcerating process that extends continuously from the rectum to the more proximal colon and frequently bleeds. Both
diseases can develop toxic megacolon, and both predispose the patient to increased risk of malignancy of the large intestine
over the long term.

2. Which answers are true? Options to consider when operating for Crohn's disease of the large intestine include:
A. Colectomy and ileorectostomy.
B. Colectomy, closure of the rectal stump, and ileostomy.
C. Colectomy and continent ileostomy (Kock pouch).
D. Proctocolectomy and ileostomy.
E. Proctocolectomy and ileal pouch–anal canal anastomosis.
Answer: ABD

DISCUSSION: Patients with colonic Crohn's disease who have minimal or mild rectal involvement can be treated by
colectomy and ileorectostomy or by colectomy, closure of the rectal stump, and ileostomy. When severe rectal involvement
is also present, proctocolectomy with permanent ileostomy is required. The Kock pouch (continent ileostomy) and the ileal
pouch–anal canal operation are not performed for Crohn's disease because of the risk of recurrence of Crohn's disease in the
ileal pouch in the postoperative period.

3. Crohn's disease:
A. Is caused by Mycobacterium paratuberculosis.
B. Is more common in Asians than in Jews.
C. Tends to occur in families.
D. Is less frequent in temperate climates than in tropical ones.
E. Is improved by smoking.
Answer: C

DISCUSSION: The cause of Crohn's disease is unknown. No specific microorganism has been identified as a pathogen, and
no clear-cut environmental factor, such as smoking, has been implicated, even though many patients with Crohn's disease
are heavy smokers. The disease does tend to occur in families. It is more common among Jews than Asians and among
people who live in temperate climates than those in tropical ones.

4. Recurrence after operation for Crohn's disease:


A. Occurs after operations for ileal Crohn's but not colonic Crohn's.
B. Is usually found just proximal to an enteric anastomosis.
C. Rarely requires reoperation.
D. Occurs in 1% of patients at risk per year during the first 10 years after the operation.
E. Is prevented by maintenance therapy with corticosteroids.
Answer: B

DISCUSSION: Recurrence after operation for Crohn's disease often occurs just proximal to an enteric anastomosis or stoma
and occurs at a rate of about 6% per year over the first 10 years after operation. Recurrence follows operations for both ileal
and colonic Crohn's and is not prevented by medical therapy using corticosteroids. Reoperation is required for 30% to 50%
of subjects at risk.

5. Excision rather than bypass is preferred for surgical treatment of small intestinal Crohn's because:
A. Excision is safer.
B. Bypass does not relieve symptoms.
C. Excision cures the patient of Crohn's disease but bypass does not.
D. Fewer early complications appear with excision.
E. The risk of small intestine cancer is reduced.
Answer: E

DISCUSSION: Bypass of segments of small bowel affected with Crohn's disease is a safe operation with few
complications, and one that usually relieves symptoms promptly. It leaves diseased bowel behind, however, which can flare
in the future and can develop carcinoma. Excision, though it does not cure the Crohn's disease, removes the areas of severe
involvement and so eliminates the risk of developing cancers in these segments.
6. Which statements about anorectal Crohn's disease are true?
A. It may be the only overt manifestation of Crohn's disease.
B. It accompanies large intestine Crohn's more often than small-intestine Crohn's.
C. It subsides when associated small intestinal Crohn's is excised.
D. It should not be treated operatively.
E. It may subside in response to metronidazole, 250 mg. q.i.d.
Answer: ABE

DISCUSSION: Anorectal Crohn's disease may be the sole gross manifestation of Crohn's disease. It more often
accompanies large-intestinal Crohn's than small-intestinal Crohn's. When present with small-intestinal Crohn's, resection of
the small-intestinal disease does not affect the course of the anorectal disease. The anorectal disease may subside with
metronidazole therapy alone, but local conservative therapy, such as draining abscesses or unroofing anal fistulas, may also
relieve symptoms and promote healing.

7. The most common indication for operation in Crohn's disease of the colon is:
A. Obstruction.
B. Chronic debility.
C. Bleeding.
D. Perforation.
E. Carcinoma.
Answer: B

DISCUSSION: Crohn's disease of the colon usually leads to operation because of chronic debility and inanition
unresponsive to medical therapy. Obstruction, perforation, and bleeding are uncommon complications of colonic Crohn's.
While for persons with Crohn's colitis the risk of carcinoma of the colon is four to six times that of a healthy control
population, the presence of cancer in the colon is an unusual cause for operation for Crohn's colitis. In fact, most patients
with Crohn's have their colons excised before sufficient time has elapsed for cancers to appear. Cancers usually do not
appear until 10 years or more after the onset of disease.

8. Which of the following statements about surgical anatomy of the colon and rectum is/are correct?
A. The cecum has the largest inner diameter of all segments of the colon (13 to 15 cm.).
B. The rectosigmoid junction is situated at approximately 15 to 18 cm. from the anus.
C. The rectum is entirely an intraperitoneal organ.
D. The ileocolic, right colic, and middle colic arteries are branches from the inferior mesenteric artery.
E. The arterial arcade created by communicating vessels at 1 to 2 cm. from the mesenteric is called the artery of
Drummond.
Answer: BE

DISCUSSION: The cecum has, indeed, the largest inner diameter of all segments of the colon; however it usually measures
between 7 and 9 cm. In patients with a distal obstruction and a continent ileocecal valve the cecum dilates. A diameter of 13
cm. or more is considered at high risk of perforation, and therefore surgery is often needed. The rectosigmoid junction is
situated approximately 15 to 18 cm. from the anus, which places lesions in the sigmoid colon within reach of a rigid
sigmoidoscope. The rectum is covered by peritoneum only in the anterior and superior aspect. The remainder of the rectum,
or most of it, is extraperitoneal. The ileocolic, right colic, and middle colic arteries are branches from the superior
mesenteric artery and supply blood to the ascending colon and the right half of the transverse colon. The artery of
Drummond is an arcade that runs 1 to 2 cm. from the mesenteric border of the entire colon and provides communication
between the superior mesenteric and inferior mesenteric arteries.

9. Which of the following statements about surgical procedures on the colon and rectum is/are correct?
A. Successful healing of colonic anastomoses depends on the adequacy of the blood supply.
B. In excising part of the colon containing cancer, the lymphatics should be avoided by dividing the mesentery close to the
wall of the colon.
C. Despite complete removal of the colon and rectum, transanal fecal flow can be preserved by means of an ileal pouch–
anal anastomosis.
D. When a colostomy is created it cannot be reversed.
E. Colostomy can be life saving in patients with colonic perforation or obstruction.
Answer: ACE

DISCUSSION: Healing of colonic anastomoses depends on the adequacy of blood supply, which in turn depends on the
tension on the anastomosis. Oncologic principles for surgery of colon cancer dictate that the mesentery be divided as close
as possible to the origin of the blood vessels, to include the lymphatic vessels and nodes draining that area. In patients at
risk for colon cancer, such as those with ulcerative colitis or familial polyposis, the construction of an ileal pouch–anal
anastomosis allows for transanal fecal flow despite complete excision of the colon and rectum. Colostomies can often be life
saving, especially in patients with colon perforation or obstruction, and are usually reversed unless the patient requires
abdominoperineal resection of the rectum for cancer.

10. Which of the following statements about colon physiology is/are correct?
A. Colonic recycling of urea is accomplished by the splitting of urea by bacterial ureases.
B. Fermentation by colonic bacteria may rescue malabsorbed carbohydrates.
C. The preferred fuel of the colonic epithelium is glucose.
D. Absorption by the colonic mucosa is a passive process.
E. Insoluble fibers create bulk in the stool.
Answer: ABE

DISCUSSION: One of the functions of the colon is to recycle nutrients used in the digestive process, such as bile salts,
water, and electrolytes. Urea reaching the colon via either the ileal effluent or the mucosal circulation is split by bacterial
ureases. The reabsorbed ammonia is returned to the liver, where it is used for amino acid and protein synthesis.
Fermentation is the anaerobic process by which bacteria can degrade carbohydrates and proteins. The normal substrate for
colonic fermentation is dietary fiber, which resists degradation by alpha-amylases in the small intestine. Starch
polysaccharides are normally degraded by amylases and absorbed in the small intestine; however, when starch
polysaccharides are not adequately degraded and absorbed, they can also be fermented and the caloric value recovered as
short-chain fatty acids. n-Butyrate, one of the short-chain fatty acids produced by bacterial fermentation, is the preferred
fuel of the colonic epithelium. The colonic epithelium utilizes n-butyrate as a fuel for the absorption of sodium and water.
Insoluble fibers retain water and are poorly fermented by bacteria, thus producing fecal bulk.

11. Which of the following statements about colonic motility is/are true?
A. Mass contractions involve only the rectum.
B. “Antiperistaltic” contractions occur in the descending colon.
C. The rectum can accommodate stool by receptive relaxation.
D. Stool in the colon is propelled by tonic contractions.
E. Defecation involves both sensory and motor pathways.
Answer: ADE

DISCUSSION: Various patterns of motility are present in the colon. In the ascending colon “antiperistalsis” waves generate
retrograde flow of colonic contents back to the cecum. In the descending colon contents are propelled caudad by tonic
contractions, separating them into a series of globular masses. A third type of contraction, called mass peristalsis, occurs at
varying intervals through the entire colon's length. Finally, the rectum will undergo receptive relaxation to accommodate
stool until defecation takes place. Normal defecation involves the perception and discrimination of stool and gas in the
rectum, deferment of stool elimination until a socially acceptable time, expulsion of stool, and sealing of the anal canal.
These processes depend on various sensory and motor pathways.

12. Which of the following statements about diagnostic studies for the colon and rectum is/are true?
A. Acetylcholinesterase staining of rectal biopsies is unreliable for the diagnosis of Hirschsprung's disease.
B. Cinedefecography is useful for detecting “hidden” prolapse or rectal intussusception.
C. A negative osmotic gap in stool is indicative of secretory diarrhea.
D. A colonic transit time study involves serial abdominal x-rays after ingestion of radiopaque markers.
E. Carcinoembryonic antigen (CEA) is useful for monitoring patients after resection for colon cancer.
Answer: BCDE
DISCUSSION: Rectal biopsy should be performed in all patients with Hirschsprung's disease. The biposy specimen is
examined for ganglion cells, but a much more reliable method is staining acetylcholinesterase, which has an accuracy rate
of 99%. Cinedefecography involves instilling contrast medium into the rectum and recording the act of defecation with
static radiographs and videofluoroscopy videotape. While the static radiographs allow measurement of the anorectal angle
and the relationship between the anus and the coccyx/pubis plane, the videotape is used to detect rectal intussusception,
which can be missed in static radiographs. In equivocal cases differentiation between malabsorptive and secretory diarrhea
can be made by measurement of electrolytes and osmolality in stool. The osmotic gap is calculated by subtracting 2 × (Na +
K) from the measured osmolality. A negative osmotic gap is indicative of secretory diarrhea, whereas a positive osmotic gap
indicates malabsorptive diarrhea. In a colonic transit study, 20 radiopaque barium-impregnated markers are ingested before
breakfast and plain films of the abdomen are obtained on days 4 and 6. The value of CEA is controversial, but most
surgeons agree that it is helpful in monitoring following resection for colon cancer.

13. Which of the following statements about anorectal functional testing is/are true?
A. Anorectal manometry is often performed through open-tipped multilumen catheters perfused with fluid.
B. Anorectal manometry can differentiate between segmental and global defects of the anal sphincter in patients with
incontinence.
C. Electromyography can demonstrate persistent contraction of the pubis rectalis muscle during defecation, which is
diagnostic of paradoxical pelvic floor contraction.
D. Measurement of sensory thresholds may reveal insensitivity in patients with chronic constipation.
Answer: ABCD

DISCUSSION: Anorectal manometry is most commonly performed through open-tipped multilumen catheters perfused
with fluid. These catheters are connected to a transducer and register internal and external sphincter pressures and presence
of the anorectal inhibitory reflex. Anorectal manometry is particularly useful in patients with fecal incontinence since it can
differentiate between segmental and global defects of the anal sphincter. Electromyography records the action potential
derived from the different muscles involved in defecation through endoscopically placed mucosal electrodes along the
colon. Electromyography can demonstrate persistent contraction of the pubis rectalis muscle during defecation, which is
diagnostic of paradoxical pelvic floor contraction. Sensory threshold measurements may disclose a high rectal threshold
(insensitivity) in patients with idiopathic constipation, extent of denervation, or myopathy in the colon wall.

14. Which of the following statements about the microbiology of the colon is/are true?
A. The colon contains no more bacteria than the stomach.
B. The predominant bacteria in the colon are aerobic.
C. Nearly one third of the dry weight of feces is bacteria.
D. Common bacteria in the colon are Bacteroides, Bifidobacterium, and Enterobacterium species.
E. The colonic microflora is relatively stable.
Answer: CDE

DISCUSSION: The number of bacteria in the colon approaches 10 12 colony-forming units per ml., which is as many as the
space in the colonic lumen allows and many more than in the stomach. The colonic microflora is relatively stable with a
predominance of anaerobic bacteria. Typical species are Bacteroides, Bifidobacterium, and Enterobacterium. Depending on
the fiber content of the diet, bacteria make up a significant proportion of the dry weight of feces. In the western world this
can be as much as one third of the dry weight of feces.

15. Which of the following statements about bowel preparation for colon surgery is/are true?
A. Bowel preparation is accomplished by a combination of mechanical cleansing and nonabsorbable antibiotics.
B. Three days of clear liquids provides sufficient mechanical cleansing.
C. Commercial electrolyte-polyethylene glycol solutions provide mechanical cleansing without inducing electrolyte
imbalance.
D. Nonabsorbable antibiotics such as neomycin and erythromycin base are administered the day before the operation in
three doses.
E. Intravenous antibiotics are also administered the day before surgery.
Answer: ACD

DISCUSSION: Bowel preparation is accomplished by a combination of mechanical cleansing and nonabsorbable


antibiotics. In the past, 3 days of clear liquids was used as a means of mechanical cleansing; this was neither sufficient nor
practical. Presently, an electrolyte-polyethylene glycol solution is commercially available that provides effective mechanical
cleansing without inducing electrolyte imbalance. Nonabsorbable antibiotics such as neomycin and erythromycin base are
administered the day before the operation, usually at 1 P.M., 2 P.M., and 11 P.M. Intravenous antibiotics are administered
within hours of surgery to achieve maximal levels in the tissues during the operation.

16. Which of the following patients generally does not require surgical intervention as a consequence of acute
diverticulitis?
A. A 35-year-old man with no history of diverticulitis.
B. A 68-year-old man status 2 weeks post–renal transplantation.
C. A 55-year-old woman with hypertension and diabetes mellitus.
D. A 50-year-old man with pneumaturia.
E. A 46-year-old man with right-sided diverticulitis.
Answer: C

DISCUSSION: The majority of patients with diverticular disease are elderly and often have comorbid illnesses. The
prognosis in these patients depends on the severity of the underlying inflammatory lesion. Certain subsets of patients,
however, have been identified whose overall prognosis is worse. Patients younger than 40 years have a higher incidence of
complications, as about 70% eventually require surgical intervention. Patients undergoing renal transplantation are routinely
immunosuppressed. Such patients do not manifest the usual signs and symptoms of an inflammatory response. Delays in
diagnosis and failure of the normal immune response mandates surgical intervention in virtually all of these patients. The
presence of pneumaturia is strongly suggestive of a colovesical fistula. All such fistulas require resection of the diseased
colon and repair the involved bladder. Patients with right-sided diverticulitis are usually misdiagnosed as acute appendicitis
and, therefore, often are not diagnosed until laparotomy.

17. The test with the highest diagnostic yield for detecting a colovesical fistula is:
A. Barium enema.
B. Colonoscopy.
C. Computed tomography (CT).
D. Cystography.
E. Cystoscopy.
Answer: E

DISCUSSION: Of the tests listed above, cystoscopy provides the highest diagnostic yield, between 80% and 95%. The most
common finding on cystoscopy is localized inflammation and bullous edema of the bladder mucosa. Actual demonstration
of the fistula is unusual, no matter which test is utilized. Barium enema usually demonstrates some abnormality; however,
precise delineation of the fistula occurs in only 30% of cases. Recently, CT has been shown to be useful in diagnosing
colovesical fistula with accuracy approaching that of cystoscopy. The low diagnostic yields (20%) of cystography and
colonoscopy have limited their use.

18. Which of the following is not true of diverticular disease:


A. It is more common in the United States and Western Europe than in Asia and Africa.
B. A low-fiber diet may predispose to development of diverticulosis.
C. It involves sigmoid colon in more than 90% of patients.
D. Sixty per cent develop diverticulitis sometime during their lifetime.
E. It is the most common cause of massive lower gastrointestinal hemorrhage.
Answer: D

DISCUSSION: Among all patients with diverticular disease only 20% can be expected ever to develop symptoms related to
their disease. The development of diverticular disease has been linked to low-fiber diets, a type of diet more common in
industrialized countries such as the United States and Western Europe. This correlates with the increased prevalence of
diverticular disease in these regions. Diverticular hemorrhage accounts for 50% to 60% of all cases of massive lower
gastrointestinal hemorrhage.

19. The most common indication for surgery secondary to acute diverticulitis is:
A. Abscess.
B. Colonic obstruction.
C. Colovesical fistula.
D. Free perforation.
E. Hemorrhage.
Answer:A

DISCUSSION: Complications of diverticular disease include obstruction, fistulization, hemorrhage, and infection. By far,
the most common indication for surgery is intra-abdominal abscess formation, accounting for 40% to 50% of all
complications of diverticulitis. Intestinal obstruction accounts for another 10% to 30%, while free perforation can be
expected in 10% to 15% of complicated cases of diverticulitis. Fistulization is the least common problem, occurring in only
4% to 10% of complicated cases. Bleeding from diverticula occurs in the complete absence of inflammation.

20. Which of the following is/are true about colorectal polyps?


A. Familial juvenile polyposis is associated with an increased incidence of colon cancer.
B. Although the propensity for development of malignancy is related to the size of a neoplastic polyp, those with mixed
tubulovillous histologic appearance are most likely to develop malignant changes.
C. The loss of a single tumor suppressor gene such as p53 is sufficient to lead to the development of malignancy in
colorectal neoplastic polyps.
D. Endoscopic polypectomy results in a decreased incidence of carcinomas of the colon and rectum.
Answer: AD

DISCUSSION: Juvenile polyps are hamartomas, and can cause symptoms in children such as bleeding, obstruction, and
intussusception. Familial juvenile polyposis is associated with increased risk of colon carcinoma. Polyps with mixed tubular
and villous appearance (tubulovillous adenomas) have an intermediate risk of malignancy; villous adenomas are the most
likely to contain malignancy in each size range. While p53 and other tumor suppressor genes may be associated with the
adenoma-to-carcinoma transition, it appears likely that multiple genetic defects are involved in this transformation.
Alterations in p53 appear to be among the last, as changes are uncommon in adenomas but very common in carcinomas.
The National Polyp Study Group (USA) demonstrated that colonoscopic polypectomy does in fact reduce the incidence of
subsequent colorectal carcinomas, which supports the concept that most carcinomas begin as polyps and supports
aggressive endoscopic removal.

21. Which of the following statements about familial adenomatous polyposis (FAP) is/are true?
A. Inherited in an autosomal-dominant manner, this genetic defect is of variable penetrance, some patients having only a
few polyps whereas others develop thousands.
B. The phenotypic expression of the disease depends mostly on the genotype.
C. Appropriate surgical therapy includes total abdominal colectomy with ileorectal anastomosis and ileoanal pull-through
with rectal mucosectomy.
D. Panproctocolectomy with ileostomy is not appropriate therapy for this disease.
E. Pharmacologic management of this disease may be appropriate in some instances.
Answer: C

DISCUSSION: The genetic defect is of high penetrance: nearly all affected patients develop hundreds to thousands of
polyps. By definition, at least 100 polyps must be present. Recent studies have shown that even patients with the identical
point mutation can exhibit variability in the phenotypic expression, suggesting that environmental or other genetic factors
play a significant role. The phenotypic variations concern age at onset, size of polyps, density of polyps, and extracolonic
manifestations of the disease. Although panproctocolectomy with ileostomy is not well-accepted by patients because of the
stoma, acceptable surgical options include panproctocolectomy with ileostomy, total colectomy with ileorectal anastomosis,
and ileoanal anastomosis with rectal mucosectomy. No pharmacologic agents have been demonstrated to be efficacious in
this condition, though several have been tried.

22. Which of the following statements about the etiology of chronic ulcerative colitis are true?
A. Ulcerative colitis is 50% less frequent in nonwhite than in white populations.
B. Psychosomatic factors play a major causative role in the development of ulcerative colitis.
C. Cytokines are integrally involved in the pathogenesis of ulcerative colitis.
D. Ulcerative colitis has been identified with a greater frequency in family members of patients with confirmed
inflammatory bowel disease.
E. Ulcerative colitis is two to four times more common in Jewish than in non-Jewish populations.
Answer: ACDE

DISCUSSION: Despite intensive investigation, the specific cause of ulcerative colitis remains unknown. There appears to
be a clear genetic component involved in the etiology and distribution of ulcerative colitis. It is significantly less frequent in
nonwhite than in white populations and significantly more frequent among Jews than among other populations. There is a
strong familial concordance by disease category: the prevalence of inflammatory bowel disease is 10% to 25% in relatives
of patients with confirmed Crohn's disease or ulcerative colitis. There is considerable uncertainty about the fundamental role
of infectious agents in the primary pathogenesis of ulcerative colitis. Psychological factors may play a role in exacerbations
of the disease, but they are not of primary importance in its pathogenesis. Recent studies have suggested that cytokines and
other immunoregulatory substances are integrally involved in the pathogenesis of inflammatory bowel disease.

23. Surgical alternatives for the treatment of ulcerative colitis include all of the following except:
A. Colectomy with ileal pouch–anal anastomosis.
B. Left colectomy with colorectal anastomosis.
C. Proctocolectomy with Brooke ileostomy or continent ileostomy.
D. Subtotal colectomy with ileostomy and Hartmann closure of the rectum.
Answer: B

DISCUSSION: Ulcerative colitis is a mucosal inflammatory disease confined to the rectum and colon. It can thus be cured
by total proctocolectomy. For that reason, the standard of therapy for many years was total proctocolectomy and ileostomy.
In an effort to avoid permanent ileostomy a number of other alternatives have been evaluated, including subtotal colectomy
with ileorectal anastomosis, proctocolectomy with continent ileostomy, and colectomy with endorectal ileal pouch–anal
anastomosis. In the past, subtotal colectomy with ileorectal anastomosis was accepted as a compromise operation, with the
knowledge that disease-bearing rectal tissue was retained. Because other definitive alternatives are currently available,
ileorectal anastomosis is no longer appropriate for elective surgical treatment of ulcerative colitis. In an acutely ill patient or
when the diagnosis is in question, subtotal colectomy with ileostomy and Hartmann closure of the rectum is the most
expeditious choice and allows later restorative surgery. Partial colectomy has never been an acceptable alternative for
elective operative management of ulcerative colitis; thus, left colectomy with colorectal anastomosis would not be an
appropriate alternative.

24. The initial management of toxic ulcerative colitis should include:


A. Broad-spectrum antibiotics.
B. 6-Mercaptopurine.
C. Intravenous fluid and electrolyte resuscitation.
D. Opioid antidiarrheals.
E. Colonoscopic decompression.
Answer: AC

DISCUSSION: Toxic colitis is a potentially life-threatening complication of chronic ulcerative colitis. Typically it manifests
clinically with the onset of abdominal pain and severe diarrhea, followed by abdominal distention and generalized
tenderness. Once megacolon and toxicity develop, fever, leukocytosis, pallor, tachycardia, lethargy, and shock set in. The
initial treatment for toxic megacolon thus includes intravenous fluid and electrolyte resuscitation, nasogastric suction,
broad-spectrum antibiotics to provide anaerobic and aerobic gram-negative coverage, and total parenteral nutrition to
improve nutritional status. Large intravenous doses of corticosteroids are generally administered to treat the colitis. In
addition, many patients with toxic megacolon are already receiving steroid therapy and, so, need stress doses of steroids to
prevent adrenal crisis. The immunosuppressive drugs 6-mercaptopurine and azathioprine may play a role in the management
of refractory ulcerative colitis; however, these drugs are not indicated in the acute management of toxicity. Cyclosporine
was shown to be effective in treating acute refractory ulcerative colitis in a single controlled trial, but this has not yet been
confirmed by other prospective studies, and it remains a potentially dangerous drug. Opioid antidiarrheals should be
avoided since they may exacerbate the colonic dilatation and increase the possibility of perforation. Limited proctoscopy
may be helpful in determining the cause of the attack, but colonoscopy may be dangerous and is contraindicated in the face
of acute toxic megacolon. If toxic colitis, with or without megacolon, does not improve within 48 hours, emergency surgery
is warranted.

25. Which finding(s) suggest(s) the diagnosis of chronic ulcerative colitis as opposed to Crohn's colitis?
A. Endoscopic evidence of backwash ileitis.
B. Granulomas on biopsy.
C. Anal fistula.
D. Rectal sparing.
E. Cobblestone appearance on barium enema.
Answer: A

DISCUSSION: It has become increasingly important to distinguish between ulcerative colitis and Crohn's colitis, since the
operative therapy for the two disease processes is quite different. Patients with ulcerative colitis are candidates for
colectomy with ileoanal anastomosis, whereas Crohn's disease is a clear contraindication to this operation. Clinical findings
suggestive of Crohn's disease include anal fistula or other perianal disease, though it must be kept in mind that
approximately 10% of patients with ulcerative colitis may also develop perianal problems secondary to their chronic
diarrhea. Endoscopic or radiographic evidence of rectal sparing is powerful evidence against a diagnosis of ulcerative
colitis. However, if patients have been treated with steroid or salicylate enemas, they may have less active disease in the
rectum than in the more proximal colon, a finding that could mislead the clinician about the presence or degree of rectal
involvement. The deep linear ulcers that lead to a cobblestone appearance on barium enema are strongly suggestive of
Crohn's disease. Typically, ulcerative colitis is confined to the rectum and colon. Frank small bowel involvement is
suggestive of Crohn's disease; however, patients with active pancolitis may have secondary inflammation of the ileum,
which has been called backwash ileitis. This clears after colectomy. The differential diagnosis may ultimately rely on
histologic evaluation. Endoscopic biopsies are not generally useful since they only sample 3-mm. deep segments of mucosa
and submucosa. Transmural inflammation and granulomas on surgical pathologic specimens are pathognomonic of Crohn's
disease.

26. An 80-year-old man who has been bedridden for many years following a stroke presents with acute onset of
abdominal distention, obstipation, and colicky abdominal pain. Abdominal x-rays reveal dilated loops of small bowel and a
dilated sigmoid colon resembling a bent inner tube. Examination reveals distention with mild direct tenderness but no
rigidity or rebound tenderness. Initial management should consist of:
A. Barium enema examination.
B. Laparotomy with resection of descending colon and descending colostomy.
C. Multiple cleansing enemas to remove impacted feces.
D. Rigid sigmoidoscopy and decompression of the sigmoid colon.
Answer: D

DISCUSSION: This patient appears to have presented with the classic signs and symptoms of acute sigmoid volvulus. The
majority of patients with colonic volvulus are elderly men with underlying neurologic dysfunction. They are commonly
referred from a chronic care facility. The patient's condition should be assessed immediately. In the absence of peritonitis the
preferred initial management consists of urgent endoscopic detorsion of the volvulus. In most patients this can be easily
accomplished with a rigid 25-cm. sigmoidoscope or a flexible fiberoptic colonoscope. Often detorsion of the sigmoid colon
results in a dramatic expulsion of gas and feces. A rectal tube should be introduced into the sigmoid and then taped to the
thigh or buttock and left in place to prevent immediate recurrence of the volvulus.

27. Axial twisting of the right colon or cecal volvulus has been shown to be associated with each of the following
except:
A. A history of abdominal operation.
B. A mobile cecum.
C. An obstructing lesion in the transverse or left colon.
D. Inflammatory bowel disease.
Answer: D

DISCUSSION: Volvulus of the right colon is less common than sigmoid volvulus and may involve either an axial twist of
the right colon or a cephalad fold of the cecum (cecal bascule). A mobile cecum is a prerequisite for cecal volvulus and may
occur in up to one third of individuals. Cecal volvulus has also been called postoperative volvulus because of its tendency to
follow abdominal surgical procedures. Obstructing lesions in the distal colon may lead to distention and torsion of the right
colon in patients with abnormalities of cecal fixation.

28. Sigmoid volvulus has been associated with each of the following except:
A. Chronic constipation and laxative abuse.
B. Chronic rectal proplapse.
C. Chronic traumatic paralysis.
D. Medical management of Parkinson's disease.
Answer: B

DISCUSSION: The development of sigmoid volvulus depends on the presence of a dilated, redundant sigmoid colon. This
acquired redundancy may be secondary to long-term ingestion of a high-residue diet, particularly in parts of the world
where the disease is common. In the United States, the most prominent association is chronic constipation and excessive
reliance on laxatives or enemas. Other contributing factors include neurologic or psychiatric conditions such as Parkinson's
disease, Alzheimer's disease, multiple sclerosis, traumatic paralysis, chronic schizophrenia, pseudobulbar palsy, and senility.
Patients are frequently bedridden and are being managed with various neuropsychotropic drugs, both of which may alter
bowel motility.

29. Which of the following statements is not true about inhereted susceptibility to colon cancer?
A. There is no known genetic susceptibility to colon cancer.
B. There are known genetic susceptibilities to colon cancer, but they are always associated with multiple adenomatous
polyps.
C. There are known genetic susceptibilities to colon cancer, but they are always associated with specific ethnic or racial
groups.
D. None of the above.
Answer: D

DISCUSSION: In addition to familial polyposis syndrome, hereditary nonpolyposis colorectal cancer (HNPCC) includes
site-specific colorectal cancer and cancer family syndrome, which are synonymous with Lynch syndromes I and II,
respectively. Autosomal-dominant inheritance of colorectal cancer is observed in both syndromes, and the average patient
age at cancer diagnosis is 45 years. Persons with HNPCC have a high occurrence of synchronous and proximal colon
malignancies. Affected persons usually exhibit one or several adenomatous polyps. The polyps on average are larger, have
more villous histology, and occur at a younger age than adenomas in the general population. Cancer family syndrome also
includes other malignancies, especially endometrial lesions but also ovarian, gastric, small intestinal, and renal cancer.

30. Which of the following recommendations for adjuvant chemotherapy of colorectal carcinoma are true?
A. Patients with Stage I or Dukes A and B1 disease should receive adjuvant treatment for 1 year with levamisole combined
with 5-FU.
B. Patients with Stage III or Dukes C disease should receive adjuvant treatment for 1 year with levamisole combined with
5-FU.
C. There is no role for adjuvant therapy for colon cancer at any stage.
D. Adjuvant chemotherapy is active in colon cancer only when combined with radiotherapy.
Answer: B

DISCUSSION: Some 50% to 60% of patients with colorectal cancer have tumors that penetrate the serosa or involve the
regional lymph nodes, eventually recur, and end fatally. Therefore, adjuvant therapy to improve the mortality was sought for
this group of patients. 5-Fluorouracil (5-FU) is the most active drug used against colon cancer, but it achieves only a 10% to
20% response in patients with advanced disease. Levamisole is thought to be an immunomodulating agent in advanced
colorectal carcinoma. Randomized controlled trials of 5-FU with levamisole, levamisole alone, and surgery in patients with
Dukes B2 or C colon cancer were performed and demonstrated that levamisole plus 5-FU and levamisole improve disease-
free survival for patients with Dukes B and C lesions. Subsequent analysis demonstrated that Dukes C patients receiving
levamisole and 5-FU also had slightly prolonged survival.
A larger, confirmatory intergroup trial was launched that demonstrated that in patients with Dukes C carcinomas of the
colon, adjuvant treatment for 1 year with levamisole combined with 5-FU reduced the risk of cancer recurrence by 41% and
reduced mortality overall by 33%, but the results in patients with Dukes B2 disease was equivocal.

31. Optimal front-line treatment of squamous cell carcinoma of the rectum includes:
A. Abdominal perineal resection.
B. Low anterior resection when technically feasible.
C. Radiation therapy.
D. Chemotherapy.
E. Combined radiation and chemotherapy.
Answer: E

DISCUSSION: Combination radiation therapy and chemotherapy is now the treatment of choice for squamous cell
carcinoma of the anus. The area of the primary lesion is biopsied, and the patient begins radiotherapy to the pelvis. If
inguinal lymph nodes are enlarged, they are also biopsied, usually by fine-needle aspiration, and if positive, they are
included in the field of radiation.
Following radiation therapy, patients receive intravenous 5-FU and mitomycin C. Patients who fail therapy have limited
options, including additional chemotherapy or radiotherapy. Salvage therapy may also include abdominoperineal resection
(APR), lymphadenectomy, or a diverting colostomy, depending on the nature of the recurrence.

32. Which of the following statement(s) is/are true about the maintenance of continence?
A. It depends on both the internal and external sphincters as well as the puborectalis.
B. Resting pressure offers a high-pressure zone that increases resistance to the passage of stools.
C. Maximal squeeze pressure can last no more than 1 minute.
Answer: ABC

DISCUSSION: Continence depends on numerous complex and interrelated anal, rectal, pelvic floor, and colon factors.
Resting pressure depends primarily on the internal sphincter and serves to increase resistance to the passage of stool.
Squeeze pressure, generated by contraction of the external sphincter, increases anal canal resting pressure and helps prevent
leakage when the rectal contents are presented to the proximal anal canal at inopportune times. It lasts but a minute before
fatiguing. The anorectal angle produced by anterior pull of the puborectalis encircles the rectum at the level of the anorectal
ring and helps to maintain continence.

33. Which of the following statement(s) about complete rectal prolapse, or procidentia is/are true?
A. Rectal prolapse results from intussusception of the rectum and rectosigmoid.
B. The disorder is more common in men than in women.
C. Continence nearly always is recovered after correction of the prolapse.
D. All of the above are true.
Answer: A

DISCUSSION: Rectal prolapse is believed to be the result of intussusception of the rectum and rectosigmoid. The condition
predominates in women, in those who strain excessively, and in those suffering from mental disorders. Pregnancy and
delivery are not implicated, as the condition can be observed in men and in nulliparous women. By the time the diagnosis is
established, 50% of patients are incontinent, and continence improves in only half of the patients after surgical correction of
the prolapse.

34. Which of the following statements about hemorrhoids is/are not true?
A. Hemorrhoids are specialized “cushions” present in everyone that aid continence.
B. External hemorrhoids are covered by skin whereas internal hemorrhoids are covered by mucosa.
C. Pain is often associated with uncomplicated hemorrhoids.
D. Hemorrhoidectomy is reserved for third- and fourth-degree hemorrhoids.
Answer: C

DISCUSSION: Hemorrhoids are specialized, highly vascularized cushions in the anal canal that aid continence. The cause
of hemorrhoids is unknown, but they may represent nothing more than the downward sliding of anal cushions associated
with straining and irregular bowel habits. They are classified and treated according to the severity of symptoms. External
hemorrhoids are covered with anoderm and are distal to the dentate line. Internal hemorrhoids are covered by the mucosal
lining of the anal canal proximal to the dentate line. They can cause painless bleeding, usually in association with
defecation. Uncomplicated hemorrhoids usually are not associated with pain, but fissures more often are.
Hemorrhoidectomy is reserved for third-degree (bleeding with prolapse requiring manual reduction) and fourth-degree
(permanently prolapsed with or without bleeding) hemorrhoids.
35. The widely accepted treatment of most localized epidermoid, cloacogenic, or transitional cell carcinoma of the anal
canal is:
A. Surgical resection.
B. Chemotherapy alone.
C. Radiotherapy alone.
D. Combined chemoradiation.
Answer: D

DISCUSSION: Tumors arising in the anal canal or in the transitional zone that have squamous, basaloid, cloacogenic, or
mucoepidermoid epithelium are similar in their clinical presentation and response to treatment. Combined chemoradiation
(the so-called Nigro protocol) promises to preserve continence, avoid colostomy, and offer a similar survival rate. Local
excision is reserved for the few very small and superficial lesions. For most lesions, chemoradiation—external-beam
radiation, 5-fluorouracil, and mitomycin C—is the treatment of choice.

36. Which statement(s) is/are true about hidradenitis suppurativa?


A. It is a disease of the apocrine sweat glands.
B. It causes multiple perianal and perineal sinuses that drain watery pus.
C. The sinuses do not communicate with the dentate line.
D. The treatment is surgical.
E. All of the above.
Answer: E

DISCUSSION: Hidradenitis suppurativa is an inflammatory process of the sweat glands characterized by abscess and sinus
formation. The disease may involve other areas where apocrine glands are present, such as the axilla, mammary, inguinal,
and genital regions. The affected areas have a blotchy, purplish appearance with numerous sinuses draining watery pus. The
condition must be differentiated from cryptoglandular fistulas, which communicate with the dentate line, and Crohn's
disease, which may track to the anorectum proximal to the dentate line. Treatment consists of unroofing sinuses for limited
disease and wide local excision for more advanced disease.

37. Which of the following statements regarding the vasculature of the colon and rectum is/are correct?

a. The middle colic artery is a branch of the superior mesenteric artery


b. The inferior mesenteric artery supplies the descending and sigmoid segments of the colon
c. An complete anastomotic arcade paralleling the colon wall is present in only 15 to 205 of individuals
d. The middle colic artery is a branch of the inferior mesenteric artery
Answer: a, b, c

Two major arterial systems supply the colon. The right colon is predominantly supplied by the superior mesenteric artery.
The major branches of this artery that perfuse the right colon include the ileocolic branch, which supplies the ileocecal
junction; the right colic, which supplies the ascending colon; and the middle colic artery, which supplies the hepatic flexure
and the transverse colon to its midpoint. The left colon is predominantly supplied by the inferior mesenteric artery, which
derives its origin from the abdominal aorta. The distal transverse colon and the descending colon obtain their blood supply
from the left colic branch of the inferior mesenteric artery, while the sigmoid colon obtains its blood supply from sigmoidal
branches. The colic arteries bifurcate and form vascular arcades so that the resultant marginal Drummond artery forms an
anastomosis between the superior mesenteric artery and the inferior mesenteric artery. However, considerable anatomic
variation exists with respect to this arterial arcade, and a complete anastomosis is present in only 15% to 20% of people.

38. Which of the following agents have been proposed as sensory neurotransmitters for the colon?

a. Acetylcholine
b. Substance P
c. Calcitonin gene-related peptide
d. Bradykinin
e. Somatostatin
Answer: b, c
Sensory neurons within the wall of the colon perceive mechanical and chemical stimuli from the luminal contents. Their
axons project both to motor neurons as well as to prevertebral ganglia and higher neural centers. Mediators of such sensory
input appear to be substance P and calcitonin-gene related peptide. The role of sensory neurons in transmitting information
remains uncertain, but since they have been shown to synapse on the excitatory and inhibitory motor neurons, they probably
play an important role in modulating spontaneous contractions.

39. How much of the daily insensible water loss is due to loss in stool?

a. 200 ml
b. 400 ml
c. 600 ml
d. 800 ml
e. 1000 ml
Answer: a

Compared with the small intestine, the epithelium of the colon is relatively impermeable and requires a considerably longer
period of time to absorb the salt, water, and carbohydrate presented to it. Between 500 and 1500 g of a semiliquid material
enter the colon on a daily basis. Most of this substance is absorbed, yielding a stool output that weighs about 200 g. The
reason for this efficient absorptive capacity, despite the relative impermeability of the colonic epithelium, relates to the close
interrelation between epithelial transport and colonic motor activity. Transit through the large bowel takes about 10 times as
long as that through the small intestine, usually requiring 2 or 3 days. Such slow passage provides optimal opportunity for
the luminal contents to come into surface contact with the absorptive epithelium. Thus, sodium and water, which are
extracted against high electrochemical and osmotic gradients, are efficiently absorbed. Further, the relatively static
conditions of the colon allow the proliferation of vast numbers of anaerobic bacteria, which in turn break down
carbohydrate to volatile fatty acids, which then are rapidly absorbed. The more rapid the colonic transit, the less efficient are
these absorptive processes.

40. A 52-year-old woman is involved in an automobile accident and sustains an open fracture of the fight femur,
compression fractures of the 10th and 11th thoracic vertebrae and right pulmonary contusion. On the fourth day after injury,
her abdomen is noted to be distended, tympanitic and diffusely tender. Abdominal radiographs reveal gaseous distension of
the ascending and transverse segments of the colon. The cecum is 13 cm in greatest diameter. Appropriate management
includes which of the following as the next step?

a. Right hemicolectomy
b. Operative cecostomy
c. Colonoscopy
d. Contrast enema
e. Observation
Answer: c

Acute obstruction of the colon in the absence of any identifiable mechanical obstructing lesion is known as Ogilvie’s
syndrome. Acute pseudo-obstruction usually involves the proximal colon, but may extend throughout the transverse colon
and, rarely, the descending colon as well. The cause of this syndrome is unknown, but it almost always arises in critically ill
or bedridden patients who have serious concomitant disease. The syndrome is occasionally seen in traumatized patients,
particularly if a vertebral fracture is present. The hallmark of the disease is severe abdominal distention. Roentgenographic
films of the abdomen show marked gaseous distention of the colon that is typically localized to the right colon.
The major problem with Ogilvie’s syndrome is the risk of cecal perforation. This risk is substantial, and the more prolonged
the cecal distention, the greater this risk becomes. Although a considerable debate exists in the literature about the extent to
which cecal dilatation can occur before perforation becomes a likely possibility, some type of decompressive procedure
should be considered when the cecum reaches 10–12 cm in diameter. Fiberoptic colonoscopy is the treatment of choice and
is initially successful in 90% or more of patients. In the event that colonoscopy does not work or is not readily accessible,
operative cecostomy is the accepted alternative therapy.

41. Which of the following features is/are consistent with a diagnosis of colonic inertia?

a. Alternating episodes of severe constipation and normal bowel activity


b. Total bowel transit time of 24 hours
c. Total bowel transit time of 48 hours
d. Total bowel transit time of 96 hours
e. Marfinoid habitus
Answer: d

Colonic inertia, or slow transit constipation, is primarily a disease of young women. Despite attempts at initiating bowel
movements with fiber supplementation, large doses of laxatives and enemas, normal bowel movement patterns are not
established. The etiology of this condition is totally unknown, but a likely etiology is some aberration in the neurochemical
control of the colon, possibly within the enteric nerves. Abnormalities within the neural elements of the myenteric plexus
suggest that disturbances in neuromodulation of colonic motility may play a role in some patients. The treatment of colonic
inertia has proved difficult and many patients have required subtotal colectomy to correct the severe constipation. Diagnosis
of this condition is usually achieved by assessing colonic transit with various radio-opaque markers. After ingestion of such
markers, sequential abdominal films are taken to assess movement of markers in each segment of the colon. Total transit
time in normal subjects averages about 35 hours. Total transit time in excess of 72 hours is clearly abnormal.

42. Which of the following statements regarding the myenteric plexus of the colon is/are correct?

a. The myenteric plexus is located between the longitudinal and circular layers of the bowel wall
b. The myenteric plexus contains only sensory neurons
c. The density of neurons with the colonic plexuses decreases along the length of the bowel
d. Neurons of the myenteric plexus control the motor function of the colon
Answer: a, c, d

Similar to other regions of the gut, two groups of plexuses exist within the wall of the colon. The submucosal plexus is
located between the muscularis mucosa and the circular muscle layer of the muscularis propria. Sandwiched between the
circular muscle and the outer longitudinal muscle is the myenteric plexus. The myenteric plexus appears to be intimately
involved with colonic motility. The plexus is composed of ganglia and clusters of nerve cell bodies that are linked together
by bundles of nerve processes. Although the myenteric plexus is well developed throughout the entire length of the colon,
the density of nerve cells is greatest in the proximal one third of the colon, similar to the density that exists throughout the
small intestine. The physiologic significance of this organization remains to be defined, but probably contributes to the
different motility patters that exist throughout the large bowel.

43. Which of the following ocular manifestations of ulcerative colitis respond to therapy with steroids or
immunosuppressive agents?

a. Iridis
b. Uveitis
c. Retrobulbar neuritis
d. Ulcerative panophthalmitis
Answer: a, b, c

A number of ocular manifestations of ulcerative colitis exist. Included in this group are iridis, uveitis, conjunctivitis,
episcleritis, retinitis, and retrobulbar neuritis. With the exception of ulcerative panophthalmitis, ocular symptoms are closely
related to disease activity and respond to therapy with steroids or immunosuppressive agents.

44. Which of the following statements regarding the risk of cancer in the context of ulcerative colitis is/are correct?

a. After 10 years of active disease, the risk of cancer approximates 20% to 30%
b. After 10 years of active disease, the risk of cancer approximates 2% to 3%
c. The risk of colon cancer in ulcerative colitis is identical to controls
d. After 20 years of disease activity, the risk of colon cancer approximates 80%
Answer: b

Significant dysplasia or suspected colon cancer is a clear indication for colectomy in patients with ulcerative colitis. Earlier
studies have suggested that the risk of cancer is relatively low for the first ten years after the onset of disease activity
(roughly 2% to 3%). The incidence of colon cancer then begins to climb at a rate of 1% to 2% per year. By the time the
patient has had ulcerative colitis for 20 years, the risk of colon cancer approximates 20%. Many epidemiologists believe that
earlier studies overestimated the risk of malignancy due to referral bias and the imperfection of retrospective surveys
performed in tertiary referral hospitals.

45. Which of the following features would be more consistent with Crohn’s disease than with ulcerative colitis?

a. Transmural inflammation
b. Microscopic evidence of granulomas within mucosal biopsies
c. Microscopic evidence of submucosal thickening and fibrosis
d. Microscopic evidence of submucosal inflammation
Answer: a, b, c, d

Transmural changes are found in Crohn’s disease of the colon in which all layers of the colonic wall may be involved in a
granulomatous inflammatory process. In its earliest stages, the lesions consist of infiltration of round cells and
polymorphonuclear leukocytes into the crypts of Lieberkühn at the base of the mucosa. Crypt abscesses are common.
Microscopy reveals vacuolization of overlying epithelial cells, swelling of mitochondria, and widening of intercellular
spaces. Submucosal thickening and fibrosis associated with recurrent submucosal inflammation are more common in
Crohn’s disease than in ulcerative colitis.

46. A 19-year-old male is seen in consultation with complaints of bloody diarrhea (10 bowel movements per day), and
weight loss (10 pounds). Physical examination reveals the presence of two circular, 4 cm erythematous lesions on the trunk.
Each lesion has an area of necrosis in the center. The abdominal examination reveals mild hypogastric tenderness. The stool
is guaiac positive. The most appropriate next diagnostic step includes which of the following?

a. Barium enema
b. Flexible sigmoidoscopy
c. Liver biopsy
d. Chest x-ray
Answer: b

The diagnosis of ulcerative colitis is one of exclusion. There are no definitive, laboratory, radiologic, or histologic features.
All patients presenting with bloody diarrhea should have an infectious etiology excluded. Stool samples and biopsy
specimens should be evaluated for Campylobacter, Salmonella, pathogenic E coli, amebic colitis, and C difficile. Flexible
sigmoidoscopy is the first step in diagnosis, because ulcerative colitis involves the distal colon and rectum in 90% to 95% of
the cases. Mild cases may show only a loss of normal vascular pattern, a granular texture, and microhemorrhages when the
friable mucosa is touched by the endoscope. More advanced cases may demonstrate spontaneous bleeding, ulceration, and
purulent exudate. Mucosal biopsies are essential in establishing the diagnosis. Lesions of the skin and oral cavity are
frequently observed in patients with ulcerative colitis. Pyoderma gangrenosum is observed in approximately 1% of patients.

47. Which of the following statement(s) regarding ulcerative colitis is/are correct?

a. The most common age of onset for ulcerative colitis is in early adulthood
b. Approximately 25% of cases of ulcerative colitis occur after the age of 60
c. Males are affected approximately twice as frequently as females
d. Approximately 10% to 25% of patients with ulcerative colitis have first degree relatives with the disease
Answer: a, d

Most cases of ulcerative colitis have onset between the ages of 15 and 40 years. While the age of onset may extend to old
age, only 3% to 5% of the cases have onset after age 60. Males and females are affected equally frequently. Clear-cut
familial patterns have been observed in ulcerative colitis. Ten to 25% of patients with this disease have first degree relatives
with ulcerative colitis. Monozygotic twins have higher concordance for inflammatory bowel disease than dizygotic twins.
Geographic as well as racial differences influence the occurrence of the disease. There is currently no definitive evidence
regarding the role of genetic and environmental influences in the determination of familial patterns.
48. Many patients with ulcerative colitis are operated upon electively with total abdominal colectomy, rectal
mucosectomy, formation of a small intestinal reservoir, and ileoanal anastomosis. The most common postoperative
complication after this operation is which of the following?

a. Enterocutaneous fistula
b. Small bowel obstruction
c. Pulmonary embolism
d. Urinary retention
Answer: b

The major postoperative morbidity following restorative proctocolectomy is small bowel obstruction. Bowel obstruction
rates requiring re-operation has been reported in 10% to 20% of most series of patients undergoing ileal pouch-anal
anastomosis. Pelvic and wound infections have been reported to occur in 10% of patients undergoing ileoanal anastomosis,
although the overall infection rate has been reduced to approximately 5% in several recent series. Conversion to a
permanent ileostomy as a result of postoperative complications is observed in fewer than 5% of patients.

49. A 25-year-old woman with known ulcerative colitis presents to the emergency room with a 24-hour history of
abdominal pain, distention, and obstipation. Physical examination reveals a temperature of 38.6° C, abdominal distention,
and diffuse abdominal tenderness. Abdominal x-rays show marked colonic dilatation, most pronounced in the transverse
colon. Laboratory examination reveals a white blood count of 19,000/mm3. Over the first 24 hours of hospitalization,
symptoms are progressive in spite of intravenous fluid resuscitation, nasogastric suctioning, and intravenous antibiotics.
The most appropriate management for this patient would include which of the following?

a. Decompressive colonoscopy
b. Proctocolectomy with formation of end ileostomy
c. Total abdominal colectomy with formation of Hartmann pouch and end ileostomy
d. Cecostomy
Answer: c

Acute toxic megacolon occurs in 6% to 13% of patients with ulcerative colitis. Initial treatment for toxic megacolon
includes intravenous fluid and electrolyte resuscitation, nasogastric suctioning, broad spectrum antibiotics, and total
parenteral nutrition. The therapeutic role of intravenous steroids in toxic megacolon is controversial. Most patients
presenting with a severe attack of ulcerative colitis are already receiving steroid therapy and require stress doses of
corticosteroids to prevent adrenal crisis. When symptomatology is progressive or when there is evidence of colonic
perforation, emergency surgery is indicated.
Postoperative complications including sepsis, wound infection, intraperitoneal abscess, fistula formation, and delayed
wound healing are common and have been reported in up to 50% of patients. The presence of colonic perforation doubles
operative risk. In the presence of toxic megacolon or colonic perforation, the operation should be definitive without being
overly aggressive. Abdominal colectomy with ileostomy and Hartmann closure of the rectum is the procedure of choice.
After recovery, delayed surgery for restoration of continence can be performed. Leaving the rectum intact allows its use for
subsequent mucosal proctectomy and ileoanal anastomosis.

50. The most common postoperative complication after formation of a continent ileostomy (Kock pouch) is which of
the following?

a. Nipple valve failure


b. Small bowel obstruction
c. Pancreatitis
d. Ischemic necrosis of the pouch
Answer: a

The continent ileostomy has been associated with a high postoperative complication rate. Most complications are related to
displacement of the nipple valve, producing fecal incontinence and difficulty in intubating and emptying the pouch. Valve
failure has been reported to occur in up to 40% of patients. Ten to 20% of patients suffer postoperative bowel obstructions.
Several syndromes of ileostomy dysfunction related to Cock pouch have also been reported. These are variably described as
stagnant loop syndrome, pouchitis, and nonspecific ileitis. Clinical features include diarrhea, malabsorption of fat and
vitamin B12, proliferation of anaerobic bacteria, inflammation of the pouch, and incontinence. Crohn’s disease is a clear
contraindication to performing this operation, as the rate of postoperative complication is much higher in this group.

51. One year following ileal pouch-anal anastomosis, the mean 24-hour stool frequency is which of the following?

a. Two to three
b. Five to six
c. Eight to nine
d. Eleven to twelve
Answer: b

Overall, mean 24-hour stool frequencies average five or six bowel movements in the late follow-up period after ileal pouch-
anal anastomosis.

52. A 30-year-old male two years postoperative after total abdominal colectomy with ileoanal anastomosis reports a
sudden increase in stool frequency, nocturnal leakage, and low-grade fevers. Physical examination is unremarkable.
Flexible endoscopic examination of the small intestinal pouch reveals a friable erythematous mucosa. Biopsies of the
mucosa are obtained. While awaiting biopsy results, which of the following is the most appropriate empiric therapy?

a. Oral corticosteroids
b. Oral vancomycin
c. Oral metronidazole
d. Corticosteroid enema
Answer: c

Nonspecific enteritis or pouchitis is the most common late complication of ileal pouch-anal anastomosis, occurring in as
many as 15% of patients. The clinical symptoms include high stool frequency, watery stools, fat malabsorption, urgency,
nocturnal leakage, and rectal bleeding. Patients may manifest fever, malaise, and arthralgias. The etiology of this condition
is unknown. Most patients respond to treatment with metronidazole.

53. A 72-year-old man returns to the hospital 2 weeks following right hip arthroplasty with complaints of 48 hours of
mucoid diarrhea, fever, and crampy abdominal pain. Physical examination reveals dehydration, diffuse abdominal
tenderness, and a temperature of 102°F. Outpatient medications have included digoxin, propranolol, and cephalothin. If
antibiotic-associated colitis is suspected, which of the following is/are appropriate diagnostic tests?

a. Fecal leukocyte smear


b. Stool culture for C difficile
c. Measurement of C difficile toxin in stool
d. Barium enema
e. Computed topography of abdomen
Answer: a, b, c

The fecal leukocyte test is a simple, rapid screening measure that is sometimes useful in supporting the diagnosis of C
difficile colitis. The finding in stained smears of 3 to 5 leukocytes in at least 5 high-dry fields suggests colitis and is strongly
against the diagnosis of benign or simple antibiotic-diarrhea, but it is not specific for C difficile. A positive test indicates
mucosal inflammation and excludes the benign form of antibiotic diarrhea; however, no more than a third of patients with C
difficile colitis have positive fecal leukocyte tests.
The isolation of C difficile from stools of a patient with diarrhea does not by itself prove the patient has colitis caused by C
difficile, because about 25% of isolates of C difficile obtained from humans are nontoxigenic and nonpathogenic. In
addition, at least 3% of healthy adults are asymptomatic carriers off toxigenic isolates of C difficile. Cell culture evidence
for cytotoxicity by cytotoxin B is presently the most reliable laboratory aid in diagnosis of pseudomembranous colitis. This
test is the "gold standard" used to evaluate all newer tests, but it is expensive, usually takes at least 24 hours, and is not
readily available in most hospitals. At least 95 percent of adults with antibiotic-associated diarrhea and toxin B positive
stools have colitis.
54. For the patient in the preceding question, after obtaining diagnostic samples, the most appropriate management
would include which of the following?

a. Oral metronidazole
b. Intravenous metronidazole
c. Oral vancomycin
d. Intravenous vancomycin
e. Oral bacitracin
Answer: c

The efficacy of vancomycin in treatment of PMC has been so well documented that it must be considered the most reliable
treatment for the disease. No isolates of C difficile are resistant to vancomycin. Metronidazole, bacitracin, and teicoplanin
are alternatives when vancomycin is not tolerated. C difficile isolates are almost always susceptible to vancomycin at
concentrations of lower than 5 mg/l, and few isolates have been identified that require more than 16 mg/l.
Because vancomycin is poorly absorbed into the systemic circulation when given orally at a dosage of 125 to 500 mg four
times per day, concentrations far exceeding 16 mg/l, the maximum concentration required for inhibition of C difficile, are
easily achieved in stools. Even patients with profuse diarrhea achieve adequate concentrations of vancomycin in stools with
these regimens. While small amounts of vancomycin may be absorbed systemically, toxic serum concentrations have never
been reported with oral therapy, even when the patient has renal failure.

55. A 72-year-old woman undergoes anterior resection for a rectal cancer located 7 cm proximal to the anal verge.
Pathologic examination of the resected specimen reveals invasion of the tumor into the muscularis propria. Five of 8 lymph
nodes contain microscopic tumor. There is no evidence of disseminated disease. Appropriate subsequent management
includes which of the following?

a. Postoperative radiation plus intravenous 5FU


b. Postoperative radiation alone
c. Observation
d. Postoperative radiation plus intravenous adriamycin
Answer: a

In rectal cancer: it is almost as important to prevent local failure and ensuing symptoms as it is to prevent death from distant
failure. Radiation therapy is recommended for patients with stage II or III rectal cancers. In a randomized, prospective
study, 204 patients with stage II or III rectal cancers were randomized to receive postoperative radiation alone versus
radiation plus 5FU and semustine chemotherapy. The group that received chemotherapy had improved local tumor control
and increased overall survival. In another prospective study, semustine was found not to be an essential component for
effective adjuvant therapy. Based on these and other clinical studies, the NIH has recommended that patients with stage II or
III rectal cancers should receive postoperative chemotherapy and radiation as standard care.

56. Which of the following are tumor suppressor genes that have been associated with the development of colorectal
cancer?

a. The DCC gene


b. The APC gene
c. The P53 gene
d. The Rb gene
Answer: a, b, c

Loss of specific chromosomal regions represent genetic alterations that are associated with the development of colorectal
neoplasms in a high percentage of cases. These chromosomal regions have been hypothesized to contain tumor-suppressor
genes whose products normally regulate growth and differentiation in a negative fashion. One such gene linked to familial
adenomatous polyposis was mapped to the long arm of chromosome 5q and referred to as the APC (adenomatosis polyposis
coli) gene. The gene codes for a 300 kilodalton protein which has recently been shown to bind to b-catenin, implying an
important role in cell adhesion and possibly cytoskeleton function. It is hypothesized that disruption of cell adhesion and
cytoskeleton function can lead to loss of contact inhibition which may promote neoplastic transformation as well as
invasiveness of cancer cells.
Another tumor suppressor gene thought to be important in colorectal tumorigenesis is the p53 gene located in chromosome
17p. Alteration in p53 is one of the most common genetic events seen in human malignancies. The p53 gene produces a
DNA-binding phosphoprotein that is important in cell proliferation, differentiation and cell survival. Allelic loss of p53 has
been observed in more than 75% of colorectal carcinomas.
Another common genetic alteration associated with colorectal tumors is an allelic loss of chromosome 18q. This is where
the "deleted in colorectal carcinoma" gene, also termed DCC is located. Mutations in DCC are present in 47% of late
adenomas and 73% of carcinomas. The DCC protein shares significant homology with the neural cell adhesion molecule
family that regulates cell adhesion and recognition.

57. Which of the following statement(s) is/are correct with regard to the use of carcinoembryonic antigen (CEA)
determinations in management of colorectal cancer?

a. CEA determination has 95% specificity when used for screening for colon cancer development in patients with
ulcerative colitis
b. CEA levels are increased in 20% of patients with local recurrence after resectional therapy
c. CEA measurements are increased in 90% of patients with disseminated disease
d. CEA levels are increased in 90% of patients with local recurrence after resectional therapy
Answer: b, c

CEA is a glycoprotein that was originally described to be a tumor-specific antigen derived from neoplasms of the
gastrointestinal tract. CEA is an oncofetal antigen because it is also expressed by early embryonic or fetal cells. It is now
known that CEA is not tumor-specific since it can be elevated by a variety of malignancies from different sites as well as in
some benign conditions. CEA is not useful as a screening or diagnostic test but is useful as a tumor marker. CEA is elevated
in over 90% of patients with disseminated colorectal cancer and in about 20% of patients with localized disease. Serum
levels generally are elevated in proportion to the mass of the tumor present and often correlate with response to therapy.
CEA levels are useful when elevated levels fall to normal levels after curative resection. In about two thirds of patients with
recurrent disease, an increased CEA level is the first indicator of the tumor, and serial CEA testing, combined with regular
physical examinations, is one of the most useful tests for detecting recurrent colorectal cancer.

58. The most common oncogene abnormality observed in association with colorectal cancer is which of the following?

a. Overexpression of the N-myc oncogene


b. Amplification of the K-ras oncogene
c. Suppression of the erbB oncogene
d. Amplification of the L-myc oncogene
Answer: b

In colon cancer an important genetic alteration that has been demonstrated is a mutation of the K-ras protooncogene. The ras
protooncogenes are a family of normal genes (N-ras, H-ras and K-ras) that are highly conserved in nature and encode for the
production of GTP-binding proteins (G proteins) which are important for signal transduction. G proteins are involved in the
transduction of proliferative signals induced by growth factors or factors involved in cell differentiation. The product of a
mutated ras gene is an abnormal G protein which has lost its ability to become inactivated; and thus, results in continuous
growth stimulation and autonomous cell growth or differentiation. Experimentally, transfection of normal fibroblasts by
mutated ras genes confer neoplastic properties to those cells.
About half of colorectal carcinomas and a similar percentage of adenomas larger than 1 cm in diameter have been found to
have the ras gene mutations. By contrast, less than 10% of patients with adenomas smaller than 1 cm have this mutation. It
has been postulated that the ras gene mutation may be the initiating event in some colorectal carcinomas or, alternatively,
may promote the clonal expansion of a mutated cell population. It appears that the ras gene mutation alone is not
responsible for tumorigenesis. Other molecular events appear to be required in addition to ras gene mutations.

59. Which of the following types of colonic polyps is associated with the highest incidence of malignant degeneration?

a. Tubular adenoma
b. Tubulovillous adenoma
c. Villous adenoma
d. Hamartomatous polyp
Answer: c

Adenomas can be divided into tubular (75% to 100% tubular component), tubulovillous (25% to 75% villous component),
or villous (75% to 100% villous) adenomas. The most common type is tubular adenoma, or adenomatous polyp, which
constitutes about 75% of neoplastic polyps. Tubulovillous adenomas represent 15% and pure villous adenomas 10% of
neoplastic polyps. All adenomas contain some degree of dysplasia or cellular atypia. This dysplasia can be graded from mild
to severe. Carcinoma in situ and severe dysplasia have been grouped together under the classification of high-grade
dysplasia. In carcinoma in situ, there is no invasion into the muscularis mucosa, as there is in invasive carcinoma. The
incidence of invasive malignancy differs markedly for the three types of adenomas and increases with size. In general,
malignancies are seen in 5% of adenomatous polyps, in 22% of tubulovillous adenomas, and in 40% of villous lesions.
Hence, although villous lesions are much less common, they are more likely to harbor a malignancy.

60. A 52-year-old man undergoes a right hemicolectomy for a carcinoma of the ascending colon. Pathological
examination of the resected specimen reveals invasion of the tumor to the level of the muscularis propria. Three of 17
lymph nodes contain microscopic tumor. What is the correct Dukes classification (Aster-Coller modification) and
associated 5-year survival for this lesion?

a. Dukes C2, 45% 5-year survival


b. Dukes B1, 75% 5-year survival
c. Dukes C1, 45% 5-year survival
d. Dukes B3, 65% 5-year survival
Answer: c

One of the more commonly used staging systems is the modified Astler-Coller system. According to this system, stage A
represents tumors that invade into the mucosa only. Stage B1 tumors invade into but not through the muscularis propria.
Stage B2 lesions invade through the bowel wall without adjacent organ involvement, whereas stage B3 tumors involve
adjacent organs. Stage C tumors involve regional lymph nodes and are subgrouped into stages C1, C2, and C3, according to
depth of bowel wall penetration. Stage D represents evidence of distant organ involvement. In general, the 5-year survival
rate for patients with stage D disease is less than 10%. Overall, the 5-year survival rates for stages A, B, and C are 90%,
77%, and 47%, respectively. Additional studies have revealed that among Dukes stage C patients, the number of positive
nodes is a significant predictor of survival.

61. An pedunculated polyp, discovered incidentally at colonoscopy, is removed by snare polypectomy from the
ascending colon. Invasive cancer to the level of the submucosa is identified histologically within the polyp. The lesion is
well-differentiated. No lymphatic or vascular invasion is noted. The cauterized margin is negative for neoplasm.
Appropriate subsequent management includes which of the following?

a. Repeat endoscopy at 6 months


b. Right hemicolectomy
c. Subtotal colectomy
d. Repeat endoscopy with fulguration of the polypectomy site
Answer: a

With the availability of colonoscopy, endoscopic polypectomy has become the standard approach for the treatment of
neoplastic polyps. The risk of this procedure is extremely low, with a complication rate of less than 1%. Almost all
pedunculated polyps can be removed endoscopically with a snare. A dilemma in treating colonic polyps occurs when a
resected lesion contains a malignant focus. A decision must then be made about the need for a colectomy. If the lesion does
not penetrate the muscularis mucosa, it should be considered an in situ malignancy that does not have the propensity to
metastasize and, therefore, does not require further surgery. If the lesion penetrates the muscularis mucosa, it is an invasive
cancer and may require surgery. In selected cases of pedunculated polyps, conservative management without colectomy
may be undertaken if the lesion does not contain poorly differentiated tumor cells or evidence of vascular invasion and if a
negative resection margin has been obtained at the level of the stalk. Lesions that are poorly differentiated or have evidence
of vascular invasion, regardless of a negative surgical margin, should be treated by colectomy.

62. Dietary risk factors thought to play a causative role in development of colorectal cancer include which of the
following?
a. High fat intake
b. Low fiber intake
c. High smoked food intake
d. High vegetable intake
Answer: a, b

Evidence from epidemiological studies suggest that dietary factors play important causative roles in the development of
large bowel cancers. From these studies, fat intake has been the most consistently positive association and fiber intake the
most consistently inverse association noted in the incidence of colorectal cancer. In comparisons between countries, the
rates of colon cancer are strongly associated with the intake of animal fat and meat. The associations between per capita
consumption of total fat, saturated fat, and cholesterol and national incidence rates of colon cancer are strongly positive. The
proposed mechanism by which dietary fat may increase the risk of developing colonic cancer is by its interaction with bile
acids.
The relation between fiber intake and colon cancer was initially noted by Burkitt, who reported low rates of colon cancer in
areas of Africa where fiber consumption and stool bulk were high. In general, epidemiological studies have demonstrated
that fiber intake is higher in non-western countries with lower incidence of colon cancer. The role of fiber was originally
seen simply as providing bulk to dilute potential carcinogens and speed their transit through the colon. This appears to be an
oversimplification and the relation between fiber intake and colon cancer is more complex. Additional studies have
suggested that certain fibers may bind mutagens, which reduces their contact with colonic epithelium, may favorably
change the fecal pH, and may participate in other complex interactions.

63. Which of the following statements with regard to resection of rectal cancers is/are true?

a. A distal margin of 5 cm should be obtained because 42% of patients have microscopic evidence of intramural
spread beyond 3 cm from the palpable tumor
b. A distal margin of 3 cm should be obtained because only 3% of patients have microscopic evidence of intramural
spread beyond 2 cm from the palpable tumor
c. Local recurrence rates correlate strongly with distal margins less than 4 cm
d. There is no correlation between local recurrence and distal margins beyond 2 cm
Answer: b, d

One of the controversies surrounding sphincter-saving procedures for rectal tumors is the length of adequate distal mucosal
margin. The traditional dictum of 5 cm for a margin is not substantiated by any studies. Only 2.5% of patients have
intramural spread beyond 2 cm from the palpable tumor, and these patients generally have dissemination of tumor despite
aggressive local therapy. There is no correlation between local recurrence and the extent of distal margin when it is greater
than 2 cm. Ideally, a surgical margin of 3 cm, measured on the fresh specimen, should be achieved.

64. A 58-year-old male undergoes resection of a Dukes C2 colon cancer via right hemicolectomy. Three years
postoperatively, rising CEA levels prompt evaluation including abdominal computed tomography. Two lesions, each
measuring 2 cm, are noted in the right hepatic lobe. No other abnormalities are noted. A right hepatic lobectomy is
performed without complication. Which of the following most closely approximates anticipated 5-year survival?

a. 85–90%
b. 65–70%
c. 45–50%
d. 25–30%
Answer: d

The liver is the most frequent site of blood-borne metastases from primary colorectal cancers. In a subgroup of patients, the
liver may be the only site of recurrent disease, and surgical excision of these metastases is the only curative option for these
patients. Overall, surgical resection is associated with a 25% to 30% 5-year survival rate. Patients eligible for hepatic
resection of metastatic disease are those who have no evidence of extrahepatic tumor, no medical contraindications for
surgery, and less than four lesions that are amenable to resection with negative surgical margins.
65. An asymptomatic 52-year-old man is undergoing screening sigmoidoscopy. In the rectum, at 6 cm from the anal
verge, a 2 cm yellow, submucosal nodule is noted. Deep endoscopic biopsies are consistent with carcinoid. Appropriate
management includes which of the following?

a. Observation
b. Transanal excision
c. Low anterior resection
d. Abdominoperineal resection
Answer: b

Most gastrointestinal tract carcinoids occur in the ileum and the appendix. The rectum is the next most common site, and
occasional carcinoid tumors occur in the colon. Tumor size is an extremely important prognostic factor. About 60% of rectal
carcinoids present as asymptomatic submucosal nodules measuring less than 2 cm in diameter. Transanal local excision
suffices for definitive therapy since small tumors rarely metastasize. Malignant potential is seen almost exclusively in
patients with tumors larger than 2 cm. More radical excisions of larger rectal lesions may be required for local control;
however, the results of radical excisions for large rectal carcinoids are poor since they are prone to metastasize.

66. A 72-year-old woman complains of anal itching and burning. Physical examination reveals an erythematous, scaly
lesion, 3 cm in circumference, within the anal canal. The intersphincteric groove can not be appreciated in the area of the
lesion. The remainder of the physical examination is normal. Appropriate initial management includes which of the
following?

a. Acyclovir 200 mg QID for 10 days


b. Hydrocortisone cream 0.1% topically for 14 days
c. Incisional biopsy
d. Metronidazole 250 mg PO QID for 14 days
Answer: c

Extramammary Paget’s disease may be found in the axilla and in the anogenital region, including the labia majora, penis,
scrotum, groin, pubic area, perineum, perianal region, thigh, and buttock. Paget’s disease of the perianal area is a malignant
neoplasm of the intraepidermal portion of apocrine glands with or without associated dermal involvement. Paget’s disease
has a long preinvasive phase, but if untreated, an invasive adenocarcinoma of the apocrine gland type develops. The disease
is more common in women than men, with the highest incidence in the seventh decade.
Macroscopically, the lesion appears as an erythematous scaly or eczematoid plaque-like lesion, similar to other benign
perianal lesions, making clinical diagnosis difficult. A definite diagnosis is made by biopsy, which shows characteristic
histologic appearance—large, pale, vacuolated cells with hyperchromatic eccentric nuclei. The cells invariably contain acid
mucosubstances, an important feature in distinguishing this lesion from melanoma and Bowen’s disease.

67. For the patient in the preceding question, biopsy revealed an invasive apocrine gland neoplasm. The deep margins
included striated muscle infiltrated by neoplastic cells. Appropriate management includes which of the following?

a. Primary radiation
b. Abdominoperineal resection with bilateral inguinal lymph node dissection
c. Abdominoperineal resection only
d. Carbon dioxide laser fulguration
Answer: c

Wide local excision is the treatment of choice in the absence of invasive carcinoma. Because of the high incidence of local
recurrence and residual tumor, it is vitally important to obtain an adequate resectional margin. Grossly, the extent of
involvement is ill defined, and multiple punch biopsies may be required to determine the extent of involvement. For more
advanced lesions with underlying carcinoma, an abdominoperineal resection is indicated. Inguinal lymph node dissection is
performed only if groin lymph nodes are clinically positive for metastasis. Because of the commonly delayed diagnosis
(average, 4 years), about 25% of patients with perianal Paget’s disease have metastases when they seek treatment. The sites
of metastases, in order of frequency, are inguinal and pelvic lymph nodes, liver, bone, lung, brain, bladder, prostate, and
adrenal gland. The prognosis is poor once metastasis has occurred.
68. A 43-year-old woman presents with complaints of anal pain and spotting of blood with defecation. Physical
examination reveals a 2 3 cm area of ulceration within the anal canal. The remainder of the physical examination is
normal. Incisional biopsy is positive for squamous cell carcinoma. Appropriate management includes which of the
following?

a. Abdominoperineal resection
b. Wide local excision, skin grafting, proximal diverting colostomy
c. Primary radiation therapy
d. Local excision and primary closure
Answer: c

For localized squamous cell cancers of the anal canal, the most effective protocol consists of primary irradiation and
chemotherapy. The treatment regimen includes:

1. External irradiation, 3000 rad (30 Gy), to the primary tumor, pelvic, and inguinal nodes from day 1 to day 21 (200
rad/d, 5 days a week)
2. Systemic chemotherapy, 5-fluorouracil at 1000 mg/m2/24 h, as a continuous infusion for 4 days, starting on day 1
of radiotherapy and repeated on days 28 through 31
3. Mitomycin C at 15 mg/m2 intravenous bolus on day 1

If the lesion disappears grossly, and its microscopic absence is confirmed by biopsy, no further treatment is necessary.

69. Recurrent episodes of sigmoid colonic diverticulitis prompt operative therapy. Which of the following describe the
appropriate margins for resection?

a. Proximal margin, splenic flexure; distal margin, rectosigmoid junction


b. Proximal margin, descending colon; distal margin, rectosigmoid junction
c. Proximal margin, descending colon; distal margin, mid-rectum
d. Proximal margin, transverse colon; distal margin, mid-rectum
Answer: b

At the time of exploratory laparotomy, if the disease is localized, a segmental colectomy should be performed. The distal
extent of the resection should always extend to the proximal rectum to decrease the chance of recurrence. The proximal
extent of resection should include the segment involved with the acute disease plus any additional colon with signs of
chronic disease or large numbers of diverticula. With this approach, the recurrence rate after surgical resection is less than
10%. The only absolute contraindications to primary anastomosis are free perforation with generalized peritonitis;
obstruction with unprepared bowel; and intraoperative conditions that do not warrant primary anastomosis, such as septic
shock, ureteral injury, or other medical conditions that make a prolonged operation inadvisable. If resection is thought to be
unsafe in the presence of a massive phlegmon or if the patient is too unstable to undergo a resection, a diverting end
colostomy with mucous fistula may be appropriate, with planned colonic resection at a later date after inflammation
subsides.

70. An elderly man presents with complaints that he is passing gas with urination. The past medical history is positive
for one episode of diverticulitis, treated medically, transurethral resection of the prostate for benign prostatic hypertrophy,
and diabetes. Which of the following diagnostic tests is most appropriate initially?

a. Computed tomography of the abdomen and pelvis


b. Cystoscopy
c. Barium enema
d. Intravenous pyelography
Answer: a

Colovesical fistulas account for about half of fistulas due to diverticulitis. Most patients with colovesical fistula present with
urinary tract symptoms, including urgency, dysuria, pneumaturia, and fecaluria. In spite of obvious symptoms, the diagnosis
of colovesical fistula can be difficult to establish conclusively. Recurrent urinary tract infections in an elderly man should
increase suspicion. Barium enema usually demonstrates diverticula, or occasionally, sigmoid narrowing. Only rarely is the
fistulous tract actually filled. Cystoscopy demonstrates hyperemia and inflammation consistent with chronic cystitis.
Although these findings may be localized to some extent, indicating the presence of a fistulous communication, the fistulous
opening is seldom seen. CT using intraluminal contrast material has emerged as the most sensitive test for the presence of a
colovesical fistula. The presence of barium in the urine is diagnostic of a colovesical fistula. In over 90% of patients, air is
visualized in the urinary bladder, and an indurated segment of sigmoid colon is observed adjacent to a locally thickened
bladder wall.

71. For the patient in the preceding question, a colovesical fistula originating from the sigmoid colon is demonstrated.
Colonoscopy reveals diverticula and excludes carcinoma. During laparotomy, a thickened sigmoid colon is found to be
adherent to the dome of the bladder. A definite fistula is not observed. Appropriate operative management includes which
of the following?

a. Sigmoid resection, primary colonic anastomosis, catheter drainage of bladder


b. En-bloc resection of sigmoid colon and adjacent bladder wall, primary colonic anastomosis, suprapubic cystostomy
c. En-bloc resection of sigmoid colon and adjacent bladder wall, formation of descending colostomy and Hartmann’s
pouch, suprapubic cystostomy
d. Sigmoid resection, primary colonic anastomosis, bilateral percutaneous nephrostomies
Answer: a

Most patients with colovesical fistulas are effectively treated by a one-stage procedure consisting of segmental colectomy
and closure of the fistulous opening in the bladder. The proximal margin of resection should include the entire segment of
thickened, contracted colon and any additional colon that is involved in the acute inflammation. If the fistulous opening is
not identifiable, implying that it is small in diameter, nothing needs to be done to identify the bladder fistula. Urinary
catheter drainage for 7 to 10 days, followed by cystographic verification of closure of the fistula, is sufficient therapy.
Depending on the severity of the related complications of diverticulitis (obstruction, inflammation, abscess, sepsis, other
fistulas), it may occasionally be necessary to perform a two-stage procedure, the first stage being segmental colectomy and
colostomy formation and the second stage consisting of closure of the colostomy. Either the one-or two-stage procedure can
be done with low morbidity and mortality and with less than a 5% recurrence rate.

72. A 65-year-old woman develops obstipation, lower abdominal pain, and fever. Physical examination reveals a
temperature of 38.5°C, left lower quadrant tenderness, and an ill-defined lower abdominal mass. White blood count is
17,500 per mm3. Intravenous hydration, broad spectrum antibiotics, and analgesics are ordered. After 48 hours, symptoms
have not improved. Appropriate management includes which of the following?

a. Barium enema
b. Computed tomography of the abdomen
c. Immediate laparotomy
d. Intravenous pyelogram
Answer: b

Signs and symptoms of diverticulitis include fever, tachycardia, leukocytosis with left shift of the differential count,
abdominal pain, and a tender lower abdominal mass. Most patients with an acute episode of diverticulitis can be treated with
intravenous fluids, bowel rest, broad-spectrum antibiotics, and analgesics.
If the patient does not improve within 48 hours, complications of diverticulitis may exist, and further investigation is
necessary. Only about 20% of patients develop complications of diverticulitis with their first episode; this rises to 60% with
recurrent episodes. Although a water-soluble contrast enema radiograph can provide the diagnosis of diverticulitis,
computed tomography (CT) has become the preferred diagnostic test in patients that do not improve within 48 hours. CT is
especially useful in delineating the complications of diverticulitis, including perforation and abscess formation. In addition,
at the time of CT scan, percutaneous drainage catheters can be placed if an abscess is identified. After CT drainage of an
abscess, 50% to 90% of patients may undergo a successful one stage procedure of segmented colectomy and primary
anastomosis. If percutaneous drainage is not feasible, or an abscess is not identified, surgical intervention is recommended.

73. Which of the following statement(s) relating to anal sphincteric function is/are correct?

a. When the rectum is distended, the internal anal sphincter relaxes and the external anal sphincter contracts
b. When the rectum is distended, the internal anal sphincter contracts and the external anal sphincter relaxes
c. The external anal sphincter is responsible for resting anal pressure
d. The internal anal sphincter is responsible for resting anal pressure
Answer: a, d

The internal sphincter, because it is innervated by the autonomic nervous system, is not subject to voluntary control. This
powerful muscle exists in a continuously tonic state, and is responsible for maintaining closure of the resting anal canal. The
high-pressure zone of the anal canal at rest is due to the actions of the internal sphincter. The external sphincter contributes
to anal pressure only when a bolus of stool is present within the anal canal. The increase in pressure during voluntary
contraction (squeeze pressure) is exclusively due to the activity of the external sphincter. The high resting pressure in the
anal canal acts as a barrier to prevent leakage of mucus and gas.
When the rectum is distended, the internal sphincter relaxes. This relaxation allows the rectal content to move down to the
anal canal. In contrast, when the rectum is distended, the external sphincter contracts. Reflex contraction of the external
sphincter prevents rectal content from leaking through the anus. Although volitional contraction of the external sphincter
can only be sustained for short periods, it is the most important mechanism of voluntary continence.

74. The most common complication after hemorrhoidectomy is which of the following?

a. Urinary retention
b. Rectal bleeding
c. Incontinence
d. Wound infection
Answer: a

Hemorrhoidectomy should be considered when the hemorrhoids are severely prolapsed through the anus, requiring manual
replacement, or in hemorrhoids complicated with associated pathology, such as ulceration, fissure, fistula, large
hypertrophied anal papilla, or extensive skin tags. An elliptic excision starts at the perianal skin, includes external and
internal hemorrhoids, and ends at the anorectal ring. The mucosa and submucosa are dissected off the underlying internal
sphincter muscle. Unless there is an associated anal stenosis or chronic anal fissure, internal sphincterotomy is not
performed. The entire wound is closed with running absorbable suture. The largest and the most redundant hemorrhoid
should be excised first. No packing is placed in the anal canal. Urinary retention is the most common complication of
hemorrhoidectomy, and can be avoided if intravenous fluids are restricted during the procedure and minimized for the next
6 to 8 hours.

75. Appropriate treatment of chlamydial proctitis includes which of the following?

a. Tetracycline 500 mg QID


b. Metronidazole 250 mg QID
c. Acyclovir 200 mg QID
d. Erythromycin 500 mg QID
Answer: a, d

C trachomatis is the most common cause of sexually transmitted disease in the United States, affecting 4 million Americans
each year. Proctoscopy reveals a picture of nonspecific proctitis with friable, granular, and edematous mucosa.
Immunofluorescent microscopy provides an accurate and a rapid diagnosis. Treatment includes tetracycline hydrochloride,
500 mg by mouth four times daily for 7 days, or doxycycline, 100 mg by mouth twice daily for 7 days. For patients in
whom tetracyclines are contraindicated, erythromycin base or stearate, 500 mg by mouth four times daily for 7 days, or
erythromycin ethylsuccinate, 800 mg by mouth four times daily for 7 days, may be used. Two new drugs have been
approved by the FDA for the treatment of chlamydia—Azithromycin, 1 gm orally in a single dose, and Ofloxacin, 300 mg
orally two times a day for 7 days. A substantial advantage of Azithromycin, in comparison with all other therapies, is that a
single dose is effective; this antimicrobial may prove most useful in situations in which compliance with a seven day
regimen of another antimicrobial cannot be ensured. In view of the high efficacy of tetracycline and doxycycline, cost also
should be considered when selecting a treatment regimen.

76. A 65-year-old man presents with complaints of mucous discharge and perianal discomfort. Physical examination
reveals a fistulous opening lateral to the anus. Anoscopic examination permits passage of a probe through the fistula tract.
The fistula traverses the internal anal sphincter, the intersphincteric plane, and a portion of the external anal sphincter. The
fistula is categorized as which type?
a. Intersphincteric
b. Transsphincteric
c. Suprasphincteric
d. Extrasphincteric
Answer: b

There are four main forms of fistula-in-ano, based on the relation of the fistula to the sphincter muscles. An intersphincteric
fistula tract is in the intersphincteric plane. The external opening is usually in the perianal skin close to the anal verge. A
transsphincteric fistula starts in the intersphincteric plane or in the deep postanal space. The fistulous track traverses the
external sphincter, with the external opening at the ischioanal fossa. Horseshoe fistulas are in this category. Suprasphincteric
fistulas start in the intersphincteric plane in the mid-anal canal and then pass upward to a point above the puborectalis. The
fistula passes laterally over this muscle and downward between the puborectalis and the levator ani muscle into the
ischioanal fossa. An extrasphincteric fistula passes from the perineal skin through the ischioanal fossa, the levator ani
muscle, and finally penetrates the rectal wall. Extrasphincteric fistulas may arise from cryptoglandular origin, trauma,
foreign body, or pelvic abscess.

77. For the patient in the preceding question, appropriate management includes which of the following?

a. Division of the tissues over the probe with electrocautery, leaving the wound open to heal by secondary intention
b. Division of the tissues over the probe with electrocautery, closing the wound using a pedicled skin flap
c. Division of the internal anal sphincter using electrocautery, encircling the external sphincter with a seton
d. Proximal diverting colostomy and antibiotics
Answer: c

In young patients, transection of internal and external sphincter muscles in the posterior half, when performed in the course
of a fistulotomy, does not always jeopardize anal continence. In older patients and in women, however, transection of the
external sphincter muscle, particularly in the anterior half, risks incontinence. When external sphincter transection appears
likely, some authors recommend the use of a seton. A seton is a suture that is drawn through a fistula. The rationale for using
a seton is to create fibrosis. The seton is threaded through the fistulous track and tied over the muscles. In the second stage
(average interval, 6 to 8 weeks), fistulotomy is performed. Incontinence after the proper use of seton is uncommon, even
when the fistula is deep.

1. Which of the following statements about the segmental anatomy of the liver are not true?
A. Segments are subdivisions in both the French and American systems.
B. Segments are determined primarily by the hepatic venous drainage.
C. The French anatomic system is more applicable than the American system to clinical hepatic resection.
D. Segments are important to the understanding of the topographic anatomy of the liver.
Answer: D

DISCUSSION: Segments are the major subdivision of the right and left lobes of the liver. In either the classic lobar
(American) or the segmental (French) system, the most variable aspect is the biliary system. Therefore the hepatic venous or
portal system defines most segments. The French system depicts eight segments, with the caudate lobe as segment I and the
other seven segments defined primarily by the hepatic venous system. Segments are not well-depicted by topography.
2. Which of the following anatomic features of the biliary system are important considerations in operative
cholangiography?
A. The left hepatic duct comes off farther anterior than the right one.
B. At the confluence there may be more than just a right and a left hepatic duct.
C. Dissection of the triangle of Calot is more important than cholangiography in preventing bile duct injury.
D. Segments V, VII, or VIII sometimes join the biliary system below the confluence.
Answer: ABCD

DISCUSSION: All of these features are important. The angle of takeoff of the left hepatic duct may make operative
visualization difficult with the patient in the supine position. Because there may be more than two major hepatic ducts,
visualization of two large ducts does not ensure that the system is normal. Ducts from any of the right-side segments can
join below the confluence. Dissecting one superior edge of the gallbladder before it joins the cystic duct is particularly
important in preventing injury.

3. The hepatic artery:


A. Supplies the same amount of blood to the liver as the portal vein.
B. Provides more blood to the bile ducts than the portal vein.
C. Is autoregulated just as the portal vein is.
D. Supplies most of the blood to hepatic metastases.
Answer: BD

DISCUSSION: The portal vein provides two thirds to three quarters of the total hepatic blood flow. The portal vein is
incapable of direct autoregulation. The hepatic artery after transplantation classically infarcts portions of the biliary system,
whereas hepatic metastases often arrive there via the portal vein. Most of their blood supply comes from the hepatic artery.

4. Bile formation is:


A. An active secretory process.
B. Determined at two sites principally.
C. Regulated physiologically by hormones.
D. Largely determined by the intactness of the enterohepatic circulation (EHC).
Answer: ABCD

DISCUSSION: Bile formation is an active process at both the canalicular and ductular sites. The paracellular pathway
probably plays a minor physiologic role. Secretin and glucagon are likely physiologic regulators of biliary secretion. Bile
salts are extremely important and are probably the most important agent in the changes that occur when enterohepatic
circulation is interrupted.

5. Generally, the two most important hepatic functions to consider after hepatic resection are:
A. Hepatic synthetic function.
B. Glucose metabolism.
C. The liver's role in lipid metabolism.
D. The liver's role in vitamin metabolism.
Answer: AB

DISCUSSION: While other functions undoubtedly may be important postoperatively, the most common abnormalities
occurring after a major hepatic resection are related to loss of protein synthesis and consequences of glucose metabolism.
Therefore, it is usually advisable to administer supplemental amounts of protein and sugar postoperatively.

6. Which of the following statements about pyogenic abscess of the liver are true?
A. The right lobe is more commonly involved than the left lobe.
B. Appendicitis with perforation and abscess is the most common underlying cause of hepatic abscess.
C. Mortality is largely determined by the underlying disease.
D. Mortality from hepatic abscess is currently greater than 40%.
Answer: AC
DISCUSSION: Involvement of the right lobe with abscess formation approximates 70% of pyogenic abscesses. This is
thought to be due to the streaming effect of superior mesenteric venous inflow to the right lobe. In addition, the greater
volume of the right lobe predisposes more tissue to seeding by bacterial organisms. While appendicitis comprised 25% to
40% of cases in early series, early recognition and operative therapy for appendicitis have reduced its importance
significantly. In current series, malignant or benign biliary obstruction is the underlying cause of 35% to 50% of cases.
Recent studies have shown that the underlying disease or an immunocompromised host is more important prognostically
than solitary versus multiple abscesses.

7. Which of the following statements most accurately describes the current therapy for pyogenic hepatic abscess?
A. Antibiotics alone are adequate for the treatment of most cases.
B. All patients require open surgical drainage for optimal management.
C. Optimal treatment involves treatment of not only the abscess but the underlying source as well.
D. Percutaneous drainage is more successful for multiple lesions than for solitary ones.
Answer: C

DISCUSSION: The development of ultrasonography and computed tomography (CT) in the past two decades has enabled
earlier diagnosis and advances in treatment of hepatic abscess. Formerly, open surgical drainage was considered necessary
in essentially all cases of pyogenic abscess. Numerous recent series, however, have reported high success rates and low
mortality from the percutaneous catheter drainage of abscesses under CT or ultrasonographic guidance. Optimal
management of pyogenic abscess, however, involves not only treatment of the abscess, whether by percutaneous or surgical
methods, but correction of the underlying source as well. All modes of therapy are more successful in treating solitary
lesions than multiple ones.

8. Which of the following statements characterize amebic abscess?


A. Mortality is higher than that for similarly located pyogenic abscesses.
B. The diagnosis of amebic abscess may be based on serologic tests and resolution of symptoms.
C. In contrast to pyogenic abscess, the treatment of amebic abscess is primarily medical.
D. Patients with amebic abscess tend to be older than those with pyogenic abscess.
BC

DISCUSSION: Mortality for uncomplicated amebic abscess should be less than 5%, in contrast to the 15% to 20% rate for
pyogenic abscess. After the demonstration by radiologic examination of an abscess, appropriate serologic tests and
resolution of symptoms after a course of treatment with an antiamebic agent such as metronidazole constitute presumptive
diagnosis of amebic abscess. Aspiration of abscess contents rarely yields amebic organisms. In contrast to pyogenic abscess,
amebic abscess rarely requires surgical or percutaneous drainage, except in the case of an extremely large abscess or
bacterial superinfection. Amebic abscess affects males in a 9:1 to 10:1 ratio and generally affects a younger population than
pyogenic abscess. Additionally, in the United States the populations most affected are immigrants from endemic areas such
as Mexico or Latin America and American tourists to those regions.

9. Which of the following statement(s) is/are true about benign lesions of the liver?
A. Adenomas are true neoplasms with a predisposition for complications and should usually be resected.
B. Focal nodular hyperplasia (FNH) is a neoplasm related to birth control pills (BCPs) and usually requires resection.
C. Hemangiomas are the most common benign lesions of the liver that come to the surgeon's attention.
D. Nodular regenerative hyperplasia does not usually accompany cirrhosis.
Answer: A

DISCUSSION: Adenomas typically enlarge and cause symptoms, may rupture, and have a definite malignant potential.
Therefore they should generally be resected when found. FNH is not a true neoplasm and generally has an uneventful
course. Both are related to BCPs, although the relationship of adenoma is more firmly established. While small bile duct
hamartomas are much more common, hemangiomas are the most common lesion to come to the attention of surgeons. They
should not generally be biopsied because of possible hemorrhage. By definition, nodular regenerative hyperplasia occurs in
the absence of cirrhosis.

10. Which of the following statement(s) about malignant neoplasms of the liver is/are true?
A. Hepatocellular carcinoma is probably the number 1 cause of death from cancers worldwide.
B. The most common resectable hepatic malignant neoplasm in the United States is colorectal metastasis.
C. Hepatoma has at least one variant that has a much more benign course than hepatomas in general.
D. Hepatomas are generally slower growing than was formerly believed.
Answer: ABCD

DISCUSSION: Although exact comparisons are impossible, hepatoma seems to be the most common cause of cancer death
worldwide, despite its relative infrequency in the United States. Colorectal metastasis is a more common indication for
surgical treatment in the United States. The fibrolamellar variant and possibly the very well-differentiated tumor probably
have a better prognosis than hepatomas in general. Previous studies from Africa in which there was a high incidence of
rupture account for the poor prognosis that was generally attributed to hepatoma. Recent studies from Europe and the
United States have shown that survival after presentation is usually measured in years.

11. Which of the following statement(s) is/are true about bile duct cancers?
A. If resected, proximal lesions are usually curable.
B. The more proximal the lesion, the more likely is resection to be curative.
C. Radiation clearly prolongs survival.
D. Transplantation is usually successful if the lesion seems confined to the liver.
E. None of the above is true.
Answer: E

DISCUSSION: Most bile duct cancers are discovered after they are incurable, and only a tiny subset of resected proximal
lesions are cured. The more distal the lesion, the more likely is resection to achieve cure (e.g., approximately 30% 5-year
survival for periampullary lesions as compared with 0% to 10% for hilar lesions). The use of adjuvant or primary radiation
remains controversial because of the heterogeneity of the patient populations on which this modality has been used. Because
of the localized nature of this disease it would seem that transplantation would produce favorable results; however, this has
not been the case.

12. Echinococcosis liver disease caused by Echinococcus granulosus:


A. Is not a neoplasm.
B. Is endemic to parts of Europe, but not the United States.
C. Is usually curable by resection.
D. Is more deadly than in its Echinococcus multilocularis form.
Answer: ABC

DISCUSSION: The parasitic infection is fairly common in certain parts of Europe but very rare in the United States.
Resection without peritoneal soilage is the treatment of choice. The E. multilocularis form, which is endemic to parts of the
United States, is more likely to be fatal because it is rarely resectable. This form is more likely to resemble a malignancy
than E. granulosus, although the natural course of the disease usually spans many years.

13. Which of the following statements about hemobilia are true?


A. Tumors are the most common cause.
B. The primary treatment of severe hemobilia is an operation.
C. Percutaneous cholangiographic hemobilia is usually minor.
D. Ultrasonography usually reveals a specific diagnosis.
Answer: C

DISCUSSION: By far the most common cause of hemobilia is trauma. Tumors also may cause the syndrome but are
relatively uncommon causes. For severe hemobilia the best therapy is arteriographic embolization. Usually the site of
bleeding or a false aneurysm can be identified. Operation should be reserved as a last resort or when the condition is
recognized intraoperatively. Percutaneous cholangiography–associated intrabiliary hemorrhage is usually, but not always,
minor and self-limiting. Ultrasonography is a very nonspecific diagnostic technique for hemobilia. Arteriography remains
the best diagnostic method.

14. Ligation of all of the following arteries usually causes significant hepatic enzyme abnormalities except:
A. Ligation of the right hepatic artery.
B. Ligation of the left hepatic artery.
C. Ligation of the hepatic artery distal to the gastroduodenal branch.
D. Ligation of the hepatic artery proximal to the gastroduodenal artery.
Answer: D

DISCUSSION: Ligation of the right or left hepatic artery frequently causes enzyme elevation but is usually tolerated by the
patient, particularly when this is a life-saving maneuver. Ligation of the hepatic artery distal to the gastroduodenal branch is
more risky but is also usually tolerated. Ligation of the hepatic artery proximal to the gastroduodenal one does not normally
cause enzyme abnormalities because of abundant collateral flow through that branch.

15. Which of the following is the most common acid-base disturbance in patients with cirrhosis and portal hypertension?
A. Metabolic acidosis.
B. Respiratory alkalosis.
C. Metabolic alkalosis.
D. Respiratory acidosis.
Answer: C

DISCUSSION: Metabolic alkalosis and hypokalemia are common in patients with cirrhosis because they often have
associated secondary hyperaldosteronism (especially those with ascites), diarrhea, and frequent emesis. Hyperaldosteronism
enhances H+ and K+ exchange for Na+ in the distal tubule of the kidney. The cause of diarrhea in patients with cirrhosis is
unknown, but malabsorption secondary to splanchnic venous hypertension may be a contributing factor. Emesis is common
in alcoholic cirrhotics and patients with tense ascites. Deleterious effects of metabolic alkalosis include impaired tissue
oxygen delivery secondary to shift of the oxyhemoglobin dissociation curve to the left and conversion of ammonium
chloride to ammonia, which may contribute to encephalopathy.

16. A portal venous pressure of 30 mm. Hg (elevated) and a hepatic venous wedge pressure of 5 mm. Hg (normal) may be
associated with which of the following causes of portal hypertension?
A. Portal vein thrombosis.
B. Alcoholic cirrhosis.
C. Schistosomiasis.
D. Alcoholic hepatitis.
Answer: AC

DISCUSSION: Pressure measured by wedging a catheter into a hepatic vein (hepatic venous wedge pressure) closely
correlates with directly measured portal venous pressure in patients with portal hypertension when the site of elevated
resistance is at the sinusoidal or postsinusoidal level. Such is the case in alcoholic cirrhosis and alcoholic hepatitis. When
the site of increased resistance is at the presinusoidal level, either within (schistosomiasis) or outside (portal vein
thrombosis) the liver, the hepatic venous wedge pressure is normal despite markedly elevated portal vein pressure. Although
schistosomiasis is one of the more frequent causes of portal hypertension worldwide, in North America presinusoidal portal
hypertension is considerably less common than alcoholic liver disease. A normal hepatic venous wedge pressure in a patient
who has bled from varices should lead one to suspect a presinusoidal cause. A specific diagnosis can often be made by
visceral angiography or liver biopsy.

17. Which of the following is the most effective definitive therapy for both prevention of recurrent variceal hemorrhage and
control of ascites?
A. Endoscopic sclerotherapy.
B. Distal splenorenal shunt.
C. Esophagogastric devascularization (Sugiura procedure).
D. Side-to-side portacaval shunt.
E. End-to-side portacaval shunt.
Answer: D

DISCUSSION: Shunt operations are the most effective means of preventing recurrent variceal hemorrhage. Rebleeding
rates after endoscopic sclerotherapy range from 40% to 60%. Although extensive esophagogastric devascularization has
effectively prevented recurrent bleeding in Japanese series, these operations have been followed by rebleeding rates in
excess of 25% in most Western series. Although one controlled trial has shown more frequent recurrent hemorrhage
following the distal splenorenal shunt than after the portacaval shunt, most series have reported rebleeding rates of less than
10% for both of these operations. Both the liver and the splanchnic viscera are important sites of ascites formation. Since
the distal splenorenal shunt maintains sinusoidal and mesenteric venous hypertension and requires interruption of important
retroperitoneal lymphatics, it tends to aggravate rather than relieve ascites. Hepatic sinusoidal pressure may be unchanged
or even increased after an end-to-side portacaval shunt. Only side-to-side portal-systemic shunts, such as the side-to-side
portacaval shunt, reliably decompress both the liver and splanchnic viscera, thus preventing ascites formation.

18. Which of the following treatments most effectively preserves hepatic portal perfusion?
A. Distal splenorenal shunt.
B. Conventional splenorenal shunt.
C. Endoscopic sclerotherapy.
D. Side-to-side portacaval shunt.
Answer: C

DISCUSSION: The conventional splenorenal shunt and side-to-side portacaval shunts completely divert portal flow away
from the liver (nonselective shunts). The distal splenorenal shunt is a selective shunt that preserves hepatic portal perfusion
in the majority of patients; however, the magnitude of portal flow is decreased because the gastrosplenic component is
diverted into the renal vein. Additionally, many patients (especially alcoholic cirrhotics) develop collaterals between the
mesenteric venous circulation and the shunt, resulting in gradual attrition of the remaining portal flow. Although there have
been anecdotal reports of portal vein thrombosis after endoscopic sclerotherapy, two controlled trials have demonstrated
better preservation of hepatic portal perfusion in sclerotherapy patients than in persons who receive the distal splenorenal
shunt.

19. Which of the following veins is preserved in performing the extensive esophagogastric devascularization procedure
described by Sugiura?
A. Left gastric (coronary) vein.
B. Short gastric vein.
C. Splenic vein.
D. Left gastroepiploic vein.
Answer: A

DISCUSSION: The Sugiura procedure consists of devascularization of the esophagus to the inferior pulmonary vein and the
proximal two thirds of the stomach, splenectomy, and distal esophageal transection. The devascularization component
should be done as close to the esophagus and stomach as possible. The coronary vein and paraesophageal collaterals are
preserved to maintain an effective portal-systemic collateral pathway and thereby discourage reformation of varices.

20. Which of the following complications of portal hypertension often require surgical intervention (for more than 25% of
patients)?
A. Hypersplenism.
B. Variceal hemorrhage.
C. Ascites.
D. Encephalopathy.
Answer: B

DISCUSSION: While many patients with portal hypertension develop hypersplenism, it is rarely clinically significant. A
splenectomy should not be performed unless platelet counts are persistently less than 20,000 per cu. mm. or white blood cell
counts are less than 1200 per cu. mm. Unfortunately, splenectomy is sometimes done for clinically insignificant
hypersplenism, thus obviating a distal splenorenal shunt if the patient should subsequently bleed from varices. The initial
treatment for most patients with bleeding esophageal varices should be endoscopic sclerotherapy; however, operation is
required for the approximately one third of patients who fail sclerotherapy and for noncompliant persons, those living in
remote geographic locations, and patients bleeding from gastric varices. Ascites can be controlled by a medical regimen of
dietary salt restriction and diuretic therapy in more than 95% of patients. When ascites is intractable to medical
management, either intermittent large-volume paracenteses or a surgical peritoneovenous shunt should be done. With rare
exceptions, encephalopathy should be treated medically. Most important is elimination of any precipitating factors that led
to the neuropsychological disturbance. Lactulose, neomycin, and dietary protein restriction may also be components of the
medical treatment regimen.

21. Which of the following effects are advantages of combined vasopressin and nitroglycerin intravenous infusion, as
compared with vasopressin infusion alone, in controlling acute variceal bleeding?
A. Lower frequency of encephalopathy.
B. Lower incidence of vasopressin side effects.
C. More effective control of bleeding.
D. Less “rebound effect” when discontinuing the infusion.
Answer: BC

DISCUSSION: Vasopressin acts through vasoconstriction of splanchnic arterioles. Both portal venous inflow and portal
venous pressure are reduced, resulting in control of acute variceal bleeding in approximately 50% of patients. However, the
adverse side effects of systemic hypertension, bradycardia, decreased cardiac output, and coronary vasoconstriction are
quite common during vasopressin infusion. Simultaneous administration of nitroglycerin or nitroprusside eliminates these
side effects—and in one controlled trial enhanced therapeutic effectiveness. Although the mechanism of action of this
combined infusion is not clear, vasodilation of portal-systemic collaterals, resulting in a further reduction in portal pressure,
may be responsible.

22. Which of the following statements about the peritoneovenous shunt (PVS) is/are correct?
A. For cirrhotic patients with intractable ascites, the LeVeen shunt is an effective “bridge” to liver transplantation.
B. Replacement of ascites with saline or lactated Ringer's solution reduces the coagulopathy following PVS.
C. For patients with cirrhotic ascites, the survival using repeated paracentesis with 5% albumin infusion is equivalent to
that with the PVS.
D. Oliguria (less than 25 ml. per hour) in the immediate postoperative period following PVS should be treated with a 5%
albumin infusion.
E. The transjugular intrahepatic portacaval shunt with stent (TIPSS) works on the same principle as the PVS.
Answer: BC

DISCUSSION: The PVS is a palliative procedure that does not prolong life. In comparing the early risks of the procedure
with those of repeated paracentesis, the shunt cannot be justified as a temporizing procedure to facilitate ascites control in
the patient awaiting liver transplantation. Oliguria is common in the first 24 hours after shunt insertion. A correctly placed
PVS (patency confirmed using an intraoperative “shuntogram”) expands the intravascular volume with a continuous
reinfusion of ascites. Inspection should identify elevation of the jugular venous pressure, and a diuretic (usually furosemide)
is needed. The mechanisms of action of the two shunts are very different. TIPSS reduces portal pressure and controls ascites
by reducing the rate of ascites formation. PVS reinfuses the ascites fluid, thereby reducing the prerenal stimulus to sodium
retention and making the patient more responsive to diuretic therapy.

23. Which of the following clinical situations are considered good indications for PVS?
A. A 50-year-old cirrhotic man had an emergency portacaval shunt for bleeding varices and postoperatively had an ascites
leak and mild superficial wound infection.
B. A 57-year-old woman with primary biliary cirrhosis (PBC) has difficult to control ascites and diuretic-induced
encephalopathy.
C. A 46-year-old resistant alcoholic has chronic ascites uncontrolled by diuretics combined with repeat paracentesis.
D. A 34-year-old woman taking BCPs had rapid onset of ascites and is found to have hepatic vein thrombosis causing the
Budd-Chiari syndrome.
Answer: C

DISCUSSION: Because of the high complication rate and the long-term failure rate, the PVS is used only when other, more
lasting options for therapy either are not available or are contraindicated. The chronic alcoholic patient may benefit from a
peritoneovenous shunt because his ascites is the dominant problem related to his chronic liver disease, and persistent
alcoholism is a contraindication to liver replacement in most centers. PVS may be quite effective for the temporary
management of acute intractable postoperative ascites, such as in patient A; however, it is absolutely contraindicated in the
presence of infection. Patient B has ascites as her dominant problem as well; however, with PBC as the underlying liver
disease, she is an excellent candidate for transplantation. Patient D also has ascites as the major problem; however, the side-
to-side portosystemic shunt is a far better long-term treatment option than PVS.
24. Which of the following explanations account(s) for the fact that hepatitis C is the most common cause of
posttransfusion hepatitis?
A. There are more carriers of hepatitis C virus (HCV) in the normal population who serve as blood donors.
B. Blood infected with hepatitis B virus (HBV) is eliminated through routine testing, leaving only HCV as the other blood-
borne pathogen.
C. Current serologic tests for HCV antigen do not exclude carriers.
D. Questions designed to eliminate risk groups for HCV from the normal donor population may not be as specific as would
be desirable.
E. Hepatitis C is a more virulent form of viral hepatitis, so it is expected that more cases of posttransfusion hepatitis would
occur.
Answer: BD

DISCUSSION: The ability to specifically identify persons infected with HCV has only recently become available.
Therefore, data about epidemiology are less than complete. It is very likely not true that more blood donors carry HCV
because of the large preponderance of HBV in the United States. It is true, however, that successful elimination of most of
the HBV carriers occurs through routine testing. Although serologic tests are available for HCV, they are tests, not of
antigen, but of antibody. Therefore, this test alone may not screen out persons who are infected but have not yet developed
or may never develop antibody. Risk groups for the relatively newly defined HCV may well not be comprehensively
established, and therefore this explanation may be a contributor. There are no differences in virulence between these classes
of hepatitis virus.

25. True or false: HBV infections:


A. Are usually asymptomatic.
B. May not be clinically recognized but may lead to chronic hepatitis.
C. Reliably protect against subsequent HBV infection regardless of the measured antibody titer to hepatitis B surface
antigen (HBsAg).
D. Are completely prevented by postexposure administration of HBIg hepatitis B immunoglobulin (HBIg).
E. Preclude subsequent infection with HDV.
Answer: TRUE: BC, FALSE ADE

DISCUSSION: Although some types of hepatitis are more often asymptomatic than symptomatic, that is not the case for
hepatitis B. Further, even if the HBV infection is asymptomatic, serious long-term side effects may occur. A prior infection
with hepatitis B confers lifelong immunity even if the antibody titer wanes below the protective level of 10 mIU. HBIg is
useful in reducing the incidence of postexposure HBV infection from around 30% with no intervention, to 15% with
standard immune globulin, to about 5% to 7% with HBIg. HBV infection is required for infection with HDV and is
therefore an essential step toward, rather than preventive of, HBV infection.

26. Which of the following statements about choledocholithiasis are correct?


A. Common duct stones can originate in the gallbladder and migrate to the common duct, and stones can form de novo in
the duct system.
B. Calcium bilirubinate stones are associated with the presence of bacteria in the duct system.
C. Common duct stones discovered at laparoscopic cholecystectomy should be treated by postoperative endoscopic
extraction.
D. The serum bilirubin value is usually greater than 15 mg. per dl. in the patient with a symptomatic common duct stone.
Answer: ABC

DISCUSSION: Most common duct stones originate in the gallbladder and migrate to the common duct, where they may
become larger. These stones tend to consist predominantly of cholesterol (about 80% of gallbladder stones are
predominantly cholesterol). Stones found in the bile ducts after cholecystectomy may have been overlooked, but de novo
stone formation does occur. Arbitrarily, stones found 2 years after cholecystectomy are assumed to have formed within the
duct system. Calcium bilirubinate stones are thought to result from precipitation of insoluble bilirubin monoglucuronide
formed by deconjugation of bilirubin diglucuronide, a reaction promoted by the enzyme beta-glucuronidase, which is
produced by bacteria in the biliary tract. Calcium bilirubinate stones are found almost exclusively in patients who have
some form of biliary tract lesion that causes partial obstruction, and these patients tend to have bactibilia. Stones smaller
than approximately 5 mm. often can be extracted through a dilated cystic duct or pushed into the duodenum. Larger stones
are best left for postoperative endoscopic sphincterotomy and extraction. Patients with more than five stones or stones larger
than 1.5 cm. should be treated by open choledocholithotomy or, when indicated, a biliary-enteric anastomosis. Not all
patients with symptomatic common duct stones have elevated serum bilirubin, but when jaundice is present the bilirubin is
only rarely greater than 15 mg. per dl.

27. A benign biliary duct stricture:


A. Need not be treated unless it causes clinical jaundice.
B. Should always be treated by percutaneous balloon drainage.
C. Is prone to recur after treatment with biliary-enteric anastomosis.
D. When due to chronic pancreatitis should be treated by side-to-side choledochoduodenostomy.
Answer: CD

DISCUSSION: Even a minor obstructing lesion in the extrahepatic duct system can produce cirrhosis over time, and the
development of portal hypertension, ascites, and esophageal varices. Therefore, all biliary strictures should be treated unless
this is not possible or there is no chance for success. The presence or absence of jaundice is of no significance. Often, the
only biochemical abnormality is mild elevation of alkaline phosphatase. The long-term results of percutaneous balloon
dilatation are not yet known, but short-term results are good. Although some argue that balloon dilatation should be the
initial treatment, its role is ill-defined, and it should not be viewed as standard therapy at this time. Biliary-enteric
anastomoses are predisposed to stricture, for reasons that are ill-understood. A mucosa-to-mucosa anastomosis, large size of
the anastomosis, a normal duct at the point of anastomosis, and stenting appear to be elements that work against stricture.
About 70% of anastomoses are not complicated by strictures. Common duct strictures caused by chronic pancreatitis are
located in the distal portion of the duct and are easily treated by side-to-side choledochoduodenostomy. A wide anastomosis
is usually possible, and because of this stenting often is not necessary. Although a Roux-en-Y biliary-enteric reconstruction
is acceptable treatment, no advantage over choledochoduodenostomy has been demonstrated.

28. Which statements about extrahepatic bile duct cancer are correct?
A. Cholangiography is essential in evaluating patients for resectability.
B. The prognosis is excellent when appropriate surgical and adjuvant therapy are given.
C. The location of the tumor determines the type of surgical procedure.
D. The disease usually becomes manifest by moderate to severe right-side upper quadrant pain.
Answer: AC

DISCUSSION: Cholangiography is essential for both diagnosis and evaluation of resectability. Brushings of the lesion for
diagnosis and temporary stenting, done percutaneously or endoscopically, are often done at the time of cholangiography.
Angiography and CT are helpful, but in the absence of hepatic artery or portal vein occlusion these tests are not accurate
predictors of resectability. The primary obstacles to complete resection are invasion of the portal vein or the hepatic artery
and proximal extension of the tumor into the liver. The long-range prognosis for patients who undergo treatment for
extrahepatic bile duct cancer is poor, even when the lesion is surgically resectable and adjuvant therapy is given. Only about
10% of patients are alive without disease at 10 years. Nevertheless, bile duct cancer tends not to metastasize to distant sites,
so resection and radiation therapy are useful in prolonging symptom-free life. Tumors in the proximal third of the
extrahepatic bile duct system are treated by a Roux-en-Y biliary-enteric anastomosis. To ensure excision of the entire tumor
this anastomosis usually must be made to the individual hepatic ducts, which must be stented individually. Tumors of the
middle third usually require anastomosis to the proximal hepatic duct. In contrast, lesions of the distal third require
Whipple's procedure with appropriate reconstruction. Thus, the treatment of extrahepatic bile duct cancer depends on the
location of the tumor. Pain is not a prominent feature of bile duct cancer. Most cases become manifest by the insidious
development of jaundice.

29. Which of the following statements about biliary tract problems are correct?
A. Choledochal cyst should be treated by Roux-en-Y cystojejunostomy.
B. Sclerosing cholangitis is characterized by long, narrow strictures in the extrahepatic biliary duct system.
C. Operative (needle) cholangiography is indicated in patients who at operation appear to have no gallbladder.
D. The long cystic duct, which appears to be fused with the common duct and enters it distally, should be dissected free and
ligated at its entrance into the common duct.
Answer: C
DISCUSSION: In the past, choledochal cyst was treated by Roux-en-Y cystojejunostomy, but long-term results were poor.
Excision of the cyst is essential to prevent recurrent pancreatitis. In addition, the development of carcinoma in about 25% of
patients mandates cyst excision. Accordingly, excision of the cyst with biliary reconstruction by Roux-en-Y
hepaticojejunostomy and diversion of the flow of pancreatic juice through the ampulla of Vater is currently the standard
treatment. Sclerosing cholangitis causes fibrosis of bile ducts both within and outside the liver. This process, which is poorly
understood, causes strictures in the duct system, characteristically with normal or dilated segments between strictures.
Unfortunately, this anatomic arrangement does not lend itself to biliary reconstructive procedures. Each case must be
analyzed, however, because in some patients the anatomic situation may lend itself to balloon dilatation or reconstruction.
When the gallbladder appears to be absent, a search should be made for an ectopically located organ in the retroduodenal
area, within the falciform ligament, and within the substance of the right lobe of the liver. With true gallbladder agenesis the
common duct may be dilated, and choledocholithiasis is present in about one fourth of those who undergo operation.
Therefore, operative needle cholangiography should always be done. Dissection of a long, fused cystic duct is fraught with
hazard because the cystic and common ducts may share a common wall and serious duct damage may occur. The cystic duct
should be ligated and divided immediately proximal to the area of fusion.

30. Which of the following statements about the diagnosis of acute calculous cholecystitis are true?
A. Pain is so frequent that its absence almost precludes the diagnosis.
B. Jaundice is present in a majority of patients.
C. Ultrasonography is the definitive diagnostic test.
D. Cholescintigraphy is the definitive diagnostic test.
Answer: AD

DISCUSSION: The presence of pain is the sine qua non of acute calculous cholecystitis. Chronic cholecystitis associated
with cholelithiasis may develop in the absence of pain, and in critically ill patients pain may not be a prominent feature of
acute acalculous cholecystitis. Only about 10% of patients with acute cholecystitis are jaundiced. Although an occasional
patient may have concomitant bile duct obstruction, the jaundice associated with acute cholecystitis is probably due to
absorption of bile pigments from the diseased gallbladder. The presence of jaundice in a patient with right-side upper
quadrant pain should also suggest the possibility of acute cholangitis secondary to bile duct obstruction. Ultrasonography is
very accurate in the detection of gallstones, but stones may be present in the absence of acute cholecystitis. Thickening of
the gallbladder wall and a collection of fluid around the gallbladder are ultrasonographic findings in some patients with
acute cholecystitis, but they are not always present and are not specific. Ultrasonography may be useful when the diagnosis
is obscure because other conditions in the liver, pancreas, and kidney can be detected; however, it is not the definitive test
for acute cholecystitis. Cholescintigraphy is specific for the diagnosis of acute calculous cholecystitis (accuracy over 95% in
experienced hands). The rapidity, simplicity, and accuracy make cholescintigraphy the definitive diagnostic test in acute
calculous cholecystitis; however, it must be interpreted cautiously in the context of another critical illness or recent surgery
or trauma, because false-positives are not unusual in these situations.

31. Which statements about acute acalculous cholecystitis are correct?


A. The disease is often accompanied by or associated with other conditions.
B. The diagnosis is often difficult.
C. The mortality rate is higher than that for acute calculous cholecystitis.
D. The disease has been treated successfully by percutaneous cholecystostomy.
Answer: ABCD

DISCUSSION: About half of the cases of acute acalculous cholecystitis are associated with other conditions, including
sepsis, sarcoidosis, polyarteritis nodosa, and systemic lupus erythematosus. A majority of cases occur after trauma, burns, or
major surgical procedures performed for other conditions. The precise pathogenesis has not been determined. The diagnosis
of acute acalculous cholecystitis is often difficult because symptoms may be masked by another illness, injury, or the
postoperative state. Unlike acute calculous cholecystitis, in which pain is always present, pain occurs in only about 70% of
cases. In addition, cholescintigraphy is sometimes inaccurate. These factors make the diagnosis difficult, and a high index of
suspicion is necessary, especially in patients who have had operations or trauma. Unexplained abdominal pain, sepsis, and
ileus should prompt a thorough investigation. The mortality rate for acute acalculous cholecystitis is higher than that of the
calculous type. The incidence of gangrene and perforation of the gallbladder is higher. The accompanying illnesses and
conditions and the frequent delays in diagnosis undoubtedly contribute to the higher death rate. Percutaneous
cholecystostomy has been used as a diagnostic and therapeutic maneuver in patients who are thought to have acute
acalculous cholecystitis. Aspiration and culture of bile assist in confirming the diagnosis, and continuous drainage
successfully treats the acute condition. Surprisingly, persistent gangrene and subsequent complications have been
infrequent. Immediate cholecystectomy should be done if significant improvement does not take place within 12 hours of
percutaneous cholecystostomy. Long-term management of the tube and the need for elective cholecystectomy must be
individualized. The experience with percutaneous cholecystostomy is too small to determine whether this technique reduces
the mortality rate.

32. True statements about the surgical management of patients with acute calculous cholecystitis include:
A. Operation should be performed in all patients as soon as the diagnosis is made.
B. Antibiotic therapy should be initiated as soon as the diagnosis is made.
C. Dissection of the gallbladder is facilitated by decompression of the organ with the use of a trocar.
D. An operative cholangiogram should be done in every patient.
Answer: BC

DISCUSSION: Cholecystectomy should be done in an otherwise healthy patient as soon as the diagnosis is made and the
patient is properly prepared for surgery. However, patients who have one or more significant risk factors such as a recent
myocardial infarction, unstable angina, clinically significant coronary artery disease, or cirrhosis should not have immediate
cholecystectomy unless they do not improve within 24 to 36 hours in response to antibiotic administration and supportive
care. Antibiotic administration should commence as soon as the diagnosis is made and should be continued for 24 hours
postoperatively—or for 7 days if significant peritonitis is present. This use of antibiotics has significantly reduced septic
complications after cholecystectomy for acute cholecystitis. In most cases the gallbladder is tensely distended, making
visualization and dissection of the cystic duct area difficult and perhaps dangerous. Decompression of the gallbladder by
insertion of a needle facilitates retraction and dissection of the gallbladder. Although some advise that operative
cholangiography be done only on a selective basis, its routine use helps to delineate anatomy and facilitates detection of an
occasionally unsuspected bile duct stone. Accordingly, it is used routinely in elective cases. In acute cholecystitis, however,
the biliary duct system may be very friable, and operative cholangiography should be done only when it is safe to do so.

33. Which of the following are indications for cholecystectomy?


A. The presence of gallstones in a patient with intermittent episodes of right-side upper quadrant pain.
B. The presence of gallstones in an asymptomatic patient.
C. The presence of symptomatic gallstones in a patient with angina pectoris.
D. The presence of asymptomatic gallstones in a patient who has insulin-dependent diabetes.
Answer: A

DISCUSSION: Cholecystectomy (and concomitant operative cholangiography) are indicated for symptomatic patients to
relieve pain and to prevent the development of acute cholecystitis and its complications. Morbidity and expense are not as
great for elective cholecystectomy as they are for cholecystectomy for acute cholelithiasis. The risk of the development of
symptoms in patients who have asymptomatic stones is approximately 2% per year, a rate associated with mortality and
morbidity that do not exceed those of elective cholecystectomy. Therefore, cholecystectomy is not indicated for
asymptomatic patients. Patients who have angina pectoris should not have cholecystectomy until their coronary artery
disease has been treated adequately, even if this requires a coronary artery bypass procedure. Heart disease is the most
frequent cause of death after cholecystectomy. Prophylactic cholecystectomy, formerly recommended for insulin-dependent
diabetics, is not indicated because several studies have shown that the mortality rate from acute cholecystitis is no higher for
diabetics than for nondiabetics.

34. Which of the following statements about laparoscopic cholecystectomy are correct?
A. The procedure is associated with less postoperative pain and earlier return to normal activity.
B. The incidence of bile duct injury is higher than for open cholecystectomy.
C. Laparoscopic cholecystectomy should be used in asymptomatic patients because it is safer than open cholecystectomy.
D. Pregnancy is a contraindication.
Answer: AB

DISCUSSION: Studies have clearly documented that postoperative pain following laparoscopic cholecystectomy is less
than that experienced after open cholecystectomy and that patients can resume normal activity sooner. This appears to be
related to the reduced trauma to the abdominal wall by virtue of the very small incisions used in laparoscopic procedures.
The best evidence is that the bile duct injury rate (0.4%) is approximately double that for open cholecystectomy. The
incidence of this serious complication will probably decrease with improved techniques, better training, and more advanced
instrumentation. Only symptomatic patients should have cholecystectomy. Prophylactic removal of the gallbladder is not
cost effective. All elective operations are contraindicated in the first trimester, so as to prevent fetal anomalies and
spontaneous abortion. The laparoscopic technique is not contraindicated thereafter except in patients in whom peritoneal
access cannot safely be established. This is rarely a problem. Premature labor is a risk in the third trimester. Thus, unless
cholecystectomy can be avoided altogether during pregnancy, the second trimester is the most propitious time.

35. Which of the following statements about cholangitis are correct?


A. Charcot's triad is always present.
B. Associated biliary tract disease is always present.
C. Chills and fever are due to the presence of bacteria in the bile duct system.
D. The most common cause of cholangitis is choledocholithiasis.
Answer: BCD

DISCUSSION: Although Charcot's triad (pain, chills and fever, jaundice) is diagnostic of cholangitis, the complete triad
occurs only in 50% to 70% of patients. Fever is the most common symptom; therefore, cholangitis should be considered in
all patients who have unexplained fever. Episodes of pain, chills, and fever are often so brief as not to concern the patient.
Cholangitis does not occur in the absence of partial or complete bile duct obstruction. All patients diagnosed as having
cholangitis should have appropriate diagnostic studies to determine the cause. This usually involves cholangiography. The
presence of bacteria in bile does not produce symptoms in the absence of partial or complete obstruction of the bile duct
system. When obstruction is present, pressure within the system increases, giving rise to reflux of bacteria or their toxic
products into the hepatic venous circulation. This cholangiovenous reflux produces chills, fever, and the hemodynamic
changes of sepsis. Death may ensue if treatment is not instituted promptly. Choledocholithiasis, the most commonly
associated problem, may produce partial or complete obstruction. When bacteria are not present in the bile duct system,
choledocholithiasis may go undetected unless the degree of obstruction is sufficient to cause jaundice. Other causes of
cholangitis are benign and malignant strictures, biliary-enteric anastomoses, invasive procedures, foreign bodies, and
parasitic infestation of the bile ducts.

36. Recurrent episodes of cholangitis:


A. Suggest the presence of undetected or overlooked bile duct pathology.
B. Occur frequently in patients who have indwelling biliary tubes or stents.
C. May be ameliorated by long-term administration of antibiotics.
D. May be associated with the development of secondary biliary cirrhosis.
Answer: ABCD

DISCUSSION: Cholangitis does not occur in the presence of a normal bile duct system, and all patients with cholangitis
have an abnormality. Thus, recurrent episodes of cholangitis signal the need for diagnostic studies. Cholangiography usually
will be necessary. The presence of any foreign body in the biliary tract is frequently associated with bactibilia and recurrent
episodes of cholangitis. Even a silk suture exposed to the lumen of a bile duct has been known to cause cholangitis. Pigment
stone and sludge formation may result from the bacterial deconjugation of bilirubin diglucuronide to bilirubin
monoglucuronide, which precipitates as calcium bilirubinate. This material can occlude indwelling tubes and predispose to
more frequent episodes of cholangitis. Long-term administration of an oral antibiotic may reduce the frequency and severity
of attacks of cholangitis; however, this method of management should not be routine. Correction of the underlying problem
is essential. Chronic obstruction and recurrent infection eventually lead to secondary biliary cirrhosis and its complications
of portal hypertension, ascites, and bleeding esophageal varices. Once this stage of the disease is reached, correction of the
underlying biliary tract problem does not reverse the changes in the liver. Once again, every effort should be made to
eliminate the cause of the cholangitis early in the course of disease. The only effective treatment for end-stage liver disease
is hepatic transplantation.

37. The initial goal of therapy for acute toxic cholangitis is to:
A. Prevent cholangiovenous reflux by decompressing the duct system.
B. Remove the obstructing stone, if one is present.
C. Alleviate jaundice and prevent permanent liver damage.
D. Prevent the development of gallstone pancreatitis.
Answer: A

DISCUSSION: Uncontrolled sepsis and the consequent multisystem organ failure are the life-threatening sequelae of acute
toxic cholangitis. Thus, the initial goal of treatment is to decompress the biliary duct system to prevent reflux of bacteria and
their toxic products into the circulation. This can be done by intubating the duct system through the percutaneous,
transhepatic, or the endoscopic route or by insertion of a T tube in the common duct at operation. Removal of the stone
causing the obstruction is not necessary to stabilize the patient. Only after the duct is decompressed should the cause of the
obstruction be addressed. When transhepatic biliary drainage has been used, endoscopic or surgical removal of the stone can
be carried out after the patient has recovered completely. When initial therapy is sphincterotomy, the stone should be
removed as part of the procedure. Often the stone falls out without manipulation. If surgical placement of a T tube is the
initial treatment, the stone should be removed only if it is convenient to do so. The long-range goal of treatment of patients
with bile duct obstruction is to prevent cirrhosis, ascites, portal hypertension, and hemorrhage from esophageal varices;
however, death from sepsis is the immediate threat in acute toxic cholangitis. Gallstone pancreatitis may occur in patients
who have an impacted stone in the distal duct, independent of the presence or absence of acute toxic cholangitis; however,
gallstone pancreatitis is more often associated with the passage of a stone into the duodenum.

38. The clinical picture of gallstone ileus includes which of the following?
A. Air in the biliary tree.
B. Small bowel obstruction.
C. A stone at the site of obstruction.
D. Acholic stools.
E. Associated bouts of cholangitis.
Answer: ABCE

DISCUSSION: An antecedent biliary-enteric fistula is necessary to allow stone migration into the intestinal tract, and this
results in air entering the biliary tree (pneumobilia). It also allows contamination of the bile ducts with intestinal bacteria,
which in fact occurs in only a minority of such cases. The stone obstructs the narrower distal bowel, producing small bowel
obstruction. Such a stone, if opaque, can be seen on plain radiography and, if not, can be appreciated by sonography. Stools
are not acholic, since the cholecystoenteric fistula allows bile access to the intestinal lumen.

39. Which of the following statement(s) about gallstone ileus is/are not true?
A. The condition is seen most frequently in women older than 70.
B. Concomitant with the bowel obstruction, air is seen in the biliary tree.
C. The usual fistula underlying the problem is between the gallbladder and the ileum.
D. When possible, relief of small bowel obstruction should be accompanied by definitive repair of the fistula since there is
a significant incidence of recurrence if the fistula is left in place.
E. Ultrasound studies may be of help in identifying a gallstone as the obstructing agent.
Answer: C

DISCUSSION: It is true that gallstone ileus occurs mostly in elderly women and should always be suspect when small
bowel obstruction presents in this age group. The great majority of cases of gallstone ileus are preceded by a spontaneous
fistula occurring between the gallbladder and duodenum, allowing gallstones to enter the intestinal tract, which can
potentially block the terminal ileum. Finding air within the biliary tree should always arouse suspicion of the possibility of
this diagnosis when it is associated with a radiographic pattern of small bowel obstruction. The initial part of the operative
approach to this disease is to relieve the bowel obstruction by performing an enterotomy just proximal to the point of
obstruction to remove the stone. Where possible, definitive repair of the fistula should be undertaken to avoid recurrent
obstruction and to obviate the possible recurring complications of cholangitis. Percutaneous drainage of bile collections
combined with endoscopic papillotomy may be sufficient treatment for external and internal biliary fistulas but is never an
allowable approach in the presence of gallstone ileus with small bowel obstruction. Relief of the obstruction is mandated in
this setting.

40. Which of the following lesions are believed to be associated with the development of carcinoma of the gallbladder?
A. Cholecystoenteric fistula.
B. A calcified gallbladder.
C. Adenoma of the gallbladder.
D. Xanthogranulomatous cholecystitis.
E. All of the above.
Answer: E
DISCUSSION: The prevalence of carcinoma of the gallbladder in patients who have or have had a cholecystoenteric fistula
is believed to be 15%. The prevalence of carcinoma in a calcified, or “porcelain,” gallbladder is reported to range from
12.5% to 61%. It is generally accepted that adenoma of the gallbladder is a precancerous lesion that presents as a polypoid
lesion. Xanthogranulomatous cholecystitis is a rare form of chronic cholecystitis believed to be associated with a higher
incidence of cancer. This form of cholecystitis is also important because, grossly, it may mimic cancer of the gallbladder.

41. The preferred treatment for carcinoma of the gallbladder is:


A. Radical resection that includes gallbladder in continuity with the right hepatic lobe and regional lymph node dissection.
B. Radiation therapy.
C. Chemotherapy.
D. Combined treatment involving surgical therapy, chemotherapy, and radiation.
E. None of the above.
Answer: E

DISCUSSION: Radical resection, radiation therapy, and chemotherapy have been effective only anecdotally. Most believe
that the dismal prognosis of carcinoma of the gallbladder does not justify anything more than palliative treatment. About
88% of patients are dead within a year of diagnosis, and only about 4% are alive after 5 years, regardless of the type of
treatment. Those whose surgeon was unaware of the presence of the tumor at the time of cholecystectomy (approximately
12% of cases) are most likely to survive long term. There are insufficient data to support conclusively the proposition that
the patient with unexpected carcinoma found on histologic examination should undergo reoperation with intent for radical
excision. There also are indirect suggestions that the prognosis of gallbladder carcinoma may be improving, but it is not
clear if this is spontaneous or due to either earlier diagnosis or surgical management.

42. Which of the following statement(s) about pancreatic embryonic malformations is/are correct?
A. Pancreas divisum can be a cause of gastrointestinal bleeding.
B. Heterotopic pancreatic tissue predisposes to pancreatic adenocarcinoma.
C. Annular pancreas may cause gastrointestinal obstruction in children or in adults.
D. Relative obstruction to the flow of pancreatic juice through the minor papilla appears to be the cause of pancreatitis in
some patients with pancreas divisum.
Answer: CD

DISCUSSION: The clinically recognized embryonic malformations of the pancreas include heterotopic pancreas, pancreas
divisum, and annular pancreas. Heterotopic pancreatic tissue most often takes the form of a firm nodule of variable size in
the stomach, duodenum, small bowel, or Meckel's diverticulum. The typical complications of heterotopic pancreas include
intestinal obstruction, ulceration, or hemorrhage. Pancreas divisum is an anatomic variant that results from failure of fusion
of the two primordial pancreatic duct systems. In pancreas divisum the major portion of the pancreas is drained via the duct
of Santorini through the minor duodenal papilla. Relative stenosis of the minor duodenal papilla can cause pancreatitis.
Pancreas divisum is not associated with gastrointestinal bleeding. Annular pancreas results when histologically normal
pancreatic tissue completely or partially encircles the second portion of the duodenum. Varying degrees of duodenal
obstructive symptoms may be observed in both children and adults with this condition.

43. The pancreas occupies a retroperitoneal position in the upper abdomen. Which statement(s) is/are correct?
A. The superior mesenteric vein and the splenic vein join to form the portal vein posterior to the neck of the pancreas.
B. The uncinate process of the pancreas extends posterior to the inferior vena cava.
C. The tail of the pancreas extends to the left of the aorta, toward the splenic hilum.
D. The head of the pancreas is jointly supplied by arterial blood from the celiac axis and the superior mesenteric artery.
Answer: ACD

DISCUSSION: The pancreas occupies a retroperitoneal position in the upper abdomen, extending obliquely from the
duodenal C loop to a more cephalad position where the pancreatic tail abuts the hilum of the spleen. The portion of the
pancreas anterior to the confluence of the superior mesenteric vein, splenic vein, and portal vein is designated the neck of
the gland. The uncinate process extends posterior to the superior mesenteric vein and approaches the superior mesenteric
artery. The head of the pancreas is intimately associated with the second portion of the duodenum, and these two structures
are jointly supplied by two arterial arcades known as the anterior and posterior pancreaticoduodenal arteries, which
originate as branches of the celiac axis and superior mesenteric artery.
44. Both endocrine and exocrine tissue comprise the pancreas. Which statement(s) is/are true?
A. The islets of Langerhans total 1 million per gland and drain their secretions via intercalated duct cells through the
ampulla of Vater.
B. Islet alpha cells produce glucagon.
C. Islet sigma cells produce somatostatin.
D. The acini and ductal systems constitute the exocrine portion of the pancreas.
Answer: BD

DISCUSSION: The endocrine portion of the pancreas is served by the islets of Langerhans, which number 1 million islets
per gland. The islets of Langerhans drain their endocrine secretions into the bloodstream. Insulin-producing beta cells
comprise the majority of the islet population. Alpha cells produce glucagon and constitute approximately 20% to 25% of the
total islet cell number. Delta cells of the islets produce somatostatin. The acini and ductal systems constitute the exocrine
portion of the pancreas. The acinar cells contain zymogen granules in their narrow, centrally located apical portion. The
pancreatic duct system includes intercalated duct cells along the ductal pathway, terminating in the main excretory duct of
the pancreas.

45. Pancreatic exocrine secretory products include a bicarbonate-rich electrolyte solution as well as digestive enzymes.
Which of the following statement(s) is/are true?
A. Cholecystokinin (CCK) is the most potent endogenous stimulant of pancreatic enzyme secretion.
B. The chloride and bicarbonate concentrations of pancreatic juice vary and depend on the secretory flow rate.
C. Secretin is the most potent endogenous stimulant of pancreatic water and electrolyte secretion.
D. The peptidases synthesized by acinar cells are released into the pancreatic duct system in active form.
Answer: ABC

DISCUSSION: CCK is the most potent endogenous stimulant of pancreatic enzyme secretion. The pancreatic acinar cells
respond to CCK with release of their zymogen granules into the ductal system. Peptidases are released in inactive form,
later to be activated by contact with duodenal enterokinase and activated trypsin. Secretin is the most potent endogenous
stimulant of pancreatic water and electrolyte secretion. The concentrations of the anions bicarbonate and chloride vary and
are largely dependent on the secretory flow rate stimulated by secretin.

46. Which of the following parameters is/are not included in the Ranson's prognostic signs useful in the early evaluation of
a patient with acute pancreatitis?
A. Elevated blood glucose.
B. Leukocytosis.
C. Amylase value greater than 1000 U per dl.
D. Serum lactic dehydrogenase (LDH) greater than 350 IU per dl.
E. Alanine aminotransferase greater than 250 U per dl.
Answer: CE

DISCUSSION: Several prognostic systems have been demonstrated to predict the severity of pancreatitis accurately. Two
Ranson prognostic criteria have been developed: one each, for pancreatitis that is not due to gallstones and pancreatitis that
is. The systems have minor differences. In both of the Ranson systems elevated blood glucose, leukocytosis, and elevations
of serum LDH have proved to have prognostic importance. The degree of amylase elevation is not one of the parameters,
nor is the degree of ALT elevation.

47. Standard supportive measures for patients with mild pancreatitis include the following:
A. Intravenous fluid and electrolyte therapy.
B. Withholding of analgesics to allow serial abdominal examinations.
C. Subcutaneous octreotide therapy.
D. Nasogastric decompression.
E. Prophylactic antibiotics.
Answer: A

DISCUSSION: Standard therapy for all patients with mild acute pancreatitis should include intravenous fluid resuscitation,
electrolyte replacement, and analgesics. Nasogastric decompression is typically reserved for patients with significant ileus
who are at risk for emesis and aspiration. Subcutaneous therapy with octreotide, the octapeptide analog of somatostatin, has
not been proven to influence the outcome in patients with mild pancreatitis. Prophylactic antibiotics are not used for mild
pancreatitis. Antibiotics are reserved for patients with severe pancreatitis (defined as greater than three Ranson prognostic
signs with associated CT evidence of pancreatic or peripancreatic necrosis).

48. Which of the following statements about chronic pancreatitis is/are correct?
A. Chronic pancreatitis is the inevitable result after repeated episodes of acute pancreatitis.
B. Patients with chronic pancreatitis commonly present with jaundice, pruritus, and fever.
C. Mesenteric angiography is useful in the evaluation of many patients with chronic pancreatitis.
D. Total pancreatectomy usually offers the best outcome in patients with chronic pancreatitis.
E. For patients with disabling chronic pancreatitis and a dilated pancreatic duct with associated stricture formation, a
longitudinal pancreaticojejunostomy (Peustow procedure) is an appropriate surgical option.
Answer: E

DISCUSSION: Chronic pancreatitis is a clinical entity that includes recurrent or persistent abdominal pain with evidence of
exocrine and endocrine pancreatic insufficiency. While chronic pancreatitis may result from repeated episodes of acute
pancreatitis, not all patients with recurring acute pancreatitis progress to chronic pancreatitis. The most common causes of
chronic pancreatitis include alcohol abuse, hyperparathyroidism, congenital anomalies of the pancreatic duct, pancreatic
trauma, and cystic fibrosis. The most useful radiographic tests in patients with suspected chronic pancreatitis are CT and
endoscopic retrograde cholangiopancreatography (ERCP). Mesenteric angiography has no role in the evaluation of most
patients with chronic pancreatitis. Patients with disabling chronic pancreatitis who require operative intervention are
candidates for a longitudinal pancreaticojejunostomy (Peustow procedure) if pancreatography demonstrates a dilated
pancreatic duct. Total pancreatectomy is rarely performed because of the significant problems associated with labile insulin
sensitivity, steatorrhea, and weight loss.

49. Which of the following statements about pancreatic ascites is/are correct?
A. Patients typically present with painful ascites, reflecting the release of toxic pancreatic enzymes into the peritoneal
cavity.
B. The standard evaluation of a patient with new-onset ascites includes abdominal paracentesis. In cases of pancreatic
ascites, the peritoneal fluid contains high concentrations of both amylase and protein.
C. Pancreatic ascites can follow an episode of acute pancreatitis.
D. Patients with pancreatic ascites may fail to improve with nonoperative therapy and require surgical procedures. At
abdominal exploration an acceptable approach to the pancreatic duct disruption involves suture ligation with omental
patching.
Answer: BC

DISCUSSION: Pancreatic ascites typically occurs because of a pancreatic duct disruption, most commonly involving
alcohol abuse and resultant acute pancreatitis. In pancreatic ascites, pancreatic exocrine secretions exit a pancreatic duct
disruption and drain anteriorly into the peritoneal cavity. Patients typically present with painless massive ascites, as the
pancreatic enzymes that extravasate into the peritoneal cavity are typically nonactivated. The diagnosis of pancreatic ascites
is best made by paracentesis, in which the analysis of the ascites fluid reveals it to be high in amylase (more than 1000 U.
per dl.) and high in albumin (more than 3 gm. per dl.). Nonoperative treatment is initially indicated in most patients with
pancreatic ascites. Should nonoperative therapy fail, surgical therapy is directed to closure of the pancreatic duct disruption.
Preoperative pancreatography is useful in directing surgical therapy. Distal pancreatic duct disruption may be treated with
distal pancreatectomy or with Roux-en-Y pancreaticojejunostomy. Pancreatic leaks in the more proximal aspects of the
gland are treated with Roux-en-Y pancreaticojejunostomy. Suture ligation of the pancreatic duct with omental patching is
not considered appropriate therapy for pancreatic duct disruptions.

50. Which of the following statements about adenocarcinoma of the pancreas is/are correct?
A. It is the fifth most common cause of cancer death in the U.S.
B. Most cases occur in the body and tail of the pancreas, making distal pancreatectomy the most commonly performed
resectional therapy.
C. For cancers of the head of the pancreas resected by pancreaticoduodenectomy, prognosis appears to be independent of
nodal status, margin status, or tumor diameter.
D. The most accurate screening test involves surveillance of stool for carbohydrate antigen (CA 19–9).
Answer: A
DISCUSSION: Adenocarcinoma of the pancreas is newly diagnosed in approximately 28,000 patients in the United States
every year. It is the fifth most common cause of cancer death in the United States, exceeded only by lung, colorectal, breast,
and prostate cancer. The majority of cases of adenocarcinoma of the pancreas occur in the head of the gland, and if
resectable, are treated via pancreaticoduodenectomy. Recent studies have shown that factors favoring long-term survival
after pancreaticoduodenectomy for adenocarcinoma of the head of the pancreas include negative nodal status, negative
margin status, small tumor diameter, and diploid DNA content. No accurate screening tests for adenocarcinoma of the
pancreas are currently available. The best serologic test appears to be the CA 19–9, which is elevated in the majority of
patients with adenocarcinoma of the head of the pancreas. Unfortunately, the test is not sufficiently sensitive or specific, and
further screening tests are needed.

51. A 35-year-old woman presents with episodes of obtundation, somnolence, and tachycardia. An insulinoma is suspected
based on a random serum glucose test value of 38 mg. per dl. Which of the following statements is/are true?
A. The most important diagnostic study for insulinoma is an oral glucose tolerance test.
B. It may be helpful to perform ERCP in an effort to localize the tumor.
C. Most patients with insulinoma present with extensive disease, rendering them only rarely resectable or curable.
D. An important component of the preoperative evaluation in patients with presumed insulinoma involves confirming
elevated C-peptide or proinsulin levels and screening for anti-insulin antibodies.
Answer: D

DISCUSSION: Insulinoma is the most common endocrine tumor of the pancreas. Insulinoma is associated with Whipple's
triad, which consists of (1) symptoms of hypoglycemia at fasting; (2) documentation of blood glucose levels of less than 50
mg. per dl.; and (3) relief of symptoms following administration of glucose. The most reliable method for diagnosing
insulinomas is a monitored fast. Neither an oral or an intravenous glucose tolerance test is indicated in the majority of
patients being evaluated for insulinoma. Support for the diagnosis of insulinoma can come from documenting elevated C
peptide and proinsulin levels. Screening for anti-insulin antibodies is indicated to rule out the possibility of surreptitious
insulin administration. Tumor localization is typically performed with CT, endoscopic ultrasonography, or angiography.
ERCP is not indicated for evaluation of most pancreatic endocrine tumors, as the tumors only rarely communicate with the
main pancreatic duct system. As many as 90% of patients with insulinoma have benign solitary pancreatic adenomas
amenable to surgical cure.

52. Which of the following statements about gastrinoma (Zollinger-Ellison syndrome) is/are correct?
A. As many as 75% of gastrinoma patients have sporadic disease; 25% have gastrinoma associated with multiple endocrine
neoplasia type 1 (MEN 1).
B. Extrapancreatic gastrinomas are common, and exploration should include careful assessment of the duodenum and
peripancreatic lymph nodes.
C. Diarrhea may be a prominent presenting feature of some patients with gastrinoma.
D. Before elective operation acid-reducing medications such as omeprazole should be administered.
Answer: ABCD

DISCUSSION: Gastrinoma patients typically present with peptic ulceration of the upper gastrointestinal tract and
abdominal pain. As many as 50% of patients may have diarrhea, which may be a prominent feature in some cases.
Approximately 25% of gastrinoma patients have the disease associated with the MEN-1 syndrome, whereas 75% have a
sporadic variety of the disease. Recent evidence indicates that extrapancreatic gastrinomas are common. Careful attention
must be paid to the duodenum and peripancreatic lymph nodes at the time of abdominal exploration. Before elective
operation it is imperative that the gastric acid hypersecretion be controlled. The control of gastric hypersecretion is best
performed by the administration of one of the substituted benzimidazoles, such as omeprazole or lansoprazole.

53. With regard to the control of pancreatic exocrine function, which of the following statement(s) is/are correct?

a.Cholecystokinin, a hormone released from the duodenal mucosa, is the predominant stimulus for pancreatic
enzyme secretion
b.Gastrin is a major stimulant for pancreatic bicarbonate secretion
c.Secretin is released from the duodenum upon mucosal acidification and stimulates pancreatic bicarbonate secretion
d.Acetylcholine, released from pancreatic nerves, stimulates enzyme secretion
Answer: a, c, d
Enzyme secretion is regulated primarily through hormonal and neural factors. The enteric hormone cholecystokinin,
released from endocrine cells in the duodenal mucosa, is the predominant regulator and stimulates acinar cells through
specific membrane-bound receptors. Acetylcholine strongly stimulates acinar cells when released from postganglionic fibers
of the pancreatic plexus and acts in synergy with CCK to potentiate enzyme secretion. Secretin weakly stimulates acinar cell
secretion and potentiates the effect of cholecystokinin on the acinar cells.
Bicarbonate is formed from carbonic acid by the enzyme carbonic anhydrase. Secretin, the major stimulant for bicarbonate
secretion, is released from the duodenal mucosa in response to a duodenal luminal pH of less than 3.0. Cholecystokinin only
weakly stimulates bicarbonate secretion, whereas it potentiates secretin-stimulated bicarbonate secretion. Gastrin and
acetylcholine are weak stimulants of bicarbonate secretion.

54. In the performance of a pancreaticoduodenectomy (Whipple procedure), the superior mesenteric vein is an
important landmark. Which of the following statements is/are true with regard to the superior mesenteric vein?

a.Small venous branches enter the superior mesenteric vein anteriorly as it courses beneath the neck of the pancreas
b.The superior mesenteric vein joins the splenic vein at the superior border of the pancreas to form the portal vein
c.Small venous branches enter the superior mesenteric vein laterally as it courses beneath the neck of the pancreas
d.The superior mesenteric vein courses anterior to the neck of the pancreas
Answer: b, c

The venous drainage of the pancreas and duodenum follows the arterial supply. The anterior and posterior venous arcades
drain the head; the body and tail drain into the splenic vein. All venous effluent from the pancreas ultimately drains into the
portal vein which is formed by the confluence of the superior mesenteric vein and the splenic vein at the superior border of
the pancreas. The anterior and posterior venous arcades in the head of the pancreas drain directly into the suprapancreatic
portal vein. The anteroinferior pancreaticoduodenal arcades drain with the right gastroepiploic vein to form a common
venous trunk with the right colic vein. This trunk is known as the gastrocolic trunk and enters the superior mesenteric vein
at the inferior border of the neck of the pancreas. The posteroinferior venous arcade empties directly into the superior
mesenteric vein. The veins of the head drain laterally into the superior mesenteric and portal veins. There are no venous
tributaries entering the superior mesenteric vein anteriorly. For this reason, it is safe to dissect the neck of the pancreas
directly anterior to the superior mesenteric and portal veins when performing a pancreaticoduodenectomy.

55. Pancreas divisum results from incomplete fusion of the ventral pancreatic duct with the dorsal pancreatic duct
during embryologic development. Which of the following statements correctly describes pancreas divisum?

a.The body and tail of the pancreas drain via an accessory ampulla distal to the ampulla of Vater. The uncinate
process drains via the ampulla of Vater
b.The entire pancreatic ductal system drains via the ampulla of Vater
c.The entire pancreatic ductal system drains via an accessory ampulla proximal to the ampulla of Vater
d.The body and tail of the pancreas are absent. The uncinate process drains via the ampulla of Vater
Answer: c

In 90% of individuals, the main pancreatic duct, or duct of Wirsung, runs the entire length of the pancreas and joins the
common bile duct to empty into the duodenum at the ampulla of Vater. The pancreatic duct is 2 to 3.5 mm in diameter and
contains 20 secondary branches, which drain the tail, body, and uncinate process. The drainage of the lesser duct, or duct of
Santorini, is variable. The lesser duct commonly drains the superior portion of the head of the pancreas. It empties
separately into the second portion of the duodenum through the lesser papilla located 2 cm proximal to the ampulla of Vater.
Pancreas divisum results from an incomplete fusion of the ventral pancreatic duct with the dorsal duct during fetal
development and is present in 5% of patients. In this anomaly, the lesser duct drains the entire pancreas via an accessory
ampulla located proximal to the ampulla of Vater. Inadequacy of this pattern of drainage can result in chronic pain.

56. Which of the following statements is/are correct with regard to the blood supply of the pancreas?

a.The inferior pancreaticoduodenal artery, a branch of the celiac artery, divides into anterior and posterior branches
to supply the pancreatic head
b.The body and tail of the pancreas are supplied by branches of the splenic artery
c.The superior pancreaticoduodenal artery is a branch of the gastroduodenal artery
d.The body and tail of the pancreas are supplied by branches derived from the left renal artery
Answer: b, c

The pancreas receives its blood supply from a variety of major arterial sources. In the head of the pancreas, there are arcades
in the anterior and posterior surfaces, which generally collateralize. These arcades arise from branches of the gastroduodenal
and the superior mesenteric arteries. Just distal to the first portion of the duodenum, the gastroduodenal artery becomes the
superior pancreaticoduodenal artery, which divides into anterior and posterior branches. The inferior pancreaticoduodenal
artery is the first branch of the superior mesenteric artery and divides into anterior and posterior branches.
The body and tail of the pancreas are supplied by the splenic artery. The splenic artery arises from the celiac trunk and
courses along the superior surface of the pancreas to the spleen. Approximately ten branches of the splenic artery supply the
body and tail of the pancreas.

57. Orally administered glucose provokes a greater insulin response than an equivalent amount of intravenously
administered glucose. The incremental response to ingested glucose is due to the effects of which of the following
hormones?

a.Gastric inhibitory peptide


b.Somatostatin
c.Pancreatic polypeptide
d.Secretin
Answer: a

Orally administered glucose stimulates a greater insulin response than an equivalent amount of intravenous glucose through
the release of enteric hormones that potentiate insulin secretion. This effect is known as the enteroinsular axis. Gastric
inhibitory polypeptide (GIP) appears to be an important regulator of this effect, although other gut peptides, such as
glucagon-like peptide I (GLP-1), may contribute to this effect as well. Nutrients that regulate insulin secretion include
amino acids, such as arginine, lysine, and leucine, and free fatty acids. Hormones that stimulate insulin secretion include
glucagon, GIP, and cholecystokinin, whereas somatostatin, amylin, and pancreastatin are inhibitory. Insulin is also
stimulated by sulfonylurea compounds, which act independently of the glucose concentration and form the basis of
treatment of type II, or insulin-independent, diabetes.

58. The islets of Langerhans contain four major endocrine cell types that secrete which of the following hormones?

a.Insulin, somatostatin, glucagon, secretin


b.Insulin, somatostatin, cholecystokinin, pancreatic polypeptide
c.Insulin, somatostatin, glucagon, pancreatic polypeptide
d.Insulin, secretin, glucagon, cholecystokinin
Answer: c

Within the pancreas are small nests of cells that are responsible for the secretion of hormones that control glucose
homeostasis. These nests are called islets of Langerhans and constitute 2% of the pancreatic mass. The islets contain an
average of 3000 cells and range in diameter from 40 to 900 mm. The islets are composed of four major cell types—alpha
(A), beta (B), delta (D), and PP or F cells, which secrete glucagon, insulin, somatostatin, and pancreatic polypeptide,
respectively. The B cells are centrally located within the islet and constitute 70% of the islet mass, whereas the PP, A, and D
cells are located at the periphery of the islet. They constitute roughly 15%, 10%, and 5% of the islet cell mass, respectively.

59. A 50-year-old man develops acute pancreatitis due to alcohol abuse. Hyperamylasemia resolves by the third day
after admission. By the eighth hospital day, the patient is noted to have recurrent fever (38.5°C), progressive
leukocytosis (18,500 WBC/mm3), and tachypnea. The most appropriate management includes which as the next
step?

a.Laparotomy with pancreatic debridement


b.CT guided aspiration of peripancreatic fluid collections
c.ERCP with sphincterotomy and placement of biliary stent
d.Intravenous amphotericin B
Answer: b
The common causes of pancreatic abscesses are infected pancreatic pseudocysts and necrotizing pancreatitis. The diagnosis
is suggested by persistent fever, leukocytosis, and a palpable abdominal mass. Bacteremia and systemic toxicity are late
clinical features. Percutaneous aspiration with positive cultures is the definitive preoperative test, facilitated by CT scanning
or ultrasound-guidance to suspicious peripancreatic fluid collections. When diagnosed, the treatment of choice is wide
surgical débridement with removal of all infected and revitalized tissues. Generous drainage is mandatory. One of the major
sources of morbidity and mortality in this situation is the late development of mycotic visceral pseudoaneurysms,
particularly involving the splenic circulation. These may be complex management problems, requiring angiographic
embolization or other innovative treatment strategies.

60. The patient in the above question is treated by observation for 8 weeks. He continues to be symptomatic with
epigastric pain. A repeat abdominal CT scan reveals a persistent 6 cm pseudocyst in the region of the body of the
pancreas. The pseudocyst is unilocular and demonstrates a well-defined rim of fibrous tissue. The gastric antrum is
displaced anteriorly. Using CT guidance, 300 ml of fluid is aspirated from the lesion which is shown to be
collapsed radiographically. No further intervention is performed. What is the risk of pseudocyst recurrence after
simple aspiration?

a.80–85%
b.60–65%
c.40–45%
d.20–25%
Answer: d

Generally, a pancreatic pseudocyst can be observed for a period of weeks or months in an effort to allow for spontaneous
resolution. Percutaneous ultrasound-or CT-directed aspiration or drainage catheter placement is an initial treatment option.
Simple aspiration is performed if the initial aspirate is sterile; if the aspirate is infected, a catheter or open drainage
procedure is appropriate. Determination of pancreatic ductal anatomy is important. Contrast injection into the pseudocyst at
the time of aspiration should be considered to assess the possibility of pancreatic ductal communication and obstruction, or
multiple cysts. The pseudocyst recurrence rate after simple aspiration is about 20% to 25%.

61. In prospective, randomized trials which of the following agents or therapeutic measures has/have been
demonstrated to accelerate recovery from acute pancreatitis?

a.Peritoneal lavage
b.Anticholinergic blockade
c.Octreotide
d.H2 receptor blockade
e.None of the above
Answer: e

A variety of pharmacologic agents that directly or indirectly reduce acinar cell enzyme release or ductal secretion have
undergone clinical evaluation for the treatment of acute pancreatitis—generally with unimpressive results. Among the first
were anticholinergic drugs. Despite extensive experience over many years, no objective data have emerged to support their
use. Clinical trials of glucagon and calcitonin based on the same principle have produced a similar lack of supportive data.
More recently, a somatostatin analog has been subjected to clinical trials for patients with acute pancreatitis. Somatostatin
inhibits pancreatic enzyme and bicarbonate secretion by preventing the normal release of cholecystokinin, secretin, and
other gut peptides. Despite the theoretical appeal, it has not been possible to demonstrate that somatostatin alters the natural
history or prognosis of simple acute pancreatitis, although it does diminish pancreatic secretion.
Peritoneal lavage as a specific therapy for acute pancreatitis was proposed after experimental studies demonstrated
improved survival in animals with fulminant pancreatitis. The concept was appealing in that activated proteases and other
vasoactive substances identifiable in peritoneal aspirates from patients with pancreatitis would be removed, rather than
systemically absorbed. Unfortunately, clinical trials using this approach have produced disappointing results, and the
eventual overall mortality rate appears unchanged.

62. Which of the following medical procedures has/have been associated with an increased risk of post-procedure
acute pancreatitis?
a.Common bile duct exploration
b.Endoscopic retrograde cholangiopancreatography
c.Coronary bypass grafting
d.Distal gastrectomy
Answer: a, b, c, d

Many surgical procedures in the upper abdomen are associated with postoperative pancreatitis. The incidence of acute
pancreatitis after gastric resection ranges from 0.6% to 1.23%. After biliary tract surgery, particularly after common bile
duct exploration itself, acute pancreatitis occurs with an incidence of 0.5% to 3%. Direct manipulation or retraction of the
pancreas or pancreatic duct appears to be the most common cause. About 1% of patients develop acute pancreatitis after
endoscopic retrograde cholangiopancreatography (ERCP). This is a predictable event, and the risk can be minimized by
limiting the pressure used for contrast injection of the pancreatic duct. Acute pancreatitis also occurs in patients after
coronary artery bypass surgery and a variety of other procedures remote from the pancreas. Although pancreatitis in this
circumstance is thought to result from ischemia, hypotension is not always noted. The systemic consequences of activation
of the inflammatory system may contribute to changes in microvascular blood flow.

63. A 42-year-old male develops acute pancreatitis in the setting of acute alcohol abuse. One week after onset of
symptoms, computed tomography of the abdomen reveals a pancreatic phlegmon and associated pseudocyst.
Which of the following factors, if present, would decrease the likelihood of spontaneous resolution of the
pseudocyst?

a.Size greater than 5 cm


b.Diffuse calcification of the pancreatic gland
c.Multilocularity
d.Location in the pancreatic tail
Answer: a, b, c

Initial management of pancreatic pseudocysts is based on symptoms. If the patient is asymptomatic and the cyst is small (<
5.0 cm) it can be safely observed as many of these will resolve over a period of weeks. Concurrent chronic alcoholic
pancreatitis (by history or as indicated by pancreatic calcification), pseudocyst size greater than 5 cm, the presence of a
multilocular or debris-filled pseudocyst cavity, and chronicity (longer than 6 weeks) are all factors that are associated with a
lower probability of spontaneous resolution.

64. Which of the following is/are prognostic signs reported by Ranson to predict outcomes associated with acute
pancreatitis?

a.Age greater than 60 years


b.Hematocrit decrease of 105 within 48 hours of hospital admission
c.Serum amylase value greater than 4 times upper limit of normal
d.Serum glucose greater than 200 mg/dL on admission
e.Ca2+ level less than 8 mg/dL within 48 hours of hospital admission
Answer: b, d, e

Ranson prognostic signs include:

ON ADMISSION
Age above 55 years
White blood cell count above 16,000/µL
Glucose level above 200 mg/dL
Lactase dehydrogenate level above 350 IU/L
SGOT value above 250 IU/L

AFTER 48 HOURS
Hematocrit decrease of 10%
Blood urea nitrogen level increase of 5 mg/dL
Ca2+ level below 8 mg/dL
PaO2 level below 60 mmHg
Base deficit value above 4 mEq/L
Fluid sequestration greater than 6 L

65. A 36-year-old woman is admitted to a the hospital with upper abdominal pain, hyperamylasemia, elevation of
serum alkaline phosphatase and ultrasound evidence of cholelithiasis. With intravenous hydration and analgesia,
symptoms rapidly resolved. After 48 hours, serum amylase and alkaline phosphatase values had returned to normal
and physical examination revealed lessening tenderness in the right upper quadrant of the abdomen. Appropriate
management consists of which of the following as the next step?

a.Cholecystectomy and intraoperative cholangiography before hospital discharge


b.Elective cholecystectomy at approximately 8 weeks
c.Endoscopic sphincterotomy before discharge followed by cholecystectomy at approximately 8 weeks
d.Observation
Answer: a

A patient who has simple cholelithiasis and an episode of acute pancreatitis is usually treated nonoperatively until clinical
resolution of the pancreatitis occurs. The rate of recurrent biliary pancreatitis is as high as 34% to 56% within 6 weeks;
therefore, an aggressive operative approach is appropriate. Cholecystectomy is often performed after the resolution of acute
pancreatitis but before hospital discharge. Common bile duct instrumentation in this setting has a substantially increased
risk of recurrent acute pancreatitis.

66. For the patient in the preceding question, symptomatic recurrence at 3 weeks after aspiration is confirmed
ultrasonographically. Endoscopic retrograde pancreatography does not demonstrate communication of a major
pancreatic duct with the pseudocyst. Appropriate management includes which of the following?

a.Pancreatectomy to include the pseudocyst


b.Cystgastrostomy
c.Repeat aspiration followed by injection of sodium morrhuate into the pseudocyst cavity
d.Pancreatic debridement followed by external drainage
Answer: b

The operative treatment for pseudocysts depends on the underlying cause of the cyst, as well as the size, location, and
maturity of the pseudocyst wall. Whenever possible, the status of the pancreatic duct should be assessed preoperatively,
preferably by ERCP. Operative drainage can be either external or internal. External drainage is chosen in the presence of
infection or an immature capsule. The disadvantages of external drainage include the risk of pancreatic fistula formation and
a pseudocyst recurrence. External drainage has been associated with a higher mortality rate, probably because it is used in
patients at higher risk, especially those with sepsis, pancreatic abscesses, or ruptured pseudocysts.
The type of internal drainage procedure selected depends on the location of the pseudocyst and whether or not there is
associated pancreatic ductal pathology. Cystogastrostomy is the simplest and safest alternative if the pseudocyst is
appropriately adjacent to the posterior wall of the stomach. Cystojejunostomy using a Roux-en-Y or loop jejunostomy may
also be appropriate, depending on the location and specific anatomy of the pseudocyst. Pancreatic resection is associated
with the lowest recurrence rate (3%), but is limited to pseudocysts occurring in the tail of the pancreas.

67. With regard to acute pancreatitis: which of the following statements is/are correct?

a.The majority of patients presenting with acute pancreatitis of biliary type are female
b.The majority of patients presenting with acute pancreatitis of alcoholic type are female
c.The most common cause of acute pancreatitis in the United States is alcohol use
d.Patients with alcohol-induced pancreatitis tend to be older than those with biliary-induced disease
Answer: a, c

In autopsy series, the evidence for past acute pancreatitis averages 0.31%. Variations among populations are highly
dependent on social factors such as ethanol use and on environmental and hereditary determinants such as the incidence of
gallstones. Acute pancreatitis may occur at any age but is most common in adults between 30 and 70 years of age. In
general, patients with gallstone-induced pancreatitis are older (age 40 to 60 years), whereas those with alcohol-associated
pancreatitis are younger (age 30 to 40 years). The sex distribution of acute pancreatitis depends on the clinical cause of the
disease, with women representing 68% of patients with gallstone-associated pancreatitis. Conversely, when alcohol is the
primary association, men account for most patients.
Clinical associations with acute pancreatitis can be divided into three broad categories-biliary stones, ethanol, and others.
Biliary tract stone disease and ethanol-induced pancreatitis account for most cases of acute pancreatitis reported worldwide.
The particular distribution of causes reflects the source of the patient population evaluated. In a summary of 18 different
reports of acute pancreatitis in the United States with a combined total of 7147 patients, 53% of patients were believed to
have ethanol-induced disease, whereas 28% had proven biliary stones. In contrast, of 1539 patients reported on from Great
Britain, 52% had gallstones, 7% were ethanol related, and 34% had no identifiable cause.

68. Which of the following statement(s) relating to chronic pancreatitis is/are correct?

a.In the United States, the most common cause of chronic pancreatitis is alcohol abuse
b.Approximately 50% of chronic alcoholics develop chronic pancreatitis
c.Clinically significant chronic pancreatitis develops on average after five years of alcohol abuse in men
d.The risk of alcohol-induced chronic pancreatitis can be decreased by consumption of a high-protein diet
Answer: a

In the United States, alcohol consumption is the major cause of chronic pancreatitis: with approximately 70% of cases
attributable to this factor. Most patients with symptomatic chronic pancreatitis have consumed large volumes of alcohol
daily for a prolonged period of time. The average daily intake of alcohol is 150 to 175 g with the mean duration of
alcoholism before recognition of pancreatitis being 18 years for men and 11 years for women. The incidence of chronic
pancreatitis on autopsy studies of chronic alcoholics is 50 times the rate of non-drinking controls. Only 10% of alcoholics
develop chronic pancreatitis—suggesting that factors other than long-term alcohol exposure may also influence
susceptibility. In both experimental and clinical studies, the risk of alcohol-induced chronic pancreatitis is increased by a
high-protein, high-fat diet.

69. The most appropriate test to confirm a clinical diagnosis of early chronic pancreatitis is which of the following?

a.Serum amylase determination


b.Calculation of urinary amylase clearance
c.Measurement of para-aminobenzoic acid absorption
d.Endoscopic retrograde cholangiopancreatography
Answer: d

Routine tests of blood or serum are not helpful in making a diagnosis of chronic pancreatitis. Although serum amylase
levels are almost always elevated in acute pancreatitis—amylase levels may be normal, elevated, or subnormal in chronic
pancreatitis. Determination of urinary amylase secretion and calculation of urinary amylase clearance does not improve
sensitivity or specificity. Indirect tests of pancreatic function which measure absorption of nutrients that first require
pancreatic digestion are not helpful in early cases of chronic pancreatitis. Clinically detectable malabsorption is absent until
90% of exocrine function is lost. Because of this, indirect tests of pancreatic function do not detect early disease. In
addition, false positive tests may occur in other disease states associated with malabsorption (Crohn’s disease, sprue,
postgastrectomy states, or in association with diabetes mellitus, cirrhosis, or renal disease. ERCP has become widely
recognized as the most sensitive and reliable method for diagnosing chronic pancreatitis. Sensitivity approaches 90% with
equal specificity.

70. A 52-year-old male, known to be alcoholic, is evaluated because of chronic abdominal pain. The clinical diagnosis
of chronic pancreatitis is supported by ERCP findings of pancreatic ductal ectasia with alternating areas of stricture
and dilatation. Several pancreatic ductal stones are also noted. With chronic pain as the operative indication, the
most appropriate procedure would be:

a.80% distal pancreatectomy with splenectomy


b.Longitudinal pancreaticojejunostomy
c.Distal pancreatectomy with end pancreaticojejunostomy
d.Total pancreatectomy
Answer: b
When patients with chronic pancreatitis have pancreatic ductal dilatation (greater than 8 mm. ductal decompression using
longitudinal pancreaticojejunostomy may be employed for relief of pain. The finding that pancreatic ductal hypertension
exists in patients with painful chronic pancreatitis and that surgical decompression reduces intrapancreatic pressure to
normal provides the rationale for this operation. The anterior surface of the pancreas is exposed through the lesser sac. The
entire pancreatic duct is opened from the pancreatic tail to a point 1 cm from the duodenum. A side-to-side anastomosis is
then performed between the opened pancreatic duct and a loop of jejunum. Splenectomy is not necessary. In-hospital
mortality rates of less than 5% have been widely reported. Approximately 80% of patients report complete or substantial
improvement of pain following longitudinal pancreaticojejunostomy.

71. For the patient in the preceding question, the most appropriate long-term management is which of the following?

a.Endoscopic stenting of the distal common bile duct


b.Choledochoduodenostomy
c.Pancreaticoduodenectomy (Whipple procedure)
d.Percutaneous transhepatic drainage of the common hepatic duct
Answer: b

Operative management of patients with stricture of the common bile duct associated with chronic pancreatitis is justified to
treat symptoms and to prevent development of biliary cirrhosis. Operative indications include progressive jaundice,
cholangitis, liver biopsy evidence of biliary cirrhosis, persistent elevation of alkaline phosphatase at greater than three times
normal, and progressive stricture demonstrated by radiologically progressive dilatation of extrahepatic and intrahepatic
biliary ducts. Both choledochoduodenostomy and choledochojejunostomy are excellent operative choices.

72. Which of the following is the most common cause of obstructive jaundice in patients with chronic pancreatitis?

a.Adenocarcinoma of the head of the pancreas


b.Choledocholithiasis
c.Fibrotic stricture of the common bile duct
d.Pancreatic pseudocyst formation
Answer: c

Biliary complications involving the common bile duct can occur in chronic pancreatitis because of the intimate association
of that structure with the head of the pancreas. In two-thirds of individuals, the common bile duct traverses the pancreatic
parenchyma and in an additional 25%, the common bile duct lies within a groove along the posterior surface of the
pancreas. Fibrosis associated with chronic pancreatitis can encase and compress the common bile duct. Common bile duct
stenosis is relatively common in chronic pancreatitis, occurring in approximately 10% of cases observed long-term.
Cholangiography typically reveals a long, gradually tapering stricture conforming to the intrapancreatic portion of the
common bile duct. In contrast, malignant strictures usually result in abrupt termination of the biliary duct. The proximal
suprapancreatic portion of the bile duct is variably dilated.

73. Alcohol-induced and hereditary chronic pancreatitis are the two most common etiologies observed in North
American patients. Most of the remaining patients fall into which of the following categories?

a.Chronic pancreatitis secondary to hyperparathyroidism


b.Chronic pancreatitis caused by protein-calorie malnutrition
c.Chronic pancreatitis secondary to congenital pancreatic ductal obstruction
d.Idiopathic chronic pancreatitis
Answer: d

After alcohol-induced and hereditary disease, idiopathic chronic pancreatitis is the most common cause of calcifying
pancreatitis in North American patients. This designation is given to those cases without a recognizable cause. Idiopathic
pancreatitis accounts for about 15% of the cases and has two peaks in incidence, suggesting that differing underlying causes
may exist. The first peak occurs in young adulthood and the second has an occurrence at approximately 60 years of age.
74. Which of the following statements regarding prognosis in chronic pancreatitis is/are correct?

a.Patients with chronic pancreatitis have decreased long-term survival compared with the general population
b.Patients with chronic pancreatitis exhibit no excess mortality relative to the general population
c.Excess mortality in patients with chronic pancreatitis is related to cancers of the aerodigestive system,
complications of diabetes, and complications of cirrhosis
d.Excess mortality in patients with chronic pancreatitis is due to development of adenocarcinoma of the pancreas and
to the complications of recurrent pancreatitis
Answer: a, c

Patients with chronic pancreatitis have decreased long-term survival compared to the general population. An excess of
mortality of 30% over 20 years has been estimated. Less than 20% of deaths are directly attributable to pancreatitis or its
complications. Excess mortality is related to extrapancreatic complications of alcoholism and smoking, including cancers of
the aerodigestive system, complications of diabetes, and complications of cirrhosis.

75. Which of the following is the most common clinical manifestation of chronic pancreatitis?

a.Epigastric pain with radiation to the hypogastrium


b.Diabetes mellitus
c.Steatorrhea
d.Epigastric pain with radiation to the upper lumbar vertebrae
Answer: d

Pain is a predominant symptom complex in most patients with chronic pancreatitis. Chronic pancreatic pain is usually
localized to the epigastrium with radiation to the back in the region of the upper lumbar vertebrae. Discomfort may be
exacerbated by eating and is usually alleviated by abstinence from food and by bending forward. Malabsorption and weight
loss, clinical manifestations of steatorrhea, are only observed when greater than 90% of exocrine tissue has been destroyed.
Clinical signs of malabsorption are a late manifestation of chronic pancreatitis. Although abnormal glucose tests can be
demonstrated in 50% to 70% of patients with chronic Pancreatitis: overt diabetes mellitus is present in only 30% to 40%.
Endocrine deficits are usually progressive. If individual patients are repetitively tested, progressive deterioration is often
observed.

76. For the patient in the preceding question, appropriate management includes which of the following?

a.Distal pancreatectomy
b.Cystjejunostomy
c.Percutaneous drainage
d.Primary radiotherapy and chemotherapy
Answer: a

The proper treatment is surgical removal of the tumor; aggressive pancreatic resection is appropriate. It is crucial to avoid
mistaking a mucinous cystic tumor for a pancreatic pseudocyst. Internal drainage of a malignant mucinous cystic tumor
results in catastrophic tumor dissemination and should never be performed. With appropriate treatment, all patients with
histologically benign tumors should be cured; for tumors demonstrating malignant change, 5-year survival after surgery is
about 60%.

77. A 72-year-old man develops jaundice and is demonstrated to have a 2.5 mass in the pancreatic head by computed
tomography. There are no signs of unresectability on CT examination. Fine needle aspiration cytology is positive
for adenocarcinoma. Which of the following intraoperative findings would indicate unresectability?

a.Fibrotic reaction in the body and tail of the pancreas


b.Microscopic tumor cells in perigastric lymph nodes on frozen section
c.Inability to develop an avascular plane anterior to the superior mesenteric vein
d.Cholelithiasis
Answer: b, c
During performance of pancreaticoduodenectomy, the lesser sac is opened widely through the gastrocolic omentum. This
maneuver allows inspection of the body and tail of the gland to determine the extent of the tumor involvement and allows
examination of lymph nodes along the superior and inferior body of the pancreas and around the celiac axis. Enlarged nodes
in these areas should undergo biopsy and be submitted for frozen-section examination, since tumor in these areas is beyond
the bounds of standard pancreaticoduodenectomy and constitutes a contraindication to resection. If there is no evidence of
lymphadenopathy, a dissection between the anterior surface of the portal vein and the posterior surface of the neck of the
pancreas is performed. Ordinarily, only thin areolar tissue lies between the pancreas and the portal vein, and a
communication behind the neck of the pancreas can be established. If there is hard tissue intervening and such
communication cannot be established, this implies invasion of the anterior surface of the portal vein and signals
unresectability by standard methods.

78. A 67-year-old male presents with complaints of itching, dark urine, and epigastric pain. Physical examination
reveals jaundice. Initial laboratory tests show total bilirubin of 6.5 mg/dL, alkaline phosphatase elevated at 3 the
upper limit of normal, and mild elevations in serum transaminases. Appropriate management includes which
diagnostic test next?

a.Abdominal ultrasonography
b.Computed tomography of the abdomen
c.Magnetic resonance imaging of the abdomen
d.Endoscopic retrograde cholangiography
Answer: a

Standard transcutaneous ultrasonography is the appropriate first test in the evaluation of the patient with jaundice, because
the presence of a dilated common bile duct or intrahepatic bile ducts is essentially diagnostic of extrahepatic biliary
obstruction. This finding directs the physician to a search for the cause of the obstruction. If the bile ducts are not dilated,
mechanical obstruction is unlikely and the diagnostic thrust should move toward hepatocellular disease. Ultrasonography is
also the best test to determine whether gallstones are present; this is extremely important because choledocholithiasis is one
of the conditions most likely to cause jaundice in the elderly population.

79. Which of the following statements regarding ductal adenocarcinoma of the pancreas is/are correct?

a.For ductal adenocarcinoma, 60–70% of tumors arise in the head of the gland, 15% in the body, and 10% in the tail,
the remainder are diffuse
b.Fifty percent of pancreatic adenocarcinomas involve the gland diffusely at the time of diagnosis
c.For ductal adenocarcinomas, tumors of the body and tail are usually larger at the time of diagnosis than those
arising in the head of the gland
d.Pancreatic adenocarcinomas occur with equal frequency within the head, body, and tail of the gland
Answer: a, c

Sixty to 70% of pancreatic ductal adenocarcinomas occur in the head of the gland. About 15% reside in the body of the
gland, another 10% are in the tail, and the remaining 5% to 15% are diffuse. The predilection of pancreatic cancer to
develop in the head of the gland is unexplained, but has the practical consequence that tumors in the head are diagnosed
earlier because they cause obstructive jaundice whereas tumors in the body and tail tend to be more advanced at the time of
symptomatic presentation. Tumors in the body and tail are typically larger at the time of diagnosis (average, 7 to 8 cm) than
in the head (average, 4 to 5 cm).

80. The most common cause of death in the postoperative period following pancreaticoduodenectomy is which of the
following?

a.Myocardial infarction
b.Intraperitoneal hemorrhage
c.Pulmonary embolism
d.Pneumonia
Answer: b
Pancreaticoduodenectomy is a formidable operation, and until recently, average operative mortality was reported to
approximate 20%. In the past few years, several centers have reported large series with operative mortalities lower than 5%.
The most dreaded complication of pancreaticoduodenectomy is disruption of the pancreaticojejunostomy, which occurs in
about 10% of patients. Anastomotic breakdown may lead to the development of an upper abdominal abscess or may present
as a external pancreatic fistula. In its most virulent form, disruption leads to necrotizing retroperitoneal infection which may
erode major arteries and veins of the upper abdomen, including the portal vein or its branches or the stump of the
gastroduodenal artery. Impending catastrophe is often preceded by a small herald bleed from the drain site. Such an event is
an indication to return to the operating room to widely drain the pancreaticojejunostomy and to repair the involved blood
vessel. Open packing of the wound may be necessary in controlling diffuse necrosis and infection. On rare occasions,
completion pancreatectomy is required to control sepsis. Intraperitoneal hemorrhage is the most common cause of death
from pancreaticoduodenectomy.

81. For the above patient, ultrasonography reveals dilated extrahepatic and intrahepatic bile ducts and the absence of
gallstones within the gallbladder. A 2 cm mass within the pancreatic head is visualized. Computed tomography of
the abdomen is performed. Which of following CT findings indicate probable unresectability?

a.Common bile duct diameter of 2 cm


b.Ascites
c.Infiltration of the transverse colonic mesentery
d.Dilatation of the main pancreatic duct to 1.5 cm
Answer: b, c

CT scans provide the best available radiologic information to determine whether or not a pancreatic neoplasm is resectable,
but they cannot be considered absolutely definitive in this regard. Only about half of pancreatic tumors that appear to be
confined to the pancreas on CT scan are found to be resectable in the operating room. CT scanning is more accurate in the
diagnosis of unresectability. CT findings that indicate that the tumor is unlikely to be surgically curable include vascular
invasion, enlarged lymph nodes outside the boundaries of resection, ascites, distant metastases (usually liver), and distant
organ invasion (usually colon). When a CT scan shows distant metastases or extensive local invasion, the positive predictive
value of the technique is high; some 90% of such patients have unresectable disease at laparotomy.

82. Which of the following have been shown to be risk factors for development of adenocarcinoma of the pancreas?

a.Cigarette smoking
b.Coffee drinking
c.Adult-onset diabetes mellitus
d.Chronic coumadin usage
e.Prior gastrectomy
Answer: a, e

Most cases of pancreatic cancer have no obvious predisposing host factors. The most consistently observed risk factor for
pancreatic cancer is cigarette smoking. Most studies estimate that smoking results in a two-to three-fold increase in risk of
developing pancreatic cancer. Alcohol consumption has been implicated in some case-control studies of pancreatic cancer,
but the overall evidence is inconsistent and alcohol is not likely to be a major factor in the development of the disease.
Although considerable public interest was focused in the past few years on coffee consumption as a risk factor for
pancreatic cancer, evidence linking coffee consumption to pancreatic cancer is not compelling.
Abnormal glucose tolerance is present in about 80% of patients with pancreatic cancer, if carefully sought. Although
diabetes and pancreatic cancer occur together far more frequently than would be expected by chance, persons with long-
standing diabetes are not at increased risk of developing pancreatic cancer. Chronic pancreatitis is a significant risk factor
for the development of pancreatic cancer. It appears that all forms of chronic pancreatitis are associated with an increased
risk of pancreatic cancer, suggesting that it is the pancreatitis and not the injuring agent which is responsible for the
augmented cancer risk. Studies indicate that patients who have previously undergone gastric resection may be from three to
seven times more likely to develop pancreatic cancer than a control population.
There have been several reports of familial clustering of pancreatic cancer. Recent epidemiological studies suggest that
about 7% of pancreatic cancer patients have a positive family history of the disease. For most cases, however, no hereditary
basis for pancreatic cancer has been identified.
83. Which of the following surgical procedures has the lowest incidence of recurrent jaundice when used in the context
of unresectable carcinoma of the head of the pancreas?

a.Choledochoduodenostomy
b.Cholecystojejunostomy
c.Cholecystoduodenostomy
d.Choledochojejunostomy
Answer: d

When jaundiced patients undergo exploration in the hopes of resection but unresectable disease is found, biliary bypass
should be performed. The jejunum is typically chosen as a conduit in preference to the duodenum because duodenal
obstruction may occur as the tumor becomes more advanced. There has been much discussion over the use of the bile duct
or the gallbladder for biliary decompression. Operative mortality and mean survival (about 6 months) do not differ between
patients with cholecystojejunostomy and choledochojejunostomy. Recurrent jaundice is more common after
cholecystojejunostomy. Because recurrent jaundice constitutes a failure of palliation, the use of the common duct for biliary
bypass is preferable in most patients. There are circumstances, however, in which it may be more appropriate to use the
gallbladder. Such instances include patients with poor performance status, cases in which the tumor is bulky and invades the
porta hepatis, or when periductal varices have developed as a result of portal vein thrombosis. The suitability of the
gallbladder as a biliary conduit must be proven intraoperatively. If, on aspiration, the gallbladder contains colorless fluid,
the cystic duct may be assumed to be obstructed, and the gallbladder should be removed and not used for bypass. If there is
green bile in the gallbladder, patency of the cystic duct should be proved by cholangiography before a bypass is performed.

84. A 45-year-old woman is evaluated for epigastric and back pain. Physical examination is normal. Computed
tomography of the abdomen reveals a 8 cm cystic lesion in the region of the tail of the pancreas. The cyst
demonstrates internal septations and papillary projections from its walls. Which of the following diagnoses is most
likely in this patient?

a.Pancreatic lymphoma
b.Retroperitoneal liposarcoma
c.Pancreatic pseudocyst
d.Pancreatic mucinous cystadenoma
Answer: d

Mucinous cystic neoplasms account for about 2% of pancreatic exocrine tumors. Most patients with mucinous cystic tumors
present with abdominal pain or an abdominal mass. There may be associated weight loss, steatorrhea, or diabetes. The
diagnosis is best made by CT scanning and ultrasonography, which demonstrate a mass containing fluid-filled structures and
internal septations. Occasionally, it is possible to see the papillary tumor excrescences on the cyst walls.
The tumor occurs six times as often in females as in males. About 80% of the tumors are located in the body and tail of the
pancreas. They present as large (average, 10 cm), soft, and somewhat irregular tumors. Microscopically, the cysts are lined
by columnar epithelium which contains mucin. Although most of the cells may appear benign histologically, most tumors
larger than 3 cm contain areas of premalignant or malignant change and all mucinous cystic tumors should be considered to
have malignant potential.

85. A 45-year-old woman develops upper gastrointestinal hemorrhage. Evaluation by upper endoscopy reveals three
ulcers in the second portion of the duodenum. Bleeding is controlled using an endoscopic heat probe. Further
investigation reveals a serum gastrin value of 240 pg/mL. Which of the following would support the presumptive
diagnosis of gastrinoma?

a.An increase of 320 pg/mL in serum gastrin upon intravenous infusion of secretin
b.Gastric acid analysis demonstrating fasting acid secretion of 3 mEq/h
c.Enlarged gastric rugae on upper gastrointestinal contrast study
d.An increase of 150 pg/mL in serum gastrin upon intravenous infusion of cholecystokinin
Answer: a, c

The indications for the measurement of gastrin include the presence of peptic ulcer disease, patients with prolonged
undiagnosed diarrhea, patients within MEN-1 families and patients with prominent gastric rugal folds on upper GI series. In
most patients with gastrinoma, the fasting serum gastrin level is elevated above 200 pg/ml. Gastrin values over 1000 pg/ml
are virtually diagnostic of gastrinoma. However, fasting hypergastrinemia alone is not sufficient for the diagnosis of
gastrinoma. Gastric acid analysis is an important test in the evaluation of patients with suspected gastrinoma, as it can
differentiate between ulcerogenic causes of hypergastrinemia and nonulcerogenic causes of hypergastrinemia. The diagnosis
of gastrinoma is supported by a basal acid output above 15 mEq/hour in nonoperated patients.
Following documentation that hypergastrinemia is associated with excessive acid secretion, provocative testing using
secretin should be performed to differentiate between gastrinoma, antral G cell hyperplasia/hyperfunction, and the other
causes of ulcerogenic hypergastrinemia. The secretin stimulation test is carried out in the fasting state by obtaining
peripheral serum samples for gastrin in the basal period, administering secretin (2 units/kg body weight) as an intravenous
bolus, and obtaining serum samples for gastrin at five minute intervals for 30 minutes. An increase in the gastrin level of
more than 200 pg/ml above the basal level is supportive of the diagnosis of gastrinoma.

86. For the patient in the preceding question, an insulin/glucose ratio of 0.5 was documented at 28 hours of fasting.
Symptoms of mental obtundation developed concurrently and were reversed by oral glucose administration.
Endoscopic ultrasonography demonstrated a 1.2 cm mass in the head of the pancreas. Appropriate management
consists of which of the following?

a.Surgical enucleation of the tumor


b.Total pancreatectomy
c.Long-term octreotide administration
d.Primary radiotherapy
Answer: a

The treatment of insulinoma is surgical in nearly all cases. Insulinomas are found evenly distributed within the pancreas,
with approximately one-third being located in the head and uncinate process, one-third in the body of the gland, and one-
third in the tail of the gland. Ninety percent of patients will be found to have benign solitary adenomas amenable to surgical
cure. Small benign insulinomas not in close proximity to the main pancreatic duct may be removed by enucleation,
independent of their location within the gland. In the body and tail of the Pancreas: insulinomas greater than 2 cm in
diameter, and those in close proximity to the pancreatic duct are most commonly excised by distal pancreatectomy. Large
insulinomas deep in the head or uncinate process of the pancreas may not be amenable to local excision, and may require
pancreaticoduodenectomy.

87. A 35-year-old woman is evaluated for seizure disorder, mental obtundation, and personality change. Physical
examination is normal. Fasting serum glucose is 44 mg/dL. Other serum values are normal. Subsequent
investigations should include which of the following?

a.Oral glucose tolerance test


b.Determination of fasting insulin/glucose ratios
c.Assay of serum C-peptide levels
d.Determination of serum prolactin levels
Answer: b, c

A common mistake made in the evaluation of a patient with suspected insulinoma is to commence the evaluation with an
oral glucose tolerance test. Instead, insulinoma is most reliably diagnosed using the technique of a monitored fast. During a
monitored fast, blood for glucose and insulin determinations is sampled every four to six hours, and at the time of symptom
occurrence. Hypoglycemic symptoms typically occur when glucose levels are less than 50 mg/dl, with concurrent serum
insulin levels often being greater than 25 µU/ml. Additional support for the diagnosis of insulinoma comes from the
calculation of the insulin to glucose ratio (I:G ratio) at different time points during the monitored fast. Normal individuals
will have I:G ratios less than 0.3, while patients with insulinoma typically demonstrate I:G ratios greater than 0.4 after a
prolonged fast. Other measurable beta cell products synthesized in excess in patients with insulinoma include C peptide and
proinsulin. Elevated levels of C peptide and proinsulin are typically found in the peripheral blood in patients with
insulinoma. The possibility of surreptitious insulin or oral hypoglycemic agent administration should be considered in all
patients with suspected insulinoma. C peptide and proinsulin levels will not be elevated in patients self-administering
insulin. Additionally, patients self-administering either bovine or porcine insulin may demonstrate anti-insulin antibodies in
circulating blood.

88. The most common location(s) for development of gastrinoma is/are which of the following?
a.Pancreas to the right of the superior mesenteric vein
b.Pancreatic body and tail
c.Gastric antrum
d.Duodenum
Answer: a, d

The majority of gastrinomas have been identified to the right of the superior mesenteric vessels within the head of the
pancreas or the duodenum. Intraoperative ultrasonography should be available to assist in tumor localization. In addition,
intraoperative upper endoscopy may be helpful by allowing transillumination of the duodenal wall and identification of
small duodenal gastrinomas. At exploration, any suspicious peripancreatic lymph nodes are excised and submitted for
frozen section. Primary tumors located within the substance of the pancreas that are small (< 2 cm) and well encapsulated
may be carefully enucleated. Pancreatic tumors without defined capsules or situated deep in the pancreatic parenchyma may
require partial pancreatic resection. In the absence of an identifiable pancreatic or duodenal tumor, a longitudinal
duodenotomy may be performed at the level of the second portion of the duodenum to allow for eversion of the duodenum
in a search for duodenal microgastrinomas. Primary gastrinomas identified within the duodenal wall are resected locally,
with primary closure of the duodenal defect.

89. Neoplastic hypersecretion of the hormone vasoactive intestinal peptide is associated with which of the following
features?

a.Hypokalemia, hypochlorhydria, diarrhea


b.Hyperglycemia, necrolytic rash, hypoaminoacidemia
c.Constipation, gallstones, hyperglycemia
d.Hyperkalemia, necrolytic rash, diarrhea
Answer: a

Patients characteristically present with intermittent severe diarrhea, typically of a watery nature, averaging 5 liters/day.
Malabsorption and steatorrhea are not common. Hypokalemia results from the fecal loss of large amounts of potassium (up
to 400 meq/day), and low serum potassium levels may be associated with muscular weakness, lethargy, and nausea. Most
patients are hypochlorhydric or achlorhydric. Half of the patients have some degree of hyperglycemia and hypercalcemia,
while cutaneous flushing can be observed in a minority of patients. The diagnosis of VIPoma is typically made after
excluding other more common causes of diarrhea. The active agent in the VIPoma syndrome is usually vasoactive intestinal
polypeptide (VIP), with a minority of patients having elevations of other candidate mediators such as peptide histidine-
isoleucine (PHI) or prostaglandins.

90. A patient with biochemically confirmed gastrinoma undergoes computed tomography for tumor localization. CT
reveals a 2 cm mass in the head of the pancreas and multiple nodules within right and left lobes of the liver.
Appropriate management includes which of the following?

a.Omeprazole administration
b.Radiotherapy
c.Pancreaticoduodenectomy
d.Proximal gastric vagotomy
Answer: a

Gastrinoma patients whose localization and staging studies are indicative of unresectable hepatic metastases should undergo
percutaneous or laparoscopically-directed liver biopsy for histologic verification. If unresectable gastrinoma is confirmed,
then open surgical exploration is not performed and the patient is maintained on long-term omeprazole therapy. Virtually all
patients can be rendered achlorhydric with appropriate dose adjustment of omeprazole. Noncompliant patients who refuse to
take appropriate doses of omeprazole and who develop complications related to their ulcer diathesis may require total
gastrectomy for management. Total gastrectomy removes the end organ (parietal cell mass) and was once the procedure of
choice for gastrinoma. Today its use in gastrinoma patients has markedly declined.

91. The following statement(s) is/are true concerning the widely accepted French or Couinaud’s nomenclature for liver
anatomy.
a.The liver is divided into eight discrete segments based on portal pedicle branches and hepatic venous drainage
b.This anatomy is particularly useful in allowing less than lobar segmental anatomical resections that minimize blood
loss and loss of hepatic reserve
c.Enumeration of the system begins from right to left
d.Segments II and III are synonymous with the left lateral segment based on English nomenclature
Answer: a, b, d

In the now widely accepted French (Couinaud’s) nomenclature, the liver can be divided into eight discrete segments based
on portal pedicle branches and hepatic venous drainage. Enumeration of the segments begins left to right, beginning with
segment I, the caudate lobe. The left lateral sector consists of a superior segment II and an inferior segment III and is
synonymous with the left lateral segment in older terminology. The major advantage to this detailed segmental anatomy,
which is based on discrete portal pedicle branches, is to accurately locate individual lesions in the hepatic substance by
preoperative imaging and intraoperative ultrasound and to allow the possibility of less than lobar segmental anatomical
resections that minimize blood loss and functional loss of hepatic reserve.

92. In the patient described above, which of the following are important operative steps in the performance of a right
hepatic lobectomy?

a.The use of an ultrasonic dissector is essential for division of the hepatic parenchyma
b.If temporary portal inflow occlusion is used (Pringle maneuver), it is not necessary to reestablish blood flow during
the course of the parenchymal division
c.The greater omentum may be used to buttress the transected liver edge
d.Control of the main right hepatic vein should eliminate all forms of venous drainage
Answer: c

The steps involved in a right hepatic lobectomy involve adherence to the tenet of optimal operative exposure and control of
vascular inflow and outflow. In select circumstances, control of the vena cava may be desired. Either the individual portal
structures can be identified and ligated early in the course of the procedure, or simply the entire portal triad can be circled
with an umbilical tape tourniquet in preparation for the Pringle maneuver. If temporary portal inflow occlusion is used,
intermittent 10 to 20 minute intervals of clamping with 3 to 5 minutes to reestablish blood flow is recommended. The
division of the hepatic parenchyma begins with scoring of Glisson’s capsule with cautery or knife and proceeds with
division of the hepatic surface using either blunt dissection by finger fracture, the blunt edge of an instrument or suction tip,
or using an ultrasonic dissector. Individual vessels and bile ducts are cauterized, sutured, or clipped in rapid succession from
anterior to posterior. The hepatic veins are encountered in the hepatic substance near the vena cava and are carefully
clamped and suture ligated to complete the resection. In addition, there are also several posterior accessory veins (up to 10
in number) which drain the medial aspect of the right lobe and empty directly into the right anterior surface of the IVC.

93. Intraoperative ultrasound is now commonly used by the hepatic surgeon. Which of the following statement(s) is/are
true concerning intraoperative ultrasound and hepatic surgery?

a.An intraoperative ultrasound offers no advantage to conventional transcorporial ultrasound in detection of hepatic
lesions
b.Portal structures can be differentiated from hepatic veins by the extension of Glisson’s capsule surrounding these
structures
c.It is difficult on ultrasound to differentiate a vascular structure from a mass
d.The short hepatic veins are difficult to detect with intraoperative ultrasound
Answer: b

Over the past 10 years, detailed anatomic description of the hepatic veins, portal pedicles, and the inferior vena cava have
been possible through the use of intraoperative ultrasound. Cooperation between radiologists and hepatic surgeon with the
use of intraoperative ultrasound has allowed the identification of lesions during surgery that were not visible by
conventional transcorporial ultrasound or CT scanning. Beginning superiorly at the inferior vena cava, the confluence and
course of each of the hepatic veins can easily be determined. More inferiorly, the main right and left portal pedicles can be
seen coursing transversely in the transverse scissura. Portal structures can easily be differentiated from hepatic veins by the
hyperechoic extensions of Glisson’s capsule which surround these structures. When a circular structure is encountered, a
mass or metastasis may be suspected. Scanning away from the mass may reveal a tubulovascular shape which has been
imaged and cross sectioned. Flattening of the circular mass by external compression with the ultrasound probe will also
differentiate a vascular structure from a solid mass.

94. The following statement(s) is/are true concerning hepatic anatomical nomenclature.

a.In the traditional English system, the right lobe is divided into anterior and posterior segments by an intersegmental
line with no topographic landmarks or interparenchymal septi
b.The caudate lobe in the French or Couinaud’s nomenclature is referred to as segment I
c.The right lobe of the liver by English nomenclature is subdivided in the French system into segments V–VIII
d.In the English system, the left lobe of the liver is divided into the medial segment and lateral segment by the
falciform ligament
Answer: a, b, c, d

Until recently, anatomic descriptions in the English literature began with the major divisions of liver into a right and left
lobe separated by a vertical line drawn from the gallbladder fossa to the inferior vena cava. The left lobe is further divided
by the falciform ligament into a medial segment and lateral segment. The right lobe is further divided into an anterior and
posterior segment by an intersegmental line which has no reliable topographical landmarks and no interparenchymal septi to
allow easy identification. The French nomenclature also known as Couinaud’s nomenclature enumerates the segments of the
liver beginning with segment I or the caudate lobe. Segments II, III, and IV make up most of the “English” left lobe, while
segments V through VIII represent the English nomenclature right lobe.

95. A 57-year-old man with a history of Duke’s C colon cancer is being evaluated for a rising CEA. Which of the
following statement(s) is/are correct concerning the use of CT scanning for this indication?

a.Conventional CT scanning will detect lesions well below 1 cm in size


b.CT arterio-portography involves immediate CT scanning after direct injection into both the common hepatic artery
and superior mesenteric artery
c.A double helical (spiral) CT scan may eliminate the need for invasive angiography
d.Magnetic resonance imaging of the liver will add little to the workup of this patient
Answer: b, c, d

CT scanning has been used increasingly to screen for hepatic and other intra-abdominal or retroperitoneal lesions.
Conventional CT scanning includes 0.5–1 cm axial images of the liver after oral administration of barium and bolus
injection of intravenous contrast. Although resolution has improved, hepatic lesions below 1 cm in size or lesions that are
isodense with hepatic parenchyma may be missed. Resolution of hepatic lesions has been greatly enhanced by the
combination of visceral angiography and CT scanning, known as CT arterio-portography (CTAP). Immediate CT scanning
after injection of contrast directly into the common hepatic artery may identify small hepatic lesions which usually show
increased density relative to the surrounding hepatic parenchyma. CT arterio-portography also includes direct injection of
contrast into the splenic or superior mesenteric arteries, with CT imaging during the portal venous phase of this injection.
Hepatic lesions supplied by the hepatic artery thus appear as discrete hypodense lesions surrounded by normal hepatic
parenchyma enhanced by portal venous contrast. Recently, double helical (spiral) CT scanning has become available and
shows considerable promise to complement or replace CTAP for preoperative imaging. This technique allows total hepatic
imaging in both the arterial and arterial/venous phases after a single rapid bolus injection of intravenous contrast during a
single breath hold by the patient. It is possible to visualize the portal structures and hepatic veins on a single scan and give a
high resolution of small hepatic lesions. In addition, three-dimensional reconstructions can be created to further delineate
hepatic parenchyma and demonstrate a CT constructed hepatic arteriogram. This technique may completely replace the need
for invasive arteriography to characterize the blood supply to the liver prior to hepatic resection or after hepatic
transplantation. Magnetic resonance imaging of the liver has results similar to CT scanning, but to date has not
demonstrated improvements sufficient to justify the increased cost associated with the technique.

96. A solitary 6 cm lesion is identified in the right hepatic lobe in the patient described above. Which of the following
statement(s) is/are true concerning the initial operative management?

a.To facilitate mobilization and assessment with intraoperative ultrasound, complete mobilization including dividing
the left and right triangular ligaments would be necessary
b.In dividing the right triangular ligament, care must be taken to avoid injury to accessory right hepatic veins
draining directly into the vena cava
c.Unless a considerable length of hepatic vein is found outside the hepatic parenchyma, early hepatic vein ligation
should be avoided
d.Ligation of the portal arterial structures is always necessary before proceeding with hepatic lobectomy
Answer: a, b, c

For major hepatic resections and for complete intraoperative ultrasound, complete mobilization of the liver will be required.
After detachment of the hepatic flexure of the colon and division of the falciform ligament, both the left and right triangular
ligaments must be sharply taken down to fully mobilize the liver. During division of the right triangular ligament, care must
be taken to avoid injury to the right diaphragm, the right adrenal gland and adrenal vein, the right phrenic vein, and several
moderate-size accessory right hepatic veins draining directly into the vena cava. After mobilization, digital and bimanual
palpation is performed and intraoperative ultrasound may be performed. Dissection of the porta hepatis is performed by
many hepatic surgeons to identify the main bifurcations of the hepatic artery, bile duct, and portal vein. This allows
individual ligation of unilateral branches of each of these structures during hepatic lobectomy but prior to parenchymal
dissection. An alternative approach has been recently described where the main portal structures are left undisturbed and
branches to a given lobe are ligated during parenchymal transection. Hemorrhage can be minimized by intermittent portal
inflow occlusion by clamping or compression of the portal triad (Pringle maneuver). There has been considerable debate
over early versus late isolation and ligation of a given hepatic vein during lobectomy since the extraparenchymal component
of the hepatic vein may be quite short or absent. Since hemorrhage in this location may be difficult to control, a safe strategy
is to always avoid early isolation of a given hepatic vein or to attempt isolation only when a considerable length of vein is
found on mobilization of the respective triangular ligament.

97. Which of the following statement(s) is/are true concerning the arterial venous anatomy of the liver?

a.Most commonly, the right, left, and middle hepatic veins join the inferior vena cava as a separate trunk
b.Most frequently, the entire length of each hepatic vein is within the parenchyma of the liver
c.A replaced right hepatic artery may be placed in jeopardy during performance of a pancreaticoduodenectomy
d.There is little collateral arterial circulation between the right and left hepatic lobes
Answer: b, c

There are three major hepatic veins which carry blood from the central veins of the hepatic substance to the inferior vena
cava (IVC). In two-thirds of patients, there is a single large right hepatic vein which joins the right anterior wall of the IVC
and a middle and a left hepatic vein which converge one-to-two cm from the IVC and enter the left anterior wall of the IVC
as a single vessel. In one-third of patients, each major hepatic vein joins at the same horizontal level of the IVC as a separate
trunk. In some patients, there is a short but definable extraparenchymal segment of one or more of the hepatic veins at the
confluence with the IVC. More frequently, the entire length of the hepatic veins is intraparenchymal, which may preclude
early, safe hepatic venous isolation during hepatic resection.
There is considerable variability in the origin and course of the right and left hepatic arteries. The most common finding
(55% of patients) is a transverse common hepatic artery from the celiac trunk which gives off the gastroduodenal, right
gastric, and supraduodenal arteries and courses obliquely in the left anterior aspect of the hepatoduodenal ligament as a
proper hepatic artery. After giving off the cystic artery to the gallbladder, there is then a fairly low trifurcation into a single
right, middle, and left hepatic arteries. Knowledge of the most common variations is extremely importance since inadvertent
division may occur during gastric, pancreatic, and hepatobiliary procedures. There may be a replaced or accessory left
hepatic artery which arises from the left gastric artery and courses transversely in the lesser omentum. With nearly equal
frequency, there is a replaced or accessory right hepatic artery from the superior mesenteric artery near its origin which
courses posterior or through the head of the pancreas obliquely along the right posterior border of the hepatoduodenal
ligament. Although original anatomic descriptions deny the existence of collateral vessels to the opposite hepatic lobe,
image perfusion studies after ligation of main or replaced hepatic arteries have clearly demonstrated the presence of
collateral flow to the deprived lobe.

98. The liver plays a vital role in carbohydrate metabolism and regulation of blood glucose. The following statement(s)
is/are true concerning carbohydrate metabolism by the liver.

a.Glycogen, a complex polymer of glucose, is synthesized by the hepatocyte in a remarkably energy efficient process
b.Glucagon stimulates glycogenesis
c.Glycolysis, the process by which glucose is converted to two molecules of pyruvate, occurs in the liver
mitochondria
d.If glycogen stores become depleted, the liver is capable of synthesizing new glucose by the process of
gluconeogenesis, which is stimulated by insulin
Answer: a

Serum glucose is tightly regulated by the liver despite wide fluctuations in dietary ingestion. The liver can take up as much
as 100 g/day of glucose and convert it to glycogen by the process of glycogenesis. The liver can also release glucose into the
blood by glycogenolysis, the breakdown of glycogen, or by gluconeogenesis, the formation of new glucose from substrates
such as alanine, lactate, glycerol or dietary amino acids. Hormones play a key role in hepatic regulation of glucose
metabolism. Insulin, for example, stimulates glycogenesis, and glucagon stimulates glycogenolysis and gluconeogenesis.
Gluconeogenesis is also enhanced by fasting, critical illness and periods of anaerobic metabolism.
Glycogen is a complex polymer of glucose. Liver cells can store up to 8% of their weight as glycogen. The first step in
glycogen storage is the transport of glucose through the hepatocyte plasma membrane. About 90% of portal venous glucose
is removed from the blood by liver cells through carrier-facilitated diffusion. The rate of glucose transport is enhanced by
insulin. Once in the hepatocyte, glucose and ATP are converted by the enzyme glucokinase to glucose-6-phosphate (G6P),
the first intermediate in the synthesis of glycogen. Because complete oxidation of one molecule of G6P generates 37
molecules of ATP, and storage only uses one molecule of ATP, the overall efficiency of glucose storage in glycogen is a
remarkable 97%. Glycolysis is the pathway by which glucose is converted to two molecules of pyruvate and occurs in the
cytoplasm in contrast to the citric acid cycle which occurs in the mitochondria.

99. Transport of substances from the blood into the hepatocyte occurs through the sinusoidal membrane. The following
statement(s) is/are true concerning this plasma membrane.

a.The high lipid content of this phospholipid bilayer allows lipid-soluble molecules to enter the cell by simple
diffusion
b.Carrier proteins within the phospholipid bilayer bind to a solute in blood and by conformational change allow it to
be transported into the cell
c.Large glycoprotein molecules of the sinusoidal membrane known as receptors always transport the binding ligand
into the cell
d.The transmission of a signal to the interior of the cell by receptor-ligand binding which generates intracellular
second messengers is known as signal transduction
Answer: a, b, d

The hepatocyte plasma membrane consists of a phospholipid bilayer in which hydrophobic fatty acid tails are oriented to the
interior membrane and hydrophilic phospholipid head groups are oriented to the exterior (sinusoidal or cytoplasmic)
membrane. Within this phospholipid bilayer are proteins which serve either structural functions or metabolic functions. The
hepatocyte sinusoidal plasma membrane is heavily studded with microvilli to increase the absorptive area in contact with
sinusoidal blood. The cell membrane, by virtue of its high lipid content, allows lipid-soluble molecules to enter the cell by
simple diffusion. Polar molecules must enter cells via membrane transport proteins. Channel proteins allow molecules to
diffuse simply into cells without binding, whereas carrier proteins first bind the solute and, by conformational change, allow
it to be transported into the cell. The glucose carrier in hepatocytes is an example of carrier-facilitated diffusion. The
sinusoidal membrane is studded with receptors, which are large glycoprotein molecules that span the plasma membrane
lipid bilayer. A ligand-binding site of this receptor molecule projects into the space of Disse. When appropriate ligand-
receptor binding occurs, the entire ligand may be internalized for intracellular degradation or biliary transport, or the ligand
may transmit a signal to the interior of the hepatocyte by a number of intracellular second messenger systems, a process
known as signal transduction. Such second messengers include cAMP, inositol triphosphate, and diacylglycerol. Each of
these structurally simple chemicals can amplify cell membrane events and bring about major changes in cellular physiology.

100.The liver is an important site of protein metabolism. Which of the following statement(s) is/are true concerning
protein metabolism by the liver?

a.Amino acids are taken up by hepatocytes by active transport mechanisms and are generally stored long-term for
later synthetic activity
b.Under certain conditions the amine group is removed from the amino acids in the liver and the carbon chain used
for carbohydrate, lipid, or nonessential amino acid synthesis
c.The most important route of detoxification of ammonia formed as the result of deamination of amino acids is via
excretion of ammonia into the urine
d.Proteins synthesized by the liver include albumin, transferrin, fibrinogen, and apolipoproteins
e.Albumin is a sensitive indicator of hepatic synthetic function
Answer: b, d

Essentially all of the end products of dietary protein digestion are amino acids, which are absorbed by the enterocytes into
the portal circulation in ionized states. Amino acids are taken up by hepatocytes by one of several active transport
mechanisms. Amino acids are not stored in the liver but are rapidly used in the production of plasma proteins, purines, heme
proteins, and hormones. Under certain conditions, the amine group is removed from the amino acids, and the carbon chain is
used for carbohydrate, lipid, or nonessential amino acid synthesis. The ammonia formed as the result of deamination of
amino acids is detoxified by one of two routes. The most important pathway involves conversion of ammonia to urea by
enzymes of the Krebs-Henseleit cycle, found only in the liver. A second route of ammonia metabolism involves deamination
of L-glutamine by the kidney, with excretion of ammonia into the urine.
Essentially all albumin, fibrinogen, and apolipoproteins are derived from the liver which can add up to 50 grams of protein
to the plasma per day. Of total hepatic protein synthesis, 75% is destined for export in plasma. Albumin, an important
plasma protein synthesized in the liver, has a long half-life in plasma of about 19 days. This long half-life makes albumin an
insensitive indicator of hepatic synthetic function.

101.The following statement(s) is/are true concerning hepatic blood flow.

a.Although constituting only 2.5% of total body weight, the liver receives 25% of the cardiac output
b.Hepatic blood flow is equally derived from the portal vein and hepatic artery
c.The liver serves as a physiologic blood reservoir either releasing blood back into the systemic circulation at times
of acute blood loss or in situations of volume overload serving as a site of extra blood storage
d.An important function of the liver is to filter particulate debris which is performed by phagocytic Kupffer cells
which line the hepatic sinusoidal endothelium
Answer: a, c, d

The liver constitutes about 2.5% of the total body weight but receives 25% of the cardiac output. Total hepatic blood flow is
100 to 130 ml/min/kg. About two-thirds of total hepatic blood flow is derived from the portal vein and one-third from the
hepatic artery. The liver also serves as a physiologic blood reservoir. About 25% to 30% of the liver volume is accounted for
by blood, and in cases of acute blood loss up to 30%, or as much as 300 ml of the hepatic blood volume can be released into
the systemic circulation without adverse effects on liver function. Conversely, in the case of right heart failure or other
causes of systemic volume overload, as much as one liter of extra blood can be stored in the liver before passive congestion
and liver injury occur. The hepatic sinusoids are lined by an endothelium punctuated with pores that allow proteins and
other particles to diffuse out of the vascular tree and into proximity with hepatocytes. This extreme permeability of the liver
allows rapid exchange of a diverse number of nutrients, hormones and environmental agents between the blood and the
hepatocyte. The liver also acts as a filter for particulate debris, which enters the portal circulation through intestinal
capillaries. Particles such as bacteria are ingested by Kupffer cells by the process of phagocytosis. Kupffer cells line the
hepatic sinusoidal endothelium where formed blood elements and matter may be in direct contact with these phagocytic
cells.

102.The liver synthesizes key metabolites essential for the production of fuel substrates for other organs. These key
metabolites include:

a.Glucose-6-phosphate (G6P)
b.Acetyl CoA
c.Pyruvate
d.Oxaloacetate
Answer: a, b, c

Hepatic processes in the liver are essential for the production of fuel substrates for other organs. The liver, by virtue of its
terminal position in the portal system, is the organ that must regulate intestinally absorbed nutrients for tissue consumption
or storage. The liver accomplishes its task by synthesizing three key metabolites: -glucose-6-phosphate, pyruvate and acetyl
CoA. G6P can be stored as glycogen or converted into glucose, pyruvate, or ribose-5-phosphate (a nucleotide precursor).
Pyruvate can be converted into lactate, alanine (and other amino acids), and acetyl CoA, or it can enter the tricarboxylic acid
cycle. Acetyl CoA is converted to HMG-CoA (a cholesterol and ketone body precursor) or citrate (for fatty acid and
triglyceride synthesis), or it is degraded to carbon dioxide and water for energy.

103.The following statement(s) concerning hepatic bile formation/secretion is/are true.

a.The adult human liver secretes less than 1000 cc of bile daily
b.Most bile is secreted by hepatocytes (canalicular bile)
c.Primary bile acids include cholic acid, chenodeoxycholic acid, and deoxycholic acid
d.The enterohepatic circulation is tremendously efficient in reabsorption of intestinal bile acids
e.Bile acids are the primary determinant of bile flow
Answer: b, d, e

The adult human liver secretes about 1.5 liters of bile daily. Eighty percent of this volume is secreted by the hepatocytes
(canalicular bile) and 20% is secreted by the bile duct epithelial cells (ductular bile). Solutes constitute about 3% of bile.
The major solutes are conjugated bile acids, phosphatidyl choline, cholesterol, protein and bilirubin. Bile acids are the main
determinant of bile production, and canalicular bile flow is traditionally divided into bile acid-dependent and bile acid-
independent components. Primary bile acids are synthesized from cholesterol in the liver and in humans consist of cholic
acid and chenodeoxycholic acid. Secondary bile acids are formed in the intestinal lumen by bacterial dehydroxylation and
consist of deoxycholic acid and lithocholic acid derived from cholic acid and chenodeoxycholic acid, respectively.
Essentially all primary and secondary bile acids are conjugated with the amino acids glycine or taurine. The human liver
synthesizes 300 to 400 mg per day of bile acids from cholesterol, or about 10% of the total bile salt pool. Normally
intestinal bile acids are efficiently (about 95%) taken up by the enterohepatic circulation. Luminal bile acids are transported
by carrier proteins in the distal ileum and appear in the portal venous effluent. The hepatocyte extracts more than 95% of
portal venous bile acids for resecretion into the bile.

104.The following statement(s) is/are true containing lipid metabolism in the liver.

a.Hepatic mitochondrial hydrolysis of fatty acids is a tremendous source of ATP


b.Significant hepatic storage of triglyceride or fatty infiltration can cause hepatic fibrosis or necrosis
c.Approximately 90% of cholesterol synthesis occurs in the liver
d.Most cells in the body are capable of phospholipid synthesis, therefore the liver plays a minimal role in this process
Answer: a, c

The liver has a number of important functions in the metabolism of lipids: 1) the synthesis of apolipoproteins, 2) the
degradation of fatty acids into energy substrates, 3) the synthesis of triglycerides from carbohydrates and proteins, and 4)
the synthesis of cholesterol and phospholipids from fatty acids. The mitochondrial hydrolysis of fatty acids is a source of
large quantities of ATP. The conversion of stearic acid to CO2 and H2O, for instance, generates 136 ATP molecules and
demonstrates the highly efficient storage of energy in fat. In times of unrestrained lipolysis, such as starvation, uncontrolled
diabetes or other conditions of triglyceride mobilization from adipose tissue, the ability of the liver to perform beta-
oxidation may be inadequate. Under these circumstances, significant hepatic storage of triglycerides or fatty infiltration of
the liver may occur. Triglyceride storage by itself does not appear to be a cause of hepatic fibrosis or necrosis, but fatty
infiltration may be a marker for derangement of normal processes by alcohol or drug toxicity, diabetes, chronic parenteral
nutrition, or morbid obesity.
Cholesterol is an important regulator of membrane fluidity and is a substrate for bile acid and steroid hormone synthesis.
Cholesterol may be available by dietary intake or by de novo synthesis. In mammals, about 90% of new cholesterol is
synthesized by the liver from its precursor, acetyl CoA. Dietary cholesterol intake suppresses endogenous synthesis by
inhibiting the rate-limiting enzyme in cholesterol by a synthetic pathway, HMG-CoA reductase. There are three major
classes of phospholipids synthesized by the liver: the lecithins, the cephalins, and the sphingomyelins. Although most cells
in the body are capable of some phospholipid synthesis, the liver produces 90%.

105.Hepatic biotransformation is defined as the intracellular metabolism of endogenous and exogenous organic
compounds. Which of the following is/are enzyme families responsible for hepatic bile transformation?

a.Cytochromes P-450
b.UDB-glucuronyl transferases
c.Glutathione-transferases
d.Sulfotransferases
Answer: a, b, c, d

The liver contains enzyme systems that can expose functional groups such as hydroxyl ions and alter the size and solubility
of a wide variety of organic and inorganic compounds by conjugation with small polar molecules. The general strategy of
the liver is to convert hydrophobic, potentially toxic compounds into hydrophilic conjugates that can then be excreted into
bile or urine. There are four general enzyme families responsible for hepatic bile transformation. The cytochromes P-450
catalyze reactions such as oxidation, hydroxylation, sulfoxide formation, oxidative deamination, dealcoholization and
dehalogination. Such reactions allow further phase II conjugation with polar groups such as glucuronate, glutathione and
sulfate. Glucuronidation is the conjugation of UDB-glucuronic acid to a wide variety of xenobiotics by either esther or ether
linkages. The glutathione transferases and sulfotransferases play a role in conjugation of P-450 derivatives. However, the
glucuronyl transferase system is the predominant mechanism.

106.The following statement(s) is/are true concerning the differential diagnosis between an amoebic and a pyogenic
liver abscess.

a.The clinical presentation is often clearly distinguishable


b.A history of travel or origin from a high risk area might suggest an amebic liver abscess
c.Routine liver chemistries frequently can distinguish pyogenic from amoebic liver abscess
d.Serologic testing for the presence of antibody to entamoeba histolyctica is the only specific and sensitive way to
confirm the diagnosis of amoebic liver abscess
e.Distinguishing pyogenic from hepatic abscesses preoperatively is not important since surgical drainage is
imperative for both
Answer: b, d

Distinguishing amoebic from pyogenic liver abscess can be a diagnostic challenge. It is of major importance, however,
because effective medical therapy with metronidazole can obviate the need for either percutaneous or surgical drainage in
most cases of amoebic abscess. The clinical presentation for both conditions with acute onset of fever, abdominal pain, and
altered liver function tests are almost identical. Important features such as travel to or origin from a high risk area is
particularly important for amebic liver abscess. Routine liver chemistries and radiographic studies can rarely distinguish
between amoebic and pyogenic liver abscesses. Specific serologic tests for the presence of antibody to E. histolytica are
specific and sensitive for amoebic hepatic abscess being positive in 95% of the cases, and therefore, are key in
distinguishing the two infections.

107.A patient is found to develop evidence of hepatitis approximately eight weeks after receiving blood transfusions
during a surgical procedure. Which of the following statement(s) is/are true?

a.The virus responsible is most likely hepatitis C


b.A chronic carrier state will ultimately develop in most patients
c.There is no role for interferon in the treatment of chronic hepatitis C viral infection
d.Chronic infection with hepatitis C is not associated with an increased risk of developing hepatocellular carcinoma
Answer: a, b

Hepatitis C virus is a virus that is responsible for more than 90% of post-transfusion hepatitis and most sporadic non-A,
non-B hepatitis throughout the world. The most common identifiable sources of acquisition of hepatitis C virus are prior
transfusion of blood or blood-derived products or a history of intravenous illicit drug use. The usual incubation period of
post-transfusion hepatitis C viral infection is 5 to 10 weeks. An initial elevation of liver enzymes may be associated with
little or no clinical disturbance. In some patients, acute hepatitis C viral infection does not progress to chronic infection,
however, chronic hepatitis C viral infection develops in up to 70% of patients with post-transfusion hepatitis C infection
with many progressing to cirrhosis. Hepatitis C does not appear to alter life expectancy at least in the first 15 years of
infection. However, once cirrhosis and end stage liver disease develop, the clinical syndrome is indistinguishable from other
forms of chronic liver disease with a predisposition to the development of hepatoma. Interferon alpha is the only FDA
approved therapy for chronic hepatitis C viral infection. There is some evidence that early administration of interferon in
acute hepatitis C viral infection may reduce the risk of progression to the chronic state. As yet, there is no evidence that
interferon alters the natural history of chronic hepatitis C viral infection or changes the incidence.
108.A surgeon is suspected of having contacted hepatitis B virus via needle stick. Which of the following statement(s)
is/are true concerning his diagnosis and outcome?

a.Incubation of hepatitis B virus is about two weeks


b.Jaundice is the first serologic indicator of hepatitis B infection
c.The patient has about a 10% chance of developing a chronic carrier state
d.All susceptible household or sexual contacts of the surgeon should receive hepatitis B viral vaccine
e.The surgeon should receive hepatitis B immunoglobulin as soon as possible after the accidental needle stick
Answer: c, d, e

Hepatitis B viral infection is insidious. The incubation period of the virus is about eight weeks. The first serum indicator of
infection by hepatitis B virus is detection of the serum hepatitis B surface antigen (HBsAg) which may proceed the onset of
jaundice. In most cases, hepatitis B infection is self-limited and does not progress to chronic hepatitis. However, some 10%
of patients with acute hepatitis B viral infection, whether it is clinical or subclinical, will develop a chronic carrier state. The
carrier state is defined by the presence of HBsAg in serum for longer than six months. The best method of treatment of
hepatitis B viral infection is primary prevention by vaccination. All susceptible household or sexual contacts of a person
with a positive serum test for HBsAg should be advised to receive a full course of hepatitis B viral vaccine. Passive
prophylaxis with hepatitis B immunoglobulin should be provided to any susceptible contact in whom there is recent
potential parenteral exposure such as an accidental needle stick.

109.The following statement(s) is/are true concerning the diagnosis and treatment of hydatid cysts.

a.Percutaneous aspiration is an important aspect of diagnosis and treatment of a hydatid cyst


b.CT scan will oftentimes show the classic findings of a cystic liver lesion with a calcific rim
c.At operation, care must be taken to protect the operative field from spillage of the cyst fluid
d.The use of a scoleocide has become obsolete with current surgical techniques
Answer: b, c

Hydatid cysts are most commonly the result of infection with the tape worm, Echinococcus granulosis. Routine laboratory
tests in patients with hydatid cysts are normal or nonspecifically abnormal. Although routine chest or abdominal radiographs
may show a mass with a calcific rim, sonography and CT scan are the favored means of imaging hydatid cysts. The
presence of calcifications and daughter cysts within the parent cyst suggests Echinococcus. Percutaneous needling of a
hydatid cyst is unwise unless precautions against anaphylaxis are undertaken. A cyst’s fluid is often under pressure, and
needling may precipitate rupture with the potential for anaphylaxis or intraperitoneal seating. The classic treatment of
hydatid cysts is operative. The surgical aim is to remove the cyst or cysts without dissemination of the organism. At
operation, the cyst is drained of fluid through a cannula after carefully protecting the operative field from fluid leakage. If
the aspirate is clear a parasiticidal fluid (ethyl alcohol or 20% sterile saline) is injected into the cyst to kill any adherent
scoleces. The cyst contents and the pericystic wall is then removed with careful surgical dissection.

110.Which of the following statement(s) is/are true concerning treatment of pyogenic liver abscess?

a.Antibiotic therapy alone may be advisable in patients with multiple small abscesses
b.Percutaneous drainage provides comparable results to surgical drainage in patients with unilocular large abscesses
c.Sufficient antibiotic coverage for most hepatic abscesses includes coverage for gram-positive aerobic bacteria only
d.In patients with a primary biliary origin for the hepatic abscess, treatment must also be addressed at underlying
biliary pathology such as choledocholithiasis or biliary ductal obstruction
Answer: a, b, d

The preferred treatment of most patients with hepatic abscesses is broad-spectrum antibiotic coverage and drainage. A
number of studies have demonstrated for most patients with large unilocular abscesses that percutaneous catheter drainage is
as effective as surgical drainage. Bacteria that predominate in pyogenic liver abscesses are gram-negative aerobes,
streptococcal species, and anaerobes. Therefore, broad-spectrum antibiotic coverage is necessary. Antibiotic coverage alone
may be advisable in occasional patients who have multiple small abscesses not accessible to percutaneous or surgical
drainage. Since many of these patients have an underlying biliary pathology as the source of the hepatic abscess, correcting
this underlying pathology, for example, establishing biliary drainage surgically or nonoperatively is important.
111. Which of the following statement(s) is/are true concerning acute, fulminant hepatic failure?

a.The most frequent cause of acute hepatic failure world-wide is hepatitis B infection
b.Higher grades of encephalopathy are associated with a worse prognosis
c.Hypoglycemia is a common complication of all liver diseases
d.Liver transplantation would appear indicated in all patients with hepatic coma secondary to acute liver failure
Answer: b

The diagnosis of acute (fulminant) hepatic failure is based on the development of encephalopathy within eight weeks of the
onset of symptoms. The overall prognosis is poor, but the hepatic lesions are potentially reversible, and recovery can lead to
restoration of normal liver function. The most frequent cause of acute hepatic failure world-wide is non-A, non-B viral
hepatitis. A variety of other viral agents and hepatotoxins can also cause this condition.
No reliable criteria predict outcome and response to treatment. Higher grades of encephalopathy (depth of coma) on
admission are associated with the worst prognosis. Management should include general supportive measures and specific
treatment for hepatic encephalopathy, cerebral edema, electrolyte and metabolic disturbances, infection, and pain.
Hypoglycemia is an unusual complication of most liver diseases except in patients with acute hepatic failure or hepatic
neoplasms. The enormous reserve capacity of the liver accounts for the rarity of hypoglycemia except as a preterminal
event. Bleeding is also a frequent cause of death in patients with acute hepatic failure secondary to depressed liver synthesis
of clotting factors and qualitative or quantitative platelet disorders. The lack of a definitive medical treatment for acute
hepatic failure makes liver transplantation seem attractive especially for patients with little or no chance of recovering
normal liver function. Perhaps the most significant drawback to widespread acceptance of liver transplantation for acute
hepatic failure is the lack of criteria reliability to predict which patients are likely to benefit from operation. Patients with
mild to moderate degrees of coma are likely to recovery spontaneously without the need for liver transplantation while rapid
deterioration and neurologic status to grade III or grade IV coma are associated in some centers with a mortality of 95%.

112.Which of the following statement(s) is/are true concerning the natural history and clinical features of alcoholic
cirrhosis?

a.In patients with compensated cirrhosis, the probability of survival at 10 years approaches 50%
b.The development of clinical evidence of hepatic decompensation reduces five year survival to less than 20%
c.Continued consumption of alcohol worsens prognosis
d.The risk of death after variceal hemorrhage depends more on the severity of underlying liver disease than the type
of therapy
Answer: a, b, c, d

Recent studies have analyzed the natural history of cirrhosis as a function of the degree of hepatic decompensation at the
time of diagnosis. A high proportion of patients with compensated cirrhosis remain well for many years after diagnosis. In
these studies the probability of remaining compensated 10 years after diagnosis was 42%, and survival probability of
compensated patients was 47%. The prognosis worsened considerably once patients developed clinical evidence of hepatic
decompensation (ascites, jaundice, encephalopathy, or gastrointestinal hemorrhage). Among these patients, the probability
of five-year survival was only 16%. The risk of death from variceal hemorrhage depends much more of the severity of the
underlying liver disease than on the type of therapy. It would also appear from natural history studies that continued alcohol
consumption does affect survival. In one study, the overall five-year survival is 63% for abstainers versus 40.5% for those
who continued to drink. Continued alcohol consumption may have less of an effect on survival than the intensity of alcohol
consumption. Furthermore, the degree of hepatic compensation at the time of inclusion into the study may have also been an
important factor.

113.Which of the following statement(s) is/are true concerning the morphologic and histologic findings of cirrhosis?

a.Micronodular cirrhosis is a pattern typical of chronic alcoholic liver disease


b.Mallory bodies and megamitochondria are typical findings of alcoholic cirrhosis
c.Bile leaks caused by rupture of bile ducts with extravasation of bile into portal triads is a common finding in post-
necrotic cirrhosis secondary to hepatitis
d.Large regenerating nodules separated by coarse irregular scars in piecemeal parenchyma necrosis is common in
liver disease secondary to chronic active hepatitis
Answer: a, b, d
Morphologic classification of cirrhosis includes micronodular, macronodular, and mixed forms. Micronodular cirrhosis is
characterized by uniform nodules and scars. The nodules are usually less than 3 mm in diameter and are typically associated
with Laennec or nutritional cirrhosis in alcoholics. Post-necrotic cirrhosis is characterized by large regenerating nodules
separated by coarse irregular broad as well as thin scars. This pattern is frequently seen in patients with viral hepatitis.
Biliary cirrhosis is characterized by a coarsely granular macronodular liver. This condition results from long-standing
cholestasis secondary to obstruction of intrahepatic or extrahepatic bile ducts. The most distinctive feature of large duct
obstruction is the presence of bile leaks caused by rupture of bile duct with extravasation of bile into portal triads. Portal
cirrhosis, which is typically observed in alcoholics, can generally be distinguished histologically by the presence of several
specific hepatocellular alterations such as Mallory bodies and megamitochondria.

114.The following statement(s) is/are true concerning the management of ascites associated with chronic liver disease.

a.Spontaneous bacterial peritonitis is an insignificant complication


b.Large volume paracentesis is unsafe due to excessive volume loss from the intervascular space
c.Peritoneovenous shunting is a trivial surgical procedure with minimal perioperative morbidity and mortality
d.Transjugular intrahepatic portosystemic shunts (TIPS) can effectively treat ascites in patients refractory to
conventional medical therapy
Answer: d

The onset of ascites usually indicates the presence of advanced liver disease. Cirrhotic ascites is usually straw colored, clear,
or greenish. Spontaneous bacterial peritonitis occurs as a complication of cirrhotic ascites in up to 10% of patients.
Spontaneous bacterial peritonitis is defined as infected ascitic fluid without a demonstrable other site of infection. This is a
serious complication with reported in-hospital mortality rates of 60% to 90%. The rational approach of therapy for ascites
includes sodium and fluid restriction, the use of diuretics, and the use of therapeutic paracentesis. Several studies have
shown that repeated paracentesis in stable cirrhotic patients may be safe and effective as medical therapy and shortens the
length of hospitalization. Single, large volume paracentesis has been reported to be effective and safe. Up to 10 liters of
ascites can be removed in one hour if salt-poor albumen is administered simultaneously. In a small percentage of patients,
surgical implantation of a peritoneovenous shunt may be advisable. The principal indication for use of peritoneovenous
shunt is to stabilize ascites that is refractory to conventional medical therapy and therapeutic paracentesis. Despite the
simplistic nature of the device, postoperative mortality and morbidity rates of 20% to 60%, respectively have been reported.
Precipitation of disseminated intravascular coagulopathy, variceal hemorrhage, or hepatic failure may complicate this
procedure. Transintrahepatic portosystemic shunts (TIPS) have been demonstrated to control ascites in one study in over
90% of patients with ascites refractory to medical management. However, patients with poor hepatic reserve in this study all
died if orthotopic liver transplantation was not performed. This data suggests that TIPS is effective for refractive ascites in
patients with good to moderate hepatic reserve but poor risk cirrhotics require orthotopic liver transplantation to correct this
problem.

115.Which of these statement(s) is/are true concerning the etiologic factors in the development of cirrhosis?

a.Viral hepatitis of any type (A, B, or non-A, non-B) can all progress to cirrhosis
b.Acetaminophen can cause acute liver failure and necrosis but will not lead to cirrhosis
c.Alcohol exerts toxic effects on the liver via reactive intermediates such as acetaldehyde
d.Long-standing congestive heart failure can lead to cirrhosis secondary to centrilobular congestion, hemorrhage, and
necrosis
Answer: c, d

Liver cells are sensitive to a variety of physical, microbiologic, and chemical agents, all of which may produce cellular
injury. The eventual development of cirrhosis is determined by the nature and severity of the cellular injury and the liver’s
ability to regenerate. Most infectious hepatitides are viral in origin. There is no documentation that hepatitis A progresses to
cirrhosis, however hepatitis B and non-A, non-B hepatitis do demonstrate a propensity to become chronic with the
development of cirrhosis. Chemical hepatotoxicity include direct and indirect actions. Acetaminophen under normal
circumstances is detoxified chiefly by conjugation with glucuronic acid or sulfate. Saturation of glucuronic pathway with
large doses of acetaminophen results in progressive depletion of intracellular glutathione stores, accumulation of toxic
intermediate, and eventual cell necrosis which may progress to either acute liver failure or chronic cirrhosis. Alcohol can
affect liver cell function in a number of ways. Like many hepatotoxins, the toxic effects of alcohol are caused indirectly by
reactive intermediates. Acetaldehyde is the principal reactive compound generated by alcohol metabolism. The
hepatotoxicity of acetaldehyde is related to its binding two major constituents of cellular membranes altering membrane
integrity and enzymatic function frequently to the detriment of the cell. Early morphologic changes of long-standing cardiac
decompensation and right-sided heart failure are central lobular congestion, hemorrhage, and necrosis combined with
phlebosclerosis of central veins and scars connecting centrizonal areas (cardiac cirrhosis).

116.Important spontaneous portosystemic collaterals which develop in the face of portal hypertension include:

a.The hemorrhoidal veins


b.Left renal vein
c.The paraumbilical venous plexus
d.The coronary, short gastric, and paraesophageal veins
Answer: a, b, c, d

If portal pressure is elevated, spontaneous portosystemic collaterals develop in an attempt to decompress the portal system.
Such collaterals increase venous return to the heart and increase cardiac output. In humans, the most important collaterals
develop as tributaries of the coronary, short gastric, and paraesophageal veins; intercostal, esophageal, and azygous veins;
the superior, middle, and inferior hemorrhoidal veins; and the peraumbilical plexus. Retroperitoneal veins and veins
draining to the left renal vein from the splenic, adrenal, and gonadal veins may also serve as sites of the development of
venous collaterals.

117.Which of the following statement(s) is/are true concerning the pathophysiology of variceal hemorrhage?

a.All patients with portal hypertension will develop esophageal varices


b.All patients with esophageal varices eventually bleed
c.Variceal size can predict the incidence of variceal hemorrhage
d.Control of acid secretion by H2 blockade can decrease the incidence of rebleeding after esophageal hemorrhage
e.None of the above
Answer: e

About two-thirds of patients with portal hypertension develop varices; of these only two-thirds subsequently experience
variceal hemorrhage. A number of factors are important in the pathogenesis of variceal hemorrhage. These include portal
pressure, intravariceal pressure, variceal size and structure, and other factors. Variceal size alone is not predictive of variceal
hemorrhage. Evidence conflicts about whether erosive esophagitis is a cause of variceal rupture. Control of acid reflux by
H2 blockade has not been shown to decrease the incidence of rebleeding after esophageal hemorrhage.

118.Which of the following statement(s) is/are true concerning the use of transjugular intrahepatic portosystemic shunts
(TIPS) in the treatment of variceal bleeding?

a.This procedure effectively creates an end-to-side portocaval shunt


b.Procedure-related mortality is generally in excess of 20%
c.TIPS has been used successfully as a pretransplant procedure to reduce portal pressure
d.The placement of a TIPS is not associated with the development of encephalopathy
Answer: c

Transjugular intrahepatic portosystemic shunts (TIPS) refer to an implantable, expandable metal stent placed radiologically
through the hepatic parenchyma to establish a track between branches of the hepatic and portal veins. TIPS results in similar
hemodynamics as a side-to-side portal systemic shunt. There is firm clinical data that TIPS provides effective control of
acute variceal hemorrhage and portal hypertension regardless of the etiology of the underlying liver disease or the degree of
hepatic decompensation. TIPS has also been used for preoperative portal decompression to facilitate orthotopic liver
transplantation. Pretransplant TIPS should reduce portal pressure thereby reducing operative time and blood loss. The major
complications of TIPS include encephalopathy and stenosis or occlusion of this stent. Encephalopathy occurs in 10% to
20% of patients after TIPS. This complication appears to correlate with increasing age of the patient and increased shunt
diameter and shunt flow.

119.Hepatic encephalopathy is a common systemic manifestation of chronic liver disease. Which of the following
statement(s) is/are true concerning this condition?
a.Blood ammonia levels correlate well with the stage of hepatic encephalopathy
b.Alterations in central nervous system neurotransmitters such as the neurotransmitter g- aminobutyric acid (GABA)
have been proposed in the pathogenesis of hepatic encephalopathy
c.Lactulose can be used to decrease intestinal ammonia absorption
d.Patients can be expected to have an increased sensitivity to benzodiazepines
Answer: b, c, d

Hepatic encephalopathy, a poorly explained neuropsychiatric syndrome, characterized by diverse neurologic abnormalities,
is the pathologic evidence of nonspecific structural changes in neurons, and a variable prognosis. Several hypotheses to
explain the pathogenesis of hepatic encephalopathy have been proposed. Ammonia has been widely implicated in the
pathogenesis of hepatic encephalopathy despite conflicting evidence. Blood ammonia levels correlate poorly with the stage
of encephalopathy, however, one of the mainstays of treatment is measures to decrease ammonia absorption from the gut
including the oral administration of lactulose. Another hypothesis has implicated false neurotransmitters in the pathogenesis
of hepatic encephalopathy. Evidence suggests that activation of the GABA system may be important in the pathogenesis of
hepatic encephalopathy. The GABA receptor binds several classes of ligands including GABA and drugs such as
benzodiazepines. For reasons that are unclear, hepatic failure appears to increase the brain density of GABA receptors. This
observation may explain the increased sensitivity to benzodiazepines and other inhibitory neurotransmitters observed with
patients with chronic liver disease.

120.Which of the following statement(s) is/are true concerning the management of gastroesophageal variceal
hemorrhage?

a.Vasopressin decreases portal pressure through the process of splanchnic vasoconstriction


b.Somatostatin is as effective as vasopressin but without the cardiac side effects
c.Balloon tamponade provides good long-term control of bleeding esophageal varices
d.Endoscopic sclerotherapy is more effective than conservative medical therapy in the treatment of bleeding
esophageal varices
e.Sclerotherapy, although excellent for the control of bleeding short-term, does not prolong overall survival
Answer: a, b, d

A number of invasive and noninvasive therapies exist for the treatment of bleeding esophageal varices. Vasopressin is
frequently used in the treatment of acute variceal hemorrhage and acts by decreasing portal venous pressure or flow through
splanchnic vasoconstriction. Vasopressin alone has been reported to temporarily control variceal hemorrhage in 50% to 75%
of patients, however, it can be associated with cardiac and peripheral extremity cutaneous ischemia. Somatostatin also acts
as a vasoconstrictor to reduce splanchnic flow, with trials demonstrating similar efficacy in controlling acute hemorrhage
when compared to vasopressin but without the cardiac side effects. Balloon tamponade is generally used for the temporary
control of acute variceal hemorrhage unresponsive to vasopressin or sclerotherapy. Initial control of acute variceal
hemorrhage occurs in about 80% of patients, but bleeding recurs promptly on deflation of the balloons in over 50%.
Endoscopic sclerotherapy has become the primary treatment for bleeding esophageal varices. A number of clinical trials
demonstrate that emergent sclerotherapy is able to halt variceal bleeding that fails to respond to more conservative
measures, with no increase in frequency or severity of complications. The long-term survival of patients treated with
sclerotherapy continues to be debated. Metaanalysis has been performed on the data from seven randomized clinical trials
evaluating the effect of repeated sclerotherapy on long-term survival. This analysis demonstrates that sclerotherapy reduces
the number of deaths by 25% therefore supporting the use of sclerotherapy as an effective means of prolonging survival in
patients who have experienced variceal hemorrhage.

121.Which of the following statement(s) is/are true concerning the surgical management of bleeding esophageal
varices.

a.A side-to-side portacaval shunt may be associated with the development of hepatofugal blood flow
b.Selective shunts preserve prograde (hepatopedal) blood flow while decompressing esophageal varices or reducing
portal pressure
c.The presence of intractable ascites is a contraindication to the Warren shunt
d.If the patient is considered a liver transplant patient, an interposition mesocaval shunt is a suitable alternative
Answer: a, b, c, d
Although portosystemic shunts are the most effective therapy for preventing recurrent variceal hemorrhage, they are
associated with the increased incidence of encephalopathy. A number of types of shunts have been described, primarily to
avoid the consequences of complete diversion of portal blood flow or to simplify the operation. End-to-side portocaval
shunts are hemodynamically unique in that all portal flow is diverted and the hepatic limb of the portal vein is ligated, thus
preventing hepatofugal blood flow from the liver. With lateral side-to-side shunts, the hepatic limb of the portal vein
remains patent. A greater compensatory increase in hepatic arterial flow occurs when the portal vein serves as an outflow
track and the liver extracts oxygen and metabolites from the blood exiting through the patent limb of the portal vein
(hepatofugal). The goal of selective shunts is to preserve prograde (hepatopedal) portal flow to the liver while selectively
decompressing gastroesophageal varices (Warren shunt) or reducing portal pressures sufficiently to prevent variceal
hemorrhage (small-diameter shunts). Most surgeons with a special interest in this field attempt to construct a selective shunt
when the operation is elective, unless contraindications are present. The distal splenorenal shunt of Warren and the small-
diameter interposition portocaval shunt of Sarfeh are the most common selective shunts used. The Warren shunt is an
ascitogenic operation and therefore the presence of ascites that is difficult to control medically is a contraindication of this
operation. An interposition mesocaval shunt is frequently preferred in emergent situations because the shunt is relatively
safe to construct and promptly halts variceal hemorrhage. If future hepatic transplantation is contemplated, this shunt offers
the advantages in that it does not involve dissection of the area of the hepatoduodenal ligament.

122.Which of the following statement(s) is/are true concerning the results of portosystemic shunting?

a.When comparing shunts with nonshunting procedures, only minor differences in long-term survival are reported,
but the mode of death usually changes
b.Distal splenorenal shunts are associated with the development of less hepatic encephalopathy
c.Survival statistics following distal splenorenal shunt in good risk patients (Child’s Class A) are in excess of 80%
d.None of the above
Answer: a, b, c

Multiple randomized comparisons of distal splenorenal shunt with other types of portosystemic shunts have been reported.
Operative mortality and long-term prognosis are similar, but patients undergoing distal splenorenal shunts develop less
encephalopathy. Less encephalopathy is also seen with the selective Sarfeh shunt and after nonshunting procedures, such as
sclerotherapy or devascularization. When comparing shunts with nonshunting procedures, only minor differences in long-
term survival are reported, but the mode of death usually changes. With nonshunting procedures, a greater proportion of
patients die of recurrent hemorrhage; after construction of a shunt, a greater proportion die of hepatic failure. Class A
patients undergoing distal splenorenal shunts can be predicted to have an actuarial survival including operative mortality in
excess of 80%. This is a substantial improvement over previous experience with portocaval shunts and is likely better than
the results that can be obtained with hepatic transplantation.

123.Which of the following statement(s) is/are true concerning radiologic examinations used for the assessment of
hepatic neoplasms?

a.Magnetic resonance imaging is considered the test of choice for distinguishing hemangiomas from other mass
lesions
b.Dynamic CT scanning is the dominant imaging modality for routine screening and diagnosis
c.Radionucleotide imaging plays an important role in modern screening and detection for liver lesions
d.CT angio-portography (CTAP) is the gold standard for early detection of metastatic hepatic lesions
Answer: a, b, d

In the Western world, the test most commonly used for screening, detection, and diagnosis of hepatic neoplasms are
dynamic bolus-enhanced CT scan, MRI, and ultrasound. Dynamic CT scanning remains the dominant imaging modality for
routine screening and diagnosis because it is sensitive, widely available, provides helpful anatomic information and allows
the evaluation of other intraabdominal structures in the detection of extrahepatic disease. The most sensitive test for the
detection of liver masses is CTAP, however, like angiography is generally reserved for preoperative evaluation of
resectability and not for screening. Hepatic MRI imaging is another useful tool in the detection and diagnosis of the liver
lesions, with sensitivities equal to that of CT. MRI with T1 and T2-weighted images and gadilinium enhancement can aid in
the differential diagnosis of a mass. For example, MRI is now considered the test of choice for distinguishing hemangiomas
from other mass lesions. Radionucleotide imaging is less sensitive and specific than CT, MRI, or ultrasound, and has a high
percentage of false-positive and false-negative results. Therefore, nuclear medicine scans play a limited role in modern
screening and detection, but they can help differentiate discrete masses.
124.A 45-year-old woman undergoes an ultrasound because of vague right upper quadrant pain and epigastric fullness.
A 7 cm cystic lesion is detected. Which of the following statement(s) is/are true concerning the patient’s diagnosis
and management?

a.Simple aspiration is indicated for treatment and diagnosis


b.Bile stained fluid suggests underlying biliary pathology
c.The cyst is likely lined by cuboidal epithelium
d.Laparoscopic unroofing of the cyst can provide satisfactory treatment
Answer: c, d

A symptomatic solitary hepatic cyst may cause vague right upper quadrant discomfort or pain, a sensation of epigastric
fullness or heaviness, and early satiety, however, most cases are asymptomatic. Complications are rare but include
hemorrhage into the cyst, secondary bacterial infection, or obstructive jaundice from compression of extrahepatic ducts. In
the absence of complications, laboratory abnormalities are uncommon. Cysts are somewhat more common in females, are
more common in the right lobe and are often multilocular rather than unilocular. They are lined with cuboidal epithelium
resembling bile duct epithelium and are filled with fluid that may be clear, mucoid, bloody, or bilious.
If the patient has no symptoms and the cyst was discovered incidentally and there is no evidence of infection or malignancy,
one may observe the patient. Neither percutaneous aspiration nor surgery is indicated. Cysts nearly always recur after
simple aspiration. Treatment of symptomatic cysts is surgical. Indications for surgery include symptoms, rupture,
hemorrhage, or infection. Asymptomatic, uninfected simple cysts are best treated by excision, if possible. Larger cysts may
be inroofed with free peritoneal drainage unless there is a history of hemorrhage or evidence of biliary communication. A
laparoscopic approach to the unroofing of cysts has recently been reported with excellent success. If this cyst communicates
with the biliary system (grossly by cholangiography) the leak may be oversewn or the cyst drained by a Roux-en-Y
cystojejunostomy.

125.Which of the following statement(s) is/are true concerning biopsy techniques for hepatic masses?

a.A fine needle aspiration (FNA for cytology is contraindicated for patients with hypervascular masses)
b.Percutaneous biopsy should be performed only if results may obviate the need for exploratory laparotomy
c.Needle track seeding of tumor is not a risk associated with percutaneous biopsy
d.Laparoscopy and biopsy play little role in the management of liver lesions
Answer: b

Biopsy of a liver mass can be done percutaneously (with or without CT or ultrasound guidance), laparoscopically, or at
laparotomy. The biopsy may be done for cytology only (FNA) or for histology (larger-core biopsy). Guided FNA has an
overall sensitivity of 77% to 94% and may allow a distinction between primary and secondary malignancy. The risks
associated with needle biopsy include bleeding, infection, needle track seeding of tumor, and sampling error. Hypervascular
masses, coagulopathy, and ascites are contraindications to percutaneous core biopsy, however, FNA biopsy is generally
considered safe under these circumstances. In evaluation of any liver mass, percutaneous biopsy should be performed only
if it can reasonably be expected to obviate the need for exploratory laparotomy. Biopsy of suspected primary metastatic
malignancy with clinical indications of unresectability may spare the patient an unnecessary laparotomy. Laparoscopy with
biopsy may also be used to evaluate liver masses and to possibly avoid laparotomy in patients considered to be borderline
resectable.

126.A 55-year-old woman presents with vague right upper quadrant pain and a palpable liver. Laboratory tests are
normal and a noncontrast CT scan (patient has a history of contrast allergy) reveals an 8 cm right hepatic mass.
Which of the following statement(s) is/are correct concerning the patient’s diagnosis and management.

a.A gadilinium-enhanced MRI would be indicated to define the extent of the lesion and confirm the diagnosis of
hemangioma
b.A fine needle aspiration should be performed regardless of radiographic workup
c.Hepatic embolization is the treatment of choice
d.The lesion should be resected because of concern for malignant degeneration
Answer: a
Cavernous hemangiomas of the liver are the most common benign hepatic tumor and are detected in some 2% to 7% of
autopsies. Overall, hemangiomas are exceeded only by hepatic metastases as the most common hepatic tumor. Cavernous
hemangiomas consist histologically of cystically dilated, endothelium-lined vascular spaces. They occur in all ages but are
observed more commonly in females. Hemangiomas are not premalignant. Less than half of affected patients have
symptoms. Those that have symptoms usually have large masses. Symptoms usually occur including vague right upper
quadrant discomfort, pain, fullness, and early satiety. Physical examination may be notable for hepatomegaly, mass or bruit.
There are no laboratory abnormalities in patients with hemangiomas. The most useful radiologic test for diagnosing
hemangiomas are MRI, CT and tagged red blood cell scanning. These tests have largely replaced angiography. CT with
vascular contrast often demonstrates a diagnostically characteristic enhancement pattern. Gadilinium-enhanced MRI has
recently been shown to be sensitive and specific in the diagnosis of hemangioma and has better resolution than tagged red
blood cell scans. FNA biopsy of suspected hemangiomas can be performed, however this procedure should be avoided if the
diagnosis is secure using noninvasive procedures. Given the natural history of hemangiomas and its low risk of rupture,
observation is indicated for asymptomatic patients, especially for lesions smaller than 4 cm. Surgical excision is the only
consistently effective treatment for symptomatic masses and should be performed if the lesion is localized and accessible
with an acceptable operative risk. Embolization is indicated only for unresectable lesions and is only modestly effective.

127.Which of the following statement(s) is/are true concerning the pathogenesis of hepatocelluar carcinoma?

a.Hepatocellular carcinoma is associated with infection with hepatitis A, hepatitis B, and hepatitis C
b.The risk of developing hepatocellular carcinoma related to hepatitis B viral infection is related to the chronicity of
the infection
c.Hepatocellular carcinoma only develops in cirrhosis associated with hepatitis
d.Alfatoxin B1 is a potent carcinogen in the development of hepatocellular carcinoma
Answer: b, d

Hepatocellular carcinoma has three well-known epidemiologic associations: hepatitis B infection, cirrhosis, and various
hepatotoxins, most notably aflatoxin B1 (a microtoxin from the fungus Aspergillus flavus). Hepatocellular carcinoma is
strongly associated with hepatitis B viral infection but there is no link between the neoplasm and the hepatitis A viral
infection. There does appear to be some link between hepatocellular carcinoma and hepatitis C virus. Cirrhosis,
predominantly macronodular, is a frequent result of hepatitis B viral infection and is a risk factor for hepatocellular
carcinoma. Nevertheless, the micronodular form, which is more common in early alcoholic cirrhosis, will progress to
hepatocellular carcinoma in about 2% to 3% of cases. Thus, hepatocellular carcinoma can develop in patients with hepatitis
B viral infection and cirrhosis, hepatitis B viral infection alone, and with cirrhosis from any cause alone.

128.A 38-year-old woman with a 17 year history of oral contraceptive use presents with right upper quadrant pain. A
CT scan demonstrates a 4 cm lesion in the right lobe of the liver. Which of the following statement(s) is/are true
concerning the patient’s diagnosis and management?

a.The lesion is likely premalignant


b.A 99mTc sulfur colloid scan will distinguish this benign lesion from a malignant hepatoma
c.The lesion, although benign, may be associated with life-threatening hemorrhage
d.The lesion would be expected to be hypervascular on angiographic study
Answer: c, d

Hepatic adenomas are clearly linked with the use of oral contraceptives. The likelihood of developing a hepatic adenoma
appears to be related to the duration and dosage of estrogen, and is greater at ages above 30 years. The majority (75%) of
these lesions occur in the right lobe of the liver. Although adenomas may be a symptomatic, most patients have symptoms,
usually of abdominal pain (as many as 50%), and 10% to 33% of patients present with acute signs and symptoms secondary
to bleeding or rupture with intraperitoneal hemorrhage.
No radiologic test is specific for adenomas. CT is the most useful preliminary test and often reveals areas of hemorrhage
and necrosis. Angiography may add to CT findings by demonstrating the hypervascular tumor with a peripheral blood
supply. 99mTc sulfur colloid scans show a cold spot thereby distinguishing adenoma from focal nodular hyperplasia but not
from other solid masses. For asymptomatic patients or patients with minimal symptoms, surgery is the treatment of choice
given the tumors have the tendency to bleed and a small chance of coexisting malignancy.
129.Which of the following statement(s) is/are true concerning focal nodular hyperplasia (FNH)?

a.The lesion predominantly affects young women


b.The lesion is associated with the use of oral contraceptives and other estrogens
c.Radionucleotide scanning can be useful in the specific diagnosis of FNH
d.Excisional biopsy is indicated in almost all cases because of the risk of bleeding
Answer: a, c

Focal nodular hyperplasia (FNH) should not be confused with a hepatic adenoma. Although FNH predominantly affects
young women, it is also found in men and children. Unlike adenomas, there is no clear relationship between oral
contraceptives and the development of FNH. FNH is most commonly asymptomatic and does not have a propensity to bleed
or undergo malignant change. Histologically, FNH contains normal-appearing hepatocytes, bile ducts, and Kupffer cells in
distinction to adenomas. Radionucleotide imaging can be useful in diagnosing FNH because FNH is the only lesion that
contains Kupffer cells and therefore appears isodense rather than a filling defect. Treatment of asymptomatic patients is
conservative when the diagnosis is clear. If there is doubt regarding the diagnosis, then excisional biopsy is indicated for
small, easily removable lesions.

130.Which of the following statement(s) is/are true concerning the prognosis of patients with hepatic metastases and
colorectal carcinoma?

a.Over half of these patients will survive one year without treatment
b.Five year survivals following hepatic resection for an isolated metastasis is in excess of 25%
c.Survival beyond five years after resection suggests a high probability of cure
d.Survival rates are improved with a margin of resection greater than 1 cm
e.The size of a liver metastasis is not a significant factor in predicting recurrence if adequate margins can be obtained
Answer: b, c, d, e

Studies consistently report five-year survival rates averaging 25% for hepatic resection for colorectal metastases. Those who
survive beyond five years seem to do well with only an additional 5% dying of recurrence within the next five years.
Median survival of patients with untreated metastases is on the order of three to 10 months, with only 20% surviving past
one year. Overall survival is significantly improved with surgical margins greater than 1 cm with decreased survival in
patients with positive margins or margins less than 1 cm in size. The number of metastasis is a less consistent but
statistically significant factor. Patients with four or more metastases have a poorer prognosis. As for the size of the
metastasis, it is not a significant factor except that a larger total liver volume of metastasis requires a larger hepatic
resection. Larger size may preclude adequate margins and indicate longer development of time with an increased likelihood
of micrometastases.

131.Which of the following statement(s) is/are true concerning the treatment and prognosis of hepatocellular
carcinoma?

a.The fibrolamellar variant of hepatocellular carcinoma has a distinctly better prognosis than other forms of the
disease
b.Patients with untreated hepatocellular carcinoma rarely survive a year
c.Multiagent chemotherapy is extremely effective in hepatocellular carcinoma and should be considered for the
treatment for most patients
d.Hepatic artery ligation or embolization has been demonstrated to be highly effective for hepatocellular carcinoma
Answer: a, b

Hepatocellular carcinoma has a tendency for local and vascular invasion. Untreated, patients have a dismal prognosis with a
median survival of only three to four months after diagnosis with only rare survival beyond one year. The fibrolamellar
variant of hepatocellular carcinoma has a distinctly better prognosis with a five year survival after resection of 50% to 60%
versus 25% for other varieties. Surgical resection offers the only chance for cure for primary hepatic malignancy, and
survival is better if tumors are small and asymptomatic. The role of orthotopic liver transplantation for hepatocellular
carcinoma continues to evolve. For patients with severe hepatic dysfunction or larger or centrally located tumors or bilobar
tumors, this may be the preferred approach. Extrahepatic disease including lymph node involvement is a contraindication to
resection or transplantation. No single-or multiple-agent chemotherapy is particularly effective in treating hepatocellular
carcinoma. The rationale for treating vascular liver tumors with arterial obstruction is the fact that the nutritional blood
supply of these tumors comes primarily from the hepatic artery. Hepatic artery ligation or embolization alone or in
combination with regional chemotherapy has been shown to be ineffective for liver cancer. In addition, hepatic artery
ligation has a high complication rate and cannot often be used in patients with compromised liver function from cirrhosis.

132.The following statement(s) is/are true concerning the relationship of the biliary tree and the hepatic artery and
portal vein.

a.The common hepatic and common bile duct lie immediately anterior to the portal vein
b.The cystic artery, which usually arises from the right hepatic artery, crosses behind the hepatic duct in most cases
c.A replaced right hepatic artery arising from the superior mesenteric artery system will run to the right of the
common bile duct
d.The arterial supply of the extrahepatic biliary ducts is derived from major trunks running along the medial and
lateral walls of the common duct at the 3 o’clock and 9 o’clock position
Answer: a, b, c, d

The common hepatic duct passes downward in the superior and lateral portions of the hepatoduodenal ligament and lies in
front of the portal vein and to the right of the hepatic artery. The cystic artery which in most cases arises from the right
hepatic artery usually crosses behind the hepatic duct (84%) of cases but is sometimes anterior to that structure (16%) of
cases. A number of anatomic variations of the arterial supply of the liver can be seen. In patients in which the right hepatic
artery arises from the superior mesenteric artery system, the "replaced" right hepatic artery usually runs to the right of the
bile duct and portal vein. The arteries to the extrahepatic biliary ducts anastomose freely within the duct walls. The ductal
arterial supply is derived primarily from the gastroduodenal and right hepatic arteries with major trunks running along the
medial and lateral walls of the common duct at the 3 o’clock and 9 o’clock positions.

133.An understanding of the anatomy of the extrahepatic biliary tree is essential in performing biliary tract surgery.
Which of the following statement(s) is/are true concerning biliary ductal anatomy?

a.The majority of patients have the "classic" anatomical description


b.The common hepatic duct unites with the cystic duct to form the common bile duct
c.An accessory right hepatic duct occurs in 5% of patients
d.A common channel or "Y" configuration of the distal bile duct and pancreatic ducts occur in approximately 70% of
patients
Answer: b, c, d

The classic anatomical description of the extrahepatic bile ducts and their arteries is present in only one-third of individuals.
The left hepatic duct usually has a longer extrahepatic course than does the right hepatic duct. The common hepatic duct is
formed by the union of the right and left hepatic ducts close to the emergence from the liver. The duct passes downward in
the superior portion of the hepatoduodenal ligament and lies in front of the portal vein and to the right of the hepatic artery.
The common hepatic duct unites with the cystic duct to form the common bile duct. An accessory right hepatic duct occurs
in 5% of cases. The cystic duct passes downward, backward, and to the left in the hepatoduodenal ligament and usually
unites with the main hepatic duct at an acute angle. Its course and mode of insertion into the common duct is highly
variable. The common bile duct is formed by the union of the common hepatic and cystic ducts and is usually about 7 to 9
cm long. The junction of the distal common bile duct and pancreatic duct at the ampulla can take one of three
configurations. In approximately 70% of patients, there is a common channel of the bile and pancreatic ducts, thus a "Y"
configuration. In approximately 20%, the common channel is nonexistent, and in another 10%, the two ducts enter the
duodenum via separate openings.

134.A 35-year-old female presents with typical biliary colic symptoms, however her sonogram shows no gallstones.
Which of the following statement(s) is/are true concerning her diagnoses?

a.Chronic acalculous cholecystitis or gallbladder dyskinesia is seldom associated with classic biliary colic symptoms
b.The most specific test for diagnosing gallbladder dyskinesia is CCK-enhanced cholescintigraphy with assessment
of gallbladder ejection fraction
c.An ejection fraction greater than 75% is considered abnormal and indicative of gallbladder dyskinesia
d.Cholecystectomy is not indicated for chronic acalculous cholecystitis
Answer: b
Motility abnormalities of the gallbladder and cystic duct present with symptoms suggesting gallstones. The most common
presentation for patients with gallbladder motility disorders such as chronic acalculous cholecystitis or gallbladder
dyskinesia is recurrent biliary-type pain. Currently, the most specific test for diagnosing gallbladder dyskinesia is CCK-
enhanced cholescintigraphy with assessment of gallbladder ejection fraction. CCK is infused intravenously 15 to 30 minutes
after ejecting an analogue of 99MTC imminodiacetic acid and calculating the ejection fraction of the isotope by the
contracting gallbladder. An ejection fraction of less than 35% is considered abnormal and cholecystectomy may be
indicated. Most patients will have relief of symptoms following cholecystectomy.

135.The following statement(s) is/are true concerning the embryology of the biliary tree.

a.The primordial anlagen of the liver and biliary tract arises from the entoderm
b.Superior and inferior caudal buds form as the hepatic diverticulum develops
c.The development of the liver is a separate process from that of the gallbladder and distal biliary tree
d.The biliary tree develops in association with the dorsal pancreas
Answer: a, b

The primordial anlagen of the liver, extrahepatic bile ducts, gallbladder, and ventral part of the pancreas develop as a
thickened area of entoderm on the ventral surface of the caudal portion of the foregut where it joins the midgut. Superior
and inferior caudal buds form as the hepatic diverticulum grows out into the ventral mesogastrium. The solid mass of
endodermal cells spreading with this cephalic bud forms the right and left lobes of the liver. The superior growth of the
cranial portion of the hepatic diverticulum, which extends from the duodenum to the liver, results in the formation of the
hepatic, common hepatic, and common bile ducts. The caudal portion of the hepatic diverticulum develops into the
gallbladder and cystic duct. The common bile duct is attached to the ventral aspects of the duodenum and is in close contact
with the ventral pancreatic bud.

136.The following statement(s) is/are true concerning biliary motor function.

a.The contracted sphincter of Oddi impairs bile flow into the duodenum and directs it into the gallbladder
b.In the postprandial state about 70% of hepatic bile flows into the gallbladder before reaching the duodenum
c.During the interdigestive period, only a small fraction of gallbladder bile enters the duodenum
d.Gallbladder emptying during fasting is associated with phase III of the interdigestive migrating motor complex
(MMC)
e.After a meal, the gallbladder empties by a steady tonic contraction thought to be due to release of endogenous
motilin from the mucosa of the small intestine
Answer: a, b, c

As bile is secreted from the liver, it flows through the hepatic ducts into the common hepatic duct and continues through the
common bile duct into the duodenum. With an intact and contracted sphincter of Oddi, bile flows directly into the
gallbladder where it is concentrated and stored. In the postprandial state, about 70% of hepatic bile flows into the
gallbladder before reaching the duodenum and entering the enterohepatic cycle. During the interdigestive phase, 90% of bile
from the liver enters the gallbladder, while only a small fraction of gallbladder bile enters the duodenum. Gallbladder
emptying during fasting is associated with phase II of the interdigestive migrating motor complex (MMC). Motilin may
account for this stimulatory effect since plasma elevations of motilin seem to correlate with the onset of phase II waves.
Following a meal, the gallbladder empties by a steady tonic contraction thought to be due to release of endogenous
cholecystokinin (CCK) from the mucosa of the small intestine.

137.The following statement(s) is/are true concerning the anatomy of the gallbladder.

a.The gallbladder lies between the right, left, and quadrate hepatic lobes or hepatic segments IV and V
b.The cystic duct contains the spiral valve of Heister which serves an important valvular function for the gallbladder
c.The cystic artery arises from the right hepatic artery in 95% of cases
d.The cystic artery crosses anterior to the hepatic duct in the majority of cases
Answer: a, c
The gallbladder is a pear-shaped organ bound to a fossa on the right inferior surface of the liver located between the right,
left, and quadrate hepatic lobes or hepatic segments IV and V. The gallbladder can be divided into four areas: the fundus,
body, infundibulum, and neck. The body of the gallbladder extends from the fundus into the tapered portion, or neck, which
curves backward and upward toward the transverse fissure of the liver and terminates in the cystic duct. The cystic duct
lumen contains a thin mucosal septum, the spiral valve of Heister; the valve may make catheterization to the cystic duct
difficult but does not have true valvular function. The arteries of the gallbladder are derived from the cystic branch of the
hepatic artery which in 95% of cases originates from the right hepatic artery. From its origin, the cystic artery usually
crosses behind the hepatic duct (84% of cases) but is sometimes anterior to that structure. The cystic artery proceeds to the
neck of the gallbladder, where it divides into anterior and posterior divisions that supply the corresponding areas of the
gallbladder. The cystic veins empty into the right branch of the portal vein indirectly into the liver.

138.The gallbladder plays an important role in altering bile composition by absorption and secretion. Which of the
following statement(s) is/are true concerning this mucosal function?

a.The absorption of water by the gallbladder can result in concentration of the solute components of bile from 2-to
10-fold
b.Gallbladder mucosal absorption can occur by both active and passive mechanisms
c.Cyclic adenosine monophosphate (cAMP) stimulates NaCl-coupled transport and may also influence tight junction
permeability
d.Secretory products of the gallbladder include bicarbonate and glycoproteins
Answer: a, b, d

The gallbladder rapidly absorbs water and solutes from bile and concentrates the solute components 2-to 10-fold. The
gallbladder has an active mucosa and is able to absorb water and solutes against significant concentration gradients. Water
absorption is linked to the transport of ions. The two major mechanisms of absorption are active and passive. In passive
absorption, sodium and chloride enter the gallbladder epithelial cells because of electrochemical gradients. This results in an
osmotic gradient, and water flows into the cell. Intracellular sodium is extruded across the basolateral membrane into the
lateral intercellular spaces by active transport. The active transport of sodium against an electrochemical gradient is
associated by an Na+-K+-ATPase pump. Cyclic AMP may inhibit NaCl-coupled transport and may also influence tight
junction permeability. Other peptides, such as secretin, glucagon, and gastric inhibitory peptide, have been shown to inhibit
absorption. Secretion by the gallbladder occurs either by inhibiting net ion and fluid absorption or with stimulation of
bicarbonate secretory mechanisms. Gallbladder epithelium may also secrete mucin and nonmucin glycoproteins which may
play a role in gallstone formation.

139.Abnormalities of the sphincter of Oddi have been recently recognized to cause symptoms which are referable to the
biliary tree or pancreas. The following statement(s) is/are true concerning sphincter of Oddi motor function.

a.The sphincter’s basal resting pressure is 10 to 15 mm Hg above duodenal pressure


b.Contraction of the sphincter occurs with CCK stimulation
c.Vagal stimulation results in relaxation of the sphincter
d.Manometry of the sphincter of Oddi may be performed at the time of ERCP to characterize basal pressure,
amplitude, frequency of contraction, and direction of propagation of contractile waves
e.Stenosis of the sphincter of Oddi is characterized by abnormally elevated basal pressure on sphincter of Oddi
manometrics
Answer: a, c, d, e

The sphincter of Oddi is about 4 to 6 mm in length. The sphincter’s basal resting pressure is about 13 mm Hg above
duodenal pressure. The sphincter exhibits phasic contractions at a frequency of 4 per minute and a duration of 8 seconds.
The regulation of bile flow is primarily controlled by the sphincter and not by the surrounding smooth muscle of the
duodenum. Relaxation of the sphincter occurs with CCK stimulation leading to diminished amplitude of phasic contractions
and reduced basal pressure, allowing increased passive flow of bile into the duodenum. Parasympathetic stimulation also
causes intermittent relaxation of the sphincter, and sympathetic splanchnic stimulation causes increased pressure.
Abnormalities of the sphincter of Oddi may cause symptoms which are referable to the biliary tree or pancreas. Manometry
of the sphincter of Oddi may be performed at the time of ERCP to characterize its basal pressure, the amplitude and
frequency of contractions, and the direction of propagation of contractile waves. Stenosis of the sphincter of Oddi is
characterized by abnormally elevated basal pressure (> 40 mm Hg) whereas dyskinesia is characterized by abnormalities of
other manometric parameters.
140.A 32-year-old woman with symptomatic gallstones wishes to discuss nonsurgical options for her gallstones. Which
of the following statement(s) are true?

a.The best commercially available oral dissolution agent, ursodeoxycholic acid, is associated with a complete
dissolution rate of less than 50%
b.If the gallstones dissolve, there is minimal risk of gallstone recurrence
c.Contact dissolution is applicable regardless of stone type
d.Extracorporial shock wave lithotripsy (ESWL) in combination with oral dissolution agents is an appropriate
technique for most patients and can result in complete stone fragment clearance in over 90% of patients by one year
Answer: a

Prior to the introduction of laparoscopic cholecystectomy, there was tremendous enthusiasm for a number of nonsurgical
techniques for managing gallstones. Dissolution of existing gallstones with pharmacologic agents has been addressed by
several national cooperative studies. Ursodeoxycholic acid is the best, safest, and most effective commerically available
drug currently available. However, it still has a rate of complete dissolution of only 40%. It is also estimated that only 10%
of patients will be suitable candidates for this therapy. Furthermore, gallstone recurrence is a major problem. Actuarial life
table analysis indicates that the risk of gallstone recurrence in patients who have undergone dissolution of gallstones with
oral bile acid therapy is 50% by five years.
The addition of extracorporial shock wave lithotripsy increases the efficiency of gallstone clearance and in selected patients
complete fragment clearance can be obtained in over 90% of patients by one year. However, optimal results can be obtained
only by setting relatively strict criteria for inclusion. In applying such criteria, less than 20% of patients in the United States
would be considered eligible for ESWL. Similar problems with gallstone recurrence have been observed with this
technique. Finally, contact dissolution primarily with the ether solvent methyl tert-butyl ether (MTBE) is extremely
effective in dissolving cholesterol gallstones. This technique, however, will work only in stones which are composed of
cholesterol and the patient must have a demonstrably patent cystic duct before considering this treatment. As with oral
dissolution and lithotripsy, gallstone recurrence will remain a problem with this technique.

141.A 48-year-old woman presents with several hours of acute right upper quadrant pain, low grade fever, and nausea
and vomiting. Which of the following statement(s) is/are true concerning her diagnosis and management?

a.A mild elevation of her bilirubin (< 3 mg/dl) would strongly suggest a common bile duct stone
b.A positive bile culture can be expected in virtually 100% of patients with this scenario
c.Laparoscopic cholecystectomy is clearly contraindicated
d.Appropriate antibiotic coverage should include coverage for gram-negative aerobes
Answer: d

Acute cholecystitis occurs in about 10% to 20% of patients with symptomatic gallstones. Although the clinical
manifestations of biliary colic and acute cholecystitis may overlap and clinical distinction is often difficult, persistence of
pain beyond a matter of hours and fever would strongly suggest acute cholecystitis. The primary events in the development
of acute cholecystitis are chemical in nature with bacterial infection playing a minor role in the genesis of the disease. In
normal healthy subjects without gallstones, incidence of positive bile cultures is essentially zero. In contrast, between 30
and 70% of patients with the clinical diagnosis of acute cholecystitis will have positive bile cultures. The incidence of
positive bile cultures who undergo cholecystectomy increases significantly with age.
Septic complications continue to be a significant source of morbidity after cholecystectomy for acute cholecystitis. These
septic complications can best be prevented by the judicious use of appropriate antimicrobial agents. The goal of
antimicrobial therapy should be establishment of adequate serum and tissue levels of antibiotic rather than selection of an
antibiotic that is excreted into the bile. Given the bacteriology that is typical in patients with uncomplicated cholecystitis, an
appropriate antibiotic regimen should provide for adequate coverage of gram-negative aerobes. Although technically more
difficult, laparoscopic cholecystectomy can be completed safely in the majority of patients with acute cholecystitis.
Significant experience and good judgment, however, is essential in insuring optimal results.
Laboratory data are often nonspecific with acute cholecystitis. Mild jaundice may be present in up to 20% of patients and is
typically due to inflammation as opposed to bile duct obstruction secondary to stones.

142.Laparoscopic cholecystectomy has become the procedure of choice for the management of symptomatic gallstones.
Which of the following statement(s) is/are true concerning laparoscopic cholecystectomy?
a.Injury to biliary tree occurs more frequently with laparoscopic cholecystectomy than open cholecystectomy
b.Laparoscopic cholecystectomy is contraindicated in patients with acute cholecystitis
c.Bile duct injuries are more likely to occur during the surgeon’s early operative experience with the procedure
d.Previous upper abdominal surgery is an absolute contraindication to laparoscopic cholecystectomy
Answer: a, c

Laparoscopic cholecystectomy was first reported in 1988 and in recent years has supplanted open cholecystectomy as the
treatment of choice in the management of calculous biliary tract disease. As experience with the technology increases,
recommendations regarding contraindications for laparoscopic cholecystectomy have evolved. Absolute contraindications
include the inability to perform laparoscopic cholecystectomy because of inadequate training or equipment, poor candidate
for general anesthesia, uncontrolled coagulopathy, peritonitis, or suspected gallbladder carcinoma. Factors previously
considered relative contraindications have been identified but with appropriate skill and judgment do not preclude the
laparoscopic technique. These include acute cholecystitis, morbid obesity, previous upper abdominal surgery, cirrhosis,
portal hypertension, and pregnancy. Laparoscopic cholecystectomy for acute cholecystitis is clearly more difficult and while
the conversion rate of elective laparoscopic cholecystectomy to an open procedure is less than 5% in the setting of acute
cholecystitis, the rate is approximately 20–30%.
The incidence of bile duct injury in large surveys in this country and abroad suggest that the incidence of bile duct injury
with open cholecystectomy is less than 0.2%. Initially much higher, (approaching 1%), several series of laparoscopic
cholecystectomy have documented that injuries to the bile ducts currently occur at the rate of 0.2–0.6% of cases. There is a
definite learning curve associated with performing laparoscopic cholecystectomy with most bile duct injuries likely to occur
during a surgeon’s early operative experience with this procedure.

143.Which of the following statement(s) is/are true concerning gallstone ileus?

a.The diagnosis may be suggested by plain abdominal radiograph


b.Primary surgical management consists of relief of obstruction and cholecystectomy
c.Gallstone ileus accounts for less than 5% of all causes of intestinal obstruction
d.Typical patients are elderly with long-standing gallstone disease
Answer: a, c, d

Cholecystoenteric fistulas are the most common form of gallbladder perforation accounting for 40% of all cases. Depending
on the size of the fistulous communication, a gallstone may pass through this tract. If the stone is large enough (> 2 cm), it
may become lodged in a portion of the gastrointestinal tract and cause mechanical small bowel obstruction. This condition,
called gallstone ileus, is relatively rare and accounts for fewer than 5% of cases of intestinal obstruction. The diagnosis of
gallstone ileus may be suggested by the presence of intrahepatic biliary air in the setting of a small bowel obstruction seen
on abdominal radiography. Patients with gallstone ileus are best managed as if they had a mechanical small bowel
obstruction with aggressive fluid resuscitation, broad spectrum antibiotics, and early laparotomy. The primary goals at
laparotomy are correction of the obstruction and removal of the offending gallstone. Since many of the patients are elderly
and ill, cholecystectomy and takedown of the biliary enteric fistula may not be appropriate. Enterolithotomy alone, without
cholecystectomy, has an associated mortality of 5% in contrast to 15% mortality rate in patients who undergo both
procedures at the same time.

144.The pathogenesis of cholesterol gallstones is multifactorial. A number of key processes, however, appear to interact
closely in the formation of cholesterol gallstones. These include:

a.Cholesterol supersaturation
b.Gallbladder stasis
c.Accelerated nucleation
d.High bilirubin concentration
Answer: a, b, c

The development of gallstones is a complex multifactorial process. Although a number of factors may contribute to
gallstone formation, a number of steps are considered critical in the formation of gallstones. These include cholesterol
supersaturation whereby the amount of cholesterol present is in excess of the ability of the micelles and vesicles to transport
cholesterol in a soluble state. In addition, the process by which cholesterol crystals form and agglomerate is referred to as
nucleation. It is likely that the process of nucleation is altered by a number of biliary proteins or other compounds which
may promote or inhibit the nucleation process. Gallbladder stasis provides the proper environment for gallbladder crystal
precipitation and stone formation. The stagnant pool of bile within the gallbladder may facilitate both cholesterol
supersaturation, the nucleation process, as well as altering the gallbladder’s absorptive or secretory functions. Although
bilirubin has been suggested to play a small role in cholesterol gallstone formation, perhaps in either nucleation or
solubilization of cholesterol, excessive production of bilirubin as seen in hemolytic disorders is associated primarily with
the formation of pigment gallstones.

145.Which of the following statement(s). is/are true concerning the diagnosis of biliary tract disease?

a.Nonvisualization of the gallbladder on oral cholecystogram is diagnostic of biliary calculous disease


b.Ultrasonography has a diagnostic accuracy and sensitivity for cholelithiasis in excess of 95%
c.Ultrasonography is the preferred test to distinguish chronic from acute cholecystitis
d.Hepatobiliary scintigraphy is primarily indicated to confirm the clinical diagnosis of acute cholecystitis
Answer: b, d

A number of radiographic studies can be used to diagnose cholelithiasis. Visualization of gallstones on plain abdominal
radiographs is possible in about 20% of patients whose stones are grossly calcified. Traditionally, oral cholecystogram has
been the gold standard for the diagnostic evaluation of patients with suspected gallstones. Although the accuracy for oral
cholecystography has been reported to be as high as 95%, several conditions preclude satisfactory examination, including
acute illness; poor patient compliance; inability to absorb the tablets as the result of emesis, malabsorption, or diarrhea; and
jaundice or hepatic dysfunction. Abdominal ultrasonography has therefore become the preferred test for evaluation of
patients with suspected gallstones. Most large series suggest that diagnostic accuracy and sensitivity for cholelithiasis
exceeds 95%. In addition to detecting gallstones, information is provided concerning the size and shape of the gallbladder,
gallbladder wall thickness, presence of pericholcystic fluid, or a sonographic Murphy’s sign. The latter has been suggested
to have an 85% accuracy rate in patients with acute cholecystitis. Nonetheless, while abdominal sonography is most helpful
in identifying the presence or absence of gallstones, it is of limited use in distinguishing chronic from acute cholecystitis.
Hepatobiliary scintigraphy provides information on the patency of the bile ducts, including the cystic duct. Recognition that
cystic duct obstruction is the sine qua non of acute cholecystitis, failure of the gallbladder to visualize during hepatobiliary
scintigraphy is highly suggestive of acute cholecystitis. Although a number of false positive exams can occur,
nonvisualization of the gallbladder during hepatobiliary scanning in the appropriate clinical setting is highly diagnostic of
acute cholecystitis.

146.Which of the following statement(s) is/are true concerning the solubilization of cholesterol in bile?

a.Cholesterol is highly soluble in both serum and bile


b.Mixed micelles are the primary transport mechanism for biliary cholesterol
c.Most cholesterol found in bile is the result of excretion from serum
d.Biliary vesicles are composed primarily of biliary phospholipid
Answer: d

Bile is secreted by the liver and is composed primarily of water, electrolytes, and organic solutes. Bile salts, cholesterol, and
phospholipids are the main solutes found within bile and account for about 80% of the dry weight of bile. Most of the
cholesterol found in bile is synthesized de novo in the liver. Cholesterol is an organic molecule that is virtually insoluble in
an aqueous medium such as bile. Therefore, mechanisms for maintaining cholesterol in solution have been evolved. For
decades, the mixed micelle which is composed of the amphiphatic bile salts and phospholipid was considered the primary
carrier of bile. More recently, it has been demonstrated that up to 70% of the total amount of cholesterol normally found in
gallbladder bile is transported and solubilized in the vesicular form. Bile vesicles are composed primarily of phospholipid of
which in the human, lecithin accounts of 90% of the phospholipid content.

147.Appropriate options for management of common bile duct stones identified at laparoscopic cholecystectomy
include:

a.Conversion to open cholecystectomy and common duct exploration


b.Transcystic duct dilatation and exploration
c.Laparoscopic choledochotomy
d.Complete the laparoscopic cholecystectomy with postoperative ERCP and stone removal
Answer: a, b, d

Common bile duct stones found unexpectedly during laparoscopic procedures presents the surgeon with a different set of
choices from that of stones found during an open procedure where one would invariably proceed to common duct
exploration. The practice of converting to an open procedure for the purposes of common duct exploration is acceptable and
should be considered based on the surgeon’s experience and expertise and other options available. Transcystic duct
dilatation and exploration with stone removal is being performed in increasing numbers and appears to be safe and effective.
Although experience with laparoscopic choledochotomy and common duct exploration has been reported with limited
experience, this technique is probably not appropriate for the average surgeon. Many surgeons, depending on their clinical
situation, may opt to complete the laparoscopic removal of the gallbladder and proceed to postoperative management of the
stone using endoscopic technique.

148.Risk factors associated with development of gallstones include:

a.Increasing age
b.Obesity
c.Rapid weight loss
d.Cirrhosis
e.Diabetes mellitus
Answer: a, b, c, d, e

Incidence of gallstones varies widely throughout the world} In the United States, about 10% of the population has
cholelithiasis. Although the physiologic explanation for increasing incidence of gallstone disease in the elderly is unclear,
epidemiologic studies have demonstrated a relationship between increasing age and the prevalence of cholelithiasis in both
males and females. Gallstone disease is a much more common in women than men, however with the higher incidence of
cholelithiasis in women persisting until the seventh or eighth decade of life, at which time the incidence approaches 20% in
both men and women. Clinical evidence suggests a two to three-fold increase in the incidence of cholelithiasis among
morbidly obese patients compared to age matched normal weight subjects. Interestingly, rapid weight loss has also been
associated with secretion of cholesterol saturated bile and increased incidence of gallstones. Patients with diabetes mellitus
have a two-fold increase of gallstone disease when compared to non-diabetics. Although the influence of alcohol on
gallstone formation remains poorly defined, autopsy studies suggests that the incidence of cholelithiasis is significantly
increased in patients with alcoholic cirrhosis.

149.Which of the following statement(s) is/are true concerning a 35-year-old woman found to have asymptomatic
gallstones?

a.The patient should undergo attempts at medical dissolution


b.Cholecystectomy should only be performed if the laparoscopic technique is an available option
c.The patient has less than 10% chance of developing significant symptoms over the next five years
d.The risk of gallbladder cancer with stones detected at such a young age, warrants cholecystectomy
Answer: c

The optimal management of individual patients with asymptomatic gallstones has been debated for years. Most evidence,
although still somewhat controversial, would indicate that the natural history of asymptomatic gallstones is benign and that
early or prophylactic cholecystectomy, either laparoscopic or otherwise, is rarely indicated. It is suggested that less than
10% of patients with asymptomatic gallstones will develop significant symptoms over a five year period. Similarly, medical
dissolution with oral agents, extracorporial biliary lithotripsy, or contact dissolution, is not indicated in these patients. The
risk of gallbladder cancer is so low as to not warrant cholecystectomy in asymptomatic patients.

150.In which of the following clinical situations is pigment rather than cholesterol gallstones a more common problem?

a.Alcoholic cirrhosis
b.Morbid obesity
c.Diabetes mellitus
d.Hereditary spherocytosis
e.Biliary tract infection
Answer: a, d, e

World wide, pigment stones are the most common type of calculus found in the gallbladder. In the United States, although
cholesterol calculi predominate, pigment stones constitute about 30% of all gallbladder stones. Pigment stones are
characterized by a high concentration of bilirubin and a low cholesterol content. The high incidence of pigment gallstones in
patients with hemolytic disorders probably results from the excessive loads of bilirubin being presented to the liver for
excretion. Infection has been thought to be a key factor in the pathogenesis of pigment gallstones in that unconjugated
bilirubin, produced by bacterial deconjugation, is insoluble in water, and combines with calcium in bile to produce a
calcium bilirubin matrix. Specific alterations in bile acid metabolism have been proposed to explain the increased incidence
of pigment gallstones found in cirrhotics. Gallstones associated with morbid obesity and rapid weight loss appear to be
primarily due to the increased hepatic secretion of cholesterol leading to a cholithogenic state.

151.The following statement(s) is/are true concerning the mode of spread of gallbladder cancer.

a.The most common mode of spread for gallbladder cancer is to the liver by hematogenous spread
b.Lymphatic drainage of the gallbladder can extend from the cystic duct lymph node to periportal lymph nodes and
the celiac and superior mesenteric lymph nodes
c.Gallbladder cancers rarely extend past the cystic duct
d.Direct extension to the liver, particularly liver segments IV and V, is the most common mode of spread for
gallbladder cancer
e.Most patients with gallbladder cancer present with a very limited extent of disease
Answer: b, d

The mode of spread of gallbladder cancer can be predicted by the lymphatic and venous drainage of the gallbladder. Venous
drainage is into the venules that drain directly into the adjacent liver. The most common spread of gallbladder cancer is
through direct extension into the liver, particularly liver segments IV and V. The lymphatic drainage of the gallbladder is to
the cystic duct lymph node, to periportal lymph nodes, and then to celiac and superior mesenteric lymph nodes. These
tumors can also spread into and around the cystic duct and may extend into the common bile duct, causing biliary
obstruction. Besides direct extension and lymphatic spread of the tumor, distant metastases is possible but is less commonly
seen. A staging system has been developed which takes into consideration lymphatic and venous drainage of the gallbladder.
The most advanced stage, stage V, includes distant spread, either into the liver or adjacent organs, which unfortunately is the
case at the time of presentation for most patients.

152.A 55-year-old woman undergoes cholecystectomy for symptomatic gallstones. The final pathology returns one
week after the operation showing gallbladder carcinoma. Which of the following statement(s) is/are true
concerning this patient’s prognosis and management?

a.If the gallbladder carcinoma is limited to the mucosa and/or submucosa, cholecystectomy alone is adequate
treatment
b.If the carcinoma involves deeper layers of the gallbladder wall, the prognosis will be significantly worse
c.If reoperation is performed, the chance of finding residual disease will be low
d.If a full thickness tumor had been recognized at the time of cholecystectomy, an "extended cholecystectomy" may
have been appropriate
Answer: a, b, d

Gallbladder cancers with the best prognosis are those that are found incidentally at the time of cholecystectomy for
symptomatic gallstone disease. This point emphasizes the importance of opening all gallbladders at the time of
cholecystectomy so that any suspicious lesions can be immediately examined histologically. When gallbladder cancer is
limited to mucosa and submucosa, cholecystectomy is adequate treatment and has a good prognosis, with up to 100% five
year survival. When the cancer involves the deeper layers of the gallbladder wall, the prognosis is more grim. A 64% five
year survival was reported in patients who had carcinoma confined to the mucosa and submucosa, whereas none of the
patients with cancer involving all layers of the gallbladder wall survived more than 2.5 years. Even though these tumors are
relatively localized at the time of cholecystectomy, cholecystectomy alone was not adequate treatment for long-term
survival. In hopes of improving these survival rates and considering the lymphatic and venous drainage of the gallbladder, it
has been recommended that gallbladder cancers be treated by cholecystectomy with wide resection of the liver around the
gallbladder bed (liver segments IV and V) and regional lymphadenectomy. This procedure has been termed an "extended
cholecystectomy." Although this treatment has prolonged median survival, an overall improvement in five year survival has
not been observed. When a gallbladder cancer has been recognized incidentally at the time of pathology examination after
routine cholecystectomy, reexploration will likely show evidence of residual disease in the lymph nodes or adjacent liver
and therefore resection of this residual disease may improve survival. More radical excision including hepatic resection and
bile duct resection and sometimes even pancreaticoduodenectomy has been reported. However, the associated increased
morbidity and mortality of these operations has not resulted in an improvement in survival.

153.Gallbladder cancer accounts for 3% to 4% of all gastrointestinal tumors. Which of the following statement(s) is/are
true concerning the incidence of gallbladder cancer?

a.Gallbladder cancer is much more frequent in men


b.Southwest Native Americans, Alaskans, Mexicans, and Hispanics have a greater incidence of gallbladder cancer
than the general population
c.The majority of patients with gallbladder cancer have gallstones
d.The association of gallstones with gallbladder cancer is greater in patients with multiple small stones
Answer: b, c

Gallbladder cancer accounts for 3% to 4% of all gastrointestinal tract tumors and about 2% of all biliary procedures are
done for gallbladder cancers. Gallbladder cancer is much more frequent in women, with a female/male ratio of 3:1. The
disease is most commonly seen in elderly women with a mean age of 65 at diagnosis. Southwest Native Americans,
Alaskans, Mexicans, and Hispanics living in the United States have an estimated five to six times greater incidence of
gallbladder cancer than the general population. On the other hand, gallbladder cancer is much less common in blacks. The
association of gallstones with gallbladder cancer is well known. Seventy to ninety percent of all patients with gallbladder
cancers have gallstones, and about 0.4% of all patients with gallstones have gallbladder cancer. The association of gallstones
with cancer may be related to gallstone size; larger stones have a greater cancer risk. There is a 10-fold increase in the
incidence of gallbladder cancer in patients who have gallstones that are larger than 3 cm in diameter.

154.The following statement(s) is/are true concerning the prognosis for gallbladder cancer.

a.Average survival is in the range of one year


b.Five year survival rates approach 50%
c.The combination of postoperative adjuvant radiation and chemotherapy have been associated with overall
improved survival
d.For most patients, the goal of treatment is palliation
Answer: d

The prognosis for gallbladder cancer remains poor, with an average survival in the range of six months. Less than 5% of
patients survive five years because 90% of gallbladder cancer patients present with stage V disease. For stage V disease, the
goal of treatment is palliation. If these patients present with obstructive jaundice, a major goal of treatment is relief of
jaundice and its attendant symptoms such as pruritus and cholangitis. Although radiation and chemotherapeutic regimens
have been tried, none have been associated with a good response.

155.Which of the following statement(s) is/are true concerning gallbladder polyps?

a.Sonographic findings of a gallbladder polyp include a filling defect which does not move with change in position
b.Benign gallbladder polyps can cause symptoms similar to those caused by gallstones
c.Ultrasonography can readily distinguish benign from malignant gallbladder lesions
d.An asymptomatic gallbladder polyp 1 cm in size can usually be followed with serial ultrasounds and does not
require cholecystectomy
Answer: a, b

Benign gallbladder polyps cause symptoms similar to those caused by cholelithiasis. Biliary symptoms including right
upper quadrant pain and discomfort, fatty food intolerance, nausea, vomiting, and an increase in flatulence are common
complaints in patients with symptomatic benign gallbladder polyps. Gallbladder tumors, when diagnosed preoperatively,
may be visualized by ultrasound, oral cholescintography, or less commonly, computed tomography. A filling defect that does
not move with changes in position is probably a benign or malignant gallbladder polyp. Symptomatic patients who have
lesions in the gallbladder should have a cholecystectomy. Since neither ultrasound nor cholecystography can distinguish
benign from malignant lesions, all gallbladders that contain polypoid lesions should be removed.

156.The following statement(s) is/are true concerning the surgical therapy of cholangiocarcinomas.

a.A resectable distal bile duct carcinoma is best treated by a Whipple procedure
b.Klatskin tumors may require hepatic resection in attempt for potential cure
c.Regardless of the surgical resection for proximal biliary tumors, stenting of the biliary anastomosis is important
d.Complete hepatic resection with hepatic transplantation has been associated with overall good results
Answer: a, b, c

The surgical treatment of a cholangiocarcinoma is dependent upon its location. For lesions of the middle third of the bile
duct, resection of the bile duct tumor with reanastomosis is the procedure of choice when possible. For distal lesions, the
Whipple procedure (pancreaticoduodenectomy) is often necessary. Overall, both middle and lower third lesions have a
better prognosis than tumors in the hilum. The prognosis for patients with hilar bile duct cancer (Klatskin tumors) is
extremely poor with mortality rates of 80% to 90% in five years. Most patients are unresectable at the time of diagnosis.
Unless contraindicated for other reasons, however, surgical exploration should be performed in all patients whose tumors
are potentially resectable. Hepatic lobectomy is indicated for potential cure if the bile duct lesion extends into the hepatic
parenchyma. Regardless of the surgical therapy, stenting of the biliary anastomosis is important since postoperative
strictures or recurrent tumors are common and long-standing stents allow for cholangiographic followup and for dilatation
should strictures arise. These stents have also been used for placement of iridium seeds used for local irradiation. More
aggressive surgical therapy including complete hepatic resection followed by hepatic transplantation have been performed.
However, the results have been associated with a very high rate of recurrence in the post-transplantation period and
therefore at the present time this treatment is not recommended.

157.The following statement(s) is/are true concerning carcinoma of the bile ducts.

a.Carcinoma of the bile ducts is more common than gallbladder carcinoma


b.Similar to gallbladder cancer, bile duct cancer is more common in females
c.Unlike gallbladder cancer, there is no association of bile duct cancer and gallstones
d.Ulcerative colitis is associated with an increased incidence of bile duct cancer
e.Choledochal cysts may be associated with the development of bile duct cancer
Answer: d, e

Cancer of the bile ducts is even less common than gallbladder carcinoma and is seen in 0.01% to 0.46% of all autopsies.
Unlike gallbladder cancer, which is more frequent in females, males have a higher incidence of bile duct cancer. The
average age range of diagnosis between is 50 and 70 years. Similar to gallbladder cancer, there is an association between
bile duct cancer and gallstones. This association is, however, not as direct as for gallbladder cancer with only 25% to 57%
of patients with bile duct cancer having gallstones. Biliary tract infection is associated with these tumors; for example,
patients with Clonorchis sinensis infection and chronic typhoid carriers have a higher incidence of bile duct cancer than the
general population. Choledochal cysts are also associated with bile duct cancer. Patients with ulcerative colitis have a
marked increase incidence of bile duct tumors which tend to follow a more aggressive course.

158.Which of the following statement(s) is/are true concerning the association of gallstones with gallbladder cancer?

a.The association of gallbladder cancer with gallstones is such that young patients with asymptomatic gallstones
should undergo routine cholecystectomy
b.Calcification of the wall of the gallbladder is associated with an increased incidence of gallbladder cancer
c.Larger gallstones are associated with a greater risk of gallbladder cancer than smaller gallstones
d.The 20 year risk of developing gallbladder carcinoma in patients in the general population with gallstones is
approximately 1%
Answer: b, c

The association of gallstones with gallbladder cancer is well known. Seventy to ninety percent of patients with gallbladder
cancer have gallstones. The association of gallstones with cancer may be related to gallstone size, with larger stones having
a greater cancer risk. The 20-year risk of developing gallbladder carcinoma in patients with gallstones has been estimated to
be in the range of 0.13% for the general population. It is evident that this small risk of developing gallbladder cancer in
patients with gallstones, even over the long-term, does not warrant routine cholecystectomy in all patients with gallstones.
Calcification of the wall of the gallbladder, the so-called porcelain gallbladder, is associated with a 25% to 60% incidence of
gallbladder cancer. In the presence of a porcelain gallbladder, a cholecystectomy should be performed unless
contraindicated for other reasons.

159.A 65-year-old man presents with obstructive jaundice. The patient’s workup begins with a CT scan. Which of the
following statement(s) is/are true concerning his diagnosis?

a.A CT scan demonstrating intrahepatic biliary obstruction with a decompressed gallbladder and a nondilated
extrahepatic biliary tree will be consistent with a Klatskin tumor
b.The presence of biliary obstruction seen on the CT scan requires further evaluation with invasive cholangiography
either percutaneously or endoscopically
c.Percutaneous transhepatic cholangiography would be the preferred technique for a suspected proximal
cholangiocarcinoma in that it will better visualize the proximal extent of the tumor in the biliary tree
d.The placement of a transhepatic biliary catheter can prove useful in surgical management of proximal bile duct
cancers
e.There is little role for angiography in the evaluation of a patient with suspected cholangiocarcinoma
Answer: a, b, c, d

Jaundice is the most frequent presenting symptom in patients with cholangiocarcinoma, occurring in 90% of patients. The
diagnosis begins with abdominal imaging to detect ductal dilatation. A CT appearance of a proximal cholangiocarcinoma or
a Klatskin tumor will show a dilated intrahepatic biliary tree with a decompressed gallbladder and normal caliber distal bile
duct. When biliary obstruction is present, further visualization of the biliary tree is required either through percutaneous
transhepatic cholangiography (PTC) or endoscopic retrograde cholangiopancreatography (ERCP). PTC is preferred for
more proximal lesions because ERCP may fail to adequately visualize the proximal portion of the biliary tree. For lower bile
duct lesions, ERCP may be the preferred route. The management of hilar tumors can often be facilitated by the placement of
transhepatic percutaneous catheters which can facilitate both resection, reconstruction, and may serve as a route for
administration of local radiation therapy. These tumors are often found to be unresectable because of direct vascular
invasion into the portal vein or hepatic arteries. Therefore, selective celiac angiography can be helpful preoperatively to
determine if these major adjacent vascular structures are involved with the tumor.

160.Benign gallbladder and bile duct tumors are extremely rare. Which of the following statement(s) is/are true
concerning benign biliary tumors?

a.Cholesterol polyps are due to foamy macrophages filled with cholesterol


b.Inflammatory polyps and cholesterol polyps are not considered to be premalignant
c.Gallbladder adenomas, like adenomas in other gastrointestinal organs, are considered premalignant
d.Most gallbladder adenomas are associated with gallstones
e.An adenomyoma is a mucosal lesion of the gallbladder not considered to be premalignant
Answer: a, b, c

Benign gallbladder and bile duct tumors are extremely rare and can include pseudotumors or hyperplastic conditions which
are thought to result from inflammatory states, or adenomas, which are most likely premalignant lesions. Cholesterolosis, or
a strawberry gallbladder, is manifested by yellow spots visible on the surface of the gallbladder mucosa. This proliferation
of foamy macrophages filled with cholesterol in the lamina propria may also result in the formation of polyps called
cholesterol polyps. These polyps are thought to result from disturbance in cholesterol metabolism. Other pseudotumors of
the gallbladder are inflammatory polyps which are composed of vascular connective tissue stalk with a single layer
columnar epithelial cells and have a chronic inflammatory infiltrate. These lesions are not considered to be premalignant
and are felt to be the result of chronic inflammation. Adenomas with hyperplasia of the epithelial layer of the gallbladder,
like adenomas in other gastrointestinal organs can be sessile or papillary. Carcinoma in situ has been reported in these
lesions, and they are thought to be premalignant. The cause of adenomas of the gallbladder is unknown; but most are not
associated with the presence of gallstones. An adenomyoma of the gallbladder is a rare intramural mass or nodule. This
lesion is characterized by a proliferation of the mucosal epithelium and hypertrophy of the mucosal layers of the gallbladder.
Histologically in addition to muscular hypertrophy, invagination of the epithelial mucosa between muscle layers is
evidenced. These lesions are also not considered premalignant.
161.Which of the following statement(s) concerning bile duct strictures due to chronic pancreatitis is/are true?

a.Most patients present with progressive jaundice


b.Strictures are classically long and tapered involving the entire intrapancreatic bile duct
c.Patients may be asymptomatic and diagnosed only by persistent elevation of serum alkaline phosphatase
d.An excellent option for surgical management is choledochoduodenostomy
Answer: b, c, d

The clinical presentation of patients with common bile duct strictures secondary to chronic pancreatitis is variable. A large
number of patients are asymptomatic with a diagnosis of bile duct strictures suggested only by abnormal liver function tests.
The serum alkaline phosphatase appears to be the most sensitive laboratory finding and is elevated in over 80% of patients.
Although in most cases, transient jaundice may occur, progressive jaundice is rare. Cholangiography will demonstrate a
classic long smooth gradual tapering of the common bile duct throughout its entire intrapancreatic segment. Biliary
reconstruction is the appropriate management for most patients. Choledochoduodenostomy is preferred by many surgeons
because it does not divert bile from the duodenum, is technically easier to perform, and leaves the jejunum intact for any
associated procedures that may be required for decompression of the obstructed gastrointestinal tract or pancreatic duct.

162.If a bile duct injury is suspected at laparoscopic cholecystectomy, appropriate management includes which of the
following?

a.Conversion to open cholecystectomy and intraoperative cholangiography


b.Small ducts (< 3 mm) demonstrated by cholangiography to drain a single liver segment can be ligated
c.If the injured segment is greater than 1 cm. in length an end-to-end ductal anastomosis is the procedure of choice
d.Postoperative external drainage should be avoided
Answer: a, b

In many cases, proper initial management of a bile duct injury recognized at the time of cholecystectomy can avoid the
development of a bile duct stricture. Unfortunately recognition of a bile duct injury is uncommon during either open or
laparoscopic cholecystectomy. It must be emphasized that should bile leakage be noted or if "a typical" anatomy is
encountered during laparoscopic cholecystectomy, early conversion to an open technique and prompt cholangiography is
imperative. If a segment of accessory duct less than 3 mm has been injured, and cholangiography demonstrates segmental or
sub-segmental drainage of the injured ductal system, simple ligation of the injured duct is indicated. If the injured duct is 4
mm or larger, however, it is likely to drain multiple hepatic segments or the entire right or left lobe and thus requires
operative repair. If the injured segment of the bile duct is short (< 1 cm) and the two ends can be opposed without tension,
an end-to-end anastomosis can be performed with placement of a T-tube through a separate choledochotomy either above or
below the anastomosis. For proximal injuries, or if the injured segment of bile duct is greater than 1 cm in length, an end-to-
end bile duct anastomosis should be avoided because of the excessive tension that usually exists in these situations. The use
of Roux-en-Y jejunal limb is preferable for creation of the anastomosis. Regardless of the type of anastomosis, all repairs at
the time of initial operation should involve some form of external drainage either with a T-tube or intraoperatively-placed
transanastomotic stent.

163.Which of the following statement(s) is/are true concerning the incidence of bile duct injury following
cholecystectomy?

a.Data from the pre-laparoscopic cholecystectomy era would suggest the incidence of bile duct injury during open
cholecystectomy to be 0.1–0.2%
b.The current incidence of bile duct injury during laparoscopic cholecystectomy is greater than 1%
c.The experience of the surgeon performing laparoscopic cholecystectomy can be correlated with the incidence of
bile duct injury
d.Intraoperative cholangiography during laparoscopic cholecystectomy will prevent bile duct injury in virtually all
cases
Answer: a, c

A number of large surveys encompassing thousands of patients undergoing open cholecystectomy would suggest the
incidence of bile duct injury to be 0.1–0.2%. Although a number of early individual series of laparoscopic cholecystectomy
reported bile duct injuries in 1% of patients, as larger series have been reported and surveys including thousands of patients
have appeared, the true incidence would appear to be 0.3–0.6%. A number of factors are associated with bile duct injury
during laparoscopic cholecystectomy including the experience of the surgeon. This reflects the steep learning curve with
this procedure. Although strongly debated, there is no evidence that intraoperative cholangiography prevents bile duct injury
during laparoscopic cholecystectomy. The use of intraoperative cholangiography may however detect the injury early in the
course of the procedure and thus minimize the extent of injury.

164.Primary sclerosing cholangitis has a number of treatment options—both medical and surgical. Which of the
following statement(s) is/are true?

a.A number of immunosuppressive oral agents can provide specific effective treatment for primary sclerosing
cholangitis
b.Biliary reconstruction with long-term transanastomotic stents can be useful in selected patients with focal strictures
at the hepatic duct bifurcation
c.Biliary reconstruction should be reserved only for patients with established biliary cirrhosis
d.Hepatic transplantation for primary sclerosing cholangitis can be associated with survival rates similar to other
indications for transplantation
Answer: b, d

There is no known specific, effective medical therapy for primary sclerosing cholangitis. Although encouraging results from
a prospective, randomized, placebo-controlled trial suggests that ursodeoxycholic acid significantly improves serum liver
function tests and clinical symptoms. Because of the lack of effective medical therapy, an aggressive surgical approach is
indicated for most symptomatic patients with primary sclerosing cholangitis. One surgical approach, in patients with a
predominant stricture at the hepatic duct bifurcation, is resection of the bifurcation and long-term transhepatic stenting with
silastic stents. Results in patients without established cirrhosis are excellent. However, in those patients with secondary
biliary cirrhosis present before surgery, perioperative morbidity and mortality have been high and long-term results poor.
Patients with established secondary biliary cirrhosis should be referred for hepatic transplantation. Recent reviews of the
experience with hepatic transplantation for primary sclerosing cholangitis would suggest survival to be similar to those
reported for hepatic transplantation for patients with any diagnosis.

165.The following statement(s) regarding the elective repair of a bile duct stricture is/are true:

a.A transanastomotic stent is necessary for a successful result


b.Long-term stenting for approximately one year is necessary for an anastomosis performed at the distal common
hepatic duct
c.A Roux-en-Y hepaticojejunostomy provides the best route for restoring biliary-enteric continuity
d.Preoperatively-placed biliary catheters facilitate dissection and identification of the stricture and are useful in
placement of transanastomotic stents when employed
Answer: c, d

Several principles are associated with successful repair of a biliary stricture. Although many surgeons favor the use of
transanastomotic stents, a number of series have reported successful results without the use of such stents. The length of
stenting depends upon the location of the stricture. If the injury involves the common bile duct or common hepatic duct at
least 2 cm distal to the hepatic duct bifurcation, and adequate proximal bile duct mucosa can be defined, the use of long-
term biliary stents is not necessary. In these situations transanastomotic stenting for 4–6 weeks postoperatively is adequate.
When adequate proximal bile duct is not available for a good mucosa-to-mucosa anastomosis, long-term stenting of the
biliary-enteric anastomosis with silastic transanastomotic stents for at least a year is recommended. For established
strictures, simple excision and end-to-end anastomosis or repair of the damaged duct can rarely be accomplished because of
the invariable loss of duct length as a result of fibrosis associated with injury. Thus in almost all cases, hepaticojejunostomy
constructed to a Roux-en-Y limb of jejunum is the preferred procedure. The use of preoperatively-placed transhepatic
biliary catheters can aid in the dissection and identification of the biliary tree especially in patients with prior attempts at
repair where scarring and fibrosis may be significant. In addition, the biliary catheters can assist in the placement of long-
term transanastomotic stents.

166.The vast majority of benign bile duct strictures occur following operations in or near the right upper quadrant.
Other causes of benign bile duct strictures include:
a.Chronic pancreatitis
b.Ulcerative colitis
c.Primary sclerosing cholangitis
d.Intrahepatic arterial infusion of 5-fluorouracil
Answer: a, c, d

Most bile duct strictures are postoperative in nature with over 80% occurring after injury to the bile duct during
cholecystectomy. A number of inflammatory conditions can also result in strictures of the biliary tree. The chronic
inflammation and fibrosis associated with chronic pancreatitis can result in a stricture of the intrapancreatic bile duct.
Primary sclerosing cholangitis is an idiopathic disease believed to be autoimmune in nature which is characterized by
intrahepatic and extrahepatic inflammatory strictures of the biliary tree. Although primary sclerosing cholangitis is
frequently associated with ulcerative colitis, this colonic disease has no direct causal relationship with benign bile duct
strictures. A rare cause of benign bile duct strictures in both the intrahepatic and extrahepatic biliary tree has been the use of
intrahepatic arterial infusion of 5-fluorouracil used to treat hepatic metastasis from colorectal carcinoma.

167.A 37-year-old female presents with obstructive jaundice due to a mid-bile duct stricture four months after
laparoscopic cholecystectomy. Which of the following statement(s) are true:

a.Surgical reconstruction is the only option for management of this patient


b.Excellent long-term results can be expected in approximately 80% of patients following surgical biliary
reconstruction
c.One year follow-up after successful repair is satisfactory regardless of the method of management
d.Surgical reconstruction offers a better chance of long-term success than either percutaneous or endoscopic
dilatation
Answer: b, d

Excellent long-term results can be achieved in 70–90% of patients who undergo surgical repair of bile duct strictures. The
definition of satisfactory results in most series requires patients with no symptoms of jaundice or cholangitis. Length of
follow-up is important in analyzing results, however, because recurrent strictures can occur up to 20 years after the initial
procedure. About two-thirds of restrictures are evident within two years and 90% within seven years. Although operative
management of bile duct strictures in most cases can result in excellent results, the nonoperative approaches of percutaneous
or endoscopic dilatation are suitable alternatives in many patients. Although comparisons between techniques are difficult,
two retrospective comparative studies from single institutions have suggested that surgical reconstruction offers a better
chance of long-term success than either percutaneous or endoscopic management.

168.Most patients with postoperative bile duct strictures after cholecystectomy present early after their initial operation.
Patients may present in which of the following manner(s)?

a.Obstructive jaundice
b.An external biliary fistula
c.Progressive accumulation of bile in the peritoneal cavity (bile ascites)
d.Biliary cirrhosis
Answer: a, b, c

Most patients with benign postoperative bile duct strictures present early after their initial operation. Those patients
suspected of having postoperative bile duct stricture within days to one week of an initial operation usually present in one of
two ways. One mode of presentation is the progressive elevation of liver function tests, particularly serum bilirubin and
alkaline phosphatase. These changes can be seen as early as the second or third postoperative day. The second mode of early
presentation is with leakage of bile from the injured bile duct. Bilious drainage from the operatively-placed drains or
through the wound after cholecystectomy is abnormal and represents some form of biliary injury. In those patients without
drains, or in whom the drains have been removed, the bile may leak into the peritoneal cavity as bile ascites or may loculate
as a collection. Patients with markedly delayed diagnosis of bile duct stricture may present with advanced biliary cirrhosis
and other evidence of liver dysfunction.

169.Nonoperative dilatation, performed either endoscopically or percutaneously, can be successfully employed in


which of the following etiologies of bile duct strictures?
a.Postoperative bile duct strictures following a hepaticojejunostomy used for reconstruction during a Whipple
procedure
b.Complete transection of the bile duct during laparoscopic cholecystectomy (the so-called "classic laparoscopic
cholecystectomy injury")
c.Primary sclerosing cholangitis
d.Oriental cholangiohepatitis
Answer: a, c

Nonoperative management of bile duct strictures is an available option at most institutions, however, it has some technical
limitations due to the anatomic situation. In the so-called "classic laparoscopic bile duct injury" however, complete bile duct
transection and discontinuity of the biliary tree eliminates the possibility of nonoperative management. Percutaneous
dilatation of biliary-enteric anastomosis has been shown in a number of series to have a success rate approaching that of
surgical reconstruction. Although limited experience with either percutaneous or endoscopic dilatation in primary sclerosing
cholangitis has been reported, this alternative may provide at least temporary improvement in symptoms and radiologic
appearance. Oriental cholangiohepatitis is an unusual infection of the biliary tree frequently associated with Clonorchis
sinensis and other parasites. Cholangiography will demonstrate multiple strictures of both the intrahepatic and extrahepatic
biliary tree with bile ducts filled with sludge and stones. Surgical management consisting of cholecystectomy and improving
biliary drainage with either a Roux-en-Y choledochojejunostomy or choledochoduodenostomy is necessary in almost all
patients. Access to the biliary tree for postoperative management of intrahepatic stones or sludge should be maintained,
however, with transhepatic biliary stents.

170.The gold standard for evaluation of patients with bile duct strictures is cholangiography. The two routes for
cholangiography are percutaneous transhepatic cholangiography (PTC) or endoscopic retrograde cholangiography
(ERC). Which of the following statement(s) is/are true?

a.PTC is generally more valuable than ERC in defining the proximal biliary tree to be used in reconstruction
b.ERC is technically easier in patients with bile leaks because the biliary tree is usually not dilated
c.Parenteral antibiotics should be administered prior to either procedure to prevent cholangitis
d.Biliary stents can be placed via either technique to control biliary leaks
Answer: a, b, c, d

The gold standard for evaluation of patients with a bile duct stricture is cholangiography. PTC is generally more valuable
than ERC in that it defines the anatomy of the proximal biliary tree that is used in the surgical reconstruction. ERC is often
less useful than PTC because the discontinuity of the extrahepatic biliary tree usually prevents adequate filling of the
proximal biliary tree. However, in patients with biliary fistulas, the proximal biliary tree is often not dilated making PTC
somewhat technically more challenging. Parenteral antibiotics should be administered prior to either procedure to decrease
the risk of cholangitis. Biliary stents used either to temporarily control biliary leaks or to stent a stricture after nonoperative
dilatation can be placed by either the percutaneous or endoscopic route.

171.The management of a suspected bile duct injury depends on a number of factors, most importantly the mode and
timing of presentation. Which of the following statement(s) is/are true concerning a patient presenting with a
suspected bile leak after laparoscopic cholecystectomy?

a. Laparotomy should be performed immediately


b. Cholangiography should be performed to determine the nature of the injury
c. Operatively-placed drains should be removed to allow the fistula to close
d. The patient should be discharged to home to allow the leak to close spontaneously
Answer: b

Patients presenting in the early postoperative period with a biliary leak may be septic with either cholangitis or intra-
abdominal bile collections. Sepsis must be controlled first with broad-spectrum parenteral antibiotics, cholangiography with
percutaneous biliary drainage and percutaneous or operative drainage of biliary leaks. Once sepsis is controlled, there is no
hurry in proceeding with surgical reconstruction of the bile duct stricture. The combination of proximal biliary
decompression and external drainage allows most biliary fistulas to be controlled or even closed. At that time the external
drains may be removed. The patients can then be discharged to home to allow several months to lapse for resolution of the
inflammation in the periportal region and recovery of overall health status.
1. As the functional anatomy of the spleen is divided into red pulp, white pulp, and marginal zone, what function is
incorporated into the anatomy of the cortical zone that relates to infection control?
A. Filtration of red cells, encapsulated bacteria, and other foreign material.
B. Red pulp for formation of red cells.
C. White pulp for its role in formation of granulocytes.
D. Gray areas, so formed because of the production of platelets.
E. Fibrous trabeculae.
Answer: A

DISCUSSION: The cortical networks described by Billroth are lined with B-, T-, and other lymphocyte subpopulations. The
open zones, called by some the cords of Billroth, serve as an anatomic microfilter as well as a functional mechanism for
removing foreign materials. For example, immature red cells with retained nuclei are “pitted” by these cords and removed
from circulation. Pneumococcal and other bacteria (encapsulated and nonencapsulated) can be removed by the spleen. The
filter mechanism of the spleen, combined with this immune capacity, is what is mainly lost in splenectomy that exposes the
asplenic patient to overwhelming postsplenectomy sepsis syndrome (OPSS).

2. During the evolution of the understanding of hematologic diseases, the indications for splenectomy have changed. The
most common indications for splenectomy are, in descending order of frequency:
A. Traumatic injury, immune thrombocytopenia, hypersplenism.
B. Immune thrombocytopenic purpura, traumatic injury, hypersplenism.
C. Hypersplenism, traumatic injury, immune thrombocytopenia.
D. Immune thrombocytopenia, hypersplenism, traumatic injury.
E. None of the above.
Answer: A

DISCUSSION: Over the years, the indications for splenectomy have varied as better understanding and altered therapy for
immune system diseases have occurred. Whereas in the past splenectomy was the available therapy for hypersplenism,
better understanding of the diseases causing hypersplenism (which is a syndrome) has allowed other therapies to emerge.
Splenectomy, therefore, is less often required for the treatment of hypersplenism associated with some types of hematologic
malignancies. Traumatic injury remains the most common indication for splenectomy, although this situation may change as
conservative management of splenic injuries continues to increase. Moreover, immune thrombocytopenia is increasing in
frequency and is definitively cured by splenectomy.

3. Useful methods for detection of splenic injury, in descending order of sensitivity, are:
A. Diagnostic peritoneal lavage.
B. CT.
C. Ultrasonography.
D. Isotope scan.
E. Magnetic resonance imaging (MRI).
Answer: B

DISCUSSION: Diagnosis of splenic injury is most specifically made and quantitated by CT. Diagnostic peritoneal lavage is
useful for detecting intra-abdominal bleeding, but not specific to splenic injury. Isotope scans are useful to a point but do not
delineate the anatomy of the spleen that is injured. Classifications of splenic injury are currently being developed based on
the images provided by CT scans and are evolving into prospective decision trees for operation or conservative
management.

4. The following statements about splenosis are correct:


A. Autotransplantation of splenic tissue is an etiology.
B. May protect against OPSS.
C. May over time be “born again” and regain some immune function.
D. May produce tuftsin and properdin.
E. All of the above.
Answer: E

DISCUSSION: Splenosis or autotransplantation of splenic tissue, which may follow splenic injury, can result in the return
of some types of immune function. The “born again” spleen refers to the detection in blood samples of the opsonin tuftsin
and properdin, which have been observed to occur in the occasional patient at a period of time remote from splenic injury.
The supposition is that as the splenic implants grow and multiply in time, they reach a critical mass whereby opsonins may
be formed and presumably the implants can provide some of the host defense mechanisms that were lost with splenectomy.

5. The following comments about immune thrombocytopenic purpura (ITP) are accurate:
A. Platelet count is low.
B. Circulating antiplatelet factor is present.
C. Antiplatelet factor is immunoglobulin G (IgG) antibody.
D. Purpura is directed against a platelet-associated antigen.
E. May be fatal.
F. All of the above.
Answer: A

DISCUSSION: ITP is a disease characterized by low platelet counts and a bone marrow with proliferative megakaryocytes.
The disease is usually diagnosed by abnormal episodes of bleeding, which may occur from trauma, menses, etc. The
causative event by which sensitized platelet-associated antibodies are developed is probably multiple. The effect of the
platelet-associated antibodies, which is usually IgG, is directed against a platelet-associated antigen and, when coating of
normal platelets occurs, results in the platelets being sequestered from the system by the reticuloendothelial system with
resultant thrombocytopenia.

6. ITP:
A. Is most common in men in their 20s.
B. Is frequently cured in adults by corticosteroid administration.
C. Usually requires splenectomy in children.
D. Is most common in the sixth decade of life.
E. Is in remission in more than 80% of patients with splenectomy.
Answer: E

DISCUSSION: ITP is characterized by a low platelet count and is definitively diagnosed by low platelet count in
association with bone marrow that shows thrombasthenia. Patients with this illness experience abnormal bleeding during
menstruation or nosebleeds or in response to minor insults. Occasionally, a patient who bleeds excessively after an auto
accident is found to have undiagnosed ITP. The usual therapy is initiated with prednisone, and some believe that the
prednisone response is predictive of success after splenectomy. Approximately 80% of patients, however, require increasing
doses of steroids to maintain satisfactory platelet counts and ultimately require splenectomy. Similarly, approximately 80%
of patients achieve normal platelet counts within 3 months after splenectomy, whereas more than 90% are remitted from
further bleeding episodes.

7. Splenectomy and perioperative therapy for ITP:


A. Follow successful steroid therapy.
B. Respond permanently to high-dose intravenous gamma globulin.
C. Are best preceded by polyvalent vaccines for Pneumococcus, Haemophilus influenzae, and Neisseria meningitidis.
D. Cannot be done laparoscopically.
E. Are associated with splenomegaly.
Answer: C

8. Thrombotic thrombocytopenic purpura (TTP) is a syndrome characterized by all of the following except:
A. Thrombocytopenia.
B. Microangiopathic hemolytic anemia.
C. Deposition of platelet microthrombi.
D. Fluctuating neurologic abnormalities.
E. Renal failure.
F. Afebrile.
Answer: F

DISCUSSION: This disease, sometimes called Moschcowitz's syndrome, is characterized by thrombocytopenia,


microangiopathy, chemolytic anemia, fluctuating neurologic abnormalities, progressive renal failure, and fever. Platelet
deposits, with hyaline material composed of aggregated platelets and fibrin, occur. The cause is unknown, and the prognosis
is very poor: survival is less than 10%. A combined approach using antiplatelet drugs and corticosteroids can be effective
and sometimes has improved results if done with splenectomy.

9. Which of the following comments does not describe hypersplenism?


A. It may occur without underlying disease identification.
B. It may be secondary to many hematologic illnesses.
C. It is associated with work hypertrophy from immune response.
D. It requires evaluation of the myeloproliferation.
E. It is associated with antibodies against platelets.
Answer: E

DISCUSSION: Hypersplenism was described by Chauffard in 1907 and initially was mostly associated with the syndrome
in which a variety of hematologic illnesses, ranging from sickle cell disease to the leukemias, could be found to have
improved circulating blood elements if the spleen were removed. Diagnosis of these illnesses has improved, and illnesses
once classified as primary hypersplenism now are frequently classified as secondary, as the primary mechanism involved is
known. The spleen may enlarge owing to differing pathophysiologic mechanisms, but when it achieves a certain size, it
removes all of the main circulating cellular blood elements (red blood cells, white blood cells, platelets). One element may
be more deficient than others.

10. Hyposplenism is a potentially lethal syndrome. Which of the following statements is incorrect?
A. It is confirmed by isotope scan.
B. It is always associated with an atrophic spleen.
C. It may be associated with overwhelming post-splenectomy sepsis syndrome (OPSS).
D. It is associated with thyrotoxicosis, corticosteroid administration, and some contrast agents.
E. It may be associated with ulcerative colitis or sickle cell anemia.
Answer: B

11. Hodgkin's disease is a malignant lymphoma with four histologic subtypes. Which of the following is not one of the
subtypes?
A. Lymphocyte predominance.
B. Nodular sclerosis.
C. Mixed cellularity.
D. Lymphocyte depletion.
E. Leukocyte-lymphocyte dominance.
Answer: E

DISCUSSION: A variety of histologic conventions for the major subtypes of Hodgkin's disease have been derived over the
years. They are useful in identifying the prognosis and predictions for morbidity and mortality. The subtypes are further
subdivided to provide more precise association with therapeutic approaches, with identification with various staging levels,
etc.

12. Which of the following statements about lymphatic capillaries are true?
A. These vessels have delicate tricuspid valves every 2 to 3 mm.
B. Lymphatic capillaries are more permeable than blood capillaries.
C. Lymphatic capillaries are less permeable than blood capillaries.
D. Lymphatic capillaries contain gaps large enough to admit particles as large as lymphocytes.
Answer: CD
DISCUSSION: The transporting lymphatic vessels have valves but lymphatic capillaries do not. The lymphatic capillaries
will accept particles including bacteria, red blood cells, and lymphocytes and transport them to regional lymph nodes.

13. Which of the following forces do not promote the formation of interstitial fluid?
A. Increased venous pressure.
B. Constrictive pericarditis.
C. Hypernatremia.
D. Hypoproteinemia.
Answer: C

DISCUSSION: Interstitial fluid production is a function of the hydrostatic and colloid osmotic pressures across the capillary
membrane. Forces tending to increase interstitial fluid flux across the capillary membrane include obstruction to outflow of
the capillary due to structural or functional obstruction in the venous system or increase in venous pressure from any cause,
reduction in osmotic pressure due to hypoproteinemia, and increase in pore size due to local mediators of inflammation.

14. The most frequent cause of primary lymphedema is:


A. A deficiency of transporting lymphatic channels.
B. Valvular incompetence in lymphatic channels.
C. Obstruction or removal of regional lymph nodes.
D. Thrombosis of lymphatic channels.
Answer: A

DISCUSSION: Primary lymphedema is most frequently due to hypoplasia or aplasia of extremity transporting lymphatic
channels.

15. Most patients with lymphedema can be managed by:


A. Pedicle transfer of lymphatic bearing tissue into the affected area.
B. Elevation, elastic support garments, and massage therapy or mechanical pneumatic compression.
C. Lymphatic bypass using an autogenous vein graft.
D. Excision of hypertrophic scarred fibrotic skin and subcutaneous tissue down to muscle fascia and coverage with split-
thickness skin grafts.
Answer: B

DISCUSSION: The vast majority of patients with mild to moderate lymphedema can be managed with leg elevation and
elastic support garments, and some will require mechanical pneumatic compression or massage therapy. Only patients with
very severe deforming elephantiasis require operative therapy.

16. Which statements about lymphangiomas are true?


A. Most lesions appear during puberty.
B. These lesions frequently respond to small doses of radiation therapy.
C. The lesions usually grow slowly but may infiltrate local tissues.
D. Malignant transformation is frequent.
Answer: C

DISCUSSION: Lymphangiomas are congenital malformations of the lymphatic vessels. Most appear at birth or during early
infancy. The lesions grow slowly and infiltrate local tissues, but actual malignant degeneration is exceedingly rare. The
lesions are not responsive to radiation therapy.

17. The two primary causes of death from sickle cell disease in the first decade of life are which of the following?

a. Sepsis
b. Splenic sequestration crisis
c. Acute chest syndrome
d. Heart failure
Answer: a, b

Sickle cell disease and thalassemia are the principal hemoglobinopathies that cause clinically important disease. Clinical
symptoms are directly dependent upon the amount of HgbS present, and this is variable. Patients who are homozygous for
HgbS usually have small sequential splenic infarcts as a result of microvascular occlusion and the spleen usually becomes
small, fibrotic, and dysfunctional by the age of 5 years. Splenic sequestration crisis is a cause of anemia that can be acute
and life-threatening. Sepsis associated with functional asplenia can be lethal as well. These are the two primary causes of
death in sickle cell disease during the first decade of life.

18. The best therapy for a patient with thrombotic thrombocytopenic purpura is which of the following:

a. Plasmapheresis
b. Corticosteroids
c. Splenectomy
d. Intravenous immune globulin
Answer: a

Thrombotic thrombocytopenic purpura (TPP) is a relatively rare syndrome with no definitive diagnostic test or clinical
characteristic. The principal clinical features are thrombocytopenic purpura, fever, microangiopathic hemolytic anemia,
mental status changes, and renal dysfunction. Treatment is generally focused on the removal of the plasma constituents that
lead to platelet aggregation. Plasmapheresis is particularly effective and has decreased the 1-year mortality rate of 50% to
80% to about 10%. Splenectomy was initially proposed for TPP but has been shown to have little benefit and instead is
associated with a considerable risk of postsplenectomy sepsis. Corticosteroids and intravenous immune globulin are
appropriate therapies for immune thrombocytopenic purpura (ITP), but not TPP.

19. Which of the following statements regarding post splenectomy sepsis are true?

a. The incidence in children is generally reported as less than 5%


b. Haemophilus influenzae, Streptococcus pneumoniae and Neiseria meningitidis are the most common causative
organisms
c. Autotransplantation techniques eliminate this risk
d. The mortality rate is now approximately 50%
e. The incidence in adults in approximately 1%
Answer: a, b, d, e

Postsplenectomy sepsis (PSS) refers to the increased risk of systemic infection in patients who have undergone
splenectomy. The incidence of PSS in children is as high as 4% during a follow-up of nearly 10 years, and from 0.3% to
1.8% in adults during an 8-year follow-up. The highest incidence of PSS is among those who have undergone splenectomy
for associated malignancies or who had an incidental splenectomy during other surgical procedures in adulthood. In
children, the patients most at risk for PSS are those who have undergone splenectomy for either congenital or acquired
anemias. It is important to remember that any patient who has undergone splenectomy or who has hyposplenism is at risk
for PSS.
The most common causative organisms are Streptococcus pneumoniae, Haemophilus influenzae, and Neiseria meningitidis.
Other less common encapsulated organisms have also been identified. In addition, organisms without classical
polysaccharide cell walls can occasionally produce overwhelming infection in postsplenectomy patients. The mortality from
PSS is approximately 50%. Splenic autotransplantation has been proposed as an alternative means of splenic preservation in
patients whose spleens are too severely injured for in situ repair. Splenic autotransplants do not appear to have normal
splenic function. Splenic autotransplants have diminished ability to protect against PSS when compared to a normal spleen
and they have not been demonstrated to prevent post-splenectomy sepsis.

20. You are consulted regarding a 50-year old male with Laennec’s cirrhosis, portal hypertension and hypersplenism. He has
no history of gastrointestinal bleeding. You would recommend which of the following?

a. Splenectomy
b. Prophylactic sclerotherapy for esophageal varices
c. Portosystemic shunt
d. Observation
Answer: d

Patients with hypersplenism from portal hypertension usually present with thrombocytopenia as the primary manifestation.
The role of splenectomy in such patients is controversial. Although splenectomy may transiently increase the number of
platelets in the peripheral circulation, it generally does not improve survival. At the time of surgery, these patients have
considerable risk for massive bleeding and subsequent complications related to hepatic dysfunction. These latter
complications include ascites, coagulopathy, and wound complications. The severity of the underlying liver disease is the
determining factor for these patients. Some believe that splenectomy is contraindicated in patients with portal hypertension.
Indeed, the perioperative risks are considerably greater than normal. This should be considered as a last option, and even
then with the realization that the operative and preoperative morbidity and mortality are high.
In the absence of gastrointestinal bleeding, this particular patient has no indication for either sclerotherapy or portosystemic
shunting.

21. Glucose-6-phosphate dehydrogenase (G6PD) deficiency is an abnormality of erythrocyte metabolism associated with
hemolytic anemia. Pharmocologic agents which induce hemolysis include which of the following?

a. Acetylsalicylic acid
b. Vitamin E
c. Sulfamethoxazole
d. Desferrioxamine
e. Nitrofurantoin
Answer: a, c, e

Generally, G6PD deficiency is not clinically apparent; hemolytic anemia occurs in most patients only after the consumption
of certain medications or exposure to specific chemicals. A partial list of oxidizing agents known to induce acute hemolytic
anemia in patients with G6PD deficiency follows:
Acetylsalicylic acid (high dose)
Phenacetin
Nitrofurantoin
Sulfamethoxazole
Doxorubicin
Nalidixic acid
Acetanilid
Primaquine
Phenazopyridine
Methylene blue
Niridazole
Furazolidone
Fava beans
Splenectomy is rarely indicated in patients with G6PD deficiency. Management generally is adequate simply by avoiding
exposure to precipitating agents. Vitamin E and desferrioxamine may be therapeutic for G6PD deficient patients,
presumably related to their antioxidant activity.

22. Hypersplenism is associated with which of the following diseases?

a. Portal hypertension
b. Lymphoma
c. Mononucleosis
d. Systemic lupus erythematosus
e. Gaucher disease
Answer: a, b, c, d, e

Hypersplenism is not a specific disease but rather a physiologic state characterized by splenomegaly, a decrease in
circulating levels of some blood cells or platelets, bone marrow hypertrophy in response to the decrease in the circulating
blood elements, and some degree of improvement by splenectomy. Disease processes associated with secondary
hypersplenism include the following:
INCREASED VENOUS PRESSURE
Portal hypertension
Splenic vein thrombosis
Severe congestive heart failure
MALIGNANCY
Leukemias (especially chronic)
Lymphoma
CHRONIC INFLAMMATORY DISEASES
Felty syndrome
Systemic lupus erythematosus
Sarcoidosis
METABOLIC ABNORMALITIES
Amyloidosis
Gaucher disease
Niemann-Pick disease
INFECTION
Mononucleosis
Bacterial endocarditis
Parasites
Fungus
OTHER
Myelofibrosis with myeloid metaplasia
Polycythemia vera

23. A 40-year old woman with chronic immune thrombocytopenic purpura (ITP) is refractory to corticosteroids. The
approximate likelihood she will benefit from a splenectomy is approximately which of the following?

a. Less than 20%


b. 40%
c. 60%
d. 80%
Answer: c

Splenectomy remains the principal treatment for ITP. Platelet counts rise to adequate levels in 60% to 80% of patients who
undergo the procedure. Ninety percent of patients who have had good responses to corticosteroids have improved platelet
counts after splenectomy. Of patients who do not respond to corticosteroids, about 60% respond to splenectomy.
Splenectomy is effective by virtue of its ability to remove the site of platelet destruction. Because the spleen is the site of
most platelet sequestration in ITP, splenectomy should eliminate this source of platelet consumption. Furthermore,
splenectomy removes a significant source of antiplatelet IgG production.

24. Which of the following statements regarding splenic function in humans are true?

a. The specific immune function of the spleen is principally related to its antigen processing role
b. The spleen is the major site of synthesis of complement pathway proteins
c. The spleen is more efficient than the liver at removing bacteria with a high density of surface opsonins
d. The spleen serves as a principal source of nonspecific opsonins
Answer: a, d

Immune function of the spleen includes both nonspecific and specific immune responses. Nonspecific immune function is
largely characterized by removal of particulate matter by the macrophages. The spleen contains 25% of the fixed tissue
macrophage population in the body. The spleen is more efficient than the liver at removal of incompletely opsonized
bacteria. The liver is most effective at removing bacteria with a high density of surface opsonins. The spleen also serves as a
principal source of nonspecific opsonins. These include tuftsin, properidin, and fibronectin. Tuftsin stimulates granulocyte
and macrophage motility and phagocytosis. Properdin activates the alternative pathway of the complement system, leading
to complement fixation. Both the activated complement complexes and the complement products facilitate the destruction
of the target organism. Fibronectin is a macromolecule that appears to have nonspecific stimulatory activity on the processes
of fibrosis and wound healing.
The specific immune functions of the spleen are principally related to its antigen-processing role, and this is in turn
dependent on its unique anatomy and the circulation of lymphocytes into the spleen.
The liver, not the spleen, is the major site of synthesis of complement pathway proteins.

1. When progressive enlargement of a multinodular goiter causes symptomatic tracheal compression, the preferred
management in otherwise good-risk patients is:
A. Iodine treatment.
B. Thyroid hormone treatment.
C. Surgical resection of the abnormal thyroid.
D. Radioactive iodine treatment.
Answer: C

DISCUSSION: When a multinodular goiter enlarges enough to cause symptoms of tracheal compression, surgical treatment
is usually required if the patient is considered a reasonable operative risk. Medical treatment may be effective in preventing
the initial growth of the goiter but is unlikely to cause enough regression to relieve symptoms. Radioactive iodine can
occasionally be used to cause some regression in patients who are poor anesthesia risks, but this is a temporizing treatment
rather than a definitive one.

2. The most precise diagnostic screening procedure for differentiating benign thyroid nodules from malignant ones is:
A. Thyroid ultrasonography.
B. Thyroid scintiscan.
C. Fine-needle-aspiration biopsy (FNAB).
D. Thyroid hormone suppression.
Answer: C

DISCUSSION: Analysis of multiple series in which patients with thyroid nodules have undergone FNAB has demonstrated
a false-negative rate of 2.4% and a false-positive rate of 3.3%. Sensitivity for this method is 92%; specificity 74%. This
surpasses the other methods for accurate selection of patients who require surgical resection.

3. The preferred operation for initial management of a thyroid nodule that is considered suspicious for malignancy by
FNAB is:
A. Excision.
B. Partial lobectomy.
C. Total lobectomy and isthmusectomy.
D. Total thyroidectomy.
Answer: C

DISCUSSION: There is a consensus that the initial minimum operation for a nodule suspected to be malignant is total
lobectomy and isthmusectomy. Partial lobectomy or excision of the nodule is associated with a higher risk of local
recurrence if the nodule proves to be malignant. Reoperation on the side of a partial lobectomy can be technically difficult
and associated with a higher risk of recurrent nerve injury. Ordinarily, total thyroidectomy is not performed until a
conclusive diagnosis of malignancy is established.

4. Advantages of total thyroidectomy for management of papillary carcinomas of the thyroid larger than 1.5 cm. include:
A. Possibility of using radioactive iodine postoperatively to identify and treat metastases.
B. The ability to use thyroglobulin levels as a marker for recurrence.
C. Lower overall recurrence rate.
D. Lower risk of hypoparathyroidism.
Answer: ABC

DISCUSSION: Following total thyroidectomy iodine 131 can be used more efficiently because of the absence of normal
thyroid tissue, which has greater affinity for iodine than papillary carcinoma tissue. When all normal thyroid tissue is
removed, serum thyroglobulin, which is produced by normal and malignant thyroid tissue, becomes a more effective marker
for recurrence. The overall recurrence rate is lower for patients undergoing total thyroidectomy, but the risk of
hypoparathyroidism is higher for patients who have total thyroidectomy instead of unilateral lobectomy.

5. Which of the following statements about follicular carcinoma is/are true?


A. It presents at a later age than papillary carcinoma.
B. It disseminates via hematogenous routes.
C. It is the most common type of well-differentiated thyroid carcinoma.
D. Extensive angioinvasion portends a poor prognosis.
E. Follicular carcinomas are frequently multicentric.
Answer: ABD

DISCUSSION: Follicular carcinoma is more common in older patients (peak incidence in the fifth decade). The tumor has a
marked propensity for vascular invasion and spreads hematogenously to bone, lung, liver, and central nervous system sites.
Local nodal metastases are less common than in papillary carcinoma. Extensive angioinvasion indicates a less favorable
prognosis. Papillary carcinoma is the most common type of well-differentiated thyroid carcinomas. Follicular carcinomas
are rarely multicentric.

6. A familial form of medullary thyroid carcinoma (MTC) should be suspected whenever:


A. The tumor is multifocal.
B. The tumor is bilateral (foci of tumor are present in both thyroid lobes).
C. Pathologic examination of the resected thyroid gland reveals the presence of C-cell hyperplasia in areas of the gland
adjacent to foci of MTC.
D. All of the above.
Answer: D

DISCUSSION: Sporadic MTC is unilateral in at least 80% of cases. However, in patients with MTC occurring as a
component of the multiple endocrine neoplasia (MEN) type 2A or type 2B syndromes, the tumor is virtually always
multifocal and bilateral. Typically, in this setting the MTC appears as multiple whitish-tan tumor nodules in the middle and
upper thirds of each thyroid lobe.
A diffuse premalignant proliferation of the C cells of the thyroid is thought to precede the development of MTC in patients
with familial MTC. This proliferation, known as C-cell hyperplasia (CCH), consists of parafollicular clusters of increased
numbers of C cells. The finding of CCH in areas of the thyroid adjacent to gross foci of MTC is strong evidence for familial
MTC.

7. All of the following are components of the MEN type 2B syndrome except:
A. Multiple neuromas on the lips, tongue, and oral mucosa.
B. Hyperparathyroidism.
C. MTC.
D. Pheochromocytoma.
Answer: B

DISCUSSION: MTC and pheochromocytoma occur in both MEN 2A and MEN 2B syndromes. Patients with MEN 2A may
also develop hyperplasia of the parathyroid glands. Although some investigators have reported equivocal histologic
abnormalities in the parathyroid glands of patients with MEN 2B, hyperparathyroidism is not a component of this
syndrome.
In contrast to patients with MEN 2A, those with MEN 2B have a characteristic phenotype, including a tall, thin “marfanoid”
habitus. Patients with MEN 2B also develop multiple neuromas on the lips, tongue, and oral mucosa, creating the
appearance of thick lips.

8. MEN 2A and MEN 2B syndromes are associated with germline mutations in:
A. The p53 tumor suppressor gene.
B. The H-ras gene.
C. The N-myc gene.
D. The RET proto-oncogene.
Answer: D

DISCUSSION: Germline mutations in the RET protooncogene, a receptor tyrosine kinase that maps to chromosome 10, are
associated with MEN 2A and MEN 2B syndromes. Homozygous loss of the tumor suppressor gene p53 is associated with
the Li-Fraumeni syndrome, and mutations of p53 are present in a variety of human neoplasms. Point mutations in the H-ras
gene are associated with carcinoma of the colon, lung, and pancreas. Amplification of the N-myc gene, when present in
neuroblastoma, suggests a poorer prognosis.

9. Which of the following are true concerning islet cell neoplasms of the pancreas in patients with MEN type 1?
A. Islet cell neoplasms in patients with MEN 1 are characteristically multicentric.
B. The most common islet cell neoplasm in patients with MEN 1 is gastrinoma.
C. Islet cell neoplasms in patients with MEN 1 may be malignant.
D. All of the above.
Answer: D

DISCUSSION: The pathologic change in the pancreas of patients with MEN 1 is typically multicentric. Diffuse hyperplasia
of islet cells and microadenoma formation are often identified in areas of the gland distant from grossly evident tumor.
Tumors are commonly multifocal. Islet cell neoplasms of the pancreas occur in 30% to 80% of patients with MEN 1. The
most common islet cell neoplasm in these patients is gastrinoma. Gastrinomas associated with MEN 1 probably account for
20% to 50% of all cases of the Zollinger-Ellison syndrome. The second most common islet cell tumor is insulinoma. Other
pancreatic islet cell neoplasms, such as glucagonoma, somatostatinoma, or vasoactive intestinal polypeptide neoplasm
(VIPoma), are rarely associated with MEN 1. Approximately 10% of insulinomas and approximately 15% or more of
gastrinomas in patients with MEN 1 are malignant.

10. Which of the following statements about the differential diagnosis of hypercalcemia is/are correct.
A. Malignant tumors typically cause hypercalcemia by ectopic production of parathyroid hormone (PTH).
B. The diagnosis of primary hyperparathyroidism is supported by these serum levels: calcium, 10.8 mg. per dl.; chloride,
104 mmol. per liter; bicarbonate 21 mmol. per liter; phosphorus, 2.4 mg. per dl.; elevated parathyroid hormone.
C. Familial hypocalciuric hypercalcemia is distinguished from primary hyperparathyroidism by parathyroid imaging.
D. Although serum albumin binds calcium, the measured total calcium value is usually unaffected in patients with severe
hypoproteinemia.
E. Thiazide diuretics are a good treatment for hypercalcemia and can be given to patients with apparent hypercalcemia of
malignancy.
Answer: B

DISCUSSION: Malignant tumors rarely secrete PTH itself; they can secrete PTHrP or cytokine activators of osteoclast
activity. The diagnosis of primary hyperparathyroidism is supported by hypercalcemia with mild hyperchloremic metabolic
acidosis and a chloride-phosphate ratio greater than 33 or a modified chloride (mmol. per liter/mg. per dl.) phosphate ratio
greater than 500. Familial hypocalciuric hypercalcemia is distinguished from primary hyperparathyroidism by a low urine
calcium. Serum calcium changes approximately 0.8 mg. per dl. for every 1 gm. per dl. change in serum albumin. Thiazide
diuretics can cause hypercalcemia and should not be given to patients who are hypercalcemic.

11. Indications for operation in a patient with previously asymptomatic hyperparathyroidism include:
A. Age older than 60.
B. Nephrolithiasis.
C. A substantial decline in renal function.
D. A substantial decline in bone mass.
E. Depression and fatigue.
Answer: BCDE

DISCUSSION: Age younger than 50 is considered an indication for operation.


12. The parathyroid glands:
A. Develop from the second and third pharyngeal pouches, along with the palatine tonsil and the thymus.
B. Migrate caudally in the neck in normal development but can be found anywhere from the pharyngeal mucosa to the deep
mediastinum.
C. Secrete PTH and calcitonin to manage calcium homeostasis.
D. Usually number four, but frequently number only two or three.
E. Contain enzymes that catalyze the conversion of 25(OH) vitamin D 3 to 1,25(OH) 2 vitamin D 3.
Answer: B

DISCUSSION: The parathyroid glands develop from the third and fourth pharyngeal pouches, along with the thymus and
the thyroid ultimobronchial body; there are four glands in the vast majority of persons. Calcitonin is secreted by the C cells
of the thyroid. Vitamin D 3 hydroxylation occurs in the kidney.

13. Hyperparathyroidism can affect which of the following organs and body systems?
A. Gastrointestinal tract.
B. Kidneys.
C. Skeleton.
D. Neuromuscular system.
E. Cardiovascular system.
Answer: ABCDE

DISCUSSION: Gastrointestinal involvement includes pancreatitis and peptic ulcer disease. The kidneys can be affected by
nephrocalcinosis or nephrolithiasis. The bones can be severely affected by bone resorption. Neuromuscular complaints can
include weakness and fatigue. Hypertension is present in as many as 70% of people with hyperparathyroidism and may be
related to the renal disease.

14. Secondary hyperparathyroidism:


A. Is a metabolic disease in which the primary abnormality is decreased glomerular filtration rate.
B. Is best treated initially by subtotal parathyroidectomy.
C. Is caused by increased production of 1,25(OH) 2 vitamin D 3, causing increasing intestinal calcium absorption and
hypercalcemia.
D. Can have severe effects on bones exacerbated by aluminum contained in phosphate binders and dialysate water.
E. Is best treated initially by total parathyroidectomy with autotransplantation.
Answer: AD

DISCUSSION: Secondary hyperparathyroidism is caused by renal disease, including decreased glomerular filtration rate
and decreased 1,25(OH) 2 vitamin D 3 production. It is best treated by medical management restricting dietary phosphate,
administering vitamin D and calcium with phosphate-binding gels, and limiting aluminum exposure, which can exacerbate
bone disease.

15. Hypoparathyroidism:
A. Is most commonly encountered as a postviral syndrome.
B. Can be associated with marked hypocalcemia after parathyroidectomy in patients with bone disease.
C. Can cause anxiety, depression, or confusion.
D. Can cause physical signs such as Chvostek's and Trousseau's signs.
E. Is treatable acutely with intravenous calcium salts and chronically with oral calcium and vitamin D.
Answer: BCDE

DISCUSSION: Hypoparathyroidism is most commonly encountered after thyroid surgery. It can be temporarily severe after
parathyroidectomy in patients with bone disease (“postoperative bone hunger”). The signs and symptoms can include
anxiety, depression, confusion, Chvostek's sign, and Trousseau's sign, as well as circumoral or extremity tingling, tetany
with carpopedal spasms, or seizures. Treatment is as noted.
16. True statements about pituitary anatomy and physiology include:
A. The pituitary has dual embryonic origin: the anterior pituitary arises from embryonic ectoderm; the posterior pituitary,
from the diencephalon.
B. The hypophyseal portal system integrates function of the anterior and posterior pituitary.
C. Adrenocorticotropin (ACTH), formed by posttranslational processing of the precursor POMC, is normally controlled by
hypothalamic CRF but may be released by immune-related mechanisms.
D. Growth hormone (GH) directly stimulates longitudinal growth of the skeleton and growth of muscles.
E. Cell types of the anterior pituitary are classified by their position in the anterior pituitary and by their staining
characteristics with histologic dyes.
Answer: AC

DISCUSSION: The anterior pituitary arises from Rathke's pouch (embryonic ectoderm), and it includes the pars distalis,
pars intermedia, and pars tuberalis. The posterior pituitary arises from the diencephalon and includes the neural stalk,
infundibulum, and posterior lobe. The hypophyseal portal system drains from the hypothalamus and integrates function of
the hypothalamus and anterior pituitary by carrying hypothalamus-derived releasing factors to target cells in the anterior
pituitary. ACTH may be released during stress in response to interleukin-1, -2, and -6. Somatotrophic actions of GH are
indirect and are mediated by the insulinlike growth factors formerly known as somatomedins. Current classification of
anterior pituitary cells is based on immunochemical identification of their secretory products (i.e., corticotropes produce
ACTH, lactotropes produce prolactin, and thyrotropes produce thyroid-stimulating hormone [TSH]).

17. Antidiuretic hormone (ADH):


A. Is related to oxytocin, and both are released from the posterior pituitary in conjunction with neurophysins.
B. Is released into the circulation by the posterior pituitary in response to a rise in plasma osmolality above 285 mOsm. or a
decrease in circulating blood volume.
C. May be stimulated by catecholamines and inhibited by phenytoin, alcohol, and lithium.
D. In excess, may produce a syndrome of euvolemic hyponatremia with inappropriately concentrated urine that is
responsive to free water restriction.
E. Deficiency causes prolonged polyuria and polydipsia and may be diagnosed by a combination of high plasma osmolality
and low urine osmolality following water deprivation.
Answer: ABCDE

DISCUSSION: ADH and oxytocin are nine–amino acid peptides derived from a common ancestral peptide, vasotocin. ADH
is released from the posterior pituitary with neurophysin II in response to a rise in plasma osmolality greater than 285
mOsm/kg. H 2O. by a 5% or greater decrease in blood volume, and by catecholamines. The syndrome of inappropriate
ADH release (SIADH) produces euvolemic hyponatremia and is responsive to free water restriction. Diabetes insipidus (DI)
reflects a deficiency of ADH and causes prolonged polyuria and polydipsia. Diabetes insipidus is diagnosed by a
combination of high plasma osmolality and low urine osmolality following water deprivation. Exogenously administered
ADH differentiates central (ADH-responsive) DI from nephrogenic (ADH-unresponsive) DI.

18. Signs and symptoms of acute pituitary apoplexy include:


A. Severe headache.
B. Meningismus.
C. Vision loss.
D. Shock.
E. May be relieved by emergent transsphenoidal decompression of the sella turcica.
Answer: ABCD

DISCUSSION: Acute pituitary apoplexy follows sudden hemorrhage into a pre-existing pituitary tumor or following closed
head trauma. Symptoms, including headache, meningismus, and vision loss, are attributable to the intracerebral blood.
Pituitary insufficiency, as well as the accompanying secondary adrenal insufficiency, may cause hypotension and shock.
Other manifestations may include DI and myxedema. Acute pituitary apoplexy is a neurosurgical emergency that requires
transsphenoidal decompression of the sella turcica.

19. Prolactinomas of the pituitary:


A. Most often produce dysfunctional uterine bleeding in women.
B. Most commonly produce infertility in men.
C. When asymptomatic, are best treated surgically early in the microadenoma stage.
D. May enlarge during pregnancy, requiring treatment with bromocriptine or surgery.
E. Commonly occur in patients with MEN 2.
Answer: D

DISCUSSION: Pituitary prolactinomas are the most common pituitary tumor and cause amenorrhea, galactorrhea, or both
in women. Men usually have macroprolactinomas and experience symptoms of a space-occupying lesion of the sella (i.e.,
headache). A minority of men experience impotence and infertility. Asymptomatic prolactinomas remain stable over time
and require observation only. Symptomatic prolactinomas require treatment, usually with bromocriptine. Prolactinomas may
enlarge during pregnancy and can produce symptoms requiring treatment with either bromocriptine or surgery. Prolactiomas
are seen in as many as 40% of patients with MEN 1.

20. Hypercortisolism:
A. Is most often ACTH-dependent, owing to an ACTH-producing pituitary adenoma.
B. Is best diagnosed by measurement of cortisol from a serum sample collected at 8 A.M.
C. Is attributable to an adrenal source if the basal serum ACTH level is above 10 pg. per ml., if the hypercortisolism is
suppressed by high-dose dexamethasone, and if an adrenal tumor is visualized radiographically.
D. May be caused by small cell carcinoma of the lung, carcinoid tumors, tumors of the endocrine pancreas,
pheochromocytoma, or medullary thyroid carcinoma (MTC).
E. In children is most often caused by adrenocortical neoplasia.
Answer: ADE

DISCUSSION: Cushing's syndrome is most often (80% to 90%) ACTH-dependent and is most often due to an ACTH-
secreting pituitary adenoma (Cushing's disease). Some 10% to 20% of ACTH-dependent Cushing's syndrome is caused by
ectopic production of ACTH from small cell carcinoma of the lung, carcinoid tumors, tumors of the endocrine pancreas,
pheochromocytoma, and MTC. ACTH-independent Cushing's syndrome secondary to primary adrenal pathology occurs in
10% to 20% of cases in adults and is the most common form in children. Measurement of cortisol in two to three
consecutive 24-hour collections of urine is the best screening test for Cushing's syndrome: plasma levels of cortisol show
marked variability, and a single random level is not helpful in establishing the diagnosis. ACTH-independent Cushing's
syndrome suppresses the pituitary and is diagnosed if the basal serum ACTH level is suppressed below 5 pg. per ml., if the
hypercortisolism is not suppressed by high-dose dexamethasone, or if the metyrapone stimulation test is negative. The
dexamethasone suppression and metyrapone tests do not, however, distinguish an adrenal from an ectopic ACTH cause of
Cushing's since both suppress the pituitary. Radiographic evidence of an adrenal tumor must be supported by biochemical
testing to confirm that the adrenal is the primary cause of Cushing's syndrome.

21. Primary aldosteronism:


A. Produces a syndrome of diastolic hypertension, hypokalemia, and edema.
B. Is suggested by findings of serum potassium less than 3.5 mEq. per liter, urinary potassium excretion greater than 30
mEq. per day, upright plasma renin below 3 ng. per ml., and a plasma aldosterone concentration–plasma renin activity ratio
greater than 20:1.
C. Is most often due to an aldosterone-producing adrenal adenoma, which may be distinguished from idiopathic adrenal
hyperplasia by its sensitivity to diurnal changes in ACTH and insensitivity to changes in posture.
D. May be diagnosed in hypertensive patients by demonstration of an adrenal mass larger than 1 cm. on computed
tomography alone.
E. Is best treated surgically if it is due to either aldosteronoma or idiopathic adrenal hyperplasia.
Answer: BC

DISCUSSION: Primary aldosteronism is a syndrome of diastolic hypertension and hypokalemia; edema typically is absent.
The diagnosis relies on demonstration of hypokalemia with inappropriate kaliuresis and inappropriately elevated plasma
aldosterone with suppression of plasma renin following sodium loading. Primary aldosteronism most often is due to an
adrenal adenoma, which must be distinguished from idiopathic adrenal hyperplasia since resection of the adenoma is nearly
always curative, whereas adrenal resection for idiopathic adrenal hyperplasia is curative less than 20% of the time. CT
evidence of adrenal tumor alone is inadequate to diagnose aldosteronism, even in a hypertensive patient.

22. Adrenocortical carcinoma:


A. May be suspected in a patient with rapidly progressive Cushing's syndrome and virilizing features or in asymptomatic
patients with adrenal tumors larger than 6 cm on CT.
B. Most often is diagnosed early in its course when disease is confined to the adrenal gland.
C. Is differentiated from benign adrenocortical adenoma by tumor necrosis, hemorrhage, and cellular features of large
hyperchromatic nuclei and more than 20 mitoses per high-power field.
D. Should be resected only if disease is localized to the adrenal gland; otherwise treatment with mitotane is indicated.
E. Carries a poor prognosis: overall 5-year survival less than 25%.
Answer: AE

DISCUSSION: Patients with adrenocortical carcinoma often present with rapidly progressive syndromes of combined
adrenocortical hormone excess. However, approximately half do not have syndromes of hormone excess but present with
abdominal pain, increased abdominal girth, weight loss, and anorexia. Furthermore, incidentally discovered adrenal masses
larger than 6 cm. harbor carcinoma in as many as 92% of cases. Adrenocortical carcinoma is differentiated from adenoma
by pathologic demonstration of either local invasion or distant metastases. Most patients with adrenocortical carcinoma
present with locally advanced (stage III) or metastatic (stage IV) disease. Prognosis for these patients is poor: 5-year
survival is less than 25% in most series. Patients with adrenocortical carcinoma should undergo primary surgical resection
or palliative surgical debulking of locally advanced, metastatic, or even recurrent disease since response of these tumors to
medical therapy, including mitotane, is poor.

23. Addisonian crisis, or acute adrenocortical insufficiency:


A. Occurs only in patients with known adrenal insufficiency or in those receiving long-term supraphysiologic doses of
exogenous steroids.
B. Can mimic an acute abdomen with fever, nausea and vomiting, abdominal pain, and hypotension.
C. May cause electrolyte abnormalities, including hypernatremia, hypokalemia, hypoglycemia, and hypercalcemia, as well
as eosinophilia on peripheral blood smear.
D. Should be diagnosed with the rapid ACTH stimulation test before steroid replacement is instituted.
E. May be effectively treated with intravenous “stress-dose” glucocorticoid and mineralocorticoid replacement.
Answer: B

DISCUSSION: Addisonian crisis, or acute adrenal insufficiency, may be seen following even mild illness in patients with
suppression of the hypothalamic-pituitary-adrenal axis. This suppression can be produced by as little as 1 week of
supraphysiologic stress dose steroids in the year before the stressful event. Addisonian crisis is a medical emergency that
requires prompt treatment based on clinical suspicion. Clinical findings include fever, nausea and vomiting, abdominal pain,
and hypotension. Laboratory analysis may reveal electrolyte abnormalities, including hyponatremia, hyperkalemia,
hypoglycemia, and hypercalcemia, as well as eosinophilia on peripheral blood smear. The rapid ACTH test is diagnostic, but
it should not delay treatment with intravenous fluid resuscitation, glucose replacement, and high-dose dexamethasone.
Dexamethasone, not hydrocortisone, should be given initially, since it does not interfere with subsequent determination of
plasma cortisol. Stress dose steroids are inadequate once adrenal crisis has occurred, and exogenous mineralocorticoids are
given when the patient resumes oral intake.

24. Preparation for surgical removal of a pheochromocytoma includes:


A. Beta-adrenergic blockade followed by alpha-adrenergic blockade.
B. Hydration.
C. Alpha-adrenergic blockade, with or without beta-adrenergic blockade.
D. Preoperative Swan-Ganz monitoring in all patients.
E. Planning removal through an anterior, posterior, or laparoscopic approach based upon tumor localization with CT,
magnetic resonance imaging (MRI), and/or 131I-MIBG.
Answer: BCE

DISCUSSION: Principles of preoperative management for pheochromocytoma include preoperative alpha-adrenergic


blockade using phenoxybenzamine or phentolamine. Beta-adrenergic blockade with propranolol is then used selectively in
patients who develop tachycardia, have a history of cardiac arrhythmia, or have primarily epinephrine-secreting tumors.
Beta-adrenergic blockade should be undertaken only after successful alpha blockade is established. Patients with
pheochromocytoma frequently exhibit intravascular volume depletion, and careful hydration is mandatory. Central venous
pressure monitoring alone is helpful to guide hydration; more intensive monitoring with a Swan-Ganz catheter is indicated
for patients with pre-existing heart disease. Formerly, the anterior approach was preferred for adrenalectomy as it facilitated
complete abdominal exploration and search for extra-adrenal pheochromocytoma. Accurate preoperative localization with
CT, MRI, and 131I-MIBG has allowed selective use of the posterior, or even the laparoscopic, approach for adrenalectomy.

25. Indications for surgical adrenalectomy include:


A. An adrenal mass larger than 6 cm.
B. Hypertensive patients with aldosteronism that is ACTH insensitive and posture sensitive and who also have multiple
adrenal nodules on CT.
C. Cushing's syndrome secondary to adrenal neoplasms or to persistent ectopic ACTH syndrome when the primary tumor is
inoperable.
D. Pheochromocytoma in adults and children.
E. Congenital adrenal hyperplasia secondary to 21-hydroxylase deficiency.
Answer: ACD

DISCUSSION: Adrenalectomy is indicated for the management of adrenal masses larger than 6 cm. on CT since tumors of
this size harbor carcinoma in more than 90% of cases. Surgical adrenalectomy is also the treatment of choice for primary
adrenal causes of Cushing's syndrome. Surgical adrenalectomy may be considered as an alternative to medical
adrenalectomy with metyrapone, aminoglutethimide, or mitotane in patients with ectopic ACTH syndrome when treatment
of the primary tumor is unsuccessful. Patients with aldosteronism that is ACTH sensitive and posture insensitive likely have
an adrenal adenoma that is curable by adrenalectomy. Aldosteronism that is ACTH insensitive and posture sensitive is likely
caused by idiopathic adrenal hyperplasia, which is best managed medically with spironolactone, triamterene, amiloride, or
nifedipine. Adrenalectomy following preoperative alpha-adrenergic blockade, with or without beta-adrenergic blockade, and
hydration is the treatment of choice of all pheochromocytomas. Adrenalectomy is not indicated in the management of any of
the congenital adrenal hyperplasias.

26. Incidental adrenal masses:


A. May be seen in as many as 10% of abdominal CT studies.
B. Most commonly represent pheochromocytoma; adrenocortical adenoma, adrenocortical carcinoma, and metastases from
other primary cancers occur less frequently.
C. May represent adrenocortical carcinoma if greater than 6 cm. in diameter.
D. Should be routinely evaluated by measurement of 24-hour urine levels of catecholamines and their metabolites, cortisol,
and aldosterone plus fine-needle aspiration.
E. Should be resected if biochemically active, if greater than 6 cm., or if they grow over six months' follow-up.
Answer: CE

DISCUSSION: The incidental adrenal mass is seen in as many as 1.3% of abdominal CT scans performed for other reasons.
Adrenocortical adenomas are most common, followed by adrenocortical carcinoma, metastases from other primary cancers,
and pheochromocytoma. Biochemical evaluation must weigh the prevalence of adrenal neoplasms against the consequences
of a missed life-threatening diagnosis, as in pheochromocytoma. All adrenal masses should be evaluated for
pheochromocytoma with measurement of 24-hour urine catecholamines and their metabolites. Aldosterone and cortisol
measurement are indicated if clinical features suggest aldosteronism or Cushing's syndrome. Fine-needle aspiration of
adrenal masses is indicated for clearly cystic lesions or if metastasis is suspected based on the presence of another known
primary. Fine-needle aspiration is not routinely indicated in the evaluation of adrenal lesions and is contraindicated until
pheochromocytoma is definitively excluded. Adrenal lesions should be resected if they are functional, are larger than 6 cm.,
or have enlarged during follow-up.

27. Which of the following statements is true about the synthesis of thyroid hormone and its physiology?
A. The iodine utilized in hormone synthesis is derived principally from dietary sources.
B. The role of thyroid-stimulating hormone (TSH) in thyroid physiology is limited to regulation of the release of thyroid
hormone in plasma.
C. Enough thyroxine (T 4) is stored in the normal thyroid to provide a euthyroid state for 3 weeks despite absence of iodine
intake.
D. The regulation of thyroid function involves pituitary, but not hypothalamic, input.
Answer: AC

DISCUSSION: Iodine is necessary for the synthesis of thyroid hormone, and approximately 200 to 500 mg is ingested daily.
Most of it is absorbed from the small intestine and is cleared from the plasma by secretion of thyroid hormone. TSH is
required for the normal production and secretion of thyroid hormone. It is generally accepted that TSH also has a major role
in thyroid growth. The thyroid gland has a storage reserve of approximately 3 weeks. The principal regulatory mechanisms
of the thyroid gland are the hypothalamic-pituitary-thyroid control system and the intrathyroidal regulatory systems. TRH is
produced by the superoptic and paraventricular nuclei within the hypothalamus and passes down their axones. Following
secretion into the hypophyseal portal blood systems TRH passes to the pituitary and induces stimulation of TSH secretion.

28. Correct statements about thyroid function tests include which of the following?
A. Contraceptive pills and pregnancy increase the amount of thyroxin-binding globulin (TBG), and, consequently, the total
T 4 level.
B. Anticonvulsive medications and chronic debilitating illnesses decrease the amount of TBG and, consequently, the total T
4 level.
C. Intravenous pyleography can lower the rate of active iodine uptake by the thyroid.
D. A triiodothyronine (T 3) suppression test that demonstrates nonsuppressibility of thyroid function is compatible with the
diagnosis of Graves' disease, toxic adenoma, or functioning carcinoma.
E. An increased serum cholesterol level in a hypothyroid patient indicates a thyroid cause.
Answer: ABCD

DISCUSSION: Hormone binding proteins are the principal intravascular factors influencing total hormone concentration.
Various factors may cause changes in the concentration of TBG. Contraceptive pills and pregnancy increase the amount of
TBG, while anticonvulsive medications and chronic debilitating disease may decrease the amount of TBG. Administration
of excess amounts of iodine may lower the rate of active iodine uptake in the thyroid gland. The thyroid suppression test is
based on the principle that administration of thyroid hormone does not suppress the patient's thyroid function when normal
homeostatic mechanisms are disrupted. Thyroid function is nonsuppressible in hyperthyroidism or in the presence of thyroid
hormone–secreting tumors. Serum cholesterol, deep tendon reflex time, and cardiovascular functions can be measured to
reflect hypothyroidism and hyperthyroidism but do not give an indication of the etiology of hypothyroidism.

29. Hyperthyroidism can be caused by all of the following except:


A. Graves' disease.
B. Plummer's disease.
C. Struma ovarii.
D. Hashimoto's disease.
E. Medullary carcinoma of the thyroid.
Answer: E

DISCUSSION: Common types of hyperthyroidism include diffuse toxic goiter (Graves' disease, named after the Dublin
physician Robert Graves who described it in 1835 but known since its original description by Parry in 1786 and described
by von Basedow in 1840) and toxic adenoma or toxic multinodular goiter (Plummer's disease). Uncommon causes include
thyrotoxicosis factitia, functioning metastatic thyroid carcinoma, trophoblastic tumors that secrete human chorionic
gonadotropin (having thyroid-stimulating properties), inappropriate secretion of thyrotropin by pituitary tumors, struma
ovarii, iodide-induced hyperfunction, and thyroiditis.

30. Which of the following is true about the use of radioiodine to treat hyperthyroidism?
A. If hyperthyroidism is secondary to radioiodine use, it will occur within 2 years of treatment.
B. There is a markedly increased risk of future thyroid cancer following radioiodine therapy.
C. The risk of leukemia following radioiodine therapy is approximately 10%.
D. Mutation abnormalities occur in 15% of fetuses in utero following internal treatment of the mother with radioiodine
during pregnancy.
E. Radioiodine may pass through the placenta and lactating breast to produce hypothyroidism in a fetus or infant.
Answer: E

DISCUSSION: Hypothyroidism secondary to radioiodine therapy increases in frequency with time to 85% after 5 years.
There has been no demonstration of an increase in malignancies following radioiodine therapy. Radioactive iodine can cross
the placenta and lactating breast to produce hypothyroidism in a fetus or a nursing infant and is, therefore, contraindicated.
31. Arrange the following complications of thyroid surgery (bilateral subtotal thyroidectomy) in decreasing order of
incidence in patients with Graves' disease.
A. Laryngeal nerve paralysis.
B. Hypoparathyroidism.
C. Hypothyroidism.
D. Recurrent hyperthyroidism.
Answer: CDAD

DISCUSSION: The incidence of recurrent disease is inversely related to the incidence of hypothyroidism and is 1% to 5%.
Within 1 to 2 years, hypothyroidism may develop in 5% to 50% of patients (with a slight additional increase in subsequent
years). The associated morbidity—related primarily to damage to the recurrent laryngeal nerves and parathyroid glands—is
estimated to be 0.5% to 3.0%.

32. The most common cause of goitrous hypothyroidism in adults is:


A. Graves' disease.
B. Riedel's thyroiditis.
C. Hashimoto's disease.
D. de Quervain's thyroiditis.
Answer: C

DISCUSSION: Hashimoto's disease was first described in Japan by Hakaru Hashimoto in 1912 and is the best-known of the
immunologic thyroid diseases. It is the most common cause of goitrous hypothyroidism in adults and of sporadic goiter in
children. The incidence is 0.3 to 1.5 cases per 1000 population per year and it is 10 to 15 times more common in women
than in men, with the highest incidence in the group aged 30 to 50 years.

33. Therapy for Hashimoto's disease includes:


A. Radioiodine.
B. Antithyroid medications.
C. Subtotal thyroidectomy.
D. None of the above.
Answer: D

DISCUSSION: There is no specific treatment for Hashimoto's disease. Patients are usually followed medically, and
replacement therapy with T 4 is begun in patients with hypothyroidism that is symptomatic or associated with a goiter that is
causing pressure symptoms. Early initiation of thyroid hormone therapy has been recommended by many to prevent further
thyroid enlargement and reduce the risk of myxedema, especially in postpartum patients. Surgical reduction of goiter should
be performed if severe pressure symptoms that have not responded to corticosteroid therapy are present. This usually
consists of subtotal thyroidectomy. Biopsy to rule out malignancy in nodules suspicious for thyroid carcinoma (usually
papillary) or lymphoma is indicated. If carcinoma is suspected, lobectomy should be performed, and if frozen section
demonstrates carcinoma, subtotal or total thyroidectomy should be performed.

34. Indications for surgical thyroidectomy for Graves’ disease include which of the following?

a. Ocular involvement
b. Symptomatic large goiter
c. Women of childbearing age
d. Concomitant thyroid nodule
e. All of the above
Answer: b, c, d

Antithyroid drugs are the initial therapy in most patients with Graves’ disease, either as a definitive therapy or in preparation
for 131I therapy or surgical ablation. Because of the high failure rate of long-term treatment with thionamides, the use of
these drugs as definitive treatment has decreased. Drawbacks include the important 0.5% incidence of agranulocytosis and a
recurrence rate as high as 43% during the first year when the drug is stopped. Five years after treatment only 25% of
patients remain in remission. Unlike other definitive treatments, hypothyroidism does not occur as a result of thionamide
treatment if an appropriate dosage is used. 131I has been used as definitive treatment for patients with Graves’ disease for
many years with predictable and long-lasting good results in most patients. It has few, if any, serious side effects. It is
ablative to the thyroid gland and hypothyroidism is a nearly inevitable result of effective therapy, although it may take years
to become clinically apparent. About 70% of patients treated with 131I are hypothyroid within 10 years of treatment. The
risk of recurrence of hyperthyroidism after an initial response is less than 5%. Most adult patients in the United States are
treated with 131I as definitive treatment for Graves’ disease. Exceptions are women in the childbearing years where a
subsequent fetus would be affected, patients with concomitant thyroid nodules where carcinoma is a concern, those with
extremely large glands and, increasingly, those who are opposed to 131I therapy. Thyroidectomy is an important alternative
in selected patients with Graves’ disease. Although controversial in the past, it now appears that ocular involvement does not
respond more favorably to thyroidectomy than to 131I ablation.

35. Which of the following statements regarding anatomic relationships of the thyroid gland are true?

a. The middle thyroid artery is intimately related to the superior laryngeal nerve
b. The superior thyroid artery is usually the first branch of the external carotid artery
c. Thyroidea ima arteries are found in approximately 20% of individuals
d. The parathyroid glands may lie within the pretracheal fascia
Answer: b, d

The thyroid gland is a vascular organ supplied by four main arteries: two superior and two inferior. The superior thyroid
artery usually arises as the first branch of the external carotid artery just above the bifurcation of the common carotid artery.
The superior thyroid artery descends medially on the surface of the inferior pharyngeal constrictor muscle to divide into an
anterior and posterior branch at the apex of the thyroid lobe on its anteromedial surface. Its relation to the external branch of
the superior laryngeal nerve is important during thyroid lobectomy. The inferior thyroid arteries usually arise from the
thyrocervical trunks and ascend behind the carotid sheath before passing downward and medial to enter the thyroid gland at
its middle portion. There are no arteries directly entering the lower poles from below with the exception of a thryoidea ima
artery that may replace an absent inferior artery. Thyroidea ima arteries arise from either the innominate artery or aorta in
1% to 4% of individuals, entering the lower surface of the isthmus after coursing on the trachea.
The pretracheal fascia is referred to as the thyroid sheath and it varies in consistency and completeness among individuals.
Posteromedially, it is usually condensed and firmly attaches the thyroid gland to the upper two or three tracheal rings and
the cricoid cartilage. The superior parathyroid gland may lie between the sheath and the thyroid capsule, within the sheath,
posterior to it in a potentially open plane, or occasionally anterior to the sheath. The inferior parathyroid gland likewise may
be within the sheath, particularly when the gland is adherent to the lower pole of the thyroid.

36. Radioactive iodine is effective treatment for metastatic lung disease for which of the following thyroid neoplasms?

a. Hürthle cell carcinoma


b. Papillary carcinoma
c. Follicular carcinoma
d. Medullary carcinoma
e. Anaplastic carcinoma
Answer: b, c

Radioactive iodine is used only in patients who have differentiated thyroid carcinomas. It is of no value in the treatment and
follow-up of patients with Hürthle cell, medullary or anaplastic carcinomas. Most papillary carcinomas are capable of
taking up radioactive iodine. Most papillary carcinomas in patients under 50-years of age do so, providing that the patient
has had a total thyroidectomy and there is no normal thyroid tissue to compete for the 131I. About 20% of all papillary
carcinomas do not trap sufficient iodine for imaging or therapy. These are usually patients with papillary carcinoma
variants: a tall cell variant of papillary carcinoma, insular carcinoma, or clear cell carcinoma.
Nearly all metastatic follicular carcinomas retain the ability to trap 131I sufficiently for imaging and for therapy. Even well-
differentiated papillary and follicular carcinoma cannot compete successfully for 131I with normal thyroid tissue and unless
this has been removed or subsequently ablated with an initial dose of 131I, many metastases cannot be detected or treated.

37. The types of thyroiditis that can cause abnormalities of surgical significance are which of the following?

a. Chronic lymphocytic thyroiditis (Hashimoto disease)


b. Riedel struma
c. Acute (viral) thyroiditis
d. Granulomatous (subacute) thyroiditis
Answer: a, b, d

There are three types of thyroiditis that can cause thyroid abnormalities of surgical significance. The most common is
chronic lymphocytic (Hashimoto) thyroiditis, an autoimmune disease that can occur in any age group. Occasionally,
Hashimoto disease causes unilateral thyroid enlargement that simulates malignancy. The rarest form of thyroiditis is Riedel
struma which can mimic a diffuse thyroid carcinoma because of the fibrotic infiltrative process that results. Hashimoto
thyroiditis is associated with reduced functional capacity of the thyroid which increases TSH secretion, and a goiter
develops. Because of the associated fibrosis, a nodular goiter or neoplasm is suggested. Thyroidectomy may be indicated for
treatment of a solitary nodule, particularly if it is cold, suspicious, definitely malignant or solid, and fine needle aspirate is
indeterminate.
Granulomatous, DeQuervain, and subacute thyroiditis are terms that refer to a disease that usually occurs in young women
within weeks of an upper respiratory or other viral infection. The disease is usually self-limited, but may persist for several
months longer. In unusual patients, the disease may be confined to one lobe and result in a firm, slightly tender mass
suggesting carcinoma. Lobectomy may be indicated to rule out the presence of malignancy. Total thyroidectomy may be
considered for persistent, painful thyroiditis after months of steroid therapy have failed.
Goiter with a woody or fibrous component involving the adjacent strap muscles and carotid sheaths is referred to as Riedel
struma. It is rare and the cause is not known. It is associated with other types of fibrotic processes including retroperitoneal
fibrosis, sclerosing cholangitis, and fibrosing mediastinitis. Although considered self-limited, the process may be associated
with considerable morbidity as a result of localized pain and compression of adjacent structures such as the airway.
Occasionally, tracheostomy is required. Airway compression may also require open biopsy resection of the isthmus with as
much as the fibrosis as possible without endangering the recurrent laryngeal nerves.

38. The principal blood supply to the parathyroid glands is which of the following?

a. Superior thyroid arteries


b. Inferior thyroid arteries
c. Thyroidea ima arteries
d. Parathyroid arterial branches directly from the external carotid artery
e. Highly variable
Answer: b

The principal blood supply to both parathyroid glands is the inferior thyroid artery. Parathyroid glands invariably have a
single end artery supplying them, and if the main trunk of the inferior thyroid artery is ligated during thyroidectomy, there is
no collateral blood supply to maintain their viability. It is preferable to divide only the branch of the inferior thyroid artery
medial to those that supply either of the parathyroid glands. This requires individual clamping of smaller vessels under the
thyroid sheath as these vessels penetrate into the thyroid capsule. Ligation of the main trunk of the inferior thyroid artery
was commonly used for bilateral subtotal thyroidectomy in the past. It did not routinely cause hypoparathyroidism only
because enough collateral blood supply was maintained to each end artery to one or more parathyroid glands. This is to be
avoided.

39. Which of the following statements regarding papillary thyroid carcinoma are true?

a. Seventy to 80% of new cases of thyroid carcinoma in the United States are of the papillary type
b. Total ipsilateral lobectomy and isthmus resection are adequate therapy for minimal thyroid carcinoma
c. Microscopic evidence of multicentric disease is present in 70% to 80% of cases
d. Nearly all patients less than 15 years of age have metastatic disease in local lymph nodes
Answer: a, b, c, d

Seventy to 80% of the 11,000 new patients with thyroid carcinoma diagnosed annually in the United States have papillary
carcinoma. Papillary carcinomas of the thyroid include minimal thyroid carcinoma, intrathyroidal, and extrathyroidal
(invasion through the true thyroid capsule) disease. Minimal thyroid carcinoma refers to those papillary carcinomas that are
less than 1 cm in diameter and not associated with any clinically apparent lymph node metastases. In contrast to clinically
significant papillary carcinomas, these are common and are found in 2% to 13% of adult thyroid glands serially sectioned
after autopsy studies of individuals who have died from other causes. For tumors between 0.5 and 1 cm, a total lobectomy
and isthmus resection are satisfactory treatment.
Most clinically significant papillary carcinomas are 1 to 4 cm in diameter and are contained within the thyroid capsule.
Multicentricity is relatively common and can be found on gross sectioning of the thyroid gland in 20% to 30% of cases.
Furthermore, after serial sectioning of the entire thyroid gland in patients with papillary carcinoma, microscopic foci are
found in 70% to 80%. Local cervical lymph node metastases are found in about 30% of all patients with papillary
carcinoma. The presence of lymph node metastases does not correlate as closely to the size of the tumor as it does to the age
of the patient. The younger the patient, the greater the likelihood of metastatic lymph node involvement. Nearly all patients
under 15 years of age have involved metastatic lymph nodes. The presence or absence of lymph node metastases in patients
with intrathyroidal primary papillary carcinomas does not appear to have an appreciable effect on long-term survival if
distant metastases are not present at the time of initial treatment.

40. A 30-year-old female presents for evaluation of a palpable thyroid nodule. Technetium-99m (99mTc) scan
demonstrates a single cold nodule. The differential diagnosis includes which of the following?

a. Carcinoma
b. A nonfunctioning adenoma
c. A thyroid cyst
d. A colloid nodule
e. An autonomous nodule
Answer: a, b, c, d

Radioisotope scanning measures the functional activity of the thyroid gland and maps its correlation with physical findings.
For routine scanning, 131I scanning has been replaced by either 123I or 99mTc. Because of the lower radiation exposure,
cost and easy use, 99mTc is preferentially used. The differential diagnosis of a solitary nonfunctioning nodule includes
carcinoma, colloid nodule, nonfunctioning adenoma, and cyst. The prevalence of carcinoma ranges from 5% to 20% in cold
nodules and such lesions require further evaluation. If the scan detects other nonfunctioning areas in addition to the palpable
nodule, the gland is at low risk for carcinoma because most patients with this finding have a multinodular goiter. A solitary,
discrete area of increased activity is more often found in a young patient with an otherwise normal thyroid gland. Multiple
hot spots are typically found in an older patient with a multinodular goiter.
A functioning solitary nodule that is independent of TSH is considered an autonomous nodule and can be the cause of
hyperthyroidism. It is hot on scintigraphic scan. The presence of a cold nodule is insufficient information to determine a
treatment plan. Although nearly all carcinomas are cold, most cold nodules are benign. Fine needle aspiration cytology is
considered the most reliable means of evaluation for the diagnosis of thyroid nodules that are nonfunctional or
hypofunctional by nuclide scan.

41. Which of the following pharmacologic agents can be used in the treatment of thyrotoxicosis to block the
production of thyroid hormone?

a. Propylthiouracil
b. Propranolol
c. Methimazole
d. Carbimazole
e. Iodine
Answer: a, c, d, e

A number of substances interfere with normal production of thyroid hormone by blocking one of the relevant enzymatic
steps. Iodine was the first effective drug to be used in the treatment of thyrotoxicosis. It can block the organification and
coupling steps in thyroid hormone synthesis as well as prevent the release of thyroid hormone. Furthermore, iodine in large
doses probably inhibits the ability of TSH to stimulate cyclic AMP release at the follicular cell membrane.
Commonly used antithyroid drugs are propylthiouracil (PTU), methimazole (Tapazole) and carbimazole. Carbimazole is
widely used in Great Britain, only PTU and methimazole are commonly used in the United States. PTU interferes with the
incorporation of iodine into the tyrosine residues of thyroglobulin, thus preventing oxidation of iodide to iodine. It also
inhibits the peripheral conversion of T4 to T3. Although both PTU and methimazole are thionamides, methimazole does not
have this peripheral effect, making PTU the preferred drug for patients with thyroid storm. About 3% of patients taking PTU
demonstrate at least one side effect during the first 3 months of therapy whereas the prevalence with methimazole is about
7%. These range from minor skin rashes to agranulocytosis that can be irreversible.
b-adrenergic antagonists have been used in the treatment of hyperthyrodism. Propranolol is the most widely used of these
drugs. However, b-blockade does not alter thyroid function per se. Rather, its effect is to provide symptomatic relief of
hyperthyroidism because of interference with the action of thyroid hormones at the cellular level.

42. A 45-year-old woman has a solitary, nonfunctioning thyroid nodule and fine needle cytology is nondiagnostic.
Which of the following is the initial surgical procedure of choice?

a. Total extracapsular thyroidectomy


b. Subtotal thyroid lobectomy and resection of the isthmus
c. Total extracapsular thyroid lobectomy, resection of the isthmus, and modified unilateral neck dissection
d. Total extracapsular thyroid lobectomy and resection of the isthmus
Answer: d

Total extracapsular thyroid lobectomy and isthmus resection is the procedure of choice when a decision has been made to
surgically remove a thyroid nodule. The entire lobe with the isthmus is submitted for frozen-section pathologic examination
if fine needle aspirate has not already resulted in a definitive diagnosis of carcinoma. In performing total lobectomy, both
parathyroid glands are carefully preserved with their blood supply. This is done in the event that total thyroidectomy is
necessary if either the frozen or permanent histologic sections confirm the presence of thyroid carcinoma. Total lobectomy
offers the best opportunity for accurate histologic diagnosis and is associated with the lowest incidence of complications
when the need for reoperation is considered. In one experience, 800 consecutive cases of total unilateral lobectomy were
performed for benign or malignant nodules suspected of cancer, and no permanent recurrent laryngeal nerve palsies
occurred. Primary total lobectomy is safer than a partial lobectomy followed by resection of the residual lobe after a delayed
diagnosis of malignancy. Reoperation to complete a lobectomy is associated with a greater risk to both recurrent laryngeal
nerve and the parathyroids on the ipsilateral side.
Although there is controversy as to whether a total lobectomy and isthmus resection or a total thyroidectomy is the best
definitive operation for unilateral papillary carcinoma, a subtotal lobectomy is universally considered an inadequate
operation. A definitive cancer operation can be accomplished with one procedure in 80% of the cases when a skilled thyroid
pathologist is available for frozen-section interpretation.

43. Which of the following statements regarding fine needle aspiration cytology of a thyroid nodule are true?

a. It differentiates neoplastic and nonneoplastic nodules in most cases


b. It does not allow differentiation of papillary, medullary and anaplastic carcinoma
c. It cannot differentiate malignant and benign follicular or Hürthle cell neoplasms
d. It is not recommended when a patient has a history of head and neck radiation
Answer: a, c, d

When interpreted by a skilled cytologist, fine needle aspiration is highly accurate and is considered the preferred method of
selecting patients with thyroid nodules for surgery. Nearly 80% of patients with thyroid nodules were spared surgical
exploration as a result of such studies in one report. Because of the risk of false-negative diagnoses (10%), advocates of this
technique emphasize the importance of clinical judgment in addition to the cytologic study in selecting operative candidates.
In most cases, fine needle aspiration cytology enables the pathologist to distinguish nonneoplastic from neoplastic nodules
and to identify the type of malignant tumor. Papillary, medullary, and anaplastic carcinoma all have a typical cytologic
appearance. Cytologic studies cannot differentiate malignant from benign follicular or Hürthle cell neoplasms. In this case, a
definitive diagnosis depends on histologic examination of the entire excised tumor.
Fine needle aspiration has dramatically reduced the number of diagnostic surgical operations for benign lesions in centers
where it is used extensively. In patients with thyroid nodules and a history of previous head and neck radiation, operation is
generally recommended regardless of cytologic findings. In these patients, both benign and malignant lesions may develop
and the chances of sampling error are considerable.

44. Hyperthyroidism results from all of the conditions noted below. Of the following which commonly require surgical
management?

a. Graves’ disease
b. Struma ovarii
c. Functioning metastatic thyroid carcinoma
d. Toxic diffuse goiter
e. Single toxic thyroid nodule
Answer: a, d, e

Hyperthyroidism is associated with clinical manifestations related to an excess of thyroid hormone. There are three causes
of primary concern to the surgeon. Graves’ disease, or toxic diffuse goiter, is most common, accounting for more than 80%
of all patients with hyperthyroidism. The other two relevant causes of hyperthyroidism are toxic nodular goiter and a single
toxic nodule. Common causes of hyperthyroidism that rarely require surgery are postpartum thyroiditis, iodine-induced
hyperthyroidism, self-administered or iatrogenic hyperthyroidism, struma ovarii, functioning metastatic carcinoma and
several rare forms of thyroiditis.

45. Which of the following statements regarding medullary carcinoma of the thyroid are true?

a. Approximately 75% of all cases are hereditary


b. The overall 10-year survival rate is less than 10%
c. Medullary carcinoma of the thyroid is associated with both multiple endocrine neoplasia IIa (MEN IIa) and
multiple endocrine neoplasia IIb (MEN IIb) syndromes
d. Prophylactic total thyroidectomy is recommended for MEN IIa and MEN IIb patients after the age of 10 years
Answer: c

Medullary carcinoma of the thyroid (MCT) accounts for about 7% of all malignant tumors of the thyroid. It is familial in
20% to 30% of all cases and its secretion of a biologic marker (calcitonin) allows detection of its presence with the tumor is
too small to palpate. MCT appears in three clinical settings. The first is a sporadic tumor, usually in patients 30 years or
older. Second, MCT occurs as a component of the MEN IIa syndrome with or without adrenal medullary disease
(pheochromocytoma) or hyperparathyroidism. A family history of thyroid carcinoma with or without pheochromocytoma is
invariably present. MCT in the MEN IIa syndrome is always bilateral and multicentric and arises from C-cell hyperplasia.
MCT usually does not develop before age 12 and is almost always clinically apparent before age 30.
Lastly, MCT is found as a component of the MEN IIb syndrome without or without bilateral adrenal medullary disease and
always with the facies and autonomic nervous system dysplasia expressed as ganglioneuromatosis from the lips to the anus.
These patients often have a marfanoid habitus and skeletal deformities as well. The MEN IIb syndrome occurs as a sporadic
mutation, but its familial occurrence is becoming more common because patients are surviving now long enough to
reproduce. MCT or its precursor, C-cell hyperplasia, develops by age 2 in MEN IIb and is always bilateral and multicentric.
As a result of its early appearance and late detection, the disease has usually been more advanced when treatment has been
instituted. Minimal treatment of MCT is total thyroidectomy. The overall prognosis for 10-year survival is approximately
50%, although the tumor growth rate in individual patients has shown great variability. Patients with the MEN IIb disease
require total thyroidectomy as soon as the syndrome is recognized, preferably by the age of 2 years. In familial cases, the
characteristic findings are sufficient even without calcitonin testing to justify operation. If the diagnosis is not made until
adolescence or later, both central compartment and lateral node involvement require neck dissection for definitive treatment.

46. Which of the following statements regarding thyroid physiology are true?

a. Normally about 20% of T3 is secreted directly from the thyroid gland


b. The thyroid gland is the only endogenous source of T4
c. Excess thyroid hormone results in an increase in the number of ATP-dependent sodium pumps on the cell
membrane
d. The majority of thyroid hormone in circulating plasma is bound to albumin, thus limiting the availability of the
metabolically active form
Answer: a, b, c

The thyroid is the only endogenous source of T4, whereas most T3 is produced by the peripheral conversion of T4. This
takes place in the liver, muscle, kidney and anterior pituitary. Under normal circumstances, only about 20% of T3 is secreted
directly from the thyroid gland. In some thyroid diseases (e.g., Graves’ disease and toxic nodular goiter), the proportion of
T3 secreted directly by the thyroid may be markedly increased.
Once thyroid hormones are released into the circulation, they are bound to thyroid-binding globulin (85%), albumin (10%)
and transthyretin (prealbumin). These binding proteins allow the thyroid hormones to remain soluble in plasma, contributing
to systemic distribution to various target-cell populations. A limited amount of thyroid hormones circulates freely in the
plasma in metabolically active form (free T4, free T3).
At the cell membrane, an excess of thyroid hormone results in an increased number of ATP-dependent sodium pumps, thus
increasing resting energy expenditure and oxygen consumption. Thyroid hormone also facilitates the transport of glucose
and amino acids across the cell membrane. In addition, T3-induced mitochondrial oxidation of substrate results in increased
ATP production.

47. The definitive treatment of choice for toxic multinodular goiter is?

a. Total thyroidectomy
b. Bilateral subtotal thyroidectomy
c. Unilateral total lobectomy on the side of dominant disease
d. 131I treatment
Answer: b

Hyperthyroidism in the patient with a multinodular goiter usually develops in women after age 50 but is seen occasionally
in younger patients. Most patients have had a nontoxic nodular goiter for many years. Preferred treatment for most patients
with toxic multinodular goiters is thyroidectomy after adequate preparation renders the patient euthyroid. 131I may be an
alternative in selected poor-risk patients with goiters that are not causing airway compression. Although 131I can be used to
treat the hyperthyroidism, larger and often repeated doses of 131I may be required. 131I does not significantly reduce the
goiter size and may, because of radiation-induced thyroiditis, cause acute enlargement. This may be hazardous in the patient
with some degree of preexisting airway compression. Any airway symptoms, particularly in patients with substernal goiters,
should be considered strong contraindications to the use of 131I.
Standard surgical treatment of toxic nodular goiter has consisted of bilateral subtotal thyroidectomy. Remnant size is not as
important as the excision of all autonomous nodules. Because thyroid replacement or suppression is used routinely to
prevent recurrence of goiter when a subtotal resection is done, the risk of hypothyroidism is not a consideration in
determining remnant size. Alternative procedures are total lobectomy with isthmus resection and contralateral subtotal
lobectomy, or total thyroidectomy. The latter is not demonstrably superior and may have more technical complications.

48. A 50-year-old male has undergone an ipsilateral thyroid lobectomy and isthmus resection for what appeared on
frozen section to be a benign nodular lesion 2.0 cm in diameter. Seventy-two hours later the final pathology returns and the
diagnosis is a high-grade angioinvasive follicular carcinoma. What do you recommend?

a. 99mTc bone scan to rule out occult bone metastases


b. Ipsilateral radical neck dissection
c. Total thyroidectomy
d. Observation with sequential 131I scans every 3 months
Answer: c

Follicular carcinoma accounts for about 10% of all new carcinomas of the thyroid. Most patients are those with minimal
invasion of the capsule or vessels within the neoplasms. Such tumors are seldom diagnosed definitively by either needle
aspiration cytology or by frozen-section diagnosis at the time of lobectomy. Most frequently, the diagnosis is made after the
study of permanent sections. Microinvasive encapsulated follicular carcinomas are rarely associated with metastatic lymph
nodes and distant metastases involving bone are also rare at the time of diagnosis.
Angioinvasive follicular carcinomas are usually large and frequently show venous invasion of perithyroidal and lateral neck
veins at the time of diagnosis. They may have already metastasized to different sites, most frequently the bone. These
tumors are often diagnosed at the time of operation because of these characteristics. Most patients younger than 40 do well,
but patients older than 50 years have a guarded prognosis. Follicular carcinomas of the thyroid are treated by total
thyroidectomy. Lymphatic dissections are not usually required because only about 5% of all patients have lymphatic
involvement. The most effective therapy for bone or pulmonary metastases is radioactive iodine. The controversy in the
surgical management of follicular carcinoma is whether patients with low-grade encapsulated neoplasms diagnosed after
total lobectomy by permanent section evaluation should be treated with total thyroidectomy. In patients with smaller
follicular neoplasms found malignant as determined by microinvasion of the capsule, completion thyroidectomy is usually
not done; instead patients are observed closely after performing a 99mTc bone scan to rule out occult bone metastases.
Patients with large or high-grade angioinvasive lesions require total thyroidectomy.

49. In 1990 a National Institutes of Health Consensus Development Conference reviewed the available evidence
regarding the management of asymptomatic primary hyperparathyroidism. The panel concluded that surgical intervention
was indicated for which of the following indications?
a. Age less than 50 years of age
b. Reduced creatinine clearance
c. Presence of kidney stone(s) (as detected by abdominal radiograph)
d. Substantial reduced bone mass as determined by direct measurement
e. Markedly elevated 24-hour urinary calcium excretion
Answer: a, b, c, d, e

All of the indications listed above are considered indications for operative treatment of asymptomatic patients with primary
hyperparathyroidism. Their conclusions regarding operative indications are summarized in Table 57-8. The NIH Consensus
Development Conference mandated close (every 6 months) follow-up for patients with known primary hyperparathyroidism
not treated by operation. In addition, surgery was agreed upon for those patients in whom medical surveillance was neither
desirable nor suitable, such as when the patient requests surgery, consistent follow-up is unlikely, co-existent illness
complicates management, or if the patient is young (< 50 year of age).
In one recent study of a group of 142 asymptomatic patients followed without operation, after 10 years, more than 20% of
the patients had required surgery for an increase in serum calcium to greater than 11 mg/dL or for specific complications
attributable to the disease. Another 20% were lost to or declined follow-up. The remainder either died of unrelated causes or
had persistent asymptomatic disease. This remains an area of considerable controversy. The best available recommendations
are derived from this October 1990 NIH Consensus Development Conference.

50. Which of the following statements regarding medullary carcinoma of the thyroid are true?

a. Bilateral medullary carcinoma of the thyroid occurs in every patient with multiple endocrine neoplasia 1, 2a or 2b
b. Medullary carcinoma of the thyroid accounts for approximately one-third of all thyroid malignancies
c. 20% of medullary carcinoma of the thyroid cases are familial
d. Provocative testing with pentagastrin and calcium infusion for family members at risk for medullary carcinoma of
the thyroid is diminished in importance by the recent development of specific genetic testing
Answer: a, c, d

Medullary thyroid carcinoma accounts for about 10% of all thyroid malignancies, and 20% of cases occur in the familial
setting of MEN 2a, MEN 2B, or familial non-MEN medullary thyroid carcinoma. It is usually the first tumor that develops
in these patients and typically occurs in the second or third decade of life. Tumors are virtually always bilateral and develop
in multiple areas of the middle and upper portions of the thyroid lobe. Detection has historically been based on elaboration
of calcitonin, particularly in response to provocative testing by the potent secretagogues calcium and pentagastrin. Patients
with medullary carcinoma of the thyroid have striking increases in plasma calcitonin levels after provocative testing, which
allows them to be readily identified. By employing provocative testing in kindred members at risk for disease, medullary
carcinoma of the thyroid was diagnosed at a preclinical stage, and a greater percentage of these patients were cured by
surgical therapy. With genetic testing now available, early treatment of medullary thyroid cancer should be possible for all
affected people, to hopefully enhance the number of people cured of this cancer.
The MEN 1, 2a and 2b syndromes are inherited in an autosomal dominant fashion with complete penetrance but variable
phenotype. Bilateral medullary carcinoma of the thyroid occurs in every affected patient.

51. While awaiting surgery on a HMO waiting list, a 50-year-old female with primary hyperparathyroidism is admitted
to the hospital with oliguria, confusion, nausea and vomiting, muscle weakness and a serum calcium of 13.5 mg/dL. Of the
treatment options below, which one is the most appropriate?

a. Administer 1 gm of hydrocortisone STAT


b. Begin an IV EDTA (chelating agent) infusion STAT
c. Administer IV Mithramycin and calcitonin concurrently and proceed to ICU for cardiac monitoring
d. Begin a normal saline infusion at 2X maintenance volume followed by 1mg/kg furosemide IV
e. None of the above
Answer: d

Patients with marked hypercalcemia or severe symptoms should be admitted to the hospital for careful observation and
monitoring. The mainstay of therapy is intravenous hydration, preferably with normal saline in sufficient quantities to
maintain the urine output above 100 mL/h. These patients are often dehydrated before therapy, and fluid can be
administered intravenously at a rate of 200 mL/h. Caution must be exercised in older patients who might have marginal
cardiac reserve. The diuretic furosemide also increases excretion of sodium and calcium but should not be employed until
the patient is well hydrated. Saline diuresis is usually effective when the hypercalcemia results from hyperparathyroidism or
from a benign cause. In contrast, the hypercalcemia of malignancy may produce severe symptoms associated with
extremely high serum calcium levels that are difficult to control. In this setting, a variety of other measures may be
considered (see Table 57-3). Some of the agents used to treat hypercalcemia have significant toxicity and require close
monitoring. Calcitonin is a fairly weak hypocalcemic agent, but it acts rapidly and is relatively less toxic. Glucocorticoids
may be particularly efficacious in patients with sarcoidosis and other granulomatous diseases. Mithramycin has proved
useful in patients with hypercalcemia of malignancy, but it has a substantial cumulative toxicity (thrombocytopenia,
hepatotoxicity, and nephrotoxicity). Intravenous phosphates and chelating agents have largely been abandoned because of
their severe toxicity.

52. Which of the following conditions are associated with hypercalcemia?

a. Hyperthyroidism
b. Thiazide diuretics
c. Terminal ileal resection
d. Breast cancer
e. All of the above
Answer: a, b, d

Multiple causes of hypercalcemia include the following:


Malignancy
Vitamin A or D intoxication
Thiazide diuretics
Hyperthyroidism
Milk-alkali syndrome
Sarcoidosis and other granulomatous diseases
Familial hypocalciuric hypercalcemia
Immobilization
Paget’s disease
Lithium therapy
Addisonian crisis
Idiopathic hypercalcemia of infancy
Hyperthyroidism is associated with increased bone reabsorption. Often, the plasma PTH is low, and a history of other
thyrotoxic symptoms can be elicited. Thiazides may increase serum calcium to a mild degree, primarily through
hemoconcentration. Serum phosphate may also be depressed. It often takes several weeks for the hypercalcemia to resolve
after the medication is discontinued. Terminal ileal resection is not associated with hypercalcemia, indeed hypocalcemia
may develop with short gut syndrome. While ileal resection is a predisposing condition for the development of calcium
oxalate kidney stones, this is the result of alterations in oxalate metabolism. Patients with solid tumors, such as lung
carcinoma, breast carcinoma, squamous cell carcinoma of the head, neck, esophagus or female genital tract or renal cell
cancer account for three quarters of all cases of humoral hypercalcemia of malignancy.

53. A 40-year-old male undergoes an apparently uneventful total thyroidectomy for follicular carcinoma of the thyroid.
48 hours later he develops circumoral numbness, followed by laryngospasm, and then has a generalized seizure. Of the
following, which is the first priority?

a. Proceed to OR for exploration of the operative site


b. Administer 25 ml of 10% calcium gluconate intravenously
c. Obtain a serum magnesium measurement and administer intravenous magnesium chloride STAT
d. Obtain a CT scan of the head to evaluate the possibility of brain metastases
Answer: b

Postoperative hypoparathyroidism commonly develops after total thyroidectomy for malignancy. Most patients undergoing
operation on the thyroid experience some alteration in serum calcium. The hypocalcemia is usually transient and is not
treated unless significant symptoms develop. The plasma calcium usually reaches its nadir at 48 to 72 hours after surgery
and then slowly returns to normal over 2 to 3 days. For these reasons, careful postoperative monitoring of serum calcium
levels is essential.
For acute symptomatic hypocalcemia, calcium should be administered intravenously. Usually 20 to 30 mL of 10% calcium
gluconate is infused over a 15 to 20 minute period and then 50 to 100 mL are administered over the next 12 hours in adults.
Calcium gluconate is less irritating to the veins than calcium chloride, and the calcium release is slower without the risk of
overcorrection. Serum magnesium should always be measured and hypomagnesemia should be corrected if present. This is
not the first priority for this patient. Symptoms should never be allowed to progress to the point demonstrated in this patient.
Postoperative routines include careful monitoring of the serum calcium until stable. There is no evidence that this patient
has a postoperative surgical problem such as a cervical hematoma. Laryngospasm and seizures are classical signs of
hypocalcemia. Lastly, the likelihood of metastatic brain disease is very low in this circumstance.

54. On routine life insurance screening, an asymptomatic 45-year-old male is found to have a serum calcium level of
12.5 mg/dL. Serum is obtained for immunoreactive parathyroid hormone level and this is 400 mEq/mL (normal range <
64mEq/mL). These findings are most consistent with a diagnosis of which of the following?

a. Primary hyperparathyroidism
b. Secondary hyperparathyroidism
c. Tertiary hyperparathyroidism
d. Ectopic hyperparathyroidism
Answer: a

Primary hyperparathyroidism develops spontaneously without apparent cause, but possibly in response to exogenous
stimuli. When the normal control of serum calcium is disturbed and there is increased autonomous production of PTH, the
state is referred to as primary hyperparathyroidism. In contrast, secondary hyperparathyroidism occurs when there is a
defect in mineral homeostasis that leads to a compensatory increase in parathyroid function. This occurs most commonly in
response to renal disease, but may also develop as a consequence of the hypocalcemia associated with some diseases of the
gastrointestinal tract, bone or other endocrine organs. Occasionally, with prolonged secondary stimulation, the
hyperfunctioning glands are no longer physiologically responsive to an increased ionized calcium. This rare, relatively
autonomous state is referred to as tertiary hyperparathyroidism and develops most commonly after renal transplantation
when the defect in calcium homeostatis is corrected. The numerical values for calcium and PTH here are consistent with
primary hyperparathyroidism.

Relation between serum immunoreactive parathyroid hormone (iPTH) and serum calcium in patients with hypoparathyroidism,
pseudohypoparathyroidism, ectopic hyperparathyroidism, and primary, secondary, and tertiary hyperparathyroidism. GP1M, guinea pig antiserum 1M.
(After Clark OH, Way LW. Thyroid and parathyroid. In: Current surgical diagnosis and treatment, ed 8. Norwalk, CT, Appleton & Lange, 1989:249)

55. A 50-year-old female is referred for treatment of a serum calcium of 11.5 mg/dL one year after resection of a right
superior parathyroid adenoma. The right and left inferior glands were normal at the initial neck exploration. The left
superior gland was not visualized. Which of the related statements below are true?

a. Recurrent postoperative hypercalcemia occurs in approximately 20% of patients with this clinical scenario
b. Localization studies via selective angiography are successful in 50% to 80% of these patients
c. The risk of permanent hypoparathyroidism is approximately 10% to 20% following reexploration in this setting
d. Surgical reexploration by an experienced endocrine surgeon has a success rate of less than 50% in this
circumstance
Answer: b, c
Persistent hyperparathyroidism occurs in less than 5% of patients after neck exploration for primary hyperparathyroidism by
an experienced surgeon. Most commonly, it is the result of a single diseased gland still remaining in the neck or in the
mediastinum. Recurrent disease develops after an interval of normocalcemia and may be the result of regrowth of diseased
tissue, implantation from a tumor broken at the initial procedure, or even recurrent parathyroid carcinoma. Documentation
of a correct initial diagnosis and review of the original operative and pathology reports are essential. It is generally agreed
that localization studies do have a place in the management of recurrent disease. Noninvasive methods are employed first,
and if these are unsuccessful in identifying the diseased gland, selective angiography and venous sampling for PTH are
employed. Selective angiography appears to be the most accurate technique, successfully localizing 50% to 80% of
parathyroid glands that cannot be detected by any other modality.
Surgical reexploration can be difficult. Generally the neck is explored first after which the mediastinum is examined if this
is unproductive. Surgical reexploration is successful in experienced hands in 60% to 80% of cases. There is, however, an
increased incidence of complications. Unilateral recurrent nerve injury occurs in 5% to 10% of patients and permanent
hypoparathyroidism occurs in 10% to 20% of patients postoperatively.

56. Which of the following statements regarding calcium and phosphate metabolism are true?

a. Parathyroid hormone excess produces a net increase in daily urinary calcium excretion
b. Calcitonin is essential for the normal control of calcium metabolism in adult humans
c. Parathyroid hormone is the single most important regulator of calcium and phosphate metabolism in humans
d. 1,25 dihydroxyvitamin D3 (calcitriol) is the active form of vitamin D in humans
Answer: a, c, d

The primary hormonal regulators of calcium and phosphate metabolism are parathyroid hormone (PTH), vitamin D and
calcitonin. Parathyroid hormone appears to be the single most important hormonal regulator of calcium and phosphate
metabolism in humans. It has direct effects on the skeleton and kidney and indirect effects on the intestine, mediated
through vitamin D. Sustained elevations of PTH stimulate osteoclasts and inhibit osteoblasts leading to absorption of
calcium from bone. In the kidney, PTH produces an increase in reabsorption at any given concentration of extracellular fluid
calcium, although excess secretion, because of the hypercalcemia, results in a net increase in daily urinary calcium
excretion.
Vitamin D3, or cholecalciferol, is produced normally by the action of sunlight on 7-dehydrocholesterol in the skin. It is then
hydroxylated in the liver (25 position) and kidney (1 position) to form the active 1,25 dihydroxyvitamin D3 (calcitriol). This
is the active form of Vitamin D in humans.
Calcitonin is a 32-amino acid protein produced by the parafollicular C cells of the thyroid. Total thyroidectomy, with
removal of all the C cells, is well tolerated, and it has been concluded that calcitonin is not essential for the normal control
of calcium metabolism in adult humans. Calcitonin does inhibit bone resorption and can produce hypocalcemia in
experimental animals. It also increases urinary calcium and phosphate excretion.

57. Multiple Endocrine Neoplasia (MEN) 2b is characterized by which of the following findings?

a. Medullary carcinoma of the thyroid, pheochromocytoma, mucosal neuromas, and a distinctive marfanoid habitus
b. Parathyroid hyperplasia, pancreatic islet cell tumors, and pituitary adenomas
c. Medullary carcinoma of the thyroid, pheochromocytoma, and parathyroid hyperplasia
d. Parathyroid carcinoma, pheochromocytoma and chronic pancreatitis
Answer: a

The multiple endocrine neoplasias are familial disorders typically characterized by predisposition to the development of
tumors of multiple endocrine organs. These disorders are all inherited in an autosomal dominant fashion, and the tumors
tend to be multicentric. The tumors may be benign or malignant and may occur metachronously or synchronously. MEN 1 is
characterized by the concurrence of parathyroid hyperplasia, pancreatic islet cell tumors, and pituitary adenomas. MEN 2a
consists of medullary carcinoma of the thyroid, pheochromocytoma, and parathyroid hyperplasia. MEN 2b includes
medullary carcinoma of the thyroid, pheochromocytoma, mucosal neuromas, and a distinctive marfanoid habitus. Together,
these syndromes encompass much of the spectrum of endocrine neoplasia.

58. Which of the following signs/symptoms are pathognomonic of hyperparathyroidism?

a. Pathologic fractures of the metacarpals


b. Calcium oxalate nephrolithiasis
c. Hypercalcemia causing mental status changes
d. Atrophy of Type II muscle fibers
e. Osteitis fibrosa cystica
Answer: e

Manifestations of hyperparathyroidism are protean, but generally nonspecific and may be difficult to elicit in the history.
The earliest complaints are often vague and include muscle weakness, anorexia, nausea, constipation, polyuria, and
polydipsia. These nonspecific symptoms may or may not cause the patient to seek medical attention. Symptomatic patients
generally have evidence of chronic disease involving the kidney or the skeleton. Renal complications develop because the
hypercalcemia leads to increased urinary calcium excretion and PTH increases the excretion of phosphate. These events
predispose to stone formation. Nephrolithiasis develops in about 30% of patients. Nephrocalcinosis occurs in 5% to 10% of
patients. These are both nonspecific for hyperparathyroidism.
Parathyroid bone disease in its most classic and severe form, osteitis fibrosa cystica, is seldom seen. However, 5% to 15%
of patients present with significant symptoms of skeletal disease. Only the skeletal disease is pathognomonic. Bone changes
include subperiosteal resorption of bone on the radial aspect of the middle phalanx of the second or third finger. Tufting of
the distal phalanges, bone cysts of the skull and long bones, “brown” tumors (i.e., localized proliferations of osteoclasts),
and diffuse demineralization or granularity are seen as well. Atrophy of Type II muscle fibers, consistent with a neuropathic
and not a myopathic cause, has been demonstrated in patients with hyperthyroidism, but this too is nonspecific.

59. The causes of Cushing syndrome may include which of the following?

a. Posterior pituitary adenoma


b. Adrenal hyperplasia
c. Small cell lung carcinoma
d. Pheochromocytoma
e. Adrenal carcinoma
Answer: b, c, e

The varied causes of cortisol excess produce clinical features that are collectively called Cushing syndrome. These include
exogenous steroid administration, Cushing disease (excessive ACTH production by the anterior pituitary gland, usually
from an adenoma), ectopic ACTH production (small cell lung carcinoma), adrenal adenoma or carcinoma, micronodular
pigmented hyperplasia, macronodular hyperplasia, and steroid-dependent adrenal hyperplasia. Pheochromocytoma is
characterized by catecholamine rather than cortisol excess as it arises from the adrenal medulla rather than the adrenal
cortex.

60. Which of the following are normal systemic effects of glucocorticoids?

a. Enhanced proteolysis
b. Increased gluconeogenesis
c. Diminished lipolysis
d. Decreased rate of intestinal epithelial replication
Answer: a, b, d

The many systemic effects of glucocorticoids are related to regulation of intermediary metabolism. In this regard, perhaps
the most important action is the effect of steroids on protein breakdown. A direct proteolytic effect of steroids is suggested
by several lines of evidence.
Glucocorticoids enhance gluconeogenesis by both a direct effect on gluconeogenic hepatic enzymes and also by provision
of substrate for gluconeogenesis by proteolysis. Glucocorticoid influence leads to the accentuation of lipolysis. The truncal
obesity seen in steroid excess is related to the predominance of the lipogenic effect of insulin on truncal adipocytes over the
lipolytic effect of glucocorticoids. The opposite relation may hold for the receptors in fat of the extremities and would
explain the comparatively scant fat in these areas with steroid excess.
The most notable effect of glucocorticoids in the intestinal tract is a decrease in the rate of mucosal cell replication. In
addition, decreased mucosal and pancreatic prostaglandin synthesis occur. This may have important implications for the
cytoprotective mechanisms in the stomach.
61. A 10-year-old child presents with hypertension, tachycardia, nervousness and sweating. The best initial diagnostic
evaluation is which of the following?

a. Radioimmunoassays for norepinephrine and epinephrine in serum


b. Magnetic resonance imaging of the adrenal gland
c. MIBG (131I-Methaiodobenzylguanidine) scintigraphy
d. Measurement of catecholamines and their degradation products in a 24-hour urine specimen
Answer: d

The first diagnostic step in determining the functional state of an adrenal gland or lesion is to screen the urine or plasma for
secretory products. Once hypersecretion is demonstrated, the specific type of pathology producing a syndrome must be
determined with the aid of functional tests and relevant scanning and imaging. The most efficient and sensitive means of
screening in a patient suspected of having a pheochromocytoma, (as indicated here) is measurement of the catecholamines
or metabolic products thereof in the urine. Although 24-hour samples can smooth out the possible episodic variations in
catecholamine secretion, shorter sampling periods can be useful, especially if corrected for creatinine excretion. Timing of
the collection is critical in patients who have only episodic hypertension. Urine collection should be started immediately
after a suspected attack of hypertension. Fluctuations in plasma catecholamine concentrations are much greater than those in
urinary excretion, even in normal subjects. Plasma determinations are quite sensitive and specific with radioimmunoassays
and HPLC determination of plasma catecholamines, but the specificity can be low because of the overlap of normal spikes
in catecholamine concentrations with concentrations produced by minimally secreting pheochromocytomas. MRI and
MIBG imaging studies are both potentially useful after catecholamine excess is confirmed. MRI demonstrates anatomy
quite well. MIBG scintigraphy is particularly useful in looking for nonadrenal and bilateral pheochromocytomas.

62. A 25-year-old male has been taking 40 mg of prednisone PO qod for ulcerative colitis for 5 years. He undergoes an
uneventful colectomy with endorectal pull-through and an ileoanal anastomosis. Which of the following statements
regarding steriod management are correct?

a. On the day of surgery he should receive 100 mg hydrocortisone IV q6h


b. The postoperative steroid dose should be halved q 12 hours to reduce the risk of infectious complications and
improve would healing
c. Prophylactic treatment with a somatostatin analogue will reduce the risk of postoperative pancreatitis
d. Exogenous steroid replacement can be stopped after 3 months
Answer: a

The postoperative course of management for a patient with pituitary adrenal suppression from exogenous steroids involves
tapering the exogenous steroid doses to maintenance levels after high dose replacement at the time of operation. One simple
regimen involves administering 100 mg of hydrocortisone intravenously every 6 hours during the first 48 hours. Some
prefer alternating doses of IM cortisone acetate in the event that intravenous access is lost. Provided that no intervening
complications arise, the doses can be halved every 48 to 72 hours. In patients who have been exposed preoperatively to
glucocorticoid excess, the maintenance dose may be as high as 100 mg/d for several months. Both high doses and normal
maintenance of 35 to 55 mg/d can be given in the form of oral cortisone acetate as long as reliable alimentation and
absorption have been achieved. The pituitary-adrenal axis remains suppressed for 6 to 12 months after operation.
Complications in the postoperative period include wound infection, pancreatitis, and thromboembolism. There is no data to
suggest that the risk of postoperative pancreatitis can be diminished with somatostatin analogue therapy.

63. Imaging of the adrenal gland is best achieved with which of the following techniques?

a. Ultrasound
b. Computed tomography (CT)
c. Arteriography
d. Scintigraphy with 131I-6 b-iodomethyl-19-norcholesterol (NP-59)
e. Scintigraphy with 131I-methaiodobenzylguanidine (MIBG)
Answer: b

Although ultrasonography is the least expensive of the imaging procedures, its value is limited by the relative inaccessibility
of the adrenal gland and by the small size of some adrenal lesions. CT is the technique most commonly used to examine
patients in whom adrenal abnormalities are suspected. CT reliably detects adrenal tumors greater than 1 cm in diameter. The
sensitivity of CT for tumors that are 1 cm in diameter is about 80% and reaches 100% for tumors that are 3 to 4 cm.
Although CT is noninvasive and reasonably sensitive, it is nonspecific. It does not distinguish functioning from
nonfunctioning tumors, nor benign from malignant tumors with any degree of reliability.
MRI has developed a certain usefulness even after retrenchment from early optimistic predictions. MRI is more expensive
and requires greater patient cooperation than CT, but it has greater versatility because of the use of TI-and T2-weighted
images. In some cases, the T2-weighted images are capable of providing a differential diagnosis and may distinguish such
entities as metastatic or primary carcinoma and pheochromocytoma from adenomas, lipomas, myelolipomas and cysts. In a
sense, MRI is complementary to CT in that the latter can better detect the lesion while the former can distinguish one type
of lesion from the other. In addition, MRI is probably better than CT for distinguishing anatomic relationships and the extent
of involvement of the surrounding tissues.
Two radiopharmaceuticals have proved useful in imaging the adrenal gland. Adrenocortical lesions can be imaged by NP-59
which is taken up as cholesterol in the adrenocortical steroidogenic pathway. The other agent is MIBG, a norepinephrine
analogue. It indicates norepinephrine accumulation in storage vesicles and can detect sympathoadrenal tumors at any site in
the body. NP-59 can localize the adrenal cortex and any functioning tumors. It can distinguish adrenocortical hyperplasia
from functioning adenomas or carcinomas. MIBG is a useful agent in localizing pheochromocytomas throughout the body,
especially when the tumors are multiple, extraadrenal, recurrent, or metastatic.
Arteriography, venography and selective venous sampling have become less popular as experience with other imaging
techniques has become greater. Disadvantages inherent in invasive procedures using intravascular contrast agents are
obvious. Arteriography is specifically dangerous in the study of patients with pheochromocytomas.

64. A 45-year-old female is found to have a 2 cm solid nodule in her right adrenal gland at the time of an abdominal
CT scan following an auto accident. With regard to the adrenal lesion, she is asymptomatic and it is found to be
nonfunctional on evaluation. You would recommend which of the following?

a. Extraperitoneal right adrenalectomy through either a flank of posterior approach


b. Suppression with 5 mg prednisone PO qod
c. Followup CT scan in 1 to 3 months
d. Excisional biopsy via laparaoscopic approach
Answer: c

Indication for operation in the patient with a unilateral functioning adrenal tumor is clear. In the patient with a
nonfunctioning adrenal tumor, the need for surgery is related to the size of the tumor and its rate of growth. There is
consensus that a tumor larger than 6 cm should be removed. Some recommend that the acceptable size limit be 3 cm,
especially when MRI suggests carcinoma or when the functional studies suggest activity. When nonoperative therapy is
elected, the patient should receive an adrenal scan 1, 3, and 6 months after the initial scan and yearly thereafter to assess the
growth of the lesion. If the tumor has grown, surgical removal is indicated.

65. A 20-year-old male with a 10 cm left adrenal mass is found to have 10 mg of norepinephrine in a 24-hour urine
collection and a plasma 18 hydroxycorticosterone level of 50 mg/dL. Initial reoperative preparation should include which
of the following?

a. Treatment with spironolactone


b. Intravenous potassium-loading to prevent intraoperative hypokalemia
c. Treatment with phenoxybenzamine
d. Treatment with labetalol
Answer: c

This patient has a pheochromocytoma. The most efficient and sensitive means of screening for pheochromocytoma is
measurement of the catecholamines, or metabolic products thereof, in the urine. The catecholamines, norepinephrine and
epinephrine, are excreted in amounts under 100 µg/d in the normal person. Because of some overlap in values, specificity
can be improved by using a normal range of up to 250 mg/d. The measurement of plasma 18-hydroxycorticosterone level is
helpful in evaluating patients with hyperaldosteronism, as it is an intermediate product in its synthesis. The 18-
hydroxycorticosterone levels are above 100 mg/dL in virtually all patients with aldosterone producing adenomas. The
plasma value for this patient is normal. Perioperative treatment with either spironolactone and potassium replacement is
appropriate for patients with hyperaldosteronism, but not pheochromocytoma.
Nonoperative treatment of pheochromocytoma is generally unsatisfactory and entails pharmacologic blockade of the effects
of catecholamines. Phenoxybenzamine and prazosin are two preferred agents that block the a-adrenergic effects of the
catecholamines preoperatively with pheochromocytoma. The use of b-adrenergic blockers, such as labetalol, may be
required in those patients with obvious b-adrenergic effects, such as resting pulse rates above 100 beats/min.
Because of the potential for wide swings in blood pressure and other effects of chronic catecholamine secretion, careful
preoperative preparation is required in patients with pheochromocytoma. It is customary to institute a-adrenergic blockade 2
to 3 weeks before anticipated surgery. This has beneficial effects of controlling blood pressure and allowing restoration of a
decreased blood volume. It is the consensus that preoperative preparation in the manner makes the intraoperative
management of the patient much more safe. In patients who require b-adrenergic blockade, it is essential to first establish
good a-adrenergic blockade. These patients are prone to cardiac failure induced by b-adrenergic blockade because of the
cardiomyopathy that may preexist. b-Adrenergic blockade in the cardiomyopathic patient with failure to first reduce the
afterload by a-adrenergic blockade, can precipitate cardiac failure.

66. Which of the following diagnostic tests will differentiate between the pituitary and adrenal gland as the cause of
hypercortisolism?

a. High dose dexamethasone suppression test


b. CRH stimulation test
c. Low dose dexamethasone test
d. AM and PM plasma cortisol determination
Answer: a, b

Diurnal variation of plasma cortisol levels is lost in hypercortisolism, caused by both adrenal tumors and pituitary lesions.
Dexamethasone, by negative feedback, suppresses the hypothalamic pituitary secretion of ACTH and effects the consequent
lowering of both plasma cortisol and urinary 17-hydroxycorticosteroid excretion. Administration of a single dose of 2 mg of
dexamethasone suppresses plasma cortisol and urinary 17-hydroxycorticosteroids by at least half when compared to control
values from untreated normal patients (low dose suppression test). In Cushing disease, with a setpoint of ACTH secretion
higher than normal, low-dose dexamethasone is insufficient to suppress ACTH. High-dose dexamethasone suppression is
achieved by 2 mg of dexamethasone administered every 6 hours for 24 hours. A normal response is to lower the 17-
hydroxycorticosteroid excretion by more than one half. In the case of Cushing disease, the hypothalamic steroid receptors
that allow negative feedback are intact but at a higher setpoint. In this case, 17-hydroxycorticosteroid secretion does
decrease significantly after high-dose dexamethasone administration. Adrenal tumors, other causes of ectopic production of
ACTH, and most cases of nodular hyperplasia do not respond to dexamethasone suppression with a decrease in steroid
secretion. With an adrenal tumor, pituitary ACTH is already suppressed; therefore, dexamethasone cannot suppress it
further. With ectopic ACTH secretion, the tissue producing ACTH has no receptors for steroids and thus negative feedback
cannot be achieved. Therefore, high dose dexamethasone suppression will differentiate hypercortisolism of pituitary and
adrenal origins.
Potentially the most helpful new test for this purpose uses the administration of recombinant CRH to release ACTH and
consequently to stimulate cortisol secretion. CRH (1 mg/kg) is administered intravenously, and serial blood samples
obtained for 3 hours after administration. The normal pituitary adrenal axis responds by a moderate increase in ACTH and
cortisol. With Cushing disease, the ACTH and cortisol rise are accentuated. With adrenal autonomous production of cortisol
and with ectopic ACTH production, there is virtually no response to CRH.

67. Which of the following statements regarding androgens and estrogens in the fetus are true?

a. The development of normal female external genitalia requires estrogen production by the ovary
b. A female with congenital adrenal hyperplasia is likely to be masculinized in appearance
c. The development of normal male external genitalia requires adrenal androgen production
d. A male with congenital adrenal hyperplasia is likely to be feminized in appearance
Answer: b, c

Adrenal androgens in the fetus stimulate wolffian duct development and elongate the genital tubercle. They promote
midline migration of the labial folds and fusion of these folds to form the scrotum. To complete the male transformation, the
urethral opening migrates to the tip of the phallus. All of these events are androgen-dependent. Since the normal female
fetus does not secrete androgens, the genital tubercle, labial folds, and urethral opening all remain in the female position in
this circumstance. Thus the female phenotype is associated with the absence of fetal sex hormone production. Excess
androgen in the female fetus causes neonatal virilization, as is seen with congenital adrenal hyperplasia. The male infant
with congenital adrenal hyperplasia is likely to have a normal appearance of the external genitalia as a neonate. Precocious
puberty will develop over a period of years in this latter circumstance.
68. Which of the following statements regarding the physiology of the adrenal gland are true?

a. Release of CRH is regulated principally by negative feedback by ACTH


b. Plasma 17-ketosteroid levels reflect the degree of adrenal cortisol production
c. Renin undergoes enzymatic cleavage in the lung to angiotensin I
d. The plasma half-life of ACTH is relatively long (> 24 hrs)
e. None of the above
Answer: e

The proximate stimulator of cortisol production is the peptide hormone, andrenocorticotropic hormone (ACTH). It
originates from the anterior pituitary gland and is regulated by corticotropin-releasing hormone (CRH). The regulation of
CRH is controlled by various neural influences. These include intrinsic central nervous system influences and a negative
feedback inhibition by cortisol. Although there is some evidence of a short-loop feedback of ACTH on CRH, both slow and
fast feedback by cortisol on the pituitary release mechanism are the primary sources of clinically relevant CRH regulation.
The steroidogenic pathway involves the conversion of cholesterol to pregnenolone, progesterone, 17-hydroxyprogesterone,
and then either to the adrenal androgens or cortisol via several intermediates. The 17-ketosteroids reflect adrenal androgen
synthesis while the 17-hydroxysteroids reflect cortisol synthesis.
Renin is produced predominantly in the juxtaglomerular apparatus of the kidney where it acts locally and is released into the
systemic circulation. Renin cleaves angiotensin I, a decapeptide derived from the liver which serves as renin substrate.
Angiotensin I undergoes enzymatic cleavage in the lung to angiotensin II, which is the biologically active form of the
peptide.
The plasma half-life of ACTH is short (measured in minutes) with a rapid onset of action. This is in contrast to a longer
plasma half-life and a slower onset of action for cortisol itself.

69. The approximate 5-year survival rate for adrenocortical carcinoma is which of the following?

a. 0%
b. 20%–25%
c. 50%–60%
d. Nearly 100%
Answer: b

The prognosis for adrenocortical carcinoma is not good. The overall 5-year survival rate is 20% to 25% for these
malignancies. When there is localized disease at the time of surgery, the 5-year survival may be higher, in the 40% to 50%
range. The true prognosis in childhood is not clear, but the data suggest a 2-year survival rate of about 20%.

70. Which of the following statements is true with respect to pheochromocytoma?

a. Pheochromocytoma associated with MEN IIa is usually unilateral and rarely malignant; therefore, unilateral
exploration through a posterior flank incision is usually sufficient
b. Clonidine fails to suppress basal plasma-catecholamine levels in patients with pheochromocytoma
c. NP-59 (131I-6 b-iodomethyl-19-norcholesterol) is taken up as cholesterol by the adrenal medulla
d. The ratio of plasma 3,4-dihydroxyphenoglycol (DHPG) to norepinephrine is generally elevated in patients with
pheochromocytoma compared to patients with essential hypertension
Answer: b, d

Pheochromocytoma associated with the MEN IIa syndrome is more often bilateral and more often malignant, therefore,
abdominal exploration through an anterior approach is indicated. The ability to measure catecholamines in the plasma has
made possible the clonidine suppression test. In patients without pheochromocytoma, clonidine suppresses high basal
plasma concentrations into the normal range, whereas concentrations in patients with pheochromocytoma are not
suppressed. Another use of plasma catecholamine measurements is in examining the ratio of 3,4-dihydroxyphenoglycol
(DHPG) to norepinephrine in plasma. DHPG is released from the chromaffin cells and adrenergic neurons to a much greater
extent than norepinephrine in pheochromocytoma patients compared with patients who have essential hypertension, i.e. the
ratio of DHPG to norepinephrine is higher in patients with pheochromocytomas.
NP-59 (131I-6 b-iodomethyl-19-norcholesterol) is taken up as cholesterol by the adrenal cortex and is incorporated in the
adrenocortical steroidogenic pathway. This is a useful agent for imaging adrenocorticol lesions. 131I-
methaiodobenzylguanidine (MIBG) is a norepinephrine analogue that is useful in localizing pheochromocytomas
throughout the body, especially when the tumors are multiple, extraadrenal, recurrent, or metastatic.

71. Which of the following statements regarding aldosterone are true?

a. Its secretion is directly related to the serum potassium concentration


b. Angiotensin II is a more potent regulatory factor than ACTH
c. Primary hyperaldosteronism is characterized by hyperkalemia
d. Secondary hyperaldosteronism occurs with renal artery stenosis
Answer: a, b, d

Aldosterone secretion is controlled by changes in the afferent arteriolar pressure in the renal cortex as well as by changes in
sodium content in the renal tubule. These changes are sensed by the juxtaglomerular apparatus and by the macula densa and
act through the renin angiotensin system. At least two other factors influence aldosterone secretion. Aldosterone secretion is
directly related to the serum potassium concentration. An increase in serum potassium directly stimulates aldosterone
production, whereas a decrease in serum potassium has the opposite effect. Because of its early point of action in the
steroidogenic pathway, ACTH also increases secretion of aldosterone, although it is much less potent in this regard than in
its stimulation of cortisol. The stimulatory effects of potassium and ACTH on aldosterone secretion can be overcome by
angiotensin II stimulation.
Primary hyperaldosteronism is characterized by mineralocorticoid hypersecretion which promotes a positive sodium
balance and hypokalemia. About 80% of patients with primary hyperaldosteronism have serum potassium levels of 3.5
mEq/L or less. Causes of secondary hyperaldosteronism are related to increased renin secretion. These include renal artery
stenosis, congestive heart failure and renal salt-wasting.

72. A term neonate is noted to have ambiguous female genitalia. This infant is at risk for which of the following
potentially life-threatening problems?

a. Cardiomyopathy with congestive heart failure


b. Sodium wasting nephropathy with hypovolemia
c. Respiratory failure from surfactant deficiency
d. Spontaneous hemorrhage from thrombocytopenia
e. Pulmonary embolus from a hypercoaguable state
Answer: b

Enzymatic defects in the steroidogenic pathway produce a syndrome known as congenital adrenal hyperplasia. This
syndrome presents predominantly in the neonatal period with sexual ambiguity. These enzymatic defects result in a lowered
cortisol secretion. The specific enzyme defects present determine the clinical form of the syndromes. These include a 21-
hydroxylase deficiency, an 11b-hydroxylase deficiency, and a 17-hydroxylase-deficiency. The 21-hydroxylase deficiency
and the 11b-hydroxylase deficiency result in excess androgen production in utero and result in masculinization with
ambiguous genitalia in the female newborn. Masculinizing effects in the male may not be detected until precocious puberty
becomes obvious. About 40% of patients with 21-hydroxylase deficiency, the most common form, have salt-wasting or
sodium loss by urine. Hypovolemic shock can result. Cardiomyopathy, respiratory failure, thrombocytopenia and pulmonary
emboli are not associated with this syndrome.

73. Which of the following adrenal lesions can be treated definitively by medical means?

a. Benign functional adrenocortical adenoma


b. Adrenocortical carcinoma
c. Congenital adrenal hyperplasia
d. Cushing disease
e. Pheochromocytoma
Answer: c
The treatment of adrenal tumors is primarily surgical removal. Although pharmaceutical agents are useful in preparing the
patient for surgery or in palliating the patient with recurrent adrenal carcinoma, no agents render definitive therapy for
adrenal tumors. Congenital adrenal hyperplasia stands alone among the primary, hyperfunctioning adrenal syndromes that
are amenable to medical therapy for definitive treatment. Functioning benign lesions of the adrenal cortex that are not
ACTH dependent, such as adenomas or macronodular hyperplasia, respond to metyrapone and aminoglutethimide, which
are inhibitors of enzymes in the adrenal steroidogenic pathway. Both agents can effect a decrease in the production of
cortisol when there is no increase in ACTH secondary to feedback stimulation. These drugs are not satisfactory long-term
agents because of their high incidence of drug reactions, patient noncompliance, and continued growth of the lesions. They
may be useful in patients whose surgery must be delayed. Although malignant, functioning adrenocortical lesions should be
debulked whenever possible. Several chemotherapy agents offer adjunct therapy. The most noteworthy is mitotane (o,p,-
DDD). This is a cytolytic agent that has a 30% to 70% response rate in terms of decreasing steroid output. Unfortunately,
patient survival is not affected. As mentioned, nonoperative treatment with cortisone acetate and possibly fludrocortisone is
definitive therapy for congenital adrenal hyperplasia. Cushing disease is best treated by transsphenoidal resection of the
pituitary adenoma. Pheochromoctyoma requires definitive surgical resection although the preoperative pharmacologic
preparation with catecholamine blockade is required.

74. Which of the following statements regarding the pituitary gland are true?

a. ADH is a product of the neurohypophysis


b. The preferred surgical approach to the pituitary gland is via the sphenoid sinus
c. Growth hormone, ACTH, LH, FSH and serotonin are products of the adenohypophysis
d. The adenohypophysis is regulated by neurotransmitters released by the supraoptic hypophyseal tract
Answer: a, b

The anterior pituitary gland is the adenohypophysis which constitutes 80% of the gland. The posterior pituitary, the
neurohypophysis, constitutes the remainder and should be considered virtually an extension of the hypothalamus of the
brain. The pituitary resides within bony confines of the sella turcica (Turkish saddle) and is bordered laterally by the
cavernous sinuses (venous), inferiorly and anteriorly by the sphenoid sinus (air), posteriorly by the dorsum sella and
superiorly by the membranous diaphragma sella. The cavernous sinuses each contain the siphon region of the internal
carotid artery and portions of the cranial nerves III, IV, V and VI all within the venous plexus. The optic chiasm lies
immediately above the diaphragma sella. Directly below the anterior and inferior portions of the sella is the aerated
sphenoid sinus. This is sufficiently large in 97% of the patients to allow a transnasal, transsphenoidal surgical approach to
the pituitary.
The adenohypophysis is regulated by a portal venous system between the median eminence of the hypothalamus and the
adenohypophysis itself. This system involves a transport of (1) thyrotropin-releasing hormone (TRH), to stimulate the
secretion of the thyroid-stimulating hormone (TSH); (2) corticotropin-releasing hormone, to stimulate adrenocorticotropic
hormone (ACTH); (3) growth hormone-releasing hormone, to stimulate secretion of growth hormone (GH); (4)
gonadotropin-releasing hormone, to stimulate luteinizing hormone (LH) and follicle-stimulating hormone (FSH); and (5)
prolactin-inhibitory factor (dopamine), to inhibit prolactin. The neurohypophysis is regulated by means of direct transport of
hormones through nerve fibers from the supraoptic and paraventricular nuclei in the hypothalamus. The neurohypophysis is
a virtual extension of the hypothalamus. Products of the neurohypophysis are antidiuretic hormone (ADH; vasopressin) and
oxytocin. The pituitary gland is not known to release serotonin.

75. Which of the following statement(s) is/are true with respect to growth hormone secreting pituitary adenomas?

a. Fewer than 50% of patients will have growth hormone levels over 10 ng/mL
b. Oral glucose administration suppresses growth hormone levels in patients with acromegaly
c. Over 80% of growth hormone-secreting microadenomas can be cured with transphenoidal resection
d. Preoperative treatment of macroadenomas with a somatostatin analogue may improve postoperative remission
rates
Answer: c, d

The endocrine diagnosis of acromegaly rests largely on serum growth hormone (GH) levels, because 90% of patients will
have levels over 10ng/mL. When acromegaly is apparent but consistently elevated growth hormone levels are not obtained,
the glucose suppression test is the most useful diagnostic procedure. In normal patients, 1 to 2 hours after the oral
administration of 100 g of glucose, the growth hormone level falls well below 5 ng/mL. This suppression is not seen with
GH-secreting adenomas, and often a paradoxical rise in GH is observed.
The goals of treatment are to lower the circulating growth hormone or somatomedin C levels to within a normal range and
to reduce the size of the mass lesion causing compression-related symptoms. When a microadenoma is removed
transsphenoidally, endocrine remission may be expected in 80% to 88% of cases. When a macroadenoma is resected,
postoperative remission is reported in 30% to 68% of cases. The rate of remission is inversely related to preoperative GH
levels and tumor size. Preoperative treatment of macroadenomas with a somatostatin analogue may improve postoperative
remission rates.

76. A 30-year-old woman presents with amenorrhea, headache and bitemporal hemianopsia. Appropriate diagnostic
tests include:

a. Cerebral angiography
b. Serum prolactin levels
c. Magnetic resonance imaging of the brain
d. Abdominal and pelvic CT scan
Answer: b, c

Patients with pituitary lesions present symptoms and signs related to a mass effect on the pituitary and its surrounding
structures, to hypersecretion of the hormones by the lesion itself, or to a combination of both. As mass lesions in the
pituitary enlarge, they encounter the various contents of the cavernous sinuses, including the third, fourth, sixth and first two
divisions of the fifth cranial nerves, as well as the internal carotid artery. The growth of a tumor in the relatively unrestricted
upward direction is much more common and often results in compression of the optic chiasm with the resultant loss of
vision, typically a bitemporal hemianopsia. Prolactin-secreting pituitary adenomas often present with endocrine symptoms
including amenorrhea and galactorrhea in women. In men, the loss of libido, infertility and visual loss are typical. Magnetic
resonance imaging (MRI) has evolved as the first choice for diagnostic imaging and is often the only tool needed to reach a
therapeutic decision with regard to pituitary adenomas. With intravenous infusion of a paramagnetic substance such as
gadolinium, MRI demonstrates intrasellar tumors as small as 5 mm. In addition, the extent of suprasellar and sphenoid sinus
extension, as well as lateral extension into the cavernous sinuses, is demonstrable. Cysts and hemorrhage can be
differentiated, as can blood flowing within an aneurysm. CT scanning has a place in pituitary imaging if MRI scanning is
unavailable. Plain skull X-rays are not needed generally. Cerebral angiography is performed only if an aneurysm is
suspected or if a lesion is so large that occlusion or compression of the internal carotid artery is in question. For this patient,
the symptoms clearly point to a central nervous system, pituitary etiology rather than abdominal end organ failure with
regard to the amenorrhea.

77. Which of the following condition(s) is/are associated with hyperprolactinemia?

a. Chronic renal failure


b. Exogenous estrogen administration
c. Diabetes mellitus
d. Cirrhosis
Answer: a, b, d

Elevated serum prolactin levels do not always indicate the presence of a pituitary tumor. Important alternative causes are
chronic renal failure, hypothyroidism, various drugs including phenothiazines, tricyclic antidepressants, exogenous
estrogen, opiates, reserpine, verapamil and others. In addition, hepatic disease, pregnancy and a variety of pituitary and
hypothalamic lesions cause hyperprolactinemia. If the prolactin level is over 150 ng/ml, a pituitary tumor is almost
invariably the cause, but often microadenomas produce prolactin levels of less than 100 ng/ml. The size of pituitary tumors
has been shown to relate to the degree of prolactin elevation, which may reach into the thousands of nanograms per
milliliter. There are no reliable provocative tests to differentiate prolactinomas from other causes of hyperprolactinemias, so
the diagnosis relies on ruling out other causes and imaging of the adenoma.

78. Pituitary adenomas are best classified according to functional hormone output. This information may be derived
from which of the following?

a. Hematoxylin and eosin staining


b. Immunohistochemical staining of pituitary tissue
c. In situ hybridization studies
d. Selective venous sampling from the inferior petrosal sinuses
Answer: b, c, d

Pituitary adenomas have been classified historically as acidophilic, basophilic and chromophobic. Adenomas may show a
variable staining pattern with conventional hematoxylin and eosin dyes, so it is difficult to classify adenomas based on these
stains. Immunohistochemistry, ultrastructural studies and in situ hybridization analyses for specific hormones are the most
reliable methods of classifying pituitary adenomas today. Immunohistochemical staining of pituitary adenomas with specific
antibodies has reliably classified adenomas using highly purified polyclonal and monoclonal antibodies against prolactin,
GH, ACTH, FSH-b LH-b, and TSH-b. Many studies with these antibodies have revealed that some pituitary tumors are
composed of several cell types, which produce various hormones. Some adenomas may not store specific hormones, so
immunohistochemical staining may be weak or absent. The mRNA is usually present in the cytoplasm of adenomas.
Localization of mRNA for specific protein hormones is becoming more widely used in the classification of pituitary
adenomas. Null-cell adenomas constitute up to 25% of pituitary neoplasms. Selective venous sampling from the inferior
petrosal sinus via transfemoral catherization is an effective method to compare venous effluent from the pituitary to
systemic levels for a specific hormone. In addition, this technique may demonstrate laterality. This latter issue is potentially
important as certain small adenomas may not be discernible from the gross appearance at surgery.

79. Which of the following statements is true with respect to Cushing Disease?

a. Pituitary microadenomas are often small and deep within the gland itself
b. The treatment of choice for hypercortisolism due to a pituitary adenoma in women of childbearing age is
transsphenoidal total hypophysectomy
c. Patients who fail to remit with both surgery and radiation to the pituitary require either medical or surgical
adrenalectomy
d. The long-term recurrence rate after resection of an ACTH-producing pituitary microadenoma is approximately
40%
Answer: a, c

Pituitary microadenomas secreting ACTH may be very small and are often located deep within the gland itself. If the tumor
is not evident on opening the dura, incisions must be made into the gland and internal exploration carried out. If no tumor is
identified, then a decision must be made as to whether to resect all or a portion of the gland. If the endocrine evidence is
convincing for pituitary origin and the patient has no desire to have children, then total hypophysectomy is warranted. If the
petrosal sinus sampling clearly indicates laterality then appropriate hemiresection of the gland may be done.
About 75% of patients have microadenomas as the source of ACTH secretion. The postoperative remission rate in these
patients is 88% to 96%, and the long-term recurrence rate appears to be no more than 5%. 10% to 20% of patients who
undergo exploration have macroadenomas, and the postoperative remission rates in these patients have been reported to be
from 33% to 61%. Most of these patients require postoperative radiation therapy. Patients who fail both surgery and
radiation require either surgical adrenalectomy or medical suppression of adrenal function.

80. A 45 year-old woman presents for evaluation of hypertension, recent onset obesity, hirsutism and depression.
Cerebral MRI does not show a pituitary lesion. Evaluation may include determination of which of the following?

a. AM serum cortisol levels after low dose dexamethasone suppression


b. Simultaneous serum ACTH measurement in peripheral and inferior petrosal sinus sites
c. Chest and abdominal CT scan
d. Urinary free cortisol excretion
Answer: b, c, d

The findings of Cushing Syndrome often include central obesity, hypertension, hirsutism, fatigue, easy bruisability, stria,
moon-like facies, dorsal fat pad, and often depression or other mental changes. Less common abnormalities include
headache, osteoporosis, diabetes mellitus, galactorrhea, peripheral edema and amenorrhea. Often, a patient presents without
the classic cushingoid appearance and complains only of severe fatigue or depression. The cause of hypercortisolism is an
ACTH-secreting pituitary adenoma (Cushing disease) in up to 80% of cases, with remainder due either to an adrenocortical
tumor or to an ectopic neoplasm secreting ACTH or corticotropin-releasing factor. Pituitary-dependent hypercortisolism is
much more common in women(80%) and an ectopic etiology more common in men.
Up to 60% of patients with pituitary etiologies have nondiagnostic imaging studies, therefore, the diagnosis often relies
completely on endocrine testing. Multiple measurements of cortisol and ACTH to evaluate the diurnal pattern are important
but often misleading. They are mainly of value when clearly elevated. The determination of urinary free cortisol excretion
over 24 hours is an extremely important measurement. If the overnight dexamethasone screening test yields an 8 AM serum
cortisol level of less than 5 ug/dl, then hypercortisolism is rarely present. Generally, patients with a pituitary etiology of
hypercortisolism do not show suppression with the low-dose dexamethasone test, but do with the higher dose test. Patients
with adrenal or ectopic etiologies do not experience suppression with either dose. Chest and abdominal CT scans are
appropriate to look for adrenal or lung tumors. The most specific test when the MRI is negative and evidence implicates the
pituitary, is simultaneous measurement of ACTH levels in both inferior petrosal sinuses and a concurrent determination of
the peripheral ACTH level. This approach produces specific information about the existence of an ACTH-secreting pituitary
tumor and even the laterality of the tumor.

81. The most common mass lesion in the sella turcica is which of the following?

a. Craniopharyngioma
b. Aneurysm
c. Benign pituitary cyst
d. Pituitary adenoma
Answer: d

Pituitary adenomas are the most common mass lesions in the sella turcica or parasellar region. They constitute 8% to 10%
of all brain tumors. Occasionally, they are cystic and may be confused with other lesions. Craniopharyngiomas are the next
most common tumor, although these are more often suprasellar in location. These are more common in children, but up to
one third occur in adults. They are usually cystic and are calcified in 70% of children and 40% of adults. More rare lesions
include meningiomas, germinomas, metastatic malignancies from lung and breast primaries, gliomas, dermoids and benign
epidermoid. Rathke cysts, aneurysms, and a variety of inflammatory and granulomatous processes.

82. Pharmacologic treatment of growth hormone (GH) excess secondary to a pituitary adenoma may include the use of
which of the following?

a. Bromocriptine
b. Vasopressin
c. Octeotide
d. Prednisone
Answer: a, c

Bromocriptine, a dopamine receptor agonist, has been demonstrated to lower GH levels in 71% of 126 acromegalic patients.
A clinical response was achieved in up to 95% of acromegalic patients, and reduced somatomedin C levels were found in
some patients with persistently elevated GH levels. Bromocriptine does not appear to be an effective primary treatment for
acromegaly, but may help to control GH and somatomedin C levels as an adjuvant therapy. A somatostatin analogue,
octeotide, has recently been used to treat acromegaly and has been demonstrated to significantly reduce GH and
somatomedin C levels in most patients and normalize values in 50%. This treatment provides only minimal tumor
shrinkage, and GH levels rise again immediately following cessation of the drug. This drug may prove to be useful as a
preoperative treatment or in surgical failures. Vasopressin and prednisone have no role in the treatment of acromegaly.

1. After intraductal papilloma, unilateral bloody nipple discharge from one duct orifice is most commonly caused by which
of the following pathologic conditions?
A. Paget's disease of the nipple.
B. Intraductal carcinoma.
C. Inflammatory carcinoma.
D. Subareolar mastitis.
Answer: B

DISCUSSION: Nipple discharge is surgically significant when it is grossly bloody and when it appears at a single duct
orifice on one nipple. Bloody discharge is usually due to a benign intraductal papilloma; however, intraductal carcinoma in
the large ducts under the nipple can be the cause of bloody discharge, and pathologically the lesion is frequently a large
papillary tumor that has become malignant. Paget's disease of the nipple is also due to intraductal carcinoma arising in
subareolar ducts, but it rarely is associated with nipple discharge. Subareolar mastitis may produce nipple discharge, but it is
purulent and not bloody. Inflammatory carcinoma is not associated with nipple discharge.

2. Which of the following conditions is associated with increased risk of breast cancer?
A. Fibrocystic mastopathy.
B. Severe hyperplasia.
C. Atypical hyperplasia.
D. Papillomatosis.
Answer: C

DISCUSSION: Fibrocystic mastopathy, or fibrocystic disease, was once thought to increase the risk of breast cancer;
however, later studies of the pathologic findings in fibrocystic complex found an increased cancer risk only for patients
whose biopsies showed atypical hyperplasia. “Severe hyperplasia” is a pathologic term that refers to the amount of
hyperplasia and is frequently seen in the biopsy specimens of young women; it is a misleading term and is not associated
with a disease risk. Papillomatosis is also part of the fibrocystic complex and is a frequent finding in benign breast biopsies;
it does not confer an increased risk of cancer.

3. Which of the following breast lesions are noninvasive malignancies?


A. Intraductal carcinoma of the comedo type.
B. Tubular carcinoma and mucinous carcinoma.
C. Infiltrating ductal carcinoma and lobular carcinoma.
D. Medullary carcinoma, including atypical medullary lesions.
Answer: A

DISCUSSION: Tubular, mucinous, and medullary carcinomas are histologic variants of infiltrating ductal cancer and are all
invasive malignancies. Infiltrating lobular cancer is a particular histologic variant of invasive breast cancer characterized by
permeation of the stroma with small cells that resemble those found in the breast lobule or acinus. Intraductal carcinoma
refers to a malignancy of ductal origin that remains enclosed within duct structures. This noninvasive proliferation can
undergo central necrosis, which frequently calcifies to form the microcalcifications seen on mammography. The central
necrosis within enlarged and back-to-back ductal structures resembles comedoes and gives rise to the term
“comedocarcinoma,” now reserved for this histologic variety of intraductal carcinoma.

4. Which of the following are the most important and clinically useful risk factors for breast cancer?
A. Fibrocystic disease, age, and gender.
B. Cysts, family history in immediate relatives, and gender.
C. Age, gender, and family history in immediate relatives.
D. Obesity, nulliparity, and alcohol use.
Answer: C

DISCUSSION: The most important risk factors for breast cancer are the patient's age, gender, and a family history of breast
cancer in immediate relatives (sisters, mother, daughter). The age-adjusted incidence of breast cancer increases with age.
Breast cancer does occur in males, but the disease is far more common in women. Family history is important when breast
cancer occurs within the immediate family; history of breast cancer in more distant relatives (grandmothers, cousins, aunts)
is less important. In addition, age factors into the risk associated with family history. An affected young primary relative is
far more significant as a risk factor than an older relative with breast cancer. The other important risk factor not listed here is
a history of breast cancer, either within the conserved ipsilateral breast or in the contralateral breast. Again, age plays an
important modifying role; as the age at which breast cancer was first diagnosed increases, the risk of a subsequent second
cancer decreases. Although patients with fibrocystic disease are at increased risk for breast cancer, risk concentrates in those
patients with fibrocystic disease who show atypical epithelial hyperplasia within breast ducts. Obesity, nulliparity, and
alcohol all appear to increase risk slightly and are important to the epidemiologic study of breast cancer; however, the effect
of these factors is not sufficient to warrant their use in common clinical practice.
5. Which of the following pathologic findings is the strongest contraindication to breast preservation (lumpectomy with
breast radiation) as primary treatment for a newly diagnosed breast cancer?
A. Grade 3, poorly differentiated, infiltrating ductal carcinoma.
B. Extensive intraductal cancer around the invasive lesion.
C. Tumor size greater than 3 cm.
D. Positive surgical margin for invasive cancer.
Answer: D

DISCUSSION: The only firm contraindication to wide excision and radiation (breast preservation, lumpectomy) as the
primary surgical treatment for a newly discovered breast cancer is the inability to achieve an uninvolved surgical margin
after excision of the tumor. A positive surgical margin requires, at least, reoperation with an attempt at re-excision of the
cancer. If the margin of removal is positive after attempts at re-excision, this is a strong reason to recommend mastectomy in
preference to breast conservation. Tumor size is a relative contraindication when the cancer is so large in relation to the
breast that excision to a clean surgical margin seems unreasonable. Other histologic findings, such as tumor grade or
vascular invasion, are not strong reasons to recommend mastectomy if the patient would prefer breast conservation.

6. Axillary lymph node dissection is routinely used for all of the following conditions except:
A. 2-cm. pure comedo-type intraductal carcinoma.
B. 1-cm. infiltrating lobular carcinoma.
C. 8-mm. infiltrating ductal carcinoma.
D. A pure medullary cancer in the upper inner quadrant.
Answer: A

DISCUSSION: Intraductal carcinoma is carcinoma in situ and does not metastasize to regional or distant sites. Lymph node
dissection is not routinely required for a pure in situ cancer of the breast. In contrast, all of the other cancers listed above
(infiltrating lobular, infiltrating ductal, and medullary carcinoma) are invasive malignancies that are capable of nodal and
distant metastasis. Lymph node dissection is commonly recommended for these invasive malignancies. Intraductal lesions
that have grown larger than 5 cm. are more apt to have become focally invasive. Since this invasive component might be
missed histologically, many surgeons advocate selective use of axillary node dissection for large intraductal lesions,
particularly high-grade tumors such as the comedo variant. However, a purely intraductal 2-cm. cancer would most likely be
treated without performing node dissection.

7. Failure to perform radiation after wide excision of an invasive cancer risks which of the following outcomes?
A. Recurrence of cancer in the ipsilateral breast.
B. Shorter survival time.
C. Regional nodal recurrence.
D. Greater chance of breast cancer mortality.
Answer: A

DISCUSSION: Retrospective reviews and prospective surgical trials agree that omission of breast radiation after wide
excision leads to a higher rate of ipsilateral breast recurrence. However, survival and the risk of distant disease are not
altered in patients treated by excision alone, within the follow-up time of the studies and given their inherent power to detect
differences in outcome. Regional node metastasis is not affected by the choice of mastectomy versus wide excision and
radiation.

8. Which of the following treatments should never be recommended to a patient with purely intraductal carcinoma?
A. Modified radical mastectomy.
B. Lumpectomy to clear surgical margins, followed by observation.
C. Incisional biopsy with an involved margin, followed by radiation.
D. Excisional biopsy to clear margins, followed by radiation.
Answer: C

DISCUSSION: The treatment approach to intraductal carcinoma depends on the extent of the disease, its multifocality, and
the involvement of the surgical margin. For extensive disease, modified radical mastectomy is appropriate, particularly if
there is a great likelihood of occult invasive disease, making axillary dissection logical. For small foci of disease excised to
clear surgical margins, observation is an acceptable recommendation to a well-informed patient. Several noncontrolled
reviews and the National Surgical Adjuvant Breast and Bowel Project (NSABP) trial for intraductal disease would indicate a
greater chance of ipsilateral breast recurrence for lumpectomy only; however, the magnitude of the risk is small, and
survival is excellent and unaffected. The only mode of treatment that cannot be recommended for routine management is
leaving residual disease in the breast and treating only with radiation.

9. The proper treatment for lobular carcinoma in situ (LCIS) includes which of the following components?
A. Close follow-up.
B. Radiation after excision.
C. Mirror-image biopsy of the opposite breast.
D. Mastectomy and regional node dissection.
Answer: A

DISCUSSION: LCIS is best thought of as a precursor lesion that confers increased risk for eventual cancer. The magnitude
of this risk appears to be in the range of seven- to ninefold over baseline risk. The chance of breast cancer is equal in both
breasts, not just in the biopsied breast, and the type of cancer is not confined to a lobular histology. After a diagnosis of
LCIS, patients are at increased risk for invasive and noninvasive ductal carcinoma in both breasts. Therefore, mirror-image
biopsy as practiced in the past has little to offer. Since LCIS is purely noninvasive, nodal dissection is not required if
mastectomy is chosen. There are no data on the use of breast radiation therapy for LCIS. Most surgical oncologists
recommend close follow-up for patients who have LCIS only; the alternative surgical treatment that makes most sense is
bilateral simple mastectomies, with or without reconstruction.

10. Which of the following statements most accurately reflects the findings of large overview analyses of clinical trials in
which adjuvant chemotherapy for early-stage breast cancer was compared to a control group treated only with surgery?
A. The benefit of adjuvant therapy is confined to young patients.
B. Adjuvant therapy benefits all patients and is independent of age or node status.
C. Adjuvant therapy does not work in estrogen-positive patients.
D. The magnitude of benefit is very large.
Answer: B

DISCUSSION: An overview analysis (meta-analysis) examined nearly all randomized clinical trials in which chemotherapy
after surgery was compared to surgery alone for treatment of early-stage breast cancer. This examination of the world's
published literature revealed that the magnitude of benefit (the reduction in the odds of recurrence) from chemotherapy was
relatively small and in the range of a 20% reduction in the chance of recurrence or death; however, this benefit extended to
patients of all ages (young and older) and to both node-positive and node-negative patients. The value of adjuvant
chemotherapy does not depend on the hormone receptor content of the cancer. It is useful to remember that a constant
reduction in the odds of recurrence results in a higher absolute benefit as the prognosis worsens. If the chance of recurrence
is 50% (for node-positive groups) the absolute reduction will be in the range of 10% or 15%. In contrast, if the recurrence
rate is 10%, the absolute difference between treated and control groups will be less than 5%. This means that many patients
need to be exposed to the risks and side effects of chemotherapy to benefit a very small number. This kind of thinking is
currently used to decide who should receive adjuvant chemotherapy after primary treatment (mastectomy or lumpectomy).

11. Which of the following statements are true about reconstruction of the breast following mastectomy?
A. A permanent prosthesis or tissue expander may be inserted at the time of the ablative surgery.
B. If the patient requires adjuvant chemotherapy or radiation therapy, reconstruction of the breast is delayed until
completion of the treatment.
C. Extensive postmastectomy defects require the use of a flap.
Answer: ABC

DISCUSSION: Reconstruction can be initiated at the time of the ablative surgery, using a 6-cm. slightly curved incision at
the level of the sixth rib through the serratus muscle. A pocket is created beneath the serratus and pectoralis major muscles,
extending medially to the perforating internal mammary vessels and inferiorly beneath the fascial insertion of the rectus
abdominis muscle. A tissue expander prosthesis is inserted into the pocket. If the patient requires adjuvant chemotherapy or
radiation therapy, reconstruction of the breast is delayed until treatment is completed and an adequate recovery period has
passed. If the quantity or quality of the chest skin or the pectoralis major muscle is insufficient, tissue must be brought in
from adjacent areas. A latissimus dorsi musculocutaneous flap may be transferred on its blood supply via the thoracodorsal
artery and vein. Extensive postmastectomy defects necessitate the use of the larger rectus abdominis musculocutaneous flap,
which is based on the superior epigastric vessels. A “free” microvascular rectus abdominis or other myocutaneous flaps may
be used. The thoracodorsal or anterior serratus vessels can usually be anastomosed to the inferior epigastric vessels of the
rectus abdominis flap.

12. Which of the following statements are true about the management of mammary hyperplasia?
A. Reduction mammaplasty can be performed only on women younger than 40 years.
B. Removal of breast tissue to reduce size of the breast is usually predicated on the use of a nipple, areola, and dermal
pedicle flap.
C. If removal of 2000 gm. of breast tissue is needed, breast amputation with immediate free nipple-areola grafting is
performed.
Answer: BC

DISCUSSION: Reduction mammaplasty can be performed at any age. Because of the increased weight of the breast
considerable shoulder and back pain, accompanied by excoriation of the skin in the inframammary area and the shoulders,
can occur. Older women frequently seek relief from these problems, which can be resolved by a reduction mammaplasty.
The reduction in breast volume is usually accomplished by moving the nipple and areola on a dermal pedicle flap. The flap
can be based inferiorly, medially, superiorly, laterally, vertically, or horizontally. It is possible to remove up to 3000 gm. of
breast tissue utilizing a pyramidal-based breast flap with an inferior dermal nipple-areola pedicle since the blood supply to
the tissues is preserved by this technique. Breast reduction involving removal of more than 3000 gm. requires a breast
amputation technique with immediate free nipple grafting.

13. Which of the following statement(s) is/are true concerning the anatomy of the breast?

a. About 25% of the lymphatic drainage of the breast courses to the internal mammary nodes
b. Nerves within the axillary fat pad include the intercostal brachial nerve, the long thoracic nerve, and thoracodorsal
nerve
c. Fascial bands projecting through the breast to the skin form a supporting framework known as Cooper’s ligaments
d. The ductal system of the breast from the alveoli to the skin are lined with columnar epithelium
Answer: b, c

The breast abuts against the fascia of the pectoralis major and serratus anterior muscles. Projections of the fascia course
through the breast to the skin, forming a supporting framework of the breast parenchyma. These fascial bands, called
suspensory ligaments of Cooper, are better developed in the upper breast. The structure of the breast can be divided into
lobular and ductal elements. The lobule is the functional unit of the breast. Within a lobule, the terminal elongated tubular
ducts are referred to as alveoli. Ten to one hundred alveoli coalesce to form a larger duct which defines a lobular unit. The
lobular ducts join to form progressively larger ducts and ultimately an excretory duct. The alveolar ducts, lobular ducts, and
excretory ducts are all lined with either cuboidal or columnar epithelium. Eventually, 10-20 excretory ducts, each dilate into
a short excretory sinus (lined with squamous epithelium) just beneath the areola. Excretory ducts then course perpendicular
to exit through the nipple.
The lymphatic anatomy of the breast is of interest to the surgeon because of the tendency of breast cancer to involve the
regional lymph nodes. Studies using radioactive tracers demonstrate at least 97% of lymphatic flow from the breast is into
the axilla; the remainder courses into the internal mammary nodes. These studies also show that lymph flowing into the
internal mammary gland chain is not restricted in origin to the medial half and sub-areolar region of the breast, as was
thought, but can originate in any quadrant of the breast. In the axilla, lymphatic vessels terminate in the lymph nodes
embedded within the axillary fat pad. Also within the axillary fat pad are the intercostal brachial nerves (a sensory nerve
supply in the under arm), the long thoracic nerve (a motor nerve to the serratus anterior and subscapularis muscles) and the
thoracodorsal nerve (a motor nerve to the latissimus dorsi adjacent to its accompanying arteries and veins).

14. Which of the following statement (s) is/are true concerning the recurrence of breast cancer?

a. The majority of patients recur within five years of diagnosis


b. More than 70% of breast cancer recurrence involve distant metastases
c. Pulmonary metastases are the most common initial site of distant recurrence
d. The local recurrence rate following breast-conserving procedures varies from 10% to 40% whether or not radiation
was used
e. Recurrent disease will be seen in at least 35% of node-negative patients undergoing appropriate primary breast
therapy
Answer: a, b, d

Metastatic disease following primary therapy for breast cancer can recur at any time. However, of those who relapse, 50%
to 70% do within two years and over 85% relapse within five years. More than 70% of recurrences are distant, but anywhere
from 10% to 30% of recurrences are local. Bone and lung are the most common initial sites of distant relapse (50% and
25%), respectively. A breast-conserving procedure can be associated with a local tumor recurrence rate. The rate of local
recurrence falls from 40% to 10% if postoperative radiation therapy is given to the entire breast. Despite potentially curative
resection, at least 20% of node-negative and 60% of node-positive breast cancer patients have recurrence of their disease at
some time after surgery.

15. Which of the following statement(s) is/are true concerning mammography?

a. Up to 50% of cancers detected mammographically are not palpable


b. One third of palpable breast cancers are not detected by mammography
c. The sensitivity of mammography increases with age
d. The American Cancer Society currently recommends routine screening mammography beginning at age 40
e. Only about 10% of nonpalpable lesions detection mammographically are found to be malignant at biopsy
Answer: a, c, d

Although mammography has been available for years, it did not become widely used until the findings of the Health
Insurance Plan of New York and the Breast Cancer Detection Demonstration project studies of screening mammography
were disseminated. These and other investigators demonstrated that 10%–50% of cancers detected mammographically are
not palpable. Conversely, palpation recognizes 10%–20% of tumors not detectable mammographically. The incidence of
breast cancer begins to rise sharply at age 40, and the sensitivity of mammograms increases with age as the dense
parenchymal tissue of young women is progressively replaced by fatty tissue. Routine screening mammography has been
shown to decrease breast cancer-related mortality in asymptomatic women over the age of 50. Controversy exists
concerning the role of screening in younger woman. However, currently the American Cancer Society recommends that
mammographic screening begin at age 40. Although sensitive, mammography is not specific. Only about 25% of
nonpalpable lesions detected mammographically are found to be malignant at biopsy. A spiculated density with ill-defined
margins on mammogram is almost certainly malignant. Most commonly, features are seen that are suggestive but not
diagnostic of cancer. These include clustered microcalcifications, asymmetric density, ductal asymmetry, and distortion of
normal breast architecture and/or skin or nipple distortion.

16. Which of the following statement(s) is/are true concerning radiation therapy after lumpectomy?

a. The total dose given to the breast is usually in the range of 2500 to 3000 cGy
b. Radiation to the axillary nodal bed is normally part of the procedure in most patients
c. Long-term complications of radiation therapy include rib fractures and arm edema
d. Breast edema and skin erythema usually resolves within a few weeks
e. None of the above
Answer: c

Breast conservation usually involves the use of lumpectomy and radiation therapy to achieve local control of breast cancer.
Any technique used for post-lumpectomy radiation of the breast must adequately cover the volume at risk, deliver a
homogenous dose throughout the target tissues, avoid overlapping or inadequate apposition of fields, and minimize the dose
reaching the heart and lung. The entire breast should be treated with a total dose of 4500 to 5000 cGy. There is no good
evidence to support a radiation boost to the site of the primary tumor. Complications from breast radiation are uncommon if
performed correctly. Acute complications of radiotherapy include fatigue, breast edema, and skin erythema; these are almost
always self-limited and resolve over weeks (fatigue) 2 months (erythema) or years (edema). The most common long-term
problems are rib fractures and minor arm edema, each of which occur about 5% of the time.

17. A 35-year-old woman, who is currently breast-feeding her firstborn child, develops an erythematous and inflamed
fluctuant area on breast examination. Which of the following statement(s) is/are true concerning her diagnosis and
management?

a. The most common organism which would expect to be cultured is Staphylococcus aureus
b. Open surgical drainage is likely indicated
c. Breast-feeding absolutely should be discontinued
d. If the inflammatory process does not completely respond, a biopsy may be indicated
Answer: a, b, d

Infection complicates breast-feeding in fewer than 1:100 women, but these lactational infections still account for 80% of all
breast infections. Presumably, gaining access via the skin of the irritated nipple of the nursing woman, Staphylococcus
aureus is by far the most common pathogen in this setting. Many breast infections begin as cellulitis, without abscess
formation. When an actual abscess is suspected, percutaneous aspiration can establish the diagnosis and allow for bacterial
culture and sensitivity testing. Open surgical drainage is the most prudent and effective treatment. Although women may
choose to cease breast feeding, there is no absolute indication for this. When mastitis or breast infection is suspected
clinically, the possibility of an inflammatory carcinoma must also be entertained. Any inflammatory process that does not
respond completely and promptly to antibiotics or drainage should be subjected to biopsy to rule out cancer.

18. Which of the following statement(s) is/are true concerning the surgical staging of breast cancer?

a. All biopsy specimens should be transported to pathology in formalin within 24 hours of the procedure
b. Removal of only level I axillary lymph nodes may understage breast cancer in up to one-fourth of patients
c. Level III axillary lymph nodes should be removed in all axillary lymph node dissections
d. A clinically negative axilla will be found to have histologically positive metastasis in approximately one-third of
patients
Answer: b, d

Pathologic staging begins with the initial biopsy. Unless previously secured, fresh tumor needs to be obtained for hormone
receptor analysis prior to placement into formalin solution. A period of warm ischemia as short as 30 minutes may cause
underestimation of estrogen receptor levels. The need to remove axillary nodes must be determined preoperatively. Axillary
lymph node metastasis will be found in approximately one-third of clinically negative axillae, but only if proper axillary
dissection is performed. Removal of only level I nodes or “sampling” of axillary lymph nodes in a haphazard fashion
increases the risk of injury to major axillary neurovascular structures and may understage up to 25% of women. Proper
staging of axillary lymph nodes should include en bloc removal and examination of level I and level II nodes. When
conducted for staging, axillary lymph node dissection should not include removal of level III axillary nodes; in fewer than
2% are metastases present in level III nodes when level I and level II nodes are negative. Removal of level III nodes,
however, does increase the incidence of postoperative arm lymph edema almost fivefold. Therapeutic axillary lymph node
dissection performed for palpable disease in the axilla should include removal of all levels to clear gross disease.

19. Which of the following statement(s) is/are true concerning the effect of various hormones on breast physiology?

a. Estrogen receptors are present only in breast cancer cells


b. Mammary ductal dilatation and differentiation of alveolar epithelial cells and secretory cells are the result of rising
progesterone levels
c. The early first trimester breast changes are primarily due to the increased progesterone effects of pregnancy
d. Milk production and secretion after childbirth are maintained by ongoing secretion of prolactin by the anterior
pituitary gland
Answer: b, d

Breast growth, development, and function are orchestrated by a variety of hormones and growth factors. Estrogen plays a
central role in breast development, growth, and differentiation. Lipid-soluble estrogens gain entry to the normal and
malignant breast cell by diffusing to the cell membrane. Once within the cell, estrogens bind with the estrogen receptor.
Both normal and malignant breast cells contain estrogen receptors, but the low levels of receptors in normal breast tissue
and in some breast cancers result in their testing negative in clinical assays. Cyclic changes associated with the menstrual
cycle have a profound influence on breast morphology and physiology. During the period of relative quiescence, increasing
Graafian follicle secretion of estrogen stimulates breast epithelial proliferation. As the luteal phase of the cycle is entered,
progesterone levels rise. Mammary ductal dilatation and differentiation of alveolus epithelial cells into secretory cells result.
At the onset of menstruation, the rapid decline of circulating sex-hormone levels leads to breast involution and the cycle
begins anew. During pregnancy, marked ductular, lobular, and alveolar growth occur under the influence of estrogen,
progesterone, placental lactogen, prolactin, and chorionic gonadotropin. These changes prepare the breasts for milk
production at parturition. Early in the first trimester, ductal sprouting and lobular formation proceed under estrogenic
influence. During the second trimester, lobular events predominate under the influence of progestins. Abrupt withdrawal of
placental lactogen and sex-hormones that occurs with delivery, leaves the breast predominately under the influence of
pituitary-derived prolactin. Milk production and secretion are maintained during lactation by ongoing secretion of prolactin
by the anterior pituitary.

20. A pre-menopausal woman three years after mastectomy for breast cancer presents with pulmonary metastases. Which
of the following statement(s) is/are true concerning her management?

a. If the patient has received adjuvant therapy, her response is likely to be better
b. If the patient is ER-positive, hormonal therapy should be the first line of treatment
c. The response to chemotherapy will likely be dose-dependent
d. Combination chemotherapy will likely work better in this patient than a woman who is post-menopausal
Answer: b, c, d

Chemotherapy for metastatic breast cancer is more likely to be employed for young women, those with ER-negative tumors,
those with visceral organ involvement and those with rapidly advancing or life-threatening disease. Generally, combinations
of agents are used in treating metastatic breast cancer with the response rate usually dose-dependent. All regimens are
slightly less active in post-menopausal women. Response rates are highest in women who have not received prior treatment
for metastatic disease. Prior adjuvant therapy is not consistently associated with a poorer response to therapy, particularly if
a long interval has lapsed between adjuvant therapy and the development of metastases. Endocrine therapy is appropriate as
the first-line treatment for nearly all women with ER-positive metastatic breast disease. Tamoxifen is the agent of choice for
first-line hormonal therapy for metastatic breast cancer. Both pre-menopausal and post-menopausal patients can receive this
agent and side effects are minimal.

21. Which of the following statement(s) is/are true concerning intraductal papilloma?

a. This lesion is the most common cause of bloody nipple discharge


b. Serous non-bloody discharge is unlikely to be due to an intraductal papilloma
c. A nonpalpable lesion can often be diagnosed with ductography
d. An isolated lesion is considered premalignant
Answer: a, c

Intraductal papilloma represents the most common cause of bloody nipple discharge, although in half of the cases, the
discharge is serous. Since the average size of an intraductal papilloma is 3–4 mm., they are rarely palpable. Ductography
may demonstrate the lesion, or it may be found after subareolar duct excision performed to treat the discharge. An isolated
intraductal papilloma is not considered premalignant nor does it place the patient at increased risk for breast cancer. Unlike
isolated papillomas, diffuse papillomatosis is associated with an increased risk of breast cancer, perhaps as high as in 40%
of women.

22. A 21-year-old woman presents with an asymptomatic breast mass. Which of the following statement(s) is/are true
concerning her diagnosis and treatment?

a. Mammography will play an important role in diagnosing the lesion


b. Ultrasonography is often useful in the differential diagnosis of this lesion
c. The mass should always be excised
d. The lesion should be considered pre-malignant
Answer: b

Fibroadenoma represents the most common tumor in adolescents and young woman, but if also frequently encountered in
older women. It generally presents as a palpable breast mass and must be differentiated from cancer. Typically,
fibroadenoma presents as a painless, slow-growing mass found incidentally on breast self examination. Palpation of a mass
usually reveals a well-circumscribed, oval or round, mobile mass with a firm, rubbery texture. Because the mammographic
appearance of a fibroadenoma is rarely characteristic, mammography plays little role in diagnosing this lesion.
Ultrasonography can differentiate a solid mass from a cyst. Additionally, the ultrasonic appearance of a well-marginated,
homogenous mass may be sufficiently characteristic to permit diagnosis of fibroadenoma. Excisional biopsy is not
necessary for every fibroadenoma. Women under 30 years of age with characteristic physical examination and sonographic
appearance of the fibroadenoma may be given the option of observation. Generally, fibroadenomas are not felt to be pre-
malignant lesions, nor to indicate any increased risk for the development of breast cancer.

23. Which of the following are factors associated with an increased risk for developing breast cancer?

a. Nulliparity
b. Oophorectomy before age 35
c. Use of oral contraceptives
d. High-fat, high-caloric diet
e. Post-menopausal use of conjugated estrogens
Answer: a, d

Women who undergo oophorectomy before age 35 and do not take replacement estrogens have a two-thirds reduction in
their breast cancer risk. Replacement estrogen therapy eliminates the beneficial effect of oophorectomy. Most investigations
of oral contraceptive use do not demonstrate an associated increased risk of breast cancer development. Studies of estrogen
replacement therapy for post-menopausal women have yielded equivocal results. Most contemporary studies fail to
demonstrate an association between breast cancer risk and post-menopausal use of conjugated estrogens.
BREAST CANCER RISK FACTORS

DEMOGRAPHIC FACTORS
Age more than 30 y
Female gender (130:1 female/male ratio)

GREATLY INCREASED RISK


Known carrier of breast cancer susceptibility gene
Strong family history—two or more first-degree relatives with
bilateral or premenopausal breast cancer
Atypical ductal or lobular hyperplasia or lobular carcinoma in situ
Ductal carcinoma in situ, risk limited to ipsilateral breast

MODERATELY INCREASED RISK


Family history—one or more relatives with breast cancer, not
bilateral or premenopausal
Menstrual history—menarche before age 12 y, menopause after
age 55 y
Parity—nulliparity or first live birth after age 30 y
Radiation—exposure to low-dose ionizing radiation in childhood or
adolescence
Previous breast cancer—low-grade, node-negative, or receptor-
positive; lobular histology
Other cancers—colon or endometrial cancer
Diet—high-fat or high-calorie diet

24. Which of the following chromosomal and/or genetic abnormalities is/are associated with the development of breast
cancer?

a. Mutations in the p53 tumor suppressor gene


b. A mutation in the short arm of chromosome 2
c. The presence of a BRCA 1 gene on chromosome 17
d. The presence of the BRCA 2 gene on chromosome 13
Answer: a, b, c, d

There are four inherited syndromes associated with the development of breast cancer. The Li-Fraumeni syndrome has an
autosomal dominant mode of inheritance. The syndrome is attributed to mutations in the p53 tumor suppressor gene, a gene
that codes for a protein that serves as a G1-S checkpoint regulator of the cell cycle. More recently, a mutation has been
characterized on the short arm of chromosome 2 in a gene associated with DNA repair. Predisposition to a wide range of
malignancies, including breast and colon cancer is associated with abnormalities at this locus. The most exciting
development in inherited susceptibility to breast cancer relate to the identification and cloning of the BRCA 1 gene, which
was initially localized on the long arm of chromosome 17 by linkage analysis. Germline abnormalities in BRCA a may be
responsible for as many as 5% of all breast cancers in the United States. The gene is characterized by autosomal dominant
inheritance with a high degree of penetrance. Almost 60% of women inheriting the gene will develop breast cancer by age
50, and a lifelong risk approaches 85%. Another breast cancer susceptibility gene, dubbed BRCA 2, has been localized by
linkage analysis to a small region of chromosome 13q12-13. BRCA 2 apparently confers the high-risk of early onset female
breast cancer. Similar to BRCA 1, the lifetime breast cancer risk approaches 90% in carriers of this gene.

25. A 45-year-old woman presents with a weeping eczematoid lesion of her nipple. Which of the following statement(s)
is/are true concerning her diagnosis and management?

a. Treatment is with warm compresses and oral antibiotics


b. Biopsy of the nipple revealing malignant cells within the milk ducts is invariably associated with an underlying
invasive carcinoma
c. The appropriate treatment is mastectomy
d. The lesion always represents a high-risk disease with a significant risk of subsequent metastatic disease
Answer: c

Paget’s disease is characterized by weeping, eczematoid lesion of the nipple. There is often accompanying edema and
inflammation. Biopsy of the nipple reveals malignant cells within the milk ducts. The lesion is invariably associated with an
underlying invasive or in situ ductal carcinoma. The prognosis of Paget’s disease is that of the underlying cancer. Standard
treatment is mastectomy with axillary lymph node dissection only if invasive cancer is present.

26. Which of the following treatment(s) is/are of proven benefit in the treatment of mastodynia associated with fibrocystic
breast disease?

a. Avoidance of methylxanthine compounds, particularly caffeine


b. Cessation of smoking
c. Vitamin E
d. Danazol
Answer: a, b, d

The relationship of methylxanthines, particularly caffeine, to mastodynia and breast nodularity remains controversial. Most
women do, however, experience diminution of their symptoms and are subject to improvement in breast nodularity by
limiting or eliminating caffeine intake. Mastodynia patients should be advised to eliminate caffeine beverages for a period
of 2 to 3 months to determine if there has been improvement in their symptoms. In addition to caffeine abstention, patients
should be urged to stop smoking because nicotine is purported to worsen mastodynia. A number of medications have been
advocated for the treatment of mastodynia. Unfortunately, because of the subjective nature of the disease and its propensity
to be better tolerated by patients with reassurance, the exact method of most of these interventions is unclear. Vitamin E has
been touted as beneficial, however, clinical data do not support the use of this or other vitamins for this condition. The use
of hormonal agents to treat mastodynia has been more extensively treated. Danazol, a weak antigen, is the most effective
drug available for treatment of mastodynia related to fibrocystic disease. Unfortunately, Danazol’s androgenic side effects
are troublesome enough to restrict its use to the most problematic cases of mastodynia. Other hormonal agents have been
investigated for the management of mastodynia. In young women, oral contraceptives have a variable effect on mastodynia.
A trial and error search for optimal preparations may be necessary as the effect of oral contraceptives is dependent on the
formulation of the pill.

27. Which of the following statement(s) is/are true concerning breast reconstruction?

a. The timing of breast reconstruction is of no oncologic significance


b. Breast reconstruction may interfere with detection of local recurrence of breast cancer
c. Maintenance of an effective subpectoral pocket for a breast implant requires preservation of the pectoralis fascia
d. Because of its complexity, the TRAM flap is seldom used for primary breast reconstruction
Answer: a, c
Breast reconstruction is suitable for any woman who has undergone mastectomy who desires reconstruction. Breast
reconstruction may be performed at the time of mastectomy (immediate) or sometime subsequently (delayed) Because the
presence of reconstruction may interfere with the accurate planning and administration of radiation therapy, reconstruction
is generally delayed if the use of local or regional radiation therapy is anticipated. Otherwise, timing of breast reconstruction
is of no oncologic significance. Because most local recurrences occur in the skin’s subcutaneous tissues, the presence of a
reconstruction will not interfere with detection. Similarly, a reconstruction does not complicate the administration of
chemotherapy.
Breast reconstruction techniques utilize either autogenous tissue or synthetic prostheses to recreate a breast mound.
Prosthetic reconstruction is usually accomplished by sub-pectoral placement of a saline-or silicone gel-filled implant.
Maintenance of an effective sub-pectoral pocket for an implant requires preservation of the pectoralis fascia and the medial
pectoral nerve during mastectomy. The transferase rectus abdominous myocutaneous (TRAM) flap is the autogenous
reconstruction of choice. The TRAM operation is complex and time consuming. Despite the magnitude of the procedure, it
is still commonly used for immediate reconstruction.

28. Which of the following statement(s) is/are true concerning the histologic variants of invasive breast carcinoma?

a. The presence of an in situ component with invasive ductal carcinoma adversely affects prognosis
b. Medullary carcinomas, although often of large size, are associated with a better overall prognosis than common
invasive ductal cancers
c. Mucinous or colloid carcinoma is one of the more common variants of invasive ductal cancer
d. Invasive lobular carcinoma is associated with a higher incidence of bilateral breast cancer
Answer: b, d

Although the breast is composed of both lobular and ductal elements, most breast cancer arises in the ductal elements.
Invasive ductal carcinoma accounts for 70% to 80% of all cases of breast cancer. Although there is no single microscopic
feature specific for infiltrating ductal carcinoma, it can be recognized histologically as an invasive adenocarcinoma
involving ductal elements. The malignant ductal cells are often dispersed within the fibrous stroma, leading to the
appellation of scirrhous carcinoma. A number of less common types of breast cancer arise from the ductal epithelium and
are hence classified as variants of invasive ductal carcinoma. There are distinct histologic criteria for classifying these
lesions; these criteria must be met throughout the entire tumor. Prognostically, histologically pure examples of these variant
tumors are associated with a better long-term survival than ordinary type invasive ductal carcinoma. When mixed
histologies are encountered, the clinical behavior parallels that of the invasive ductal element, not the other sub-type. Hence,
these mixed tumors are considered together with pure invasive ductal carcinoma for prognostic purposes. In many cases,
when areas of in situ ductal carcinoma are seen, the presence of an in situ component does not adversely affect prognosis,
although it jeopardizes the attempts at breast conservation. Medullary carcinoma is one of the more common variants,
accounting for approximately 6% of all invasive breast cancers. These tumors may grow to be a rather large size within the
breast (5 to 10 cm) and are characteristically well-circumscribed. Mucinous carcinoma, also referred as colloid carcinoma,
is encountered in 1% to 2% of breast cancer cases. Invasive lobular carcinoma arises from the lobular component of the
breast and in most series accounts for approximately 10% of breast cancers. Almost every series has stressed the higher
incidence of bilateral cancer in patients with invasive lobular carcinoma. The contralateral breast is involved either
synchronously (3% of patients) or metachronously in up to 30% of patients.

29. Which of the following statement(s) is/are correct concerning cystosarcoma phyllodes?

a. The tumor is most commonly seen in post-menopausal women


b. Total mastectomy is necessary for all patients with this diagnosis
c. Axillary lymph node dissection is not necessary for malignant cystosarcoma phyllodes
d. Most patients with the malignant variant of cystosarcoma phyllodes die of metastatic disease
Answer: c

Cystosarcoma phyllodes is a tumor arising in the mesenchymal tissue of the breast. The tumors usually present as a painless
breast mass. Phyllodes tumor is most commonly encountered in women age 30–40 years of age but can occur at any age,
even before puberty. The differentiation of a benign from a malignant phyllodes tumor may be difficult. About one-fourth of
all phyllodes tumors are histologically malignant, but only a fraction of these patients actually develop metastatic disease.
The optimum treatment for benign or malignant phyllodes tumor is wide excision with a margin of normal breast tissue. The
margin must be histologically free of involvement because even benign lesions can recur after incomplete excision. If this
can be done leaving an adequate cosmetic appearance, mastectomy is not necessary. Total mastectomy is reserved for large
lesions in small-breasted women or recurrences after previous local excision that is not amenable to repeat local excision.
Axillary lymph node dissection is not performed in the absence of biopsy-proven nodal involvement, even for malignant
phyllodes tumors, because axillary metastases are uncommon.

30. Which of the following statement(s) is/are true concerning local recurrence of breast cancer?

a. The percentage of patients with chest wall recurrence as their initial site of failure following mastectomy is similar
for node-negative and node-positive patients
b. Most patients with local-regional recurrence of their disease will eventually die of metastatic disease
c. The treatment of local recurrence following mastectomy includes local radiation therapy and systemic
chemotherapy
d. In-breast recurrence following breast conserving surgery is not a negative prognostic factor
e. Regional lymph node recurrence following axillary node dissection is rare
Answer: a, b, c, e

Recurrence in the chest wall after mastectomy is ominous. In a large series of patients treated with mastectomy, 6.5% of
node-negative and 8.8% of node-positive women had chest wall recurrence as their initial site of failure. By ten years after
local-regional recurrence, about 60% of initially node-negative and almost all (> 90%) of initially node-positive patients had
evidence of metastatic disease. Patients with local recurrence, who have not had prior chest wall radiation, should receive
radiation therapy. A full course of at least 4500 to 5000 cGy should be delivered to the entire chest wall, with consideration
given to a boost dose at any sites of gross tumor. Because post-mastectomy recurrence is often rapidly followed by
metastatic disease, it is logical to postulate a role for adjuvant systemic therapy once local measures have achieved control
of chest wall disease.
Recent data suggests that in-breast recurrence following breast conservation is a prognostic factor. Women who develop an
in-breast recurrence have a higher likelihood of developing systemic disease than do women who remain disease-free in
their breast. Fewer than 3% of patients develop recurrence of disease in the axilla after axillary node dissection.

31. Which of the following statement(s) is/are correct concerning prognostic factors for breast carcinoma?

a. Prognosis is improved with estrogen or progesterone receptor positivity


b. Increased thymidine labeling index, a measure of the proportion of cells in the DNA synthetic phase (S-phase), is
associated with improved survival
c. High tumor levels of cathepsin D are associated with an improved prognosis
d. Immunohistochemical demonstration of active angiogenesis correlates with increased metastatic potential and poor
prognosis
Answer: a, d

32. Which of the following statement(s) is/are true concerning adjuvant systemic therapy?
a. Adjuvant tamoxifen in post-menopausal, node-positive, ER-positive women is equivalent to cytotoxic
chemotherapy
b. Tamoxifen clearly improves survival in all hormonal receptor-positive patients
c. CMF is associated with improved overall survival in both pre-menopausal and post-menopausal node-positive
patients
d. There is no evidence to suggest a role for chemotherapy in node-negative patients
Answer: a

Adjuvant tamoxifen leads to a prolonged disease-free interval in post-menopausal ER-positive women with histologically
positive nodes and in pre-menopausal and post-menopausal ER-positive women with negative nodes. Because of similar
results and, because tamoxifen is generally less toxic than chemotherapy, this treatment is the treatment of choice for post-
menopausal, node-positive, ER-positive women. CMF (cyclophosphamide, methotrexate, and 5-fluorouracil) is associated
with both a longer disease-free survival and overall survival time in pre-menopausal patients with positive lymph nodes. In
post-menopausal women with positive nodes, there is an improved disease-free survival, but there is no significant
difference in overall survival. Several trials of adjuvant chemotherapy with CMF or related regimens have been conducted
in node-negative patients. The early results of all of these trials have been similar: disease-free survival is definitely
improved with adjuvant chemotherapy. These studies are definitely not mature enough to draw definitive conclusions
regarding overall survival. Therefore, the National Cancer Institute has recommended the use of adjuvant chemotherapy for
all patients with tumors large enough to have hormonal receptor levels measured.

33. Which of the following statement(s) is/are true concerning tissue sampling techniques for breast masses?

a. The sensitivity of fine needle aspiration biopsy is such that mastectomy can be performed in the case of malignant
diagnosis
b. The accuracy of mammographic-directed fine needle aspiration biopsy is comparable to that achieved for that of
palpable lesions
c. Core-needle biopsy showing normal breast tissue is an acceptable diagnosis
d. The technique of core-needle biopsy is not applicable to radiographically detected lesions
Answer: b

Whatever tissue sampling method is chosen, only biopsy (examination of cells or tissue) and not physical examination or
mammography can establish a definitive diagnosis and avoid delay in treatment. Fine needle aspiration biopsy (FNAB)
permits rapid, minimally invasive diagnosis of many palpable and some non-palpable, radiologically detected breast masses.
The technique is both reliable and accurate. The incidence of false-positive findings is generally less than 0.5%. FNAB is
not, however, so highly specific that definitive surgery (particularly mastectomy) should be performed without prior
intraoperative frozen-section confirmation of the presence of cancer. Reported sensitivity of FNAB ranges from 7% to 99%;
with 85% a good estimate of the true sensitivity in clinically relevant settings. Recently, x-ray-guided FNAB has been used
to offer minimally invasive diagnosis in nonpalpable breast lesions detected mammographically. The technique is quite
effective, especially for mass lesions. Accuracy is comparable to that achieved with FNAB of palpable lesions. Core-needle
biopsy is a helpful tissue sampling method for palpable masses. The tissue obtained is useful for histologic analysis
although inadequate for cytosol hormone receptor determination. The technique is also applicable by using mammographic
guidance for nonpalpable lesions.

34. A 42-year-old woman undergoes her first mammogram. Clustered microcalcifications are seen but there is no mass
palpable. Which of the following statement(s) is/are true concerning this patient’s diagnosis and management?

a. A needle localization and excision of the mass is necessary to establish the diagnosis
b. Frozen-section examination is particularly useful in the diagnosis of this lesion
c. Intense interlobular fibrosis and proliferation of small ductules with loss of orientation of lobules and epithelial
cells may suggest carcinoma
d. This finding is associated with an increased risk of cancer
Answer: a, c

Sclerosing adenosis is a histologic subtype of fibrocystic change that is not associated with an increased risk of cancer
development. It is, however, one of the benign breast processes most likely to be confused radiologically and histologically
with cancer. Most commonly, it is detected on routine mammography as cluster microcalcifications without an associated
palpable mass. In these cases, needle localization and excision are required to establish a diagnosis. Sclerosing adenosis
microscopically is characterized by interlobular fibrosis and proliferation of small ductules. If the fibrous component is
particularly intense, the orientation of lobules and epithelial cells may be lost, mimicking carcinoma. Differentiating
sclerosing adenosis from cancer on frozen-section examination can be particularly difficult and should not be attempted.

35. Which of the following conclusion(s) can be drawn from the results of the NSABP prospective randomized trials
completed in the 1970’s and 1980’s?

a. Delay of axillary node dissection until there is clinical evidence of disease does not influence overall survival
b. Removal of clinically negative nodes has no therapeutic benefit
c. Breast irradiation reduces both local recurrence and overall survival
d. Modified radical mastectomy offers no advantage of lumpectomy with axillary node dissection
Answer: a, b, d

The scientific basis of local-regional treatment strategies for stage I and stage II breast cancer was established by a series of
studies conducted during the 1970’s and 1980’s by the NSABP. In the first of these protocols, total mastectomy with delayed
node dissection only for nodes that subsequently turned positive, total mastectomy with local-regional radiation therapy, and
radical mastectomy were clinically equivalent. Furthermore, the finding that delay of axillary node dissection until there is
clinical evidence of disease does not influence survival emphasizes that the role of axillary dissection in clinically node
negative patients is solely for staging. The removal of clinically negative nodes has no therapeutic benefit if regional
recurrences are detected and treated promptly. In the second of these protocols, modified radical mastectomy, lumpectomy
with axillary node dissection, and lumpectomy, axillary node dissection, and breast or irradiation were compared in small
breast cancers. Modified radical mastectomy offered no advantage over other treatments when analyzed by disease-free or
overall survival in either node-negative or node-positive patients. Breast irradiation after lumpectomy reduced the likelihood
of in-breast tumor recurrence from 39% to 10% but did not affect overall survival when compared with lumpectomy alone.

36. Which of the following statement(s) is/are true concerning non-invasive breast carcinoma?

a. Ductal carcinoma in situ (DCIS) is associated with a significant risk of development of invasive ductal carcinoma
in the same quadrant of the same breast as the initial lesion
b. DCIS should not be treated with breast conservation therapy
c. Lobular carcinoma in situ (LCIS) is the most common form of non-invasive breast cancer
d. When LCIS is found, there is an up to 50% chance of lobular carcinoma in situ of the contralateral breast
e. About one-third of patients with biopsy-proven LCIS develop invasive cancer, always of the same breast
Answer: a, d

Non-invasive (in situ) cancer is defined as a neoplastic entity within the epithelium of origin and without invasion to the
basement membrane. Ductal carcinoma in situ (DCIS) arises from the ductular elements. The age distribution of DCIS does
not differ significantly from that of invasive ductal carcinoma. Not every woman who undergoes complete excision of a
focus DCIS develops invasive ductal cancer. Various studies suggest half or more patients develop invasive breast cancer
after excisional biopsy alone. When a subsequent invasive cancer does occur, it is almost always of the invasive ductal type
and located in the same quadrant of the breast as the initial DCIS. The latent period before the development of invasive
cancer usually exceeds five years. Total mastectomy is usually associated with a nearly 100% cure rate for this condition.
Although total mastectomy remains the gold-standard for treatment of DCIS, there is increasing experience with breast-
conserving therapy. Breast conservation may be offered to DCIS patients in whom the entire tumor can be surgically
removed with negative histologic margins and in whom the remaining breast tissue can be reliably assessed clinically and
radiographically. It would appear that the disease-free survival following lumpectomy and radiation therapy is worse than
that achievable with simple mastectomy. Therefore, breast conservation for DCIS commits patients to more careful long-
term follow-up and will likely subject them to additional subsequent treatment to deal with the recurrences. Lobular
carcinoma in situ (LCIS) accounts for one-third of the non-invasive breast cancers. LCIS patients are significantly younger
than patients with invasive breast cancer. Three-fourths of affected women are pre-menopausal. LCIS is an infrequent
finding in women over 75. When the opposite breast is sampled at the time of diagnosis, contralateral LCIS is found in 30–
50% of cases. The prognosis of LCIS is solely related to the subsequent development of invasive carcinoma. About one-
third of patients with biopsy-demonstrated LCIS develop invasive cancer; half occur in the index breast and half in the
contralateral breast. The subsequent breast cancers can be either lobular or ductal in histology.

37. A 33-year-old woman is referred with nipple discharge. Which of the following statement(s) is/are true concerning her
diagnosis and management?

a. Bilateral galactorrhea is suggestive of an underlying endocrinopathy


b. Brownish discharge is usually suggestive of old blood and is worrisome for an underlying breast cancer
c. Expressible bloody nipple discharge should be evaluated with a ductogram
d. Milky breast discharge would not be expected one year after discontinuation of breast feeding
Answer: a, c

At one time or another, many women notice a nipple discharge. The most common physiologic basis for nipple discharge is
lactation. Milk may continue to be secreted intermittently for as long as two years after breast feeding has stopped,
particularly with breast stimulation. A milky whitish discharge, usually bilateral, that is not related to lactation or breast
stimulation is termed “galactorrhea.” The presence of bilateral galactorrhea should prompt an evaluation for underlying
endocrinopathy causing increased prolactin secretion by the pituitary. Classically, this is associated with amenorrhea, but
galactorrhea may be the only sign of hypoprolactinemia. Nipple discharges associated with fibrocystic disease are generally,
green, yellow, or brown, Intraductal papillomas and cancer lead to a bloody or blood-tinged serous discharge. The brownish
discharge of fibrocystic disease can easily be confused with old blood. A guaiac test or simply dabbing the discharge with a
gauze pad and examining the stain can usually differentiate the two. A bloody or blood-tinged discharge must be promptly
evaluated to exclude carcinoma. If the discharge is expressible at the time the patient is seen, a contrast ductogram may be
obtained.

38. Clinical features of breast cancer which are associated with a particularly poor prognosis include:

a. Edema of the skin of the breast


b. Skin ulceration
c. Lateral arm edema
d. Dermal lymphatic invasion
Answer: a, b, c, d

The histologic hallmark of inflammatory breast cancer is dermal lymphatic invasion demonstrable on skin biopsy. The
stigmata of this clinical syndrome include breast warmth, tenderness, erythema, and edema.

39. Which of the following statement(s) is/are associated with gynecomastia?

a. If the disease is unilateral, it is unlikely drug-related


b. The standard surgical treatment is subcutaneous mastectomy
c. The presence of gynecomastia is often associated with the subsequent development of breast cancer
d. A formal endocrine evaluation is indicated in most patients with gynecomastia
Answer: b

Gynecomastia is defined as palpable enlargement of the male breast. Pathologic causes of estrogen excess or testosterone
deficiency are associated with gynecomastia. In many cases, no cause is found. Clinically significant gynecomastia has been
associated with the use of a number or drugs including cimetidine, digoxin, spironolactone and tricyclic antidepressants.
The use of marijuana has also been associated with gynecomastia. Drug-related gynecomastia is often unilateral or unequal
between the two breasts, and discontinuation of the offending drug does not always lead to resolution of the condition. A
formal endocrine evaluation is not indicated for gynecomastia unless some other sign of hormonal imbalance is found on
routine evaluation. The standard surgical treatment of gynecomastia consists of subcutaneous mastectomy performed under
local anesthesia. The presence of gynecomastia is not associated with the subsequent development of cancer, yet protracted
hyperestrogenemic states, which are associated with gynecomastia are linked to breast cancer development.

1. The bronchial circulation:


A. Is the blood supply to the conducting airways.
B. Drains into a peribronchial venous network that may expand considerably with conditions such as bronchiectasis and
chronic obstructive pulmonary disease.
C. Is an especially important consideration in pulmonary transplantation.
D. All of the above.
Answer: D

DISCUSSION: The bronchial circulation is the primary blood supply for the conducting airways, pulmonary vessels,
lymphoid tissue, and squamous cell carcinomas. In conditions such as mitral stenosis, bronchiectasis, or chronic obstructive
pulmonary disease, the rich peribronchial venous network that drains the bronchial circulation may expand considerably,
creating significant left-to-right shunts. Whenever the pulmonary artery circulation is obstructed, there is a tendency for
bronchial circulation to increase; thus, the bronchial circulation is an important consideration during lung transplantation as
well as in the surgical treatment of cyanotic congenital heart disease and chronic pulmonary embolism.

2. Clearance of mucus produced in the tracheobronchial tree in chronic bronchitis secondary to smoking may:
A. Be hampered by the fact that the amount of mucus is increased by the number of mucus-producing cells at the expense
of ciliated cells.
B. Be slowed if patients have decreased lung volume and are therefore unable to generate a vigorous cough that would
cause an inflammatory process.
C. Cause a decrease in diffusion capacity and associated hypoxemia.
D. All of the above.
Answer: A

DISCUSSION: Chronic bronchitis may have an acute component, and in these patients therapy with antibiotics and
bronchodilators may improve the flow rate as measured by pulmonary function tests within 3 or 4 days of the cessation of
smoking and treatment of the acute condition. However, the chronic bronchitic will continue to produce large amounts of
mucus, most evident in the morning, even after the acute process has been resolved. Clearance of these secretions is
hampered by the inability to cough, perhaps secondary to the pain of thoracotomy or abdominal surgery or by a decrease in
the number of ciliary cells that help move mucus up the tracheobronchial tree. This causes plugging of small airways and
atelectasis, which may progress to pneumonia. For this reason, cessation of smoking for 3 to 5 days before surgery is very
beneficial in preventing pulmonary complications during the postoperative period.

3. The pulmonary circulation:


A. Is the only vascular system in which the veins do not have the same course as the arteries.
B. Has a direct connection of vein to adjacent lung tissue by connective tissue fibers, making the diameter of the tissue
fibers dependent upon lung volume.
C. Supplies the metabolic needs of the alveoli.
D. All of the above.
Answer: C

DISCUSSION: Pulmonary artery circulation transports oxygenated blood to the alveoli level where gas exchange occurs,
and it is here that the matching of ventilation and perfusion is so important during the postoperative period. The loss of lung
volume that generally occurs after all surgical procedures does not return to baseline for 5 to 7 days and may play an
important role in the ventilation-perfusion ratio. Improving or returning lung volume to normal is performed by
manipulating functional residual capacity (FRC) and preventing atelectasis, which in turn maintains circulation to the
alveolus and optimizes the ventilation-perfusion ratio.

4. Which of the following screening tests are important for preoperative evaluation of pulmonary function?
A. History and physical examination.
B. Room air arterial blood gases.
C. Chest film.
D. Vital capacity and forced expiratory volume in 1 second (FEV 1).
E. Cardiopulmonary exercise testing.
Answer: ABCDE

DISCUSSION: The most important clues to impairment of respiratory function are found in the history and physical
examination. A negative history and physical examination in combination with a relatively normal room air arterial blood
gas and normal chest film are sufficient to screen patients to support the clinical impression that there is minimal pulmonary
disease. Patients with symptoms, positive physical findings, and/or abnormalities in the arterial blood gases or chest film
can be screened most effectively with an additional evaluation of the vital capacity and FEV 1. More elaborate tests such as
cardiopulmonary exercise testing are reserved for patients with obvious and marked impairment of pulmonary function who
are being evaluated for the feasibility of surgical intervention.

5. Carbon monoxide diffusion capacity (DLCO) has been shown to correlate with:
A. The thickness of the alveolar lining membrane.
B. The permeability of the erythrocyte to carbon dioxide.
C. Pulmonary emboli.
D. Total alveolar-capillary capacity.
Answer: ABCD

DISCUSSION: The single-breath DLCO is a screening test that has been shown to be decreased in all of the above
examples. It is an estimate of the total capacity of the functional alveolar microarchitecture and has been demonstrated to be
an independent measure of physiologic capability apart from the FEV 1 and forced ventilatory capacity.
6. The closing volume is:
A. The volume remaining in the lung at the end of expiration below which alveolar collapse begins to occur, resulting in
physiologic shunting.
B. Higher in young persons.
C. Not changed during surgery.
D. Relative to the oxygen content of mixed venous blood.
Answer: AC

DISCUSSION: The closing volume is conceptually the remaining lung volume at the end of expiration below which
alveolar collapse begins to occur, causing intrapulmonary right-to-left shunting and thus desaturation of blood in the left
atrium. In a normal young person this closing volume is well below the functional residual capacity (FRC); thus, such
physiologic shunting does not occur until there is a decrease in the elastic properties of the lung. Although FRC gradually
increases with age, so does the effective closing volume. Eventually some alveoli are being underventilated (at end-
expiration), allowing physiologic right-to-left shunting to occur. Closing volume is unchanged, but FRC decreases during
surgery (i.e., shunting occurs). Closing volume has no direct relationship to the oxygen content of the mixed venous blood.

7. The effect of high positive end-expiratory pressures (PEEP) on cardiac output is:
A. None.
B. Increased cardiac output.
C. Decreased cardiac output because of increased afterload to the left ventricle.
D. Decreased cardiac output because of decreased effective preload to the left ventricle.
Answer: D

DISCUSSION: Higher levels of PEEP can be associated with decreases in cardiac output as a consequence of an effective
decrease in the preload to the left ventricle owing to impaired left ventricular filling.

8. Weaning patients from maximum ventilator support usually involves:


A. Weaning PEEP first, tidal volume second, and the fraction of inspired oxygen (FIO 2) third.
B. Weaning FIO 2 first, ventilator rate second, and PEEP third.
C. Weaning FIO2 first, PEEP second, and tidal volume third.
D. Weaning FIO 2 first, PEEP second, and ventilator rate third.
Answer: D

DISCUSSION: When the inspired oxygen concentration is greater than 60% for more than 24 to 28 hours, the risk of
oxygen toxicity increases. PEEP is usually weaned to physiologic levels (i.e., 5 to 7 cm. H 2O) before weaning either rate or
tidal volumes. Generally, the optimal tidal volume to achieve alveolar recruitment is selected and usually is not decreased
unless peak airway pressures increase. If decreases in ventilatory rate are not tolerated, airway pressure support can be
added.

9. Which of the following statements about bronchoscopy is false?


A. The morbidity and mortality are approximately 0.2% and 0.08%, respectively.
B. The most common complications of bronchoscopy are related to premedication of patients.
C. Adjunctive cancer therapy such as laser treatment and brachytherapy may be administered via this route.
D. A chronic cough and unilateral wheezing are accepted indications for bronchoscopy.
E. Early postoperative bronchoscopy for atelectasis is contraindicated following pulmonary resection.
Answer: E

DISCUSSION: The stated complication rates are true and reported from a comprehensive review of over 24,000 patients.
Although the most common complications are related to the premedication, significant hemorrhage, pneumothorax,
bronchospasm, and dysrhythmias have been reported. In addition to laser and brachytherapy, phototherapy and
immunotherapy have been given by bronchoscopy. Unilateral wheezing may represent a bronchial foreign body, and a
chronic cough could signify myriad pulmonary disorders. Accordingly, both are amenable to diagnostic bronchoscopy. Most
thoracic surgeons favor early bronchoscopy for lobar atelectasis following pulmonary surgery.

10. Flexible bronchoscopy is preferred over rigid bronchoscopy for all of the following except:
A. Patients with cervical spine injuries requiring intubation.
B. The evaluation of a smoke inhalation injury.
C. Transcarinal needle aspiration of an enlarged subcarinal lymph node.
D. The removal of a bronchus intermedius foreign body from an infant.
E. A cost-effective evaluation of mild hemoptysis.
Answer: D

DISCUSSION: Neither patients with significant cervical spine disease or injuries nor those with large aortic arch aneurysms
should undergo rigid bronchoscopy, given the greater risk of complications. Even if severe, smoke inhalation injury can be
assessed adequately by flexible bronchoscopy. Transbronchial needle aspiration of lesions that on computed tomography
(CT) look suspicious is safe and quite easily performed with fluoroscopic guidance. Hemoptysis as a presenting symptom
should be evaluated by flexible bronchoscopy. In contrast, for massive hemoptysis an airway should be secured for
ventilation with a rigid bronchoscope. Similarly, an airway needs to be maintained while removing endobronchial foreign
bodies from infants or children. Since adequate port sites for instrumentation are also needed, the rigid bronchoscope is
preferred in this setting.

11. Which of the following approaches is/are currently acceptable for the management of spontaneous pneumothorax?
A. Chest tube replacement alone for the patient with a first episode.
B. Operation on presentation for any patient with a first episode.
C. Video-assisted thoracic surgery (VATS) bleb excision and pleurodesis for recurrent pneumothorax on the same side.
D. Thoracotomy with bleb excision and pleurodesis for unilateral recurrent pneumothorax.
E. Operation after a first episode in an airline pilot.
Answer: ACDE

DISCUSSION: Primary spontaneous pneumothorax typically occurs in young patients with congenital blebs at the apices of
the lungs. Rupture of these blebs causes pneumothorax, which recurs in about 30% of patients. Standard care on initial
presentation is chest tube placement alone. Operation traditionally has been performed during the initial episode only if
there is prolonged air leakage (longer than 7 days); in patients with bilateral pneumothorax; those who live in an area where
prompt medical care, if needed, is not available; and those frequently exposed to extremes of pressure (e.g., airline pilots).
Both VATS and thoracotomy approaches to excision of blebs and pleurodesis have been shown to be effective. The
availability of the apparently safer VATS approach has led some to favor earlier operation.

12. For which patient(s) with a pulmonary infiltrate of uncertain cause would you favor VATS over open wedge excision?
A. An AIDS patient with a diffuse infiltrate who is ambulatory but requires supplemental oxygen. Bronchoalveolar lavage
is negative.
B. A 64-year-old previously healthy man with increasing shortness of breath, a diffuse infiltrate, and restrictive lung disease
as shown by pulmonary function studies.
C. A 74-year-old diabetic woman with a rapidly progressing process throughout the right lung who is ventilator- and
pressor-dependent.
D. A 44-year-old man with fever, left-sided infiltrate, and shortness of breath.
E. A 79-year-old man on a ventilator for right lower and middle lobe pneumonia which has been culture negative.
Answer: ABD

DISCUSSION: Lung biopsy by VATS or minithoracotomy is often indicated in the work-up of a pulmonary infiltrate that
has not been successfully diagnosed by less invasive studies. This procedure probably is not indicated for cancer patients
with acute pneumonitis, as broad-spectrum antibiotics frequently are successful treatments. For those who do require the
procedure, the choice between VATS and thoracotomy is determined by the severity of illness. In those who are critically ill
and ventilator dependent, changing the tube to an endobronchial tube for thoracoscopy may be risky, and in these cases an
anterior thoracotomy with single-lumen ventilation is indicated. These patients, who are frequently heavily sedated and are
likely to remain so for some time postoperatively, are unlikely to benefit from the greatest advantage of VATS, the reduction
of postoperative pain. It is, then, the ambulatory patient with a chronic interstitial process who benefits the most from the
VATS approach.

13. Which of the following statements about the cause and prevention of postintubation tracheal stenosis are correct?
A. Postintubation airway stenosis can largely be avoided by providing assisted ventilation via endotracheal tube rather than
tracheostomy tube.
B. Postintubation tracheal stenosis at the cuff level results, more or less equally, from low blood pressure, advanced age,
steroids, high intracuff pressure, sensitivity to tube materials, gas sterilization elution products, and systemic disease.
C. In women and smaller men large endotracheal tubes can produce lesions of the glottis and subglottis that can progress to
stenosis.
D. Stomal stenosis is due principally to cicatricial closure of large stomas resulting from removal of a disk or segment of
tracheal wall during tracheostomy.
E. A large-volume tracheostomy tube cuff such as that now used on most available tubes can become a high-pressure cuff if
filled beyond its resting maximal volume.
Answer: CE

DISCUSSION: Use of an endotracheal tube, of course, avoids a stoma and related complications. Cuff lesions, however, are
incurred from cuffs on endotracheal tubes, cricothyroidostomy tubes, and tracheostomy tubes. A cuff is the common factor.
Endotracheal tubes, on the other hand, cause erosion at the level of the glottis and subglottis, in particular. Although many
factors may play some role in the origin of airway stenosis, the principal factor is pressure necrosis of the mucosa,
submucosa, and ultimately of the cartilage, with subsequent cicatrization. Large endotracheal tubes do, indeed, cause
necrosis and airway injury at narrow areas in the upper airway, at glottic and cricoid levels. Such injuries lead to posterior
commissure stenosis, arytenoid fixation, vocal cord erosion and granulomas, anterior commissure stricture, and subglottic
circumferential stenosis. While excision of a large amount of tracheal wall can lead inevitably to healing by contraction with
narrowing of the tracheal circumference, the most usual cause of stomal stenosis is erosion of the stoma by pressure from
the tracheostomy tube. This in turn may relate to leverage by equipment. Additional factors appear to be subsidiary. A large
volume, low pressure tracheostomy tube cuff, such as those currently available, if properly constructed, will seal the trachea
before it is necessary to stretch the cuff by adding an increasing volume of air. If the cuff is stretched beyond that resting
volume, which usually occludes the normal trachea, high pressures will develop because the plastic material from which all
of these cuffs are now made is not very extensible. Therefore, the pressure-volume curve rises sharply once the limit of
unstretched volume is passed. A low-pressure cuff then becomes a high-pressure cuff.

14. Which of the following statements about the treatment of postintubation airway stenosis are correct?
A. Emergency management of airway obstruction due to stenosis at the level of a prior tracheal stoma is best accomplished
by establishing a new tracheostomy in normal tracheal tissue just below the scar of the old stoma.
B. Radial lasering and dilatation usually leads to permanent resolution of postintubation tracheal stenosis.
C. Splinting of a cervical trachea with a silicone T-tube for 6 to 8 months generally leads to permanent resolution of
stricture.
D. Postintubation tracheal stenosis that extends into the subglottic larynx is treated by resection of a cylindrical sleeve of
stenotic airway and end-to-end reconstruction.
E. Acquired tracheoesophageal fistula due to intubation injury is corrected by surgical closure of the fistula concurrent with
resection and reconstruction of the damaged trachea.
Answer: E

DISCUSSION: Emergency management of postintubation tracheal stenosis is accomplished by dilatation under general
anesthesia using rigid bronchoscopes and dilators. Tracheostomy is employed only when the patient requires a prolonged or
a permanent airway before or instead of resection and reconstruction. If a new tracheostomy is needed it is preferable to
place it through the area of existing stenosis—in this case the site of prior tracheostomy—rather than to injure normal
trachea that will be needed for resection and reconstruction. If the stenosis is below the sternal notch, a long tracheostomy
tube is inserted at the usual position (second and third rings) but extends past the now dilated stenotic lesion.
Lasering almost never results in a permanently satisfactory airway; the exception is the very limited lesion described as a
thin, weblike stenosis. Such lesions are rare. Unfortunately, laser is widely used and often compounds existing damage by
concurrent placement of a tracheostomy tube below the lesion in the normal trachea. T-tubes are very useful for temporizing
when repair is not possible or must be delayed. It almost never leads to resolution of a stricture unless it is an extremely
limited one. Soon after removal of the T-tube the stricture reasserts itself.
If the lesion involves the subglottic larynx, complex repair is required to preserve the recurrent laryngeal nerve's anatomy
and function. The posterior cricoid plate is salvaged and resurfaced with a flap of membranous tracheal wall; the anterior
subglottic larynx is reconstructed with a “prow” of distal tracheal cartilage and mucosa.
A tracheoesophageal fistula is managed (after weaning from a respirator) by layered closure of the esophagus, interposition
of a flap of well-vascularized tissue (such as a pedicled strap muscle), and resection and reconstruction of the damaged
trachea. Since the fistula results from the pressure of a cuff, often against an esophageal feeding tube, there is
circumferential damage to the trachea at the level of the fistula. Resection and reconstruction are therefore necessary, in
addition to closure of the fistula, for successful treatment of this complex lesion.
15. Which of the following statements are true?
A. Pyogenic lung abscess occurs most frequently in the lower lobe of the left lung.
B. Anaerobic bacteria are commonly present in pyogenic lung abscess.
C. Operation is usually required to eradicate a pyogenic lung abscess.
D. Penicillin is the treatment of choice for lung abscess.
Answer: BD

DISCUSSION: Pyogenic lung abscess is the result of aspiration of debris from the oropharynx. Since most patients are
unconscious and supine when this occurs, the aspirated material usually finds its way into the most dependent bronchi.
These are the superior division of the right lower lobe and the posterior segment of the right upper lobe. The organisms most
commonly responsible for pyogenic lung abscess are the same anaerobic bacteria found in the mouths of patients with poor
oral hygiene. In addition to anaerobic organisms alpha- and beta-hemolytic streptococci, staphylococci, nonhemolytic
streptococci, and Escherichia coli may be present. Gram-negative rods and staphylococci are particularly common in
hospital-acquired infections. These organisms are almost always penicillin sensitive. Surgical therapy is rarely necessary to
eradicate a pyogenic lung abscess. Penicillin, alone or in combination with metronidazole, is the drug of choice.
Metronidazole alone probably lacks sufficient activity against anaerobic and microaerophilic streptococci. Clindamycin is
also effective against most anaerobic bacteria present in pyogenic lung abscesses.

16. Amphotericin B is effective for the following lung infections:


A. Histoplasmosis.
B. North American blastomycosis.
C. Aspergillosis.
D. Mucormycosis.
E. Sporotrichosis.
Answer: ABD

DISCUSSION: Amphotericin B is effective against most fungal infections, including histoplasmosis and North American
blastomycosis. Aspergillosis is caused by the fungus Aspergillus fumigatus, an organism that is resistant to treatment with
iodides, nystatin, hydroxystilbamidine, and amphotericin B. The treatment for this fungal infection is a surgical procedure,
if the patient's condition permits. Surgical excision and amphotericin B usually are necessary to treat mucormycosis.
Itraconazole is the drug of choice for sporotrichosis. Itraconazole, ketoconazole, and fluconazole should be considered as
primary or secondary drugs when treating systemic fungal infections.

17. The following statements are true.


A. A distinguishing roentgenographic appearance of lung abscess, the air-fluid level can be seen only on roentgenograms
obtained in the upright or lateral decubitus position.
B. The fungus ball characteristic of aspergillosis can be seen roentgenographically in either the upright or recumbent
position.
C. Actinomycosis and nocardiosis are both fungal diseases of the lung that respond to treatment with the newer azole
antifungal agents.
D. The commonest fungal lung infection in the United States is due to Histoplasma capsulatum.
Answer: ABD

DISCUSSION: An air-fluid level distinguishes a lung abscess. While this can be seen easily in an upright or lateral
decubitus roentgenogram, it cannot be seen when the patient is recumbent. The fungus ball characteristic of aspergillosis is
identified by its crescent-shaped shadow on a roentgenogram. When the patient changes from an upright to recumbent
position, the fungus ball may also change position in the cavity in the lung. Both actinomycosis and nocardiosis are bacterial
infections and do not respond to antimycotic treatment. Actinomyces israelli is treated with penicillin and Nocardia
asteroides is sensitive to trimethoprim-sulfamethoxazole. The most common fungal infection in North America is
histoplasmosis. More than 30 million people have been infected, most of whom are asymptomatic.

18. Pneumocystis pneumonia is an opportunistic infection caused by Pneumocystis carinii. Which of the following
statements are true?
A. P. carinii is a fungus.
B. Pneumocystis pneumonia is the most common opportunistic infection in patients with AIDS.
C. The diagnosis of Pneumocystis pneumonia depends on the demonstration of P. carinii organisms in lung tissue.
D. There is no effective treatment for Pneumocystis pneumonia.
Answer: BC

DISCUSSION: P. carinii is a protozoan that stains with silver methenamine and resembles a fungus. It responds to
antiprotozoal drugs. Pneumocystis pneumonia occurs in 80% of AIDS patients. The diagnosis is made by demonstrating the
organisms in lung tissue by transbronchoscopic lung or brush biopsy, percutaneous needle biopsy, or open lung biopsy. Both
trimethoprin-sulfamethoxazole and pentamidine isethionate are effective against P. carinii.

19. Which of the following statements are true?


A. The pleural space does not extend into the neck.
B. Positive intrapleural pressures as high as 40 cm. H 2O and negative pressures as low as -40 cm. H 2O are possible.
C. The pleural cavities cannot absorb more than 500 ml. of fluid per day.
D. All pleural effusions are of clinical significance and should be investigated.
Answer: BD

DISCUSSION: The pleural spaces extend into the neck as well as retrosternally and into the costophrenic sinuses. This
should be kept in mind when performing procedures such as subclavian and jugular puncture, to avoid pneumothorax.
Markedly elevated intrapleural pressures are obtained with the Valsalva maneuver, and extreme negative pressures can be
produced with forced inspiratory effort against a closed glottis. Because of the many microvilli present on the mesothelial
cells that line the pleural cavity, a liter or more of fluid is easily secreted or absorbed within a 24-hour period. Most pleural
effusions are caused by infection, tumor, or congestive heart failure and should be investigated to determine the proper
course of management.

20. Which of the following statements are true?


A. Chylothorax, or chyle in the pleural cavity, usually is not a serious condition.
B. Chyle is easily identified by its milky appearance, which looks like no other kind of pleural effusion.
C. The commonest causes of chylothorax are trauma and tumor.
D. The thoracic duct can be ligated with impunity.
Answer: CD

DISCUSSION: Chylothorax is most often the result of trauma; however, spontaneous chylothorax is also a manifestation of
tumor and should be investigated to identify occult malignancies. Chyle in the thorax is characteristically milky white but
can be mistaken for the pseudochylothorax of rheumatoid disease or tuberculosis. If necessary, a diagnosis can be confirmed
by lymphangiography. This also facilitates ligation of the thoracic duct, should this become necessary to control the loss of
chyle.

21. Which of these statements about pleural tumors is/are true?


A. The commonest type of pleural tumor is primary pleural mesothelioma.
B. Exposure to asbestos dust is causally related to the development of malignant mesothelioma.
C. Localized benign mesotheliomas are asymptomatic.
D. Complete pleurectomy for malignant mesothelioma usually results in cure.
Answer: B

DISCUSSION: Pleural involvement by metastatic disease is much more common than primary pleural tumors. Patients with
localized benign pleural mesotheliomas may have symptoms of arthralgia, clubbing of the fingers, or fever, which usually
disappear after excisional surgery. The evidence relating industrial exposure to asbestosis and malignant pleural
mesothelioma is quite strong. Excisional surgery for malignant mesothelioma is usually only palliative. Most patients
succumb within 1 to 2 years of the diagnosis, regardless of the kind of treatment they receive.

22. Which of the following correctly describe a patient with spontaneous pneumothorax?
A. The patient is almost always elderly and debilitated.
B. An unsuspected primary or metastatic lung tumor may be present.
C. The administration of supplemental oxygen is of little benefit to the patient.
D. The patient should always be treated with an intercostal tube and closed pleural drainage.
E. Video-assisted thoracic surgery (VATS) should be considered for persistent air leak in patients with secondary
spontaneous pneumothorax.
Answer: BE

DISCUSSION: A patient with spontaneous pneumothorax may be old and debilitated, but the typical patient is an otherwise
healthy young adult, usually one who smokes. An incidental, unsuspected lung cancer is discovered on rare occasions when
operation is performed to control a persistent air leak. Perhaps smoking is a common factor. Absorption of air from the
pleural space can be facilitated by the administration of supplemental oxygen. Increasing the oxygen tension lowers the
partial pressure of nitrogen (P N2) of the capillary blood and increases the partial pressure difference between the pleural
space and the pulmonary capillary. If the pneumothorax results in less than 20% collapse of the lung an asymptomatic
patient can be safely observed; however, a larger or persistent pneumothorax is best treated with an intercostal tube
thoracostomy. Patients with bullous emphysema may require stapling of bullae and pleurectomy, which can be done by open
thoracotomy or thoracoscopically (VATS).

23. Which of the following statements about spontaneous pneumothorax (PSP) is/are correct?
A. The risk of recurrence after resolution of the first episode of PSP or secondary spontaneous pneumothorax (SSP) is 35%
to 45%.
B. Patients with PSP are typically tall, thin, young adult males with a history of smoking.
C. Secondary spontaneous pneumothorax is associated with family history in 10% of cases.
D. For bleb resection and pleurodesis thoracoscopic thoracotomy and open thoracotomy provide similar cure rates for
patients with primary spontaneous pneumothorax.
E. Causes of secondary pneumothorax include trauma and iatrogenic needle puncture.
Answer: ABD

DISCUSSION: Patients with PSP are usually 20- to 40-year-old males with a common long-chested body habitus. The
majority of PSP patients have a history of tobacco use and 10% have a family history of PSP. The majority of cases of SSP
are due to advanced emphysema in a population of patients aged 50 to 70 years. Additional causes of SSP include
tuberculosis, cystic fibrosis, P. carinii infection, lung cancer, and lung abscess. For patients with PSP bleb resection and
pleurodesis performed thoracoscopically provides cure rates similar to those of open thoracotomy. Because of the nature of
underlying pulmonary diseases, open thoracotomy appears to provide better results for patients with SSP.

24. Which of the following are relative contraindications for surgical management of emphysema?
A. Rapidly progressive dyspnea.
B. Bullae occupying less than one third of a hemithorax on plain chest radiography.
C. Elevated room air PCO 2.
D. “Pink puffer” patients.
E. FEV 1 less than 35% of predicted value.
Answer: BCE

DISCUSSION: Relative contraindications to operation for bullous emphysema include patients with carbon dioxide
retention, FEV 1 less than 35% of predicted value, small bullae that occupy less than one third of a hemithorax, and “blue
bloaters,” who are prone to the sequelae of chronic bronchitis. Patients who have primarily emphysema (“pink puffers”) and
rapidly progressive dyspnea are usually good candidates for operation.

25. Which of the following treatments would be appropriate therapy for symptoms that persist on medical therapy and
bronchiectasis involving, in order of decreasing severity, the left lower lobe, the right middle lobe, and the left upper lobe?
A. Left pneumonectomy.
B. Wedge resection of the left lower lobe.
C. Left lower lobectomy.
D. Simultaneous left lower lobectomy and right middle lobectomy.
Answer: C

DISCUSSION: Pneumonectomy is seldom indicated today for bronchiectasis. Anatomic resection of involved segments
with either segmentectomy or lobectomy is preferred to nonanatomic wedge resection. Bilateral pulmonary resections
should generally be done as staged procedures, the most symptomatic side being resected first. Then, the contralateral side
is resected only if symptoms persist during a prolonged course of medical therapy.
26. Which of the following would not be acceptable sequences of preoperative studies in a patient being prepared for
lingulectomy for bronchiectasis?
A. CT alone.
B. CT, bronchoscopy, bronchography.
C. Bronchoscopy alone.
D. Bronchoscopy, bronchography.
Answer: C

DISCUSSION: Bronchoscopy alone is generally not diagnostic for bronchiectasis. Thin-section, high-resolution CT can
diagnose bronchiectasis and define the airway anatomy sufficiently for resection. Bronchography is performed less
frequently today but can be very useful in diagnosing bronchiectasis and defining airway anatomy for pulmonary resection.

27. Which of the following statements about pulmonary mycobacterial infection is/are correct?
A. Worldwide, tuberculosis no longer represents a significant public health problem.
B. Mycobacterium tuberculosis is responsible for the majority of cases of pulmonary mycobacterial disease.
C. Mycobacterium kansasii pulmonary infection almost always requires surgical treatment.
D. Atypical mycobacteria are never primary pulmonary pathogens in humans.
E. Mycobacterium avium-intracellulare is generally resistant to most antimycobacterial drugs in vitro.
Answer: BE

DISCUSSION: Tuberculosis remains the leading infectious killer in the world today. M. tuberculosis is responsible for the
vast majority of pulmonary mycobacterial disease. M. kansasii infection responds to multiple drug chemotherapy and
relatively infrequently requires surgical treatment. Atypical mycobacteria can be primary pulmonary pathogens in humans.
M. avium-intracellulare is usually resistant in vitro to most antituberculosis drugs.

28. Which of the following chemotherapeutic regimens are currently recommended for the treatment of pulmonary
infection caused by M. tuberculosis?
A. Isoniazid, rifampin, pyrazinamide, and streptomycin for 24 months.
B. Isoniazid for 9 months with ethambutol for the first 3 months.
C. Isoniazid and rifampin for 6 months with pyrazinamide added for the first two months.
D. Isoniazid alternating with rifampin at 3-month intervals for 12 months.
E. Isoniazid and rifampin for 9 months.
Answer: CE

DISCUSSION: Treatment of tuberculosis with a single drug leads to rapid emergence of drug-resistant organisms. Any
treatment regimen that employs only one drug for a period of time encourages the development of drug-resistant
tuberculosis. Currently, the American Thoracic Society recommends either (1) a 6-month regimen consisting of isoniazid,
rifampin, and pyrazinamide for 2 months followed by isoniazid and rifampin for 4 months or, alternatively, (2) a 9-month
course of isoniazid and rifampin. Prolonged courses of treatment beyond 9 to 12 months no longer are considered necessary.

29. Which of the following are appropriate indications for pulmonary resection for mycobacterial disease?
A. Localized pulmonary disease caused by M. avium-intracellulare.
B. Advanced lobar tuberculous pneumonia with massive hilar lymphadenopathy and bronchial obstruction in a young child.
C. Localized pulmonary disease due to multiple drug–resistant M. tuberculosis.
D. An asymptomatic tuberculous cavity greater than 12 cm. in diameter.
E. Massive hemoptysis from a right upper lobe cavity occurring during an appropriate course of chemotherapy for
pulmonary tuberculosis in a sputum-negative patient.
Answer: ACE

DISCUSSION: Modern antimycobacterial chemotherapy is very effective. Surgical treatment of pulmonary mycobacterial
disease is rarely necessary; however, pulmonary disease caused by M. avium-intracellulare or multiple drug–resistant M.
tuberculosis is not likely to respond to chemotherapy and should be resected if the disease is localized. Chemotherapy for
tuberculosis is almost invariably curative in children, regardless of the extent of disease. The size of a tuberculous cavity is
not an indication for resection. Massive hemoptysis from a cavitary lesion is life threatening and is an indication for
pulmonary resection.

30. Which statements about squamous papillomatosis of the trachea is/are correct?
A. It is the most common type of benign tracheal tumor in adults.
B. It is the most common type of benign tracheal tumor in children.
C. Most are treated with segmental tracheal resection.
D. There is no risk of malignant degeneration.
E. It is associated with a herpesvirus.
Answer: A

DISCUSSION: Squamous papillomatosis is the most common benign tracheal and bronchial tumor in adults. Up to 50% of
untreated lesions may degenerate into squamous cell carcinoma. The lesion is associated with human papillomavirus types 6
and 11, and therefore, interferon therapy is under investigation. Most patients can be treated successfully by repeated
bronchoscopic fulguration, laser ablation, or cryotherapy.

31. Which of the following statements about pulmonary hamartomas is/are true?
A. Hamartomas are benign chondromas.
B. Most are located in the conducting airways.
C. Wedge resection is curative.
D. A lobectomy is necessary to obtain draining hilar lymph nodes.
E. Hemoptysis is common.
Answer: C

DISCUSSION: Pulmonary hamartomas are benign masses consisting of cartilage, lymph tissue, fat, and epithelial elements.
Eighty per cent are located in the lung periphery and are treated by a small wedge resection, usually with a thoracoscope.
Most are asymptomatic, and there is no risk of malignant degeneration.

32. Which of the following statements about typical carcinoid tumors are true?
A. They make up the majority of bronchial adenomas.
B. They frequently have lymph node metastases.
C. The carcinoid syndrome is observed in 33%.
D. Overall survival at 5 years is 90%.
E. Overall survival at 5 years is 50%.
Answer: AD

DISCUSSION: Eighty-five per cent of bronchial adenomas are carcinoid tumors. Typical carcinoid tumors have few mitotic
figures and infrequent lymph node metastases (fewer than 10%). Only 10% to 15% of patients present with the carcinoid
syndrome (flushing, wheezing, diarrhea). Survival after resection is more than 90% at 5 years but decreases to
approximately 50% for atypical histology.

33. Which is/are true of adenoid cystic carcinoma?


A. It is a common type of salivary gland tumor.
B. Another name is cylindroma.
C. Most patients are completely resected for cure.
D. Different histological types have different prognoses.
E. Tissue invasion is rare.
Answer: ABCD

DISCUSSION: Adenoid cystic carcinomas (cylindromas) are commonly observed salivary gland tumors that can occur in
the conducting airways. The undifferentiated solid type is associated with distant metastases, of which the cribriform and
tubular types are associated with perineural and submucosal invasion. Most patients (60%) can be resected for cure.
34. A solitary pulmonary nodule is discovered in an asymptomatic 55-year-old smoker with no evidence of extrathoracic
dissemination. The most appropriate management would be to:
A. Obtain serial chest films every 3 months to determine the growth potential of the nodule.
B. Perform transthoracic needle aspiration (TTNA) before considering pulmonary resection to confirm malignancy.
C. Conduct an extensive systematic evaluation to exclude the possibility that the nodule represents a metastatic lesion.
D. Proceed with pulmonary resection after ascertaining that the patient would tolerate removal of the requisite amount of
lung.
E. Obtain baseline serum levels of carcinoembryonic antigen and p53.
Answer: D

DISCUSSION: A patient with a solitary pulmonary nodule—a single spherical lesion within the lung— represents an
important and challenging diagnostic problem in thoracic oncology. A solitary pulmonary nodule is assumed to be primary
lung cancer until proved otherwise; the differential diagnosis includes metastatic carcinoma, granuloma, and benign
pulmonary tumors. In most cases, solitary pulmonary nodules should be resected after thorough investigation to establish
that systemic dissemination has not already occurred. CT of the chest, liver, and adrenals is performed to confirm the
location of the tumor, to evaluate the mediastinum, and to assess the abdomen for systemic disease. If there is no evidence
of metastases on CT, the patient should undergo bronchoscopy, which may establish the histologic diagnosis and determine
resectability if an endobronchial lesion exists.
Pulmonary function studies are obtained preoperatively to assess the potential for pulmonary resection. A thorough review
of systems is undertaken to rule out medical contraindications to thoracotomy. TTNA is not performed routinely and should
be reserved for patients with marginal pulmonary function, for whom thoracotomy would be performed only after
verification of a malignant histologic diagnosis.

35. After thoracotomy, pulmonary resection, and mediastinal lymph node dissection, a patient is determined to have a
squamous cell carcinoma 2 cm. in diameter, located 1 cm. from the carina along the right mainstem bronchus. Three
peribronchial lymph nodes are positive for cancer, and all other lymph node stations are negative. The correct stage,
according to the TNM system, is:
A. T1N0M0 Stage I.
B. T1N1M0 Stage II.
C. T2N1M0 Stage II.
D. T3N1M0 Stage IIIa.
E. T2N3M0 Stage IIIb.
Answer: C

DISCUSSION: The TNM staging system for carcinoma of the lung provides a consistent, reproducible description of the
anatomic extent of disease at the time of diagnosis. In the TNM system, T represents the primary tumor and numerical
suffixes describe increasing size or involvement; N represents regional lymph nodes with suffixes to describe levels of
involvement; and M designates the presence or absence of distant metastases.
TUMOR (T)
TX Occult carcinoma (malignant cells in sputum or bronchial washings but tumor not visualized by imaging studies or
bronchoscopy)
T1 Tumor 3 cm. or less in greatest diameter, surrounded by lung or visceral pleura, but not proximal to a lobar
bronchus
T2 Tumor larger than 3 cm. in diameter, or with involvement of main bronchus at least 2 cm. distal to carina, or with
visceral pleural invasion, or with associated atelectasis or obstructive pneumonitis extending to the hilar region but not
involving the entire lung
T3 Tumor invading chest wall, diaphragm, mediastinal pleura, or parietal pericardium; or tumor in main bronchus
within 2 cm. of, but not invading, carina; or atelectasis of obstructive pneumonitis of the entire lung
T4 Tumor invading mediastinum, heart, great vessels, trachea, esophagus, vertebral body, or carina; or ipsilateral
malignant pleural effusion
NODES (N)
N0 No regional lymph node metastases
N1 Metastases to ipsilateral peribronchial or hilar nodes
N2 Metastases to ipsilateral mediastinal or subcarinal nodes
N3 Metastases to contralateral mediastinal or hilar, or to any scalene or supraclavicular nodes
DISTANT METASTASES (M)
M0 No distant metastases
M1 Distant metastases
The TNM subsets are subsequently grouped in a series of stages of disease to identify groups of patients with similar
prognosis and therapy.

STAGE T N M
Occult TX N0 M0
Stage I T1-2 N0 M0
Stage II T1-2 N1 M0
Stage IIIa T3 N0-1 M0
T1-3 N2 M0
Stage IIIb T4 N0-2 M0
T1-4 N3 M0
Stage IVAny T Any N M1

36. After complete resection of Stage I non-small cell lung cancer (NSCLC), the role of adjuvant therapy is best
summarized thus as:
A. Postoperative radiation therapy improves disease-free survival.
B. Postoperative radiation therapy improves overall survival.
C. Postoperative chemotherapy improves disease-free survival.
D. Postoperative chemotherapy improves overall survival.
E. Adjuvant therapy is not indicated after complete resection of Stage I NSCLC.
Answer: E

DISCUSSION: Prospective randomized trials conducted by the Lung Cancer Study Group demonstrate that postoperative
chemotherapy may be responsible for significantly longer disease-free survival in patients with Stage III (and perhaps Stage
II) NSCLC. The efficacy of postoperative chemotherapy and radiotherapy in patients with extensive lymph node
involvement or positive surgical margins in reducing systemic recurrences and prolonging disease-free survival has also
been demonstrated. Adjuvant therapy is not associated with improved overall survival and has not been shown to be
beneficial in patients with Stage I NSCLC.
Radiation therapy is an effective adjuvant treatment in many patients with carcinoma of the lung. Adjuvant radiotherapy,
applied to patients with completely resected Stage II or Stage III (but not Stage I) NSCLC, has been shown to decrease local
recurrence but has no significant effect on survival. However, postoperative irradiation may provide a survival advantage in
patients who have resection and are found to have metastases to hilar or mediastinal lymph nodes. Thus, the purpose of
adjuvant radiotherapy is prevention of local tumor recurrence, especially when lymph node sampling of the mediastinum at
thoracotomy is incomplete.

37. Compared to segmentectomy or wedge resection, lobectomy for NSCLC is associated with:
A. Similar operative morbidity but higher operative mortality.
B. Similar operative mortality but higher operative morbidity.
C. More severe postoperative pulmonary dysfunction.
D. Lower incidence of locoregional recurrence.
E. Equivalent locoregional recurrence.
Answer: D

DISCUSSION: The risk of recurrence after surgical resection according to the magnitude of the resection has been analyzed
by the Lung Cancer Study Group. In a prospective, randomized trial involving more than 400 patients with T1N0 lung
cancer, lobectomy was compared to segmentectomy and wedge resection. There was no significant difference in morbidity
and mortality among the procedures. Furthermore, no difference was observed in postoperative pulmonary function between
patients who underwent lobectomy and those who underwent lesser procedures. The rate of locoregional recurrence was
significantly lower in patients who underwent lobectomy (5%) as compared with those who underwent either
segmentectomy or wedge resection (15%). In another study, segmentectomy was compared to lobectomy in patients with
Stage I lung cancer. In this study, the rate of locoregional recurrence was lower in patients who underwent lobectomy (5%),
as compared with those who underwent segmentectomy (23%). Furthermore, there was a survival advantage in the patients
undergoing lobectomy for T2 disease.

38. In contrast to NSCLC, small cell lung cancer (SCLC) is characterized by:
A. Greater response rate to chemotherapy.
B. Inability to achieve surgical cure.
C. Less frequent association with paraneoplastic syndromes at the time of diagnosis.
D. Lower likelihood of metastases present at the time of diagnosis.
E. Slower growth.
Answer: A

DISCUSSION: For the purposes of staging, estimating prognosis, and selecting therapy, lung cancer is divided into two
categories: NSCLC and SCLC. SCLC is characterized by more rapid growth, higher prevalence of metastases at the time of
diagnosis, and greater responsiveness to chemotherapy and radiation therapy. After ascertaining the histological diagnosis of
SCLC, staging is performed, including thorough neurological examination and CT evaluation of the chest, abdomen, and
brain. For most patients with limited-stage disease, treatment is initiated with six cycles of combination chemotherapy.
Radiotherapy to the chest is usually employed after three initial cycles of chemotherapy and is continued for 4 weeks.
Among patients with limited-stage disease, thoracotomy for pulmonary resection is recommended for the subset of patients
with stage I SCLC.

39. Which of the following statements about the diagnosis and staging of mesothelioma is/are correct?
A. Fluid obtained by thoracentesis is usually adequate for accurate diagnosis.
B. Open biopsy or thoracoscopy should be performed to obtain tissue for diagnosis.
C. Immunohistochemistry should be performed in all cases of suspected mesothelioma.
D. Chest CT and/or magnetic resonance imaging (MRI) are useful in the staging of mesothelioma.
E. Head CT and bone scans are useful in the staging of mesothelioma.
Answer: BCD

DISCUSSION: Approximately 90% of patients with mesothelioma develop pleural effusion, but cytologic specimens from
pleural fluid are inaccurate for the diagnosis of mesothelioma, and open or thoracoscopic biopsy is required. Accurate
diagnosis of mesothelioma is difficult: the epithelial variant must be differentiated from adenocarcinoma, whereas the
sarcomatous form often resembles benign sarcomas. Immunohistochemistry using a panel of antibodies, and sometimes
electron microscopy, is required for all cases. Relentless local spread is typical, and chest CT or MRI is essential to evaluate
potential local extension into the chest wall, pericardium, mediastinum, or diaphragm. Metastatic disease is less common
and occurs late (if at all) in the disease course, so head CT and bone scans are indicated only if clinical findings are
suspicious for metastasis.

40. Which of the following statements about therapy for malignant pleural mesothelioma is/are correct?
A. The role of surgery is confined to biopsy for diagnosis and pleurodesis for palliation of effusion.
B. Extrapleural pneumonectomy involves resection en bloc of the lung, visceral and parietal pleura, pericardium, and
diaphragm.
C. If a lesion is unresectable by extrapleural pneumonectomy, pleurectomy/decortication is contraindicated.
D. Neither surgery, chemotherapy, nor radiation therapy as a single therapy improves survival.
E. Multimodality therapy, combining surgery, chemotherapy, and radiation therapy may improve survival in select patients.
Answer: BDE

DISCUSSION: In debilitated patients, palliation by pleurodesis is indicated; however, cytoreductive techniques, including
pleurectomy/decortication, and extrapleural pneumonectomy, are indicated for patients who can tolerate surgery. For Stage I
disease, extrapleural pneumonectomy is offered. If the patient cannot tolerate pulmonary resection or if the lesion is
unresectable by extrapleural pneumonectomy, pleurectomy/decortication is appropriate. Both cytoreductive procedures,
when used in a multimodality setting, may improve survival in selected patients. They also improve quality of life by
relieving or delaying two severe symptoms of mesothelioma: dyspnea secondary to lung restriction by the tumor and pain
from tumor invasion. No single modality (surgery, chemotherapy, or radiation therapy) improves survival.

41. All of the following may be acceptable operative approaches to management of the thoracic outlet syndrome except:
A. Scalenectomy.
B. Excision of a cervical rib.
C. Thoracoplasty.
D. First rib resection.
E. Division of anomalous fibromuscular bands.
Answer: C
DISCUSSION: Supraclavicular decompression of the thoracic outlet is the preferred operative approach for the thoracic
outlet syndrome. This procedure consists of extensive anterior scalenectomy, middle scalenectomy, removal of a cervical rib
(if present), and, on occasion, first rib resection. Transaxillary first rib resection has been widely used as well but is
associated with a greater risk for complications. Numerous fibromuscular anomalies have been described in association with
the thoracic outlet syndrome. Thoracoplasty has no role in the management of this disorder.

42. Initial conservative (nonsurgical) management of the thoracic outlet syndrome may include all of the following except:
A. Weight reduction.
B. Improvement of posture.
C. Exercises to strengthen the muscles of the shoulder girdle.
D. Pentoxifylline.
E. Avoiding hyperabduction.
Answer: D

DISCUSSION: The initial management of the thoracic outlet syndrome is nonoperative. A trial of weight reduction,
shoulder girdle strengthening exercises, improvement of posture, and avoidance of hyperabduction should be recommended
for 4 months or longer. These measures are successful in 50% to 70% of patients, particularly in young to middle-aged
females with poor posture. Pentoxifylline is a hemorrheologic agent used in selected patients with peripheral arterial
insufficiency and has no known benefit in the thoracic outlet syndrome.

43. Which of the following statements about pectus excavatum are correct?
A. It is the most common congenital malformation of the chest wall.
B. The most frequent presenting complaint is the cosmetic deformity.
C. The manubrium and first and second costal cartilages typically are involved in the deformity.
D. It may be associated with cardiac defects and other skeletal defects such as scoliosis.
E. Restrictive alterations in chest wall mechanics and abnormalities in pulmonary function tests have been documented.
Answer: ABDE

DISCUSSION: Congenital deformities of the chest wall represent a spectrum of deformities ranging from minor cosmetic
defects to gross deformities incompatible with life. Pectus excavatum, or funnel chest, is the most common of the congenital
deformities of the chest wall, accounting for 90% of such defects. It is characterized by a concave, posteriorly displaced
sternum due to overgrowth of the costal cartilages. Most commonly the defect begins at the junction of the manubrium and
the body of the sternum and becomes progressively deeper toward the xiphoid. The manubrium and the first and second
costal cartilages typically are normal. The defects have both physiologic and psychologic consequences and are often
associated with other abnormalities, including congenital heart disease, Marfan's syndrome, and other skeletal defects,
including scoliosis. Patients most often present because of the cosmetic defect but frequently are found to have other
symptoms, including impaired cardiopulmonary function and scoliosis. Pulmonary complaints include dyspnea and
respiratory tract infections. Restrictive alterations in chest wall mechanics and abnormalities in pulmonary function tests,
including decreased vital capacity, decreased total lung capacity, decreased maximal ventilatory volume, and decreased
maximal breathing capacity, have been documented.

44. Surgical correction of pectus excavatum is characterized by which of the following?


A. Significant cosmetic improvement initially but a high incidence of recurrence of the defect on late follow-up.
B. An increase in exercise tolerance and respiratory reserve postoperatively.
C. Improvement in FEV 1, vital capacity, and total lung capacity.
D. Improvement in maximal ventilatory volume, total progressive exercise time, and maximal exercise capacity.
E. Prevention of the development of “thoracogenic scoliosis.”
Answer: BDE

DISCUSSION: Because of the significant cosmetic and psychological improvement, subjective increase in exercise
tolerance, documented improvement in cardiac and respiratory status, and prevention of the development of scoliosis
following surgical intervention in these patients, surgical correction should be considered for all patients with moderate to
severe deformity. Cosmetic results of surgical correction are excellent, and recurrence is uncommon. Objective
improvement in cardiac function has been documented postoperatively, owing to relief of the sternal compression.
Postoperatively, worsening of the FEV 1, vital capacity, and total lung capacity have been noted, whereas a significant
improvement in maximal ventilatory volume, total progressive exercise time, and maximal oxygen consumption has also
been documented. Following surgical correction there is a consistent increase in maximal exercise capacity at every level of
workload, a lower heart rate at every workload, and an increase in exercise duration.

45. Which of the following statements about the diagnosis of chest wall tumors is/are correct?
A. Pain is a common presenting symptom.
B. Firmness and fixation to underlying bone and muscle are important to note in the physical examination as aids to
diagnosis.
C. In general, chest wall tumors are slow growing and produce symptoms late in their course.
D. CT is the most useful imaging study for making the diagnosis and for planning surgical resection of chest wall tumors.
E. Angiography should be performed routinely.
Answer: BCD

DISCUSSION: Seventy-five per cent of patients present with a slow-growing, painless chest wall mass. A firm mass that is
fixed to an underlying rib is more likely to be of bony or cartilaginous origin. Conversely, soft, mobile tumors are more
likely to be of soft tissue origin. CT defines depth of invasion and extent of tumor and is the most useful imaging modality.
Angiography should be employed selectively, primarily for very large and vascular tumors.

46. Which of the following statements about chest wall resection and reconstruction is/are correct?
A. Most tumors of soft tissue and bone require 4-cm. margins to be adequately resected.
B. At least one normal rib above and below the primary tumor should be included in the resection.
C. Techniques of chest wall reconstruction are directed at the prevention of paradoxical chest wall movement with
respiration.
D. Soft tissue defects are most conveniently addressed by stretching the existing skin over the defect under tension.
E. Chest wall defects that are covered by the scapula require no special reconstructive procedures, even if the defects are
quite large.
Answer: ABCE

DISCUSSION: Margins of resection of chest wall tumors should be at least 3 cm. of skin, 4 cm. of muscle, and 6 cm. of
bone. Old biopsy sites should be included in the specimen. A normal rib above and below the specimen should also be
included. Prevention of paradoxical chest wall movement is the primary goal of chest wall reconstruction. Large soft tissue
defects are best managed by myocutaneous pedicle flaps. In general, defects larger than 5 cm. require reconstruction.
Defects covered by the scapula require no reconstruction.

47. Prolonged extracorporeal membrane oxygenation (ECMO):


A. Is highly successful in the treatment of severe respiratory failure in newborn infants.
B. Is contraindicated in adult respiratory distress syndrome (ARDS).
C. Causes hemolysis and renal failure.
D. Requires total systemic heparinization (activated clotting time longer than 500 seconds).
E. Is identical to heart/lung bypass for cardiac surgery.
Answer: A

DISCUSSION: The survival rate of newborn infants who are moribund from respiratory failure with ECMO is 80% to 90%.
ECMO is also indicated in ARDS with a survival rate from 40% to 50%. Hemolysis and renal failure are rare complications.
ECMO requires low-dose partial heparinization, with clotting times in the range of 200 seconds. Several modifications in
the conventional heart/lung machine permit the extension of ECMO from hours to days.

48. Indications for ECMO include:


A. Newborn infants with pulmonary hypoplasia secondary to congenital diaphragmatic hernia.
B. Meconium aspiration syndrome in full-term babies (at least 35 weeks).
C. Children with pulmonary infection after bone marrow transplantation.
D. Adults with acute viral pneumonia.
E. Adults requiring mechanical ventilation and 100% oxygen for 2 weeks or longer.
Answer: BD
DISCUSSION: At present ECMO is not used for infants smaller than 1500 gm. because of a high risk of intracranial
bleeding. ECMO is very successful in the treatment of respiratory failure in full-term newborn infants. Immunosuppression
is a relative contraindication to ECMO. ECMO is indicated in adults with acute, potentially reversible respiratory failure,
but mechanical ventilation and high oxygen concentration for more than 10 days are contraindications.

49. Venovenous ECMO:


A. Avoids major arterial access.
B. Provides cardiac and pulmonary support.
C. Can be accomplished via cannulation at separate venous sites or at a single venous site using a double-lumen catheter.
D. Provides greater venous drainage than venoarterial ECMO.
E. Maintains the normal pulsatile blood flow to the systemic circulation.
Answer: ACE

DISCUSSION: Venovenous ECMO has become the access technique of choice for patients with respiratory failure without
significant requirement for cardiac (hemodynamic) support. In neonates, a double-lumen cannula allows ECMO to be
performed through a single incision over the right internal jugular vein. It can also be performed by separate cannulation of
the femoral and jugular veins. In either configuration, venovenous ECMO avoids cannulation of any major arteries and
maintains the normal pulsatile circulation through the heart and lungs. Venous drainage is no different with venovenous
ECMO.

50. As compared with venovenous ECMO, venoarterial ECMO:


A. Requires cannulation of a major artery and vein.
B. Provides both cardiac and respiratory support.
C. Can be performed with less anticoagulation.
D. Usually maintains a normal pulse pressure.
Answer: AB

DISCUSSION: Venoarterial ECMO can provide total cardiorespiratory support via cannulation of a major vein and artery
(usually the right internal jugular vein and common carotid artery in neonates). With most roller and vortex pumps, the
arterial inflow from the ECMO circuit is nonpulsatile, and therefore pulse pressure is often reduced or absent. Venoarterial
ECMO requires the same degree of anticoagulation as venovenous techniques.

51. A 24-year-old male has new onset of chest pain. Chest films demonstrate a large anterosuperior mass. Appropriate
evaluation should include:
A. CT of the chest.
B. Measurement of serum alpha-fetoprotein and beta–human chorionic gonadotropin.
C. A barium swallow.
D. A myelogram.
Answer: AB

DISCUSSION: Elevated levels of serum alpha-fetoprotein and beta–human chorionic gonadotropin are indicative of a
malignant nonseminomatous germ cell tumor. Optimal therapy for such a tumor is based on a cis-platinum-containing
chemotherapeutic regimen. After normalization of serum markers, resection of residual disease is performed. Extensive
surgical procedures prior to chemotherapy are not warranted. Confirmation of the diagnosis can usually be obtained using
needle biopsy techniques. In some institutions patients are treated based on elevated serum markers alone. CT imaging is
useful to evaluate tumor invasiveness, airway compression, vascular involvement, and the likelihood of resectability.
Barium swallow may be helpful in the evaluation of enteric cysts. Myelography may be useful in patients with posterior
mediastinal masses to evaluate for spinal column involvement.

52. Systemic syndromes frequently associated with mediastinal tumors include:


A. Myasthenia gravis.
B. Hypercalcemia.
C. Malignant hypertension.
D. Carcinoid syndrome.
Answer: ABC

DISCUSSION: Myasthenia gravis occurs in 10% to 50% of patients with thymoma. The incidence with which myasthenia
gravis occurs in patients with a thymoma increases with the age of the patient. In males over 50 and females over 60 years
of age, the incidence appears to be greater than 80%. Hyperparathyroidism due to a mediastinal parathyroid adenoma is a
cause of hypercalcemia. Although parathyroid glands may occur in the mediastinum in 10% of the patients, they are usually
accessible through a cervical incision. A sternotomy is required infrequently, even in those patients with a mediastinal
parathyroid gland. Most often the adenomas are found embedded in or near the superior pole of the thymus. Mediastinal
paraganglioma may produce significant catecholamines, predominantly norepinephrine. Catecholamine production causes a
classic group of symptoms associated with pheochromocytomas, including periodic sustained hypertension often
accompanied by orthostatic hypotension, and hypermetabolism manifested by weight loss, hyperhydrosis, palpitation, and
headaches. Mediastinal carcinoid tumors have been more frequently associated with Cushing's syndrome because of the
production of adrenocorticotrophic hormones. These tumors uncommonly cause the carcinoid syndrome.

53. A 36-year-old female developed dyspnea on exertion that has progressed over 3 months. Chest film reveals a left
anterior mediastinal mass with evidence of elevated left hemidiaphragm. CT indicates probable invasion of the pericardium.
Paratracheal or subcarinal adenopathy is not identified. Appropriate intervention in this patient would include:
A. A median sternotomy with radical resection of the tumor, sacrificing the left phrenic nerve and excising the involved
pericardium.
B. A mediastinoscopy with biopsy.
C. A left anterolateral thoracotomy or median sternotomy with generous biopsy of the tumor.
D. Observation with repeat chest radiography in 3 months.
Answer: C

DISCUSSION: The differential diagnosis of an invasive anterosuperior mediastinal mass includes thymoma, lymphoma,
germ cell tumor, undifferentiated carcinoma, and carcinoid tumors. These tumors often have a very similar histologic
appearance, which may cause an inaccurate diagnosis based on light microscopy alone. Use of electron microscopy and
immunohistochemistry may be necessary to correctly determine the specific histologic diagnosis. Frozen section should be
used to determine adequacy of tissue biopsy. Histologic diagnosis based on frozen section examination in many of these
tumors may be erroneous. Although radical resection of tumor is indicated for thymoma, chemotherapy and radiotherapy are
the modalities used for the treatment of patients with lymphomas and germ cell tumors. Exact determination of tumor
histology by permanent section should precede radical resectional therapy. Generous tissue biopsy is necessary for the
precise subtyping of lymphomas. Mediastinoscopy is useful in patients with paratracheal and pericarinal masses or
adenopathy, particularly when right-sided. Observation of a patient with invasive mediastinal mass is not warranted.

54. An 18-year-old male presents with a history of increasing shortness of breath that worsens in the recumbent position.
On physical examination, the neck veins are noted to be distended, with facial plethora that is accentuated by lying the
patient down. A 2.5-cm. left supraclavicular lymph node is palpable. Chest film reveals an extensive right anterosuperior
mediastinal mass. Appropriate intervention may include:
A. An urgent biopsy of the mediastinal mass under general anesthesia with subsequent initiation of therapy.
B. CT.
C. Pulmonary function testing in the sitting and supine positions.
D. A biopsy of the right supraclavicular lymph node under general anesthesia.
E. A biopsy of the supraclavicular lymph node under local anesthesia.
Answer: BCE

DISCUSSION: Although most patients with a mediastinal mass may undergo surgical procedures under general anesthesia
with a minimal risk, patients with a large anterior, superior, or middle mediastinal mass, particularly those with posture-
related dyspnea and superior vena caval syndrome, have an increased risk of developing severe respiratory complications
during general anesthesia. Useful techniques for identifying less symptomatic patients who have significant airway
compression include CT imaging and pulmonary function tests. A reduction of the tracheal diameter by more than 35% on a
CT scan and reduction of peak expiratory flow during pulmonary function testing are sensitive indicators of functional
airway compression. In patients with airway compression and superior vena caval obstruction, the risk of general anesthesia
is significant. Attempts to obtain a histologic diagnosis should be limited to needle biopsies or open procedures performed
under local anesthesia. In situations in which histologic diagnosis cannot be obtained using these methods, therapy may be
initiated with radiation, corticosteroids, and chemotherapy. However, a histologic diagnosis may not be obtainable in as
many as 40% of these patients after initiation of treatment. Some proceed with biopsy of the mediastinal mass under general
anesthesia. However, alterations in anesthetic management include: (1) induction of anesthesia in a semi-Fowler's or upright
position, (2) availability of rigid bronchoscopy to allow reestablishment of an adequate airway, (3) use of a long
endotracheal tube to allow advancement of the tube beyond the site of obstruction, (4) avoidance of muscle relaxants and
the use of spontaneous ventilation when possible, (5) lower extremity intravenous cannulation, and (6) standby
cardiopulmonary bypass.

55. A 42-year-old male who is scheduled to undergo elective knee surgery has a preoperative chest film that demonstrates a
5-cm. posterior mediastinal mass. The patient denies any neurologic symptoms and physical examination fails to elucidate
any neurologic deficit. CT confirms the presence of a 5-cm. mediastinal mass in the left costovertebral gutter with minimal
enlargement of the seventh thoracic foramen. Appropriate intervention includes:
A. Resection of the posterior mediastinal mass using a standard posterolateral incision.
B. A CT with myelography or magnetic resonance (MR) imaging.
C. Two-stage removal of the tumor, performing the resection of the thoracic component first with subsequent removal of
the spinal column component at a later date.
D. One-stage removal of the dumb-bell tumor, excising the intraspinal component prior to resection of the thoracic
component.
Answer: BD

DISCUSSION: Approximately 10% of neurogenic tumors extend into the spinal column and are termed dumb-bell tumors
because of the characteristic shape. Although 60% of patients with such tumors have neurologic symptoms related to spinal
cord compression, the significant proportion of patients without symptoms underscores the importance of evaluating all
patients with a posterior mediastinal mass for possible intraspinal extension. CT, MR imaging, and vertebral tomography
may demonstrate an enlargement of the foramen, erosion of bone, or intervertebral widening, which are indicative of a
dumb-bell tumor. If these findings are present, CT with myelography or MR imaging is indicated to evaluate the presence
and extent of the intraspinal component. A one-stage removal of the tumor is recommended, with excision of the intraspinal
component prior to resection of the thoracic component to minimize the risk of spinal column hematoma.

56. True statements regarding patients with a mediastinal mass include:


A. Asymptomatic patients have a benign mass in over 75% of cases.
B. Symptomatic patients are more likely to have a malignant lesion than a benign lesion.
C. In a patient with a chest film demonstrating a mediastinal mass, a Tru-cut needle biopsy is a safe procedure.
D. Seminomas usually produce alpha-fetoprotein.
Answer: AB

DISCUSSION: Seventy-six per cent of the asymptomatic patients with a mediastinal mass seen in one series over a recent
20-year period had a benign leison. In contrast, 62% of the symptomatic patients had a malignant neoplasm during this
period. A number of intrathoracic and extrathoracic lesions may have an appearance similar to a primary mediastinal mass
on routine chest films, as do a large number of cardiovascular lesions. Although angiography was used in the past for this
differentiation, CT with contrast and MRI now distinguish a primary mediastinal mass from a cardiovascular lesion. Tru-cut
needle biopsy of a cardiovascular lesion may be associated with significant hemorrhagic complications. Seminomas rarely
produce beta–human chorionic gonadotropin and never produce alpha-fetoprotein. In contrast, over 90% of the
nonseminomas secrete one or both of these hormones.

57. Which of the following would be the least appropriate in the management of acute suppurative mediastinitis?
A. Wide débridement.
B. Irrigation under pressure.
C. Topical antibacterials.
D. Long-term systemic antibacterials.
E. Closure with muscle flaps.
Answer: D

DISCUSSION: Acute suppurative mediastinitis is a classic wound problem and forms a paradigm for principles of
management. Wide débridement is perhaps the most important step in correcting this type of invasive wound sepsis.
Drainage requires removal of tissue with vascular compromise. Tissue that is infected and can serve as an ongoing nidus for
infection, particularly cartilage, must be removed. Irrigation is effective only when the irrigation fluid reaches into and
flushes out débris and bacteria. The irrigation is insufficient if only dilutional and not also mechanically effective. Since
infected tissue tends to become isolated from the systemic circulation the direct application of antibacterials reaches
avascular areas. Some, such as silver sulfadiazine, penetrate avascular tissue better than, for instance, ointments or povidone
iodine, and such an agent should be chosen. Wide débridement and the washing of debris with pressure irrigation make the
wound then available to topical applications, which are often best packed into these deep, irregular cavities. Long-term
systemic antibacterials serve no purpose and lead to potential resistant bacterial overgrowth. Although systemic
antibacterials provide a measure of protection up to the margin where vascularized and nonvascularized tissues meet, topical
agents are better in the actual infected site. Once closed, these wounds rapidly become sterilized. Even the infection at the
bone level is far different from traditional osteomyelitis, and long-term systemic therapy is unnecessary. Muscle flaps are a
great advance in closure technique, since they provide bulky protection, obliterate dead space, and help vascularize the
wound.

58. Each of the following is appropriate for managing acute suppurative mediastinitis except:
A. Alloplastic material and skin flaps.
B. Rectus abdominis muscle flaps.
C. Omentum.
D. Pectoralis major muscle flaps.
E. Rigid internal fixation.
Answer: A

DISCUSSION: Alloplastic materials may be nonreactive in the laboratory and biologically acceptable in other areas
(artificial hips, breast prostheses). Their introduction into a contaminated wound, however, would more likely promote
rather than reduce infection. Various meshes and other types of “protection” devices are not necessary. Skin flaps alone do
not obliterate dead space and have not been shown either to reduce or resist infection. The rectus abdominis muscle is a
superb source of readily available tissue that can be rotated into very large cavities. The nature of the muscle allows it to be
“dressed into” irregular cavities. It has an excellent, easily movable skin territory overlying it, which can also be transferred
if locally available skin is wanting. The omentum has the great ability to fit into the many irregularities of some defects. For
appropriately selected cases it is excellent. The pectoralis major muscle flaps are the usual initial choice since they are in the
operative field. When the musculotendinous insertion is released their mobility is often sufficient. Additionally, it avoids the
need for abdominal incisions. The latissimus dorsi muscle as a flap is dependable and includes sternal defects in the scope
of its arc of rotation. It requires rotating the patient on the operating table and thus is less readily available than the other
flaps.

59. Clinical features suggestive of myasthenia gravis include all of the following except:
A. Proximal muscle weakness.
B. Diplopia.
C. Sensory deficits of the extremities.
D. Dysphagia.
Answer: C

DISCUSSION: Weakness of proximal weight-bearing muscle groups is the hallmark of the clinical diagnosis of myasthenia
gravis. The weakness or fatigue occurs with repetitive activity and improves with rest. The majority of patients (90%)
experience ocular muscle involvement, manifested as diplopia or ptosis most easily demonstrated with sustained upward
gaze. Cranial nerve involvement is uncommon but can be present, with symptoms of dysphagia, nasal regurgitation, and
aspiration. Since myasthenia gravis is a disorder of neuromuscular transmission at the motor end plate, deep tendon reflexes
and sensory examination are normal.

60. The diagnosis of myasthenia gravis can be confirmed most reliably using:
A. Anti–acetylcholine receptor antibody titers.
B. The Tensilon test.
C. Electromyography (EMG).
D. Single-fiber EMG.
E. Physical examination.
Answer: D

DISCUSSION: Although findings from a careful history and physical examination are suggestive of the diagnosis of
myasthenia gravis, specific diagnostic testing is required to confirm the diagnosis. Elevated anti–acetylcholine receptor
antibodies are present in 85% to 90% of patients with generalized myasthenia but are often negative in patients with early or
ocular myasthenia gravis. The Tensilon test is also positive in approximately 90% of patients with generalized myasthenia
gravis, but both false-negative and false-positive results occur, especially in patients with mild or early disease. Standard
EMG studies are helpful if positive, but their overall sensitivity may be as low as 35%. The specialized technique of single-
fiber EMG is the most reliable diagnostic test, being abnormal in 90% of patients with mild disease and in virtually 100% in
patients with severe generalized myasthenia gravis.

61. All of the following statements are true about the pathogenesis of myasthenia gravis except:
A. The number of functional acetylcholine receptors at the motor end plate is reduced.
B. An autoimmune mechanism involving antibodies to the acetylcholine receptor has been proposed.
C. Complement system involvement has been demonstrated.
D. A nonspecific “thymitis” may initiate the autoimmune response.
E. Clinical improvement following thymectomy is correlated with decreased acetylcholine receptor antibody titers.
Answer: E

DISCUSSION: Myasthenia gravis is generally regarded as an autoimmune disorder due to antibodies directed toward the
acetylcholine receptor. A variety of autoimmune mechanisms have been proposed; the ultimate result is a reduction in the
number of functional acetylcholine receptors at the motor end plate. Proposed immune mechanisms include
complementmediated receptor destruction, antibody-induced accelerated receptor turnover, and simple receptor blockade. In
spite of these proposed immune mechanisms, the severity of myasthenia symptoms and improvement following therapy do
not correlate with antibody titers. Although the source of these autoantibodies is not proven, it is generally felt that a
nonspecific thymitis may trigger the autoantibody response, the thymic myoid cells serving as the source of the antigen.

62. Which of the following statements about the relationship of the thymus and myasthenia gravis is/are true?
A. Thymic abnormalities are present in up to 80% of patients with myasthenia gravis.
B. Thymoma is present in up to 20% of patients with myasthenia gravis.
C. Myasthenia gravis will occur in up to 60% of patients with thymomas.
D. Myasthenia patients with thymoma respond more favorably to thymectomy.
E. Thymoma is the most common abnormality of the thymus in patients with myasthenia gravis.
Answer: ABC

DISCUSSION: The central role of the thymus gland in the pathogenesis of myasthenia gravis is based on the observation
that more than 80% of patients have histologic abnormalities of the thymus and on the beneficial effect of thymectomy on
patients' symptoms. Of the patients with documented abnormalities of the thymus the majority have B-cell lymphoid
hyperplasia; only 20% have a thymoma. Conversely, up to 60% of patients with known thymoma will have or ultimately
develop myasthenia gravis. In these patients, with thymoma and myasthenia gravis, the response to thymectomy is less
favorable than in those without thymoma.

63. Which of the following statements about the results of thymectomy for myasthenia gravis are true?
A. Patients with ocular symptoms experience clinical improvement in 90% of cases.
B. Clinical remission can be expected in 90% of cases.
C. The response rate to thymectomy for patients with generalized symptoms is 90%.
D. Patients with thymoma experience improvement in 75%.
E. Continued medical therapy is required in 75%.
Answer: C

DISCUSSION: Overall, improvement can be expected in 90% of patients who undergo thymectomy for generalized
myasthenia gravis. In general, the results are more favorable in patients with mild generalized myasthenia. In patients with
only ocular symptoms, the benefit following thymectomy is less clear; improvement is documented in 80%. The response
rate is even less (30%) in patients with thymoma. Complete remission occurs in 40% to 50% of patients following
thymectomy, and the remainder require some continued medical therapy.

64. All of the following are true of the treatment of myasthenia gravis except:
A. The transcervical approach to surgical thymectomy is less likely to benefit the patient with myasthenia gravis.
B. Corticosteroids result in improvement in 80% of patients.
C. Plasma exchange is associated with improvement in up to 90% of patients.
D. Medical therapy with Mestinon (pyridostigmine) is associated with remission in approximately 10% of patients.
E. Surgical thymectomy, regardless of the approach, is associated with improved remission and response rates as compared
with medical therapy.
Answer: A

DISCUSSION: Although Mestinon therapy results in clinical improvement in most patients, complete remission can be
expected in only 10%. In addition, intolerable side effects may limit their usefulness. In patients who fail to respond to
Mestinon therapy, and in those who experience significant side effects, corticosteroids can be utilized, with improvement
expected in 80% of patients. Plasma exchange results in improvement in 90% of patients, but the cost of therapy and its
transient duration of benefit limit the use of pheresis therapy to special circumstances such as preoperative preparation or in
myasthenic crisis. Overall, response rates to surgical thymectomy range from 80% to 95%, and complete remission occurs
in 30% to 50%. This benefit following thymectomy has not been shown to depend on the particular technique utilized.
Remission and response rates are similar for transcervical, standard transsternal, and the “maximal thymectomy”
techniques.

65. Which of the following is/are acceptable alternatives in the management of malignant pericardial effusion?
A. Pericardiocentesis.
B. Subxiphoid pericardiotomy (“pericardial window”).
C. Thoracotomy with pericardiectomy.
D. Instillation of tetracycline or bleomycin into the pericardial space.
E. Treatment of the underlying malignancy.
Answer: ABCDE

DISCUSSION: In patients with symptomatic malignant pericardial effusions, management options may be designed to
establish a diagnosis, relieve symptoms, or prevent recurrence. Pericardiocentesis is very successful in removing fluid for
diagnosis and alleviating symptoms; however recurrence rates are greater than 50%. This rate can be reduced to around 20%
with instillation of sclerosing agents such as tetracycline or bleomycin. Surgical techniques, including subxiphoid
pericardiotomy and thoracotomy with pericardiectomy, offer the highest success rates (approximately 90%) but are more
invasive and usually require general anesthesia. Systemic antitumor therapy with chemotherapy or radiation therapy can be
effective in controlling malignant effusions in cases of sensitive tumors such as lymphomas, leukemias, and breast cancer.

66. Which of the following statements about cardiac tamponade is/are correct?
A. At least 500 ml. of fluid must be present in the pericardium of an adult to cause symptoms of tamponade.
B. A drop in systemic blood pressure of greater than 20 mm. Hg during inspiration (pulsus paradoxus) is a finding specific
to cardiac tamponade.
C. The vast majority of patients with cardiac tamponade demonstrate a low QRS voltage, nonspecific ST T-wave
abnormalities, and electrical alternans (alternation of QRS amplitude) on the electrocardiogram.
D. In trauma victims with cardiac tamponade, the three components of “Beck's triad” (hypotension, elevated jugular venous
pressure (JVP), and muffled heart sounds) are almost always present.
E. When the diagnosis is made, treatment must be instituted rapidly and may include pericardiocentesis, creation of a
pericardial window, and identification and treatment of the underlying cause.
Answer: E

DISCUSSION: Development of tamponade symptoms depends on the rate of accumulation of fluid. As little as 100 to 200
ml. accumulating rapidly may cause symptoms, whereas a slowly developing pericardial effusion of over 1 liter may remain
asymptomatic. Pulsus paradoxus is not specific for tamponade; it may occur in patients with severe congestive heart failure,
chronic obstructive pulmonary disease, hypovolemia, acute pulmonary embolism, or shock. Electrocardiographic findings
of low QRS voltage and nonspecific ST T-wave changes are common in this condition, but electrical alternans, often
considered pathognomonic of cardiac tamponade, is present in only a small number of patients. Trauma victims with
tamponade frequently lack one or more of the elements of Beck's triad; for example, associated hypovolemia may lead to
low or normal jugular venous distention. Since cardiac tamponade is life threatening, therapy designed to drain the
pericardial fluid must be provided quickly and the underlying cause must be established and controlled.

67. Which of the following statements about constrictive pericarditis is/are correct?
A. Most patients who develop constrictive pericarditis after cardiac operation present with symptoms within 6 months of
the procedure.
B. Results of pericardiectomy for constrictive pericarditis are worse in patients who develop constriction after mediastinal
irradiation.
C. Drainage of asymptomatic pericardial effusions arising from acute pericarditis is advised to prevent development of
constrictive pericarditis.
D. If surgical treatment is planned for constrictive pericarditis it should involve total or complete pericardiectomy.
E. Echocardiography can usually make the diagnosis by imaging a thickened pericardium.
Answer: BD

DISCUSSION: The time course in the development of constrictive pericarditis after cardiac surgery ranges from 1 month to
nearly 9 years, but the mean interval from surgery to presentation is about 23 months. Most series have reported poorer
outcomes from pericardiectomy for postirradiation constrictive pericarditis, possibly owing to underlying myocardial
fibrosis. In this subset, 5-year survival averages 50%, as compared with 75% for constrictive pericarditis of all causes.
Constrictive pericarditis is a rare complication of acute pericarditis. As a result, drainage of asymptomatic (nonpurulent)
pericardial effusions from acute pericarditis is not required. Patients with significant symptoms from constrictive
pericarditis should undergo total pericardiectomy, even though this procedure carries an operative mortality rate of
approximately 10%. Limited pericardiectomy has proven to be ineffective for this condition. It can be difficult to distinguish
constrictive pericarditis from restrictive cardiomyopathy. Echocardiography may help by demonstrating chamber
dimensions and wall motion abnormalities, but CT and MRI more accurately assess pericardial thickness.

68. The relationship between small-cell and non-small cell lung cancers can be described by the following:

a. They differ by histology, clinical behavior and cell of origin


b. Of all lung cancers, approximately 80% are non-small cell and 20% are small cell
c. Both cell types are predictably responsive to chemotherapy
d. The International Staging System can be applied to both tumor types
e. The majority of non-small cell cancer patients vs. the minority of small cell cancer patients are candidates for
pulmonary resection
Answer: b

Although small cell and non-small cell lung cancers do differ by histology and clinical behavior, they probably have a
common origin since c-myc or n-myc amplified small cell lung cancer lines will undergo transition to non-small cell
phenotypes after insertion of an activated ras/gene. The overall incidence of lung cancers is 80% non-small cell and 20%
small cell. Only the small cell carcinoma is predictably responsive to chemotherapy.
The staging system for small cell lung cancer is based on limited vs. extensive disease outside of a tolerable radiotherapy
portal while the International Staging System uses TNM descriptors for 4 clinical stages. Unfortunately, only about 30% of
patients with non-small cell lung cancer have potentially resectable tumors.

69. A 62-year-old male smoker presents with right anterior chest pain. There is a 3 cm mass attached to the chest wall
with radiographic evidence of rib erosion and positive cytology for non-small cell carcinoma. Which of the follow is/are
true:

a. The patient is inoperable due to tumor size and chest wall involvement
b. Radiation therapy is the preferred initial treatment
c. Operative resection should be performed with en bloc removal of the tumor and adjacent chest wall as well as a
mediastinal lymph node resection
d. Positive mediastinal nodes will have little effect on survival
e. The patient would be classified Stage IIIa
Answer: c, e

Survival after resection for non-small cell lung cancer is related to the stage of the disease with a strong adverse effect from
nodal involvement. This is true even for large peripheral tumors that extend into the chest wall as in this case where a 40–
50% survival would be expected in the absence of nodes (T3N0:Stage IIIa) but only a 15% survival with nodal
involvement. Radiation therapy would be a postoperative consideration to reduce the incidence of local recurrence. En bloc
operative resection of the involved lobe and mediastinal nodes for staging would offer the greatest likelihood of cure.
70. For the patient in the pervious question to become an operative candidate which of the following must be met?

a. Extrathoracic metastases must be able to be controlled by another modality, e.g. radiotherapy


b. Tumor doubling time must exceed 40 days
c. If there is recurrence at the primary site, it must be treated before the metastatic disease
d. Even if effective systemic therapy is available, resection of metastases is preferred
e. If pulmonary reserve is marginal, resection of the maximal number of metastatic foci should be performed
Answer: c

There are a number of controversial areas in the area of operative approaches to metastatic disease in the lung, but there is
general agreement that any extrathoracic metastases preclude eligibility for pulmonary resection. Although tumor doubling
time is a measure of its aggressiveness, it is too variable to have prognostic significance and is generally disregarded as a
criterion for resection. Primary site recurrence must be treated before the metastatic focus to prevent further seeding. If
effective systemic therapy is available as would be expected in breast and testicular cancer or osteogenic sarcoma, it is
preferred over surgical resection. Similarly, pulmonary resection should not be undertaken unless the pulmonary reserve
will allow all metastatic foci to be resected.

71. Biopsy of the lesion in the previous question is reported as “bronchial carcinoid with no signs of atypia.” Which of
the follow is/are true?

a. Sleeve resection of the bronchus would be appropriate


b. Lymph node biopsy at time of resection is unnecessary
c. Associated carcinoid syndrome is very unlikely
d. If carcinoid syndrome were found in a tumor this size, hepatic metastases would be likely
e. When bronchial carcinoid syndrome occurs, right-sided cardiac valves are affected
Answer: a, c, d

In the absence of atypia, carcinoids are only locally malignant and can be managed by limited lung and/or bronchial
resection. Therefore, a sleeve resection of the bronchus preserving distal lung would be appropriate. Lymph node sampling
at the time of resection, however, is advisable to ensure that a complete resection has been performed. The carcinoid
syndrome is rarely found except in the presence of a large primary tumor or hepatic metastases. When the carcinoid
syndrome does occur, it is left-sided cardiac valves that are affected rather than right, which one would expect with
gastrointestinalcarcinoids.

72. In the evaluation and preparation of a 55-year-old smoker for resection of a 3 cm pulmonary adenocarcinoma, the
following is/are true:

a. Preoperative cessation of smoking does not reduce postoperative pulmonary complications


b. Resting PaCO2 is of more value than PaO2
c. FEV1 is of more value than measured vital capacity
d. Diffusion capacity should be measured routinely
e. V/Q lung scan is useful when pulmonary reserve is marginal
Answer: b, c, e

Preoperative cessation of smoking for a period of 2 weeks can reduce pulmonary complications and should be required. In
the preoperative assessment for pulmonary resection, the PaCO2 is of more value than the PaO2 since an elevated PaCO2 >
50 mmHg identifies the very high risk patient with chronic lung disease. Hypoxemia may be secondary to the mechanical
effects of the tumor producing ventilation/perfusion mismatch. The latter can be confirmed by V/Q lung scan which also
serves to identify areas of functioning lung in patients with marginal pulmonary function. The best screening test for
adequacy of pulmonary reserve is the FEV1. It identifies obstructive pulmonary disease which is more important than the
restrictive lung disease identified by vital capacity measurement. Diffusion capacity measurement provides little additional
information of value.

73. Following resection of a T1N1 squamous cell cancer in a 47-year-old male, the following is/are true:
a. There is a higher risk of local recurrence than with any other histologic type of non-small cell cancer
b. The greatest risk to the patient is a distant metastasis
c. Of all metastatic sites, liver is most likely
d. If the patient survives five years, there is a greater risk of a new lung cancer than recurrence
e. To improve survival, the patient should be considered for adjuvant chemotherapy
Answer: a, b, d

The risk of local recurrence for non-small cell carcinomas of the lung is much more common for those of squamous cell
histology than the others and averages 20%–30% overall. The greatest risk, however is of distant metastases which occur in
70%–80% of patients, regardless of stage. Almost all recurrences are seen within five years, and of the distant metastatic
sites, the brain is most commonly affected. In this patient with Stage II disease, radiation therapy would be a consideration
to reduce the incidence of local recurrence, but not chemotherapy. After five years, the highest risk would be from a new
lung cancer rather than a recurrence.

74. A 42-year-old woman with hemoptysis is seen to have a 2 cm mulberry appearing polypoid lesion in the left
mainstem bronchus suspicious for bronchial adenoma. The differential diagnosis includes which of the following:

a. Mucoepidermoid carcinoma
b. Plasma cell granuloma
c. Carcinoid tumor
d. Adenoid cystic carcinoma
e. Mucous gland adenoma
Answer: all of the above

The term bronchial adenoma includes a spectrum of tumors arising from epithelial stem cells which vary from the benign
mucous gland adenoma to the malignant adenoid cystic and mucoepidermoid carcinomas as well as the carcinoid tumors of
similar varied behavior. Among these variants, the carcinoid are most common representing 80%–90% of all bronchial
adenomas.

75. A 42-year-old man has a solitary “coin lesion” 2 cm in diameter in the area of the right upper lobe on a routine
chest radiograph. Which of the following is/are true?

a. A previous radiograph from five years prior showing the lesion to be 1.2 cm in diameter indicates malignancy
b. If a CT scan shows mediastinal adenopathy, mediastinoscopy is preferable to thoracotomy
c. In the absence of previous radiographs, the lesion should be followed by serial films at 6 month intervals
d. Calcification in a concentric or “popcorn” configuration denotes a benign lesion
e. Needle aspiration showing “chronic inflammatory cells” denotes a benign lesion
Answer: b, d

In the evaluation of a solitary lung lesion, previous radiographs are important, particularly if the lesion is new. A coin lesion
that is growing slowly does not necessarily indicate malignancy, since the most common benign tumor, hamartoma, has a
variable pattern of slow growth and typically will show “popcorn” calcification. Concentric calcification is also most
suggestive of a benign granuloma. In the absence of previous radiographs, the lesion must be assumed to be malignant until
proved otherwise and should not be dismissed to follow-up. If a CT scan shows mediastinal adenopathy, then
mediastinoscopy with biopsy is appropriate to make a diagnosis. Needle aspiration results of “chronic inflammatory cells” is
non-diagnostic.

76. A 2 cm peripheral squamous cell carcinoma in the lung of a 60-year-old male with a pleural effusion positive for
malignant cells would be classified as:

a. T1N0M1
b. T3N0M0
c. T3N0M1
d. T4N0M0
e. T4N0M1
Answer: d, e

The presence of a pleural effusion in association with a primary lung cancer is usually an ominous sign precluding surgical
resection. However, if more than one sample of the effusion is negative for malignant cells and it is non-bloody, it can be
considered unrelated to the tumor and excluded as a staging element. When the effusion cytology is positive, the tumor is
considered T4 regardless of size or nodal status.

77. A 53-year-old woman who had a malignant tumor removed 2 years ago presents with a solitary lung nodule 1.5 cm
in diameter. The following is/are true:

a. If the primary tumor originated in the breast, the lesion is most likely to represent a new primary lung cancer.
b. If the primary tumor was melanoma, the lesion is most likely to be metastatic
c. If the remainder of the lung fields are clear, a CT scan is unnecessary
d. If the primary tumor was in the GI tract, there is very little chance that the lesion is a new primary lung cancer
e. Fine needle aspiration should always be performed prior to resection of the lung lesion
Answer: a, b

A new pulmonary lesion in a patient with a history of a previously treated malignancy poses a diagnostic and therapeutic
challenge. A CT scan should always be obtained since plain radiographs can detect lesions only 9 mm in diameter or greater.
The lesion is most likely to be metastatic if the prior malignancy was sarcoma or melanoma and most likely to be a new
primary lung cancer if the prior malignancy originated in the head, neck or breast. When the original lesion was in the GI or
GU tract, there is an equal chance that it is metastatic or a new primary. Fine needle aspiration does not usually alter the
plan for excision and is done only when the patient is not an operative candidate or desires to know the diagnosis.

78. A 61-year-old male presents with a painful mass 3.5 cm in diameter below the clavicle and attached to the chest
wall. The following is/are true:

a. A CT scan is the best study to determine rib destruction


b. The lesion should be removed enbloc without biopsy to minimize the chances for local recurrence
c. The chances are approximately 40% that the lesion is metastatic
d. If it is metastatic, the most likely primary tumor is in the lung or pancreas
e. Fortunately, less than 50% of chest wall tumors are malignant
Answer: c

Chest wall tumors are uncommon, accounting for only 1–2% of all body tumors. About 57% of chest wall tumors are
primary, whereas 43% are metastatic. Solitary metastases most frequently arise from the thyroid gland, the GU tract and the
colon. Overall, about 60% of chest wall tumors are malignant, most arising form bone or cartilage. The CT scan is of value
in demonstrating the relationship between the mass and contiguous structures, but of little value in determining bone
destruction because of the oblique course of the ribs. Specific rib films are most helpful. Now that multimodality therapy is
available, core needle biopsies are recommended and have not increased the incidence of local recurrence.

79. Concerning the sternum, the following is/are true:

a. The xiphoid process is the anterior border of the thoracic outlet


b. Gladiolus is the body of the sternum
c. The angle of Louis is at the level of the 2nd costal cartilage
d. The 11th rib is attached via costal cartilage to the xiphoid
e. The sterno-manubrial junction is at the level of T4 posteriorly
Answer: a, b, c, e

The sternum consists of 3 segments, the upper manubrium, the body or gladiolus, and the xiphoid process which ends in the
rectus sheath and has no costal attachments. The xiphoid marks the anterior border of the thoracic outlet. The junction of the
manibrium and body is the sternal angle or angle of Louis which corresponds to the level of T4 posteriorly and attaches to
the 2nd costal cartilage anteriorly.
80. A 22-year-old woman recovering from a traumatic head injury is noted to have bright red bleeding when her
tracheostomy is suctioned. The following is/are true statement(s):

a. Antibiotics should be administered to treat the bronchitis


b. Deflation of the tracheal tube cuff is a useful diagnostic maneuver
c. If massive bleeding occurs, a finger should be used to compress the innominate artery against the sternum
d. Operative treatment of a tracheoinnominate fistula includes resection and prosthetic replacement of the innominate
artery
e. Tracheal resection is usually required for a tracheoinnominate fistula to prevent recurrence
Answer: b, c

The complication of tracheoinnominate artery fistula characteristically occurs in young women and is often heralded by
bleeding during the tracheostomy suctioning. Deflation of the tracheal tube cuff confirms the diagnosis if massive bleeding
occurs. At that point the tracheal tube cuff should be overinflated and a finger inserted into the tracheostomy incision to
tamponade the bleeding. Throughout this, the airway must be protected. Operative repair through an upper sternal split
requires resection of the innominate and coverage of the oversewn vessels with viable tissue since the wound is
contaminated. No prosthetic material should be inserted and tracheal resection is not necessary.

81. A 52-year-old alcoholic with fever and a cough productive of purulent sputum is found to have the opacity on chest
film as shown (Fig. 62-15). The following is/are true statement(s):

a. The findings suggest a parapneumonic empyema


b. If pus is found on aspiration of the pleural space, a chest tube should be placed
c. If pus is found on aspiration, bronchoscopy is a necessary part of the patient’s evaluation
d. In this situation, rib resection for drainage is preferred to a large-bore chest tube
e. Decortication of the lung should be considered if the lung fails to expand within 4 weeks
Answer: a, b, c

The posterior location of the infiltrate and fluid collection is typical of a parapneumonic empyema. The most important test
is pleural aspiration which will usually yield frank pus, at which time a chest tube should be placed. Formerly, oily Dionosil
was used to perform an empyemagram; this substance is now no longer commercially available. In the case of
parapneumonic empyemas, tube drainage alone may be sufficient to allow full expansion of the lung. If this is not the case,
a formal rib resection or early decortication should be performed. Decortication or marsupialization is indicated if the lungs
fail to expand after 6–8 weeks. Every patient with spontaneous empyema should undergo bronchoscopy to rule out
endobronchial obstruction by foreign body or tumor.

82. The lesion shown (Fig. 62-6) was found on a 32-year-old male on a routine chest film required for his employment.
Which of the following is/are true?

a. The stippled calcification and intact cortex of the rib are characteristic of osteochondroma
b. The stippled calcification is characteristic of osteogenic sarcoma
c. If the lesion is osteogenic sarcoma, the optimal treatment is resection and radiation therapy
d. If the lesion is an osteochondroma, it need not be resected in this age group
e. The radiographic picture is typical for Ewing sarcoma
Answer: a

Osteochondroma is the most common benign rib tumor and has a 3:1 male incidence. The stippled calcification and intact
rib cortex are characteristic for this lesion in contrast to the bone destruction of Ewing sarcoma and combined bone
destruction and “sunburst” calcification of osteogenic sarcoma. For both Ewing and osteogenic sarcoma, multimodality
therapy using preoperative chemotherapy followed by resection yields better results than with radiation therapy.
Osteochondromas in prepubertal children can be observed unless they become painful or enlarged, but are routinely resected
in adults.

83. To resect a chondrosarcoma of the chest wall in a 42-year-old man, ribs 2–4 were removed, leaving a defect 8 x 8
cm. For reconstruction, the following is/are true:
a. If this were to be posterior, beneath the scapula, reconstruction would not be required
b. If this defect is anterior, the primary benefit of reconstruction is an improved cosmetic result
c. Whenever chest wall reconstruction is considered, it should be delayed 6–12 months to allow detection of recurrent
tumor
d. If Marlex is used for reconstruction, no wound drainage tube is necessary
e. If PTFE is used for reconstruction, both pleural and wound tubes should be used
Answer: a, d, e

Skeletal chest wall defects that are full-thickness and occur posteriorly where they can be covered by the scapula do not
require reconstruction. Anterior chest wall defects do require reconstruction, primarily to stabilize the chest wall and prevent
paradoxical motion. The reconstruction should be immediate for optimal physiological benefit. Since Marlex mesh is
porous, only a wound catheter is needed as pleural fluid will drain through it. PTFE, however, is a solid sheet necessitating
both pleural and wound drainage.

84. An upright chest film of a cachectic, homeless 47-year-old woman shows blunting of the right costophrenic angle.
The following is/are true:

a. A lateral decubitus film should be obtained to confirm the presence of fluid rather than a CT scan
b. Tuberculous effusion can readily be identified by stain and culture of aspirated fluid
c. A pleural fluid glucose level lower than in the serum is diagnostic of empyema
d. Bloody pleural effusion in this patient is diagnostic of an underlying malignancy
e. Pleural fluid cytology report of lumphoma should be viewed with skepticism
Answer: a, e

Although the CT scan is a very sensitive indicator of pleural effusion, a lateral decubitus is the simplest way to differentiate
fluid from pleural thickening or fibrosis. Tuberculous pleuritis is difficult to diagnose by stain or culture which have a 30%
yield, but the diagnosis is facilitated by needle biopsy of the pleura. Pleural fluid glucose lower than in serum is
characteristic of rheumatoid arthritis, neoplasms, and tuberculosis as well as empyema. A red-tinged fluid can occur from
needle trauma, but even frankly bloody fluid in this patient may reflect trauma as well as underlying malignancy. Pleural
inflammation induces reactive changes in mesothelial cells that makes them resemble lymphocytes, so a lymphoma
diagnosis is suspect.

Following shotgun wound of the chest wall, a 39-year-old woman desires reconstruction without a foreign-body prosthesis.
Old incisions prohibit use of her rectus abdominus muscles. Considering chest wall muscles for reconstruction, the
following is/are true statement(s):

85. The pectoralis major muscle is available and innervated by the medial and lateral pectoral nerves so named because
it describes their relationship to the pectoralis minor
a. The serratus anterior muscle is available since its absence has no functional significance
b. There is no serratus posterior muscle
c. The latissimus dorsi muscle is available and supplied by the thoracodorsal artery
d. The latissimus dorsi is innervated by the thoracodorsal nerve with fibers from C6, C7 and C8
Answer: d, e

The pectoralis major muscle can be used for reconstruction but the medial and lateral pectoral nerves are named from their
respective cords of the brachial plexus. The serratus anterior muscle holds the scapula to the chest wall and its absence
produces the functional and cosmetically disabling winged scapula. The serratus posterior muscle is attached to the 7th
cervical and first three thoracic vertebrae posteriorly and functions as an accessory muscle of respiration. The constancy of
the vascular pedicle to the latissimus dorsi and its size allow this muscle to be used to reconstruct defects of the head, neck,
chest wall and pleural cavity. It is innervated by the thoracodorsal nerve with fibers from C6, C7 and C8.

86. A 38-year-old man presents with facial and upper extremity edema, venous distention in the neck and arms and a
cyanotic appearance. The following is/are true statement(s):

a. The most likely cause of the problem is mediastinal granulomatous disease


b. A venogram should be obtained to confirm the diagnosis
c. Mediastinoscopy for diagnosis is contraindicated
d. If a malignancy is identified, resection is indicated for palliation
e. If the etiology is benign disease, gradual improvement without operation is to be expected
Answer: e

Although mediastinal granulomatous disease is one cause of the superior vena cava syndrome described, the most common
cause (75%) is malignant disease. A venogram adds little information to the typical findings and increases risk from
extravasation of contrast medium subcutaneously from the venous hypertension. Mediastinoscopy can be used for diagnosis
with recognition of increased risk of bleeding and airway problems from the edema associated with the endotracheal
intubation required for the procedure. If a malignancy is found, operative resection is usually precluded by the extent of
mediastinal invasion. Fortunately, in the case of benign disease, the symptoms tend to improve with time as chest wall and
mediastinal collaterals enlarge.

87. A 39-year-old woman with hypertension and radicular chest wall pain was found to have the lesion seen on chest
radiograph (Fig. 63-23). The following is/are true statement(s):

a. The location of the lesion suggests a teratoma


b. High urinary vanillylmandelic acid levels would indicate that the lesion is a paraganglioma
c. If the lesion was seen on a film 5 years earlier, resection would not be indicated
d. A neurosurgical consultation should be obtained
e. Vasoactive intestinal polypeptide level elevation suggests a ganglioneuroma
Answer: d, e

The posterior mediastinal location of the tumor is most indicative of a neurogenic tumor while teratomas are
characteristically found in the anterior mediastinum. Neurogenic tumors can undergo malignant degeneration and should be
resected, particularly in this symptomatic patient even if known to be present for years. The radicular pain suggests the
possibility of intraspinous extension of the tumor, and therefore a neurosurgical consultation is appropriate. Both urinary
vanillylmandelic acid elevation and vasoactive intestinal polypeptide can be produced by ganglioneuroma but would not be
characteristic of a paraganglioma.

1. Which is not true of cardiopulmonary resuscitation (CPR)?


A. Closed chest massage is as effective as open chest massage.
B. The success rate for out-of-hospital resuscitation may be as high as 30% to 60%.
C. The most common cause of sudden death is ischemic heart disease.
D. Standard chest massage generally provides less than 15% of normal coronary and cerebral blood flow.
Answer: A

DISCUSSION: Closed chest massage is not as effective as open-chest massage in normalizing blood pressure or perfusion
of vital organs, and closed chest massage does generally deliver 5% to 15% of normal coronary and cerebral blood flow.
The success rate for out-of-hospital resuscitation has been as high as 30% to 60% when communities are prepared to
institute CPR early after a cardiac arrest. Ischemic heart disease is the most common cause of sudden death.

2. Which maneuver generally is not performed early before chest compression in basic life support outside the hospital?
A. Call for help.
B. Obtain airway.
C. Electrical cardioversion.
D. Ventilation.
Answer: C

DISCUSSION: Basic life support does involve calling for help, obtaining an airway, and beginning ventilation before
starting chest compression. Electrical cardioversion requires special equipment and trained personnel and thus is part of
advanced cardiac life support.
3. Which treatment would be least effective for asystole?
A. External pacemaker.
B. Intravenous epinephrine, 10 ml. of 1:10,000.
C. Intravenous calcium gluconate, 10 ml. of 10% solution.
D. Intravenous atropine, 0.5 mg.
Answer: C

DISCUSSION: Recommended treatment for asystole is administration of atropine. If atropine is unsuccessful epinephrine is
given. Ultimately cardiac pacing is necessary if atropine and epinephrine do not establish an adequate heart rate. Calcium
has no clear role in treating asystole.

4. The most important factor that influences the outcome of penetrating cardiac injuries is:
A. Comminuted tear of a single chamber.
B. Multiple-chamber injuries.
C. Coronary artery injury.
D. Tangential injuries.
Answer: C

DISCUSSION: Multiple studies in the literature confirm that injuries to the coronary arteries are the most important factor
in determining outcome after a penetrating cardiac injury. Tangential injuries are the least serious. Injury to a single chamber
—even if comminuted—or to multiple chambers is less likely to be fatal than are injuries that involve a major coronary
artery.

5. The most useful incision in the operating room for patients with penetrating cardiac injury is:
A. Left anterior thoracotomy.
B. Right anterior thoracotomy.
C. Bilateral anterior thoracotomy.
D. Median sternotomy.
E. Subxyphoid.
Answer: D

DISCUSSION: The subxyphoid incision is useful for determining if there is blood in the pericardium and if there is an
intracardiac injury; however, exposure is extremely limited, and definitive repair can rarely be performed through the
incision. Left (or right) anterior thoracotomy is easily performed, especially in the emergency room, and gives adequate
exposure to certain areas of the heart. However, each has significant limitations in exposure. Either may be extended across
the thoracotomy into the other side of the chest, thus producing a bilateral anterior thoracotomy. Exposure is excellent
through this incision, and most injuries can be satisfactorily repaired through this approach. Most cardiac operations today
are performed through median sternotomy incisions. If the patient is in the operating room, this incision is easily performed
and always provides excellent exposure for all areas of the heart.

6. In patients who present with a penetrating chest injury, injury to the heart is most likely when the following physical
sign(s) is/are present:
A. Hypotension.
B. Distended neck veins.
C. Decreased heart sound.
D. All of the above.
Answer: D

DISCUSSION: Hypotension, increased venous pressure (distended neck veins), and decreased heart sounds make up the
classic Beck's triad associated with cardiac tamponade. If these three findings are present in a person who has a penetrating
chest wound, intracardiac injury is almost certain and operative intervention is mandatory.

7. Which of the following would be an acceptable method of repair for a neonate with symptomatic isolated coarctation
of the aorta?
A. Resection with end-to-end anastomosis.
B. Prosthetic patch aortoplasty.
C. Subclavian flap aortoplasty.
D. Prosthetic tube graft repair.
Answer: AC

DISCUSSION: The most commonly used methods for coarctation repair are resection with anastomosis and subclavian flap
aortoplasty. Both have been shown to provide adequate relief of the obstruction with acceptable rates of restenosis. The
choice of repair depends on the patient's anatomy and the surgeon's experience. Patch aortoplasty was used frequently in the
past; however, because of concern over restenosis and aneurysm formation it is no longer commonly performed. Prosthetic
tube graft repair is avoided except in some complex cases and some cases of recoarctation.

8. Which of the following constitutes a true vascular ring?


A. Pulmonary artery sling.
B. Double aortic arch.
C. Anomalous origin of right subclavian artery from the descending aorta.
D. Cervical aortic arch.
Answer: B

DISCUSSION: Only the double aortic arch secondary to persistence of the right and left fourth aortic arches forms a true
vascular ring. Pulmonary artery sling may cause symptoms that are due to compression of the trachea, and an anomalous
right subclavian may cause dysphagia, but these anomalies do not constitute complete rings. Cervical aortic arch, which is
thought to be secondary to persistence of the third aortic arch, is not a complete ring and usually is asymptomatic.

9. Which of the following may be physical examination findings in a young adult with coarctation of the aorta?
A. Posterior systolic murmur between the scapulas.
B. Diminished femoral pulses.
C. Elevated blood pressure in left arm as compared with right arm.
D. Peripheral cyanosis.
Answer: ABC

DISCUSSION: A systolic murmur that radiates posteriorly is characteristic of coarctation of the aorta. Coarctation produces
obstruction to aortic flow, and thus the femoral pulse has a diminished volume with delayed upstroke. Hypertension in
coarctation is multifactorial, but the most important factors are diminished renal flow (single clip, single kidney-Goldblatt
model) and mechanical factors. If the right subclavian artery is anomalous and arises distal to the coarctation, blood
pressure may be greater in the left arm than in the right. Isolated coarctation does not produce cyanosis.

10. In a premature infant with hyaline membrane disease and inability to be weaned from mechanical ventilation, which of
the following would suggest hemodynamically significant patent ductus arteriosus (PDA)?
A. Continuous murmur.
B. Hyperactive precordium with bounding peripheral pulses.
C. Jaundice.
D. Diminished femoral pulses.
Answer: AB

DISCUSSION: PDA causes a left-to-right shunt that produces left ventricular volume overload. Physical findings include
evidence of hyperdynamic circulation with a prominent apical impulse and bounding peripheral pulses. The classic murmur
of PDA is a continuous or mechanical murmur heard over the precordium and radiating to the medial third of the clavicle.
Diminished femoral pulses are not seen with isolated PDA and would suggest other anomalies. PDA may result in
hepatomegaly but does not cause jaundice.

11. In an infant with suspected PDA, which of the following would be the optimal method of confirming the diagnosis?
A. Chest film.
B. Cardiac catheterization.
C. Retrograde aortography via an umbilical artery catheter.
D. Two-dimensional echocardiography with continuous-wave and color-flow Doppler echocardiography.
Answer: D

DISCUSSION: Echocardiography is the best method for confirming the diagnosis of a PDA. Two-dimensional
echocardiography can demonstrate PDA and exclude associated anomalies. Doppler echocardiography can demonstrate the
shunt, determine direction of shunting, and provide an estimate of shunt magnitude. The chest film is not particularly
helpful and may be normal or show cardiomegaly with pulmonary congestion. In general, cardiac catheterization should be
reserved for older patients and those with suspected associated anomalies or pulmonary hypertension.

12. Which of the following are potential complications of untreated coarctation of the aorta?
A. Endocarditis.
B. Pulmonary vascular disease.
C. Cerebrovascular accident.
D. Congestive heart failure.
Answer: ACD

DISCUSSION: Coarctation of the aorta produces an obstruction to blood flow and hypertension, turbulent flow, and
increased left ventricular afterload. There is an increased incidence of coronary artery disease. Prior to the introduction of
effective techniques for relief of coarctation, the most common causes of death were endocarditis, aortic rupture, congestive
heart failure, and cerebrovascular accident. Pulmonary vascular disease does not occur with isolated coarctation.

13. The atrial septal defect (ASD) most commonly associated with partial anomalous pulmonary venous return (PAPVR)
is:
A. Secundum defect.
B. Sinus venosus defect.
C. Ostium primum defect.
D. Complete atrioventricular (AV) canal defect.
E. Coronary sinus defect.
Answer: B

DISCUSSION: Although partial anomalous return of the pulmonary veins can occur with any of the ASDs listed, it is
particularly common with sinus venosus defects and is considered by many to be part of this lesion. The most common
anomaly is drainage of the right superior pulmonary vein to the lateral aspect of the superior vena cava.

14. The direction of an intracardiac shunt at the atrial level is controlled by:
A. The size of the defect
B. The compliance of the right and left ventricles.
C. The systemic oxygen saturation.
D. Right atrial pressure.
E. The presence or absence of an associated ventricular septal defect (VSD).
Answer: B

DISCUSSION: The direction of an intracardiac shunt is governed by the compliance of the downstream chambers. For an
atrial level shunt, the compliance of the right and left ventricles and their ability to distend with increased volume during
diastolic filling dictates the direction of the shunt flow. Since the right ventricle is usually a more compliant—and therefore
more distensible—chamber than the left ventricle, flow across an ASD occurs from left to right across the open tricuspid
valve during diastole. The size of an ASD does not correspond to the degree of shunt as long as the defect is large enough to
be unrestrictive to flow. A large shunt can occur through a relatively small defect if the ventricular compliance is favorable.

15. The ASD most commonly associated with mitral insufficiency is:
A. Secundum defect
B. Sinus venosus defect
C. Ostium primum defect.
D. Coronary sinus defect.
Answer: C

DISCUSSION: Ostium primum defects, or “partial” AV canal defects, are commonly associated with a “cleft” of the
anterior leaflet of the mitral valve. Depending on the deformity of the mitral valve, these defects can be accompanied by
variable degrees of mitral insufficiency. This cleft of the mitral valve needs to be repaired at the same time that the defect is
closed. Although other types of ASDs can be associated with mitral insufficiency, this is not as common. When mitral
stenosis exists with a secundum ASD the condition is often referred to as Lutembacher's syndrome.

16. An electrocardiogram (ECG) in a patient with a systolic ejection murmur that shows an incomplete bundle branch
block in the precordial lead is most consistent with:
A. A secundum ASD.
B. A sinus venosus ASD with PAPVR.
C. An ostium primum ASD.
D. A complete AV canal defect.
Answer: A

DISCUSSION: Many patients with secundum ASDs have an incomplete bundle branch block on their ECG. This is in
contradistinction to patients with ostium primum defects, who often have a left axis deviation. Although the ECG is not
pathognomonic of the defect, the findings are sometimes helpful along with other clinical and diagnostic information toward
elucidating the nature of the defect.

17. ASDs can all be closed with a pericardial or prosthetic patch. Which of the following ASDs can also be safely closed
primarily without the use of a patch?
A. Secundum ASD.
B. Sinus venosus ASD with PAPVR.
C. An ostium primum ASD.
D. A complete AV canal defect.
Answer: A

DISCUSSION: Secundum ASDs can frequently be closed primarily, although the use of a prosthetic or pericardial patch is
indicated for large defects. The other types of ASDs are more safely closed with a patch.

18. Obstruction to pulmonary venous return is associated with which of the following anomalies?
A. Partial anomalous pulmonary venous connection (PAPVC) to the superior vena cava.
B. Infracardiac (Type III) total anomalous pulmonary venous connection (TAPVC).
C. Pulmonary vein stenosis.
D. Cor triatriatum.
E. Supracardiac (Type I) TAPVC.
Answer: BCDE

DISCUSSION: Obstruction to pulmonary venous return is the most important factor affecting circulatory function when
pulmonary venous anomalies exist. This obstruction is most prevalent and severe in patients with infracardiac TAPVC, but
it also occurs in as many as 50% of patients with supracardiac TAPVC and 20% of patients with intracardiac TAPVC to the
coronary sinus. Obstruction to pulmonary venous return is also the primary pathophysiologic effect of both pulmonary vein
stenosis and cor triatriatum. Obstruction, however, is rare with partial anomalous pulmonary venous connection, especially
with the common form of PAPVC to the superior vena cava.

19. Postoperative complications associated with repair of TAPVC include:


A. Complete heart block.
B. Acute pulmonary hypertensive crisis.
C. Pleural effusions.
D. Pulmonary venous obstruction.
Answer: BD
DISCUSSION: In the early postoperative period after repair of obstructed forms of TAPVC, acute episodes of pulmonary
hypertension may develop as a response to stress. To minimize this potentially fatal complication, infants are kept
anesthetized with fentanyl and pancuronium for at least 48 hours. Residual or recurrent pulmonary venous obstruction
occurs in only 5% to 10% of patients after TAPVC repair, but if identified it requires early reoperation. Reoperation is
usually successful if the obstruction is at the level of the anastomosis. Unfortunately, in some cases, the obstruction is in the
pulmonary veins and surgical relief is less successful. Although complete heart block and pleural effusions can occur after
any cardiac operation, they rarely occur after TAPVC repair.

20. Which of the following statements about VSDs is/are correct?


A. Perimembranous lesions are located in the region of the membranous portion of the interventricular septum near the
anteroseptal commissure of the tricuspid valve.
B. Muscular VSDs are holes in the interventricular septum that are bordered by muscle on three sides and by the pulmonary
and the aortic valve annulus superiorly.
C. VSD, in its isolated form, is the most commonly recognized congenital heart defect.
D. The conduction bundle runs along the posteroinferior rim of a perimembranous VSD.
Answer: ACD

DISCUSSION: Perimembranous VSDs occupy the area of the membranous portion of the interventricular septum adjacent
to the anteroseptal commissure of the tricuspid valve. Often a remnant of the membranous portion of the interventricular
septum (the membranous flap) is left hanging on the posteroinferior rim of the defect. The annulus of the tricuspid and
aortic valves often form a part of the rim of the defect, but in some patients they are separated from the VSD by a thin rim
of muscle tissue that protects the conduction bundle. Muscular VSDs have exclusively muscular rims on all four sides.
VSDs in the outlet septum that extend to the annuluses of the aortic and pulmonary valves are called doubly committed or
juxta-arterial defects. Isolated VSDs occur at an approximate rate of 2 per 1000 live births and represent 30% to 40% of all
congenital heart malformations at birth. The conduction bundle in patients with perimembranous VSDs does run along the
posteroinferior rim of the defect on the left ventricular side. Sutures used for repair of a perimembranous VSD should be
placed well away from this area to avoid the creation of surgically induced complete heart block.

21. Which of the following statements about VSDs is/are true?


A. When coarctation of the aorta is associated with VSD, it most commonly occurs in infants with large lesions who have
to undergo repair before age 3 months.
B. In some patients with VSD, aortic valve incompetence develops over time and progresses.
C. In the United States doubly committed or juxta-arterial VSDs are most commonly associated with aortic insufficiency.
D. PDA is present in approximately one fourth of infants with a VSD and concomitant congestive heart failure.
Answer: ABD

DISCUSSION: VSD in combination with severe coarctation of the aorta occurs in approximately 17% of patients. This
combination is more common among infants with large VSDs undergoing operation before age 3 months. Aortic valve
incompetence does develop over time in some patients with VSD, presumably as a result of progressive prolapse of the right
aortic cusp through the defect. In the United States two thirds of patients with VSD and aortic insufficiency have
perimembranous lesions and one third have a doubly committed or juxta-arterial lesion. In Japan, however, the reverse is
true: two thirds have doubly committed or juxta-arterial lesions and one third have perimembranous lesions. A moderate- or
large-sized PDA is associated with VSD in approximately 6% of patients of all ages; however, in infants with VSD and
concomitant congestive heart failure, PDA is present in approximately 25%.

22. Which of the following statements about VSD is/are correct?


A. A large VSD is approximately the size of the pulmonary valve orifice or larger.
B. Large VSDs associated with high pulmonary blood flow result in an enlarged left atrium on chest x-ray.
C. Patients with small (restrictive) VSDs tend to have normal right ventricular and pulmonary arterial pressures with
normal pulmonary vascular resistance and no evidence of pulmonary vascular disease.
D. A pulmonary vascular resistance greater than 10 to 12 units per sq. m. is considered a contraindication to operation.
Answer: BCD

DISCUSSION: A large VSD is approximately the size of the aortic valve orifice or larger and causes systemic right
ventricular systolic pressures. In the absence of right ventricular outflow tract obstruction, the pulmonary artery systolic
pressure will also be systemic in the presence of a large VSD. Large VSDs associated with a high pulmonary blood flow do
result in an enlarged left atrium because of increased pulmonary venous return. When marked enlargement of the left atrium
is present in a patient suspected of having a VSD, the presence of coexisting mitral valve regurgitation should also be
considered. Patients with small VSDs do have normal right ventricular and pulmonary arterial pressures. There is only a
slight elevation of pulmonary blood flow relative to the systemic flow, and the pulmonary vascular resistance is normal
without evidence of pulmonary vascular disease. At any age, the presence of pulmonary vascular disease so severe that the
pulmonary vascular resistance is fixed and greater than 10 to 12 units per sq. m. is considered a contraindication to
operation.

23. Which of the following statements about VSDs is/are correct?


A. Spontaneous closure of VSDs occurs in 25% to 50% of patients during childhood.
B. Tachypnea and failure to thrive are symptoms frequently associated with large VSDs.
C. Patients with normal pulmonary vascular resistance and left-to-right shunting across the VSD have Eisenmenger's
complex.
D. Patients with a large VSD and low pulmonary vascular resistance can present with a middiastolic murmur at the apex.
Answer: ABD

DISCUSSION: Spontaneous and complete closure of VSDs, even large ones, has been estimated to occur in 25% to 50% of
patients during childhood. The probability of eventual spontaneous closure is inversely related to the age at which the
patient is observed. Tachypnea, poor feeding, growth failure, recurrent respiratory infections, exercise intolerance, and
severe cardiac failure may develop in patients with large VSDs. Patients with Eisenmenger's complex are cyanotic,
polycythemic, and severely limited in their exercise tolerance, owing to markedly elevated pulmonary vascular resistance
associated with a predominantly right-to-left shunt across the VSD. A middiastolic murmur can be present at the apex in
patients with a large VSD associated with low pulmonary vascular resistance. This indicates high pulmonary blood flow
with a large flow across the mitral valve into the left ventricle.

24. Which of the following is/are true of the surgical treatment of VSDs?
A. A right ventricular approach is employed for the repair of most perimembranous VSDs.
B. Intracardiac repair is advisable for patients with intractable symptoms and for asymptomatic infants with evidence of
increasing pulmonary vascular resistance.
C. Complete heart block is a common complication.
D. Hospital mortality after repair of VSD in infants approaches 20%.
Answer: B

DISCUSSION: The right atrial approach is preferred for the repair of most perimembranous VSDs. Prompt intracardiac
repair is indicated for infants with large defects, large shunts, and pulmonary hypertension who present with intractable left
ventricular failure, recurrent pulmonary infections, severe growth failure, or evidence of increasing pulmonary vascular
resistance. In the modern era, complete heart block requiring a permanent pacemaker is a very uncommon complication of
surgical closure of a ventricular septal defect. Hospital mortality after closure of a VSD currently approaches zero. While in
earlier years younger age was an incremental risk factor for hospital death in some surgical experiences, this risk has been
neutralized during the past decade.

25. Tetralogy of Fallot consists of all of the following features except:


A. ASD.
B. VSD.
C. Dextroposition of the aorta.
D. Pulmonary stenosis.
E. Right ventricular hypertrophy.
Answer: A

DISCUSSION: Although ASD is a frequent component of tetralogy of Fallot, it was not included by Fallot as part of his
classic tetralogy. Occasionally, the inclusion of an ASD prompts use of the term pentalogy of Fallot. The other four
anomalies listed were all mentioned by Fallot in his original description of this defect.

26. Which of the following has the greatest impact on the physiology of tetralogy of Fallot?
A. The size of the ASD.
B. The size of the VSD.
C. The degree of pulmonary stenosis.
D. The amount of aortic overriding.
Answer: C

DISCUSSION: The VSD in tetralogy of Fallot is nonrestrictive, and therefore its size does not affect the degree of shunting
that can occur. Likewise, an ASD, which may or may not be a component of tetralogy of Fallot, can provide right-to-left
shunting at the atrial level but is not the main contributor to the cyanosis of this disease. The degree of right-to-left shunt
across the VSD is dictated by the variable compliance of the downstream chambers, and the increased resistance imposed
by severe pulmonary stenosis creates greater amounts of right-to-left shunting and, therefore, more intense cyanosis. The
position of the aorta in relation to the VSD is not important as long as no subaortic obstruction exists.

27. Which of the following anomalies is not associated with tetralogy of Fallot?
A. Absence of the left pulmonary artery.
B. A right aortic arch.
C. A retroesophageal subclavian artery.
D. Anomalous origin of the left anterior descending coronary artery from the right coronary artery.
E. Primary pulmonary hypertension.
Answer: E

DISCUSSION: The first four defects listed occasionally are associated with tetralogy of Fallot. A right aortic arch is seen in
25% of patients with that lesion. Anomalous coronary arteries or a retroesophageal subclavian artery are found in as many
as 5% to 10% of patients. Absence of a pulmonary artery is unusual but can present in as many as 3% of patients.
Pulmonary hypertension is distinctly unusual with tetralogy of Fallot unless the patient has had excessive pulmonary blood
flow from collaterals or systemic-to-pulmonary artery shunts for a long time. It is because these patients usually do not have
pulmonary hypertension that infant correction with transannular patches can be performed with such great success.

28. Surgical treatment of a patient with tetralogy of Fallot can include any of the following except:
A. Maintenance of ductal patency with prostaglandins (PGE 1) to provide pulmonary blood flow while the baby is
transferred to an institution equipped to provide more definitive therapy.
B. Banding of the pulmonary artery in an acyanotic patient with tetralogy of Fallot to control pulmonary blood flow and
prevent the development of pulmonary hypertension.
C. Placement of a subclavian-to-pulmonary artery shunt on the side opposite the aortic arch in a 3-day-old infant with
severe cyanosis.
D. Closure of the VSD and transannular patching of the right ventricle onto the main pulmonary artery in a 2-day-old
infant.
Answer: B

DISCUSSION: Patients with tetralogy of Fallot who do not appear cyanotic still have mild arterial hypoxemia by arterial
blood gas determination. Patients with tetralogy of Fallot rarely have excessive pulmonary blood flow, and the development
of pulmonary hypertension is not a concern in this population. Banding of the pulmonary artery is never a consideration in
patients with tetralogy of Fallot, since the predominant physiologic effect of the defect results from too little pulmonary
blood flow to begin with. Acyanotic patients with tetralogy of Fallot (“pink tets”) can usually be followed for several
months and their defects repaired electively as a first-stage procedure (usually by age 6 months). All of the other therapies
are appropriate treatment for babies with tetralogy of Fallot. Prostaglandins maintain patency of the ductus arteriosus,
providing an anatomic systemic-to-pulmonary artery shunt that sustains pulmonary blood flow until a more permanent
surgical solution can be provided. The advent of prostaglandin therapy has enabled numerous critically ill infants to become
stabilized enough to reach a tertiary care institution and receive proper surgical therapy who might not otherwise have
survived had it not been for the ability of pulmonary blood flow to be maintained through the reversal of duct closing. The
choice of palliative shunting or total anatomic correction rests largely with the experience and skill of the surgical team and
is dictated in part by the anatomy of the pulmonary arteries. Either of these options is acceptable.

29. The predominant determinant of outcome for patients with pulmonary atresia and an intact ventricular septum revolves
around:
A. The size of the ASD.
B. The baby's age at presentation.
C. The size of the right ventricular cavity and tricuspid valve.
D. The presence of a tricuspid—as opposed to a bicuspid—pulmonary valve.
E. The level of hypoxemia at presentation.
Answer: C

DISCUSSION: The long-term outcome for babies with pulmonary atresia and intact ventricular septum depends on the
ability to convert the cardiac circulation into a two-ventricle versus one-ventricle physiology. Patients with a good-sized
right ventricle and tricuspid valve can often be treated with pulmonary valvotomy or right ventricular outflow patching
alone and can have a fairly acceptable outcome. Patients with a small right ventricle that cannot provide adequate pumping
to the pulmonary bed and is often associated with a small tricuspid valve annulus may need to be staged toward a Fontan
procedure—and, consequently, a less acceptable outcome. The size of an ASD is not relevant except that in patients with
this syndrome, the right side of the heart will decompress across the ASD until antegrade flow can be established.
Therefore, an ASD in some part is an essential feature of this lesion. The degree of arterial hypoxemia, the nature of the
pulmonary valve, and the patient's age at presentation may all be factors that relate to clinical management, but they do not
imply specific consequences with respect to long-term outcome.

30. Which of the following statements about double-outlet right ventricle are true?
A. A VSD is usually present.
B. In the Taussig-Bing type of double-outlet right ventricle, the VSD is usually noncommitted.
C. Patients with double-outlet right ventricle and a subaortic VSD usually have pulmonary stenosis.
D. Patients with double-outlet right ventricle with a subpulmonary VSD (Taussig-Bing malformation) tend to mimic
patients with transposition of the great arteries and VSD in their presentation and natural history.
Answer: ACD

DISCUSSION: A VSD is usually present in patients with double-outlet right ventricle and is the only outlet from the left
ventricle. Both great arteries may arise totally from the right ventricle, or one or both may overlie the ventricular septum
immediately above the VSD. To categorize the heart as having a double-outlet right ventricle, more than 50% of each great
artery must arise from the right ventricle. In the Taussig-Bing type of double-outlet right ventricle, the VSD is related to the
pulmonary valve annulus and is termed a subpulmonary defect. Additional morphologic characteristics peculiar to this
entity have been described. Most patients with double-outlet right ventricle and a subaortic VSD have concomitant
pulmonary stenosis that protects the lungs from pulmonary vascular disease and also results in a clinical course similar to
that of patients with tetralogy of Fallot. In the absence of pulmonary stenosis the presentation, clinical course, and natural
history of the Taussig-Bing malformation are similar to those of transposition of the great arteries with VSD. Cyanosis is
present, usually from birth, since streaming directs the desaturated systemic venous return toward the aorta and the
oxygenated left ventricular blood toward the pulmonary artery. These patients tend to develop early congestive heart failure
and can develop severe pulmonary vascular disease early in life. They usually experience symptoms within the first few
months of life.

31. Which of the following statements about the surgical repair of double-outlet right ventricle are true?
A. In double-outlet right ventricle with a subaortic or doubly committed VSD, a tunnel-type repair connecting a committed
VSD with its respective great artery is usually employed.
B. Repair of the Taussig-Bing malformation can be accomplished using an intraventricular tunnel technique or by
performing a straight patch closure of the VSD combined with an arterial switch procedure.
C. The hospital mortality rate is highest when a subaortic VSD is present.
D. Some hearts with double-outlet right ventricle and a noncommitted VSD must be repaired using a modification of the
Fontan procedure.
Answer: ABD

DISCUSSION: When the VSD is subaortic or doubly committed, the tunnel-type repair connects the left ventricle via the
VSD and tunnel to the aorta. The Taussig-Bing malformation can be repaired using an intraventricular tunnel technique
described by Kawashima. This repair can best be accomplished when the great arteries are in a more or less side-by-side
relationship with the aorta to the right of the pulmonary artery. The infundibular septum is generously resected and the VSD
is connected to the aorta by an intraventricular tunnel that runs posterior to the pulmonary artery. The most common
approach for the repair of the Taussig-Bing malformation involves patch closure of the VSD to the pulmonary artery. This
creates transposition of the great arteries with an intact interventricular septum. An arterial switch procedure then establishes
ventriculoarterial concordance. Of all the types of double-outlet right ventricle the hospital mortality is lowest when a
subaortic or doubly committed VSD is present. Double-outlet right ventricle is associated with a noncommitted VSD in
approximately 10% of patients in surgical series. The repair of this subset of patients is associated with a relatively high
mortality, as compared with the results obtained after repair of other forms of double-outlet right ventricle. At times,
because of the remote location of the VSD and because of other compelling anatomic features, complete repair cannot be
performed. In this case, a modification of the Fontan procedure must be employed.

32. Management of a patient with tricuspid atresia within the first month of life may include:
A. Creation of a systemic artery–to–pulmonary artery shunt.
B. Observation.
C. Creation of a bidirectional superior cavopulmonary anastomosis.
D. Pulmonary artery banding.
E. Fontan procedure.
Answer: ABD

DISCUSSION: Initial management of newborn infants with tricuspid atresia is determined by the anatomic and physiologic
factors that affect the balance of pulmonary and systemic blood flow. Infants with severely limited pulmonary blood flow
and arterial oxygen saturations of less than 70% should be stabilized with PGE 1 to maintain patency of the ductus
arteriosus until a systemic-to-pulmonary artery shunt can be performed. Patients with unobstructed pulmonary blood flow
may exhibit only mild cyanosis but suffer from significant congestive heart failure. Many of these patients are best managed
by pulmonary artery banding to decrease the volume overload on the left ventricle and to prevent the early development of
irreversible pulmonary vascular disease. Some patients with moderate restriction of pulmonary blood flow may have
balanced delivery of blood to the systemic and the pulmonary circulation. These patients can be carefully followed until
such time as an imbalance develops or they become candidates for a bidirectional superior cavopulmonary (Glenn)
anastomosis or a Fontan procedure. The normally high pulmonary vascular resistance present in the first month of life
precludes the performance of either the Glenn or the Fontan procedure in the newborn.

33. Which of the following should contraindicate performance of the Fontan procedure?
A. Patient age of 25 years.
B. Severe mitral insufficiency.
C. Left ventricular end-diastolic pressure of 18 mm. Hg.
D. Right pulmonary artery stenosis.
E. Pulmonary vascular resistance of 6 Woods units.
Answer: CE

DISCUSSION: Good ventricular function and low pulmonary vascular resistance are essential requirements for a successful
Fontan procedure. The Fontan operation should not be performed when ventricular ejection fraction is less than 30% or
ventricular end-diastolic pressure is greater than 15 mm. Hg. Pulmonary vascular resistance in excess of 4 Woods units
should also be considered an absolute contraindication for Fontan correction. Age at the time of Fontan procedure does not
appear to be a major risk factor, except before age 2 years. Although patients who have survived into the third or fourth
decade are likely to have ventricular dysfunction, a Fontan procedure can be performed successfully in these older patients
if ventricular function and pulmonary vascular resistance meet the above criteria. In patients with tricuspid atresia a
competent mitral valve is important for satisfactory cardiac output after the Fontan procedure. The presence of severe mitral
insufficiency, however, should not necessarily contraindicate the procedure. In these cases it is recommended that the mitral
valve be repaired or replaced in combination with the creation of a bidirectional superior cavopulmonary anastomosis. A
completion Fontan operation is performed later. Distorted or stenosed pulmonary arteries are common sequelae of systemic-
to-pulmonary artery shunts and may result in unsatisfactorily high pulmonary vascular resistance. In most cases, these
stenoses can be repaired at the time of Fontan correction or with a bidirectional superior cavopulmonary anastomosis.

34. Initial management of a newborn infant with hypoplastic left heart syndrome should include:
A. Intravenous administration of PGE 1.
B. Supplemental oxygen.
C. Routine intubation and mechanical ventilation to achieve a PCO 2 between 30 and 35 mm. Hg.
D. Cardiac catheterization and balloon atrial septostomy.
Answer: A

DISCUSSION: Postnatal stabilization of infants with hypoplastic left heart syndrome requires patency of the ductus
arteriosus and balance of the systemic and the pulmonary circulation. Because the ductus is the only pathway from the right
ventricle to the systemic circulation, duct patency must be maintained with intravenous PGE 1. To minimize the workload
on the single ventricle and ensure adequate delivery of oxygen to the tissues, an equal delivery of blood to both the lungs
and the body is sought. The normal postnatal decrease in pulmonary vascular resistance often results in overperfusion of the
pulmonary circulation and underperfusion of the systemic circulation. Maneuvers that further decrease pulmonary vascular
resistance, such as the addition of supplemental oxygen, lowering the PCO 2 to less than 35 mm. Hg, or eliminating any
resistance at the atrial septum by balloon septostomy only worsens the imbalance.

35. The performance of a bidirectional superior cavopulmonary (Glenn) anastomosis as the second stage in the
reconstructive approach to hypoplastic left heart syndrome:
A. Provides early relief of volume load on the single right ventricle.
B. Increases peripheral oxygen saturations to greater than 90%.
C. Permits concomitant repair of pulmonary artery or aortic arch stenoses.
D. Improves mortality and morbidity of subsequent Fontan procedure.
Answer: ACD

DISCUSSION: After the first-stage reconstructive (Norwood) procedure, the circulation is inherently inefficient because of
the obligatory recirculation of a portion of both saturated and desaturated blood. Closure of the arterial shunt and creation of
a bidirectional Glenn anastomosis eliminates this inefficient recirculation and significantly diminishes the volume load on
the single right ventricle. Distorted and stenosed central pulmonary arteries or aortic arch obstructions should be repaired at
the same time the bidirectional Glenn procedure is performed. In almost all series the mortality of the Fontan procedure has
decreased since the adoption of the three-stage approach for hypoplastic left heart syndrome. Because systemic and
pulmonary venous blood continue to mix in the right atrium after a bidirectional Glenn procedure, cyanosis persists with
peripheral oxygen saturations between 75% and 85%.

36. Which of the following statements about truncus arteriosus are true?
A. Most infants survive without operations until late childhood.
B. Most infants present with cyanosis.
C. Most infants present with congestive heart failure.
D. Repair requires a conduit from right ventricle to pulmonary arteries.
Answer: BCD

DISCUSSION: While an occasional child survives to age 3 or 4 years, without either palliative or totally corrective surgical
treatment few live past early infancy. The lesion is one of excessive pulmonary blood flow because of the origin of the
pulmonary arteries from the truncus arteriosus; physiologically, the pulmonary arteries arise directly from the aorta.
Although the aortic saturation can never be 100% because of some element of bidirectional shunting at the VSD, the
physiologic manifestations are congestive heart failure and excessive pulmonary blood flow rather than cyanosis. The
congestive heart failure becomes severe as pulmonary vascular resistance drops. If congestive heart failure later improves
spontaneously, it is because of the development of pulmonary vascular disease. Complete repair always requires closure of
the VSD, detachment of the pulmonary arteries from the common trunk, and re-establishment of an outflow tract from the
right ventricle to the pulmonary artery. This conduit usually contains a valve and can be either a homograft or a synthetic
conduit containing a porcine valve.

37. Truncus arteriosus is a diagnosis with anatomic components including:


A. VSD.
B. Abnormal origin of pulmonary arteries.
C. Subaortic stenosis.
D. Single ventricular outflow valve.
Answer: ABD

DISCUSSION: By definition, a VSD is always present immediately beneath the truncal valve. The pulmonary arteries arise
abnormally from the single trunk, which is due to failed partitioning of the embryonic conus during the first few weeks of
fetal development. In the classification of Collett and Edwards, Type I truncus arteriosus has a single arterial trunk giving
rise to an aorta and a main pulmonary artery; in Type II the right pulmonary arteries arise immediately adjacent to one
another from the dorsal wall of the truncus; in Type III the right and left pulmonary arteries originate from either side of the
truncus; and in Type IV the proximal pulmonary arteries are absent and pulmonary blood flow is by way of major
aortopulmonary atresia and is no longer considered truncus arteriosus. Subaortic stenosis cannot occur in this anomaly. The
single ventricular outflow valve is the truncal valve. It may contain from two to six cusps, but most often there are three
and, next most often, four.

38. Optimal treatment for the neonate who presents with transposition of the great arteries {S,D,D}* and intact ventricular
septum includes:
A. PGE 1 infusion to maintain duct patency.
B. Administration of intravenous fluid to increase intravascular volume.
C. Hyperventilation to decrease pulmonary resistance.
D. Oxygen administration to increase arterial oxygen tension.
E. Atrial balloon septostomy to improve atrial mixing.
Answer: ABE

DISCUSSION: Because with transposition of the great vessels the systemic and the pulmonary circulations exist in parallel
rather than in series, survival depends on mixing between pulmonary and systemic circulations. Initially infants with
transposition and intact atrial septum survive because of aortopulmonary flow through PDA, which may be maintained with
prostaglandin infusions. Although increased pulmonary flow may cause enlargement of the left atrium and stretching of the
foramen ovale resulting in atrial-level mixing of oxygenated and nonoxygenated blood, inadequate mixing at the atrial level
will result in marginal tissue oxygenation, which does not improve with oxygen administration. Atrial balloon septotomy
results in improved admixture and oxygen delivery in these patients and should be performed promptly if peripheral
acidemia and severe cyanosis are present. Relative dehydration may decrease the degree of interatrial shunting and volume
infusion often improves hemodynamics in infants. Decreased pulmonary vascular resistance may increase pulmonary blood
flow at the expense of systemic blood flow and alter the loading conditions of the left ventricle, which may complicate early
arterial repair.

39. Factors that preclude the use of a single-stage arterial switch reconstruction of dextrotransposition of the great vessels
include:
A. Age older than 6 weeks with a left ventricular pressure of less than 50% of systemic pressure.
B. Dynamic left ventricular outflow tract obstruction.
C. Intramural coronary artery anatomy.
D. Valvar pulmonic stenosis.
E. Subpulmonary VSD.
Answer: AD

DISCUSSION: Single-stage arterial switch procedure for reconstruction of transposition of the great vessels, with or
without associated VSD has become the standard of treatment in the majority of cardiac centers. Contraindications to
arterial switch repair include fixed types of left ventricular outflow tract obstruction, including valvar pulmonic stenosis,
which would render the systemic semilunar valve stenotic or incompetent. Anatomic abnormalities without stenosis, such as
a bicuspid valve, however, are suitable for surgical correction. The location of VSD does not affect surgical outcome, and
most VSDs can be approached adequately through the right atrium or the anterior great vessel. Most dynamic forms of left
ventricular outflow tract obstruction are often relieved partially or completely by realignment of the ventricular septum with
the hemodynamic changes following successful arterial switch repair. When, however, the left ventricle has not been
prepared to sustain the pressure load of the systemic circulation by the decrease in pulmonary vascular resistance that occurs
in the first few weeks of life and when the ventricular pressure is less than 50% of the systemic ventricular pressure, one-
stage repair is contraindicated, and staged repair with pulmonary banding and shunt followed by arterial switch must be
contemplated.

40. Complications commonly associated with the atrial (Senning and Mustard) repairs of transposition of the great arteries
include:
A. Atrial arrhythmias.
B. Systemic or pulmonary venous obstruction.
C. Right ventricular outflow tract obstruction.
D. Systemic ventricular failure.
E. Progressive elevation of pulmonary vascular resistance.
Answer: ABD
DISCUSSION: The atrial repair of transposition of the great arteries—rerouting systemic and pulmonary venous blood at
the atrial level—results in the right ventricle's becoming the systemic ventricle. This results in an anatomic right ventricle
with abnormal geometry sustaining the afterload of a more ideally suited left ventricle. Long-term complications of
ventricular dilatation, AV valve regurgitation, and right ventricular failure have been reported in as many as 10% of patients
many years following the atrial operation. The multiple suture lines in the atrium have been associated with a high incidence
of late atrial arrhythmias and a low incidence of sinus rhythm following the Mustard and Senning operations. These
complications do not appear to be as frequent with the arterial switch repair. In addition, the complicated interatrial baffles
have been associated with pulmonary or systemic venous baffle obstruction. Because the right ventricular outflow tract is
not addressed during an atrial switch operation, right ventricular outflow tract obstruction is not a recognized complication
following the repair. Right ventricular outflow tract and supravalvar pulmonic stenosis, however, have been reported in
patients after the arterial switch repair, owing to the reconstruction of the right ventricular outflow tract in that operation.
Although progression of pulmonary arterial obstruction has rarely been reported following early repair with the atrial or the
arterial switch procedure, it is an unusual complication if operation is undertaken in infancy. Delayed repair beyond age 6
months to 1 year, however, has been associated with a higher incidence of progressive development of pulmonary vascular
obstructive disease. The rapidity of the development of pulmonary vascular disease is increased by the coexistence of a
VSD.

41. Critical aortic stenosis in the neonate is characterized by which of the following?
A. It is most often due to commissural fusion of a trileaflet valve.
B. It may be associated with coarctation of the aorta, PDA, and mitral stenosis.
C. It can be managed medically until the child is large enough to undergo aortic valve replacement.
D. Success of valvotomy is determined by the adequacy of the left ventricle.
Answer: BD

DISCUSSION: Critical aortic stenosis in the neonate most often presents in the first week of life with severe and
progressive congestive heart failure and may be associated with coarctation of the aorta, PDA, and mitral stenosis. The
prognosis is poor unless valvotomy can be performed expeditiously. Medical management cannot stabilize these infants for
valve replacement at a later age. Infants whose left ventricle is too small to sustain the systemic circulation are unlikely to
survive aortic valvotomy and, thus, should be managed as patients with hypoplastic left heart syndrome. The aortic valve in
neonatal aortic stenosis is most commonly unicuspid or bicuspid.

42. Surgical management of aortic valve disease in an older child may include:
A. Enlargement of the aortic annulus.
B. Incision of fused commissures.
C. Insertion of a porcine valve prosthesis.
D. Transfer of the pulmonary valve to the aortic position.
Answer: ABD

DISCUSSION: The majority of older children with aortic stenosis and significant transvalvular gradients can be treated
successfully by aortic valvotomy. This can be done percutaneously with balloon dilatation or surgically with direct
visualization of the aortic valve and incision of the fused commissures. Aortic valve replacement is rarely necessary as a
primary procedure but may be required in children who develop progressive aortic insufficiency after a previous
intervention. When valve replacement is performed it is desirable to insert the largest prosthesis possible, to allow for
growth. Enlargement of the aortic annulus is commonly performed for this purpose. If a true valve prosthesis is employed, a
mechanical valve is preferred. Durability of xenograft valves in children is limited owing to early calcification and leaflet
degeneration. The pulmonary autograft technique may be the best method of aortic valve replacement in children. With this
operation the patient's own pulmonary valve is transferred to the aortic position and a pulmonary allograft is inserted to
replace the pulmonary valve. Although the pulmonary autograft may not achieve the long-term durability of a mechanical
valve, the patient does not face the long-term complications of thromboembolism and bleeding imposed by a mechanical
valve and lifelong anticoagulation.

43. Which of the following statements about subvalvular aortic stenosis are true?
A. Most patients present in early infancy with severe congestive heart failure.
B. An ejection click is a specific physical sign of subaortic stenosis.
C. The subaortic membrane is approached surgically via the aorta and aortic valve.
D. A concomitant septal myectomy decreases the incidence of recurrent subaortic stenosis.
Answer: CD

DISCUSSION: Subaortic stenosis is rarely encountered in neonates. Most often it is discovered in an asymptomatic child
during a routine physical examination. A loud crescendo–decrescendo systolic murmur without an ejection click is usually
noted. The presence of an ejection click is more consistent with isolated valvular aortic stenosis. Discrete subaortic stenosis
is approached surgically with cardiopulmonary bypass, aortic cross-clamping, and cardioplegic arrest. The aorta is opened
and the aortic valve leaflets are retracted, exposing the fibrous membrane. The fibrous ring is carefully excised, taking care
to avoid injury to the anterior leaflet of the mitral valve and the penetrating conduction bundle. Once the subaortic
membrane is excised a septal myectomy further opens the left ventricular outflow tract and diminishes the likelihood of
recurrent subaortic stenosis.

44. Management of hypertrophic obstructive cardiomyopathy may include:


A. Propranolol and verapamil.
B. Aortic valve replacement.
C. Dual-chamber sequential pacing.
D. Combined septal myectomy and mitral valve plication.
Answer: ACD

DISCUSSION: The majority of patients with hypertrophic obstructive cardiomyopathy are treated medically with beta-
blockers such as propranolol and calcium channel blockers such as verapamil. Patients whose symptoms do not respond to
medical therapy are treated surgically with a transaortic septal myectomy. Recent reports indicate that simple plication of
the anterior leaflet of the mitral valve performed in addition to the septal myectomy further opens the left ventricular
outflow tract by eliminating systolic anterior motion of the mitral valve. Aortic valve replacement is not an appropriate
treatment for hypertrophic obstructive cardiomyopathy. Some patients who are poor surgical candidates may experience
relief of symptoms and left ventricular outflow gradients with dual-chamber permanent pacing. Appropriate pre-excitation
of the ventricular septum can prompt the septum to move away from the left ventricular wall during systole and open the
outflow tract.

45. Which of the following statements about supravalvular aortic stenosis are true?
A. Surgical repair is indicated only when the systolic gradient exceeds 75 mm. Hg.
B. Simple excision of the supravalvular membrane results in satisfactory relief of the stenosis in most patients.
C. The diffuse form of supravalvular aortic stenosis may cause obstruction to branches of the aortic arch.
D. Reoperation after repair of discrete supravalvular aortic stenosis is rare unless abnormalities of the valve itself also exist.
Answer: CD

DISCUSSION: Supravalvular aortic stenosis is a progressive disease and should be repaired surgically if symptoms are
present or the systolic gradient exceeds 50 mm. Hg. In addition to excision of the supravalvular membrane, a patch of
dacron or pericardium must be placed across the area of narrowing and down into at least one of the sinuses of Valsalva.
Reoperation is rare after this procedure unless associated aortic valve disease is also present. In the diffuse form of the
disease the thickening of the aortic wall commonly results in significant luminal narrowing of the ascending aorta and its
major branches.

46. Each year the approximate number of Americans who die from complications of coronary artery disease is:
A. 100,000.
B. 250,000.
C. 500,000.
D. 1,000,000.
E. Over 2,000,000.
Answer: C

DISCUSSION: It is estimated that approximately 7,000,000 Americans currently have symptomatic coronary artery disease.
Of these some 1,500,000 experience myocardial infarction annually and approximately 500,000 die each year from
complications.

47. Which of the following arteries is most likely to be involved with serious atherosclerosis?
A. The right coronary artery.
B. The left coronary artery.
C. The anterior descending coronary artery.
D. The circumflex coronary artery.
Answer: C

DISCUSSION: In order of frequency, the anterior descending coronary artery is the most commonly involved with
atherosclerosis, followed by the right coronary, the circumflex, and the left main coronary artery.

48. Which of the following statements about collaterals in the normal coronary circulation is true?
A. There is a rich and quite effective collateral circulation in the coronary arterial bed.
B. The coronary arterial bed has minimal effective collaterals.
C. The coronary arterial bed is an absolute example of anatomic end-arteries.
Answer: B

DISCUSSION: The collateral circulation to the heart is relatively poor. In the human heart there are few natural collaterals
of sufficient diameter for delivery of a significant quantity of blood. Most of the collaterals are approximately 200 mm. or
smaller, and channels of this size cannot conduct significant quantities of blood for cardiac requirements. There is no
absolute example of anatomic end-arteries in humans. While the magnitude of arterial collateral circulation varies
considerably, all organs have some collaterals.

49. If blood entering the normal arterial circulation of the heart is 100% saturated with oxygen, oxygen saturation of blood
in the coronary sinus can be expected to be approximately:
A. 75%.
B. 60%.
C. 50%.
D. 35%.
E. Less than 20%.
Answer: D

DISCUSSION: The heart has an unusually high rate of oxygen utilization and consumes approximately two thirds of the
oxygen in the arterial blood. The oxygen saturation of the blood in the coronary sinus is usually about 30% to 35% and
varies with the magnitude of cardiac disease. The body as a whole extracts approximately 25% of the oxygen it receives,
thus emphasizing the great need of the heart for oxygen at rest as well as at exercise.

50. Coronary bypass procedures have been demonstrated to:


A. Reduce the incidence of myocardial infarction.
B. Significantly relieves angina symptoms.
C. Statistically improve the life span.
D. Improve the ejection fraction of the left ventricle in many patients in whom it is significantly depressed preoperatively.
Answer: ABCD

DISCUSSION: In a variety of studies, coronary bypass procedures have been demonstrated to reduce the incidence of
subsequent myocardial infarction as well as to relieve significantly anginal symptoms. They also improve the life span of
most patients as well as the ejection fraction of the left ventricle in those in whom it was depressed preoperatively.

51. The following patients are best treated with coronary artery bypass grafting (CABG):
A. A 60-year-old man with class II angina, 75% proximal right coronary artery lesion, and normal ventricular function.
B. A 60-year-old man with unstable angina, three-vessel disease, and an ejection fraction of 35%.
C. A 60-year-old nondiabetic man with class III angina symptoms and focal discrete lesions in the mid-right coronary artery
and mid-left circumflex artery.
D. A 60-year-old man with diabetes, class IV angina, 75% proximal left anterior descending and 75% proximal right
coronary artery obstruction, and left ventricular ejection fraction of 60%.
Answer: BD
DISCUSSION: CABG has been shown to prolong patient survival compared with medical therapy in those patients with left
main occlusive disease and those with three-vessel or two-vessel disease with proximal left anterior descending involvement
in association with class III or greater anginal symptoms, impaired ejection fraction, or easily inducible ischemia with
exercise. Although percutaneous transluminal coronary angioplasty (PTCA) appears to be comparable to CABG in
nondiabetic patients, patients with diabetes appear to have a significant survival advantage when CABG is used. Similarly,
patients with more extensive coronary artery disease are better treated with CABG than with PTCA.

52. Sternal wound infections that spread to the mediastinum are associated with a mortality rate of:
A. 60%.
B. 30%.
C. 25%.
D. Less than 15%.
Answer: D

DISCUSSION: Although the mortality rate following sternal infections with mediastinitis formerly was high, it is now
greatly reduced. In most series, mediastinitis is cured in more than 90% of patients who are treated aggressively with
débridement and placement of muscle flaps or omentum into the mediastinum to speed wound healing.

53. Perioperative myocardial infarction occurs following coronary bypass procedures in approximately:
A. 15%.
B. 10%.
C. 7%.
D. Less than 5%.
Answer: D

DISCUSSION: Following improvements in myocardial protection and coronary grafting techniques, perioperative
myocardial infarction now occurs in less that 2% to 4% of patients in most series.

54. Following acute myocardial infarction, ventricular septal defects occur in:
A. 20%.
B. 10%.
C. 15%
D. 2% or less.
Answer: D

DISCUSSION: Postmortem studies indicate that 8% to 10% of fatal cases of myocardial infarction are due to rupture of the
heart. In addition, infarction of the interventricular septum with subsequent formation of a ventricular septal defect occurs in
1% to 2% of patients with acute myocardial infarction. The usual interval between the acute infarction and septal rupture—4
to 12 days—correlates with the histologic finding of maximal cardiac muscle degeneration.

55. Which of the following clinical characteristics is/are associated with a higher mortality after emergency CABG for
failed PTCA?
A. Multivessel disease.
B. Rescue atherectomy.
C. Cardiogenic shock prior to CABG.
D. Previous bypass surgery.
E. All of the above.
Answer: ACD

56. Which statement(s) about operative mortality and perioperative incidence of myocardial infarction for elective CABG
(X) versus emergency CABG following failed PTCA (Y) is/are accurate?
A. The operative mortality is higher for Y but the incidence of perioperative myocardial infarction is unchanged between X
and Y.
B. The operative mortality is unchanged between X and Y but the perioperative incidence of myocardial infarction is higher
in Y.
C. The operative mortality and perioperative incidence is higher in X than in Y.
D. The operative mortality and perioperative incidence of myocardial infarction are no different for X and for Y.
Answer: C

57. Which of the following statements about patients treated by placement of an internal mammary artery (IMA) bypass
graft at primary CABG is/are correct?
A. The risk for morbidity and mortality from reoperative coronary bypass grafting is increased.
B. Left ventricular function is better preserved at the time of reoperation.
C. The risk of sternal wound complications is greatly increased if the contralateral IMA is harvested at the time of
reoperation.
D. A light clamp should be applied to the IMA pedicle to limit cardiac warming during cardioplegic arrest at the time of
reoperation.
E. A functional study demonstrating a large portion of myocardium at risk should be obtained before reoperation.
Answer: BDE

DISCUSSION: Patients who have an intact IMA graft should have severe anginal symptoms and a significant portion of
myocardium at risk before reoperative coronary bypass grafting is considered. A functional study may better define the
proportion of myocardium at risk for ischemia and infarction. Patients with an intact IMA graft are less likely to require
reoperation, but if stenosis distal to the IMA and disease in other vein grafts have progressed or if a large portion of
myocardium is at risk, reoperation is recommended. The presence of an intact IMA is not a contraindication to reoperation;
in fact, this population of patients have better-preserved ventricular function and are, perhaps, better candidates for
reoperation. Placement of an IMA graft at the time of the first operation was critically important, neutralizing the adverse
effects of elevated serum cholesterol, hypertension, and smoking on reoperation-free survival. The risk of damaging an
intact IMA graft is 3% to 5%. A lateral projection of the IMA at cardiac catheterization will define its course, particularly in
relation to the sternum, to allow more careful sternal re-entry. The IMA should be minimally dissected and a light clamp
applied during cardioplegic arrest to limit cardiac warming and improve myocardial protection. The IMA may be detached
and recycled if needed. The use during reoperation of the contralateral IMA does not increase the risk of sternal wound
complications.

58. Considering the results of coronary reoperation in comparison to primary CABG, choose the incorrect statement:
A. Operative morbidity and mortality are increased over those for primary CABG.
B. Mortality most often stems from cardiac causes after reoperation.
C. Survival of patients after hospital discharge following coronary reoperation is nearly equivalent to survival after primary
CABG.
D. Compared to primary CABG, return of anginal symptoms is delayed after reoperative CABG.
E. Myocardial protection and the risk of myocardial infarction in reoperation are complicated by increased noncoronary
collaterals, patent atherosclerotic saphenous vein grafts, and more diffuse coronary atherosclerosis.
Answer: D

DISCUSSION: The mortality and morbidity after reoperative CABG are approximately two to three times that of primary
CABG. In contrast to primary CABG, where the majority of deaths are a result of failure of other organ systems, 75% to
85% of deaths after reoperative CABG are due to cardiac causes. The increased risk of reoperation results from more
advanced native vessel disease, a longer cross-clamp time, a longer cross-clamp time per graft, a longer time to initiate
cardiopulmonary bypass, and increased blood loss. The increased frequency of pulmonary complications, myocardial
infarction, neurologic injury, and death, stems from the technical factors of reoperation and the characteristics of the patient
population. Technical factors include difficulty in finding targets secondary to pericardial reaction and more diffusely
diseased vessels, the risks of injuring the heart or great vessels on sternal re-entry, increased blood loss and risk of requiring
transfusion, less available conduit for bypass, and greater difficulty in providing optimal myocardial protection.
Characteristics of this patient population that increase risks include advanced age and diminished ventricular function.
While survival after reoperation is nearly equivalent to that after primary CABG, angina symptoms return at twice the
frequency in the first year after operation (47% versus 20%) then return at a similar annual rate (2% to 3%).

59. Which statements are correct comparisons of gated equilibrium and initial-transit radionuclide measurements of left
ventricular function?
A. Gated equilibrium techniques provide more accurate measurements of ejection fraction than initial-transit methods.
B. Left ventricular imaging time for a gated equilibrium study is at least 10 times that of an initial-transit study.
C. Both techniques require the same radiopharmaceuticals.
D. Both techniques require a bolus injection.
Answer: B

DISCUSSION: Both techniques are equally accurate for measuring left ventricular ejection fraction. The left ventricular
imaging time for gated equilibrium studies is at least 10 times that of initial-transit radionuclide angiocardiography. Initial-
transit techniques use data from fewer than 10 heartbeats, whereas equilibrium studies require more than 100 heartbeats to
acquire data with similar information density. The initial-transit study can be performed with any radioactive substance, but
the gated equilibrium technique requires a radiopharmaceutical that remains within the blood pool for imaging. The initial-
transit radionuclide study requires a bolus injection, but an equilibrium study can be acquired up to several hours after
injection and must be acquired while the tracer is at equilibrium.

60. The radionuclide variable that contains the greatest amount of prognostic information in patients with coronary artery
disease is:
A. Exercise ejection fraction.
B. Change in regional wall motion from rest to exercise.
C. Maximal cardiac output during exercise.
D. Change in heart rate during exercise.
Answer: A

DISCUSSION: The exercise ejection fraction is the single most important radionuclide variable relating to subsequent
cardiac death or myocardial infarction, and this single variable contains 80% of the prognostic information in the test.

61. Which of the following statements about left ventricular aneurysm is/are correct?
A. Ventricular aneurysms are commonly associated with systemic arterial embolization.
B. Absent collateral circulation in an area of myocardium supplied by an acutely occluded artery favors aneurysm
formation.
C. Posterobasal aneurysms are more common than those located in the anteroapical region.
D. Aneurysm repair can improve associated cardiac valve dysfunction.
E. Persistent ST segment elevation after acute myocardial infarction suggests aneurysm formation.
Answer: BDE

DISCUSSION: The mural thrombus frequently present on the endocardial surface of an aneurysm is usually adherent and
rarely embolizes. Collateral circulation, when present, often prevents transmural necrosis following arterial occlusion. Since
the left anterior descending coronary artery is the vessel most commonly occluded in patients with ventricular aneurysms,
most of the aneurysms are anteroapical. Improvements in ventricular contour and reduction in ventricular volume
accompany aneurysm repair. Although persistent elevation of ST segments following myocardial infarction is very
suggestive of aneurysm formation, the diagnosis should be confirmed by more definitive tests.

62. Which of the following factors does/do not increase early mortality associated with repair of left ventricular
aneurysm?
A. Class IV cardiac status.
B. Size of aneurysm.
C. Presence of left main coronary disease.
D. Emergent operation.
E. Location of aneurysm.
Answer: BE

DISCUSSION: Class IV cardiac status and emergent operation both imply extensive myocardial damage and in most
reported series are associated with increased operative mortality. Similarly, the presence of significant stenosis of the left
main coronary artery increases the operative mortality of virtually all cardiac procedures. On the other hand, neither the size
of the aneurysm nor its location affect early operative mortality, despite the fact that posterior aneurysms are technically
more difficult to repair and are much less common.

63. The most effective medical therapy in ameliorating the symptoms of Kawasaki's disease and preventing the
development of giant coronary artery aneurysms is administration of:
A. Antibiotics.
B. Antiviral agents.
C. Aspirin.
D. Gamma globulin.
E. Glucocorticoids.
Answer: D

DISCUSSION: Kawasaki's disease is a multisystemic disorder of unknown cause and is the leading cause of acquired heart
disease in children in both Japan and the United States. Although many clinical aspects of Kawasaki's disease suggest an
infectious agent, the search for a single agent has been unsuccessful; neither antibacterials nor antivirals have a role in the
therapy of Kawasaki's disease. The goal of initial therapy of Kawasaki's disease is the reduction of inflammation, including
coronary and myocardial inflammation. After the diagnosis of Kawasaki's disease is secured, patients are treated with
intravenous gamma globulin and large doses of aspirin. Gamma globulin, 2 gm. per kg., is administered as a single infusion
over 12 hours. Treatment with intravenous immune globulin has been shown to decrease the duration of fever, to decrease
the prevalence of cardiovascular complications, and to prevent the progression to giant coronary aneurysms. High-dose
aspirin therapy contributes to the resolution of the acute manifestations of Kawasaki's disease. When Kawasaki's disease is
diagnosed, children are given a regimen of aspirin, 100 mg. per kg. per day, which is continued until defervescence.
Thereafter, they are maintained on small doses of aspirin, 3 to 5 mg. per kg. per day, for 8 weeks. The goal of aspirin
therapy is amelioration of symptoms and prevention of the thrombotic and embolic complications of Kawasaki's disease.
Aspirin does not decrease the risk of the development of coronary aneurysms. There is no role for glucocorticoids in the
treatment of Kawasaki's disease.

64. Indications for surgical intervention in Kawasaki's disease include which of the following?
A. The presence of multiple coronary artery aneurysms.
B. Myocardial infarction and severe left ventricular dysfunction.
C. The presence of a 5 mm. aneurysm in the right coronary artery.
D. Progressive stenosis in the left anterior descending coronary artery.
E. None of the above.
Answer: D

DISCUSSION: The indications for surgical treatment of Kawasaki's disease include: (1) progressively stenotic coronary
lesions demonstrated on coronary arteriography, with no distal coronary aneurysms with stenosis; (2) localized aneurysm
with significant stenosis in the left main coronary artery; (3) significant stenosis in two coronary arteries; (4) presence of
collateral vessels arising from a coronary artery with a proximal aneurysm; (5) progressive stenosis in the left anterior
descending coronary artery; and (6) presence of a left ventricular aneurysm. Advanced thrombosis of coronary aneurysms
causing critical stenoses in multiple coronary arteries is the most common indication for surgical intervention.

65. Which of the following statements about the pathophysiology of Ebstein's anomaly is/are true?
A. The tricuspid valve is usually insufficient.
B. Typically there is a left-to-right shunt across the ASD.
C. The redundant anterior leaflet of the tricuspid valve may cause obstruction of the right ventricular outflow tract.
D. Pulmonary hypertension is a common late complication.
E. High pulmonary vascular resistance in neonates exacerbates tricuspid regurgitation and cyanosis.
Answer: ACE

DISCUSSION: Ebstein's anomaly is characterized by downward displacement of the tricuspid valve into the right
ventricular cavity. The anterior leaflet is large and “sail-like,” while the other two leaflets are rudimentary. Although the
tricuspid valve occasionally may be stenotic, it is usually regurgitant. The tricuspid regurgitation and functional right
ventricular outflow tract obstruction caused by the large anterior leaflet lead to right-to-left shunting across the ASD.
Systemic venous hypertension is often present, but pulmonary hypertension almost never occurs with this malformation.
Finally, neonates that present with Ebstein's anomaly are markedly cyanotic, owing to their high pulmonary vascular
resistance. This causes a functional pulmonary atresia, which increases right-to-left shunting across the ASD.

66. In the surgical treatment of Ebstein's anomaly, which of the following is/are true?
A. In neonates, the tricuspid valve orifice may be oversewn and a systemic-pulmonary shunt created to provide pulmonary
blood flow.
B. Techniques in repair of the tricuspid valve do not utilize plication of the atrialized right ventricle.
C. Closure of the ASD alone is adequate repair of the malformation.
D. If tricuspid valve replacement is performed, the valve should be sutured above the coronary sinus to avoid injury to the
conduction system.
E. Currently, mechanical prostheses are recommended for tricuspid valve replacement because the durability of
bioprosthetic valves in the tricuspid position is so poor.
Answer: AD

DISCUSSION: In a recent report on the surgical treatment of Ebstein's anomaly in neonates, Starnes described a technique
consisting of oversewing the tricuspid valve, atrial septectomy, and placement of a systemic-pulmonary shunt. These
patients are then later staged to a modified Fontan procedure when they outgrow their shunts. Repair of the ASD alone was
performed early in the treatment of Ebstein's anomaly and was associated with high mortality rates. It is not considered an
adequate repair. Most techniques in tricuspid valve repair for Ebstein's malformation utilize plication of the atrialized right
ventricle in addition to excision of redundant atrial tissue. If tricuspid valve replacement is necessary, current approaches
utilize bioprosthetic valves because of their excellent durability in the tricuspid position. Placement of the valve ring above
the coronary sinus has been associated with a lower rate of postoperative heart block.

67. Which of the following congenital lesions of the coronary circulation causes a cardiac murmur that is similar to the
murmur produced by a PDA?
A. Origin of the left coronary artery from the pulmonary artery.
B. Origin of the right coronary artery from the pulmonary artery.
C. Coronary artery fistula.
D. Membranous obstruction of the ostium of the left main coronary artery.
Answer: C

DISCUSSION: The major clinical finding with a coronary artery fistula is a continuous murmur over the site of the
abnormal communication. This murmur may closely resemble that of PDA.

68. The congenital coronary lesion most likely to cause death in infancy is:
A. Coronary artery fistula.
B. Origin of the left coronary artery from the pulmonary artery.
C. Origin of the right coronary artery from the pulmonary artery.
D. Congenital coronary aneurysm.
Answer: B

DISCUSSION: The prognosis for most patients with origin of the left coronary artery from the pulmonary artery is poor. It
has been estimated that 95% of patients with this anomaly die within the first year of life unless surgical therapy is
undertaken. Patients whose right coronary artery originates from the pulmonary artery are usually asymptomatic. Patients
with coronary fistulas occasionally suffer congestive heart failure early. Congenital aneurysms of the coronary arteries are
most often asymptomatic until complications occur, usually later in life.

69. The congenital coronary lesion associated with minimal or absent clinical manifestations and nearly normal life
expectancy is:
A. Congenital origin of both coronary arteries from the pulmonary artery.
B. Congenital coronary artery fistula.
C. Membranous obstruction of the ostium of the left main coronary artery.
D. Congenital origin of the right coronary artery from the pulmonary artery.
Answer: D

DISCUSSION: Clinical manifestations of congenital origin of the right coronary artery from the pulmonary artery are
usually minimal or absent. This malformation is thought to have been associated with death. The oldest reported patient
with this malformation died at age 90 years from unrelated problems.

70. Which of the following is/are indications for aortic valve replacement for aortic stenosis?
A. Syncope.
B. Congestive heart failure.
C. Angina.
D. Transvalvar gradient of 35 mm. Hg without symptoms.
Answer: ABC

DISCUSSION: With progressive narrowing of the aortic valve area from the normal 3 to 4 sq. cm. to 1 sq. cm., patients
become symptomatic. The classic symptoms produced by aortic stenosis are syncope, congestive heart failure, and angina.
Once symptoms occur, life expectancy is limited to 2 to 5 years. Therefore, symptomatic aortic stenosis is an indication for
aortic valve replacement. The risk of death with asymptomatic aortic stenosis is quite low, and aortic valve replacement is
not indicated for asymptomatic patients with a transvalvar gradient less than 50 mm. Hg.

71. Under which of the following circumstances is medical management logical?


A. Moderate aortic insufficiency seen on echocardiography with normal left ventricular end-systolic dimensions.
B. Moderate to severe aortic insufficiency seen on echocardiography with cardiomegaly on chest roentgenography.
C. Moderate aortic insufficiency seen on echocardiography with symptoms of congestive heart failure.
D. Moderate aortic insufficiency with an end-systolic left ventricular dimension of 70 mm. as seen on echocardiography.
Answer: A

DISCUSSION: The left ventricle is usually able to compensate for a long time for the increased volume load imposed by
aortic insufficiency. The natural history of asymptomatic aortic stenosis is excellent; 10-year survival for moderate aortic
insufficiency managed medically is as high as 85% to 95%. Medical management typically consists of diuretics and
afterload reduction; however, once the compensatory mechanisms begin to fail, survival is limited. Half of patients with
signs or symptoms of congestive heart failure die within 2 years. Therefore, evidence of left ventricular dilation by
echocardiography (left ventricular end-systolic dimension greater than 55 mm., cardiomegaly on chest roentgenography) or
symptoms of congestive heart failure are indications for aortic valve replacement.

72. Which of the following may be indications for operation for mitral stenosis?
A. Systemic embolization.
B. Infective endocarditis.
C. Onset of atrial fibrillation.
D. Worsening pulmonary hypertension.
Answer: ABCD

DISCUSSION: Although each is only a relative indication for operation for mitral stenosis, systemic embolization, infective
endocarditis, onset of atrial fibrillation, and worsening pulmonary hypertension may each be an indication for operation for
mitral stenosis. Systemic embolization, infective endocarditis, and onset of atrial fibrillation are each complications of
mitral stenosis that portend a risk of further complication with continued medical therapy. Patients older than 40 years with
mild class II congestive heart failure stand to gain symptomatically from operation for significant mitral stenosis and do not
run excessive risk of multiple reoperative procedures.

73. Which of the following is/are not true?


A. Operation improves survival in patients with severe, symptomatic mitral valve disease.
B. Left ventricular dilatation with class I or class II heart failure is an indication for operation with mitral regurgitation.
C. Tricuspid regurgitation is most commonly caused by abnormalities of the leaflets themselves.
D. Mitral valve replacement requires resection of the mitral valve leaflets and chordae.
Answer: CD

DISCUSSION: Relative to medical therapy alone, surgical therapy has been shown to improve survival in patients with
severe, symptomatic mitral valve disease. In mitral regurgitation, left ventricular dilatation is an indication for surgical
intervention regardless of failure symptoms. The most common cause of tricuspid regurgitation is tricuspid annular
dilatation without abnormalities of the leaflets themselves. Mitral valve replacement with preservation of both leaflets or at
least the posterior leaflet is well described and is probably advisable for most patients to preserve left ventricular function
and reduce the probability of ventricular-annular separation.

74. Which of the following generally are not symptoms of tricuspid valve disease?
A. Pulmonary edema.
B. Hepatic failure.
C. Anasarca.
D. Hoarseness.
Answer: AD

DISCUSSION: Hepatic failure and anasarca are indeed common symptoms of severe, long-standing tricuspid valve disease
with increased venous pressure. Pulmonary edema is a consequence of left-sided heart disease and does not result from a
tricuspid lesion. Similarly, hoarseness is most common after mitral valve disease with left atrial enlargement and is rarely
due to tricuspid valve disease alone.

75. Which of the following are relative indications for mitral valve replacement, as opposed to mitral valve repair?
A. Extensive leaflet calcification.
B. Mitral regurgitation.
C. Chordal rupture of the anterior mitral leaflet.
D. Significant annular dilatation.
Answer: A

DISCUSSION: Extensive mitral valve calcification is a relative indication for mitral valve replacement. Mitral regurgitation
or significant annular dilatation may, however, be amenable to mitral valve repair. Chordal rupture of the anterior leaflet is
generally reparable using chordal transposition or polytetrafluoroethylene (PTFE) chordae.

76. Which of the following are not true?


A. Tricuspid regurgitation due to annular dilatation alone generally does not require valve replacement.
B. Mitral valve replacement with either a bioprosthesis or a mechanical valve requires warfarin anticoagulation.
C. Tricuspid valve replacement is generally an indication for using a tissue valve.
D. Chronic renal failure is a relative indication for tissue valves.
Answer: B

DISCUSSION: Tricuspid regurgitation due to annular dilatation alone generally can be treated with tricuspid annuloplasty
or with correction of associated mitral valve disease. Mitral valve replacement with a mechanical valve does require
warfarin anticoagulation; however, mitral valve replacement with a bioprosthesis may be managed with aspirin alone.
Tricuspid valve replacement is an indication for using a tissue valve because of the significant incidence of valve
thrombosis when a mechanical valve is in the tricuspid position. Chronic renal failure is a relative indication for tissue
valves because valve calcification is rare and because anticoagulation of patients on dialysis carries high risks of morbidity
and mortality.

77. Which of the following are relative indications for mechanical, as opposed to tissue, valve replacement?
A. Patient younger than 30 years.
B. Young female patient who desires children.
C. An elderly patient.
D. Tricuspid valve replacement.
Answer: A

DISCUSSION: Age younger than 30 years is a relative indication for mechanical valves because of an increased incidence
of calcification of tissue valves in younger persons. A young female who desires children would be a relative
contraindication to mechanical replacement because of the risk of teratogenesis and hemorrhage during pregnancy
secondary to warfarin therapy. Advanced age is a relative indication for biologic valves to avoid complications of
anticoagulation and because the probability of reoperation is low. Tricuspid valve replacement is a relative contraindication
to mechanical valve replacement, owing to the increased incidence of tricuspid valve thrombosis with a mechanical
prosthesis.

78. Which of the following statements are not true?


A. Bioprosthetic valves have a relatively high incidence of hemolysis.
B. Bioprosthetic valves have a lower incidence of postoperative prosthetic valve endocarditis.
C. Mechanical valves develop structural failure after an average of 7 to 10 years.
D. Mortality attributable to warfarin therapy approaches 5% per patient-year.
Answer: ABCD

DISCUSSION: Bioprosthetic valves have a relatively low incidence of hemolysis. Bioprosthetic and mechanical valves do
not differ significantly in the associated incidences of postoperative prosthetic valve endocarditis. Bioprosthetic valves
develop structural failure after an average of 7 to 10 years, whereas mechanical valves have a life span of well beyond 10
years. The mortality attributable to warfarin therapy approaches 1% per patient-year.

79. Which of the following are not generally associated with mitral stenosis without regurgitation?
A. Pulmonary hypertension.
B. Pulmonary edema.
C. Left ventricular dilatation.
D. An opening snap after the second heart sound.
Answer: ABD

DISCUSSION: Pure mitral stenosis without regurgitation may be associated with pulmonary hypertension, pulmonary
edema, and an opening snap after the second heart sound. Left ventricular dilatation would be rare in pure mitral stenosis
and generally occurs with volume or pressure overload of the left ventricle, as with mitral regurgitation.

80. The most common location of accessory pathways in patients with the Wolff-Parkinson-White syndrome is the:
A. Left free wall.
B. Right free wall.
C. Posterior septum.
D. Anterior septum.
Answer: A

DISCUSSION: All major published series of the Wolff-Parkinson-White syndrome indicate that the majority of all
accessory pathways appear in the left free wall space. In one series, approximately 60% of all accessory pathways occur in
the left free wall space. In Ebstein's anomaly, pathways are usually located in the posterior septum and/or right free wall
spaces. If these patients are excluded, approximately 70% of pathways occur in the left free wall space.

81. The anatomic electrophysiologic basis of AV node re-entry tachycardia is dual AV node conduction pathways. AV
node re-entry tachycardia is most likely to occur with which of the following electrophysiologic aberrations?
A. Proximal antegrade block in the slow conduction pathway.
B. Proximal retrograde block in the slow conduction pathway.
C. Proximal antegrade block in the fast conduction pathway.
D. Proximal retrograde block in the fast conduction pathway.
Answer: C

DISCUSSION: A retrograde conduction block in either the slow or fast pathway would be likely to prevent a re-entrant
circuit from developing. A proximal antegrade block in the slow conduction pathway is extremely unusual because of the
short refractory period of the slow conduction pathway. The most common conduction block that occurs in patients with
dual AV node physiology is a proximal antegrade conduction block in the fast pathway because of its longer refractory
period. This antegrade block in the fast conduction pathway allows AV conduction to occur via the slow pathway and to
return in retrograde fashion up the fast pathway to establish the re-entrant circuit responsible for AV node re-entry
tachycardia.

82. Match the four surgical procedures that have been developed for the treatment of atrial fibrillation with the major
detrimental sequela(e) of atrial fibrillation that each corrects.
A. His bundle ablation.
B. Left atrial isolation procedure.
C. Corridor procedure.
D. Maze procedure.
1. Patient's sensation of irregular heart rhythm.
2. Hemodynamic compromise because of loss of AV synchrony.
3. Increased vulnerability to thromboembolism.
Answer: A-1. B-1,2. C-1. D1,2,3

DISCUSSION: The surgical procedure most commonly employed for the treatment of atrial fibrillation is catheter ablation
of the His bundle. The International Catheter Ablation Registry reveals that more than 60% of patients who undergo elective
catheter ablation of the bundle of His do so for the treatment of atrial fibrillation. His bundle ablation is an isolation
procedure, in that it confines the atrial fibrillation to the atria and protects the ventricles from the unpleasant sensation of an
irregular heartbeat. Because the atria continue to fibrillate there is no restoration of AV synchrony, and therefore there is no
improvement in cardiac hemodynamics. Moreover, the continuing fibrillation of the left atrium means that postoperatively
the patient is still at the same risk for thromboembolism. Thus, His bundle ablation corrects only one of the three
detrimental sequelae of atrial fibrillation, namely the arrhythmia problem.
The left atrial isolation procedure confines atrial fibrillation to the left atrium, allowing the sinus node to drive the
remainder of the heart in a normal sinus rhythm. Thus, it alleviates the unpleasant sensation of an irregular heartbeat. In
addition, because AV synchrony is re-established between the right atrium and right ventricle, right-sided cardiac output is
restored to normal. This means that normal cardiac output is delivered through the lungs to the left side of the heart. In the
presence of a normal left ventricle the left-sided cardiac output is also normal, despite the fact that left-sided AV synchrony
is not present; however, because the left atrium is allowed to fibrillate, the vulnerability to thromboembolism remains
unchanged postoperatively.
The corridor procedure allows the sinus node to drive the heart in normal sinus rhythm, but because of the total isolation of
the sinoatrial and AV nodes from the remainder of the atria, the atria may continue to fibrillate. Even if they do not, in effect
they are isolated from their respective ventricles so that AV synchrony is lost on both sides of the heart. As a result, the
corridor procedure alleviates the sensation of arrhythmia but does not restore normal hemodynamics, nor does it decrease
vulnerability to thromboembolism. The maze procedure ablates the re-entrant circuits responsible for atrial fibrillation and
restores the normal sinus rhythm. Thus, it alleviates the sensation of arrhythmia, restores normal hemodynamics, and
alleviates the vulnerability to thromboembolism.

83. All of the following statements about nonischemic ventricular tachyarrhythmias are true except:
A. They usually occur in the right ventricle.
B. They are usually associated with a left bundle branch block pattern during the tachycardia.
C. They are usually more refractory to medical therapy than ischemic ventricular tachyarrhythmias.
D. They usually occur as a result of automaticity rather than re-entry.
Answer: D

DISCUSSION: Nonischemic ventricular tachyarrhythmias usually occur in the right ventricle, and as a result the ECG
shows a left bundle branch block–type pattern during ventricular tachycardia. These arrhythmias are notoriously refractory
to medical therapy and they occur almost exclusively on a re-entrant basis.

84. Which of the following statements about left atrial myxoma are true?
A. This lesion, by site and histology, is the most common primary cardiac tumor.
B. It is best diagnosed by cardiac catheterization and angiography.
C. The symptom complex can mimic collagen vascular disease.
D. It has an intracavitary growth pattern.
E. It has a multicentric origin in the chamber wall.
Answer: ACD

DISCUSSION: Eighty per cent of primary cardiac tumors are benign, and half of these benign tumors are myxomas.
Seventy-five per cent of myxomas arise in the left atrium in the region of the fossa ovalis. Echocardiography is the
technique of choice in the evaluation of intracardiac tumors, and findings suggestive of myxoma occur in 95% of patients
examined. Invasive procedures, with the attendant risk of tumor embolization, are not warranted. Owing to an autoimmune
phenomenon, left atrial myxomas can present with systemic constitutional symptoms of fever, malaise, weight loss,
polymyositis, and blood dyscrasias that mimic collagen vascular disease. Of surgical significance is the fact that most
myxomas rarely extend deeper than the endocardium but grow as polypoid, intracavitary masses. Attachment by a vascular
stalk thus allows tumor mobility, predisposing to embolization and interference with mitral valve competence and causing
characteristic echocardiographic findings.
85. Which of the following statements about malignant cardiac tumors are true?
A. Sarcomas are the most frequent primary malignancy.
B. Metastatic tumors are usually asymptomatic.
C. Adjuvant chemotherapy and irradiation are efficacious in prolonging survival.
D. Intra-atrial extension of renal neoplasms is a contraindication for surgical resection.
E. Constrictive physiology is an indication for operation.
Answer: AB

DISCUSSION: Twenty per cent of primary cardiac tumors are some variant of sarcoma. Precise histologic classification is
not imperative, as all have a similar clinical picture with rapid systemic dissemination and aggressive local invasion. In
contrast, metastatic tumors cause symptoms in only 10% of patients. Unfortunately, most primary and secondary cardiac
malignancies infrequently respond to systemic chemotherapy or mediastinal irradiation. Surgical treatment is most
successful with renal tumors extending into the right atrium. Significant 5-year survival can be achieved with concomitant
nephrectomy and intra-atrial resection of the tumor thrombus. Relief of tamponade is worthwhile; however, extensive
decortication provides little help.

86. Disadvantages of temporary pacing through skin electrodes applied to the anterior chest wall include all of the
following except:
A. Skin burns.
B. Painful chest wall muscle contractions.
C. Ventricular fibrillation.
D. Inability to pace.
Answer: C

DISCUSSION: In 1952 Zoll first described successful pacing through external metal electrodes applied to the anterior chest
wall. Clinical experience with this technique has shown that it is both feasible and lifesaving for temporary pacing;
however, disadvantages of the external pacing technique include skin burns when too little electrode jelly is applied, painful
chest wall muscle contractions, and inability to pace in thick-chested or emphysematous patients. Ventricular fibrillation
induced by external temporary cardiac pacing is exceedingly rare.

87. In adults the most common cause of acquired complete heart block is:
A. Ischemic heart disease.
B. Sclerodegenerative disease.
C. Traumatic injury.
D. Cardiomegaly.
Answer: B

DISCUSSION: Before permanent pacemakers were available, 50% of patients with complete heart block died within 1 year.
The most common cause of acquired complete heart block in adults is sclerodegenerative disease of the cardiac skeleton and
AV conduction system. Other less common causes of complete heart block include ischemic heart disease, cardiomyopathic
processes, Chagas' disease, and traumatic injury.

88. The most common indication for permanent pacing is:


A. Complete heart block.
B. Second-degree AV block.
C. Chronic bifascicular block.
D. Sick sinus syndrome.
Answer: D

DISCUSSION: Patients with sinus node dysfunction may develop a number of arrhythmias, such as inappropriate sinus
bradycardia, chronotropic incompetence, sinoatrial exit block, and sinus arrest. This group of rhythm disorders typically
occurs in older patients with or without underlying heart disease and is collectively known as the “sick sinus syndrome.” In
addition, many patients with sick sinus syndrome have associated atrial tachyarrhythmias, particularly atrial fibrillation.
This association of atrial tachyarrhythmias in patients with the sick sinus syndrome is called the tachycardia-bradycardia (or
tachy-brady) syndrome. The most common indication for permanent pacing occurs in patients with the sick sinus syndrome.
89. Decreasing pacemaker electrode tip size results in:
A. Lower pacing thresholds.
B. Improved electrogram sensing.
C. Decreased battery life.
D. Less patient discomfort.
Answer: A

DISCUSSION: Decreasing pacemaker electrode tip size results in lower pacing thresholds, both at the time of implant and
subsequently, because of higher current density. However, better sensing function is directly related to electrode area and is
adversely affected by small electrode size. Therefore, a compromise between pacing and sensing efficiency is required.
Typical electrode surface areas for pacing are between 8 and 10 sq. mm.

90. At the time of ventricular pacemaker implantation, lead resistance is determined at a voltage near that of the
pacemaker's output. The calculated resistance at 5 volts should range from:
A. 10 to 100 ohms.
B. 125 to 250 ohms.
C. 300 to 800 ohms.
D. 1000 to 1500 ohms.
Answer: C

DISCUSSION: At the time of pacemaker implantation, in addition to measuring pulse amplitude (voltage and current) and
pulse width, resistance is also determined. As described by Ohm's law, resistance is calculated by dividing voltage by
current. Resistance calculations are made at a voltage near that of the pacemaker's output. The calculated resistance at 5
volts should range from 300 to 800 ohms. An unsatisfactorily low resistance is unsatisfactory because current is wasted and
battery life is shortened. Conversely, excessively high resistance (more than 800 ohms) increases battery life but decreases
the current delivered to the heart for pacing.

91. A ventricular inhibited-demand pacemaker using the Intersociety Commission for Heart Disease Resources (ICHD)
code is designated as:
A. DVI.
B. VVI.
C. VOO.
D. VDD.
Answer: B

DISCUSSION: A ventricular inhibited-demand pacemaker using the ICHD code is designated as VVI. As the ICHD code
states, the pacemaker senses intrinsic ventricular activity and is inhibited when this activity exceeds the standby or escape
rate of the pacemaker. When the intrinsic ventricular rate falls below the escape rate of the pulse generator, the pacemaker
begins to function at its programmed rate.

92. In rate-modulated pacing, the pacing rate is determined by a physiologic parameter other than atrial rate and is
measured by a special sensor in the pacemaker or pacing lead. The most commonly used physiologic parameter in rate-
modulated pacemakers is:
A. QT interval.
B. Venous blood temperature.
C. Mixed venous oxygen saturation.
D. Body motion.
Answer: D

DISCUSSION: During exertion, the required increase in cardiac output is obtained mostly by the increase in paced heart
rate, although increased venous filling and maintenance of AV synchrony are also important contributors. The most
commonly used physiologic parameters in rate-modulated pacemakers at the present time are body motion and minute
ventilation. Other parameters that are less commonly used or under evaluation include QT interval, venous blood
temperature, mixed venous oxygen saturation, contractility, stroke volume, venous pH, and the paced depolarization
gradient.
93. The most common pacing mode used in patients with symptomatic bradycardia and an underlying sinus rhythm is:
A. AAI.
B. DVI.
C. DDD.
D. VVI.
Answer: C

DISCUSSION: “Universal,” or DDD, pacing has been shown to have many benefits over other pacing modalities, including
the ability to track the intrinsic sinus rate, pace the atrium and ventricle, maintain atrioventricular synchrony, and avoid the
pacemaker syndrome. Recognition of these benefits has steadily increased the use of DDD pacemakers in the last decade,
and at the present time DDD is the most common pacing mode.

94. A transvenous pacemaker generator pocket should be placed on the patient's nondominant side over the:
A. Anteromedial chest wall.
B. Anterolateral chest wall.
C. Inferomedial chest wall.
D. Inferolateral chest wall.
Answer: A

DISCUSSION: Bipolar impulse generators can be placed either in the subcutaneous tissue or beneath the muscle. Migration
of the impulse generator most commonly occurs in infraclavicular pacemakers pockets. Migration tends to follow the
curvature of the chest wall, and the impulse generator tends to migrate laterally. This can be prevented by creating an
anteromedial pocket large enough to contain the impulse generator and lead. In susceptible persons the impulse generator
can be further secured to the chest wall to prevent migration.

95. Pacemaker-mediated tachycardia is caused by:


A. Pacemaker induction of atrial fibrillation.
B. Sensing of retrograde atrial activation.
C. Inappropriate ventricular sensing.
D. Lead fracture.
Answer: B

DISCUSSION: Pacemaker-mediated tachycardia occurs in the setting of intact ventriculoatrial conduction. Typically,
premature ventricular contractions may be conducted retrogradely through the AV conduction system and cause retrograde
activation of the atrium. If this retrograde atrial activation occurs after completion of the programmed pacemaker
ventriculoatrial refractory period, the atrial event is sensed by the DDD pacemaker and evokes a paced ventricular event that
may cause further VA conduction. If each ventricularly paced event results in atrial activation sensed by the pacemaker,
pacemaker-mediated tachycardia will be generated.

96. Which cardiovascular pharmacologic agents are safe to use during routine abdominal surgery in a 75-year-old woman
with documented hypertension and mild coronary artery disease?
A. Nifedipine.
B. Atenolol.
C. Hydralazine.
D. Captopril.
E. Reserpine.
Answer: ABD

DISCUSSION: Nifedipine is tolerated fairly well by elderly patients and is safe to use in the perioperative period with close
hemodynamic monitoring. Atenolol is a safe beta-blocker to use during the perioperative period and provides protection
from cardiac rhythm disturbances and rebound hypertension. Hydralazine, if given without a beta-blocker, often elicits
reflex tachycardia, which limits its usefulness. Captopril is a safe agent that does not appear to interfere with the normal
cardiovascular response to anesthesia, and abrupt withdrawal of this agent may result in severe hypertension and should be
avoided. Reserpine is an adrenergic inhibitor that may depress cardiac output and result in hypotension, so its use in the
perioperative setting is limited.

97. Which inotropic drugs are safe for use in elderly patients with mild congestive heart failure in the postoperative
period?
A. Digitalis compounds.
B. Dopamine.
C. Amrinone.
D. Melrinone.
E. Dobutamine.
Answer: BCDE

DISCUSSION: Dopamine and dobutamine stimulate cardiac beta-receptors and are very useful in providing inotropic
support for patients in the postoperative period. Melrinone and amrinone are phosphodiesterase inhibitors that have strong
inotropic effects while causing arterial and venous dilation. Melrinone and amrinone are useful in patients with low cardiac
output, especially in the setting of congestive heart failure. Digitalis compounds can be troublesome in the postoperative
period owing to the toxic effects of these agents. Furthermore, perioperative hypoxia and hypokalemia increase myocardial
susceptibility to digitalis-induced ventricular arrhythmias.

98. Which anticoagulation treatment plan(s) is/are appropriate for a 72-year-old man with a mechanical heart valve in
place who takes Coumadin (warfarin) and now requires elective left colon resection?
A. Discontinuation of Coumadin therapy on the day of the operation.
B. Discontinuation of Coumadin therapy on the day of the operation with replacement of clotting factors with fresh frozen
plasma (FFP) before the start of the surgical procedure.
C. Discontinuation of Coumadin therapy 5 days before operation with no further anticoagulation therapy before surgery.
D. Discontinuation of Coumadin therapy 5 days before operation with the institution of intravenous heparin as the
prothrombin time normalizes.
E. Discontinuation of Coumadin therapy 2 days before operation followed by large doses of aspirin.
Answer: D

DISCUSSION: Many patients who require anticoagulation with Coumadin for underlying cardiac disease need to undergo
routine general surgical procedures. The current recommendations for patients who have been on long-term Coumadin
therapy is to discontinue Coumadin 5 days before an operative procedure. As the patient's prothrombin time normalizes
intravenous heparin should be started. The patient should be maintained on a therapeutic dose of heparin with an activated
partial thromboplastin time (aPTT) of at least 60 seconds. Heparin should then be withheld approximately 4 to 6 hours
before the surgical procedure. The operation is then performed in a “heparin window,” where the level of anticoagulation
can easily be titrated or totally reversed with protamine if necessary.

99. Which of the following treatment plans is appropriate for a 68-year-old patient with moderate to severe congestive
heart failure following a major abdominal surgical procedure?
A. Aggressive use of inotropic support with epinephrine.
B. Aggressive diuresis with furosemide and inotropic support with dopamine.
C. Afterload reduction with nitroprusside and inotropic support with dopamine.
D. Close perioperative monitoring and inotropic support with melrinone.
E. Intravenous digitalis with diuresis using furosemide as needed.
Answer: CD

DISCUSSION: Treatment of congestive heart failure using epinephrine alone is contraindicated owing to the profound
vasoconstrictive properties of epinephrine, which only exacerbate the heart failure. Diuresis with furosemide and inotropic
support with dopamine is acceptable for patients with mild congestive heart failure; however, in the postoperative period
pharmacologic diuresis can lead to profound hypovolemia requiring continuous invasive hemodynamic monitoring. The
ideal choice for the postoperative management of patients with severe congestive heart failure is afterload reduction using
nitroprusside and inotropic support with dopamine. This helps to stimulate the failing heart while decreasing the afterload
pressure against which the heart must pump. Melrinone is a useful phosphodiesterase inhibitor, which has been shown to be
useful in the treatment of mild to moderate congestive heart failure. Digitalis along with a diuretic in the postoperative
period can be troublesome owing to the potential toxicity of digitalis while the patient has ongoing fluid and electrolyte
shifts.

100. Which of the following steps is/are appropriate for a 65-year-old woman who develops atrial fibrillation with
associated mild hypotension and rapid ventricular response following partial gastric resection?
A. Correction of electrolytes and blood chemistries.
B. Evaluation for possible myocardial infarction.
C. Treatment with intravenous lidocaine.
D. Attempt to limit the ventricular response with digitalis.
E. Immediate cardioversion.
Answer: ABD

DISCUSSION: When a patient develops postoperative atrial fibrillation following an extracardiac procedure, correction of
the patient's blood chemistries and electrolytes is essential. The patient must also undergo evaluation for a possible
myocardial infarction as the cause of the atrial dysrhythmia. The first rule in treatment is to slow the ventricular response
and attempt to limit hemodynamic instability. Digitalis is effective in slowing down the ventricular response and thus
improving the hemodynamic status of the patient. Lidocaine has little use in controlling atrial dysrhythmias but is very
effective in decreasing ventricular ectopy. Immediate cardioversion is rarely indicated for new-onset atrial fibrillation. Only
after correction of all underlying metabolic and electrolyte defects as well as an attempt at medical conversion and
ventricular rate control is cardioversion recommended.

101. The damaging effects of cardiopulmonary bypass are, to a large degree, due to activation of the humoral amplification
system. The humoral amplification system includes which of the following?
A. The coagulation cascade.
B. The fibrinolytic cascade.
C. Complement activation.
D. A and C.
E. A, B, and C.
Answer: E

DISCUSSION: Cardiopulmonary bypass stimulates a whole-body inflammatory response, and the concentrations of several
inflammatory mediators (e.g., complement fraction C5a) have been associated with subsystem dysfunction following
cardiopulmonary bypass. This inflammatory response is complex and has several arms, including the coagulation,
fibrinolytic, and complement systems. Simply blocking one pathway is unlikely to completely prevent bypass-induced
injury.

102. Adequate flow during cardiopulmonary bypass is best indicated by:


A. Systemic blood pressure of 90/50 mm. Hg.
B. Arterial PO 2 of 230 mm. Hg.
C. Mixed venous hemoglobin saturation of 78%.
D. Central venous pressure of 1 mm. Hg.
E. Plasma lactate value of 6 mg. per dl.
Answer: C

DISCUSSION: The purpose of cardiopulmonary bypass is to provide adequate circulation of blood to sustain aerobic
metabolism. Oxygen consumption during bypass depends on bypass flow until a critical flow is attained. With higher flows
there is no further increase in oxygen consumption (i.e., oxygen consumption becomes flow independent), and the mixed
venous hemoglobin saturation increases. A mixed venous hemoglobin saturation of 78% indicates that bypass flow is above
the critical level and that flow is adequate. The other variables do not ensure adequate bypass flow.

103. Which of the following does not typically occur during the first few minutes of cardiopulmonary bypass?
A. Interstitial fluid increases.
B. Blood flow becomes nonpulsatile.
C. Platelet count decreases.
D. Complement is activated.
E. Systemic vascular resistance falls.
Answer: A

DISCUSSION: Several events occur during the first few minutes of bypass. The tubing and oxygenator surfaces are coated
by serum proteins that in turn activate platelets. This reduces the platelet count. The roller pump produces nonpulsatile flow,
which is different from the usual pulsatile cardiac flow. Serum complement is activated by exposure of blood to the
nonphysiologic surfaces of the pump-oxygenator, and systemic vascular resistance falls. Interstitial fluid accumulates during
bypass; however, this occurs later during bypass.

104. Which of the following are physiologic benefits of intra-aortic balloon counterpulsation to the ischemic ventricle?
A. Preload reduction.
B. Afterload reduction.
C. Coronary blood flow enhancement.
D. Decreased ventricular end-diastolic pressure.
Answer: BCD

DISCUSSION: In general, preload relates to the volume of blood or fluid presented to the left ventricle. Although wall
tension does increase with increased volume, Starling properties are called forth for added efficiency. Preload is controlled
by volume status as well as capacity of the venous system. The effects of balloon counterpulsation on cardiac preload are
minimal and secondary to other changes. As the balloon collapses in the aorta, the absence of the balloon volume, or
“abyss,” creates a decrease in ventricular afterload. In effect this decreases ventricular wall tension, reducing myocardial
oxygen consumption significantly. During counterpulsation, the intra-aortic balloon inflates in diastole, elevating coronary
perfusion pressure significantly. Maximal coronary artery perfusion occurs in this part of the cardiac cycle. Thus, ischemic
ventricles benefit especially from balloon pumping. The balloon pump does not directly decrease the left ventricular end-
diastolic pressure. However, in ventricles failing from ischemia the combination of afterload reduction and improved
coronary blood flow usually augments cardiac function, producing decreased cardiac filling pressure or left ventricular end-
diastolic pressure.

105. Which of the following are the major indications for instituting intra-aortic balloon pumping?
A. Medically refractory angina.
B. Acute papillary muscle rupture.
C. Left main coronary artery lesion.
D. Ventricular failure after cardiac surgery.
E. PTCA failure.
Answer: ABDE

DISCUSSION: Medically refractory angina is one of the major indications for implementing the intra-aortic balloon pump.
When intravenous nitroglycerin becomes ineffective at relieving chest pain or results in early hypotension, the balloon
pump should be used in preparation for surgical revascularization or percutaneous angioplasty. By reducing left ventricular
afterload, the pump reduces regurgitation into the left atrium. Thus, balloon counterpulsation is very helpful for treating
patients with acute mitral insufficiency secondary to papillary muscle rupture. Patients should undergo valve surgical
procedures emergently, as balloon pump support is only temporizing. The mere presence of a left main coronary lesion is
not an indication for use of the balloon pump. In former years such pumps were inserted prophylactically before induction
of anesthesia for coronary bypass surgery. Newer anesthetic techniques have largely obviated this; however, in the presence
of a left main lesion and medically refractory angina the balloon pump should be used. The balloon pump is quite effective
in helping to wean patients who have postcardiotomy left ventricular failure from cardiopulmonary bypass. This is one of
the major uses of this device. The Emory University group was the first to expound on the efficacy of the balloon pump in
stabilizing patients following percutaneous angioplasty failure. With the pump inserted, most patients can be transported to
the operating room safely, many being stable enough to harvest an internal mammary graft instead of having to defer to the
more accessible but less preferable saphenous vein.

106. Which of the following are the most frequent complications of intra-aortic balloon counterpulsation?
A. Stroke.
B. Limb ischemia.
C. Arrhythmias.
D. Aortic thrombosis.
Answer: B

DISCUSSION: Stroke rarely occurs secondary to intra-aortic balloon pump use. The balloon must be positioned well below
the aortic arch vessels and never proximal to the left subclavian artery origin. Strokes have been reported from emboli being
thrown retrograde from the balloon; however, this is very rare. Limb ischemia is one of the most frequent complications of
balloon pumping. The combination of iliofemoral atherosclerosis and catheter luminal obstruction may impede distal flow.
This may require catheter removal to re-establish flow. In 2% to 10% of patients, arterial reconstruction is necessary to
repair balloon-related complications. Smaller catheters have helped prevent limb ischemia. Arrhythmias in general are not
complications of balloon pumping. In fact, arrhythmias related to ischemia may be controlled by the balloon pump. Aortic
thrombosis can occur very rarely with pump use. A more frequent occurrence is distal embolization with limb ischemia.
Patients should be heparinized while the balloon catheter is in place. Following cardiac surgery heparinization is usually
delayed for 12 to 24 hours.

107. Permanent artificial hearts are being developed that are electrically powered. Wireless techniques are used to transmit
the electrical energy across the body wall using the principle of:
A. Infrared sensor.
B. Inductive coupling.
C. Thermionic coupling.
D. High-pressure liquid chromatography (HPLC).
E. Infrared spectroscopy.
Answer: B

DISCUSSION: Electrical energy can be transmitted across the body wall by tunnelling an electric wire; however,
experience has shown that infection, starting at the skin line and burrowing deeper into the body, will occur over time. This
infection can be delayed, but not stopped, by the use of a velour covering on the wire. Wireless electrical energy
transmission was first used in clinical surgery by W.W.L. Glenn in the 1950s for powering pacemakers. The remarkable
advances in electronics have facilitated this technique; however, the placement of the two coils parallel to one another (with
the skin between), as opposed to interlocking as in an industrial transformer, reduces the efficiency of transmission from
approximately 99% to 70%.

108. The following statements about the pneumatic artificial heart is/are correct:
A. It can support the circulation for over 1 year.
B. It may be complicated by infection or thromboembolism.
C. When further developed, it will be an ideal permanent heart substitute.
D. It is an ideal “bridge” for transplantation.
E. It can be implanted using techniques similar to those used for heart transplantation.
Answer: ABE

DISCUSSION: The pneumatic artificial heart was developed as a permanent cardiac substitute, but the need for two tubes to
pass through the chest wall and the bulky power unit have relegated the pneumatic heart to short-term use as a bridge to
transplantation. The heart is implanted using similar techniques as a heart transplantation. The presence of foreign surfaces
and crevices make the device prone to thromboembolism and infection. Most surgeons feel that left ventricular support or
biventricular assist pumps represent a better option for those patients with end-stage congestive heart failure who require
use of a bridge device.

109. A cyanotic infant has echocardiographic evidence of a univentricular heart (UVH). The following is/are true:

a. The most common form of the disorder is a double-inlet right ventricle


b. To be classified as a ventricle, the chamber must receive at least half of an inlet valve
c. This infant is a good candidate for a Blalock-Taussig shunt
d. Optimal correction of UVH diverts all vena caval blood flow into the pulmonary arteries (Fontan procedure)
e. In the absence of pulmonic stenosis, UVH usually presents as congestive heart failure
Answer: b, c, d, e
Univentricular heart is defined by the connection of the atria to only one ventricular chamber, usually the left as a double
inlet left ventricle. A chamber must receive at least half of an inlet valve to be considered a ventricle. The presentation of
UVH depends on the pulmonary blood flow; if pulmonary stenosis is present there is increased cyanosis and the infant is a
candidate for a Blalock-Taussig shunt. In the absence of pulmonic stenosis, pulmonary flow is excessive and the
presentation is congestive heart failure. Optimal correction of UVH diverts all vena caval flow into the pulmonary arteries
as the Fontan procedure.

110. A 9-year-old boy with hypertension has no palpable femoral pulses. Coarctation of the aorta is suspected. The
following is/are true:

a. The most common associated abnormality is a bicuspid aortic valve


b. Chest radiograph is likely to show rib notching
c. The etiology is felt to be secondary to an inflammatory aortitis
d. In infancy, coarctation may present with a pink upper body and cyanotic lower body
e. “Paradoxical hypertension” seen after operative repair indicates residual stenosis from incomplete correction
Answer: a, b, d

Coarctation of the aorta occurs just distal to the origin of the left subclavian artery and results from contraction of ectopic
tissue from the ductus arteriosus. The most common associated abnormality is a bicuspid aortic valve. Extensive collateral
development involves the mammary and intercostal arteries producing rib notching on the chest radiograph. In infancy, flow
to the lower body is from the ductus arteriosus before it closes, producing differential cyanosis. The “paradoxical
hypertension” seen postoperatively is thought to relate to sympathetic nerve stimulation and does not reflect an incomplete
repair.

111. A 48-year-old woman with episodic syncope has echocardiographic evidence of a mass in the left atrium. The
following is/are true statement(s):

a. Transseptal puncture should be used for definitive diagnosis


b. If this is a primary cardiac tumor it is most likely to be malignant
c. If this is a myxoma attached to the atrial septum, the adjacent septum should be removed with it
d. In infancy, the most common cardiac tumor is a rhabdomyosarcoma
e. The most common primary malignant tumor of the heart is angiosarcoma
Answer: c, e

Primary cardiac tumors commonly arise in the left atrium and can present with dyspnea, syncope, congestive failure and
systemic embolism. Transseptal puncture should not be used for diagnosis because of the risk of embolism. Most primary
cardiac tumors are benign by a 3:1 ratio. The most common malignant tumor is the angiosarcoma. Myxoma is the most
common benign tumor, but it can recur and the adjacent atrial septum should be resected with it. In infancy, the most
common cardiac tumor is a rhabdomyoma.

112. A 2-month-old boy who appeared normal at birth has become cyanotic and is found to have a systolic ejection
murmur over the pulmonic area and a boot-shaped heart on chest radiograph. The following is/are true:

a. Echocardiography alone is sufficient to confirm the diagnosis of Tetralogy of Fallot


b. Cyanotic spells may be appropriately treated by propranolol
c. The Blalock-Taussig shunt connects the right ventricle to the pulmonary artery
d. Increasing cyanotic spells is the most common indication for operation
e. Operative repair of right ventricular outflow obstruction is never extended across the pulmonic valve since
intolerable pulmonary insufficiency would result
Answer: a, b, d

In this typical scenario for Tetralogy of Fallot, echocardiography can confirm the diagnosis with no need for cardiac
catheterization. Cyanotic spells are treated by supplemental oxygen, sedation with morphine and a beta blocker such as
propranolol. For palliative increase in pulmonary blood flow, the Blalock-Taussig shunt is utilized connecting the
subclavian artery to the pulmonary artery. Increasing cyanosis and cyanotic spells are the most common indication for
operative repair. To correct the right ventricular outflow obstruction in Tetralogy, a transannular patch may be required
extending into the pulmonary artery. Fortunately the pulmonary valvar insufficiency that results is well tolerated in the
absence of tricuspid insufficiency or ventricular dysfunction.

113. A 12-year-old boy is found to have an ejection systolic murmur over the aortic region with a precordial thrill and
normal cardiac size on chest radiograph. The following is/are true:

a. A systolic ejection click would signify that the stenosis is supravalvar


b. In the absence of cardiomegaly, cardiac catheterization to measure the pressure gradient is necessary
c. Development of syncope would suggest an intracranial lesion
d. In valvar aortic stenosis a pressure gradient of 80 mmHg is an indication for operative repair regardless of
symptoms
e. In membranous subvalvar aortic stenosis a pressure gradient of 40 mmHg is an indication for operative repair
Answer: d, e

In the patient with findings of aortic stenosis, a systolic ejection click is evidence that the obstruction is valvular. Cardiac
size does not provide an indication of the severity of the stenosis and is frequently normal. The development of angina or
syncope reflects inadequate cardiac output and signifies late-stage disease. A pressure gradient over 75 mmHg is an
indication for operation in valvar aortic stenosis even if the patient is asymptomatic while a lesser gradient of 30 mmHg or
more is considered sufficient for operative correction of membranous subvalvar stenosis.

114. Within 2 hours of birth, a baby girl is obviously cyanotic and chest radiograph shows the heart to appear like “an
egg on its side.” The following is/are true:

a. The most common cause of cyanosis this early is transposition of the great vessels (TGV)
b. If TGV is present, echocardiography will show that the posterior vessel leaving the left ventricle is a pulmonary
artery
c. If TGV is confirmed by echocardiography, cardiac catheterization has little to add
d. The EKG is helpful in making the diagnosis of TGV since it shows reversed dominance of the ventricles
e. To improve mixing of pulmonary and systemic circulations, prostaglandin should be used to increase pulmonary
vascular resistance
Answer: a, b

TGV is the most common cause of cyanosis in the first week of life, and this diagnosis can be confirmed by
echocardiographic demonstration of a posterior pulmonary artery attached to the left ventricle. Cardiac catheterization is
useful to confirm the anatomy, detect other lesions, define the coronary anatomy and improve cardiac mixing by balloon
atrial septostomy. The EKG is not helpful in the diagnosis of TGV since it shows only normal right ventricular dominance.
Prostaglandin improves the mixing of the circulation by opening the ductus arteriosus and reducing pulmonary vascular
resistance.

115. A one-year-old boy thought to have Tetralogy of Fallot is found on cardiac catheterization to have double-outlet
right ventricle (DORV). The follow is/are true:

a. Spontaneous closure of the VSD is rare


b. Location of the VSD has little effect on the degree of cyanosis
c. Double outlet left ventricles do not occur
d. Coincidental aortic stenosis with DORV is not compatible with life
e. Doubly committed VSD refers to its relationship to the great vessels
Answer: a, e

In DORV, the location of the VSD affects the direction of flow of oxygenated blood and thus determines the degree of
cyanosis. Fortunately, the VSD rarely closes since that would result in severe decompensation or death. Double outlet left
ventricles occur but are less common than DORV. A number of other anomalies are associated with DORV including both
valvar and subvalvar pulmonary and aortic stenosis. The VSD may be directed to either or both great vessels (doubly
committed) or remote from them (noncommitted).
116. A 5-year-old girl is found on routine examination to have a pulmonic flow murmur, fixed splitting of P2 and a right
ventricular lift. The following is/are true:

a. Cardiac catheterization is indicated if the chest film shows cardiomegaly


b. Radiology report of “scimitar syndrome” findings on the chest film would indicate need for an arteriogram
c. If the catheterization report is “ostium secondum defect,” at least one pulmonary vein drains anomalously
d. Measured pulmonary vascular resistance of 14 Woods units/m2 with an ASD mandates early repair
e. An ASD with Qp/Qs of 1.8 can be observed until symptoms occur
Answer: b

The findings suggest an atrial septal defect (ASD) that can be confirmed by 2D echocardiography eliminating the need for
cardiac catheterization. The ostium secondum type defect is most commonly found, but it is the sinus venosus type that is
associated with anomalous pulmonary venous drainage. In the scimitar syndrome, the anomalous pulmonary vein can be
seen on a chest radiograph and, since these are associated with a hypoplastic lung that is supplied by an anomalous systemic
artery from the aorta, an arteriogram is appropriate. An ASD with a significant left-to-right shunt as demonstrated by a
Qp/Qs ratio in excess of 1.5 should be repaired. When the pulmonary vascular resistance is elevated above 10–12 Woods
units/m2 the patient is not a candidate for repair due to fixed pulmonary hypertension.

117. A 2-month-old infant has EKG evidence of myocardial ischemia and echocardiography suggests anomalous origin
of the left coronary artery from the pulmonary artery. The following is/are true:

a. Ischemia is due to perfusion of the myocardium with inadequately oxygenated blood


b. Selective coronary angiography should not be attempted because of the risk of myocardial infarction
c. Conservative treatment is preferred to allow the coronary artery to grow to a size that will allow bypass
construction
d. If the infant deteriorates, ligation of the coronary at its origin is a viable option
e. The severity of the abnormality insures that it will always be detected in the first year of life
Answer: d

Anomalous origin of the left coronary artery from the pulmonary artery results in reverse flow in the coronary into the low-
pressure system as a steal from the coronary circulation. If collaterals from the right coronary develop to allow adequate
myocardial perfusion, the disorder is frequently not diagnosed until later in life when a murmur is heard. Selective coronary
arteriography is appropriate to define the anatomy and operative repair is undertaken promptly. Ligation of the anomalous
coronary can be lifesaving but leaves the child dependent on a single vessel and coronary bypass is preferred.

118. A 2-month-old boy is found to be in congestive heart failure manifested by tachypnea, tachycardia and diaphoresis
with poor weight gain. The physical findings suggest a ventricular septal defect (VSD). Management should include:

a. Pulmonary artery banding


b. Urgent closure if a VSD is found on echocardiography
c. Medical treatment only with digitalis and diuretics
d. If a VSD is found, repair is unlikely to be possible because of elevated pulmonary vascular resistance
e. If a restrictive VSD is found, spontaneous closure is a possibility and operative repair should be delayed
Answer: c, e

Large VSDs present at 6–8 weeks of age when the normally elevated pulmonary vascular resistance falls, allowing an
increase in the left-to-right shunt. Since roughly half of all VSDs undergo spontaneous closure, particularly with restrictive
defects, the initial management is medical. The diagnosis is confirmed by echocardiography and cardiac catheterization.
Advanced pulmonary vascular changes do not occur usually until 2 years of age and banding is only rarely indicated for
palliation for multiple complex muscular VSDs.

119. A 1-year-old girl with dyspnea and poor feeding is found to be in congestive heart failure. Echocardiography shows
an atrio-ventricular septal defect (AVSD). The following is/are true:

a. The second heart sound will show fixed splitting


b. Despite diagnostic echocardiography, cardiac catheterization is indicated to assess pulmonary artery resistance
c. Pulmonary artery banding is indicated to limit pulmonary flow and allow the child to grow
d. AVSD is classified according to the morphology of the anterior leaflet of the common A-V valve
e. Operative repair is best performed after 2 years of age
Answer: a, b, d

AVSD is a defect of endocardial cushion development which produces morphologic abnormalities of both AV valves and
both atrial and ventricular septa. It is usually classified according to the morphology of the anterior leaflet of the AV valve.
The pulmonary vascular resistance remains elevated in infancy delaying diagnosis and producing fixed splitting of the
second heart sound. Cardiac catheterization is indicated to assess pulmonary vascular resistance, but pulmonary artery
banding is no longer performed to protect the pulmonary bed. Instead, operative repair is made, preferably before the age of
6 months.

120. The child in the previous question undergoes cardiac catheterization confirming a VSD with Qp/Qs ratio of 2.0 and
right ventricular systolic pressure half of systemic pressure. The following is/are true:

a. If aortic insufficiency is detected, the defect is likely to be subpulmonic in location


b. Finding aortic stenosis in addition to the VSD would be highly unlikely
c. The cath data indicate a restrictive type of VSD
d. If pulmonary vascular resistance falls with tolazoline administration, it is safe to close the VSD
e. Operative closure of VSDs is possible without ventriculotomy
Answer: a, c, d, e

The finding of aortic insufficiency in a patient with VSD suggests prolapse of the aortic valve due to a subpulmonic or
supracristal defect. Associated aortic stenosis, mitral stenosis and coarctation are common with VSDs. The finding of a
moderate left-to-right shunt and a right ventricular pressure well below systemic levels indicates a restrictive VSD. If
elevated pulmonary vascular resistance is found, the ability to respond to a vasodilator like tolazoline indicates that the
resistance is not fixed and operative repair is possible. Operative repair of VSDs is frequently possible via atriotomy or
through the pulmonary artery.

121. A premature infant in respiratory distress is found to have a continuous “machinery” murmur over the precordium.
The following is/are true:

a. The most likely diagnosis is coarctation of the aorta


b. If large pulmonary arteries are noted, a patent ductus is likely
c. To discriminate between a and b, prostaglandin administration can be used which will constrict the patent ductus
arteriosus
d. If a ductus if found, operative repair should be delayed until the respiratory symptoms improve to reduce mortality
rates
e. Normal ductus closure depends on increased oxygen saturation in the pulmonary artery
Answer: b, e

A continuous “machinery” murmur is characteristic of patent ductus arteriosus typically seen in the premature infant.
Normal closure of the ductus is prompted by a fall in pulmonary vascular resistance that increases the left-to right shunt and
oxygen levels from the aorta. Indomethacin can cause ductus closure by cyclooxygenase inhibition which decreases
endogenous prostaglandins. Prostaglandin infusion would keep the ductus open. Operative closure can be done safely in
even the smallest neonates and usually promptly relieves the respiratory distress.

122. A neonate in congestive heart failure has echocardiographic evidence of a single truncal vessel from which the
pulmonary arteries arise, a VSD and truncal valvar stenosis. The following is/are true:

a. Natural history of this anomaly allows only 20% one-year survival


b. The most likely configuration of the truncal valve is bicuspid
c. Location of the pulmonary arteries minimizes the risk of pulmonary vascular obstructive disease (Eisenmengers)
d. Repair of the lesion requires an extracardiac conduit
e. Optimal timing of operative repair is at 6–12 months
Answer: a, d
The defect described is truncus arteriosus which carries an 80% one year mortality rate uncorrected. The truncal valve is
most commonly tricuspid (65%) or quadricuspid (25%); least likely bicuspid (9%). The large left-to-right shunt makes these
patients particularly likely to develop pulmonary vascular obstruction (Eisenmenger’s syndrome). Operative repair requires
detachment of the pulmonary arteries which are reconnected to the right ventricle by an extracardiac conduit, and the
optimal timing for repair is within the first 6 months of life.

123. A neonate in respiratory distress has echocardiographic evidence of hypoplastic left heart syndrome (HLHS). The
following is/are true:

a. Initial management should include prostaglandin infusion


b. Ventilatory adjustment should maintain PaCO2 at approximately 40 mmHg
c. Survival depends on sustained patency of the ductus arteriosus
d. Cardiac transplantation for HLHS requires inclusion of the donor aortic arch
e. Reconstruction for HLHS converts the pulmonary artery into the main outlet for a functional single ventricle
(Norwood)
Answer: a, b, c, d, e

The neonate with HLHS has a severely underdeveloped left ventricular and aortic arch and is dependent on patency of the
ductus which is facilitated by prostaglandin infusion. Ventilator adjustment to reduce supplemental oxygen and maintain
PCO2 of 40 mmHg avoids excessive pulmonary flow. The options for treatment include cardiac transplantation which
requires a donor aortic arch and reconstruction by the Norwood procedure which converts the pulmonary artery into the
main outlet for a functional single ventricle.

124. A 52-year-old man with known aortic stenosis develops angina pectoris and has a single episode of syncope. The
following is/are true:

a. Onset of angina indicates concomitant coronary artery disease independent of valvular lesion
b. Percutaneous aortic balloon valvuloplasty should be considered since it has generally favorable results
c. Patient is not an operative candidate since heart failure has not occurred
d. A measured transvalvular pressure gradiant > 50 mmHg would be an operative indication
Answer: d

The ventricular hypertrophy which accompanies aortic stenosis increases oxygen demand while mechanical forces increase
resistance to perfusion, resulting in ischemia. Only one half of these patients with angina have coronary artery disease.
Percutaneous balloon valvuloplasty of the aortic valve has high complication and recurrence rates. Any such patient with
symptoms has an indication for operations as would the patient with a transvalvular gradiant > 50 mmHg.

125. The patient in the previous question with AI progresses to profound heart failure requiring medical management.
The following is/are true:

a. Perperal vasdilators are contraindicated


b. The inta-aortic balloon pump can be used to improve cardiac output
c. Furosemide and nitroglycerin would be appropriate
d. Valve replacement is necessary
Answer: c, d

Peripheral vasodilators are key to the treatment of AI favoring peripheral blood flow. The intraaortic pump is
contraindicated because diastolic augmentation worsens aortic regurgitation. Both furosemide and nitroglycerin would be of
value to treat the failure, but the most effective treatment requires replacement of the valve.

126. A 42-year-old woman has noted progressive exercise intolerance and fatigability. Examination discloses an
opening snap in the mitral area suggestive of mitral stenosis. The following is/are true:

a. Critical mitral stenosis is defined as an orifice area reduced to 2 cm2


b. With a fixed mitral orifice, the change from sinus rhythm to atrial fibrillation has little effect on cardiac output
c. Mural thrombi and thromboembolism are directly related to the presence of atrial fibrillation
d. Depressed cardiac output is usually due to depressed myocardial contractility
Answer: c

Normal adults have a 4–6 cm2 mitral orifice and reduction to 2 cm2 is mild stenosis while reduction to 1 cm2 is considered
critical mitral stenosis. Even with a fixed orifice, the onset of atrial fibrillation reduces cardiac output by 20%. Mural
thrombi and thromboembolism are directly related to the presence of atrial fibrillation. Mitral stenosis spares ventricular
function, and the loss of cardiac output is from decreased preload.

127. Concerning valvular heart disease, the following is/are true:

a. Mitral stenosis is the most common lesion


b. Of all cardiac valves, the aortic is the most anterior
c. Stenosis is the most common lesion of the aortic valve
d. Rheumatic heart disease is the most common cause of valve dysfunction
Answer: c, d

Aortic valvular stenosis is the most common type of valvular lesion followed by mitral stenosis. Anatomically, the pulmonic
valve is the most anterior of the cardiac valves. Rheumatic heart disease is the most common cause of valve dysfunction and
the most common cause of multivalvular disease.

128. A 47-year-old male with fatigue and cardiac failure has a high-pitched, decrescendo diastolic murmur along the left
sternal border and an apical diastolci rumble. His blood pressure is 148/45 mmHg. The following is/are true:

a. Chest radiograph will show cor bovinum


b. The apical murmur is due to the Gallavardin phenomenon
c. A carotid shudder would be expected
d. Abdominal exam will show a pulsatile liver
Answer: a

This patient with aortic insufficiency has a volume loading strain on the heart which produces cor bovinum as dramatic
enlargement. The apical murmur produced by turbulence with mitral forward flow mimics mitral stenosis and is called an
Austin-Glint murmur. A carotid shudder occurs with aortic stenosis and a pulsatile liver is typical of tricuspid insufficiency.

129. Concerning the adaptation to cardiac valvular dysfunction, the following is/are true:

a. Severe heart failure is more likely from acute than chronic valvular dysfunction
b. Valvular dysfunction produces both volume and pressure afterload stress on the heart
c. Early cardiac dilation from valve dysfunction shifts the Frank-Starling curve to depress cardiac output
d. The LaPlace law predicts that wall stress decreases with increasing ventricular radius
Answer: a, b

Valvular dysfunction produces both volume and pressure overload representing afterload stress on the heart. Although
cardiac reserves allow for gradual adaptation to chronic valvular dysfunction, acute dysfunction is less well tolerated and
more likely to result in severe heart failure. The increase in diastolic filling which initially dilates the heart, shifts the Frank-
Starling curve to improve ejection and cardiac output. The LaPlace law predicts that wall stress increases with increasing
ventricular radius but is inversely related to wall thickness.

130. A 31-year-old male drug abuser presents with fever, chills and multiple bilateral lung abscesses. Right heart
endocarditis is suspected. The following is/are true:

a. The organisms most likely responsible are gram-negative and fungal


b. The pulmonic valve is most likely to be affected
c. A negative echocardiogram is useful to exclude the diagnosis
d. Valve replacement is necessary if the native valve is excised to treat infection
Answer: a

The typical endocarditis in a drug-abuser involves fungal and gram-negative organisms which infect the tricuspid rather
than the pulmonic valve. An echocardiogram is useful to confirm the presence of vegetations but it may overlook smaller
ones so it cannot be used to exclude the diagnosis. Although valve replacement is usually preferable, the infected tricuspid
valve can be excised without prosthetic replacement.

131. In the initial management of the patient in the previous question with suspected acute MI, the following is/are true:

a. Oxygen and lidocaine should be administered prophylactically


b. If chest pain persists, IV nitroglycerin should be used to limit infarct size
c. Ca-channel blockers are also of value to limit infarct size
d. Morphine IV can be used but has no therapeutic effect
Answer: b

Initial treatment during an early evolving MI should include oxygen, but lidocaine should be used only if arrhythmias occur.
Nitroglycerin IV is of value to limit infarct size but not Ca-channel blockers which have no such benefit. By decreasing pain
and anxiety, morphine IV has a significant therapeutic effect in decreasing myocardial oxygen demand.

132. Concerning the physiology of the coronary circulation, the following is/are true:

a. Under circumstances of increased oxygen demand by the myocardium, O2 extraction from arterial blood can
increase
b. Coronary flow is maximal during systole
c. Adenosine is the most important metabolic regulator of coronary blood flow
d. Sympathetic nerve stimulation constricts coronary arteries despite the need for increased cardiac output
Answer: c, d

Since myocardium maximally extracts O2 from blood at rest, increased demand requires increased delivery. Systolic
pressures compress intramyocardial vessels, so maximal coronary flow is during diastole. Adenosine, a breakdown product
of ATP, is a vasodilator and the most important metabolic regulator of coronary blood flow. Although sympathetic nerves
produce coronary vasoconstriction, the autoregulatory vasodilatory responses to increased myocardial demand overwhelm
that effect.

133. True statement(s) concerning cardiac vascular anatomy include the following:

a. In 80%–85% of cases the posterior descending coronary artery (PDA) arises from the circumflex coronary artery
b. The PDA gives off the AV nodal artery
c. The great cardiac vein ascends along the right coronary artery to empty into the coronary sinus
d. Thebesian veins drain from only left and right ventricles
Answer: b

In 80%–85% of cases the circumflex coronary artery ends with branches to the left ventricle while the PDA originates from
the right coronary in 80%–85% of cases. The PDA gives off the AV nodal artery and its occlusion can result in heart block.
The great cardiac vein ascends along the left anterior descending coronary artery and the Thebesian veins drain all 4
chambers.

134. In the medical management of coronary artery disease, the following is/are true:

a. Nitroglycerin primarily dilates coronary arterioles


b. b-blocking agents act to reduce myocardial O2 demand
c. Ca-channel blocking agents reduce ventricular contractility
d. Ca-channel agents should not be used if there is an element of coronary vasospastic disease
Answer: b, c
Nitroglycerin primarily dilates venous capacitance vessels, but at higher doses can produce coronary and systemic arterial
dilation. b-adrenergic blocking agents reduce myocardial O2 demand by decreasing heart rate and contractility. Ca-channel
blocking agents reduce ventricular contractility, produce vasodilation and may protect myocytes. They are particularly
effective for coronary vasospastic disease.

135. A 67-year-old man with documented acute MI progresses in 24 hours to cardiogenic shock. The following is/are
true:

a. The mortality rate for cardiogenic shock after acute MI is increased more than 10 fold in comparison to no shock
b. Age, ejection fraction, MI size and previous MI serve as predictors of cardiogenic shock
c. Acute loss of more than 20% of myocardium frequently results in cardiogenic shock and death
d. Emergency revascularization is contraindicated for the MI patient in cardiogenic shock
Answer: a, b

Cardiogenic shock is unusual after acute MI but increases the mortality rate from 4% to 65%. All of the risk factors
described plus a history of diabetes mellitus can predict cardiogenic shock. The volume of myocardium lost acutely that is
associated with shock is 40%. Recent studies suggest that emergency coronary bypass can be used within 18 hours of shock
to reduce the mortality rate to 7%.

136. A 52-year-old man with chest pain and tachycardia has ECG evidence of an acute MI. The following is/are true:

a. Thrombolytic therapy should be considered immediately since the benefit is greater the earlier it is given
b. Of the drugs available, recombinant tPA produces better results than SK or APSAC although it is more expensive
c. Thrombolytic therapy requires catheterization for intracoronary administration
d. Addition of heparin and antiplatelet drugs produces no incremental benefit
Answer: a

Thrombolytic therapy for acute MI is of significant value in reducing mortality with benefit related to early administration.
Although rtPA can produce higher coronary patency rates, the results of treatment are no better than with SK or APSAC.
Thrombolytic drugs were initially given intracoronary but can be used effectively when given systemically IV. There is an
added benefit from heparin and antiplatelet drugs to prevent rethrombosis.

137. Following repair of an abdominal aortic aneurysm, a 66-year-old man develops severe chest pain, diaphoresis,
bradycardia and hypotension. The following is/are true:

a. The electrocardiogram is most likely to show a prominent Q in lead 3 if this is an MI


b. If Q wave is present, the infarct is subendocardial rather than transmural
c. Creatine kinase measurement alone is diagnostic of MI
d. Since bradycardia rarely occurs with MI, another diagnosis should be considered
Answer: a

Pain is the most common complaint in patients with myocardial infarction although 20%–25% are asymptomatic. Inferior
MIs involving the right coronary frequently have parasympathetic activity with bradycardia, hypotension and a prominent Q
wave in lead 3. The presence of a Q wave indicates a transmural MI which can be confirmed by measurement of the specific
isoenzyme for cardiac tissue (CK-MB) since creatine kinase can be elevated non-specifically after stroke or operation.

138. A 70-year-old woman with intractable angina pectoris undergoes cardiac catheterization for possible mechanical
intervention. She prefers PTCA to open correction. The following is/are true:

a. A long symmetric lesion in the left main coronary artery would be appropriate for PTCA
b. Multiple obstructive lesions in the same artery would be a contraindication to PTCA
c. A focal lesion in the left anterior descending coronary artery where the vessel is 1 mm in diameter would allow
PTCA
d. Successful PTCA for a simple lesion carries a recurrent stenosis risk of less than 10%
Answer: b
The ideal lesion for PTCA is focal symmetric stenosis in an epicardial vessel. However, it is relatively contraindicated for
significant disease in the left main coronary, for multiple obstructive lesions in the same artery, and for vessels less than 2
mm in diameter. Restenosis rates of 20% to 40% occur within the first 4–6 months after successful dilation for simple
lesions.

139. A 78-year-old patient who is a candidate for CABG is concerned about the risks/benefits of the procedure. The
following is/are true:

a. Operative mortality in patients > 70 years is more than double that of younger patients
b. If the patient is a woman, the risk is higher than it would be for a man
c. A previous CABG procedure increases the complexity and complication rate, but does not alter mortality rate
d. Results are better if there is ischemic cardiomyopathy than if there is hibernating myocardium
Answer: a, b

Operative mortality for patients > 70 years was 8% in the CASS study as compared to 3% in younger patients. For reasons
not entirely clear, the risk of CABG is higher in women than in men. Reoperative procedures carry a higher operative
mortality due to technical difficulties, more advanced disease, and less complete revascularization. Congestive heart failure
is a major determinant of poor surgical outcome, but the results are better when there is viable myocardium (hibernating)
than when there is irreversible ischemic cardiomyopathy.

140. Four days after a transmural MI, a 74-year-old man develops hypotension and congestive heart failure. The
following is/are true:

a. An intra-aortic balloon pump should be used and cardiac catheterization performed


b. If the infarct was posterior, this is most likely due to a ventricular septal defect
c. Pulmonary wedge pressure tracing of prominent V waves without an O2 step-up suggests papillary muscle rupture
d. Operative repair of a post MI VSD should be delayed to allow strengthening of the myocardium to hold sutures
Answer: a, c

Both ventricular septal defect (VSD) and ruptured papillary muscle occur from 3–5 days post-MI and should be managed by
intra-aortic balloon pump, decreasing afterload and cardiac catheterization for diagnosis. A VSD is most likely in an elderly
hypertensive female who has sustained an anterior transmural MI; posterior MIs typically lead to papillary muscle rupture
which is diagnosed by prominent V waves on pulmonary wedge pressure tracing. Survival rate for both of these
complications is improved by early rather than late repair.

141. A 52-year-old woman with chest pain is considered for coronary arteriography on the basis of her risk factors. The
following is/are true statement(s):

a. All patients with typical anginal symptoms should have coronary arteriography
b. Atypical patients with borderline positive stress tests should have arteriography
c. Patients who require valve procedures do not require arteriography
d. Patients in refractory heart failure awaiting cardiac transplantation should have coronary arteriography
Answer: b

Patients with typical angina and ECG changes should have angiography only if they are refractory to medical management
and/or a candidate for revascularization. Patients with atypical signs and symptoms should have angiography to confirm or
exclude the diagnosis. Patients with valve disease and risk of coronary artery disease should have angiography but patients
awaiting cardiac transplantation are not candidates for revascularization and do not require coronary angiography.

142. The patient in the previous question is found to have disease unsuitable for PTCA. Concerning operative
revascularization (CABG) the following is/are true:

a. CABG is more effective than medical treatment for relieving angina and improving physical work capacity
b. In CABG for unstable angina, there is no difference in late outcome between stable and unstable cohorts
c. For CABG, the most common arterial graft is the left internal mammary artery
d. Long term patency is improved when arterial grafts are used but there is no difference in the early mortality rate
Answer: a, b, c

Randomized studies show that CABG is more effective than medical therapy for relieving angina, improving physical work
capacity and improving overall quality of life. When CABG is used for unstable angina, the initial complication and
mortality rates are higher than for stable angina, but the late outcomes are similar. Use of arterial grafts for CABG has
increased with the left internal mammary artery used most commonly; when at least one mammary artery is used, the early
mortality rate is improved.

143. In the workup of a 45-year-old man with suspected coronary artery disease, the following is/are true:

a. Thyroid tests are included to rule out hyperthyroidism


b. Typically positive stress ECG would show elevated ST segments
c. Dipyridamole is a useful adjunct to thallium scanning as it increases coronary perfusion pressure
d. Persisting defects on thallium scan indicate reversible myocardial ischemia
Answer: a

Diagnostic studies for coronary artery disease should detect risk factors such as diabetes mellitus, hyperlipidemia and
hyperthyroidism. The stress ECG typically shows downward sloping ST segment depression. Dipyridamole is a coronary
artery vasodilator that reduces systemic and coronary perfusion pressures. The persisting thallium scan defect reflects
irreversibly scarred myocardium.

144. Following successful thrombolytic treatment of the patient in the previous question, he develops recurrent chest
pain in 24 hours. The following is/are true:

a. Rethrombosis is most likely and thrombolytic therapy alone should be repeated


b. The problem could have been prevented by early elective catheterization and PTCA
c. Patient has an indication for catheterization and PTCA if single vessel disease is found
d. Findings of multivessel disease at catheterization would indicate need for operative bypasses
e. If operative bypass is deemed necessary, there should be a 30-day delay to allow myocardial healing
Answer: c, d

After thrombolytic therapy for acute MI, angina recurs in 30%–35% and is an indication for cardiac catheterization and
mechanical intervention to prevent infarct extension. Prophylactic catheterization, however, has not been found to provide
benefit. If the findings at catheterization show limited disease treatable by PTCA, then it should be performed. But if
multivessel disease or unfavorable anatomy is found, operative bypass should be carried out early since results are best
within 30 days of the MI.

145. A 59-year-old male has undergone successful CABG with 4 grafts constructed but remains in low cardiac output (<
2L/min/m2) postoperatively. The following is/are true:

a. An inotropic drug should be used initially to increase cardiac output


b. If low cardiac output persists despite optimal physiological and pharmacological support, a balloon pump (IABP)
should be inserted
c. Decreased cardiac filling pressures suggest the possibility of cardiac tamponade
d. When IABP is used, the balloon is inflated during diastole
Answer: b, d

Initial efforts to improve cardiac output should include correction of poor oxygenation or acidosis and optimization of
rhythm, preload and afterload before an inotropic agent is used. If low cardiac output persists despite physiological and
pharmacological support, an IABP should be inserted. It improves coronary artery perfusion by counterpulsation during
diastole. Cardiac tamponade is heralded by increased cardiac filling pressures, narrowed pulse pressure and pulsus
paradoxus.
146. A 42-year-old asymptomatic attorney undergoes a routine exercise test which is reported positive for myocardial
ischemia. The following is/are true:

a. This is a rare event since less than 5% of patients with coronary artery disease (CAD) are asymptomatic with
exercise
b. Such a patient could progress to heart failure from ischemic cardiomyopathy
c. Typical angina pectoris is promptly relieved by rest or relaxation
d. Dyspnea on exertion can represent an angina equivalent
Answer: b, c, d

As many as 25% of CAD patients found by exercise testing are asymptomatic. Progressive coronary obstruction in these
patients can produce heart failure from ischemic cardiomyopathy. Typical angina is relieved promptly by rest or relaxation.
Ischemic reductions in ventricular contractility and compliance can produce dyspnea on exertion as an angina equivalent.

147. A 52-year-old man develops postoperative supraventricular tachycardia to a rate of 180/min. and hypotension. The
following is/are true:

a. Since a heart rate of 180/min should be tolerated at his age, the hypotension must have another cause
b. A vagal maneuver that breaks the tachycardia suggests atrial flutter as the etiology
c. Atrial overdrive pacing should be tried for paroxysmal atrial tachycardia (PAT)
d. Verapamil IV should be used for rate control
e. Cardioversion is preferred for patients on digoxin
Answer: c, d

A tachyarrhythmia over 150 beats/min can produce hypotension and myocardial ischemia and demands urgent therapy.
Vagal maneuvers may break PAT but are not usually effective for atrial flutter or fibrillation. Atrial overdrive pacing should
be attempted for PAT or atrial flutter. Verapamil is the most effective approach to rate control for supraventricular
arrhythmias, but cardioversion of patients on digoxin should be undertaken cautiously since they are prone to ventricular
tachycardia.

148. A 77-year-old man with a healed transmural myocardial infarction has a medically refractory ventricular
arrhythmia. The following is/are true:

a. Direct current catheter endocardial ablation has a high likelihood of success.


b. If the arrhythmia is inducible at EP study, there is an indication for operative intervention.
c. A recent MI would be a contraindication to operation
d. Ventricular failure would be a contraindication to operation
e. Monomorphic ventricular tachycardia is least amenable to surgical resection.
Answer: b, c, d

After catheter ablation, only 25% of patients remain free of ventricular arrhythmia off of drug therapy. If the arrhythmia is
inducible at EP study and the patient is an acceptable risk, with a myocardial scar he has an indication for operation. Both
recent MI and ventricular failure are contraindications to operation. Monomorphic ventricular tachycardia is the arrhythmia
most amenable to surgical resection.

149. A 68-year-old man suffers sudden cardiac death (SCD) but is resuscitated and brought to the hospital for evaluation
and treatment. The following is/are true:

a. The most likely cause of SCD is ventricular arrhythmia


b. There is 30–40% chance of recurrent SCD in one year
c. Empiric antiarrhythmic drug therapy improves survival
d. An inducible ventricular tachyarrhythmia at EP study carries a favorable prognosis
e. If a ventricular aneurysm is found with arrhythmia, aneurysm resection is adequate treatment
Answer: a, b
Ventricular arrhythmias cause 75% of SCD, while 25% are due to acute MI. There is a 30–40% chance of recurrent SCD in
one year. An inducible ventricular tachyarrhythmia carries a poor prognosis with < 50% five year survival from SCD unless
it can be abolished. Empiric antiarrhythmic drug therapy does not improve survival. Aneurysmectomy alone is not adequate
therapy for arrhythmias associated with aneurysms since the arrhythmia usually originates in adjacent mechanically stressed
myocardium.

150. The following is/are true concerning the anatomy of the conduction system:

a. There is no special conduction path from the sinoatrial (SA) to the atrioventricular (AV) node
b. The blood supply to the AV node is from the anterior descending coronary artery
c. The only normal muscular connection between atria and ventricles is the bundle of His
d. The aortomitral continuity is the only area where supraventricular accessory pathways cannot occur
e. The sinus node artery arises from the right or circumflex coronary artery
Answer: a, c, d, e

The SA node is located at the junction of the superior vena cava and the right atrial appendage and receives its blood supply
from the right or circumflex coronary artery. There is no special conduction path between SA and AV nodes. The bundle of
His is the only normal atrioventricular muscle connection but abnormal pathways can occur anywhere except the area
known as the aortomitral continuity. The blood supply to the AV node is from the posterior descending coronary artery.

151. The following is/are true concerning the physiology of arrhythmias:

a. A physical or electrical stimulus causes sodium fast channels and calcium slow channels to open
b. During the effective refractory period, only the slow calcium channels are closed
c. Rapid repolarization follows potassium egress from the cell
d. Extracellular hypokalemia increases sodium channel size increasing automaticity
e. Catecholamines increase outward potassium flow from myocytes
Answer: a, c, d

Physical or electrical stimuli cause sodium fast channels and calcium slow channels to open. During the effective refractory
period, both slow calcium channels and fast sodium channels are closed and the myocardium cannot be excited. Then
potassium channels reopen, allowing potassium out, and rapid repolarization occurs. Extracellular hypokalemia increases
transmembrane potassium gradient and sodium channel size increasing automaticity. Catecholamines decrease outward
potassium flow from myocytes enhancing automaticity.

152. A 72-year-old man has had several episodes of ventricular tachycardia and is a candidate for electrophysiological
(EP) study. The following is/are true:

a. The goal of the EP study is either sustained or non-sustained ventricular tachycardia


b. Patients with less than 5 repetitive complexes in response to stimulation are considered noninducible
c. An induced reentry from ventricular stimulation is not necessarily pathological
d. Microreentry arrhythmias are typical after myocardial infarction
e. Macroreentry arrhythmias are typical of myocardial ischemia
Answer: a, b

For arrhythmias of ventricular origin, the EP study goal is either sustained or nonsustained ventricular tachycardia. Patients
with less than five repetitive complexes in response to stimulation are considered noninducible. Ventricular reentry does not
occur in normal myocardium, so all reentrant arrhythmias are pathological. Macroreentry arrhythmias are typical after
myocardial infarction, while microreentry arrhythmias are typical of myocardial ischemia.

153. A 29-year-old male with supraventricular tachyarrhythmias is suspected to have Wolff-Parkinson-White (WPW)
syndrome. The following is/are true:

a. Electrophysiologic studies (EPS) will require catheters in or at the right atrium, His bundle, right ventricle and
coronary sinus
b. Pacing for EPS uses stimuli twice the diastolic threshold
c. The anomalous conducting bundle (Kent) is found in the right free wall if the coronary sinus catheter records the
earliest atrial activity during reciprocating tachycardia
d. If the atrial catheter records the earliest activity during tachycardia, the bundle of Kent is located in the left free
wall
e. If neither left or right bundle-branch block prolong the VA interval, the anomalous bundle is in the septum
Answer: b, e

For supraventricular arrhythmias, EPS requires catheters placed in the right atrium and ventricle, coronary sinus and His
bundle. A programmable stimulator is used for stimuli that are twice the diastolic threshold and 2 msec in duration. When
the coronary sinus catheter records the earliest activity during reciprocating tachycardia, the bundle of Kent is in the left free
wall while it is in the right free wall if the earliest activity is in the atrial catheter. When neither left or right bundle-branch
block prolong the VA interval, the bundle is in the septum.

154. A 62-year-old woman whose arrhythmia is noninducible at EP study has depressed LV function without aneurysm.
The following is/are true:

a. If her arrhythmia is ventricular tachycardia, she is not a candidate for an Automatic Implantable Cardiac
Defibrillator (AICD) since it only recognizes fibrillation
b. If an AICD is appropriate, it offers a 50% improvement in mortality compared to drug therapy
c. Poor ventricular function is a contraindication to AICD implantation
d. AICD should not be used for patients awaiting cardiac transplantation
e. AICD can provide antitachycardia pacing as well as defibrillation
Answer: b, e

The AICD is capable of recognizing ventricular tachycardia as well as fibrillation and can provide antitachycardia pacing,
low or high-energy defibrillation or some combination. It offers a 50% improvement in mortality with 95% freedom from
SCD at 5 years after implantation. Neither poor ventricular function nor pending transplantation are contraindications to
AICD implantation.

155. A 27-year-old surgery resident has had multiple episodes of supraventricular tachycardia (SVT) and on EP study is
felt to have WPW syndrome. The following is/are true:

a. The pathophysiology of WPW is a single bundle of Kent


b. Dangerous ventricular responses in WPW are due to the shorter refractory period of the accessory pathway
c. Identification of the accessory pathway of WPW is an indication for its interruption
d. Approximately half of the patients who have successful division of accessory pathways demonstrate VA block
postop
e. Both radiofrequency catheter and surgical ablation offer excellent results with low morbidity
Answer: b, d, e

The pathophysiology of WPW is the Kent bundle of which 10–20% are multiple rather than single. The shorter refractory
periods permit rapid and dangerous ventricular responses to atrial flutter or fibrillation. In 0.25% of the population,
accessory pathways of WPW can be identified, but in the absence of a history of SVT, they have a normal life expectancy.
Approximately half the patients who have successful division of accessory pathways demonstrate VA block postop. Both
radiofrequency catheter and surgical ablation offer excellent results with low morbidity and the catheter technique is less
costly.

156. In the pharmacological management of cardiac arrhythmias, the following is/are true:

a. Membrane-stabilizing local anesthetics (Class 1) act via sodium channel blockage


b. Class 1 drugs also shorten the refractory period
c. b-blocking drugs (Class 2) block the sympathetic nervous system but not circulating catecholamines
d. Bretylium and other Class 3 drugs inhibit potassium influx into cells
e. Calcium channel blockers (Class 4) directly affect the SA and AV nodes
Answer: a, d, e
Class 1 drugs are local anesthetics that act via sodium channel blockade, and lengthen the refractory period. Class 2 b-
blocking drugs inhibit both the sympathetic nervous system and circulating catecholamines. Class 3 drugs inhibit potassium
influx into cells and Class 4 drugs affect slow channel-dependent pacemaker tissue (SA and AV nodes).

vascular

1. A 55-year-old woman gives a history of tiredness, aching, and a feeling of heaviness in the left lower leg for the past 3
months. These symptoms are relieved by leg elevation. She is also awakened frequently by calf and foot cramping, which is
relieved by leg elevation, walking, or massage. On physical examination there are superficial varicosities, nonpitting edema,
and a slightly painful, 2 cm. diameter superficial ulcer 5 cm. above and behind the left medial malleolus. What is the most
appropriate diagnosis?

A. Isolated symptomatic varicose veins.

B. Superficial lymphatic obstruction.

C. Deep venous insufficiency.

D. Arterial insufficiency.

E. Incompetent perforating veins.

Answer: C

DISCUSSION: The most common symptoms associated with venous insufficiency are aching, swelling, and night cramps
of the involved leg, which often occur after periods of sitting or inactive standing. Leg elevation frequently provides relief
of symptoms due to venous insufficiency, while it increases pain due to arterial insufficiency. Although edema can occur
with varicose veins alone, usually it is associated with deep venous abnormalities and incompetent perforators. In such
cases, ulcers usually are located above and posterior to the malleoli, reinforcing their relationship with perforator
abnormalities. The ulcers associated with arterial insufficiency may occur anywhere on the lower leg, eventually penetrate
the fascia, and are more painful than venous ulcers.

2. The best treatment plan for the patient described in the preceding question should include:

A. Varicose vein ligation and stripping as soon as possible.

B. Ulcer débridement, vein stripping, and skin grafting.

C. Ligation of the medial perforating veins.

D. Transposition of saphenous vein valve.

E. Leg elevation, external Unna boot support, and ambulation without standing.

Answer: E
DISCUSSION: Operative treatment of venous insufficiency is in most instances an adjunct after failure of aggressive
conservative management. Leg elevation, active exercise, and elastic compression form the cornerstones of nonoperative
management. The goals of compression are to relieve symptoms and reduce swelling. The indications for superficial vein
ligation and stripping are moderate to severe symptoms without other signs of deep venous insufficiency. If ulceration
persists despite appropriate conservative management, ligation of the underlying incompetent perforators helps ulcer
recurrence, and split-thickness skin grafting provides immediate coverage and healing of the ulcer. The patient must,
however, comply with a program of external stocking support and prevention of leg edema since the underlying venous
pathophysiology remains and ulcers tend to recur.

3. In patients who develop a documented episode of deep venous thrombosis (DVT) the most significant long-term sequela
is:

A. Claudication.

B. Recurrent foot infections.

C. Development of stasis ulcer.

D. Pulmonary embolization.

E. Diminished arterial perfusion.

Answer: C

DISCUSSION: The increased hydrostatic pressure from incompetent venous valves following DVT predisposes to
erythrocyte extravasation, hemosiderin deposition, and brown pigmentation around the ankle. Although the edema that
occurs with deep venous insufficiency can predispose to skin infections, these usually are located about the ankle and
resolve with adequate short-term care. When patients with a history of DVT are followed beyond 10 years, as many as 80%
ultimately develop venous stasis ulcers. While there may be recurrence of DVT in a minority of patients, the incidence of
pulmonary embolization is no greater than with the initial episode. Even in a leg severely affected by venous stasis changes,
the arterial circulation is unimpaired unless there is concomitant arterial obstructive disease.

4. A 28-year-old woman developed a painful thrombosis of a superficial varix in the left upper calf 2 days previously. After
spending the 2 days in bed with her leg elevated, she felt better and the tenderness resolved; however, when out of bed she
developed a twinge of right-sided chest pain when walking and a feeling of heaviness in the calf. Which treatment is most
appropriate?

A. Check for leg swelling, tenderness, and Homan's sign, and obtain a Doppler ultrasound study.

B. Begin antibiotics for a probable secondary bacterial infection.

C. Order emergency venography, and if it is abnormal, begin heparin administration.

D. Begin ambulation and discontinue bed rest that probably caused muscle pain by hyperextension of the knee.

E. If there is no pain on dorsiflexion of the left foot reassure her, since a negative Homan's sign precludes the diagnosis of
DVT.

Answer: C
DISCUSSION: Associated DVT may occur during treatment of superficial venous thrombosis, especially if the process is
near the groin or popliteal fossa. Although a positive Homan's sign or calf, popliteal, or groin pain is suggestive of DVT,
clinical examination alone may be incorrect in more than 50% of cases. Noninvasive tests, including Doppler
ultrasonography, are accurate for diagnosing DVT in the thigh but are less dependable in the calf. Emergency venography
performed on an outpatient basis remains the most accurate and cost-effective technique for diagnosing DVT of the calf
veins. Because 85% of pulmonary emboli arise from the lower extremity, early diagnosis and aggressive treatment are
important.

5. In a 55-year-old grocery store cashier with an 8-month history of leg edema increasing over the course of a work day,
associated with moderate to severe lower leg bursting pain, the most appropriate investigative study or studies are:

A. Doppler duplex ultrasound.

B. Brodie-Trendelenburg test.

C. Ascending and descending phlebography.

D. Measurement of ambulatory and resting foot venous pressure.

E. Venous reflux plethysmography.

Answer: A

DISCUSSION: While the Brodie-Trendelenburg test was an early attempt to clinically evaluate valve competence and
function, it is neither quantitative nor precise. The development of phlebography allowed anatomic delineation of normal
and abnormal veins and, when used in combination with invasive measurement of venous pressures in the foot at rest and on
ambulation, helped correlate the venous hypertensive state with postphlebitic changes. Noninvasive plethysmography to
quantitate the degree of venous valvular incompetence was more easily accepted; however the combination of B-mode
duplex ultrasound (to accurately locate the vein of interest) plus pulsed Doppler flow signal is now the “gold standard” for
venous assessment.

6. Which of the following statements are true of pulmonary embolism?

A. Most cases occur postoperatively.

B. In the majority of patients pulmonary emboli are ultimately lysed in situ without the administration of pharmacologic
agents.

C. The preferred therapy for most patients is intravenous heparin.

D. It is generally safe to give thrombolytic agents as early as 48 hours postoperatively.

Answer: BC

DISCUSSION: Although many patients develop pulmonary embolism postoperatively, the majority of such lesions reported
in most series do not follow operation. These patients develop thromboembolism as a complication of an underlying
condition such as congestive heart failure, cerebrovascular accident, malignancy, chronic infection, and a variety of other
debilitating diseases. Generally, postoperative patients comprise approximately one third of those with pulmonary
embolism. Serial pulmonary scans following pulmonary embolism generally show gradual clearing of the emboli with re-
establishment of perfusion in the occluded vessels. Depending on the magnitude of the embolism, most patients show the
clearing at the end of a month to 6 weeks. The presence of persistent congestive heart failure, chronic infection, and
atelectasis retard thrombolysis. This dissolution of emboli is generally agreed to be caused by naturally circulating
thrombolysins. In fewer than 1% of cases the emboli persist and often increase with the passage of time, with the
development of chronic pulmonary embolism leading to severe respiratory insufficiency, chronic cor pulmonale, pulmonary
hypertension, right ventricular failure, and death. The majority of patients with pulmonary embolism are managed by
continuous intravenous heparin. Thrombolytic agents are generally reserved for the management of extensive
thromboembolism in patients with a stable cardiovascular system. Thrombolytic agents are generally withheld from
postoperative patients until at least the fifth postoperative day, or preferably later. Earlier administration of these agents is
apt to produce bleeding at the operative site. While it may occasionally be indicated to proceed earlier, it is generally best to
wait until the thrombi in the vessels divided at the time of the surgical procedure have become organized.

7. Which of the following can cause a radioactive pulmonary perfusion scan to demonstrate an appearance similar to that of
acute pulmonary embolism?

A. Atelectasis.

B. Pneumonitis.

C. Pleural fluid.

D. Emphysematous bullae.

Answer: ABCD

DISCUSSION: It has been shown by routine radioactive pulmonary perfusion scans that atelectasis, pneumonia, pleural
fluid, emphysematous bullae, and pulmonary embolism may reduce pulmonary arterial blood flow to the involved segment.
For this reason, it is imperative simultaneously to obtain chest film to exclude any significant radiopacity, which is usually
associated with any defect that causes diminished pulmonary vascular perfusion.

8. In an otherwise healthy male with previously normal pulmonary and cardiac function, how much of the pulmonary
vascular bed must usually be occluded to produce an unstable cardiovascular state (shock)?

A. 10%.

B. 20%.

C. 40%.

D. More than 50%.

Answer: D

DISCUSSION: If the patient had normal cardiovascular and respiratory function before the onset of pulmonary embolism,
experimental and clinical studies have documented the fact that more than 50% of the pulmonary circulation must be
occluded to produce cardiovascular collapse or shock. This can be considered similar to ligation of the pulmonary artery
during the course of pneumonectomy, as occlusion by this procedure is also tolerated well, without development of
hemodynamic instability. While bronchoconstriction may reduce pulmonary function in the normally perfused lung after
embolism, this effect is generally short lived, as demonstrated by pulmonary ventilation scans.

9. Lytic therapy in pulmonary embolism:


A. Should precede anticoagulation.

B. Can be considered for all patients.

C. Can be considered for hemodynamically unstable patients.

D. Is indicated for the majority of patients with documented pulmonary embolism.

Answer: C

DISCUSSION: Thrombolytic therapy in pulmonary embolism involves use of streptokinase or urokinase. This therapy is
indicated for hemodynamic compromise from documented pulmonary embolism that has not responded to anticoagulation
and inotropic support. Additionally, lytic therapy is contraindicated after neurosurgery or cranial trauma and in persons who
have a history of internal bleeding or hemorrhagic cerebral infarction. The majority of patients with documented pulmonary
embolism do not require lytic therapy.

10. The single most important indication for emergency pulmonary embolectomy is:

A. The likelihood of another episode of embolism.

B. The inability to determine whether the problem is acute pulmonary embolism or acute myocardial infarction.

C. The presence of persistent and intractable hypotension.

D. Pulmonary emphysema.

Answer: C

DISCUSSION: The likelihood of another episode of pulmonary embolism is not an indication for pulmonary embolectomy
since recurrent embolism may also be managed nonoperatively in most cases. Myocardial infarction should definitely be
ruled out before pulmonary embolectomy is considered. In the presence of intractable cardiopulmonary collapse the only
measure apt to correct the condition is emergency pulmonary embolectomy employing extracorporeal circulation. In such
instances emboli generally are massive, with more than 50% of the pulmonary arterial circuit being occluded. The presence
of cardiac and/or respiratory insufficiency before the attack of embolism is important, as that is apt to mean that a smaller
pulmonary embolism is present that can cause serious cardiovascular changes.

11. In prevention of the fat emboli syndrome the primary therapy can be accomplished by which of the following?

A. Systemic anticoagulation achieving a partial thromboplastin time greater than 50 seconds.

B. Intravenous administration of alcohol.

C. Prophylactic administration of methyl prednisolone.

D. Maintaining a serum albumin value greater than 3 gm. per 100 ml. in the days immediately following injury.

Answer: D

DISCUSSION: The mechanism producing fat emboli syndrome is primarily release of free fatty acids that produce a change
in the capillary alveolar membrane. Albumin is the primary binder of fatty acids. With serum levels greater than 3 gm. per
100 ml., 99% of the circulating free fatty acid remains in the bound configuration. Alcohol, heparinization, and steroids
have been shown to have no direct effect on fat emboli syndrome, and serious side effects are related to their use.

12. Significant tachypnea and hypoxia follow development of fat emboli syndrome, and the goal of ventilatory support
should be:

A. Keeping the respiratory rate below 30.

B. Preventing respiratory alkalosis.

C. Reversing pulmonary shunting using positive end-expiratory pressure.

D. Maintaining an adequate total volume.

Answer: C

DISCUSSION: The primary defect is increased pulmonary shunting secondary to reduced functional residual capacity. Use
of positive end-expiratory pressure reverses this and is the primary goal of therapy.

13. Which of the following statements about the role of the endothelium is/are correct?

A. Endothelial cells only mediate vasorelaxation.

B. Endothelial cell–derived nitric oxide (NO) is produced by a constitutive and an inducible NO synthase.

C. The anticoagulant properties of the endothelium reside in its barrier function.

D. A local renin-angiotensin system is found in the walls of arteries and veins.

E. There are no significant differences in the vasomotor characteristic of endothelial cells in the macro- and
microcirculation.

Answer: BD

DISCUSSION: Endothelial cells have the ability to mediate both contractile (prostanoids, oxygen free radicals, endothelins,
angiotensins, and uncharacterized endothelium-derived constriction factors) and relaxant (prostanoids, nitric oxide and
nitric oxide–containing compounds, hyperpolarization factor) responses. NO is synthesized from the conversion of L-
arginine to citrulline by at least two categories of enzymes— constitutive nitric oxide synthases (cNOS, predominantly
membrane bound) and inducible nitric oxide synthases (iNOS, predominantly cytosolic), both of which are calcium- and
calmodulin dependent. Constitutive NOS is present in endothelial cells, and following cytokine stimulation endothelial cells
also synthesize iNOS. In addition to its basic barrier function, the endothelium regulates intravascular coagulation by three
other separate, but related, mechanisms: participation in procoagulant pathways, inhibition of procoagulant proteins,
regulation of fibrinolysis and production of thromboregulating compounds. There is a local renin-angiotensin system in the
vessel wall of both vein and arteries, and this system is considered important in the maintenance of blood pressure, the
control of hypertension, and the response of the vessel wall to injury. In contrast to the macrocirculation, the
microcirculation has distinctive vasomotor characteristics. Vessels of the microcirculation are the conduits that are
responsible for the local delivery and transfer of cell substrates and metabolites. The endothelium regulates the
microvasculature, reacting to the metabolic needs of tissue; it is essential in organ autoregulation and in the responses of
these microvasculatures to changes in local blood flow. Reactive hyperemia appears to be dependent on the immediate
production of endothelium-derived cyclo-oxygenase products, predominantly prostaglandin E 2 (PGE 2), as opposed to PGI
2, in the short term.
14. Which of the following statements on smooth muscle cells is/are correct?

A. Smooth muscle cells can undergo pheotypic changes in response to injury.

B. Platelet-derived growth factor (PDGF) requires a progression factor to initiate smooth muscle cell growth.

C. NO can be produced by smooth muscle cells.

D. Changes in the composition of the extracellular matrix modulate smooth muscle cell growth.

E. Smooth muscle cells are the principal cell involved in the development of intimal hyperplasia.

Answer: ABCDE

DISCUSSION: Smooth muscle cells are the principal cells found in the media of a vessel. They are embedded in a matrix of
connective tissue elements and provide mechanical and structural support to the vessel. In addition to their vasoreactive
characteristics, smooth muscle cells are capable of synthesizing and secreting elements of the extracellular matrix,
particularly proteoglycans. The exact mechanisms whereby smooth muscle cell proliferation is initiated, controlled,
reduced, and eventually suppressed are not fully understood. Quiescent vascular smooth muscle cells are well-differentiated
cells characterized by an abundance of contractile proteins, predominantly smooth muscle cell actin and myosin but little
rough endoplasmic reticulum. Once activated, smooth muscle cells lose their differentiated state, acquire abundant
endoplasmic reticulum, and commence the synthesis of extracellular matrix. After cytokine stimulation, smooth muscle cells
synthesize iNOS and produce nanomoles of NO for at least 24 hours. Smooth muscle cell proliferation depends on the
presence of PDGF, basic fibroblast growth factor (bFGF), and insulin-like growth factor 1 (IGF-1). IGF-1 acts as a
progression factor for PDGF in smooth muscle cells, and both in vivo and in vitro there is synergism between these two
growth promoters. Smooth muscle cell growth in the wall can be modulated by various extracellular matrix substances such
as collagen (type V), several glycoproteins, and the glycosoaminoglycans. Both NO and prostacyclin inhibit cell
proliferation. Intimal hyperplasia is a structural lesion that develops in injured blood vessels after injury and is the result of
smooth muscle cell migration into and proliferation within the intima of a blood vessel.

15. Which of the following statements correctly characterizes the healing of prosthetic arterial grafts in humans?

A. Complete healing occurs within 3 months of graft implantation.

B. Complete healing occurs within 1 year of graft implantation.

C. Prosthetic grafts do not heal completely in humans.

D. Polytetrafluoroethylene (PTFE) grafts heal completely whereas Dacron grafts do not.

E. Dacron grafts heal completely but PTFE grafts do not.

Answer: C

DISCUSSION: Prosthetic grafts of any kind do not heal completely in humans as they do in experimental animals. Some
healing occurs in the perianastomotic regions, but the majority of the graft's luminal surface never develops a living
neointima; rather, it remains covered with a compacted layer of fibrin.

16. Which of the following adversely influence the patency of lower extremity autogenous vein grafts?
A. Poor arterial outflow from the distal anastomosis of the graft.

B. Diabetes.

C. Small-caliber (less than 4 mm. in diameter) veins.

D. Use of reverse, rather than in situ, grafting technique.

E. Grafts performed for limb salvage indications rather than claudication.

Answer: ACE

DISCUSSION: Small-caliber veins perform poorly as arterial substitutes. The long-term patency of infrainguinal vein grafts
is better when the indication for operation is claudication rather than limb salvage and is improved when there is a good
outflow tract. Reverse vein and in situ bypass have equal patency rates at all levels. Surprisingly, the presence of diabetes
has not been shown to negatively affect the patency of autogenous vein infrainguinal arterial bypass.

17. Arterial autografts are:

A. Limited by the length of available artery.

B. When available, always appear to function superiorly to venous autografts.

C. Are the graft of choice for pediatric renal artery grafting.

D. Are performed infrequently.

E. Are immune from the fibrointimal hyperplasia that frequently complicates venous autografts and prosthetic grafts.

Answer: AC

DISCUSSION: Arterial autografts have limited but well-established clinical uses. They are the graft of choice for pediatric
renal artery revascularizations, as they do not appear to be subject to the aneurysmal dilatation that complicates pediatric
renal artery vein bypass. The internal mammary artery autograft provides superior patency for coronary artery bypass as
compared with the saphenous vein and is thus a very common arterial autograft procedure. Radial artery autografts for
coronary bypass are, however, rapidly narrowed by fibrointimal hyperplasia.

18. PTFE grafts:

A. Are a variant of a woven textile graft.

B. Provide patency superior to that with Dacron grafts for suprainguinal revascularization procedures.

C. Provide patency equal to that of autogenous saphenous vein for above-knee femoropopliteal bypass.

D. May be more resistant than Dacron grafts to pseudoaneurysm formation.

E. Are currently the prosthetic graft of choice for hemodialysis access.

Answer: DE
DISCUSSION: PTFE grafts are manufactured from a unique polymer extrusion process and are not textile grafts. For most
surgeons they are currently the prosthetic graft of choice for hemodialysis access. For suprainguinal bypass, PTFE and
Dacron provide comparable patency, although PTFE grafts may be more resistant than Dacron grafts to pseudoaneurysm
formation. PTFE at all levels in comparable patients is inferior to good-quality saphenous vein for infrainguinal bypass.

19. Which of the following statements about the evaluation of arterial substitutes are correct?

A. A graft is considered to have continued primary patency, even if it requires revision, as long as it has not actually
thrombosed.

B. Secondary patency refers only to grafts whose patency has been restored following an episode of thrombosis.

C. Patency figures should be derived from life table calculations.

D. Primary patency is the best indicator of the natural history of an arterial substitute.

E. Secondary patency is the best indicator of the natural history of an arterial substitute.

Answer: CD

DISCUSSION: Patency is the most important variable in the evaluation of the clinical effectiveness of an arterial substitute,
and it should always be derived from life table calculations. Both primary and secondary graft patency are important.
Primary patency is the best indicator of the natural history of a graft. A graft remains primarily patent as long as it has not
thrombosed or had any graft- or anastomosis-directed procedures. Grafts are considered to be secondarily patent if patency
has been restored after an episode of thrombosis or if, in order to maintain patency, operation on a patent graft is directed
toward the graft itself or its anastomoses.

20. Which of the following statements about aneurysms of the sinus of Valsalva is/are true?

A. Aneurysms of the sinus of Valsalva are dilatations of the aortic sinuses that eventually rupture into a cardiac chamber, the
pulmonary artery, or the pericardium.

B. The most common cause of an acquired sinus of Valsalva aneurysm is bacterial endocarditis.

C. Congenital aneurysms of the sinus of Valsalva usually cause symptoms long before they rupture.

D. The most common defect associated with congenital sinus of Valsalva aneurysms is aortic insufficiency.

E. The most common symptoms of sinus of Valsalva aneurysm rupture include symptoms caused by obstruction of the
ventricular outflow tract, heart block, and embolization.

Answer: AB

DISCUSSION: A sinus of Valsalva aneurysm is a dilatation of the aortic sinus that eventually ruptures into a cardiac
chamber, the pulmonary artery, or the pericardium. The cause may be either congenital or acquired, and the most common
cause of an acquired sinus of Valsalva aneurysm is bacterial endocarditis. The vast majority of sinus of Valsalva aneurysms
are asymptomatic before they rupture, but if symptomatic they present as obstruction to either the left or right outflow tract,
heart block, or embolization. When sinus of Valsalva aneurysms rupture, they present with symptoms of an acute left-to-
right shunt and these include dyspnea, palpitations, and chest pain. Finally, associated defects occur commonly with
congenital sinus of Valsalva aneurysms, the most common being ventricular septal defect followed by aortic insufficiency.
21. Which statements about treatment for sinus of Valsalva aneurysms are correct?

A. Close observation is appropriate for patients who have an asymptomatic sinus of Valsalva aneurysm without rupture.

B. Patients with sinus of Valsalva aneurysms that rupture should undergo operative repair because progressive heart failure
may well lead to death.

C. All patients with suspected sinus of Valsalva aneurysm ruptures need to undergo cardiac catheterization prior to
operation.

D. The best operative approach for closure of a ruptured sinus of Valsalva aneurysm is a dual approach through the aorta
and the chamber of entry of the fistula.

E. When a sinus of Valsalva aneurysm ruptures into the pericardium, emergency operation is required.

Answer: ABDE

DISCUSSION: When a found intact sinus of Valsalva aneurysm is asymptomatic, the patient should be followed closely for
symptoms but does not usually require operative intervention without symptoms or rupture. A ruptured sinus of Valsalva
aneurysm should be repaired by surgical intervention because the significant left-to-right shunt that occurs often leads to
progressive heart failure and death. While it is true that cardiac catheterization was the gold standard, the accuracy of
noninvasive transesophageal color-flow Doppler echocardiography has replaced it in some cases recently. Operative
intervention is not emergent unless the aneurysm ruptures into the pericardial space. If this occurs, symptoms of cardiac
tamponade are present and emergent operation is required. Finally, the best operative approach is a dual approach via the
aorta and the chamber of entry of the fistula. The major advantage of this approach is that the fistula can be identified
through the aorta into the chamber involved and closed securely from both sides. The aortic valve can be protected or
replaced as necessary, and the ventricular septum can be inspected for a ventricular septal defect.

22. Transection of the thoracic aorta following trauma usually:

A. Is located just distal to the left subclavian artery.

B. Produces a false aneurysm.

C. Is fatal in 80% of cases.

Answer: ABC

DISCUSSION: Transection of the thoracic aorta may occur at multiple sites from the isthmus down into the abdominal
aorta. Transection of the ascending aorta occurs but is uncommon. Those who survive the initial injury and do not rupture
into the pleural space do so because the false aneurysm is supported by the adventitia and pleura. The majority of
transections occur just distal to the left subclavian and the immediate mortality is about 80%.

23. Which of the following confirms the diagnosis of transection of the descending thoracic aorta?

A. Widened mediastinum.

B. Fractured first rib.


C. Left pleural effusion.

D. Positive aortogram.

E. All of the above.

Answer: D

DISCUSSION: The diagnosis of transected aorta should be suspected with severe injury especially if the patient is thrown
from the vehicle. A widened mediastinum, fractured first rib, or a pleural effusion can each be seen with chest trauma that
does not involve the thoracic aorta. The diagnosis is best made by the use of the aortogram even though the diagnosis can be
made with CT, MRI, and occasionally echocardiography.

24. The following is/are true of a descending dissecting aortic aneurysm:

A. It originates distal to the subclavian artery.

B. It is usually found in hypertensive patients.

C. It may extend the entire length of the aorta.

Answer: ABC

DISCUSSION: Descending dissections are almost always seen in hypertensive patients. The diagnosis is made by locating
the tear in the aorta distal to the left subclavian artery. Type III dissections may involve only the descending thoracic aorta
but can extend the entire length of the aorta.

25. The optimal management of Type A or ascending aortic dissection includes:

A. Aortography.

B. Hemodynamic monitoring and frequent recording of blood pressure, urinary output, and neurologic status.

C. Emergency operation.

Answer: ABC

DISCUSSION: Type A dissections are Type I or Type II dissections that originate in the ascending aorta. These may be
associated with aortic insufficiency or hypertension or hypotension depending upon whether or not there has been bleeding
into the pericardial space and an element of tamponade and/or neurologic deficit has occurred. Aortography is the best
method to establish this diagnosis. Patients with dissecting aortic aneurysms originating in the ascending aorta are
considered as emergencies. The operation involves grafting of the ascending aorta to prevent rupture in the mediastinum or
into the pericardium with correction of the aortic insufficiency while redirecting the blood into the true lumen.

26. Aneurysms of the ascending aorta may be caused by:

A. Type II aortic dissection.


B. Atherosclerosis.

C. Cystic medial necrosis.

Answer: ABC

DISCUSSION: Aneurysms of the ascending aorta are classically associated with Marfan's syndrome, cystic medial necrosis,
and, in past years, syphilis. Today, aneurysm formation in the ascending aorta is secondary to the manifestations of
degenerative disease occurring within the aorta and the aorta wall.

27. When complications occur after operating on a descending thoracic aorta, perhaps the most devastating is:

A. Recurrent nerve injury.

B. Bleeding with hemothorax.

C. Paraplegia.

D. Renal insufficiency.

Answer: C

DISCUSSION: Although the recurrent nerve can be stretched and hoarseness seen after operations in the area of the isthmus
of the thoracic aorta and bleeding associated with aneurysm repair as well as renal insufficiency from the cross-clamping of
the vessel above the renals, they are usually insignificant compared to the problem that occurs when paraplegia follows
operation on the descending thoracic or thoracoabdominal aorta. Not only is there a loss of function in the legs, but bowel
and bladder function are usually seriously affected. The psychological effects of paraplegia may also be devastating.

28. The most common risk associated with carotid artery aneurysm is:

A. Thrombosis of the aneurysm.

B. Embolization of mural thrombus.

C. Rupture of the aneurysm.

D. Compression of the hypopharynx.

Answer: B

DISCUSSION: Thrombosis of the aneurysm and internal carotid artery may occur through deposition of laminated clot
within the aneurysm and kinking of the outflow tract of the internal carotid artery. However, this is a relatively uncommon
clinical presentation for such an aneurysm. The majority of patients with carotid artery aneurysm present with transient
ischemic attacks in the ipsilateral cerebral hemisphere secondary to embolization of laminated clot lining the aneurysm wall.
If the aneurysm is left uncorrected, most patients ultimately suffer a stroke. Although rare, rupture of a carotid aneurysm is
almost always fatal. Rupture usually occurs into the oral cavity or nasopharynx, and death occurs from suffocation.
Aneurysms may present with symptoms of dysphagia secondary to pressure on neighboring cranial nerves and the lateral
wall of the pharynx. Nevertheless, these presentations do not represent serious risks to the patient.
29. Risks associated with carotid artery aneurysms are treated most successfully by which of the following?

A. Proximal ligation.

B. Observation.

C. Resection and graft replacement.

D. Resection and reanastomosis.

Answer: CD

DISCUSSION: Proximal ligation was the first method of surgical treatment and was employed during the nineteenth and
early twentieth centuries, but a high incidence of stroke was associated with its use. Gradual occlusion over several days
with a Crutchfield clamp can be used in the rare instance when resection and restoration of flow cannot be performed.
Reports from the first part of this century indicate that observation of symptomatic carotid aneurysms yielded a high
incidence of stroke and subsequent death. The preferred management of carotid aneurysms is resection and either
reanastomosis of the internal and common carotid arteries or interposition graft replacement. Large aneurysms frequently
require interposition grafting. Most aneurysms can be removed successfully, and flow can be re-established by
reanastomosis. Currently, this is being achieved with a combined operative stroke and mortality rate of less than 2%.

30. Which of the following statements about carotid body tumors are true?

A. Cells from which carotid body tumors arise normally sense changes in systemic blood pressure.

B. Most carotid body tumors are malignant and usually metastasize to the ipsilateral cerebral hemisphere.

C. Carotid body tumors are extremely vascular.

D. Carotid body tumors most frequently present as a palpable, painless mass at the carotid bifurcation.

Answer: CD

DISCUSSION: Cells of the carotid body function as receptors for changes in PO2, PCO2, and pH. Interestingly, carotid
body tumors are more prevalent in persons living at high altitudes. Although carotid body tumor cells may have
characteristics of malignancy, only about 5% of them metastasize. A carotid body tumor characteristically appears as a
hypervascular oval mass situated at the carotid bifurcation and widening the angle between the origins of the internal and
external carotid arteries. Although large or malignant carotid body tumors may cause dysfunction of the vagus and
hypoglossal nerves and cause dysphagia from their mass effect on the hypopharynx, most tumors are recognized when they
still have no associated symptoms and are only a painless mass over the area of the carotid bifurcation.

31. Treatment of carotid body tumors most frequently consists of:

A. Radical neck dissection, including the extracranial carotid artery.

B. Radiation therapy.

C. Resection of the common, internal, and external carotid arteries with interposition grafting.

D. Subadventitial dissection of the carotid bifurcation and simple excision of the tumor.
Answer: CD

DISCUSSION: Malignant carotid body tumors are uncommon. They are treated either by limited resection of the structures
involved with the tumor or by radiation therapy. Although radiation therapy has been demonstrated to shrink some tumors, it
should be used only for the rare tumor that is unresectable. Most tumors are benign and can be separated from the wall of
the carotid artery without resection of the bifurcation. Such resection is possible because the tumor is located in a plane
outside the media. Therefore, upon creation of a subadventitial plane, one can separate the tumor from the carotid artery
bifurcation without entering the vessel.

32. The cause of subclavian arterial aneurysms is most often:

A. Sepsis.

B. A congenital defect.

C. Atherosclerosis.

D. Fibromuscular dysplasia.

Answer: C

DISCUSSION: The most common cause of subclavian aneurysms is atherosclerosis followed by trauma. The thoracic outlet
syndrome may cause dilatation of the distal subclavian artery secondary to compression with poststenotic dilatation. Mural
thrombi are quite common in these aneurysms and may cause embolism in an arm, a serious complication.

33. Of the visceral aneurysms, which is the most common?

A. Celiac.

B. Superior mesenteric.

C. Hepatic.

D. Splenic.

Answer: D

DISCUSSION: Splenic aneurysms are the most common visceral type. They are usually caused by medial degeneration of
the arterial wall, although fibromuscular dysplasia can be a cause.

34. Aneurysms of the renal artery are most common:

A. At its origin from the aorta.

B. In the main renal artery or the bifurcation into the primary branches.

C. Within the kidney.


Answer: B

DISCUSSION: The majority of renal aneurysms are located either in the main renal artery or at the point of bifurcation into
the branches. The appropriate treatment is resection of the aneurysm with restoration of continuity of the artery whenever
possible, often with the use of a graft.

35. An aortic abdominal aneurysm was first successfully resected by:

A. Matas.

B. Linton.

C. Dubost.

D. None of the above.

Answer: C

DISCUSSION: Charles Dubost was the first to successfully correct an aortic abdominal aneurysm by resecting the entire
aneurysm and replacing it with an aortic homograft. In his paper he presented preoperative as well as postoperative
arteriograms demonstrating correction of both the aortic aneurysm and severe stenosis of the left iliac artery.

36. Evaluation of the natural history of abdominal aortic aneurysms in patients who are followed without any surgical
procedure indicates that:

A. Approximately 20% are alive at the end of 5 years.

B. Seventy-five per cent of patients succumb by the end of the first year.

C. Aortic rupture is quite common in this group, occurring in more than half by the second year.

D. None of the above.

Answer: A

DISCUSSION: In the classic study of Estes in 1951, during an era before surgical correction, the survival at 5 years of
patients with aortic abdominal aneurysms was 20%.

37. The appropriate treatment in most situations of an aortic abdominal graft that has become infected is:

A. Intravenous antibiotics and observation for future complications.

B. Catheter drainage at the site of infection.

C. Replacement of the infected graft with another prosthetic graft.

D. Excision of the entire graft and insertion of axillobifemoral grafts.


Answer: D

DISCUSSION: Many forms of management have been tried and occasionally recommended. The most appropriate
treatment for most patients with an infected graft is removal of the entire graft, closure of the iliac or femoral arteries distal
to the insertion of the graft, and placement of axillobifemoral grafts.

38. In a patient with an abdominal aortic aneurysm and a history of several previous abdominal procedures for release of
dense peritoneal adhesions causing episodes of intestinal obstruction, consideration should be given to which one of the
following at operation?

A. Cardiopulmonary bypass.

B. An incision from the xiphoid process to the symphysis pubis.

C. Incision in the left flank with a retroperitoneal approach.

D. An axillobifemoral graft.

Answer: C

DISCUSSION: A retroperitoneal approach has been recommended, particularly for patients who have had previous
abdominal procedures when peritoneal adhesions might represent a serious technical problem. In these circumstances, the
retroperitoneal approach can be quite satisfactory, especially in the management of small aneurysms and those located in the
midportion of the abdominal aorta between the renal arteries and aortic bifurcation. However, prolonged postoperative
incision pain may be a complication in some patients with this incision.

39. After emergency correction of an aortic abdominal aneurysm, the two most common causes of mortality are:

A. Acute renal insufficiency.

B. Severe hemorrhage from dehiscence of the suture line postoperatively.

C. Myocardial infarction.

D. Infection of the graft.

Answer: AC

DISCUSSION: Although all these complications may follow correction of an aortic abdominal aneurysm, the most common
ones are fatal complications from myocardial infarction and acute renal insufficiency.

40. The incidence of inflammatory aortic abdominal aneurysms with dense periaortic adhesions and possible involvement of
adjacent structures such as the duodenum, renal vein, and ureter is approximately:

A. 2%.

B. 10%.
C. 25%.

Answer: B

DISCUSSION: Increased attention is being given to inflammatory aortic abdominal aneurysms. These are characterized by
aneurysms surrounded by dense periaortic inflammation and at times involvement of surrounding structures such as the
duodenum, renal vein, and ureter. This problem occurs in some 7% to 10% of patients undergoing aneurysmectomy.

41. Which of the following statements about true femoral artery aneurysms is/are correct?

A. All three layers of the blood vessel wall are involved in true aneurysms.

B. There is a very high association with aortoiliac and popliteal aneurysms.

C. Femoral artery aneurysms occur bilaterally in about 10% of cases.

D. Type I femoral artery aneurysms involve the orifice of the deep femoral artery.

E. The most common complication of femoral aneurysms is rupture.

Answer: AB

DISCUSSION: By definition, a true aneurysm involves the intima, media, and adventitia. Femoral aneurysms are very often
associated with other aneurysms, ranging from 70% to 95% in most reports. They are most commonly associated with
aortoiliac or popliteal aneurysms. Approximately 50% of femoral aneurysms are bilateral. Type I femoral aneurysms are
limited to the common femoral artery; type II aneurysms involve the orifice of the deep femoral artery. Rupture of femoral
artery aneurysms is rare. The most common complications are due to thrombosis, embolism, or local compression.

42. Which of the following statements about false aneurysms of the femoral artery is/are correct?

A. The incidence of iatrogenic false aneurysms has increased in recent years.

B. Arteriography is the most useful study for diagnosis of iatrogenic femoral aneurysms.

C. Ultrasound-guided compression of iatrogenic false aneurysms is usually successful in achieving thrombosis.

D. Femoral anastomotic aneurysms usually involve the proximal anastomosis of a prosthetic infrainguinal bypass.

E. Rupture is the most common complication associated with femoral anastomotic aneurysms.

Answer: AC

DISCUSSION: With the introduction of invasive percutaneous procedures such as angioplasty, valvuloplasty, atherectomy
and coronary stenting, the incidence of iatrogenic false aneurysms has risen to about 0.6% to 1.0% in recent years. Color-
flow duplex ultrasound is the best test for diagnosis of these aneurysms, owing to its ability accurately to delineate the
anatomy. Additionally, compression of these false aneurysms using ultrasound guidance is very effective in achieving
thrombosis. Femoral anastomotic aneurysms are most commonly seen after aortofemoral bypass grafting and only rarely are
associated with infrainguinal procedures. Thromboembolic complications are the most common complication of these
lesions.
43. Which of the following statements about popliteal artery aneurysms is/are correct?

A. They are the most common site of peripheral artery aneurysms.

B. For a patient with an abdominal aortic aneurysm the risk of a popliteal aneurysm is approximately 50%.

C. For a patient with a popliteal artery aneurysm the risk of a contralateral popliteal aneurysm is approximately 50%.

D. Popliteal artery aneurysms most commonly present with local symptoms secondary to compression of the adjacent vein
or nerve.

E. Arteriography is the most accurate test for the diagnosis of popliteal artery aneurysm.

Answer: AC

DISCUSSION: Although rare, popliteal artery aneurysms are the most common site of peripheral artery aneurysms. While
the incidence of associated aneurysms in patients with popliteal aneurysms is high, the opposite is not true. The likelihood
of a patient with an abdominal aortic aneurysm having a popliteal artery aneurysm is less than 10%. About 50% of popliteal
aneurysms are bilateral. The most common presenting symptom in patients with popliteal aneurysms is leg ischemia,
secondary to either thrombosis or embolism. Local symptoms due to the aneurysm are relatively rare. Ultrasonography is
the best test for diagnosis of a popliteal aneurysm. Arteriography should be reserved for patients undergoing operative
repair.

44. Which of the following statements about management of popliteal artery aneurysms is/are correct?

A. All symptomatic aneurysms should be treated with surgery.

B. The most common operation is excision of the aneurysm with arterial reconstruction.

C. Thrombolytic therapy may be useful when there is thrombosis of the aneurysm and the distal runoff vessels.

D. The results for surgery for asymptomatic aneurysms are better than those for symptomatic ones.

E. The long-term results with prosthetic grafts are equivalent to those of autogenous vein grafts.

Answer: ACD

DISCUSSION: It is generally felt that all symptomatic popliteal aneurysms should be treated with surgery. Treatment of
asymptomatic aneurysms is more controversial; some advocate repair of all of these as well. A reasonable approach is to
consider surgery for asymptomatic aneurysms larger than 2 cm. Aneurysm ligation with bypass is the most commonly
performed operation for popliteal aneurysm, excision being reserved for aneurysms that cause local compressive symptoms.
Thrombolytic therapy, given either before or during operation, is useful in clearing the tibial and pedal vessels in cases of
acute thrombosis. The patency and limb salvage rates are better for asymptomatic lesions than for symptomatic ones.
Autogenous vein is superior to prosthetic material and is the conduit of choice for ligation and bypass of popliteal
aneurysms.

45. Which of the following statements about thrombo-obliterative disease of the aorta and its branches are correct?

A. The most common cause of obstructive disease is thrombi.

B. Atherosclerosis is the most common pathologic cause of arterial obstruction.


C. Lesions occur with greater frequency at the origin of vessels from the aorta.

D. Obstructive lesions are preferentially managed by endarterectomy.

Answer: BC

DISCUSSION: Thrombo-obliterative disease of the aorta and its branches is primarily due to atherosclerosis. The lesions
occur most frequently at the origin of blood vessels and are usually localized and short. Surgical management consists
primarily of bypass grafts since endarterectomy is often followed by later reocclusion with reappearance of symptoms.

46. Which of the following statements about Takayasu's disease is/are correct?

A. Atherosclerosis is restricted to the ascending aorta and innominate artery.

B. It primarily affects patients of Asian descent.

C. It is a nonspecific arteritis affecting the thoracic and abdominal aorta and its major branches.

D. The disorder is also characterized by systemic symptoms, including fever, malaise, arthritis, and pericardial pain.

E. Surgical bypass of the involved vessel should be undertaken in nearly all patients, since the results are excellent.

Answer: BCD

DISCUSSION: While it may affect others, Takayasu's disease occurs primarily in those of Asian descent and usually attacks
young females. It is basically a nonspecific arteritis involving all layers of the arterial wall with proliferation of connective
tissue and degeneration of elastic fibers. Clinical manifestations include fever, malaise, arthritis, and pericardial pain. Some
believe that it may be an autoimmune disease, and steroids have been effective in some patients. Late manifestations include
ischemia of both cerebral and upper extremity circuits. While surgical management is occasionally successful, it usually is
not recommended except for patients with disabling symptoms.

47. A 65-year-old man complains of having had slurred speech and no motor function or sensation of his right hand for 15
minutes. A left carotid bruit is heard in the neck. Which of the following diagnostic studies should be done?

A. Carotid duplex scan.

B. Electroencephalography (EEG).

C. Carotid arteriography.

D. Computed tomography (CT) of the brain.

Answer: CD

DISCUSSION: Although a carotid duplex scan provides valuable information on the presence of significant carotid artery
disease at its bifurcation, it cannot be used as the final test of the circulation to the brain in this particular case. Carotid
duplex scanning has its greatest value in the assessment of asymptomatic patients who have cervical bruits. In that
circumstance, it can provide information on which the decision for further workup can be based. When symptoms suggest a
transient ischemic attack, no noninvasive study provides a complete evaluation and arteriography is mandatory. EEG is a
valuable test for evaluating patients when seizure activity is suspected. In this patient, an EEG would be valuable only if all
other diagnostic studies had been unrevealing. An arteriogram is mandatory in this patient to adequately evaluate the entire
extracranial and intracranial cerebral circulation. If a duplex scan had been performed and had revealed no disease, an
arteriogram would still be necessary to exclude brachiocephalic trunk disease as a source of emboli. Likewise, intracranial
narrowing would also be missed by such a noninvasive study. For these reasons, duplex scanning might be considered
redundant and inadequate in these circumstances. CT of the head would be imperative in the evaluation of this patient. Its
primary role is to exclude cerebral infarction, even in the presence of transient symptoms. It also distinguishes an ischemic
cerebral event from an intracranial hemorrhage and rules out other potential causes of the symptoms, such as a brain tumor,
other space-occupying intracranial lesions, and arteriovenous malformation.

48. Carotid artery occlusive disease most often produces transient ischemic attacks or stroke by which of the following
mechanisms?

A. Reduction of flow to the affected area of the brain through stenotic or occluded vessels.

B. Embolization of atheromatous debris and/or clot with occlusion of intracranial branches of the carotid artery.

C. Thrombosis and propagation of the clot into the intracranial branches.

D. All of the above are equally common.

Answer: B

DISCUSSION: The collateral network to the brain is extensive. Collateral flow to an area supplied by a carotid artery is
provided by the contralateral carotid artery and vertebrobasilar system around the circle of Willis, by the external carotid
artery branches around the eye, and by direct intracerebral connections between the anterior cerebral arteries. For these
reasons, low-flow cerebral symptoms are extremely rare, even when carotid lesions are present bilaterally. Approximately
70% of all cerebral symptoms produced by carotid artery occlusive disease are embolic in origin. The surface of an
atherosclerotic lesion at the carotid bifurcation is thrombogenic and acts as a nidus for the accumulation of platelet-rich
thrombi. Similarly, the interior of the plaque can degenerate and rupture into the lumen, embolizing its contents into the
distal bed of the carotid circulation. The final event in the progression of an atherosclerotic carotid lesion is total
thrombosis. Because there are no extracranial branches of the internal carotid artery, the thrombus propagates distally. If the
propagation of the clot stops at the first major intracranial branch (the ophthalmic artery) and does not disturb collateral
flow through it, no cerebral ischemic event may occur. When propagation continues into the intracranial branches, a massive
stroke may occur. This mechanism accounts for a minority of strokes.

49. The majority of patients with “subclavian steal” syndrome have which of the following conditions?

A. Reversed flow in the involved vertebral artery.

B. Disabling neurologic symptoms.

C. Upper extremity claudication.

D. Decreased systolic blood pressure in the ipsilateral arm.

Answer: AD

DISCUSSION: Subclavian steal syndrome results from occlusion of a subclavian artery, rarely the innominate, with
decreased systolic pressure distal to this obstruction. This causes blood to flow up the contralateral vertebral area and across
the basilar artery (from which more blood is “stolen”) as it courses down (in a retrograde manner) the ipsilateral vertebral
artery to help supply that subclavian artery. Most patients with this phenomenon are asymptomatic, although limb weakness
and paresthesias or symptoms of vertebral basilar insufficiency may occur. Strokes do not occur in patients with subclavian
disease alone. Most affected patients, however, have associated atherosclerotic disease of other extracranial arteries,
particularly the carotid vessels, which may contribute to symptoms of cerebral ischemia.

50. Which of the following treatments is/are appropriate for symptomatic subclavian steal syndrome?

A. Subclavian endarterectomy.

B. Carotid-subclavian bypass.

C. Subclavian-carotid transposition.

D. Intra-arterial streptokinase.

Answer: BC

DISCUSSION: Because most patients with the abnormal flow phenomena demonstrated in subclavian steal syndrome are
asymptomatic, surgical treatment is not necessary. Furthermore, some patients with subclavian steal have symptoms caused
by other extracranial arterial disease and benefit from therapy directed at carotid disease. For patients whose symptoms are
attributed to the subclavian steal phenomenon, a variety of surgical procedures have been proposed. Relief of symptoms has
been obtained in 70% to 80% of these patients. Currently, both carotid-subclavian bypass and subclavian-carotid
transposition are considered acceptable.

51. Which statements about thrombo-obliterative disease of the terminal abdominal aorta (Leriche syndrome) are true?

A. It is characterized by a combination of atherosclerotic and thrombotic occlusion of the terminal aorta.

B. It is characterized by acute thrombosis of the terminal aorta.

C. It requires emergency revascularization when the diagnosis is made.

D. It is often associated with distal obstructive lesions.

E. Patients often show signs of ischemia in the legs, and males may have difficulty maintaining a stable erection.

F. The preferred surgical procedure is bypass with a prosthetic graft.

Answer: ABEF

DISCUSSION: Leriche syndrome usually affects males 35 to 60 years of age. It is caused by a combination of thrombosis
superimposed on atherosclerosis. The symptoms are usually those of ischemia of the legs, including weariness or
claudication, atrophic changes, and the inability to maintain a stable erection owing to inadequate arterial flow to the penis.
Distal sites of segmental occlusion are not uncommon. This disorder is often tolerated well for 5—and even 10—years, but
ultimately serious symptoms of ischemia require operation. The procedure of choice is a bypass with a prosthetic graft from
the aorta to the iliac or femoral arteries bilaterally.

52. Following surgical correction of Leriche syndrome, which of the following are true?

A. Gangrene is usually prevented, but symptoms of claudication persist.


B. The symptoms of claudication are usually improved.

C. Sexual function is improved in some patients.

D. Patients who continue to smoke following surgical correction have a higher incidence of reocclusion.

E. Prevention of damage to the sympathetic and parasympathetic nerves in the periaortic region reduces the postoperative
incidence of retrograde ejaculation.

Answer: BCDE

DISCUSSION: Following corrective operation, patients with a history of claudication as well as ischemic manifestations
are usually considerably improved. Those who have one or more patent internal iliac (hypogastric) arteries are more likely
to have improved sexual function. It is important that patients cease smoking following bypass grafts since there is objective
evidence that thrombosis of grafts is appreciably increased by continuance of smoking. Minimal dissection in the region of
the abdominal aorta is important in minimizing damage to the sympathetic and parasympathetic nerves, since injury is
related to the incidence of postoperative retrograde ejaculation.

53. In a patient who has chronic, complete occlusion of a common iliac artery, which of the following are true?

A. The primary symptom is claudication of the calf muscles.

B. Symptoms are usually claudication of the thigh and calf.

C. The decision as to whether or not to operate can be based on clinical examination findings.

D. Collateral iliac arterial vessels are prevalent.

E. Balloon angioplasty is appropriate in some patients.

Answer: BDE

DISCUSSION: Occlusion of the common iliac artery is usually associated with claudication of the thigh and calf.
Arteriography should be done to establish the diagnosis and to assess the peripheral arterial system. Arteriography is also
quite helpful in deciding whether or not balloon angioplasty is indicated, since it can be used successfully in some patients.
Collateral vessels are usually apparent on the arteriogram. Bilateral involvement of the iliac vessels is quite common, and if
symptoms are not present at the outset they may develop later. The preferred surgical management is bypass grafts from the
aorta to the patent distal circulation.

54. Which of the following does not describe intermittent claudication?

A. Is elicited by reproducible amount of exercise.

B. Abates promptly with rest.

C. Is often worse at night.

D. May be an indication for bypass surgery.

Answer: C
DISCUSSION: Intermittent claudication characteristically is elicited by a relatively reproducible amount of exercise, and it
is promptly relieved by cessation of that exercise. Furthermore, it is usually felt in a defined muscle group such as the calf
muscles. Rest pain, indicating impending limb loss, is felt in the toes and forefoot as opposed to a muscle group, and this
pain often begins at night. Claudication, when sufficiently disabling, may be an indication for revascularization, particularly
if it interferes with gaining a livelihood.

55. In terms of long-term graft patency, the best results in the femoral tibial bypass position have been achieved with:

A. A modified human umbilical cord graft.

B. Polytetrafluoroethylene (PTFE [Gore-Tex]).

C. Saphenous vein allograft.

D. Segments of greater and lesser saphenous and cephalic veins spliced together.

Answer: D

DISCUSSION: Particularly for distal bypass grafting, the autogenous saphenous vein is the graft material of choice. When
this is not available intact, several segments of this vein, as well as lesser saphenous and upper extremity veins, can be used
for creating a graft of sufficient length. When the distal anastomosis is below the popliteal artery autogenous tissue grafts
are clearly superior to synthetic ones.

56. Which of the following statements about femoral popliteal bypass grafting is/are true?

A. Long-term graft surveillance by duplex scanning has no effect on graft patency rates.

B. Graft failure and amputation occur in half the patients within 5 years.

C. If grafting is successful, long-term mortality is improved.

D. Patency rates of 80% to 90% at 1 year currently are expected.

Answer: D

DISCUSSION: Graft surveillance and repair of acquired defects in the vein before graft thrombosis improve long-term
patency rates. Patients who require lower extremity bypass for limb salvage have poor 5-year survival because of
concomitant coronary and cerebrovascular disease. Current graft patency rates approach 90% at 1 year.

57. Which of the following statements about percutaneous renal artery transluminal angioplasty (PRTA) are true?

A. Patients with renovascular hypertension are usually cured after successful PRTA.

B. Patients with renovascular hypertension due to atherosclerosis are more likely to benefit from PRTA than those in whom
it is due to fibromuscular dysplasia.

C. PRTA of ostial atherosclerotic lesions is more successful than PRTA of nonostial lesions.
D. PRTA is associated with a higher morbidity and mortality than angioplasty for peripheral vascular disease.

Answer: D

DISCUSSION: The results of treatment for renovascular hypertension depend on the nature of the obstructing lesion and its
anatomic location. Treatment of obstruction due to fibromuscular dysplasia is technically successful in 90% of patients and
helps to control hypertension in almost 85%. However, fewer than 60% are cured. The results of treating atherosclerotic
causes are substantially worse. Approximately 60% of patients show improvement in blood pressure control, but fewer than
25% are cured. The results of treating nonostial atherosclerotic lesions are better than those for ostial lesions. Unfortunately,
the majority of patients with renovascular hypertension have ostial atherosclerotic lesions as the obstructing lesion. The
complication rate associated with PRTA is substantially higher than that for angioplasty of vascular disease of the lower
extremities, averaging almost 20% in patients with atherosclerotic lesions. Renal insufficiency is more likely. The mortality
averages 1%, but higher rates are reported.

58. True statements about percutaneous transluminal angioplasty (PTA) of peripheral vascular lesions include which of the
following?

A. PTA of iliac lesions is more often successful than PTA of femoral artery lesions.

B. PTA of iliac occlusions produces results similar to PTA of iliac artery stenoses.

C. PTA of infrapopliteal occlusive disease is associated with an increased rate of vasospasm, which can cause thrombosis.

D. A short, singular arterial stenosis is the optimal situation for a successful angioplasty.

Answer: ACD

DISCUSSION: Clinical results using PTA to treat occlusive vascular disease of the lower extremities depend on the
morphologic nature of the obstruction as well as the anatomic location. PTA of iliac lesions is technically successful in
approximately 90% of patients with 3-year patency rates of approximately 80%. PTA of superficial femoral artery lesions is
technically successful in 75% to 85% of patients with 3-year patency rates of approximately 60% to 70%. PTA of
infrapopliteal vessels is associated with increased risk of vasospasm, which can cause thrombosis. Improvements in the
technique and pharmacologic treatment of vasospasm have reduced the risk of vessel thrombosis to less than 10%. Although
some series have reported PTA of infrapopliteal lesions in patients with claudication, most authors recommend treating only
patients with threatened limbs. Ideal patients for PTA are those with stenotic lesions less than 5 cm. long; the best results
occur in the shortest lesions. Although total occlusions and multiple stenoses have been treated successfully, there are fewer
technical successes, and long-term patency rates are substantially lower than in the treatment of stenotic lesions.

59. Advantages of PTA, as compared with surgical revascularization, include which of the following?

A. Decreased initial cost, shorter convalescence, and earlier return to full activity.

B. Because PTA is performed under local anesthesia, it is applicable to a greater number of patients with peripheral vascular
disease.

C. PTA is more durable and requires fewer subsequent procedures.

D. Repeat PTA is well-tolerated, morbidity is equivalent to that for the initial procedure, and success rates are comparable to
those expected with the initial procedure.

Answer: AD
DISCUSSION: The primary advantages of PTA over surgical revascularization are the lower initial cost of the procedure,
decreased hospital stay, earlier return to full activity, lower morbidity, and the ability to repeat the procedure without
markedly increasing patient morbidity or compromising clinical results. The procedure can be performed in patients who
would have a high operative risk related to coexisting medical conditions; however, angioplasty is not applicable to the
majority of patients with vascular occlusive disease because of the presence of long stenoses, occlusion, or multiple lesions
in the vascular system. Restenosis and progression of disease in the vascular system also limit the long-term results of PTA
and are the cause of a greater need for subsequent procedures.

60. Which of the following statements are true?

A. All arterial injuries are associated with absence of a palpable pulse.

B. Preoperative arteriography is required to diagnose an arterial injury.

C. The presence of Doppler signals indicates that an arterial injury has not occurred.

D. Patients with critical limb ischemia have paralysis and paresthesias.

E. In all patients with multiple trauma, arterial injuries should be repaired before other injuries are addressed.

Answer: D

DISCUSSION: Although arteriography may be required to diagnose partially severed or damaged arteries with persistent
flow, it is not required for the diagnosis of most injuries. In a patient who is not in shock, a diminished pulse on physical
examination indicates an arterial injury; however, patients with only partial severance or intimal injury may maintain axial
flow through the injury and distal pulses will be palpable. In a patient who has adequate collateral flow, Doppler signals
may be heard even in the presence of complete occlusion of an axial artery. These patients may have audible flow, but their
Doppler-derived pressure will be decreased. Patients who do not have adequate collateral flow present with critical ischemia
manifested by paralysis and paresthesias, the latter in a stocking or glove distribution. Patients who have neither paralysis
nor paresthesia do not have immediately limb-threatening injuries and, for them, arterial reconstruction can be deferred until
more acute life- and limb-threatening lesions have been treated.

61. A patient presents with a gunshot wound of the mid-neck. Although drunk, he exhibits no lateralizing neurologic signs.
After control of his airway is achieved, he is taken directly to the operating room for control of hemorrhage. The common
carotid artery has a 2-cm. destroyed segment. There is also a major esophageal injury. The best treatment for this carotid
injury is:

A. Vein graft replacement of the common carotid artery.

B. Ligation of the common carotid artery.

C. Ligation of the common carotid artery proximally with a subclavian carotid bypass.

D. Ligation of the common carotid artery with sympathectomy.

E. Prosthetic graft replacement of the common carotid artery.

Answer: A

DISCUSSION: The treatment of carotid artery injury in a patient without neurologic deficit consists of reconstruction of the
carotid. This patient had no neurologic deficit preoperatively and should have a good result. Autogenous material is
preferred over prosthetic material. Ligation without reconstruction may be tolerated in the common carotid artery but is not
recommended. Subclavian carotid bypass has no advantage over reconstruction unless it can be performed in a clean
operative field. Cervical sympathectomy is not performed in these circumstances. Although the presence of an esophageal
wound means that there is a potential for infection, it should not deter reconstruction with autogenous material. This
potential for infection, however, should alert the physician to the possible development of false aneurysm in the
postoperative period. If an infected false aneurysm develops, alternative methods of carotid reconstruction must be utilized.

62. Which of the following statements about iatrogenic arterial injuries are true?

A. Femoral artery pseudoaneurysms occurring after arteriography require urgent operative intervention.

B. Symptomatic axillary sheath hematomas require urgent operative intervention.

C. Arterial occlusions after catheterization occur more commonly in the femoral artery than in the brachial artery.

D. The Allen test identifies patients with an incomplete palmar arch.

Answer: BD

DISCUSSION: Small pseudoaneurysms after arterial puncture often close spontaneously. Duplex-guided compression may
be used to cause thrombosis of the pseudoaneurysm. Surgery is usually reserved for large or expanding pseudoaneurysms
that are not successfully occluded by duplex-guided compression. On the other hand, axillary sheath hematomas can cause
permanent nerve injury and require urgent surgical decompression. Occlusions after catheterization are more common in
smaller arteries such as the brachial artery, but ischemic signs and symptoms are more likely with larger vessel occlusions.
The Allen test should be performed before all radial artery cannulations to show that there is an adequate palmar arch.

63. A 35-year-old man involved in a motor vehicle accident presents with a knee dislocation that is easily reduced.
Radiography of the knee shows no fracture. Which of the following statements about his treatment are true?

A. If he has normal pulses he can be discharged.

B. If he has normal pulses he requires either close observation or arteriography.

C. If he has absent distal pulses and severe ischemia he should undergo arteriography in the radiology suite.

D. A popliteal vein injury is best treated with ligation.

E. A popliteal artery injury should be repaired with the ipsilateral saphenous vein if available.

Answer: B

DISCUSSION: Knee dislocations commonly cause popliteal vessel injury. Early after injury, flow may be sustained in the
injured popliteal artery; therefore, these injuries require close observation or arteriography. When an occluded popliteal
artery is evident clinically, immediate repair is indicated and arteriography is unnecessary. Repair should ideally utilize the
contralateral saphenous vein, to maintain optimal venous return in the injured leg. Injured popliteal veins should be repaired
if at all possible. Ligation may jeopardize the arterial repair and increase the likelihood of chronic venous insufficiency and
the risk of pulmonary embolism.
64. A 24-year-old man is involved in an industrial accident in which he sustains a crushed pelvis. Diagnostic peritoneal
lavage is positive. At exploration, a large pelvic hematoma is found. What is the best treatment?

A. Explore all the major arteries and veins of the pelvis and surgically control the bleeding if possible.

B. Do not explore the pelvic hematoma. Close the abdomen and apply a MAST suit.

C. Do not explore the pelvic hematoma. Apply a pelvic fixator and send the patient to radiology for possible embolization of
bleeding pelvic vessels.

D. Use sustained hypotensive anesthesia to try to control bleeding.

E. Open the pelvic hematoma and apply laparotomy pads with topical hemostatic agents.

Answer: C

DISCUSSION: Pelvic hematomas are not usually the result of common or external iliac artery disruption. Therefore, unless
the hematoma is rapidly expanding in the area of one of the major iliac arteries, it is not opened. Instead, the pelvic fracture
is fixed externally and arteriography is performed. Usually, bleeding is from smaller arteries as they exit through the
fracture site. These are best controlled with embolization. The other options are poor surgical choices that have little chance
of success.

65. Which of the following statements about acute arterial occlusion today is/are not true?

A. Most arterial emboli originate in the heart as a result of underlying cardiac disease.

B. It can be treated under local anesthesia.

C. It is usually due to atherosclerotic disease.

D. Surgical treatment can usually be avoided if the lesion is diagnosed early.

Answer: D

DISCUSSION: The majority of arterial emboli originate in the heart as a result of atherosclerotic disease manifested by
myocardial infarction, atrial fibrillation, congestive heart failure, or ventricular aneurysm. The incidence of embolization
from rheumatic mitral stenosis has decreased significantly. Elderly and debilitated patients who are poor risks for general
anesthesia can be treated with the balloon embolectomy catheter technique, which can be performed under local anesthesia
via groin incisions. Atherosclerotic disease in the peripheral vessels can cause thrombosis at the site of atheroma. Acute
arterial occlusion represents an emergency that is best treated promptly via heparinization and balloon catheter
embolectomy.

66. Which of the following statements about lytic agents is/are true?

A. They were first introduced well after the advent of balloon embolectomy.

B. Streptokinase is nonantigenic.

C. Systemic use is the most effective means of delivery.

D. The interval to reperfusion limits their utility in the treatment of advanced ischemia.
Answer: D

DISCUSSION: The balloon catheter technique for embolectomy and thromboembolectomy was introduced in 1963. Lytic
agents were undergoing clinical investigation at that time. Streptokinase, derived from streptococci, is antigenic. Urokinase
is a naturally occurring enzyme and is not antigenic. Lytic agents have yet to play a major role in the treatment of advanced
ischemia in the periphery because of delayed reperfusion, cost, risk of complications, the danger of increased tissue loss.

67. Which of the following is not an indication for postoperative heparinization?

A. Suspected venous thrombosis.

B. Risk of embolism following acute myocardial infarction.

C. Advanced ischemia secondary to acute embolic occlusion.

D. Dissolution of residual thrombus after balloon thromboembolectomy.

Answer: D

DISCUSSION: Conditions that would cause undesirable consequences from rethrombosis (e.g., venous thrombosis,
additional embolization from the cardiac source, pre-existing ischemia) are indications for postoperative heparinization.
Heparin prevents additional clotting; it is not a thrombolytic agent.

68. Which of the following is/are not true of the embolectomy catheter technique?

A. The balloon should be inflated by the same person who withdraws the catheter.

B. Distal exploration should be carried out in all major branches of the affected extremity.

C. The balloon is designed to dilate as it traverses areas of luminal narrowing.

D. Removal of adherent thrombus requires alternate catheter-based therapy in addition to balloon exploration.

Answer: C

DISCUSSION: The embolectomy balloon should be adjusted in diameter as the catheter is withdrawn to accommodate
changes in luminal diameter and to effect appropriate traction. To do so, the operator must simultaneously control the
withdrawal and the balloon inflation. The propensity of clot to propagate distally and proximally requires nearby branches
to be explored. When it is technically feasible, major branches should be explored. The embolectomy balloon is made of a
distensible elastomer that conforms to the intra-arterial surface to maintain wall contact. Dilatation balloons, in contrast, are
made of a nondistensible material that can effect dilating force on the arterial narrowing. An increase in the incidence of
adherent thrombus from chronic atherosclerotic disease and failed synthetic grafts has promoted the development of more
aggressive catheter-based systems. Graft thrombectomy and adherent clot catheters are specifically designed to remove
adherent thrombus left behind after balloon exploration.

69. Which of the following is the least reliable indicator of successful thrombectomy?

A. Vigorous back-bleeding after removal of thrombotic material.


B. Arteriographically demonstrated patency of all runoff vessels.

C. Normal distal pulses.

D. Return of normal skin color and temperature.

Answer: A

DISCUSSION: The significant incidence of discontinuous thrombus makes the presence of vigorous back-bleeding an
unreliable indicator of distal patency. Proximal material can be cleared, precipitating back-bleeding from all side branches
yet still leaving the main vessel occluded more distally. Arteriography, normal distal pulses, and the return of normal skin
color and temperature are the best indicators of distal patency, and thus of successful reperfusion. It must be borne in mind
that the primary goal of most thromboembolectomy procedures is to return the limb to its preocclusive state. Optimal distal
perfusion may require additional therapy for the patient with chronic atherosclerotic disease.

70. Which of the following statements about arteriovenous fistula are correct?

A. The local features characteristic of an arteriovenous communication are demonstrated by the presence of a thrill and bruit
with aneurysmal dilatation.

B. An arteriovenous fistula is best managed by ligation of the feeding vessels.

C. The most common type of arteriovenous fistula is iatrogenic, created for vascular access.

D. Branum's or Nicoladoni's sign is increased heart rate when the fistula is compressed.

Answer: AC

DISCUSSION: Local features characteristic of an arteriovenous fistula include a thrill and bruit at the site of the fistula
associated with aneurysmal dilatation. Simple ligation of feeding vessels often leads to vascular insufficiency of the
peripheral vascular bed secondary to drainage of the flow via collateral circulation. Repair of the artery and vein is
considered the optimal therapy. If this is not possible, ligation of the feeding artery and vein proximally and distally can be
performed if sufficient collateral circulation has developed, which generally takes months or more after the fistula develops.
Branum's or Nicoladoni's sign is a decrease in the heart rate following compression of the fistula.

71. Of the following statements about congenital arteriovenous malformations, which are correct?

A. Patients with complex congenital arteriovenous malformations should as early as possible undergo ligation of feeding
vessels.

B. Embolizing large arteriovenous malformations has not been demonstrated to be beneficial.

C. The most common complications of a large arteriovenous fistula are symptoms of congestive heart failure, pain,
ulceration, and cosmetic deformity.

D. Most congenital arteriovenous malformations are easily managed with simple excision.

Answer: ABC

DISCUSSION: Complex congenital arteriovenous malformations are often one of the greatest challenges a surgeon may
encounter. Management includes accurate diagnosis and determination of the extent of the lesion. The site of
communication should be localized by arteriography; however, computed tomography (CT), magnetic resonance imaging
(MRI), and duplex Doppler imaging may prove useful in the diagnosis of arteriovenous communications. Ideal surgical
management includes closure of the fistula with restoration of arterial and venous continuity. Complex arteriovenous
malformations require a multidisciplinary approach, including intra-arterial embolization in conjunction with surgical
therapy. Indications for surgery include secondary ischemic complications and congestive heart failure, pain, nonhealing
ulcers, and a cosmetic deformity. Ligation of feeding vessels is effective only temporarily, making further treatment,
especially embolization, difficult or impossible. Preoperative embolization may allow surgical resection and reduce
operative blood loss.

72. Thrombosis occurs frequently in thromboangiitis obliterans (Buerger's disease) in which of the following vessels?

A. Superficial femoral artery.

B. Radial or ulnar artery.

C. Digital arteries.

D. Superficial veins.

Answer: BCD

DISCUSSION: Thromboangiitis obliterans is characterized by thrombosis and inflammation of small and medium-sized
peripheral arteries and veins, and by migratory superficial phlebitis. Unlike atherosclerosis, it often involves the upper
extremity. Thromboangiitis obliterans rarely involves large vessels such as the brachial or femoral arteries.

73. Which of the following statements about thromboangiitis obliterans (Buerger's disease) are true?

A. The disease affects only young men.

B. The disease is more common in Asia and the Middle East than in the United States.

C. Life expectancy is limited.

D. The usual cause is smoking.

Answer: BD

DISCUSSION: Thromboangiitis obliterans was originally described in young men, who still constitute the overwhelming
majority of patients. In recent years, however, several cases have been reported in women, perhaps a reflection of the
marked increase in the number of female smokers. For reasons unknown, there is marked geographic variation in the
prevalence of thromboangiitis obliterans. The number of cases in the United States has decreased markedly since World War
II, but the incidence is much higher in other countries, particularly in the Far East. Although thromboangiitis obliterans
often results in peripheral gangrene and may necessitate amputation, long-term life expectancy differs little from that for the
general population, unlike that of atherosclerosis. Although rare instances of thromboangiitis obliterans have been reported
in nonsmokers, the most common cause is cigarette addiction, which usually starts at an early age. Prolonged remissions
often follow cessation of smoking.
74. A 52-year-old man presents with sudden onset of profound cyanosis of the second and third digits of the right hand with
gangrene of the tip of the second digit. The remaining digits and the other hand are not affected. Which of the following
statements are true?

A. This is characteristic of vasospastic Raynaud's syndrome.

B. Evaluation should include arteriography.

C. A coagulation abnormality may be the cause of this problem.

D. Thoracic sympathectomy is the first-line treatment.

Answer: BC

DISCUSSION: Gangrene is indicative of occlusive disease and not of vasospastic disease. The other factor that makes this
unlikely to be vasospasm is the involvement of only two fingers. Because of the possibility of a surgically correctable
upstream vascular lesion such as a subclavian aneurysm, patients presenting with embolic-type lesions should have
arteriography. Thus this patient, presenting with ischemia of sudden onset in just two digits, should undergo arteriography.
Coagulation abnormalities such as antithrombin-III, protein C, or protein S deficiency, or the presence of antiphospholipid
antibodies or the lupus anticoagulant, may cause digital gangrene. Conservative therapy—local wound care and
débridement—leads to healing in most patients. Sympathectomy should be reserved for patients who fail to heal after
months of conservative therapy.

75. A 21-year-old woman presents with digital color changes in response to cold stimulation. Physical examination and
laboratory data, including an autoimmune disease screen, are normal. She should be advised that:

A. Her condition is characteristic of vasospastic Raynaud's syndrome and, while she may be at a slightly higher risk for
developing a connective tissue disease in the future, there is no evidence of one currently.

B. Her problem with her fingers will get progressively worse and she will eventually lose fingers.

C. She has scleroderma, which will manifest itself at a later date.

D. Her problem is “all in her head.”

Answer: A

DISCUSSION: This presentation is classic for vasospastic Raynaud's syndrome. While these patients are probably at a
slightly higher risk for developing a connective tissue disorder in the future, this risk is low. These patients do not invariably
progress to tissue loss. This patient has no evidence of scleroderma at the present, and there is no reason to predict that she
will develop it. Even though her physical and laboratory examinations are negative, her history is positive for Raynaud's
syndrome.

76. Obstructive Raynaud's syndrome can be differentiated from vasospastic Raynaud's syndrome by the:

A. Ice water test.

B. Digital hypothermic challenge test.

C. Antinuclear antibody levels.


D. Digital blood pressure measurement.

Answer: D

DISCUSSION: Patients with obstructive Raynaud's syndrome have fixed palmar and digital arterial lesions with a decrease
in digital blood pressure at rest. In contrast, patients with vasospastic Raynaud's syndrome have normal digital arteries at
rest and normal digital blood pressure. Therefore the only test that differentiates the two conditions is measurement of
digital blood pressure. The ice water and digital hypothermic challenge tests both test for the presence of Raynaud's
syndrome, but do not differentiate the spastic from the obstructive type. The presence or absence of antinuclear antibodies
does not determine whether obstruction is present, although it is true that many patients with obstructive Raynaud's
syndrome will have a positive test.

77. Which of the following statements about upper extremity arterial insufficiency is/are true?

A. Symptomatic ischemia is more common in the upper extremity than in the lower extremity.

B. Vascular injuries from blunt trauma are more common in the upper extremity arteries than in the lower extremity ones.

C. Arteriovenous fistulas frequently follow blunt trauma.

D. The inflammatory process of arteries obstructed by an arteritis should be controlled before a bypass graft is inserted.

Answer: D

DISCUSSION: The larger muscle mass and heavier work loads of the lower extremities may lead to the development of
ischemic symptoms in them more often than in the upper extremities. Blunt trauma occurs more often in the lower
extremities. Arteriovenous fistulas more frequently follow penetrating trauma than blunt trauma. If the arteritis is not
controlled before a bypass graft is placed, the inflammatory process may contribute to early occlusion of the graft.

78. Which of the following statements about upper extremity edema is/are true?

A. Lymphedema is more common than venous edema.

B. Signs and symptoms of venous obstruction include edema, distention of superficial veins, tightness, aching, cyanosis,
and pain.

C. Distal venous obstructions are more likely than proximal venous obstructions to cause symptoms in the upper extremity.

D. All patients with symptomatic upper extremity venous thrombosis should receive fibrinolytic therapy.

Answer: B

DISCUSSION: Venous edema is much more common than lymphedema. Any of the signs and symptoms listed may be seen
in patients with venous obstruction. The proximal venous obstructions in the axillary, subclavian, and innominate veins are
more likely to cause symptoms in the upper extremities than is obstruction of the distal veins. The treatment for
symptomatic upper extremity venous thrombosis varies according to the cause of the thrombosis.

79. Which of the following statements about chronic mesenteric ischemia due to atherosclerosis is/are correct?
A. Postprandial pain in these patients is due to gastric hyperacidity and in most cases is relieved with H 2 blockers.

B. Men are more often affected than women.

C. Mesenteric endarterectomy is the surgical treatment of choice, since long-term patency rates are superior to mesenteric
bypass.

D. Arteriography is no longer necessary in these cases since noninvasive diagnosis can be established using duplex
ultrasound scanning.

E. Surgical treatment is indicated to prevent intestinal infarction in symptomatic patients.

Answer: E

DISCUSSION: The precise cause of postprandial pain in patients with chronic mesenteric ischemia is unknown.
Hyperacidity has been observed in some patients with this disease, and gastric pH may be reduced after successful
revascularization. No medical therapy, including H 2 blockers, has provided symptomatic relief. Unlike most syndromes of
ischemia due to atherosclerosis (coronary, cerebrovascular, peripheral vascular), chronic mesenteric ischemia occurs more
frequently in women. The long-term success rates for mesenteric bypass and mesenteric endarterectomy are equivalent;
either technique is acceptable. While duplex scanning is a useful noninvasive screening technique in these cases,
arteriography is required for definitive diagnosis and to plan revascularization. While prospective, natural history studies
have not been done, an increased risk of intestinal infarction is undeniable in these patients. When this occurs, patients
rarely survive. Elective surgical revascularization is indicated in symptomatic patients with severe mesenteric arterial
occlusive disease.

80. In patients with acute mesenteric ischemia due to mesenteric embolism, which of the following statements is/are
correct?

A. Patients often have a history of postprandial pain and weight loss.

B. The use of digitalis may be a predisposing factor to the acute event.

C. Thrombolytic therapy may be attempted in patients without signs of bowel infarction or gastrointestinal bleeding.

D. Arteriography usually reveals the embolus lodged at the orifice of the superior mesenteric artery.

E. At the time of exploration, the proximal jejunum is often viable and ischemia is most severe in the more distal small
bowel and colon.

Answer: CE

DISCUSSION: Patients who suffer mesenteric embolism usually have otherwise normal mesenteric arterial anatomy, and
ischemic symptoms are acute and profound. A history of chronic gastrointestinal symptoms is most often seen in patients
with mesenteric thrombosis. Although cardiac arrhythmias like atrial fibrillation (which predispose to mesenteric emboli)
may be treated with digitalis, this has no causative role in mesenteric embolism. Digitalis use has been associated with the
development of “nonocclusive” mesenteric ischemia. Thrombolytic therapy with streptokinase or urokinase has been used
successfully to treat mesenteric emboli; however, patients with any signs of local or generalized peritonitis should have
immediate surgical exploration. Mesenteric emboli usually lodge distally in the main superior mesenteric artery beyond the
first jejunal branches and the origin of the middle colic artery. The orifice of the superior mesenteric artery and the proximal
branches are normal, which explains the “jejunal sparing” often observed at the time of surgical exploration, even when
arteriography has not been performed.
81. Which of the following statements about angiotensin II is correct?

A. It is a decapeptide.

B. It is an enzyme found in high concentration in the pulmonary circulation.

C. It is a direct vasoconstrictor and stimulates aldosterone production.

D. It is a vasoconstrictor and inhibits aldosterone secretion.

Answer: C

DISCUSSION: Angiotensin II is an octapeptide cleaved from angiotensin I by angiotensin-converting enzyme. A direct


vasoconstrictor, it also stimulates the adrenal cortical production of aldosterone.

82. Which of the following statements about atherosclerotic obstruction of the renal arteries is true?

A. Lesions are usually short.

B. These lesions are usually found in the distal renal arteries, particularly just beyond branch points.

C. Ostial lesions are best treated with balloon angioplasty.

D. Lesions of this type are the second most common cause of renal artery stenosis.

Answer: A

DISCUSSION: Atherosclerotic renal artery lesions are usually short and found in the proximal renal arteries. Ostial lesions
in particular do not respond well to balloon angioplasty, and such lesions are responsible for more than two thirds of renal
artery obstructions.

83. Which of the following statements about the treatment of renal artery stenosis is/are true?

A. Though a significant cause of hypertension, renal artery stenosis seldom results in loss of renal function.

B. In patients with medically controlled renovascular hypertension there is no need to consider revascularization.

C. Balloon angioplasty is more effective in patients with atherosclerotic disease as compared with those with fibromuscular
disease.

D. In patients with severe atherosclerosis of the aorta, bypass from the splenic or hepatic arteries should be considered.

Answer: D

DISCUSSION: In a significant percentage of patients with renovascular hypertension there is loss of renal mass, which may
progress to end-stage renal insufficiency. Ostial lesions respond poorly to balloon angioplasty. If the aorta is severely
diseased bypass can be effective using either the hepatic or the splenic artery to supply blood to the kidney.
84. Which of the following statements about venous trauma is/are current?

A. All injured veins can be ligated without any immediate or long-term complications.

B. Attempted repair of injured veins is associated with increased incidence of thrombophlebitis.

C. Attempted repair of injured veins is associated with a high incidence of fatal pulmonary embolism.

D. Careful consideration should be made to repair the injured popliteal vein.

E. Anatomy of the venous system is more variable than anatomy of the arterial system.

Answer: DE

DISCUSSION: Many patients treated by ligation of injured veins suffer no immediate or long-term sequelae. Unfortunately,
there are patients who are difficult to identify initially who may have either acute venous hypertension or later suffer
chronic venous insufficiency following ligation of large-caliber veins in the lower extremities. The popliteal vein is
particularly important to repair if at all possible following injury, as are many of the larger-caliber central veins. Both
civilian and military reports have documented that there is not an increased incidence of thrombophlebitis and/or pulmonary
embolism associated with attempted repair of injured veins. In contrast to the arterial system, there are many more
variations in venous anatomy. This complicates the evaluation of patients with injured veins with examples of a possible
bifid popliteal vein and one to five channels comprising the superficial femoral vein.

85. All but one of the following statements is true. Which is not true?

A. Successful clinical repair of injured veins had been effected by the turn of the twentieth century.

B. Initial large experience in managing injured veins came from the battlefields of twentieth-century wars.

C. More than 50% of repaired injured veins thrombose.

D. Phlebography is useful in evaluating variable venous anatomy.

E. Repeated phlebography following attempted venous repair is useful in determining the success rate.

Answer: C

DISCUSSION: Murphy advocated venous repair as early as 1897, after others, including Schede (1882), noted success.
Extensive experience in managing large numbers of venous injuries has resulted from major armed conflicts of the twentieth
century. Phlebography is particularly valuable for determining venous anatomy and the success or failure of attempted
venous repair, and repeated phlebography over the first 72 hours has proven particularly informative. There is a high degree
of success in performing lateral suture of lacerations of large-caliber veins as demonstrated by follow-up phlebography.

86. Which of the following statements about injured veins is/are correct?

A. In contrast to the arterial system, it is more difficult to evaluate the patient for suspected venous trauma.

B. There is no simple method similar to palpating distal pulses following arterial repair to evaluate the status of attempted
venous repair.

C. Recanalization of initial thrombosis of attempted venous repair is more common than in the arterial system.
D. Doppler ultrasound—and more recently color-flow duplex—have been increasingly helpful in evaluating integrity of the
venous system.

E. Prevention of venous stasis is important in the immediate postoperative period after attempted venous repair.

Answer: ABCDE

DISCUSSION: There are significant differences between the arterial system and the lower-pressure venous system.
Presence or absence of distal pulses associated with possible arterial injuries provides a simple mechanism for evaluating
patients with potential injury, while there is no simple method of evaluating patients for potential venous injury. Doppler
ultrasound—and more recently color-flow duplex—have been increasingly helpful in evaluating the integrity of the venous
system. If thrombosis of an attempted venous repair occurs, there is a high probability that recanalization of the thrombus
will occur. This is in contrast to the arterial system, where recanalization of thrombosis of an attempted arterial repair is a
rare occurrence. Venous stasis must be prevented in the immediate postoperative period following attempted venous repair
to reduce the possibility of thrombosis.

Concerning the normal structure of blood vessels, the following is/are true:

a. In utero, hemangioblasts give rise to both vascular conduits and hematopoietic tissue

b. In development, smooth muscle tubes precede endothelium

c. After birth, growth of large vessels does not change the number of elastic and smooth muscle layers

d. Adventitia includes the external elastic lamina

Answer: a, c

The earliest vascular primordia in the embryo are isolated hemangioblasts that give rise to both vascular conduits and
hematopoietic tissue. Endothelial cells organize at sites of vessel development followed by mesenchymal cells that form the
outer layers. The number of elastic and smooth muscle layers remains constant after birth, although wall mass increases due
to proliferation. Adventitia lies outside the external elastic lamina

Among the theories of atherosclerosis, the following is/are true:

e. Fatty streaks in the aorta of children do not predict atherosclerosis or heart attacks

f. Aging induces non-atherosclerotic thickening of the intima

g. T-lymphocytes are present in atheromas

h. The reaction-to-injury hypothesis serves to explain the characteristic lipid accumulation

Answer: a, b, c

It is true that fatty streaks in the aorta and coronary arteries of children are found in populations without increased incidence
of atherosclerosis or heart attacks. Similarly, aging induces gradual thickening of the intima throughout the arterial tree
which is not atherosclerotic. A variety of leukocytes including T-lymphocytes are present in atheromas. The reaction-to-
injury hypothesis explains smooth muscle growth in atherogenesis but fails to provide an explanation for lipid accumulation
or the monoclonal nature of the atherosclerotic plaque.
Concerning in-vivo regulation of the anticoagulated state by endothelium, the following is/are true:

i. Heparan-ATIII inactivates only thrombin

j. Thrombomodulin serves only to bind thrombin

k. Production of von Willebrand factor (VWF) inactivates platelets

l. Endothelial cells can secrete tissue factor

Answer: d

Endothelium synthesizes heparan which, like heparin, increases the affinity of ATIII for thrombin which is inactivated along
with other serine proteases, including factors VII, IX and X. Thrombomodulin, in addition to binding thrombin, activates
protein C which binds with protein S to inactivate factor Va. On the procoagulant side, endothelial cells produce VWF
which binds platelets and are capable of secreting tissue factor.

Concerning medial and intimal thickening, the following is/are true:

m. Increase of wall mass is a consequence primarily of smooth muscle cell proliferation

n. Smooth muscle cells are normally quiescent at maturity

o. Transplanting a vein into the arterial circuit causes both endothelial and smooth muscle proliferation

p. Heparin can suppress both proliferation and migration of smooth muscle cells

Answer: a, b, d

Endothelial proliferation does not contribute to an increase in wall mass which is secondary to smooth muscle cell
proliferation. Smooth muscle cells are normally quiescent at maturity and their proliferation and migration are inhibited by
heparin. Transplanting a vein into the arterial circuit causes some endothelial cell loss and smooth muscle cell proliferation.

Concerning regulation of arterial luminal area, the following is/are true:

q. The major determinant of arterial diameter is blood pressure

r. Compensatory vasodilation occurs until more than 40% of area inside the internal elastic lamina is obstructed in coronary
arteries

s. Vasodilating nitric oxide is derived from adenosine

t. When endothelium is absent, thrombin causes vasoconstriction.

Answer: b, d

The major determinant of arterial diameter is blood velocity as demonstrated by post-stenotic dilation. This adaptation by
wall relaxation is limited to 40% of the area inside the internal elastic lamina in coronary arteries. The predominant
vasodilator, nitric oxide, is derived from arginine. When endothelium is absent, a number of vasodilators, including
thrombin, produce vasoconstriction.

Concerning regulation of smooth muscle cell growth, the following is/are true:

u. Serum derived from plasma has substantially more growth promoting activity than serum from whole blood.

v. Fibroblast growth factor (basic) is responsible for the first wave of proliferation in experimental arterial injury

w. The gene for platelet derived growth factor (PDGF) is nearly identical to the oncogene v-sis

x. Sympathectomy promotes the increase in DNA in the media of developing arteries and in hypertension

Answer: b, c

The observation that serum derived from whole blood has substantially more growth promoting activity than serum from
plasma led to the discovery of PDGF. Basic FGF is responsible for the first wave of proliferation in experimental carotid
artery injury. The gene for PDGF is nearly identical to the oncogene v-sis raising the possibility that wound healing and
malignant growth might have similarities of regulation. Sympathectomy inhibits the increase in DNA in the media of
developing arteries and in hypertension.

A 21-year-old man with premature arteriosclerosis and mental retardation is found to have homocystinuria. The following
is/are true:

y. Presence of mental retardation is atypical for homocystinemia

z. The specific enzyme deficiency responsible is homocysteine methyl transferase

aa. Arteriosclerotic plaques in this condition are atypically void of lipid deposition

bb. Homocysteine exists in plasma in three forms: protein bound, mixed and free

Answer: c, d

Homocystinuria reflects homocystinemia which is associated with ectopia lentis, mental retardation and thromboembolic
disorders as well as arteriosclerosis. There are three enzyme deficiencies known to cause the disorder as well as deficiencies
of the cofactors pyridoxine, cobalamin and folate. Lipid deposition in plaques is characteristically absent. Homocysteine
exists in plasma as the mixed disulfide homocysteine cysteine, as free and as protein bound homocysteine.

A 22-year-old male basketball player with back pain is found to have a dissecting aortic aneurysm. the follow is/are true:

cc. In Marfan’s syndrome, a disorder of type I collagen underlies the observed cystic medial necrosis

dd. In type IV Ehlers-Danlos syndrome, little or no type III collagen is produced and arterial rupture is likely

ee. In pseudoxanthoma elasticum, the medial elastic fibers are replaced by xanthoma cells which calcify

ff. In arteria magna syndrome, the media is devoid of elastic tissue and coronary artery disease is common
Answer: a, b, d

Cystic medial necrosis is associated with aortic dissection at an early age and can be due to Marfan’s syndrome with its
disorder of type I collagen or type IV Ehlers-Danlos where little or no type III collagen is produced. In pseudoxanthoma
elasticum, the medial elastic tissue is replaced by calcific deposits and there are xanthoma-like cutaneous papules. In arteria
magna syndrome, elastic tissue is absent in the media and associated coronary artery disease is common.

A 38-year-old male smoker with gangrenous changes in the toes of both feet has an arteriogram showing normal vessels to
the popliteal trifurcation and multiple occlusions distally in small vessels. The following is/are true:

gg. Hyperlipidemia, diabetes, and autoimmune disease must be ruled out to make the diagnosis of Buerger’s disease

hh. Plethysmographic evidence of digital obstruction in all four extremities with normal proximal vessels is sufficient
evidence for Buerger’s disease without arteriography

ii. The most important treatment for Buerger’s disease is regional surgical sympathectomy

jj. In contrast to the lower extremities, Buerger’s involvement of the upper extremities rarely leads to amputations

Answer: a, b, d

Buerger’s disease is a panarteritis associated with intraluminal thrombus in young male smokers. Diabetes, hyperlipidemia
and autoimmune diseases must be ruled out to fulfill the diagnostic criteria, but the diagnosis can be made
plethysmographically by evidence of small vessel obstruction in all four extremities. Cessation of all tobacco use is the most
important treatment. Management is conservative with only rare limb loss in the upper extremities as opposed to the lower.

A 42-year-old Asian woman with a history of recurrent deep venous thrombosis presents with a pulsatile mass in the
abdomen confirmed on ultrasound to be an abdominal aortic aneurysm. The following is/are true:

kk. History and findings suggest Kawasaki disease

ll. History and findings suggest polyarteritis nodosa

mm. Venous thrombosis is more common than arterial disease in these patients and the presence of an aneurysm portends a
high mortality rate

nn. Replacement of an aneurysm with a graft in Behcet’s disease is associated with recurrent aneurysms and thrombosis

Answer: c, d

Kawasaki disease is a disorder of infants and children with coronary aneurysms. Polyarteritis nodosa usually occurs in
males and the inflammatory process involves small and medium-sized muscular arteries. Behcet’s disease is a vasculitis that
produces venous thrombosis, and when arterial aneurysms are present, the mortality rate approaches 20%. Because of the
fragility of the arteries, recurrent aneurysm formation is likely.

A 32-year-old woman with severe hypertension is found to have renal artery changes as shown in Figue 69-1. The following
statement/s/ is/are true:

oo. Next to the renal artery, this process affects the carotid and coronary arteries most commonly
pp. In the most common variant of this disorder, the media is infiltrated with increased collagen, fibrous connective tissue
and glycosaminoglycans

qq. If similar disease is found in the carotid, it should be treated, even if asymptomatic

rr. Appropriate treatment includes percutaneous transluminal balloon angioplasty

Answer: b, d

Fibromuscular dysplasia is an abnormality of unknown etiology primarily affecting women (90%) and in the renal arteries.
The carotid and iliac arteries are the next most frequently affected. Medial fibroplasia is the most common pathology with
the pathological findings in answer b. Surgical treatment is indicated only for symptomatic stenoses since many
asymptomatic cases exist for which the natural history is unknown. In addition to surgical procedures, balloon angioplasty
of main renal artery lesions is acceptable treatment.

Ten years after irradiation of the neck for a tonsillar carcinoma, a 59-year-old woman is found to have symptomatic carotid
artery disease. Arteriogram shows a 70% irregular stenotic lesion. The following is/are true:

ss. Replacement of the artery should be planned due to radiation induced arterial injury

tt. The pathology is most likely to be an inflammatory reaction with endothelial sloughing and thrombosis

uu. If atherosclerotic disease is found, the plaque will be no different than nonirradiated plaques

vv. The patient should be managed medically because of the radiation arterial injury

Answer: c

Radiation-induced arterial injury produces three types of injury, the earliest post-treatment consisting of inflammatory
reaction with endothelial slough and thrombosis. Later, there may be fibrotic changes in the wall producing stenosis or
accelerated atherosclerosis. The latter lends itself to standard endarterectomy and the plaque is indistinguishable from non-
irradiated plaque.

A 23-year-old woman with fever, myalgia and anorexia presents with hypertension and a cool, ischemic left arm.
Angiography shows multiple stenoses of the subclavian and renal arteries. The following is/are true:

ww. Coronary angiography is indicated with high likelihood of finding coronary disease

xx. Endarterectomy of the lesions would be preferred to transluminal angioplasty

yy. The presentation is more suggestive of Behcet’s disease than Takayasu arteritis

zz. Preferred management consists of corticosteroids

Answer: d

The presentation is most suggestive of Takayasu arteritis which tends not to involve the coronary arteries. A variety of
operations have been used in these patients but endarterectomy is not recommended because of a high incidence of early
failure. The preferred management is corticosterioids.
A 58-year-old woman presents with a history of severe headache, visual field loss and a transient myalgia involving the
back and shoulders. The following is/are true:

aaa. A tender, nodular temporal artery would indicate a picture compatible with temporal arteritis

bbb. The presentation is most compatible with giant cell arteritis

ccc. Steroids should be avoided if an operation is planned

Angiography is most likely to show irregular surface stenosis

Answer: a, b

The presentation is typical for a patient with temporal arteritis, which is a form of systemic giant cell arteritis. It is
characterized by chronic inflammation of the aorta and its major branches. Corticosteroid therapy is indicated because of its
success in relieving symptoms whether or not an operation is planned. Angiographic findings in this condition show smooth
rather than irregular surface stenoses.

Concerning the fibrinolytic system the following is/are true:

ddd. Plasminogen is an a-globulin

eee. Fibrin but not fibrinogen is lysed by plasmin

fff. The main inhibitor of plasmin is a2-macroglobulin

ggg. TAP is activated during fibrin bonding to plasminogen

Answer: d

Plasminogen is a b-globulin that is converted to plasmin by a number of activators. Plasmin lyses and destroys both fibrin
and fibrinogen. The main inhibitor of plasmin is a2-antiplasmin; a2-macroglobulin, ATIII and antitrypsin are less important
inhibitors. During fibrin formation, both plasminogen and TAP bind to it specifically, and TAP is activated.

Concerning platelet function in vascular disease, the following is/are true:

hhh. Platelet aggregation is the initial step in thrombogenesis when subendothelial structures are exposed.

iii. Non-homogeneous distribution of platelets towards the vessel wall is enhanced at increasing shear rates

jjj. The platelet glycoprotein (GP) which is the principal collagen receptor is GPIb-IX

kkk. Fibrinogen binding to GPIIb-IIIa is a prerequisite for all platelet aggregation

Answer: b, d

Platelet adhesion is the initial step in thrombogenesis when subendothelial structures are exposed. Platelet adhesion and
their non-homogeneous distribution toward the vessel wall are enhanced at increasing shear rates. The platelet GP which is
the principal collagen receptor is GPIa-IIa, GPIb-IX is the one which binds to von Willebrand factor. Fibrinogen binding to
GPIIb-IIIa is a prerequisite for all platelet aggregation.
Concerning the inhibition of intravascular coagulation, the following is/are true:

lll. Heparin accelerates the effects of ATIII to a greater extent than native heparin sulfate

mmm. Protein C but not protein S is vitamin K dependent

nnn. Protein Ca stimulates the release of TAP from endothelial cells

ooo. ATIII neutralizes factors Xa, IXa and IIa

Answer: a, c, d

The rate of inhibition of activated coagulation factors by ATIII is dramatically accelerated by heparin, and to a lesser extent
by heparin sulfate. Both protein C and protein S are vitamin K dependent, and protein Ca does stimulate the release of TAP
from endothelial cells. ATIII neutralizes factors Xa, IXa and IIa (thrombin).

Concerning hypercoagulable syndromes, the following is/are true:

ppp. Acquired hypercoagulable states are more common than congenital disorders

qqq. Fatal neonatal thrombosis is associated with severe dysfibrinogenemia

rrr. Heparin associated thrombocytopenia is due to an antibody that attaches to the platelet Fc receptor

sss. The lupus anticoagulant induces a hemorrhagic diathesis

Answer: a, c

Acquired hypercoagulable states are much more common than congenital disorders at our present level of understanding.
Fatal neonatal thrombosis is associated with homozygous ATIII deficiency. Heparin associated thrombocytopenia is
associated with an IgG antibody that attaches at the Fc receptor and triggers platelet secretion and aggregation. The lupus
anticoagulant is an antiphospholipid antibody that induces a thrombotic rather than a hemorrhagic disorder.

Concerning the treatment of thrombotic vascular disease, the following is/are true:

ttt. Aspirin is successful in preventing venous as arterial thromboembolism

uuu. Dipyridamole enhances the ability of aspirin to prevent arterial thrombosis

vvv. Ticlopidine is more effective than aspirin in patients with cerebrovascular disease

www. Ticlopidine prevents fibrinogen binding to the GPIIb-IIIa receptor complex

Answer: c, d

Aspirin and other anti-platelet drugs are effective in preventing thrombosis in arterial but not venous thrombotic disorders.
Dipyridamole has been found to be ineffective in rigorous clinical trials. Ticlopidine is more effective than aspirin in
patients with cerebrovascular disease, and serves to prevent fibrinogen binding to the platelet GPIIb-IIIa complex.
An 82-year-old man with a long history of coronary and peripheral vascular disease presents with an acutely ischemic right
lower extremity. The following is/are true:

xxx. The first step in management should be an arteriogram

yyy. If intractable congestive heart failure is present, non-operative treatment with heparin would be appropriate

zzz. If prolonged ischemia has occurred, reperfusion should be accompanied by sodium bicarbonate

aaaa. Regardless of the period of ischemia, fasciotomy should be based on the findings postoperative

Answer: b

The first step in the management of acute limb ischemia in any patient is heparin anticoagulation. If intractable heart failure
is present, heparin treatment would be appropriate without operation. If prolonged ischemia has occurred, the venting of the
first 3–500 ml of venous outflow will allow conservation of RBCs and avoidance of the consequences of high levels of
potassium. If the duration of ischemic has exceeded 4 hours, a 4 compartment fasciotomy should be performed at the time
of restoration of perfusion.

Two days following coronary angiography and angioplasty, a 47-year-old male diabetic develops painful blue toes on both
feet. The following is/are true:

bbbb. It is very unlikely that there is any connection between the catheterization and the extremity problem

cccc. The appropriate treatment is vasodilators and an antiplatelet agent

dddd. If both superficial femoral arteries are obstructed, the most likely etiology is in-situ microvascular thrombosis

eeee. If renal failure or pancreatitis develops, the outlook for long term survival is very poor

Answer: d

Artheroembolism results from plaque rupture or manipulation at catheterization and is much more frequent after
catheterization than suspected clinically. Since repetitive events and additional complications are expected, prompt
arteriography should be performed to delineate the possible site of origin which is then excised, endarterectomized or
bypassed as the only effective treatment. Since plaque debris is very small, it can readily pass through collateral vessels to
lodge in arterioles, and major vascular occlusion is no barrier. The kidney is the most common organ affected and if renal
failure or pancreatitis develops as a sign of generalized atheroembolism, the outlook is poor with life expectancy measured
in months.

A 39-year-old woman with embolic occlusion of an iliac artery is subject to an operating room delay before perfusion can be
restored. The following is/are true:

ffff. Ischemia for longer than 3 hours will result in muscle fiber autolysis

gggg. Earliest ultrastructural changes of ischemia in muscle include mitochondrial swelling and loss of glycogen granules

hhhh. Phosphocreatine mediated rephosphorylation of ADP occurs for about 3 hours after ischemia
iiii. Capillary thrombosis is the most likely explanation for the “no-reflow” phenomenon

Answer: b, c

Skeletal muscle can tolerate ischemia by anaerobic glycolysis for up to 6 hours. The earliest ultrastructural changes in
ischemic muscle include mitochondrial swelling and loss of glycogen granules. During ischemia, ATP levels are maintained
by phosphocreatine mediated rephosphorylation of ADP until phosphocreatine levels are exhausted after about 3 hours. Of
the possible causes of the “no-reflow” phenomenon, capillary obstruction by leukocytes is more likely than capillary
thrombosis.

A 70-year-old man presents with sudden pain and ischemic changes in his left leg. An arterial embolus is suspected. The
following is/are true:

jjjj. The most likely source of an arterial embolus is from intracardiac thrombus on a previous MI

kkkk. If atrial fibrillation (AF) is present, it is known that chronic AF is less likely to produce embolism than paroxysmal
AF

llll. Currently, the most common cause of AF is ischemic rather than rheumatic heart disease

mmmm. Aspirin is more effective than coumadin in AF for reducing risk of stroke and cardiovascular mortality

Answer: c

Approximately 80–90% of arterial emboli originate in the heart, and 2/3 are secondary to AF. Chronic AF carries an annual
risk of 3–6% of significant embolic complications while paroxysmal AF has a lower risk. Traditionally, rheumatic heart
disease was the most common cause of chronic AF, but with its decline, ischemic heart disease has become the most
common cause. Drug therapy in AF will reduce the risk of stroke, but aspirin is less effective in this regard than coumadin.

In discussing risk and outcomes of the patient in the previous question, the following is/are true:

nnnn. If renal failure occurs, the mortality rate is about 50%

oooo. If arterial embolism is confirmed, the patient should receive lifelong anticoagulation

pppp. Postoperative amputation is unlikely if the embolectomy is successful

qqqq. Postoperative death from pulmonary embolism is unlikely

Answer: a, b

There has been only modest improvement in the mortality and morbidity after arterial embolectomy in the past 40 years,
and if renal failure occurs, the mortality rate is about 50%. Recurrence of arterial embolism without anticoagulation occurs
in 28–45% of patients and justifies prolonged anticoagulation which reduces the incidence of recurrent embolism. In
addition to a high postoperative mortality rate, amputations are required in about 15% of patients. Pulmonary embolism is
the 2nd most common cause of death after embolectomy, reflecting the incidence of DVT in 7–27% of patients after arterial
embolectomy.

The clinical manifestations of the patient in the previous question would include:
rrrr. Loss of sensation to deep pain as one of the earliest signs

ssss. Paresthesia would be noted in a classical dermatome distribution

tttt. Early pallor is due to both diminished skin blood flow and reflex vasoconstriction

uuuu. Involuntary muscle contraction indicates that restored flow cannot save the extremity

Answer: c, d

The earliest limb changes in ischemia are in sensory nerves with small nerve fibers having increased sensitivity resulting in
loss of sensation to light touch. Sensation to deep pain, pressure and temperature are preserved until late. The paresthesias
that occur are in a glove or stocking-like distribution rather than by dermatome. Early pallor is due to decreased blood flow
as well as reflex vasoconstriction. Among signs of irreversible limb ischemia are complete anesthesia and involuntary
muscle contraction.

A 51-year-old man with a history of transmural MI one month ago presents with sudden occlusion of his abdominal aorta.
The following is/are true:

vvvv. Most likely location of the MI is anterolateral

wwww. The vast majority of emboli occur within 6 weeks of the occurrence of the MI

xxxx. Occurrence of arterial embolism does not affect the overall mortality

yyyy. Heparin can reduce the incidence of embolism after MI

Answer: a, b, d

Acute MI with endocardial thrombus is the second most common cause of arterial embolism most commonly within 6
weeks, and the most typical location postmortem is anterolateral. Arterial embolism after MI is associated with an increase
in mortality rate. Heparin following acute MI has been shown to reduce the incidence of systemic arterial embolism.

A 67-year-old man with acute popliteal arterial embolism has a negative cardiac echo for source of the thrombus. The
following is/are true:

zzzz. Most likely non-cardiac source is a thoracic aortic aneurysm

aaaaa. Embolism is more common from femoral than popliteal arterial aneurysms

bbbbb. Emboli from popliteal aneurysms are often clinically silent

ccccc. Embolism is rare from subclavian artery aneurysms

Answer: c

The most likely non-cardiac source of an arterial embolism is an infrarenal abdominal aortic aneurysm. Arterial embolism is
more frequent from popliteal than femoral aneurysms, and these embolic events are often clinically silent. Subclavian artery
aneurysms give rise to peripheral embolism in up to 33% of patients.
In regards to the previous case, the following statement(s) concerning the distribution of arterial emboli is/are true:

ddddd. Change in arterial diameter is a more important determinant of embolic site than flow rate

eeeee. Aortic valvular disease is more often associated with cerebral embolism than mitral valve disease

fffff. Among embolic sites, renal emboli are least detected clinically

ggggg. The most common site for an arterial embolus is the aortic bifurcation

Answer: a, c

Most arterial emboli lodge at bifurcations, where there is a sudden change in arterial diameter. Flow rate does not correlate
with sites of embolism. It is mitral valve disease with associated atrial fibrillation that is most frequently associated with
cerebral embolism. The discrepancy between clinical and autopsy evidence of embolism is significant for renal emboli
where the clinical diagnosis is made in less than 1% of cases. The most common site for an arterial embolus is the common
femoral artery.

Concerning cellular metabolism, the following is/are true:

hhhhh. Anaerobic metabolism is about half as efficient in energy production as normoxic metabolism.

iiiii. Loss of cellular Ca++ as ion pumping fails activates phospholipase

jjjjj. Cellular swelling serves to protect the cell

kkkkk. Ketone as an alternative energy source has been shown to be beneficial in ischemia

Answer: b, d

The shift from normoxic to aerobic metabolism results in a dramatic loss of efficiency from 38 mol ATP to a net of 2 mol
per molecule of glucose, a 94% reduction. With loss of ion pumping, free Ca++ accumulates and triggers phospholipase
activation. Cellular swelling and interstitial edema increase diffusion distances further compromising oxygen and substrate
delivery. In a number of experimental conditions, ketone as an alternative to glucose has proved beneficial.

A 62-year-old woman with embolic femoral artery occlusion is facing a delay before circulation can be restored. To
minimize ischemic injury, the following is/are true:

lllll. Increased oxygen saturation is beneficial

mmmmm. Injury will continue during reperfusion

nnnnn. Low collateral flow is more harmful than no flow

ooooo. Concern regarding reperfusion injury should not delay revascularization

Answer: b, c, d
A significant part of ischemic injury occurs during reperfusion from generation of oxygen free radicals, but this should not
delay efforts to provide revascularization. Excess oxygen levels are harmful rather than helpful and low collateral flow has
been demonstrated experimentally to be more harmful than no flow.

Concerning tolerance to tissue ischemia, the following factor(s) are important variables determining organ failure:

ppppp. Resting metabolic rate

qqqqq. Anaerobic glycolysis

rrrrr. Autonomic nerve supply

sssss. Efficiency of existing collaterals

Answer: a ,b, d

Although the specific tolerance of various tissues and organs to ischemic injury is variable, the resting metabolic rate,
anaerobic glycolysis and efficiency of existing collaterals are important variables. Autonomic nerve supply does not play a
major role.

Concerning ischemic cellular injury, the following is/are true:

ttttt. Prolonged hypoxic metabolism is inefficient but not harmful to cells

uuuuu. Microvascular endothelium is a significant source of xanthine oxidase

vvvvv. The “no-reflow” phenomenon is due to arteriolar spasm

wwwww. Endothelial cells lack the defense system for oxidative damage

Answer: b, d

Prolonged hypoxic metabolism is not only inefficient but the accumulation of lactic acid produces direct injury to
organelles, alters enzyme activity, and enhances cytokine production. The microvascular endothelium reacts to ischemia by
production of xanthine oxidase, but lacks the defense system for oxidative damage. The “no-reflow” phenomenon is
associated with an increased number of WBCs adherent to the luminal surface of the microcirculation and cellular swelling.

Concerning remote effects of localized ischemia, the following is/are true:

xxxxx. Lower torso and limb ischemia induce pulmonary injury via TXA2 and WBC effects

yyyyy. ATP degradation produces adenosine which induces systemic hypertension

zzzzz. Adenosine also causes renal and pulmonary vasoconstriction

aaaaaa. Oxygen radicals are continuously produced by normal metabolic processes

Answer: a, c, d
Both experimental and clinical studies have linked WBC dependent pulmonary injury mediated by TXA2. Adenosine
release from ATP degradation contributes to systemic hypotension since it is a vasodilator in most vascular beds but a
vasoconstrictor in the renal and pulmonary vasculature. Oxygen radicals are continuously produced by normal metabolic
processes and handled by quenching and scavenging systems.

Which of the following is/are true regarding exercise testing and reactive hyperemia in patients with peripheral vascular
occlusive disease?

bbbbbb. Normal individuals walk on a treadmill at 2 m.p.h. at a 10% grade without experiencing leg pain and ankle pressure
remains unchanged after exercise

cccccc. In patients with arterial obstruction, pain usually forces cessation of walking after 2–3 minutes and the ankle
pressure measured immediately after exercise is diminished

dddddd. The time required for pressure to return to baseline is usually 2–3 minutes

eeeeee. Reactive hyperemia may be used as a substitute for treadmill exercise

Answer: a, b, d

Normal subjects walk without pain and do not drop their ankle pressure after exercise. In patients with arterial obstruction
there is a drop in ankle pressure after exercise and the severity of that drop is roughly proportional to the severity of the
occlusive process. Likewise the time for pressure to return to pre-exercise levels is proportional to the severity of the
occlusive process and may exceed 20 minutes in severely diseased extremities. The reactive hyperemia test is quite sensitive
in patients who cannot exercise, and may be used as a substitute for treadmill exercise.

Which of the following is/are true with respect to transcutaneous PO2 (TcPO2) measurements?

ffffff. TcPO2 levels provide an index of the adequacy of tissue perfusion and depends on the quantity of oxygen delivered
and that extracted to meet metabolic demands

gggggg. Extremity TcPO2levels are typically normalized to a well perfused area, such as the infraclavicular skin

hhhhhh. TcPO2levels average about 60 mmHg in normal limbs

iiiiii. Patients with limb threatening ischemia usually have values less than 20 mmHg and may approach 0

Answer: a, b, c, d

Transcutaneous PO2 levels provide an index of the adequacy of tissue perfusion. They depend on the quantity of oxygen
delivered by the blood and that extracted to meet metabolic demands. Because oxygen supply is a function of the arterial
PO2, cardiac output and age peripheral measurements must be compared with levels from a well perfused central area, such
as the infraclavicular skin. In normal limbs transcutaneous oxygen levels average 60 mmHg or 90% of the infraclavicular
value. Many claudicants have resting values in the normal range, whereas in patients with limb threatened ischemia the
values are usually less than 20 mmHg and many be zero.

Which of the following is/are appropriate candidates for exercise testing?

jjjjjj. The patient with symptoms of intermittent claudication but normal resting ankle brachial indices
kkkkkk. The patient with rest pain, nonhealing ulcers or gangrene

llllll. If the resting ankle pressure is below 30–40 mmHg

mmmmmm. The patient with blue toe syndrome and readily palpable pedal pulses

Answer: a

When the patient’s presenting complaints are compatible with claudication and the ABI is normal or nearly so treadmill
exercise is quite helpful in unmasking significant arterial occlusive disease. In patients with obvious rest pain, nonhealing
ulcers or gangrene the diagnosis of significant peripheral vascular disease is obvious, likewise for the patient’s whose
resting ankle blood pressure is below 30 or 40 mmHg. Finally, in patients with atheroemboli and palpable pedal pulses, toe
brachial indices may be helpful but exercise testing is not. Search for the embologenic source is more appropriate.

Which of the following statements is/are true regarding the use of duplex scanning as a means to follow and monitor bypass
grafts?

nnnnnn. Duplex scanning is accurate and cost effective

oooooo. A localized increase in systolic velocity greater than 25% compared to adjacent segments in the graft identifies a
diameter reduction of at least 50%

pppppp. Peak systolic velocities should be less than 40 cm/sec throughout the graft

qqqqqq. Arterial venous fistulas associated with in situ bypass grafts are difficult to detect with a duplex scanner

Answer: a

Duplex scanning is an excellent way to monitor bypass grafts. It can detect graft threatening defects before the patient
becomes symptomatic and before the ankle pressure begins to drop. A localized increase in systolic velocity greater than
100% of that of the adjacent graft identifies a diameter reduction of more than 50%. Peak systolic velocities less than 40
cm/sec are an ominous sign of markedly reduced flow. Arterial venous fistulas are regularly recognized by their pattern of
localized flow disturbances with increased velocities at the site of the fistula and immediately proximal to the fistula and
concomitant decreased velocities just below the fistula.

Which of the following is/are true with respect to ankle blood pressure and ankle brachial index (ABI)?

rrrrrr. An ABI of less than 0.92 almost always indicates hemodynamically significant arterial disease

ssssss. Claudicants have a wide range of ABIs with average values of 0.6 +/– 0.15

tttttt. In limbs with rest pain the mean ABI is typically 0.25 +/– 0.13

uuuuuu. In limbs with impending gangrene ABIs seldom exceed 0.25 and average about 0.05 +/– 0.08

Answer: a, b, c, d

The normal person’s resting ankle brachial blood pressure usually exceeds brachial blood pressure. In persons with stenotic
lesions that do not reduce the diameter of the arterial lumen by more than 50% there may be no change in resting ankle
brachial blood pressure. Absolute ankle pressures of less than 40 mmHg always indicate severe arterial compromise
regardless of the ABI. The ranges given above have been empirically derived from assessing large numbers of patients.
When the arteries are incompressable secondary to calcification spuriously high ankle pressures may be obtained.

Which of the following is/are true regarding Doppler assessment?

vvvvvv. Conventional Dopplers admit an ultrasonic beam in the frequency of 2–10 MHz

wwwwww. The sound frequency changes in inverse proportion to the velocity of the moving particles (red blood cells) and
the cosign of the angle of insonnation

xxxxxx. The frequency shift is not audible

yyyyyy. More information can be obtained by spectral analysis

Answer: a, d

Of all of the diagnostic methods used in the noninvasive lab, Doppler ultrasound has the most utility. In clinical usage
Doppler instruments emits an ultrasonic beam at 2–10 MHz. The frequency of the sound is changed in proportion to the
velocity of moving particles (red blood cells) and the cosign of the angle of insonnation that the beam makes with the
velocity vector. The frequency shift is in the audible range and listening with a pocket Doppler provides a quick and simple
method of assessing blood flow. Spectral analysis with frequency on the vertical axis, time on the horizontal axis and
amplitude changes indicated by an increasing intensity on the grey scale provides considerably more information,
particularly with respect to flow disturbance which produce a broadening of the normally narrow bands of frequencies
which parallel the flow envelope.

Which of the following is/are true with respect to assessment of the carotid circulation?

zzzzzz. The external carotid artery flow pattern resembles those obtained from peripheral arteries

aaaaaaa. The internal carotid artery maintains forward flow throughout the cardiac cycle

bbbbbbb. Peak systolic velocity exceeding 200 cm/sec suggests a stenosis greater than 50%

ccccccc. An end diastolic velocity greater than 120 cm/sec suggests a stenosis greater than 80%

Answer: a, b, d

The brain is a low resistance vascular bed and internal carotid artery blood flow is positive throughout the cardiac cycle.
The external carotid artery which supplies the muscles of the face and neck resembles peripheral arteries. A great deal of
attention has focused on determining criteria for estimating stenosis severity. A peak systolic velocity exceeding 130 cm/sec
suggests a stenosis of greater than 50% and end diastolic velocity greater than 120 cm/sec suggests a stenosis greater than
80%. An internal carotid/common carotid peak systolic velocity ratio of 4.0 or more and an end diastolic velocity of 100
cm/sec have been proposed as criteria for identifying diameter stenosis of 70%.

Which of the following is/are true regarding the assessment of renal artery obstruction with duplex scanning?

ddddddd. There is no flow reversal in early diastole in the renal artery

eeeeeee. Renal artery to aortic peak systolic velocity ratios that exceed 3.5 indicates the presence of a 60% diameter stenosis
fffffff. Duplex scanning regularly identifies accessory renal arteries

ggggggg. Duplex scanning cannot be recommended as a means for monitoring renal artery reconstruction

Answer: a, b

The kidneys represent a low resistance vascular bed, therefore flow in renal arteries is positive throughout the cardiac cycle
and there is no flow reversal in early diastole as is seen during the assessment of peripheral arteries. A ratio of renal artery to
aortic peak systolic flow that exceeds 3.5 indicates the presence of a 60% diameter stenosis. Sensitivities greater than 80%
and specificities greater than 90% have been reported using these criteria to predict significant stenoses of main renal
arteries. Unfortunately duplex scanning often fails to detect accessory renal arteries or segmental branch disease. It is quite
reasonable as a screening technique for patients with suspected renal artery hypertension and as an accurate method for
monitoring renal artery reconstruction.

Which of the following is/are true regarding normal peripheral arterial flow waves?

hhhhhhh. Flow is antegrade and rapidly accelerated in early systole

iiiiiii. There is a rapid deceleration phase during which velocities fall to 0

jjjjjjj. A short period of flow reversal occurs in early diastole

kkkkkkk. Low level forward flow continues throughout the remainder of diastole

Answer: a, b, c, d

The charact

1. The “father of experimental surgery” who performed pioneering research, including several transplantation procedures,
was:
A. Homer, the Greek who described the Chimaera in his Iliad.
B. Gasparo Tagliacozzi, the Italian who described a method of reconstructing the nose.
C. John Hunter, the Scot who performed autografts and xenografts.
D. Emrick Ullmann, the Austrian who performed the first successful renal allograft.
E. Alexis Carrel, the Franco-American who described a successful technique for vascular anastomosis.
Answer: C

DISCUSSION: All of the descriptions are correct and represent important contributions to the history of transplantation.
However, the Scottish surgeon John Hunter (1728–1793), is rightfully known as the father of experimental surgery because
of his pioneering research. Several of his experimental procedures involved transplantation, including autografting of a
cock's spur to its comb and xenografting of a human tooth to the comb of a cock.

2. Transplantation terminology contains terms to describe the relationship of the graft donor to the graft recipient. Historical
terms such as “homograft” and “heterograft” have been replaced by less ambiguous terms. The correct modern terminology
for a graft between genetically nonidentical members of the same species is:
A. Allogeneic graft.
B. Autogeneic graft.
C. Isogeneic graft.
D. Syngeneic graft.
E. Xenogeneic graft.

Answer: A
3. The modern era of clinical organ transplantation began with the advent of chemical immunosuppression. The important
drug discovery that produced the initial success of cadaveric transplantation was:
A. Cyclophosphamide.
B. Azathioprine.
C. Cyclosporine.
D. Antilymphocyte serum.
E. Monoclonal antibody OKT3.
Answer: B

DISCUSSION: All of the listed drugs have immunosuppressive activity that has proved useful in transplant recipients.
However, the discovery in 1959 by Schwartz and Dameshek that 6-mercaptopurine blocked antibody production and the
subsequent creation by Hitchings in 1961 of its safe, convenient imidazole derivative named azathioprine produced the first
consistently effective immunosuppression for successful cadaveric renal transplantation.

4. Which of the following statements correctly characterize the genetic basis of histocompatibility?
A. Histocompatibility is determined by a series of genes inherited as a complex and subject to the mendelian rules that
characterize recessive traits.
B. Histocompatibility depends in part on the inheritance of histocompatibility genes and in part on the inheritance of T-cell
receptor genes.
C. Major histocompatibility genes are polymorphic.
D. Histocompatibility genes are independently segregating and co-dominant.
E. Histocompatibility is learned.
Answer: CDE

DISCUSSION: Histocompatibility refers to the genetic determinants of graft rejection. The determinants of overwhelming
importance consist of a series of histocompatibility genes that segregate independently during meiosis. Each gene has
multiple, dominant alleles. Histocompatibility genes and the proteins they encode are highly polymorphic (i.e., they exist in
multiple forms).

5. The major histocompatibility complex (MHC) includes genes that encode which of the following proteins?
A. HLA-A.
B. HLA-DR.
C. TAP-1.
D. 21-Hydroxylase.
E. HLA-L.
Answer: ABCD

DISCUSSION: The major histocompatibility complex (MHC) includes genes encoding histocompatibility antigens, some
other proteins, and a number of pseudogenes that do not encode proteins. The class I region encodes more than 15 genes,
including the classical transplant genes A, B, and C as well as HLA-E, F, and G and four pseudogenes, H, J, K, and L. The
class II region contains more than 25 genes, including those for the transplantation antigens HLA-DR, DQ, and DP. The
region also includes two alpha genes, DMA and DNA, and two beta genes, DMB and DOB, genes for the low-molecular-
weight proteins (LMPs) LMP2 and LMP3 and for the transporter molecules TAP1 and TAP2. The class III region, lying
between class II and class I, contains more than 30 genes, among which are the genes encoding the complement components
factor B, C2, and both C4 molecules, both tumor necrosis factor genes alpha and beta, and the heat shock proteins Hsp 1H
and Hsp 70 2, and 21-hydroxylase.

6. Which of the following distinguish MHC class I from MHC class II antigens?
A. MHC class I and class II antigens are encoded in different regions of the MHC complex.
B. MHC class I antigens are expressed on specialized antigen-presenting cells, whereas MHC class II antigens are
expressed on all cells.
C. MHC class I and class II are members of different supergene families.
D. MHC class I are considered to be the major histocompatibility antigens and MHC class II the minor histocompatibility
antigens.
E. MHC class I is recognized by the CD8 glycoprotein, whereas MHC class II is recognized by the CD4 glycoprotein.
Answer: AE

DISCUSSION: MHC class I and class II antigens are encoded by genes in different regions of the MHC. The genes and the
proteins they encode are homologous to immunoglobulins and thus are members of the immunoglobulin supergene family.
MHC class I antigens are expressed on the surface of all cells, whereas MHC class II antigens are largely restricted in
expression to antigen-presenting cells and endothelial cells. Both MHC class I and class II antigens are major
histocompatibility antigens because their incompatibility in the donor and recipient can lead to very rapid and vigorous
rejection of an allograft. The T cells that have antigen receptors specific for MHC class I plus peptide express CD8, a co-
receptor that binds to the MHC class I molecules. The T cells that have antigen preceptors specific for MHC class II plus
peptide express CD4, a co-receptor that binds to MHC class II molecules.

7. Which of the following characterize the role of the major histocompatibility antigens in immune responses?
A. The major histocompatibility antigens are critical in antigen processing and presentation.
B. Major histocompatibility antigens contribute to the maturation of T cells in the thymus.
C. T cells recognize only foreign antigens that are complexed with major histocompatibility antigens.
D. Expression of major histocompatibility antigens is increased in inflammation.
E. Recognition of major histocompatibility antigens is critical to the development of tolerance.
Answer: ABCDE

DISCUSSION: Once thought to be solely markers of individuality, MHC antigens are crucial to cell-mediated immune
responses. Foreign antigens taken up by antigen-presenting cells are degraded and then become complexed with MHC
molecules and expressed on the cell surface, and these events are enhanced in inflammation. Since T cells recognize only
foreign antigens expressed as peptides in association with MHC antigens, the possibility for recognition is increased as a
consequence of inflammation. Since T cells recognize only antigens expressed in association with MHC antigens,
recognition of these antigens is critical to the development of tolerance to “self.”

8. The unusual intensity of alloimmune responses reflects which of the following characteristics?
A. The presence of a peptide-binding groove in the MHC molecule.
B. Recognition of the native structure of allogeneic MHC molecules.
C. The high frequency of T cells able to recognize directly allogeneic MHC antigens.
D. Stimulation of many T-cell receptors during the interaction of a T cell with an antigen-presenting cell.
E. The high frequency of antigen-presenting cells able to be recognized by T cells.
Answer: BCDE

DISCUSSION: Allotransplantation evokes an unusually intense and rapid cellular immune response. In contrast to
conventional cellular immune responses, in which foreign antigens are recognized only as peptides in the groove of self
MHC antigens, allogeneic MHC antigens are recognized directly as native proteins on the surface of allogeneic antigen-
presenting cells. Thus, a large fraction of antigen-presenting cells is able to present alloantigen in this fashion, and a large
fraction (up to 10%) of T cells is able to respond.

9. Which of the following statements correctly characterize the role of histocompatibility typing in transplantation?
A. Histocompatibility typing must be carried out before transplantation can safely be undertaken.
B. The “rules” of histocompatibility were established shortly after the advent of immunosuppressive therapy made
transplantation feasible.
C. Histocompatibility typing may involve serologic, cellular, and molecular procedures for typing.
D. The role of histocompatibility matching in transplantation is controversial.
E. The cross-match test is carried out to determine whether a potential graft recipient has antibodies against the donor.
Answer: CDE

DISCUSSION: The concept of histocompatibility and the rules governing the susceptibility to rejection were deduced early
in this century by such investigators as Jensen, Little, and Tyzzer, who were interested in the inherited resistance or
susceptibility to transplanted tumors. The application of histocompatibility to clinical transplantation, however, had to await
the advent of immunosuppressive therapy. Despite the practice of organ transplantation for more than 30 years, the role of
histocompatibility typing in transplantation is controversial. Although grafts between HLA-matched donors and recipients
exhibit better survival than HLA-mismatched grafts, matching is not routinely performed before transplantation of the heart
or liver, and the outcome of these grafts may be very good. Histocompatibility typing involves the use of a variety of
techniques—serologic, cellular, and molecular—to identify the antigens carried by the donor and the recipient. In addition
to formal typing, the recipient is tested via cross-match for antibodies against the donor.

10. Activation of T cells requires:


A. Stimulation of the antigen receptor.
B. Stimulation of the MHC antigen.
C. Co-stimulation.
D. Anergy.
E. CD3.
Answer: ACE

DISCUSSION: The activation of T cells generally involves the delivery of two types of signals. One signal is initiated when
the T-cell antigen receptor binds in a cognate manner to an MHC antigen bearing an antigenic peptide expressed on the
surface of an antigen-presenting cell. This interaction is enhanced by the co-ordinate binding of CD4 or CD8 to the MHC
antigen complex. This interaction initiates signaling through CD3, as well as through CD4 or CD8, both of which are
associated with tyrosine kinases. Full activation of the T cell also requires the delivery of “co-stimulatory” signals. These
signals may arise through the interaction of CD28 expressed by the T cell with B7-1 or B7-2 expressed on antigen-
presenting cells. If only the T-cell antigen receptor is stimulated (and co-stimulation is not provided) the T-cell becomes
anergic, that is, resistant to further stimulation. Anergy may be an important mechanism contributing to tolerance.

11. Which of the following statements characterize the biology of allotransplantation?


A. The rejection response is systemic.
B. The rejection response is learned.
C. The rejection response involves a constellation of immunologic and environmental factors.
D. Allotransplantation evokes a cellular immune response.
E. Allotransplantation evokes a humoral immune response.
Answer: ABDE

DISCUSSION: Medawar and Gibson elucidated some of the basic principles of transplantation biology. Rejection of a
second skin graft from the donor of a first graft is very much hastened, indicating that the response is learned and that the
second response evokes “memory.” The second graft is rejected rapidly, regardless of its location, indicating that the
response is systemic. The major immune reaction causing rejection of a first graft is a cellular immune response; however,
the recipient exposed to allogeneic cells develops antibodies against alloantigens, indicating that a humoral response has
also occurred.

12. Allograft rejection may involve which of the following?


A. Helper T cells.
B. Veto cells.
C. Cytotoxicity.
D. Cytokines.
E. The Arthus reaction.
Answer: ACD

DISCUSSION: The effector mechanisms that underlie the pathogenesis of allograft rejection remain the subject of
controversy. Rejection, like delayed-type hypersensitivity, may be mediated by helper T cells, which release cytokines that
activate other cells such as macrophages and directly alter endothelial cell functions. On the other hand, rejection may be
mediated by cytotoxic T cells, which kill or injure target cells through direct interactions. Both types of cells can be found in
grafts undergoing rejection, and there is experimental evidence suggesting that both may be involved. Veto cells kill T cells,
which recognize MHC antigens on the veto cell surface; this action is thought to contribute to tolerance and not to rejection.
The Arthus reaction is an immune response that, in contrast to allograft rejection, is mediated primarily by antibodies and
not by cells.

13. Which of the following statements about allograft rejection are true?
A. In the absence of immunosuppression, the time and intensity of rejection of transplants between unrelated donors and
recipients is highly variable.
B. Allograft rejection may be mediated by antibodies or by cells.
C. Allograft rejection is thought to be caused by Th2 cells.
D. Acute cellular rejection is the major cause for loss of clinical organ transplants.
E. An individual with “tolerance” is unable to reject an allograft.
Answer: B

DISCUSSION: In the absence of immunosuppression, allografts from randomly selected donors are always rejected, and the
rate of rejection is rapid as compared with the rate of development of most immune responses. Although allograft rejection
in naive recipients is mediated predominantly by cells, antidonor antibodies can cause very severe types of rejection,
including hyperacute and acute vascular rejection. Antidonor antibodies or cellular responses may contribute to the
development of chronic rejection, which is now the most common cause of graft loss. Recent studies demonstrate that
helper T cells may differentiate along one of two pathways. The Th1 pathway leads to secretion of interferon-gamma and
other cytokines and is associated with delayed-type hypersensitivity and allograft rejection. The Th2 pathway is associated
with secretion of interleukin-10 (IL-10) and IL-4 and may actually inhibit alloimmune responses. The development of Th2
responses may thus contribute to tolerance. Like allograft rejection, tolerance is highly specific. Thus, a person who is
tolerant to one antigen or one individual is still able to mount an immune response against other antigens and other
individuals.

14. The presence of donor-reactive lymphocytotoxic antibodies in the serum of a potential kidney transplant recipient:
A. Can be detected by in vitro testing with recipient leukocytes and donor serum.
B. Is a contraindication to kidney transplantation.
C. Can be found in all male patients older than 20 years.
Answer: B

DISCUSSION: The presence of donor-reactive antibodies, detected by incubation of the recipient's serum with donor
lymphocytes in the presence of complement, results in a “positive crossmatch,” and is a contraindication to renal
transplantation. They occur as a result of pregnancy, blood transfusions, or previous organ transplants.

15. Utilization of a living related donor instead of a cadaver donor is no longer an advantage in renal transplantation
because:
A. Public recognition of transplantation as a successful therapy has facilitated obtaining family permission for recovery of
transplantable organs. Thus, because sufficient kidneys are available from “brain-dead” accident victims, there is no need to
use related donors.
B. Cyclosporine therapy after cadaveric renal transplants has improved their outcome, which is now comparable to related-
donor transplants.
C. Modern preservation techniques can maintain viability of kidneys from cadaver donors for many hours, consistently
allowing their early function to be as good as that of kidneys from living donors.
D. None of the above.
Answer: D

DISCUSSION: It is generally accepted that transplantation is a useful therapy; however, the number of recipients continues
to greatly exceed the number of suitable cadaver donors whose families grant permission for organ recovery. Thus,
availability of a living donor may shorten the waiting period for a transplant by several years. Cyclosporine has improved
the short-term results of cadaveric transplantation, but the attrition of these grafts is greater than that for living-donor
transplants, especially those with close histocompatibility. The predicted 10-year survival of grafts from HLA-identical
siblings is 80%, whereas for cadaver grafts it is only 40%. Although preservation techniques can maintain viability of
kidneys for 36 to 48 hours, cadaver kidneys suffer a much higher rate of posttransplant acute tubular necrosis than those
from related donors. Acute tubular necrosis has been shown to have a definite detrimental effect on long-term graft survival.

16. Large volumes of urine in the early postoperative course of renal transplant patients:
A. Result from osmotic stimuli to diuresis.
B. May signify reversible polyuric acute tubular necrosis.
C. Should be replaced by administration of equal volumes of crystalloid.
D. Facilitate the diagnosis of rejection and obstruction of the renal artery and/or collecting system.
Answer: ABCD
DISCUSSION: Factors responsible for the brisk diuresis following renal transplantation include osmotic stimuli secondary
to high urea and/or glucose concentrations in the serum, and mild proximal tubular damage resulting from allograft
ischemia. To avoid severe dehydration in the early postoperative period, an attempt should be made to replace urine losses
with equal volumes of 0.45% NaCl solution to which 20 to 30 mEq. NaHCO 3 per liter may be added. The diagnosis of
rejection and/or obstruction to urine flow is made easier when a transplanted kidney is undergoing voluminous diuresis
rather than demonstrating oliguria or anuria secondary to severe acute tubular necrosis.

17. As compared with the early immunosuppressive drugs (azathioprine, steroids, antilymphocyte serum) some newer
agents have the following specific advantages:
A. Cyclosporine, which interferes with lymphokine production, exhibits neither bone marrow nor renal toxicity.
B. Monoclonal antibody (OKT3) is more available and has greater specificity and fewer side effects than antilymphocyte
serum.
C. Tacrolimus (FK506) has properties similar to those of cyclosporine but is especially valuable for rescue of grafts that are
failing on cyclosporine therapy.
D. None of the above.
Answer: C

DISCUSSION: Cyclosporine interferes with production of cytokines and lacks the bone marrow toxicity of azathioprine.
Unfortunately its chief toxicity is renal. Although OKT3 is more available, more uniform, and more specific than
antilymphocyte serum, some of its side effects are even greater, such as fever, chills, nausea, vomiting, diarrhea, and
pulmonary edema. Tacrolimus has been used most extensively for liver grafts. It has been found especially valuable in
reversing rejection of failing grafts.

18. Survival rates for patients on dialysis are better than those for patients receiving renal allografts in the following
circumstances:
A. A living related donor is available.
B. A cadaver donor must be used.
C. The recipient's renal failure is secondary to diabetes.
D. None of the above.
Answer: D

DISCUSSION: Patients receiving chronic dialysis have a mortality rate of 6% to 20% per year, every year. The mortality
rate is as high as 11% to 25% per year in diabetic dialysis patients. Patients undergoing renal transplantation have an
operative mortality rate of less than 2%, and the 1-year survival for recipients of living related kidneys is better than 95%.
Survival is greater than 90% for recipients of cadaver kidneys. The 5-year patient survivals are approximately 80% for
nondiabetic recipients of living related and cadaver kidneys, and 60% to 70% for diabetic recipients. Thus, a well-
functioning renal allograft provides a greater chance for a longer life than does chronic dialysis.

19. Posttransplantation hypertension can be caused by:


A. Rejection.
B. Cyclosporine nephrotoxicity.
C. Renal transplant artery stenosis (RTAS).
D. Recurrent disease in the allograft.
Answer: ABCD

DISCUSSION: Both acute and chronic rejection may result in hypertension. The former causes acute fluid retention and
plugging of peritubular capillaries with inflammatory cells. This may progress to intimal swelling and medial necrosis and
eventuate in ischemia secondary to endothelial proliferation and obliteration of small vessels. Chronic rejection, thought to
be related to protracted humoral injury, results in obliteration of capillaries via the development of intimal hyperplasia.
Cyclosporine has a vasoconstrictive effect which, through activation of the renin-angiotensin system, may lead to
hypertension. RTAS is responsible for hypertension in 4% to 12% of renal allograft recipients. It responds well to
percutaneous angioplasty. A careful trial of angiotensin-converting enzyme inhibitors may be diagnostic of RTAS. Recurrent
disease such as membranoproliferative glomerulonephritis and focal glomerular sclerosis may result in significant
hypertension in renal allograft recipients.
20. Which of the following statements about posttransplantation malignancy is correct?
A. Certain immunosuppressive agents increase the incidence of malignancy in transplant recipients, whereas others do not.
B. Those malignancies most commonly seen in the general population (breast, colon) are substantially more common in
transplant recipients.
C. Lymphoproliferative states and B-cell lymphomas are associated with Epstein-Barr virus.
D. None of the above.
Answer: C

DISCUSSION: Both naturally occurring and iatrogenic states of immune deficiency are associated with an increased rate of
de novo malignancy. Transplant recipients have a rate of malignancy approximately 100 times that of the normal population.
The degree of immunosuppression, rather than a specific immunosuppressive agent, appears to be responsible. Squamous
and basal cell carcinomas of the skin are most common; however other tumors that are common in the general population,
such as breast and colon cancers, do not appear to be increased in incidence. Lymphomas, which occur at a rate that is 350
times normal, and the lymphoproliferative states that often precede them appear to be associated with Epstein-Barr virus.
Possible explanations for these high malignancy rates include defective immunosurveillance, chronic stimulation of the
reticuloendothelial system by the allograft, the carcinogenic effect of immunosuppressive drugs, and viral oncogenesis.

21. One week after receiving a cadaver renal allograft, the recipient remains oliguric and dialysis dependent.
Ultrasonography reveals a larger perigraft fluid collection. Your next step in management includes:
A. No further investigations (since perigraft collections are fairly common after renal transplantation).
B. Aspiration of the perigraft fluid collection and instillation of a fibrosis-inducing agent to obliterate the dead space.
C. Angiography for localization of a bleeding site in the renal allograft.
D. Aspiration of the perigraft fluid collection for chemical analysis.
Answer: D

DISCUSSION: Urine leaks usually occur early after transplantation, and the most frequent site of leakage is from the
ureteroneocystostomy or ischemic ureter. The clinical signs are pain, swelling, and deterioration of renal function before
leakage from the wound is observed. Aspiration of the perigraft fluid collection for chemical analysis of blood urea nitrogen
(BUN) and creatinine would aid the differentiating urinoma from lymphocele. The composition of urinoma reveals BUN
and creatinine concentrations several orders of magnitude higher than those of a lymphocele, which are comparable to the
values in blood.

22. Regarding access for hemodialysis, which of the following statements is/are incorrect?
A. Some patients are not candidates for hemodialysis.
B. Some complications can lead to exsanguination.
C. The best access to place for a patient beginning dialysis is a leg polytetrafluoroethylene (PTFE) graft from the femoral
artery to the saphenous vein.
D. First of all one should attempt to create a Brescia-Cimino fistula.
E. The leading complication of PTFE grafts is infection.
Answer: CE

DISCUSSION: Some patients do not tolerate hemodialysis because of cardiac difficulties or because they cannot be
heparinized for hemodialysis. If a peripheral shunt becomes disconnected, the patient can exsanguinate. This can occur if a
cap or clamp is inadvertently removed from a central dialysis catheter. The ideal location for a dialysis fistula or graft is in
the upper extremity as far distal as possible, such as with a Brescia-Cimino fistula or a forearm loop graft. The leading
complication of PTFE grafts is thrombosis caused by intimal hyperplasia in the venous limb. Infection is the second leading
complication with these grafts.

23. Access to the peritoneal cavity for peritoneal dialysis can be gained:
A. Percutaneously.
B. Surgically.
C. Using laparoscopy.
D. Only using general anesthesia.
Answer: ABC
DISCUSSION: Chronic ambulatory peritoneal dialysis (CAPD) catheters may be placed at the bedside with a straight
Tenckhoff catheter under local anesthesia. They can be placed laparoscopically or by making a small perimedian incision
and placing the catheter through the rectus muscle. All of the techniques can be performed using local anesthesia; however,
use of the laparoscopy commonly calls for general anesthesia.

24. Which of the following are true concerning immunosuppression?


A. Current immunosuppressive agents function in a nonspecific manner to suppress rejection.
B. The use of immunosuppressive agents is associated with an increased rate of opportunistic infections.
C. An increased rate of malignancy is not associated with the use of immunosuppressive agents.
D. In almost all cases, the graft is rejected if immunosuppression is discontinued.
Answer: ABD

DISCUSSION: At the present time, clinical immunosuppression involves the use of agents that function in a nonspecific
manner to prevent rejection. These agents suppress almost all aspects of the immune response. Because of their mechanism
of action, they have associated toxicities and side effects, such as an increased rate of opportunistic infections. An increase
in certain malignancies is also associated with use of these agents. In almost all cases, the graft is rejected if the
immunosuppression is discontinued. Therefore, immunosuppression must be continued for the life of the graft.

25. Which of the following is true for hyperacute rejection?


A. It is mediated by preformed cytotoxic antibody.
B. It occurs late in the life of the graft.
C. It is usually reversible with a bolus of steroids.
D. None of the above.
Answer: A

DISCUSSION: Hyperacute rejection is mediated by preformed cytotoxic antibody. It can be screened for by cross-matching
procedures. It usually occurs immediately after graft placement or within the first 24 to 48 hours after graft placement. It is
almost never reversible.

26. The major components of the immune system include which of the following?
A. T lymphocytes.
B. B lymphocytes.
C. Cytokines.
D. Macrophages.
Answer: ABCD

DISCUSSION: The development of the lymphoid system begins with a pluripotent stem cell in the liver and bone marrow
of the fetus. With maturation of the fetus toward term, the bone marrow becomes the primary site for lymphopoiesis. It
produces the T lymphocytes, B lymphocytes, and macrophages that are critical to the immune response. These cells then
produce cytokines or soluble growth factors, which amplify the immune response.

27. The most common types of immunosuppressive agents used clinically include which of the following?
A. Antimetabolites.
B. Alkylating agents.
C. Inhibitors of helper T-cell activation.
D. Irradiation.
E. Lymphocyte depletion compounds.
Answer: ABCDE

DISCUSSION: Much of the susceptibility of lymphocytes to immunosuppression is due to the vast cellular changes that
follow immune stimulation. The biosynthetic events that take place make the lymphocytes vulnerable to inhibition at
various stages of the cell cycle. Cyclosporine inhibits cytokine gene expression, whereas alkylating agents and radiation
produce cross-linkages and breaks in DNA strands that interfere with cell differentiation and division. Agents that produce
depletion of lymphocytes include antilymphocyte globulin (ALG) and monoclonal antibodies (OKT3).
28. Which of the following is/are true of the antiproliferative agents?
A. They act by preventing the differentiation and division of the immunocompetent lymphocyte after it encounters antigen.
B. The antimetabolites in this group have a structural similarity to cell metabolites and either inhibit enzymes of a
metabolic pathway or are incorporated during synthesis to produce faulty molecules.
C. The most frequently used antiproliferative agent is azathioprine.
Answer: ABC

DISCUSSION: Antiproliferative agents inhibit the full expression of the immune response by preventing the differentiation
of the immunocompetent lymphocyte after it encounters antigen. They act in one of two ways: they either structurally
resemble necessary metabolites, or they combine with certain cellular components, such as DNA, and thereby interfere with
function. Until recently, azathioprine was the most widely used immunosuppressive drug in transplantation, and it still has a
major clinical role in preventing rejection.

29. Which of the following is the one true statement about acute rejection.
A. Acute rejection is mediated by T lymphocytes.
B. Acute rejection is mediated by preformed cytotoxic antibody.
C. Acute rejection most frequently occurs over months.
Answer: A

DISCUSSION: Acute rejection is mediated primarily by T lymphocytes. It occurs over 1 to 3 weeks after placement of an
allograft. Hyperacute rejection is mediated by preformed cytotoxic antibody. It occurs within 48 hours of placement of a
graft. Chronic rejection is mediated by both T cells and B cells and occurs over months.

30. Which of the following are true of cyclosporine?


A. It was the first immunosuppressive agent to be used clinically.
B. It acts selectively on T cells to suppress rejection.
C. Toxic effects include hirsutism, hypertension, nephrotoxicity, and increased risk of opportunistic infections.
Answer: BC

DISCUSSION: Cyclosporine is a product of a fungus and was discovered in 1972. It has contributed very significantly to
the development of the field of transplantation. The mechanism of action is relatively specific for T lymphocytes. Other
inflammatory cells are much less sensitive to its immunosuppressive effects. It inhibits activated T lymphocytes and
prevents the cells from manufacturing and releasing interleukin 2 (IL-2). Toxicities include hirsutism, hypertension,
nephrotoxicity, and increased risk of opportunistic infections (because it still functions as a nonspecific immunosuppressive
agent).

31. Which of the following are true of OKT3?


A. It is not a monoclonal antibody.
B. It binds to the T-cell receptor and inactivates T-cell function.
C. It is the monoclonal antibody most frequently used in clinical transplantation.
Answer: BC

DISCUSSION: OKT3 is a monoclonal antibody produced in limitless supply by a hybridoma. It binds to a site associated
with the T-cell receptor complex to inactivate the T cell. It is the most widely used monoclonal antibody in clinical
transplantation.

32. Hypothermia (0‫ ؛‬to 4‫ ؛‬C) is a critical component of successful organ cold storage because:
A. Oxygen is more soluble in cold solutions and provides a continual supply for energy metabolism.
B. There is no way to suppress microbial growth except by cooling and slowing the growth rate.
C. Hypothermia diminishes energy requirements and allows the limited energy reserve to keep the organ alive.
D. It slows metabolism and the enzymic processes that would destroy the cell.
Answer: D
DISCUSSION: Hypothermia in simple organ cold storage serves one primary function but secondary ones as well. The
primary function is to slow metabolism. Metabolic rates decrease about twofold for every 10‫ ؛‬C drop in temperature.
Cooling an organ from 37‫ ؛‬to 0‫ ؛‬to 4‫ ؛‬C drops metabolism about 12- to 13-fold. This is related to the activation energy of
enzymatic processes as expressed by Arrhenius and van't Hoff. Thus, catabolism of structural and functional cellular
element is retarded for a long period—up to 3 days for some organs. The cold also suppresses microbial growth, but this can
be accomplished by other means as well. The cold also allows time for the transplantation operation, and during this time it
is important to be quick or to keep cooling the organ.

33. Is the following statement true or false? Organs should be preserved only for short periods of time (4 to 8 hours)
because longer periods lead to too many complications, and even loss of the organ.
Answer: FALSE

DISCUSSION: The logistics of organ transplantation make it very difficult to use all available cadaver organs within 4 to 8
hours. To use all the organs requires the capability to preserve them for at least 24 hours. It has been shown that most organs
can be matched to a recipient within about 17 to 24 hours. For all but the heart and lung, most intra-abdominal organs
tolerate preservation for 20 to 30 hours and perform as well as those preserved for 4 to 8 hours. Although undue delay
should not be purposefully used, certainly, most organ transplants do not need to be done on an emergency basis. However,
this is not true for hearts and lungs, which should be transplanted as quickly as possible.

34. Which of the following statements about hepatic artery thrombosis following liver transplantation is/are correct?
A. Thrombosis of the hepatic artery following liver transplantation is more common in children than in adult patients.
B. Thrombosis of the hepatic artery usually occurs several weeks after transplant as a result of arteriosclerosis.
C. Thrombosis of the hepatic artery in the early days following transplantation is a serious complication leading to death
unless retransplantation can be performed within 36 to 72 hours.
D. Late thrombosis of the hepatic artery may present as biliary complication or intrahepatic abscesses.
E. Thrombosis of the portal vein is more frequent than hepatic artery thrombosis following liver transplantation.
Answer: ACE

DISCUSSION: Thrombosis of the hepatic artery remains one of the most serious early complications of liver
transplantation. This complication is three to five times more common in children than in adults. The major cause of this
complication is related to technical error, although the hypercoagulable state may play a significant role in some situations.
Early thrombosis of the hepatic artery leads to rapid liver failure with a fatal outcome unless a transplant can be performed
within 36 to 72 hours. Although thrombolytic therapy through percutaneous or surgical access can be successful, most of
these patients require retransplantation. Stenosis of the hepatic artery or late thrombosis of the hepatic artery can lead to
multiple intrahepatic strictures of the bile duct and/or hepatic abscesses. This complication also often requires
retransplantation. Portal vein thrombosis is a rarer complication. It is a devastating condition when it occurs early, but can
be tolerated well if it develops after several months. Portal hypertension due to late portal vein thrombosis can often be
treated successfully by a shunt procedure.

35. Which of the following statements about fulminant hepatic failure (FHF) is/are correct?
A. Fulminant hepatic failure can occur in the setting of pre-existing chronic liver disease.
B. Coagulopathy and coma are important findings in patients with FHF.
C. Liver transplant should not be attempted in patients with FHF because of the high mortality rate, regardless of the
treatment used.
D. The main cause of death in these patients is cerebral edema.
E. One of the most important factors in prognosis of FHF is the cause of liver disease.
Answer: BDE

DISCUSSION: FHF corresponds to the rapid loss of hepatic function in the absence of pre-existing liver disease, causing
jaundice, coagulopathy, and coma. One of the major prognostic factors is the cause of the liver disease. Early admission to
an intensive care unit and management by physicians experienced in liver transplantation are mandatory. The major cause of
death in these patients is cerebral edema. In patients who rapidly develop coma, subdural intracerebral pressure monitoring
is mandatory for optimal management as well as for identification of patients who can benefit from liver transplantation.
The survival rate for patients with FHF who underwent liver transplantation is currently above 65%. This is the only cure in
most patients with FHF.
36. Which of the following statements about immunology in liver transplantation is/are correct?
A. Good human leukocyte antigen (HLA) matching between recipient and donor is mandatory for a good outcome for liver
transplantation.
B. Hyperacute rejection is almost nonexistent following liver transplantation.
C. Acute rejection occurs in more than 50% of patients and is reversible in most patients with large doses of steroids.
D. Acute rejection is very rare later than 2 months after liver transplantation unless the patient is inadequately
immunosuppressed.
E. Chronic rejection is different from acute rejection, is usually irreversible, and often requires retransplantation.
Answer: BCDE

DISCUSSION: Immune-mediated reactions following liver transplantation are clearly different from those that follow other
solid organ transplants. The liver is tolerated quite well, and currently donors and recipients are matched only for their ABO
group. Even when the ABO barrier is not respected, survival is still over 60%. T cell–mediated acute rejection occurs in
about half of the patients within 6 weeks after liver transplantation, and acute rejection is reversed by large doses of steroids
in most cases. Chronic rejection, on the other hand, is a different entity that is ill-understood and corresponds to destruction
of small arteries and bile ducts. Change in the immunosuppression regimen sometimes may hinder the progression of this
disease, but often retransplantation is required.

37. An elevated serum amylase level following pancreas-kidney transplantation may be due to:
A. Preservation/procurement injury.
B. Rejection.
C. Reflux pancreatitis.
D. Duodenal segment leak or bladder leak.
E. Native pancreatitis.
F. Constipation.
Answer: ABDEF

DISCUSSION: During the immediate postoperative period, an elevated serum amylase is usually due to preservation or
procurement injury to the transplanted pancreas. If UW (University of Wisconsin) solution and a good flushout technique is
used in an acceptable donor, amylase is usually elevated only several hundred points and will decline in a day or two.
Rejection of the pancreas can also cause elevated serum amylase and is usually accompanied by a rise in the creatinine
value due to concomitant renal transplant rejection. Reflux pancreatitis is generally caused by bladder dysfunction:
increased pressure transmitted back through the pancreatic ducts causes pancreatitis. It is generally relieved by bladder
decompression with a Foley catheter. Naturally, leakage from the anastomosis of the pancreas transplant to the bladder
causes absorption of amylase from the peritoneal cavity and an elevated serum amylase value. Constipation causes a rise in
the amylase level of pancreas-kidney transplant recipients, for reasons that remain unclear. Native pancreatitis has to be
borne in mind in the differential diagnosis of hyperamylasemia in transplant patients. Contributing factors may include
underlying gallbladder disease, as well as side effects of steroids and Imuran.

38. Complications of a pancreas transplant drained into the bladder include:


A. Duodenal segment leak.
B. Recurrent urinary tract infections.
C. Recurrent hematuria.
D. Urethritis.
E. Refractory loss of bicarbonate.
Answer: ABCDE

DISCUSSION: All of the listed problems are potential complications of bladder drainage. The most useful diagnostic tests
for a duodenal segment leak include CT cystogram and technetium-based nuclear cystogram. Cystoscopy should be
performed in patients with recurrent urinary tract infections to evaluate the presence of sutures or foreign bodies acting as a
nidus for infection. Severe recurrent hematuria, as well as urethritis (most commonly affecting males), may occur with
bladder drainage. Finally, severe bicarbonate loss may be associated with bladder drainage, and some patients may have
difficulty keeping up with the loss by oral bicarbonate replacement. All of the above situations can be effectively treated
with enteric conversion
39. Patient selection criteria for simultaneous pancreas-kidney transplantation should include:
A. Type I diabetes mellitus.
B. Type II diabetes mellitus.
C. Dialysis dependence.
D. Renal dysfunction with a creatinine value greater than 3.0.
E. Minimal extrarenal morbidity related to diabetes.
Answer: ADE

DISCUSSION: Simultaneous pancreas-kidney transplantation should be reserved for patients with Type I, juvenile, insulin-
dependent diabetes mellitus. Although patients with Type II diabetes could potentially be helped, these older patients
generally are not in good condition for simultaneous pancreas-kidney transplantation, nor are they reliably cured. In order to
reliably monitor renal transplant function, the pretransplant creatinine value should be above 3, but the patient must not
necessarily be on dialysis. To achieve good long-term results with pancreas-kidney transplantation it is appropriate to select
patients with minimal extrarenal morbidity related to their diabetes.

40. Criteria for a pancreas donor include:


A. No history of diabetes.
B. No liver donation.
C. No replaced hepatic artery vessels arising from the superior mesenteric artery (SMA).
D. No previous splenectomy.
E. No pancreatitis.
Answer: AE

DISCUSSION: Combined liver-pancreas procurement should be routine, even if the right hepatic artery arises from the
superior mesenteric artery. In this situation, since the transplanted liver is the life-saving organ, the proximal superior
mesenteric artery should remain with the liver and the distal superior mesenteric artery supplying the head of the pancreas
can be reconstructed on a Y-graft of iliac artery with the splenic artery. Successful pancreas transplantation can be
performed using donors who have previously undergone splenectomy; however, there should be no significant pancreatitis
and no history of diabetes in the donor.

41. For which of the following clinical scenarios would cardiac transplantation be an appropriate therapeutic modality?
A. A 50-year-old man with angina pectoris, three-vessel coronary artery disease, and a left ventricular ejection fraction of
25%.
B. A 75-year-old woman with irremediable heart failure secondary to critical aortic stenosis.
C. A 25-year-old male athlete with insidious onset of heart failure secondary to idiopathic dilated cardiomyopathy.
D. A 55-year-old woman who is status post two previous surgeries for coronary artery revascularization, now presenting
with heart failure in the absence of angina, left ventricular ejection fraction of 15%, and insufficient target coronary arteries
for a third bypass procedure.
E. A newborn infant with hypoplastic left heart syndrome and no other congenital anomalies.
F. A 30-year-old woman who develops irremediable heart failure due to postpartum cardiomyopathy after giving birth.
Answer: CDEF

DISCUSSION: Scenarios A and B are not appropriate for cardiac transplantation. The patient in example A would be far
better served by a conventional revascularization procedure such as coronary artery bypass grafting. The risk might be
somewhat greater than normal because of his depressed left ventricular ejection fraction; however, cardiac transplantation is
a therapy that is necessarily reserved for persons for whom no other procedure is available. That clearly is not the case in
this example. In example B, despite the fact that this patient's disease might be benefited by cardiac transplantation, she is
too old to withstand the rigors of this procedure and its attendant therapies. Examples, C, D, E, and F, all represent situations
in which cardiac transplantation would be appropriate. In all these cases there is end-stage heart disease, and no other
therapies are available that are likely to have any substantial benefit. Therefore, it is appropriate to consider cardiac
transplantation for these patients, as a last resort.

42. Suitable donors for heart transplantation have which of the following characteristics?
A. Normal electrocardiogram (ECG).
B. Normal echocardiogram.
C. Positive serology for HIV or hepatitis B or C.
D. Patient requiring high-dose epinephrine to maintain a systolic blood pressure of 90 mm. Hg.
E. Age over 70 years.
Answer: AB

DISCUSSION: To be suitable for cardiac donation, individuals must have a normal ECG and a normal echocardiogram.
Clearly, positive serologic tests for HIV or hepatitis B or C would render donors unsuitable for solid organ transplantation.
Similarly, high-dose pressor support and age greater than 60 years, in most programs, contraindicate cardiac donation.

43. Heart-lung transplant is currently the therapy of choice for which of the following conditions?
A. Primary pulmonary hypertension with reasonably well-preserved right ventricular function.
B. Eisenmenger's syndrome due to single ventricle and truncus arteriosus.
C. Validated cardiomyopathy in a patient with cystic fibrosis and end-stage lung disease.
D. Cystic fibrosis and end-stage lung failure with normal heart function.
E. Eisenmenger's syndrome due to an atrial septal defect.
F. End-stage lung disease secondary to emphysema.
Answer: BC

DISCUSSION: Heart-lung transplantation is now properly used only for persons with end-stage disease of the heart and
lungs. Therefore, a patient with primary pulmonary hypertension and reasonably wellpreserved right ventricular function is
best treated with a single or bilateral lung transplant. A person with complex congenital heart disease and Eisenmenger's
syndrome or one who has end-stage disease of the heart and lungs would be better treated with combined heart-lung
transplantation. Patients with Eisenmenger's syndrome secondary to relatively straightforward defects (e.g., atrial septal
defect, ventricular septal defect) are best treated with concomitant correction of the congenital defect and single or bilateral
lung transplantation. Similarly, the patient with end-stage emphysema with normal heart function can be treated very well
with single or bilateral lung transplantation, preserving the donor heart for someone who truly has heart failure.

44. Both single and bilateral lung transplantation are suitable technical alternatives for which of the following conditions?
A. Obstructive lung disease (chronic obstructive pulmonary disease, emphysema).
B. Restrictive lung disease (pulmonary fibrosis).
C. Primary pulmonary hypertension.
D. Cystic fibrosis.
Answer: ABC

DISCUSSION: Single-lung transplantation is inappropriate for cystic fibrosis or for any patient with chronic bilateral
pulmonary sepsis. Leaving a septic native lung in situ in an immunocompromised patient would leave the patient at risk for
local and systemic septic complications. Single-lung transplantation with contralateral pneumonectomy would be associated
with a high risk of empyema in the pneumonectomy space and also disruption of the bronchial stump. Both single- and
bilateral lung transplantation have been applied successfully in all of the other disease categories listed here.

45. Which of the following are contraindications to lung transplantation?


A. Age 65 years or older.
B. Current corticosteroid therapy.
C. History of thoracotomy.
D. Ventilator-dependent respiratory failure.
Answer: D

DISCUSSION: Single-lung transplantation is still offered up to age 65 years. Current low-dose corticosteroid therapy has
not been demonstrated to lead to a higher risk of airway complications after lung transplantation. Advancements in
operative technique have lessened the risk of surgery, so prior thoracotomy is no longer a contraindication to lung
transplantation. However, patients with chronic ventilator-dependent respiratory failure who have no potential for
cardiopulmonary rehabilitation currently are not accepted for evaluation for potential lung transplantation.

46. Which of the following is the single most useful approach for diagnosing acute lung allograft rejection?
A. Clinical diagnosis.
B. Decline in spirometry and oxygenation.
C. Chest radiographic abnormalities.
D. Fiberoptic bronchoscopy with transbronchial lung biopsy.
Answer: D

DISCUSSION: Virtually all lung transplant patients experience at least one episode of acute rejection during their
postoperative recovery in hospital. All of the approaches mentioned above are useful in leading to the diagnosis. Clinically,
the patient experiences malaise and fever. There is also typically a slight decline in spirometry and arterial oxygenation. The
chest radiograph typically shows a hilar or basal shadow; however, although these findings all suggest acute rejection, they
are not specific. The one test with high specificity for detection of acute rejection is bronchoscopy with transbronchial lung
biopsy.

47. Advantages of split-thickness skin grafts over full-thickness skin grafts include:
A. Split-thickness grafts include only part of the epidermis and none of the dermis.
B. Split-thickness grafts offer better pigment matching.
C. Split-thickness grafts offer better resistance to contraction.
D. Split-thickness grafts offer better resistance to infection.
E. Split-thickness grafts survive better on surfaces with compromised blood supply.
Answer: DE

DISCUSSION: Split-thickness grafts include all of the epidermis but only a part of the dermis. Full-thickness skin grafts
include all of both layers, so surgical closure of the donor wound is necessary whereas the portion of dermis left at the split-
thickness skin donor site regenerates a skin covering. Because all layers of the skin are included in a full-thickness skin
graft, pigment matching is better and less contraction occurs than with split-thickness grafts. Full-thickness grafts require a
better blood supply for survival than the split-thickness grafts because the graft vessels are cut below the level of the dermal
branching. Relatively fewer cut vessels are available to absorb nutrients from the wound bed to meet the relatively greater
nutritional needs of the thicker graft. The poor resistance of full-thickness grafts to infection precludes their use on
contaminated wounds, whereas split-thickness skin, which is more richly supplied with open blood vessels on its underside,
is able to survive on compromised surfaces, including granulating wounds contaminated with bacteria.

48. The most commonly used substitutes for peripheral arteries are:
A. Dacron grafts.
B. Expanded polytetrafluoroethylene (Gore-Tex) grafts.
C. Internal, external, and/or common iliac artery autografts.
D. Bovine carotid artery xenografts.
E. Saphenous vein autografts.
Answer: E

DISCUSSION: The greater saphenous vein has proved to be the most satisfactory and most commonly used arterial
substitute. The wall is sufficiently strong to withstand arterial pressures without becoming dilated or aneurysmal, yet is
flexible and easily sutured. The diameter is sufficiently great to avoid thrombosis and nourishment is provided by the
intraluminal blood flow. The smooth, natural endothelial lining is less thrombogenic than any known synthetic surface. The
lining surface heals itself and may sequester white cells to fight infection, unlike Dacron grafts, which provide a haven for
infecting organisms in the interstices of their synthetic fibers. Saphenous vein autografts heal even when placed into the
infected bed of a previous synthetic graft.

49. Endocrine autografts were among the first successful transplantation procedures. The demonstration by Berkhold in
1849 that autotransplanted testes led to the acquisition of secondary sexual characteristics in castrated cocks marked the
beginning of experimental endocrinology. Endocrine autografts used successfully in modern surgical practice include:
A. Adrenal medulla to the brain.
B. Thyroid to the forearm.
C. Parathyroid to the forearm.
D. Testicle to the scrotum.
E. Pancreatic islets to the liver.
Answer: CDE
DISCUSSION: The report in 1987 of open microsurgical autotransplantation of the adrenal medulla to the caudate nucleus
of the brain for treatment of intractable Parkinson's disease aroused great interest in the topic. Subsequent multicenter trials
showed improvement but not cure of the disease and substantial post-operative morbidity, so the technique was not
recommended for widespread use. Excellent synthetic hormone replacement is available for thyroid insufficiency, so
implantation of the thyroid gland in the forearm is not necessary. However, parathyroid hormone replacement is not
available, and medical therapy for hypoparathyroidism is complicated. When parathyroid tissue is removed it should be
autografted to prevent the deficiency symptoms of tetany, psychological disturbances, convulsions, coma and death. One-
millimeter pieces may be implanted into pockets in the sternocleidomastoid muscle. When all glands are removed for
diffuse parathyroid hyperplasia, implantation of fragments into the forearm muscles facilitates subsequent removal of more
tissue under local anesthesia if hyperparathyroidism persists. Autotransplantation is the treatment of choice for undescended
testes. The cryptorchid or ectopic testicle must be taken out of the abdomen and placed into a cooler location prior to age 6
(preferably at 1 year) for normal spermatogenesis to occur. Approximately half of the pancreatic islet transplants performed
after pancreatectomy for relief of chronic pancreatitis pain have produced patients who are insulin independent. Islets for
autotransplantation are difficult to isolate in sufficient quantities from the fibrotic adult pancreas. Dispersed islets injected
directly into the human portal vein have occasionally produced untoward effects such as disseminated intravascular
coagulation, portal hypertension, and even hepatic necrosis.

50. Several types of gastrointestinal autografts have been used to replace the esophagus after extirpation of carcinomas.
Successful reconstructions have been achieved most frequently with:
A. Stomach.
B. Jejunum.
C. Ileum.
D. Ascending colon.
E. Descending colon.
Answer: A

DISCUSSION: Although all of the listed bowel segments have been used successfully for reconstruction of the esophagus
following removal of carcinomas, the stomach remains the most frequently used autograft for esophageal reconstruction.
Because of its excellent blood supply the procedure can be performed at little risk as a single operation and achieve
satisfactory long-term relief of dysphasia in at least 90% of patients. Either the entire stomach can be drawn into the chest or
a gastric tube created in an isoperistaltic or antiperistaltic manner of sufficient length to reconstruct the entire esophagus.
The advantages of a mucosal lining, serosal covering, natural opening into the stomach, and excellent blood supply based on
the gastroepiploic vessels make the stomach the autograft of choice in most situations.

51. Which of the following statement(s) is/are true concerning the options for managing the exocrine secretions following
pancreatic transplantation?

a. Ductal ligation is associated with no adverse effects to pancreatic parenchyma


b. Drainage of the pancreatic ductal system into the bladder is useful in the early diagnosis of rejection
c. All pancreatic grafts should be placed in a retroperitoneal position
d. Complications following enteric drainage of the pancreas (without the duodenum) are primarily associated with
anastomatic leakage
Answer: b, d

There are, in principal, three options in managing the exocrine secretions following pancreatic transplant. In the first option,
maintenance of exocrine secretions by internal drainage of the exocrine pancreas can be achieved by anastomosing the
ductal system to either the intestinal tract (stomach, small intestine) or the urinary tract (ureter, bladder). These techniques
are the most common in use today and provide the best overall results. The second technique, free drainage of the pancreatic
juice into the peritoneal cavity, is certainly the least technically demanding method of transplantation. It is, however,
associated with many other complications. Ablation of the exocrine secretion, the third option, can be accomplished by two
techniques. The first, duct ligation, has been associated with exocrine atrophy and extensive fibrosis, usually resulting
eventually in endocrine insufficiency. Ductal ligation has also had unpredictable effects on the exocrine tissue, associated
with a high risk of acute pancreatitis and peripancreatic sepsis. The other method of ductal ligation involves injecting the
pancreatic system with a synthetic polymer that solidifies within several minutes, with a result that exocrine secretion is
completely blocked. The enterically drained pancreas (without duodenum) has in the past been associated with a significant
incidence of anastomatic leakage, leading to pancreatic fistula, perigraft abscess, and systemic sepsis. Many of these
allografts had to be removed. These problems can be oveated to a large extent if the donor duodenum (removed in block
with the pancreas) is used to establish anastomosis. The bladder drainage technique greatly facilitates early diagnosis of
rejection by providing a means to measure the output of amylase from the graft, as determined by the urinary amylase
activity.
Regardless of the type of graft transplanted (either whole organ or segmental), most transplant surgeons agree that graft
should be placed intraperitoneally. The extensive surface of the peritoneum is probably of considerable help in absorbing the
exudate that escapes from the surface of the pancreas. The incidence of anastomatic leaks and wound complications has
been greatly reduced with the intraperitoneal placement of grafts.

52. The term “tolerance” refers to responses observed which include long-term graft acceptance without the need for
chronic immunosuppression. There are a variety of specific ways in which T and B lymphocytes can be tolerant or
nonresponsive to antigen. Which of the following is/are mechanisms of tolerance?

a. Clonal abortion
b. Clonal deletion
c. Clonal anergy
d. Suppression
Answer: a, b, c, d

Clonal abortion refers to the developmental process whereby nascent T and B cell clones, which recognize autoantigen with
high affinity, are eliminated. Clonal deletion may encompass the processes of clonal abortion but it also refers to the
elimination of mature T and B cell clones. Clonal anergy is a state in which the potential relative reactive clones and their
receptors are physically present but fail to respond to antigen. Suppression generally refers to an active process in which a
leukocyte and/or its soluble products inhibit the development or effector function of immune lymphocytes.

53. Which of the following statement (s) is/are true concerning currently approved immunosuppressant agents?

a. Azathioprine (Imuran) is useful in the treatment of acute ongoing rejection


b. Methylprednisolone is particularly useful in immunosuppression as it has lesser toxicity than Prednisone
c. Cyclosporine blocks transcription of several early T-cell activation genes
d. FK-506 is both more potent and less toxic than cyclosporine
e. The monoclonal antibody OKT3 interferes with T-cell antigen recognition function
Answer: c, e

The major principle of immunosuppression is to induce the patient with high doses of drugs at the time of allografting in
order to prophylax rejection. The drugs are then reduced rapidly within a period of days to weeks to less toxic maintenance
levels. The anti-metabolite azathioprine (Imuran) interferes with nucleic acid metabolism inhibiting proliferation and clonal
expansion of activated lymphocytes, eliminating alloantigen specific immune responses. This agent is used during induction
immunosuppression and for maintenance immunosuppression but has little role for treating an acute, ongoing rejection.
Glucocorticoids are the mainstays of virtually all immunosuppressive regimens. All glucocorticoids have similar
immunosuppressive actions and none is more effective than any other at equipotent doses. Complications and side effects
are equivalent at all equipotent doses. Cyclosporine inhibits the rotamase activity of cyclophilin. Therefore the major
immunosuppressive activity of cyclosporine is to block transcription of several early T-cell activation genes. The macrolide
antibiotic, FK-506 is 10-100 times more potent than cyclosporine on a molar basis but it too is associated with a number of
significant and similar toxicities. Antibodies are given for only short periods of time to prophylax rejection and to treat acute
ongoing rejection. There are two major types of antibody preparations—polyclonal antibodies such as antilymphocyte
(ALG) or antithymocyte globulin (ATG) or monoclonal antibodies. The only monoclonal antibody currently available is
OKT3 which is the used for both induction and treatment of rejection and is the most efficacious agent currently available
for the treatment of rejection.

54. Which of the following patients would be a candidate for a liver transplant?

a. A 48-year-old man with end-stage liver disease secondary to non-A, non-B hepatitis
b. A 35-year-old man with both primary sclerosing cholangitis and ulcerative colitis and end-stage liver disease
c. A 22-year-old woman with fulminant hepatic failure secondary to acetaminophen overdose
d. A 4-year-old child with congenital biliary atresia having failed a previous Kasai procedure
e. A 48-year-old patient with alcoholic cirrhosis and a 2.5 cm central unresectable hepatoma
Answer: a, b, c, d, e

In the absence of contraindications, virtually any disease resulting in liver failure is amenable to liver transplantation.
Primary sclerosing cholangitis is a common indication for transplantation since there is no other effective treatment. The
common association with inflammatory bowel disease can somewhat complicate the timing of the procedure, however, in
general hepatic transplantation does not affect the outcome of the ulcerative colitis. Non-A, non-B hepatitis is the most
common form of hepatitis leading to liver transplantation. Recurrence of viral hepatitis in the transplanted liver occurs, but
usually follows an indolent course. Biliary atresia is by far the most common indication for hepatic transplantation in
pediatric patients. Recommended treatment includes creation of a portoenterostomy (Kasai procedure), if this can be done
before three months of age. After this point, success rates diminish markedly. Patients without a satisfactory course, multiple
revisions of the portoenterostomy should be avoided to facilitate subsequent transplantation. The most common cause of
fulminant hepatic failure are non-A, non-B hepatitis, hepatitis B, and various drug toxicities. In the latter group,
acetaminophen toxicity is particularly prominent. Primary hepatic malignancy, most often hepatoma, is sometimes an
indication for transplantation but the results are usually worse than in other disease states because of recurrent disease.
Transplantation is justified in the occasional case in which the tumor is central but relatively small, if the patient is
otherwise healthy, and there is no evidence of extrahepatic disease after exhaustive evaluation.

55. Which of the following statement(s) is/are true concerning changes in physiology following lung transplant?

a. In patients with pulmonary hypertension, changes in right ventricular function and pulmonary artery pressure takes
weeks to months to resolve
b. In single lung transplantation, changes in pulmonary function are seen almost immediately following
transplantation
c. Patients with double lung transplants have both better pulmonary function studies as well as better exercise
capabilities
d. After single-lung transplant, ventilation perfusion mismatch persists and carbon dioxide retention is seen
Answer: b

Performing single-lung transplantation in a patient with pulmonary hypertension has been particularly illustrative in
demonstrating the potential for reversal of right ventricular dysfunction. As soon as the lung is implanted, the morphology
of the right ventricular changes significantly as assessed by transesophageal echocardiography. The intraventricular septum,
previously bulging into the left ventricle, immediately assumes the normal position. An increase in contractility of the right
ventricle occurs with significant decrease in dilatation. The pulmonary artery pressure immediately decreases and is
essentially normal by the time the patient leaves the operating room.
One would also expect significant ventilation perfusion mismatch to occur with ventilation to the native lung occurring
preferentially because the native lung is significantly more compliant. Conversely, perfusion should preferentially go to the
newly transplanted lung because of lower pulmonary vascular resistance. Despite this occurrence, patients with this
operation do well from a functional standpoint. By three months after transplantation, the ventilation/perfusion mismatch
narrows. Despite this mismatch, patients do not demonstrate carbon dioxide retention. From a clinical standpoint,
improvement in pulmonary function is seen almost immediately after transplantation. The measurement most often used is
FEV1 and marked improvement is seen within two weeks. The FEV1 essentially triples and then remains fairly stable.
Improvement after bilateral lung transplant is slightly better. Although patients who receive two lungs may do better on
pulmonary function tests, this benefit is not translated to significantly better exercise capability.

56. Current clinical protocols determine a limited number of variables and parameters for matching and allocation of donor
organs to potential recipients. Which of the following statement(s) is/are true concerning aspects of immunity
important for clinical transplantation?

a. HLA matching is important for kidney, pancreas, and liver transplantation


b. A cross match assay determines if there are preformed antibodies in the recipient’s serum which will react with
antigens on the cell surface of the potential donor’s lymphocytes
c. A patient with a history of multiple transfusions or previous transplant will have a high panel reactive antibody
(PRA)
d. A normal heterozygous individual with a complete donor-recipient match will have a four-antigen match
Answer: b, c
ABO compatibility is obviously required for successful transplantation. The central position of the MHC in immune
regulation suggests that HLA matching is also very important for allografting. There is significant data to prove that HLA
matching is important for kidney and pancreas transplantation. There is good data also to show that HLA matching is not
important for liver transplantation and does not affect graft survival. The main loci typed are HLA-A, HLA-B, and HLA-
DR. Thus, for a normal completely heterozygous individual this results in six antigens typed and a complete donor-recipient
match is referred to as a six-antigen match. An important test for graft compatibility is the cross match. This assay
determines if there are preformed antibodies in the potential recipient’s serum which will react with antigens on the cell
surface of the potential donor’s lymphocytes. A positive cross-match means that such antibodies are present and that
hyperacute rejection will ensue if the transplant were to be performed. Another important test which is also a reflection of
the presence of host anti-donor antibodies is the panel reactive antibody (PRA). Most recipients on transplant lists send
serum samples to the transplant center on a regular basis which are tested against a panel of typing cells of known HLA
specificities. Most individuals should have no anti-HLA antibodies and have a low PRA (0–5%). Patients who have been
transfused, pregnant, previously transplanted, or have an autoimmune disorder which induces a lot of antibodies might have
a high PRA (50–99%). The presence of a very high PRA suggests that a patient is likely to have a positive cross-match.

57. T-lymphocytes are divided into two main sub-classes: CD4+ and CD8+. Which of the following statement(s) is/are
true concerning these classes of T-cells?

a. CD4+ T-cells are restricted to recognizing antigens of the class II major histocompatibility complex (MHC)
b. CD8+ T-cells perform primarily cytotoxic functions
c. CD4+ 8+ double positive cells are well-differentiated mature cells
d. CD4+ T-cells also perform suppressor functions
Answer: a, b, d

T-cells are divided into two main sub-classes: CD4+ and CD8+. CD4+ 8+ double positive cells are usually immature T-cells
or thymocytes while the fully differentiated T-cell is usually single positive. Because of molecular interactions, CD4+ T-
cells are restricted to recognizing antigens in the context of class II major histocompatibility complex (MHC) and usually
perform roles related to B-cell help, T-cell help, and inflammatory responses such as delayed and contact hypersensitivity.
CD8+ T-cells are restricted to class I MHC and perform cytotoxic functions. In addition, experimental studies have
demonstrated that both CD4+ and CD8+ T-cells can act as T suppressor cells.

58. Correct statement(s) concerning postoperative complications after hepatic transplantation include:

a. Primary nonfunction occurs in 5 to 10% of transplanted livers in the immediate postoperative period
b. A biliary leak, although a common complication, is usually of minimal clinical importance
c. Portal vein thrombosis occurs much more commonly than hepatic artery thrombosis
d. If postoperative bleeding is encountered, immediate return to the operating room is indicated
Answer: a

Primary nonfunction of the allograft occurs in about 5% to 10% of transplanted livers. Most cases of nonfunction are related
to inadequate tissue preservation or occult organ dysfunction in the donor but a sizeable percentage may arise from
immunologic mechanisms. In the worst case scenario, the patient does not regain consciousness, a coagulopathy ensues, and
multiple organ failure develops. Liver enzymes show hepatocellular injury with SGOT and SGPT values in the range of
5000 to 10,000 and little bile production. Hepatic artery thrombosis occurs in 5% of adult hepatic transplantation cases and
up to 25% of pediatric cases. Postoperative vein thrombosis is much less common than hepatic artery thrombosis, occurring
in 2% to 3% of cases. Laparotomy to control postoperative bleeding is required in 15% of cases. In about half of the
reoperations, a specific bleeding point is identified. Survival is higher in these cases in contrast to those in which diffuse
bleeding is encountered, presumably since the latter circumstance is usually associated with poor allograft function and
resultant coagulopathy. If significant bleeding occurs after hepatic transplantation, a common and sensible policy is to
transfuse the patient until hypothermia and coagulopathy are corrected with subsequent (one to three days) evacuation of
blood from the peritoneal cavity. Biliary leakage is a feared complication, with a high (50%) mortality. The high mortality
may be the result of a concomitant hepatic arterial thrombosis and infection of the leaked bile, or difficulty of bile duct
repair in the area of inflamed tissue.

59. Which of the following statement(s) is/are true concerning renal transplantation?
a. Living-related donor transplants typically can be expected to have one-year graft survival rates of over 90%
b. Preconditioning of the recipient with the use of donor-specific blood transfusions from their living donor improves
graft survival and therefore should be used routinely
c. Pre-transplant blood transfusions result in improved graft survival following cadaveric renal transplant in the
cyclosporine era
d. Age of the recipient over 50 years is generally associated with a poorer outcome due to graft rejection
Answer: a

The use of living-related donor renal transplant has multiple advantages including improved short-and long-term graft
survival, routine immediate allograft function, and fewer rejection and infectious episodes. Nearly all transplantation centers
that perform living-related donor transplantations report one-year graft survival rates of over 90%. The use of
preconditioning of the recipient with donor-specific blood transfusions from their living donor can improve graft survival.
The major drawback to this maneuver is the development of recipient anti-donor antibodies (sensitization) which occurs in
nearly one-third of recipients. The development of sensitizing antibodies eliminates the use of that donor. With the
introduction of cyclosporine, the use of donor-specific transfusions with subsequent immunosuppression, was compared to
nontransfused recipients treated with cyclosporine and prednisone. These investigations have demonstrated excellent graft
survival rates over long-term follow-up and therefore routine donor-specific transfusions are seldom performed in adults. In
the azathioprine and prednisone immunosuppression era, several immunologic and nonimmunologic risk factors were
identified as having an adverse effect on graft outcome. Historically, older renal allograft recipients (older than 50 years) did
poorly compared with younger counterparts. Much of the graft loss was found to be associated with patient deaths, and
usually was the result of overwhelming infection. With the cautious use of cyclosporine and prednisone, however, excellent
patient and graft survival rates are now reported. Data from the azathioprine and prednisone era show a clear-cut benefit
from improved graft survival after multiple random blood transfusions. More recent studies again showed no advantage to
blood transfusion when cyclosporine is used. Since transfused patients have a risk of developing anti-HLA antibodies, these
patients may become more difficult to undergo organ transplantation in a timely fashion.

60. Which of the following statement(s) is/are true concerning clinical syndromes of rejection?

a. Hyperacute rejection occurs with kidney, heart, liver and lung transplants
b. The histologic characteristics of acute rejection include lymphocyte infiltration accompanied by plasma cells,
eosinophils, or neutrophils
c. Vascular atherosclerosis and obliteration are characteristic of chronic rejection
d. Transplantation across major ABO incompatibility will result in hyperacute rejection of a renal or cardiac
transplant
Answer: b, d

Hyperacute rejection is the result of pre-formed antibody binding to the allograft at the time of revascularization in the
operating room. Complement is activated resulting in endothelial cell destruction, vascular leak, recruitment of platelets and
neutrophils, thrombosis of vessels, and destruction of the graft in a period of minutes to hours. Kidney, heart, pancreas, and
lung allografts are all susceptible to hyperacute rejection; however, liver grafts are relatively resistant to this process and are
often transplanted across antibody differences and even across an ABO difference. Acute rejection usually occurs days to
weeks after transplantation and is initiated by T-cell dependent immunity characterized microscopically by lymphocytic
infiltration accompanied by plasma cells, eosinophils, and a few Mast cells or neutrophils. Chronic rejection usually occurs
months to years after transplant. It is characterized by loss of normal histologic structure, fibrosis and atherosclerosis.
Chronic rejection is the major cause of graft failure and patient loss with all organs.

61. Which of the following statement(s) is/are true concerning techniques for multiple organ procurement and
preservation?

a. The liver and pancreas are generally removed en bloc and separated as a bench procedure
b. Renal allograft function is improved by the use of machine perfusion
c. UW (University of Wisconsin) cold storage solution is the method of choice of most programs for hepatic and
pancreatic transplantation
d. Cardiac allografts have the shortest limit of cold ischemia
Answer: a, c, d
The complexity of multiple organ procurement involves the coordination of at least two teams (thoracic and abdominal).
The liver and pancreas are generally removed en bloc with the organs separated as a bench procedure, retaining the celiac
axis for the liver. The kidneys are also removed en bloc. Studies indicate that post-transplantation renal allograft function is
similar regardless of whether simple hypothermia or the more cumbersome technique of machine perfusion are used. For
decades, the primary solution used for cold storage preservation of kidneys was Euro-Collins solution. Recently, a new
solution, UW solution, has been developed with ingredients designed to provide high-energy phosphate precursors,
hydrogen ion buffering capacity, and anti-oxidant properties. Although the advantage of this solution over Euro-Collins
solution for kidneys is unclear, UW solution is used as the preservation method of choice by nearly all programs performing
hepatic and pancreatic transplantations. Both organs can reliably be stored for 24 hours. Kidneys can generally be safely
stored for 36 to 48 hours before transplantation. Cardiac preservation has changed relatively little in recent years.
Hyperkalemic crystalloid cardioplegia solution is used at 4°C and four hours is generally the accepted limit of cold
ischemia. The current limit of cold ischemia for small bowel is approximately 12 hours.

62. Which of the following statement(s) is/are true concerning the outcome of renal transplantation?

a. Two-thirds of all graft losses alone (without death) occur from one to six months after transplantation
b. The most common cause for graft loss after one year following transplantation is patient death
c. Most patient deaths following transplantation are related to immunosuppression
d. An acute rejection episode in a renal allograft recipient is the most important clinical event, determining both
short-term and long-term graft survival
e. The period between the six months and one year following transplantation is the most critical time period
following renal transplant
Answer: a, b, d

There are two ways to lose a renal allograft—graft loss alone and death of the patient regardless of the degree of graft
function at the time of death. Two-thirds of all graft losses alone (without death) occur from one to six months after
transplantation. Only 14% of all graft losses occur after one year. In contrast, half of the patient losses (most dying with
functioning grafts) occur more than one year after transplantation. More than half the deaths are due to cardiovascular
complications not related to immunosuppression but closely related to comorbid cardiovascular variables present at the time
of transplantation. Less than 25% of deaths are related to immunosuppression. The period between one and six months after
transplantation is the most active and crucial time in the clinical course of a patient with a renal transplant. During this time
63% of all graft losses, 22% of deaths, and 74% of all acute rejection episodes occur. An acute rejection episode in a renal
allograft recipient is the single most important clinical event determining both short and long-term graft survival. The post-
transplant period that begins at six months and continues to the one year mark is the quiescent time with very few influential
clinical events. Only 9% of all graft losses and 9% of all acute rejection episodes occur during this time period.

63. Which of the following characteristics or conditions will exclude a patient as a suitable cadaveric organ donor?

a. Active systemic bacterial infection


b. Primary CNS malignancy
c. Age over 65
d. History of prior cholecystectomy for a possible hepatic donor
Answer: a

The characteristics of a suitable cadaveric organ donor can be divided into those that are general in nature and those that are
organ-specific. Broadly stated, the general attributes of an acceptable organ donor include the establishment of a diagnosis
of brain death, previously good general health, and relative hemodynamic stability from the time of the advanced
precipitating brain death until organ procurement is complete. As experience has been gained with donors considerably less
than ideal, it has become apparent that arbitrarily defined chronological age limits for organ donors are unnecessary.
Active systemic infection is an absolute contraindication to organ donation. Documented positive blood cultures for known
systemic infection that has not been completely eradicated rule out the potential organ donor because of risk of transmission
of infection to an immunosuppressed recipient. Furthermore, all potential organ donors, regardless of whether they are
considered high risk, should be tested for infection with human immunodeficiency virus as well as hepatitis B and C.
Cancer, whether treated or not, has long been considered to contravene organ donation. The only exception to this rule has
been the donor with a primary malignancy of the central nervous system.
The condition of particular organs in great measure dictate their individual suitability for transplantation. Preexisting hepatic
disease can usually be identified before organ procurement. A history of hepatitis or cirrhosis of any kind preclude donation.
Although calculous biliary tract disease would appear at first blush to be a contraindication of hepatic procurement, prior
cholecystectomy for uncomplicated cholelithiasis is not an absolute contraindication to liver donation.

64. Which of the following statement(s) is/are true concerning associated renal and pancreatic transplantation?

a. The most important advantage is the use of renal function as an early indicator of pancreatic graft rejection
b. After renal transplant, there is no additional risk associated with immunosuppression
c. A major disadvantage of simultaneous renal/pancreatic transplant is the potential adverse effect on renal allograft
as the result of a pancreatic complication
d. A diabetic with a renal transplant continues to be at risk for diabetic nephropathy
Answer: a, b, c, d

In the patient with a functional renal transplant, because of the need for long-term immunosuppression, the demonstration of
a prior allograft acceptance and a continued risk for recurrent diabetic nephropathy are compelling reasons to offer
pancreatic transplantation. The advantages to simultaneous renal-pancreatic transplantation compared to a sequential
procedure (renal followed by pancreatic) include 1) the recipient’s need to accept only one set of donor antigens; 2) the
ability to monitor rejection of the pancreas by identifying the well-recognized signs of renal allograft rejection; 3) the
immunosuppressive effect of uremia; 4) transportation in patients who have not been maintained on chronic
immunosuppression; and 5) a single albeit longer anesthetic exposure. Of these advantages, the most important is the use of
renal function as an early indicator of pancreatic graft rejection. The disadvantages of simultaneous renal-pancreatic
transplantation include extensive surgery in a uremic diabetic patient and the potential adverse effect on renal allograft
function as the result of a pancreatic complication. Ideally, pancreatic transplantation should be performed in patients who
do not yet have, but are designed to develop, secondary complications to diabetes that are more serious than the potential
side-effects of immunosuppression. In recipients of a pancreas after a kidney, the only risks of pancreatic transplant are
related to the surgery since immunosuppression is already obligatory.

65. There are numerous toxicities and adverse effects associated with immunosuppression. Which of the following
statement(s) is/are true concerning complications of immunosuppression?

a. Transplant recipients are susceptible primarily to infections with unusual organisms (fungus, virus, atypical
bacteria)
b. Immunosuppressive agents may blunt the inflammatory response to infection leading to a late presentation of an
infectious process
c. The development of malignancy appears primarily due to direct mitogenic effects of the agent
d. Lymphomas are the most common malignant tumors developing in the transplant patient
e. Graft-vs-host disease is a progressive condition and extremely difficult to treat
Answer: b

The most obvious complication of immunosuppression is infection. As immunosuppression becomes stronger and more
effective, the recipient’s ability to resist infection diminishes. Transplant recipients are susceptible both to typical bacterial
infections (UTI, pneumonia, wound infections) and to infections with unusual organisms (fungus, virus, atypical bacteria).
Immunosuppressives also block the inflammatory response to infection so that patients present with very subtle signs and
symptoms or they present late in the infectious process.
Another complication in allograft recipients is malignancy. The immunosuppressive drugs do not appear to be directly
mitogenic or transforming, but rather probably suppress immune mechanisms which keep transformed cells in check.
Squamous cell carcinomas of the exposed area of the skin are by far the most common malignancy. Lymphomas are the next
most common tumor and are 10–100 times more common in transplant recipients than in the general population. These are
usually non-Hodgkins B cell lymphomas and are often related to malignant transformation by Epstein-Barr virus (EBV).
Another complication of organ allografting is graft-vs-host disease (GVHD). GVHD is usually self-limited as donor cells,
stimulated by the host alloantigen, are eliminated either by immunosuppression or by host anti-donor responses.

66. Which of the following statement(s) is/are true concerning the results of lung transplantation?

a. One year survival following single lung transplant is significantly better than following bilateral transplant
b. The worst survival is seen in patients with pulmonary hypertension
c. Patients with cystic fibrosis have a markedly poorer result than do patients with emphysema
d. Infection is a common cause of mortality in both the early and late post-transplant period
Answer: b, d

In just over ten years since the first successful lung transplant, approximately 3000 transplants have been performed.
Overall, one-year actuarial survival following lung transplant is approximately 70% (single lung = 70%; bilateral lung =
74%). At two years, survival drops to 63%. Patients with emphysema have the best survival at one and two years while
those with pulmonary hypertension had the worse (77% vs 61%). Patients with cystic fibrosis do almost as well as the group
with emphysema (72%). Overall there is some continuing to fall off in survival at three years with an overall survival of
57% which drops to 51% at four years and 46% at five years. Causes of recipient death can be categorized according to the
time frame in which they occur. Early (less than 90 days following transplant) death most commonly results from bacterial
infection. Infection also accounts for approximately one-third of late deaths (greater than 90 days) following transplantation.
A similar percentage results from manifestations of chronic rejection and obliterative bronchiolitis.

67. Categories of patients in which pancreatic transplantation is applicable includes:

a. Diabetics with a functioning renal transplant


b. Diabetic patients with end-stage renal disease requiring renal transplantation
c. Nonuremic diabetics with other complications of their disease
d. Well-controlled adult onset diabetics
Answer: a, b, c

Pancreatic transplantation can be applied to three categories of patients. In the first category are diabetic patients who
already have undergone successful renal transplantations. In the patient with a functioning renal transplant, because of the
need for long-term immunosuppression, the demonstration of prior allograft acceptance, and the continued risk of recurrent
diabetic nephropathy, are compelling reasons to offer pancreatic transplantation. The second group of patients are those with
end-stage renal disease requiring renal transplantation. These people may benefit either from simultaneous or sequential
renal-pancreatic transplantation. The final and largest potential group of patients are nonuremic diabetics with other
complications of their disease.

68. Which of the following statement(s) is/are true concerning the results of cardiac transplantation?

a. Overall one-year survival is approximately 80%


b. Survival following transplant in the pediatric age group is significantly worse than in adults
c. There is no difference in survival when cardiac transplantation is performed in a heterotopic position versus an
orthotopic position
d. The survival rate for retransplantation is approximately 50%
Answer: a, d

Collected data from a multi-center registry has shown that the overall one-year survival following cardiac transplantation is
80%. Overall five-year survival is approximately 65%. Survival in patients receiving heterotopic cardiac transplants is
significantly lower than in patients receiving hearts in the orthotopic position. The overall one-year survival rate for
retransplantation as reported from the same registry is only 54%. In the pediatric age group, actuarial survival at two years
is 80% and 76% at five years.

1. Cytotoxic T cells (CTL) are capable of recognizing:


A. Peptide antigens associated with major histocompatibility complex (MHC) molecules.
B. Membrane-bound antigens.
C. Cytoplasmic antigens.
D. Nuclear antigens.
E. All of the above.
Answer: E

DISCUSSION: It was long thought that CTL recognize antigen expressed on the surface of the target cell destined to be
killed, similar to the recognition of an antigen by an antibody; however, it was found that the mechanism of CTL
recognition of antigens was fundamentally different from the mechanism of antibody recognition. CTL can detect antigens
derived from cell surface–associated proteins, but in addition can recognize proteins that are normally in the cytoplasm or in
the nucleus. In fact, the normal location of the protein can be anywhere within the cell.
To understand how a CTL can distinguish an antigen from a protein that is normally located within the cytoplasm or nucleus
of the cell, investigators had to determine exactly what was being recognized by the CTL. T cells were found to recognize
short linear fragments of processed or even denatured protein. It was found that proteins that were synthesized
endogenously were degraded within the cytoplasm into 9– or 10–amino acid long peptides. These peptides were then
transported to the endocytoplasmic reticulum and associated with newly synthesized MHC class I molecules. Certain
peptides could fit within the MHC class I molecule and were then transported to the cell surface as a complex. It was this
complex, consisting of a 9– to 10–amino acid peptide within a MHC class I molecule that was presented to, and recognized
by, CD8+ CTL.

2. Adoptive immunotherapy with lymphokine activated killer cells (LAK) and tumor infiltrate (TIL) cells are characterized
by:
A. Nonspecific stimulation of effector cells.
B. Expansion ex vivo of large numbers of lymphocytes.
C. Infusion with interleukin 2 (IL-2).
D. Significant toxicity at high doses.
E. All of the above.
Answer: E

DISCUSSION: LAK cells, generated by short-term culture of peripheral blood lymphocytes in the presence of high
concentrations of IL-2, lyse transformed target cells, and have minimal lytic activity for most normal tissues. Up to 10 11 in
vitro generated LAK cells have been administered in a single intravenous infusion to cancer patients. Therapeutic trials have
also combined short courses of high-dose systemic IL-2 administration with LAK cell transfer to promote LAK function
and viability, with apparently enhanced efficacy. The shortcomings of LAK and IL-2 therapy included a larger degree of
toxicity (including pulmonary, renal, and hepatobiliary) with a significant proportion of patients requiring intensive care unit
admissions and 2% to 5% treatment-related mortality. Despite this, response rate remained relatively low.
Another therapy using in vitro expanded lymphocytes derived from a TIL has been evaluated in clinical trials. In humans,
TIL cell lines have been generated by mincing tumor specimens and culturing eluted lymphocytes with high concentrations
of IL-2. TIL lines can be expanded to 10 8 to 10 11 cells over 3 to 8 weeks in culture, and some lines appear to function as T
cells with lytic specificity for autologous—but not allogeneic—tumor targets, whereas others function as LAK cells and
lyse both autologous and allogeneic tumor targets. Adoptive transfer of 5 × 10 10 TIL alone has not been associated with
significant toxicity, and administration of 5 × 10 10 TIL cells with concurrent systemic IL-2 has caused toxicities that are
attributable to the IL-2.

3. Previous clinical studies with cancer vaccines have:


A. Clearly demonstrated induction of tumor-specific immune response.
B. Repeatedly demonstrated clinical response to large tumor burden.
C. Not clearly demonstrated induction of tumor-specific immune response.
D. Not been performed to date.
Answer: C

DISCUSSION: Several different approaches in experimental animal studies in the 1970s formed the basis for human
clinical trials of SAI against human cancers. Although several thousand patients have been injected with a variety of tumor
cell preparations in this country and elsewhere during the past 25 years, the complexity of the studies has made it difficult to
definitively assess the value of this approach to cancer therapy. Nonetheless, a number of clinical trials have suggested a
therapeutic benefit of SAI. Most of these trials have two features in common: (1) a therapeutic effect was not seen or, if
seen, has not been confirmed independently; and (2) no acceptable information was provided on the presence of tumor
antigens in the vaccines and the immune response of patients to these antigens.

4. Which of the following statements is/are true of the epidemiology and etiology of melanoma?
A. Most patients are diagnosed after age 60 years.
B. Skin color has no association with risk of melanoma.
C. Sun exposure is the only risk factor for melanoma.
D. The per capita incidence of melanoma is highest in Australia.
Answer: D

DISCUSSION: The median age at diagnosis is in the late forties; so, a minority of patients are diagnosed after age 60. The
risk of melanoma is closely tied to skin color: Caucasians are at highest risk, and those with a Celtic complexion at even
higher risk. Australia has a large Celtic population living near the equator, and it has the highest per capita incidence of
melanoma in the world. Sun exposure is believed to be an important cause of melanoma, but the data remain incomplete.
Some melanomas arise in sites not exposed to the sun (e.g., mucous membranes), so sun exposure is not the only risk factor.

5. Which of the following variables best predicts prognosis for patients with a recent diagnosis of cutaneous melanoma and
no clinical evidence of metastatic disease?
A. Breslow thickness.
B. Clark's level.
C. Ulceration.
D. Gender.
E. Celtic complexion.
Answer: A

DISCUSSION: The prognosis for melanoma is best predicted by the thickness, measured in millimeters (Breslow
thickness). Clark's level and ulceration are also predictive, but less so. Gender is a secondary prognostic factor. Skin color
may have a mild impact on outcome, but it is primarily a risk factor for developing melanoma.

6. A 38-year-old man presents with a melanoma on the skin of the right calf measuring 5 mm. in thickness. Several large
nodes are palpable in the right inguinal region. Which of the following statements about the appropriate management of this
clinical problem is false?
A. In the absence of systemic disease, the primary melanoma of the right calf should be excised with at least a 2-cm.
margin.
B. Complete right inguinal node dissection should be performed if there is no evidence of systemic metastasis.
C. If further work-up reveals multiple lung metastases of melanoma, they should be excised as soon as possible.
D. Chemotherapy for melanoma is primarily palliative; so surgical therapy is preferred if there is no evidence of metastatic
disease beyond the inguinal region.
E. If the nodes do not contain metastatic disease but are simply reactive, the chance of 5-year survival is 50% or less.
Answer: C

DISCUSSION: This patient should be evaluated with computed tomography in addition to a careful history, review of
systems, and physical examination. If there is no evidence of metastatic disease beyond the inguinal region, therapy should
focus on the local and regional disease. The primary lesion should be excised with a 2- to 3-cm. margin. The enlarged nodes
can be evaluated by fine-needle aspiration (FNA) or node excision. If the FNA or excisional biopsy does show metastatic
melanoma, then complete inguinal node dissection should be performed. Alternatively, complete node dissection can be
justified simply on the basis of clinical findings. Chemotherapy does not provide any significant chance of cure, whereas a
significant proportion of patients with isolated nodal metastases will live long. The depth of the primary lesion would
predict chances of 5-year survival at less than 50%, even if the nodes are not metastatic. If work-up does reveal multiple
pulmonary nodules, chemotherapy or experimental systemic therapies should be considered. Surgery has no role in the
therapy of multiple lung metastases of melanoma.

7. A 42-year-old woman presents with an 8 cm. × 6 cm. × 4 cm. mass in the posterior thigh. Incisional biopsy reveals a
high-grade liposarcoma. Her management should include:
A. High thigh amputation.
B. Extracompartment excision with negative margins.
C. Complete excision with negative margins.
D. Adjuvant radiation therapy.
E. Adjuvant chemotherapy.
Answer: CD

DISCUSSION: This is a common presentation of soft tissue sarcoma. Surgery remains the principal modality for curative
therapy for all soft tissue sarcomas. Provided the entire tumor is removed, less radical procedures have not been
demonstrated to adversely affect local recurrence or outcome. The surgical objective is complete removal of the tumor with
negative margins and maximal preservation of function. Adjuvant radiation improves local control. Adjuvant chemotherapy
has not proven to be efficacious.

8. Biologic features of adult soft tissue sarcomas include the following:


A. Mutations of p53 in metastatic liposarcoma.
B. A low (less than 1%) risk of metastasis for small, low-grade lesions.
C. Recurrent disease in at least 33% of patients.
D. Lymph node metastasis in less than 3% of patients.
E. Mutations of p53 in Li-Fraumeni syndrome.
Answer: BCDE

DISCUSSION: Inactivation of the p53 tumor suppressor gene is involved in tumorigenesis of several sarcomas, but not
specifically liposarcoma. The relevance of this gene is underscored by the frequent occurrence of soft tissue sarcomas in the
Li-Fraumeni syndrome, where all families studied have p53 germ line mutations. Low-grade lesions have a low (less than
1%) risk of subsequent metastasis, whereas high-grade lesions have a high (greater than 50%) risk of subsequent metastasis.
Despite optimal multimodality treatment, at least 33% of patients develop recurrent disease (median disease-free interval 18
months). Lymph node metastasis occur in fewer than 3% of adult soft tissue sarcomas.

9. Which of the following statements describes an ideal tumor marker?


A. The ideal tumor marker should be tumor specific; that is, in the normal population or patients with benign diseases,
false-positive test results are rare.
B. The ideal marker must have a low false-negative rate; that means that all patients with a particular type of cancer should
test positive.
C. The circulating level of an ideal tumor marker should correlate directly with the amount of viable tumor and be a
measure of the response to therapy.
D. The ideal tumor marker should act as a prognostic indicator.
E. All of the above.
Answer: E

DISCUSSION: Each of the above is a criterion for an ideal tumor marker.

10. A marker for the diagnosis of pancreatic cancer is:


A. CA 15-3.
B. CA 19-9.
C. Alphafetoprotein (AFP).
D. Carcinoembryonic antigen (CEA).
E. CYFRA 21-1.
Answer: B

DISCUSSION: CA 15-3 is a marker for breast cancer. AFP is a marker for hepatocellular and testicular cancer. CEA is a
marker for colon cancer. CYFRA 21-1 is a marker for non–small-cell lung cancer. CA 19-9 is a marker for pancreatic
cancer.

11. Which of the following tumors may cause elevated CEA levels?
A. Breast cancer.
B. Colorectal cancer.
C. Gastric cancer.
D. Lung cancer.
E. All of the above.
Answer: E

DISCUSSION: CEA is relatively nonspecific, and each of the listed cancers can cause elevated levels of CEA.

12. The presence of which marker is a significant poor prognosis variable for patients with breast cancer:
A. CEA.
B. C-erb B-2.
C. AFP.
D. Human chorionic gonadotropin (hCG).
E. RB-1.
Answer: B

DISCUSSION: Some 20% to 30% of breast cancers have C-erb B-2 gene expression. Studies have demonstrated that it is an
independent prognostic tumor marker. Tumors that express C-erb B-2 have a poorer prognosis.

13. The most useful circulating marker for patients with hepatocellular carcinoma is:
A. CA 50.
B. Levels of vitamin B 12.
C. CEA.
D. AFP.
E. hCG.
Answer: D

DISCUSSION: Eighty percent of patients with hepatocellular carcinoma have elevated serum levels of AFP.

14. In patients with colorectal cancer the serum CEA level is a clinically useful measure for all reasons except:
A. Prognosis.
B. Detection of recurrence.
C. Guiding second-look operations.
D. Following treatment response.
E. Early diagnosis.
Answer: E

DISCUSSION: CEA levels may be used for each of the indications except early diagnosis.

15. Which serum markers are useful in the management of patients with testicular cancer?
A. hCG.
B. AFP.
C. CA 15-3.
D. Two of the above.
E. None of the above.
Answer: D

DISCUSSION: AFP and hCG are useful in managing testicular cancer. Approximately 90% of patients with testicular
cancer have elevated levels of one or both of these markers.

16. Which tumor marker is useful for the management of patients with breast cancer?
A. CA 125.
B. Inhibin.
C. CA 19-9.
D. CA 15-3.
E. CEA.
Answer: D

DISCUSSION: CA 15-3 is the only useful marker for patients with breast cancer. Approximately 73% of patients with
breast cancer have elevated levels.

17. A new marker that has possible utility in the management of patients with non–small-cell lung cancer (NSCLC) is:
A. Calcitonin.
B. Neuron-specific enolase.
C. CYFRA 21-1.
D. Glucagon.
E. Chromogranin A.
Answer: C

DISCUSSION: CYFRA 21-1 is a new marker for non–small-cell lung cancer (NSCLC). Approximately 40% to 50% of
patients with NSCLC will have elevated levels.

18. A circulating marker that may be useful in the management of patients with any neuroendocrine tumor is:
A. Chromogranin A.
B. Neuron-specific enolase.
C. hCG.
D. Two of the above.
E. None of the above.
Answer: D

DISCUSSION: Although most endocrine tumors have a specific marker hormone that is the mainstay of diagnosis, both
chromogranin A and neuron-specific enolase may be useful as serum markers in patients with all types of neuroendocrine
tumors.

19. A 65-year-old man is seen two years following right hemicolectomy for a Duke’s B-2 carcinoma of the cecum.
Although asymptomatic, the CEA level has risen four-fold from a value obtained six months previously. Computed
tomography reveals a single, 3 cm lesion in the right hepatic lobe. There is no evidence of extra-hepatic metastatic
disease and the patient undergoes right hepatic lobectomy. Which of the following correctly represents the chance of
overall 5 year survival?

a. 15%
b. 33%
c. 50%
d. 66%
Answer: b

Surgeons have a definite role in excising isolated metastatic lesions, especially those in the brain, lung, liver, and soft
tissues. Patient selection is critical and such operations should be performed only when the potential benefits clearly
outweigh the risks. The best results are obtained when surgical excision of metastatic lesions can be combined with effective
chemotherapy. Several institutions have demonstrated a 20% to 30% five-year survival rate in selected patients whose
pulmonary metastases from sarcomas, melanomas, colon cancers, and testicular carcinomas have been resected. In a recent
national survey of patients undergoing liver resection for colorectal metastases, the overall five-year survival rate was 33%.
Excision of metastatic tumors to the brain is also indicated in selected patients, especially when combined with irradiation.

20. The most serious long-term side effect of bleomycin therapy is which of the following?

a. Pulmonary fibrosis
b. Cataract formation
c. Cardiomyopathy
d. Aplastic anemia
Answer: a

While any chemotherapeutic drug may exhibit a diverse array of toxicities, most classes of drugs are known for particular
adverse effects. These are often graded by their severity from 0 (no toxicity) through 4 (life-threatening toxicity); grade 5
toxicity is equivalent to death. Most therapies allow for dosage reductions for toxicities greater than grade 2 or 3; one
exception is the therapy of acute leukemia, where a grade 4 hematologic toxicity is expected. Some toxicities are only
observed with prolonged drug administration, such as anthracycline-induced cardiotoxicity, bleomycin-and nitrosourea-
related pulmonary fibrosis, or secondary leukemias associated with the nitrosoureas, etoposide, or mechlorethamine
(nitrogen mustard), or when drugs are given at very high doses (nitrosourea-related hepatic veno-occlusive disease), or are
given regionally (chemical peritonitis, arteritis, or biliary sclerosis).

21. Which of the following malignancies have declined in incidence in the United States over the past two decades?

a. Breast carcinoma
b. Gastric carcinoma
c. Endometrial carcinoma
d. Prostate cancer
e. Carcinoma of the uterine cervix
Answer: b, c, e

Estimates of cancer incidence in the United States reveal that in men, lung, prostate, and colorectal carcinomas have the
highest incidence. Breast carcinoma accounts for 29% of all cancer in women, followed by colorectal, lung, and uterine
carcinomas. From 1973 to 1987, there was a 15% increase in incidence for all cancer sites. Cutaneous melanoma showed
the greatest increase at 83%, and non-Hodgkin’s lymphoma and prostate carcinoma followed with increases of about 50%.
Only a few cancers declined in incidence over the same interval. Carcinoma of the cervix showed the greatest reduction at
36%, followed by endometrial carcinoma and gastric carcinoma. The reduction in carcinoma of the cervix is likely due to
the early detection and treatment of cervical dysplasia resulting from the widespread use of routine cervical cytological
screening.

22. Oncogenes have been implicated in the development of a number of human neoplasms. Oncogene activation is
believed to be required for oncogenesis. Which of the following potential mechanisms are relevant to these processes?

a. Chromosome translocation
b. DNA point mutation
c. Amplification
d. Gene deletion
Answer: a, b, c

Cellular proto-oncogenes are key regulatory components in cell growth, and many proto-oncogenes are expressed during
periods of proliferation, development, and differentiation. Abnormal proto-oncogene function or activation may, therefore,
result in the deregulated proliferation and dedifferentiation characteristic of the neoplastic state. Various activation
mechanisms can change a normal gene into an oncogene. Point mutations (single nucleotide changes in DNA) are one such
mechanism. For example, members of the ras gene family are some of the most frequently activated oncogenes in solid
human tumors. The ras gene products are plasma membrane proteins capable of binding guanosine triphosphate (GTP).
These proteins, called G proteins, are involved in intracellular signal transduction. In tumors with ras activation, the
difference in the oncogene compared to the normal gene is a single nucleotide change. Point mutations resulting in single
amino acid substitutions in the ras protein change a normal protein into one with transforming activity.
Another important mechanism of oncogene activation is chromosomal translocation, the shifting of a segment of one
chromosome to another chromosome. One of the most well-known examples is the formation of the Philadelphia
chromosome in chronic myelogenous leukemia. The proto-oncogene c-abl is translocated from chromosome 9 to the bcr
locus on chromosome 22. Transcription of the new bcr/abl locus results in the formation of a fusion protein. This new
protein has enhanced tyrosine kinase activity and transforming capabilities. In other tumors, an oncogene may be found in
an increased number of gene copies per cell, termed amplification. DNA amplification results in an increased level of gene
expression. Amplification of N-myc has been found to correlate with prognosis in patients with neuroblastoma.

23. A 45-year-old woman undergoes excision of a 1 cm breast mass. Histologic examination reveals invasive ductal
carcinoma. Flow cytometric analysis is also performed which determines that a fraction of the tumor cells are
“aneuploid”. The patient asks for an explanation of this term. Which of the following is/are correct?

a. The cells have a DNA content 1 times the baseline content


b. The cells have hyperchromatic nuclei
c. The cells have a DNA content 2 times the baseline content
d. The cells have squamous morphology
e. The cells have a DNA content not an even multiple of baseline content
Answer: e

The development of flow cytometry (FCM) has enabled analysis and sorting of individual cells. FCM is now widely used
clinically to rapidly measure the DNA content and estimate the proliferative rate of normal cells and populations of cells
comprising malignancies. Normal human resting somatic cells contain 46 chromosomes and a baseline diploid DNA content
of 1; cells in mitosis are tetraploid, with a DNA content of 2; and cells undergoing DNA synthesis have a DNA content
between 1 and 2. Tumor cells often exhibit gross chromosomal karyotypic changes in both structure and number, which is
frequently reflected in a DNA content other than 1 or an even multiple of 1, known as aneuploidy.
The clinical application of FCM attempts to identify aggressive populations or subpopulations of tumor cells within a
localized primary lesion that may predict early-stage patients at risk for micrometastatic disease. For most solid tumors, it
has been difficult to demonstrate the presence of aneuploidy as negative prognostic variables independent from other
negative prognostic variables, such as high nuclear grade, vascular or lymphatic invasion, or the presence of regionally
involved lymph nodes. There does seem to be a subpopulation of node-negative breast cancer tumors with aneuploidy that
are more biologically aggressive other node-negative breast cancers.

24. A 45-year-old man with long-standing gastroesophageal reflux undergoes upper endoscopy that reveals patchy
areas of epithelium resembling gastric mucosa extending 5 cm proximal to the esophagogastric junction. Biopsies are
obtained. The pathologic report describes “Barrett’s epithelium”. Which of the following processes does this finding
represent?

a. Cellular hyperplasia
b. Cellular hypertrophy
c. Metaplasia
d. Carcinoma in situ
Answer: c

Hyperplasia is an increase in cell number. For example, an increase in growth rate over baseline is a normal response in
wound healing. Liver regeneration after hepatic injury or hepatectomy results in both a dramatic hyperplastic response with
an increase in cell number and a hypertrophic response with an increase in cell size. Metaplasia is a reversible
transformation of one mature cell type to another in a given tissue. Epithelial metaplasia secondary to chronic inflammation
has been extensively investigated. In cases of chronic gastroesophageal reflux, the normal distal esophageal squamous cell
mucosa may undergo metaplasia to a “gastric-type” columnar epithelium. In the respiratory tract, the normal
pseudostratified columnar epithelium may be replaced by squamous cells. Dysplasia is a term usually applied to epithelial
tissues, characterized by altered cell size, shape, and organization. Dysplasia is classified as mild, moderate, or severe,
depending on the degree of cellular dedifferentiation. The gastrointestinal tract, respiratory tree, urinary bladder, cervix,
vagina, breast, and the skin all may exhibit foci of dysplasia. Nuclear polymorphism and hyperchromatism are present and
often accompanied by total loss of cellular and epithelial polarity. Mitoses are more frequently seen in the afflicted areas
than in the surrounding normal epithelium. Although disordered growth of superficial tissue is present, there is no
penetration of abnormal cells through the epithelial basement membrane. Carcinoma in situ demonstrates all of the above
changes, with an increased number of mitoses, but it is still confined by the basement membrane and may be considered the
most disordered extreme of dysplasia.

25. Which of the following statements regarding the inherited form of retinoblastoma is/are correct?

a. Retinoblastoma results from amplification of the H-ras oncogene


b. Clinical disease results after chromosomal loss in a retinal cell after birth
c. Retinoblastoma results from the loss of a tumor suppressor gene
d. Clinical disease results from chromosomal translocation
Answer: b, c

Genetic events leading to oncogene activation result in either increased gene expression or increased activity of the
oncoprotein. Another distinct class of genes has been described as negative regulators of cell proliferation. These genes have
been referred to as tumor-suppressor genes. The loss of these genes allow tumor development by the loss of normal growth-
inhibitory signals.
Genetic analysis of families with hereditary and sporadic retinoblastomas (Rb) has provided evidence for the role of tumor-
suppressor genes in human tumors. In inherited retinoblastomas, an abnormal Rb gene is transmitted to half the offspring.
As long as a normal allele is present, the tumor does not develop, and the abnormal germ-line mutation behaves in a
recessive fashion; however, a second mutation of the normal allele (somatic cell mutation) may occur in a retinal cell,
leading to the development of the tumor. In sporadic retinoblastomas, development of a tumor requires two somatic
mutations. The genetic loss in retinoblastomas involves deletion of DNA from chromosome 13q14.
Using the technique of restriction-fragment-length polymorphism (RFLP), tumor suppressor genes may be identified
through the loss of heterozygosity of the allele in question or a specific marker associated with that allele. Heterozygosity
refers to the presence of two different alleles for a given gene. In the Rb gene example, presence of heterozygosity at the Rb
locus prevents tumor development because of the influence of the normal wild-type allele. Loss of this wild-type allele in a
second genetic event results in the absence of a normal tumor suppressor gene product and subsequent tumor formation.

26. Analysis of metastatic tumor cells has revealed expression of factors promoting tissue invasion. Which of the
following is/are among such factors?

a. Collagenase
b. Plasminogen activator
c. Fibroblast growth factor
d. Interleukin-2
Answer: a, b

Evidence for the involvement of tissue-degradative enzymes in neoplastic invasion is convincing. The production and
activation of specific tissue-destructive enzymes such as lysosomal hydrolases and collagenases has been documented in
tumor invasion. Destruction of host tissue by hydrolytic enzymes, aided by pressure atrophy and the occlusion of blood and
lymph vessels by an expanding tumor mass, facilitates infiltration of neoplastic tissue. Histologic examination of tissues
obtained from sites of tumor invasion displays considerable variation in the degree of tissue damage, and many human and
animal malignant neoplasms express higher levels of lytic enzymes than benign tumors or corresponding normal tissues.
Lysosomal catheptic activities are elevated within some tumor tissues, and increased production of cathepsin B in breast
carcinomas has been observed compared with that in normal or benign tissue. Enhanced production and secretion of the
serine protease plasminogen activator has been associated with the neoplastic transformation of a variety of cell types.
Both intravasation and extravasion are pivotal in hematogenous, systemic metastasis. In either instance, the tumor cells are
confronted with a barrier significantly comprised of type IV collagen. Type IV collagen is a major structural protein of
basement membranes between parenchymal cells and the connective tissue on which cells rest. A strong correlation exists
between the ability of murine tumor cells to produce spontaneous metastases and their ability to produce increased levels of
collagenase IV. Tumor cells that invade blood vessels or leave capillaries of distant organs in which they have lodged must
penetrate the basement membrane. Dissolution of the basement membrane, suggestive of enzymatic action, has been
observed in areas adjacent to arrested tumor cells.

27. The most common complication that requires alteration of planned chemotherapy regimens is which of the
following?

a. Pulmonary fibrosis
b. Gastrointestinal ulceration
c. Hematologic suppression
d. Hepatotoxicity
Answer: c

In chemotherapy regimens, it is important not only to deliver adequate doses with each treatment cycle but also to deliver
them in a timely fashion. The interval between each treatment cycle is determined by the toxic effects experienced by
normal tissues and the amount of time required for resolution of the effects. The scheduling of treatment courses is crucial.
If a course follows too closely upon the preceding one, additive toxicities are noted, much to the detriment of the patient,
and often the ability to deliver further therapy is at least temporarily compromised. For most agents, the dose-limiting
toxicity is myelosuppression, usually leukopenia and/or thrombocytopenia. Nadir blood counts are reached approximately
14 days after the initiation of each cycle and begin to improve 3 to 5 days later, often with complete resolution by day 28.
The resiliency of the bone marrow reserve is dependent upon prior chemotherapy and radiotherapy. Agents such as
busulfan, mitomycin C, procarbazine, and the nitrosoureas display delayed myelosuppression, often up to four weeks after
the initiation of therapy and lasting several weeks. Therapy with these drugs may be delivered only every 6–8 weeks; in
addition, therapy with these agents may lead to chronic cumulative myelosuppression and, in some cases, marrow failure.
28. Resistance of tumors to multiple chemotherapeutic agents is often due to the MDR (Multiple Drug Resistance)
gene. This gene encodes a protein that acts by which of the following mechanisms?

a. As a transmembrane efflux pump for chemotherapeutic agents


b. As a DNA repair molecule
c. As an isoform of superoxide dismutase
d. As a membrane stabilizer
Answer: a

Resistance to chemotherapeutic agents may become manifest during the administration of drugs of very different classes,
with a variety of presumed intracellular targets. Such resistance, a function separate from the state of proliferation of the
malignant cells, has been termed multidrug resistance. This form of resistance is mediated by the cell surface protein, P-
glycoprotein, a product of the MDR-1 gene. A magnesium-dependent ATPase, P-glycoprotein acts as a transmembrane
efflux pump and appears to serve as a cellular detoxifier and possibly as a chloride pump. P-glycoprotein causes the
extrusion of diverse agents such as anthracyclines, epipodophyllotoxins, Vinca alkaloids, and actinomycins out of the cell
before they are able to reach their intracellular targets. The protein is normally found on the luminal surface of such organs
as the colon and rectum, small intestine, proximal tubules of the kidney, and gravid uterus; the acinar and bile canalicular
surfaces of pancreatic and hepatic parenchymal cells, respectively; and in cells of the adrenal cortex, where there is no
polarity to the cell surface location. P-glycoprotein has likewise been found in tumors derived from these organs.
P-glycoprotein can be induced in vitro by exposing cell lines to increasing concentrations of various agents, a phenomenon
which also extends to tissues and cell lines not known to normally express significant amounts of P-glycoprotein, such as
melanoma, ovarian carcinoma, small cell carcinoma of the lung, and adenocarcinoma of the breast.

29. The high incidence of hepatitis B infection in Africa and parts of Asia is thought to be causally associated with
increased incidence of which of the following malignancies?

a. Hepatocellular carcinoma
b. Esophageal cancer
c. Burkitt’s lymphoma
d. Gastric carcinoma
Answer: a

Although a genetic predisposition exists for some cancers, most available information favors environmental variation as the
major contributor to the disparate geographic incidence of different cancer types. In many cases, the high incidence of a
specific cancer in a particular region or country is linked to a specific causative agent. The chewing of a betel nut, tobacco,
and lime mixture in regions of India has resulted in a high incidence of oropharyngeal tumors. The high incidence of gastric
carcinoma in Japan is linked to a diet of smoked and highly salted foods, and food contaminated with aflatoxin. The high
incidence of hepatitis B infections in parts of Africa and China is thought responsible for the high incidence of
hepatocellular carcinoma in these regions. Geographic differences in several other common solid tumors are less clearly
explained. Breast and colon carcinomas are common in North America, but rare in Africa and Asia. Carcinomas of the
stomach, esophagus, and cervix are more common in developed countries.

30. Workers exposed to asbestos are at increased risk for which of the following tumors?

a. Thoracic mesothelioma
b. Bladder carcinoma
c. Laryngeal carcinoma
d. Testicular carcinoma
e. Non-Hodgkin’s lymphoma
Answer: a, b, c

Exposure to asbestos fibers or dust markedly increases the risk for neoplastic disease. Among the inorganic carcinogens,
asbestos is unusual in its induction of otherwise rare mesotheliomas. In addition to the mesothelioma, workers exposed to
asbestos show increased incidence of bladder carcinoma, gastrointestinal tract neoplasms, and laryngeal and esophageal
tumors. Clinical and experimental model investigations reveal that asbestos may act as a primary carcinogen or as a
cocarcinogen. Thus, individuals who smoke tobacco and are exposed to asbestos fibers increase their risk beyond that
associated with the individual susceptibilities.
31. The Lynch Syndrome is also known as hereditary non-polyposis colorectal cancer. Which of the following is/are
features of this syndrome?

a. Left sided colon cancers


b. Autosomal dominant inheritance
c. Multiple polyps beginning in adolescence
d. Multiple cutaneous nevi
Answer: b

The most widely investigated familial cancer of the colon is the heredity non-polyposis colorectal cancer (HNPCC).
HNPCC syndrome is divided into two main categories: (1) hereditary site-specific colon cancer, or Lynch Syndrome I, and
(2) colorectal cancer in association with other forms of cancer or Lynch Syndrome II. The main features of Lynch Syndrome
I are an autosomal dominant model of heredity, no associated polyposis, right-sided colon cancers, multiple colon cancers,
and long survival. In Lynch Syndrome II, many of the characteristics are the same as Syndrome I, with the added burden of
increased risk for endometrial, ovarian, stomach, and urinary tract tumors. The HNPCC group comprises nearly 5% of all
colorectal cancers, and in some centers, this figure approaches 15 to 20%. The potential for control of this form of colorectal
cancer lies in use of computer registries and clinical surveillance of the high risk group. The molecular basis of HNPCC has
been identified as defective DNA mismatch repair gene(s).

32. A number of clinical factors have been noted to decrease sensitivity of tumors to the effects of ionizing radiation.
Which of the following is most important in this regard?

a. Increased tissue vascularity


b. High tumor mitotic rate
c. Tissue hypoxia
d. Subcutaneous tumor location
Answer: c

The radiosensitivity of cells depends on their position within the cell cycle. Cells in the M phase (mitotic phase) are most
sensitive. At the time of any given dose, only a fraction of cells will be in a vulnerable position within the cell cycle.
Subsequently, the sensitivity of the tumor versus normal tissue depends on the ability of cells to redistribute and repopulate
within the radiated volume. In tumors that are particularly radiosensitive, this balance between cell killing and repopulation
favors the normal tissues over the tumor. The ability to recruit cells from adjacent undamaged areas should also favor
normal tissues.
The presence or absence of molecular oxygen greatly influences the proportion of cells killed by a given dose of radiation.
The availability of oxygen is clearly the most important factor for enhancing the formation and prolonging the survival of
free radicals. Therefore, radiosensitivity of cells within a given tumor varies based on their location relative to oxygen-
carrying capillaries. Cells in hypoxic regions of a tumor may escape the effects of radiation. As conditions change with
progressive cell killing, fewer cells theoretically exist in hypoxic regions, and oxygen may become more evenly distributed
within the tumor volume.

33. Brachytherapy involves the delivery of radiation therapy locally via specially designed catheters placed in direct
apposition to the treated tissue. The most common radioisotope used in this application is which of the following?

a. 125I
b. 14C
c. 3H
d. 34P
Answer: a

Brachytherapy is a technique in which the source of the radiation is adjacent to or within the targeted tissue. In this method,
isotopes such as 198Au and 125I are placed in special catheters, the positioning of which is based on precise geometric
considerations. The placement of the radiation source is critical because, in principle, a high dose of energy is delivered to
the immediate vicinity of the target and then decreases rapidly with distance. The advantage of this technique is the ability
to deliver high, concentrated doses to the target while limiting damage to nearby normal tissue. Clinical applicability is
somewhat limited, but there is considerable experience with brachytherapy in the treatment of some oral tumors,
gynecologic malignancies, and soft tissue tumors of the extremities.

34. Patients that have acquired immunodeficiency syndrome are at increased risk for which of the following
neoplasms?

a. Colorectal cancer
b. Meningioma
c. Kaposi’s sarcoma
d. Hepatocellular carcinoma
e. Esophageal carcinoma

Answer: c

The role of the immune system in the development of neoplastic disease has been the subject of extensive debate. Clinical
observations and the results from experimental animal models are not completely congruent. For example, patients with
immune deficiency or immunosuppressed transplant recipients are at greater risk for neoplastic disease. Frequently these are
tumors of the lymphoreticular system, and only selected nonlymphoid tissues exhibit increased incidence of neoplasia.
Patients with acquired immunodeficiency syndrome and some transplant patients are at an increased risk for Kaposi’s
sarcoma.

35. DNA viruses have been implicated as etiologic agents in several human tumors. Evidence for a causative role
exists for which of the following neoplasms?

a. Burkitt’s lymphoma
b. Testicular carcinoma
c. Cervical carcinoma
d. Osteogenci sarcoma
e. Esophageal carcinoma
Answer: a, c

Three groups of DNA viruses have oncogenic strains. The papovaviruses include the papilloma, polyoma, and simian
viruses, which induce tumors in animals. The second group is made up of adenoviruses, and the third group comprises the
herpesviruses. Adenoviruses have been isolated from a variety of animal tumors; most of these are tumorigenic in newborn
animals. An example of the herpesvirus group is the Epstein-Barr virus (EBV), which has been implicated in the etiologies
of Burkitt lymphoma and nasopharyngeal carcinoma. The World Health Organization has indicated that previous and
continued exposure to EBV results in high titers of antibody to the virus capsid antigen and a 30-fold increase in risk
compared with control populations.
Herpes simplex virus type 2 (HSV-2) is the etiologic agent of genital herpes. Infected women with early sexual activity and
a large number of sexual partners have significantly increased risk for cervical carcinoma. Women with invasive carcinoma
have antibodies to HSV-2, and about 40% of cervical biopsies with severe dysplasia or carcinoma display HSV-2-specific
DNA-binding protein. A similar relation has been established for human papovavirus.

36. When a chemotherapeutic agent is stated to have caused a partial response this implies what degree of reduction in
measurable tumor volume?

a. 0–9%
b. 10–29%
c. 30–49%
d. 50–99%
Answer: d

A partial response is regarded as a 50%–99% reduction in all bidimensionally measurable disease, lasting at least four
weeks or one treatment cycle. A complete response is defined as a total disappearance of all signs and symptoms associated
with all malignant lesions, and complete resolution of all laboratory parameters associated with the disease, lasting at least
four weeks or one treatment cycle. A minor response is a reduction of less than 50% of all bidimensionally measurable
disease, or a reduction of 50% or greater which does not last at least four weeks or a treatment cycle. A minor response is
not considered significant, though it may indicate potential biologic activity. Neither a minor nor a partial response is
curative, though they may be associated with palliation of symptoms. The presence of a residual mass does not always
indicate viable disease, as it may be composed of necrotic or fibrotic tissues.

37. Which of the following represent obstacles to the use of retroviruses in therapeutic gene transfer?

a. Viral receptors may not be present on target cell membranes


b. For integration, the host cell must undergo mitosis
c. Virus particles are labile
d. Viral purification is difficult

Answer: a, b, c, d

There are several limitations of recombinant retroviruses which make them unsuitable for some gene transfer protocols.
First, retrovirus entry requires that target cells contain the appropriate viral receptor. In many cases, these receptors are not
known. Difficulties in efficiently transducing certain target cell populations may be due to low levels or absence of
appropriate virus receptors. A second requirement for efficient retroviral gene integration is the process of cell proliferation.
The dependence of integration on mitosis is thought to be due to the need for nuclear membrane breakdown to enable the
viral integration complex to enter the nucleus. Production is another problem, as retrovirus particles are relatively labile
when compared to other viruses. In general, retroviruses cannot be purified without significant loss of infectivity.

38. Which of the following statements regarding alpha-1-antitrypsin deficiency is/are correct?

a. Alpha-1-antitrypsin is a plasma elastase inhibitor


b. Most homozygous patients develop chronic obstructive pulmonary disease
c. The spleen is the primary site of alpha-1-antitrypsin synthesis
d. Intracellular accumulation of abnormal protein occurs in hepatocytes
Answer: a, b, d

Alpha 1-antitrypsin (a-1-AT) is a plasma protease inhibitor with elastase as a major physiological substrate. Synthesis, post-
translational modification, and secretion occur primarily in the hepatocyte. Disease-causing alleles are inherited in an
autosomal recessive manner. Homozygotes have only 15 to 20% of the normal plasma levels of a-1-AT. Defects in normal
a-1-antitrypsin expression and processing, brought about as a result of mutation, can lead to pathology in the lung and liver.
Diminished levels of a-1-AT in the blood result in an imbalance between proteases and protease inhibitors, allowing
destruction of lung parenchyma. Chronic obstructive pulmonary disease is the most common clinical manifestation of a-1-
AT deficiency, with basal lung parenchyma most severely affected.
The liver is the primary site of a-1-AT biosynthesis. In some cases, alteration in the protein sequence of a-1-AT prevents
normal post-translational processing and results in the intracellular accumulation of large quantities of abnormal protein,
detectable as intracytoplasmic inclusions. One pathologic consequence of abnormal protein accumulation is the
development of liver injury.

39. Which of the following statements regarding retroviruses is/are correct?

a. The genetic material contained within a retrovirus is RNA


b. Inside the host cell the viral RNA is converted to single-stranded DNA
c. Proviral DNA is integrated into the host chromosome
d. Retroviruses can be used to transfect both replicating and non-replicating cells
Answer: a, c

Retrovirus particles are composed of a ribonucleic acid (RNA) genome encapsidated into a complex virus particle structure
containing both viral and cellular components The virus enters cells primarily on the basis of interactions between viral coat
proteins and complementary proteins on the host cell membrane. Once internalized, the viral RNA is converted to a double
stranded DNA sequence, the provirus. The proviral DNA is integrated into the host chromosome by means of an integrase
protein utilizing proviral DNA sequences known as long terminal repeats. An important point is the need for host cell
replication for successful proviral DNA integration.

40. Which of the following statements relating to adenoviruses is/are correct?

a. Adenoviral infection is a common cause of upper respiratory tract infection


b. Adenoviral genetic material consists of double-stranded DNA
c. Adenovirus can be produced in large quantity and easily purified
d. Adenoviral infection requires host cell mitosis
Answer: a, b, c

There are over 40 human adenovirus serotypes, many of which are pathogenic in humans. Diseases caused by adenoviruses
include hepatitis, conjunctivitis, upper respiratory tract infection, and diarrhea. Multiple exposures to adenoviruses
throughout life result in the development of humoral immunity.
Human adenovirus contains 36 kilobases (kb) of double stranded DNA. Adenoviruses are internalized by receptor-mediated
endocytosis and transported to the nucleus where the immediate early genes are expressed. Unlike retroviruses, adenoviral
infection does not require host cell division. Early gene units express a variety of proteins involved in the regulation and
control of the viral life cycle. The concerted activities of the early genes contribute to initiation of the late phase of viral
replication whose hallmark is the onset of DNA replication. Adenoviral mRNA undergoes extensive post-transcriptional
processing, leading to expression of five sets of late proteins comprising structural components of the virion. Production of
the virus, which can be grown in large quantities and highly purified, is relatively easy.

41. Which of the following statement relating to cystic fibrosis is/are correct?

a. Cystic fibrosis is inherited as an X chromosome-linked recessive trait


b. Cystic fibrosis is caused by a defective chloride channel
c. Cystic fibrosis is caused by defective acetylcholine receptors
d. Cystic fibrosis is inherited as an autosomal recessive trait
Answer: b, d

Cystic fibrosis is a lethal autosomal recessive disorder caused by mutations of the cystic fibrosis transmembrane
conductance regulator (CFTR) gene. The CFTR gene encodes a 170 kDa protein that functions as a cAMP-regulated
chloride-channel on the apical surface of epithelial cells.

42. Hemophilia B has been treated in a pre-clinical model by gene transfer for which deficient clotting factor?

a. Factor II
b. Factor VII
c. Factor IX
d. Factor X
Answer: c

Hemophilia B is a chromosome X-linked blood clotting disorder which results when the gene encoding Factor IX is
deficient or functionally defective. The enzyme is synthesized in the liver, and the existence of animal models for this
genetic disease have permitted the development of somatic gene therapy protocols aimed at transfer of functional copies of
the factor IX gene into the liver. A recombinant retroviral vector was used for the transfer of human factor IX cDNA in
rabbit hepatocytes. The infected rabbit hepatocytes produced human factor IX that was indistinguishable from functional
protein derived from human plasma.
Factor IX hepatic gene transfer has also been accomplished in vivo by the direct infusion of recombinant retroviral vectors
into the portal vasculature. Canine factor IX cDNA was transduced directly into the hepatocytes of affected dogs in vivo.
The dogs transduced with canine factor IX recombinant retrovirus constitutively expressed canine factor IX for 5 months
after the procedure at a high enough concentration to improve the whole blood clotting time and partial thromboplastin time.

43. Familial hypercholesterolemia has been proposed as a disease to be treated by gene therapy. The molecular basis of
familial hypercholesterolemia is which of the following?
a. Absence of hepatic low density lipoprotein receptors
b. Overproduction of high density lipoprotein
c. Absence of lipoprotein lipase
d. Overproduction of hepatic ornithine transcarbamylase
Answer: a

The molecular basis of familial hypercholesterolemia is a mutation in the gene that encodes the low density lipoprotein
(LDL) receptor. Patients who inherit one abnormal allele have moderate elevations in plasma LDL and suffer premature
coronary heart disease. The prevalence of heterozygotes in most populations is 1 in 500, representing approximately 5% of
patients under 45 who suffer myocardial infarction. Patients with two abnormal LDL receptor alleles have severe
hypercholesterolemia and life-threatening coronary artery disease. Characterization of mutant alleles has revealed a variety
of mutations including deletions, insertions, missense mutations, and nonsense mutations. Class 1 mutations are associated
with no detectable protein and are often caused by gene deletions. Class 2 mutations lead to abnormalities in intracellular
processing of the protein. Class 3 mutations specifically affect binding of the ligand LDL, and class 4 mutations encode
receptor proteins that do not internalize within hepatocytes.

44. Antisense oligodeoxynucleotides have been proposed as agents for cancer-directed gene therapy. When delivered
intracellularly, antisense molecules act to block which of the following?

a. Transcription
b. Translation
c. Post-translational processing
d. Ribosylation
Answer: b

Many strategies for oncogene-directed cancer gene therapy involve the use of antisense oligodeoxynucleotides. Instead of
engineering a virus or other gene delivery vehicle, a piece of DNA is inserted which is complementary to messenger RNA
molecules. The antisense molecules bind to the sense mRNAs so that mRNA translation cannot occur. Should the bound
mRNA subsequently escape its antisense partner, translation could proceed. Antisense molecules are essentially drugs which
do not require virus-mediated gene transfer. The oligonucleotides can be made resistant to nuclease degradation.

45. Which of the following viruses is/are considered to be neurotropic?

a. Adenovirus
b. Herpes simplex virus
c. Retrovirus
d. Adeno-associated virus
Answer: b

The ability to manipulate the expression of genes in the mammalian brain leads to opportunities to treat neurologic
disorders. DNA viruses have been used for most attempts at gene therapy of non-malignant CNS disease. The herpes
simplex type I (HSV I) has been used extensively because of its neurotropic host cell range. Recombinant HSV vectors
retain functional viral genes which may be cytotoxic or may reactivate preexisting latent virus in recipient cells. To
circumvent these problems, defective HSV viruses have been engineered to eliminate viral genes while retaining certain
recognition sequences.

1. Polyhydramnios is frequently observed in all of the following conditions except:


A. Esophageal atresia.
B. Duodenal atresia.
C. Pyloric atresia.
D. Hirschsprung's disease.
E. Congenital diaphragmatic hernia.
Answer: D

DISCUSSION: Polyhydramnios is defined as excessive amounts of fluid (>2000 ml.) in the amniotic sac during pregnancy.
The amniotic pool is a dynamic pool with a relatively rapid turnover. In the fourth intrauterine month the fetus begins to
swallow amniotic fluid (25% to 40% of the volume) and absorbs the fluid from the upper gastrointestinal tract. The fluid is
urinated back out into the amniotic pool by the fetal kidneys and a functioning bladder. Although there are maternal causes
of polyhydramnios (cardiac failure, renal failure, other causes of fluid retention) and some idiopathic cases, many instances
are related to the presence of fetal anomalies. These include central nervous system problems such as anencephaly, which
prevents normal swallowing, and any high alimentary tract obstruction that blocks the passage of the amniotic fluid and
prevents its absorption (including esophageal atresia, pyloric atresia, and duodenal atresia). In addition, infants with
congenital diaphragmatic hernia have obstructions due to herniation of the stomach and bowel into the thoracic cavity. This
is a poor prognostic finding in these infants. Hirschsprung's disease is a form of low intestinal obstruction, and therefore an
adequate length of proximal patent intestine is available for absorption of the swallowed amniotic fluid and polyhydramnios
is usually not present.

2. Which of the following statements about Hirschsprung's disease is/are true?


A. There are no ganglion cells seen in Auerbach's plexus.
B. There is an increased incidence of Down syndrome.
C. It is more common in girls.
D. It may be associated with enterocolitis.
E. It may involve the small intestine.
Answer: ABDE

DISCUSSION: The affected segment of bowel in patients with Hirschsprung's disease has hypertrophic nerves in
Auerbach's intermyenteric plexus, but no ganglion cells are present. Ganglion cells are also absent in Meissner's submucosal
plexus. Some 3% to 5% of babies with Hirschsprung's disease also have Down syndrome. Hirschsprung's disease should be
suspected in infants with Down syndrome that manifest evidence of abdominal distension and constipation. Hirschsprung's
disease is much more common in boys (4:1). The enterocolitis of Hirschsprung's disease is a condition associated with delay
in diagnosis, low bowel obstruction, severe abdominal distension, explosive diarrhea, and colonic mucosal ulceration. The
course may be fulminant. This complication is associated with increased morbidity and mortality. Bacterial translocation
and endotoxemia may complicate the condition. Treatment includes nasogastric suction, intravenous fluids, antibiotics, and
rectal tube decompression of the obstructed rectosigmoid segment. In approximately 10% of cases aganglionosis extends
into varying lengths of small bowel. In rare instances, the entire small bowel and colon may be aganglionic.

3. Which of the following statements is/are true of infants with gastroschisis?


A. It is associated with malrotation.
B. There is a high incidence of associated anomalies.
C. There is prolonged adynamic ileus following repair.
D. It is complicated by intestinal atresia in 10% to 12% of cases.
E. It is associated with chromosomal syndromes.
Answer: ACD

DISCUSSION: Because of intrauterine herniation of bowel to an extra-abdominal location, normal intestinal rotation and
fixation do not occur. Most infants with gastroschisis have nonrotation. In contrast to infants with omphalocele, in which a
high incidence of associated anomalies coexist, babies with gastroschisis have little else wrong. Following repair of the
abdominal wall defect, infants with gastroschisis have a long delay in return of intestinal function. They usually require total
parenteral nutrition to supply adequate caloric intake until gut function returns (3 to 4 weeks). Intestinal atresia is observed
in 10% to 12% of neonates with gastroschisis. This is caused by bowel ischemia due to intrauterine volvulus or compression
of the herniated viscera in a small, tight defect in the abdominal wall. Although infants with omphalocele frequently have
chromosomal syndromes such as Beckwith syndrome or trisomy 13 to 15 or 16 to 18, babies with gastroschisis do not.

4. In neonates with congenital diaphragmatic hernia, which of the following statements is true?
A. The defect is more common on the right side.
B. Survival is significantly improved by administration of pulmonary vasodilators.
C. An oxygen index of 20 is an indication for extracorporeal membrane oxygenation (ECMO).
D. Oligohydramnios is a frequent occurrence.
E. Mortality is the result of pulmonary hypoplasia.
Answer: E
DISCUSSION: In infants with congenital diaphragmatic hernia the defect is more common on the left side (85%).
Polyhydramnios is sometimes noticed and is a poor prognostic indicator of survival. Oligohydramnios is noted in fetuses
with urinary tract obstruction and may be associated with pulmonary hypoplasia with an intact diaphragm. Although
pulmonary vasodilators were used extensively in babies with congenital diaphragmatic hernia, they have not significantly
improved survival. An oxygen index of greater than 40 is the usual indication for ECMO. Pulmonary hypoplasia is the main
cause of mortality in babies with congenital diaphragmatic hernia.

5. Which of the following statements are true regarding the premature neonate?
A. A 15% to 20% right-to-left shunt occurs across the foramen ovale and patent ductus arteriosus.
B. Surfactant levels are normal after 30 weeks' gestation.
C. Fluid requirements are higher than in a full-term baby.
D. Rectal temperature is the best indicator of core body temperature.
E. They are more at risk for infection than the full-term infant.
Answer: ACE

DISCUSSION: The newborn infant has a relatively elevated pulmonary artery pressure and shunts a significant amount of
unoxygenated blood through the foramen ovale and patent ductus arteriosus. The normal PaO 2 below the ductus, as
measured through an umbilical artery catheter, would be between 60 and 80 mm. Hg. Surfactant levels do not approach
normal until after the 34th week of gestation, when enzyme levels in the surfactant pathway mature. Amniocentesis is
performed to measure the lethicin-to-sphingomyelin ratio (L:S ratio) and determine whether maturation has occurred. Fluid
requirements in the premature infant are between 140 and 150 ml. per kg. per day in comparison with those of the normal
neonate in whom 80 ml. per kg. per day would be adequate. Increased insensible losses and the need for overhead warmers
play a role in this increase. Axillary or skin probe temperature monitoring is more accurate than the rectal temperature in the
neonate. The rectal temperature is not a good indicator of core body temperature until approximately 18 months of age.
Premature infants lack immunoglobulin A (IgA) and have low levels of IgM, the C3b component of complement, and
decreased opsonins. In addition, the leukocytes have reduced phagocytic ability, creating an increased risk of infection.
Escherichia coli and beta-hemolytic streptococcus are the two most common infectious agents affecting the neonate.

6. In neonates with necrotizing enterocolitis, which of the following findings is an indication of significant bowel ischemia?
A. Increased gastric residuals.
B. Septic shock.
C. Cardiac failure due to a patent ductus arteriosus.
D. Elevated platelet count.
E. Erythema of the abdominal wall.
Answer: E

DISCUSSION: Necrotizing enterocolitis (NEC) is a condition that occurs in 2% of babies admitted to neonatal intensive
care facilities. Increased gastric residuals can occur for a number of reasons and are seen as an early indicator of NEC, but
they may not reflect the presence of ischemic bowel. Septic shock may be due to a wide variety of causes besides NEC.
Cardiac failure due to patent ductus arteriosus may predispose to NEC but is not necessarily an indicator of ischemic bowel.
Most babies with NEC have a progressive decrease in their platelet count in association with bowel ischemia. Erythema of
the abdominal wall is an indication for surgical exploration and is consistent with NEC with perforation and inflammation
of the peritoneum and abdominal wall.

7. The treatment of choice for neonates with uncomplicated meconium ileus is:
A. Observation.
B. Emergency laparotomy, bowel resection, and Bishop-Koop enterostomy.
C. Intravenous hydration and a gastrograffin enema.
D. Emergency laparotomy, bowel resection, and anastomosis.
E. Sweat chloride test and pancreatic enzyme therapy.
Answer: C

DISCUSSION: Meconium ileus is a form of intestinal obstruction that occurs in 10% to 15% of neonates with cystic
fibrosis. The obstruction is related to intraluminal concretions of abnormal meconium. The treatment of choice is adequate
hydration and evacuation with a hypertonic gastrograffin enema. The hyperosmolar contrast material causes an outpouring
of fluids into the bowel lumen, which flushes out the obstructing meconium and negates the need for laparotomy.
Observation alone is not a useful method of treatment. When gastrograffin evacuation fails, laparotomy, placement of a
pursestring suture in the bowel wall, and intraluminal irrigation with saline and gastrograffin (administered through a
catheter inserted through a small enterotomy within the pursestring) will often clear the obstructing meconium. This
obviates the need for resection or enterostomy in most cases. Postoperatively, a sweat chloride test should be obtained to
confirm the diagnosis of cystic fibrosis. Pancreatic enzyme should be given when diet is initiated.

8. The pentalogy of Cantrell includes all of the following except:


A. Epigastric omphalocele.
B. Sternal cleft.
C. Intracardiac defect.
D. Pericardial cyst.
E. Ectopia cordis.
Answer: D

DISCUSSION: The pentalogy of Cantrell includes an epigastric-located omphalocele, ectopia cordis, anterior
pleuropericardial defect in the diaphragm, sternal cleft, intracardiac defect (most commonly a ventricular septal defect), and
in approximately one third of the cases a diverticulum of the left ventricle. Pericardial cysts are not part of the pentalogy.

9. In infants with duodenal atresia all the following statements are true except:
A. There is an increased incidence of Down syndrome.
B. Duodenal atresia can be detected by prenatal ultrasound examination.
C. It may occur in infants with situs inversus, malrotation, annular pancreas, and anterior portal vein.
D. It is best treated by gastroenterostomy.
E. There is a high incidence of associated cardiac defects.
Answer: D

DISCUSSION: The diagnosis of duodenal atresia can be made prior to the infant's birth with a prenatal ultrasound
examination. Infants with duodenal atresia are often premature and have a high incidence of associated anomalies,
especially congenital heart disease. Duodenal atresia may also coexist in patients with annular pancreas, situs inversus,
malrotation, and anterior portal vein. Approximately one third of the cases occur in babies with Down syndrome. The
operative treatment of choice is a duodenoduodenostomy. Duodenojejunostomy is an alternative procedure.
Gastrojejunostomy is not recommended.

10. The initial treatment of choice for a 2.5-kg. infant with a 20.0-cm. long proximal jejunal atresia and 8.0 cm. of distal
ileum is:
A. Laparotomy, nasogastric suction, proximal dilatation to lengthen the atretic jejunum, total parenteral nutrition, and
delayed anastomosis.
B. Laparotomy and proximal end-jejunostomy.
C. Laparotomy and immediate small bowel transplantation.
D. Laparotomy and double-barrel enterostomy (jejunum and ileum), with refeeding of jejunal contents into distal ileum and
delayed anastomosis.
E. Laparotomy, tapering jejunoplasty, and end-to-oblique jejunoileal anastomosis.
Answer: E

DISCUSSION: The patient has short bowel syndrome with most of the bowel length involving the dilated proximal jejunal
atresia. The treatment of choice is to perform a tapering jejunoplasty to preserve bowel length and construct an anastomosis.
Early feedings are initiated when bowel function returns in order to stimulate bowel adaptation. Jejunal dilatation will not
significantly lengthen the atretic jejunum and will not alter its abnormal motility. End-jejunostomy decompresses the
obstruction but produces a high ostomy with excessive loss of succus entericus. A double-barrel enterostomy might allow
refeeding of jejunal content into the distal ileum and colon, but the proximal atretic loop may have poor function. Small
bowel transplantation is not a feasible alternative in the neonate at the present time.
11. A 2.8-kg. neonate with excessive salivation develops respiratory distress. Attempts to pass an orogastric catheter fail
because the catheter coils in the back of the throat. A chest film is obtained and shows right upper lobe atelectasis and a
gasless abdomen. The most likely diagnosis is:
A. Proximal esophageal atresia without a fistula.
B. Proximal esophageal atresia with a distal tracheoesophageal (TE) fistula.
C. “H-type” TE fistula.
D. Esophageal atresia with both proximal and distal TE fistula.
E. Congenital esophageal stricture.
Answer: A

DISCUSSION: Proximal esophageal atresia will result in excess salivation and aspiration of saliva. It is often associated
with right upper lobe collapse. Infants with a distal TE fistula will have air in the stomach and intestine, as will babies with
both proximal and distal fistulas, “H-type” fistula, and an esophageal stricture. Infants with proximal atresia but without a
TE fistula will not have air beneath the diaphragm. Attempts at passing an orogastric catheter will be met by an obstruction
and coiling of the catheter in the infant's mouth. The catheter will pass into the stomach in infants with “H-type” TE fistula
and in most with esophageal stricture.

12. Neonates with NEC may demonstrate all of the following findings on abdominal films except:
A. Pneumatosis intestinalis.
B. Portal vein air.
C. Pneumoperitoneum.
D. Colovesical fistula.
E. Fixed and thickened bowel loops.
Answer: D

DISCUSSION: Infants with NEC do not develop a colovesical fistula as an initial x-ray finding. Pneumatosis, portal vein
air, pneumoperitoneum, and fixed intestinal loops with thickened bowel wall are all observed with some regularity in babies
with NEC.

13. The most common type of congenital diaphragmatic hernia is caused by:
A. A defect in the central tendon.
B. Eventration of the diaphragm in the fetus.
C. A defect through the space of Larrey.
D. An abnormally wide esophageal hiatus.
E. A defect through the pleuroperitoneal fold.
Answer: E

DISCUSSION: Eventration of the diaphragm is related to phrenic nerve paralysis. It is more commonly observed after a
breech delivery and may be associated with torticollis and Erb's palsy. The space of Larrey is located anteriorly just off the
midline. A Morgagni hernia passes through this potential space. An abnormally wide esophageal hiatus would most likely
create a sliding hiatal hernia. The most common type of congenital diaphragmatic hernia in the neonate is the posterolateral
Bochdalek hernia, which passes through a defect in the developing pleuroperitoneal fold.

14. The calorie-nitrogen ratio for an infant should be maintained at:


A. 75:1.
B. 100:1.
C. 50:1.
D. 150:1.
E. 25:1.
Answer: D

DISCUSSION: The calorie-nitrogen ratio should be maintained at 150:1 for most infants. Fever, major illness, sepsis, or
trauma may increase the caloric requirements significantly.

15. All of the following conditions are derived from the primitive embryonic foregut except:
A. Bronchogenic cyst.
B. Cystic adenomatoid malformation.
C. Gastric duplication.
D. Mesenteric cyst.
E. Pulmonary sequestration.
Answer: D

DISCUSSION: Mesenteric cysts are derived from lymphatic anlage in the abdomen and are unassociated with foregut
development. The lung buds arise from the primitive foregut and anomalies associated with tracheopulmonary development
are therefore all derived from foregut. These include tracheoesophageal fistula, congenital lobar emphysema, enteric cysts
(which may communicate to the normal esophagus, lung, or spinal canal; e.g., neurenteric cyst), cystic adenomatoid
malformations, solitary lung cysts, intra- and extralobar sequestrations, and bronchogenic cysts. The stomach and first part
of the duodenum are also of foregut origin, and thus a gastric duplication is by definition derived from foregut.

16. For a 22-kg infant, the maintenance daily fluid requirement is approximately which of the following?

a. 1100 ml
b. 1250 ml
c. 1550 ml
d. 1700 ml
e. 1850 ml
Answer: c

Maintenance water and electrolyte requirements are illustrated in the table indicated below. The volume calculation is
demonstrated. The composition of the intravenous fluids is generally that of D5 1/4 normal saline or D5 1/2 normal saline
with 10 mEq/l of KCL.

17. Which of the following statements regarding nutritional requirements in infants are true?

a. The total daily water requirement is estimated to be 100 ml/100 kcal ingested
b. The resting energy expenditure is approximately twice that of an adult
c. The highest rate of nitrogen retention with parenteral nutrition occurs in infants given approximately 40% of the
calories as carbohydrate and the remainder as fat
d. The protein requirement for a newborn infant is approximately 2.5 g/kg/day
Answer: a, b, c, d

Taking all factors into account, total daily water requirements for a term infant are estimated to be 100 ml/100 kcal ingested,
assuming an insensible loss of 50 ml/kg/day and a growth requirement of approximately of 15 ml/kg/day. The energy
expenditure of normal neonates is approximately twice that of normal adults (50 kcal/kg/day versus 25 kcal/kg/day).
In most circumstances, high carbohydrate/low fat ratios in parenteral nutrition result in high rates of energy expenditure and
decreased nitrogen retention, while low carbohydrate/high fat ratios result in excessive fat deposition. A balanced ratio
(approximately 40% carbohydrate) provides the highest rate of nitrogen retention, and is consistent with the proportion of
carbohydrate found in breast milk and with the estimates of minimal carbohydrate needs determined by isotope infusion
studies. A consensus statement by the World Health Organization and the United Nations University estimates the protein
requirement at 2.5 g/kg/day for an infant and 1.25 g/kg/day for a one-year-old child. For preterm infants, the protein need
ranges from 2.5 to 3.9 g/kg/day if the weight is less than 2.5 kg.

18. A term infant 48 hours of age suddenly develops hypoxemia, irritability, and glucose and temperature instability.
Which of the following statements are true?

a. Empiric antibiotic coverage for b-hemolytic Streptococci and Escherichia coli should be initiated
b. An intravenous infusion of prostaglandin E1 should be initiated immediately
c. Exogenous surfactant should be given immediately
d. The mortality rate for this child is approximately 50%
Answer: a,d
This infant has the classical findings of neonatal sepsis. This is defined as a generalized bacterial infection accompanied by
a positive blood culture during the first month of life. Early onset sepsis occurs during the first week of life, and is due
primarily to maternal organisms, such as b-hemolytic Streptococci, Escherichia coli or Listeria monocytogenes. The
mortality rate of early onset sepsis is approximately 50 percent. Late onset sepsis is due primarily to hospital acquired
organisms such as Staphylococcus epidermidis, Staphylococcus aureus or Pseudomonas species, and the mortality rate for
this entity is approximately 20 percent.
The signs and symptoms of neonatal sepsis are subtle and nonspecific. Early signs include lethargy, irritability, temperature
instability, change in the respiratory pattern, or changes in the feeding pattern. Hematologic findings include
thrombocytopenia, leukocytosis, or leukopenia. Hemodynamic manifestations occur late. Presumptive therapy should be
based upon the suspected organism, but often includes Ampicillin or an anti-Staphylococcal agent plus an amino glycoside.
A prostaglandin E1 infusion is inappropriate as this relates to patients with ductal-dependent congenital heart disease.
Exogenous surfactant is unlikely to be helpful in a full-term infant who has previously been well and can be expected to
begin his illness with a normal complement of pulmonary surfactant.

19. Which of the following statements about pulmonary surfactant are true?

a. Endogenous surfactant deficiency is the key physiologic problem in preterm infants with the infant respiratory
distress syndrome
b. Surfactant function can be restored to normal using aerosolized phosphatidylcholine administration
c. Exogenous surfactant replacement has been shown to reduce mortality in preterm infants with the infant respiratory
distress syndrome
d. Surfactant is produced by Type I alveolar epithelial cells
Answer: a, c

The pulmonary surfactant complex lowers surface tension, stabilizing the alveolus even at low lung volumes. It is a
complex material secreted by Type II alveolar epithelial cells. It is composed of 80 to 90 percent phospholipid (primarily
phosphatidylcholine) and unique surfactant-associated proteins (10 percent). Surfactant proteins appear to play a critical role
in the organization of the phospholipid molecules, and modify the surface-active properties of the lipids. Phospholipid
synthesis and the expression of surfactant proteins increase with advancing gestational age. Amniotic fluid surfactant
concentrations have been used for many years to predict pulmonary maturity. Surfactant deficiency is the primary factor in
the pathophysiology of the neonatal respiratory distress syndrome. The use of exogenous surfactant replacement therapy is
currently under investigation for the treatment of neonatal respiratory distress and has been shown to reduce mortality in a
variety of specific circumstances involving preterm infants. Several commercially available preparations are available and
are undergoing clinical investigation.

20. Which of the following statements regarding premature infants are true?

a. Complications of prematurity account for approximately 85% of fetal deaths


b. Prematurity is defined by the World Health Organization as birth prior to 35 weeks gestation
c. Infants with intrauterine growth retardation have physiologic problems which are more dependent on the birth
weight than the gestational age
d. Preterm infants are at increased risk for hypocalcemia and hypoglycemia when compared to term infants
Answer: a, d

Prematurity is defined by the World Health Organization as a gestational age at birth of less than 37 weeks. Complications
of prematurity account for approximately 85% of fetal deaths. These deaths are commonly due to perinatal asphyxia,
respiratory failure and infection. The term intrauterine growth retardation describes a pathophysiologic process that results
in restriction of fetal growth. Fetal, placental or maternal abnormalities are common. These infants are a heterogeneous
population and they tend to have neonatal problems related more to their gestational age than to their birth weight. These
problems include asphyxia, hypoglycemia, hypothermia, hypocalcemia, pulmonary hemorrhage, necrotizing enterocolitis
and other complications related to specific syndromes or congenital anomalies.

21. Other than the history and physical exam, which of the following tests is considered an essential feature of the
preoperative evaluation of a patient with a suspected thyroglossal duct cyst?

a. Cervical ultrasound
b. Thyroid scan
c. Serum T3 and T4 levels
d. Needle aspiration
e. None of the above
Answer: e

A thyroglossal duct cyst is typically a midline structure connected to the foramen cecum at the base of the tongue, that is
pulled proximal and superior as the tongue protrudes. It may be superior or inferior to the hyoid and is occasionally slightly
off the midline. Because the thyroglossal cyst may rarely contain the patient’s only thyroid tissue, some have recommended
a technetium-99m radioisotope thyroid scan before excision. However, excision of the cyst is indicated regardless, because
infection of the cyst is likely, and the dysgenetic thyroid tissue in the cyst has malignant potential. For these reasons,
patients with suspected thyroglossal duct cysts require routine surgical excision. Preoperative ultrasound, thyroid scan, T3
and T4 levels, or needle aspiration is not necessary. For those patients who have thyroid tissue in their cysts by pathologic
examination, postoperative thyroid function tests identify those who have no remaining thyroid tissue and replacement
therapy can be prescribed.

22. Suppurative cervical lymphadenitis in a 3-year-old child is commonly related to which of the following organisms?

a. Staphylococcus aureus
b. Atypical mycobacterial organisms
c. Streptococcal organisms
d. Lymphoma with secondary pyogenic organisms
e. Cat scratch
Answer: a, c

Acute suppurative lymphadenitis related to bacterial pathogens is generally straightforward to diagnose. There is often
accompanying infectious illness. The lymph nodes enlarge rapidly, are tender and erythema of the overlying skin is present.
Fever and an elevated white blood count with a left shift are usually present. Fluctuant nodes may be aspirated.
Streptococcus and Staphylococcus aureus are the most common organisms and the initial course of antibiotic therapy is
directed to these organisms. If the adenopathy fails to resolve in 2 to 3 weeks, then the patient should likely undergo an
excisional biopsy. Atypical Mycobacteria, cat scratch disease, and Mycobacterium tuberculosis are more uncommon than
bacterial lymphadenitis and do not typically produce tenderness or systemic signs. Lymphoma with secondary pyogenic
infection is similarly uncommon.

23. Branchial cleft remnants most often present with which of the following clinical problems?

a. Infection
b. Airway obstruction
c. Hemorrhage
d. Malignant degeneration
e. Pain
Answer: a

The more common second branchial cleft anomalies present typically as a pinpoint opening on the anterior border of the
sternocleidomastoid muscle. Attention is drawn to the defect often by the appearance of small drops of clear fluid at the
orifice or by occurrence of infection in the tract itself. Less frequently, a mass may present anterior to the upper portion of
the sternocleidomastoid muscle representing a cyst derived from this tract. Infection is a more common problem in the older
age group. Airway obstruction and hemorrhage are rare presentations. Pain is usually secondary to infection and malignant
degeneration is reported but exceedingly rare. Treatment consists of surgical excision. If infection is present, a course of
antibiotics is administered first.

24. Proximity to which of the following structures places it at risk during surgical excision of a second branchial cleft
remnant?

a. Internal carotid artery


b. External carotid artery
c. Hypoglossal nerve
d. All of the above
e. None of the above
Answer: d

Operative excision of a second branchial cleft remnant begins with an elliptical transverse incision at the sinus opening and
cephalad dissection of the tract to its furthest extent, generally reaching the floor of the tonsillar pillar. The dissection is kept
directly on the tract to avoid injury to contiguous structures such as the internal jugular vein, the internal or external carotid
arteries (between which it passes), and the hypoglossal nerve. The operation can almost always be carried out through a
single cosmetic incision if the tract is kept under traction and digital pressure is exerted through the tonsillar fossa.
Dissection of the sinus tract may be facilitated by passing a fine silver probe or piece of heavy nylon suture through the
length of the tract.

25. Standard therapy for acute epiglottitis in a child is:

a. Tracheostomy
b. Intravenous antibiotic treatment in an ICU setting
c. Endotracheal intubation in the operating room and intravenous antibiotic therapy
d. Indirect laryngoscopy and intravenous antibiotics
e. Intravenous steroids and antibiotics
Answer: c

Acute epiglottitis is a common cause of acquired airway obstruction in the pediatric age group. Haemophilus influenzae B
is nearly always the causative organism, and most children are toxic at presentation with an elevated temperature and an
increased pulse and respiratory rate. Prolonged inspiratory stridor that worsens in the supine position is characteristic. The
child usually sits erect, anxious and drooling and becomes increasingly exhausted with air hunger. No attempt should be
made to visualize the larynx outside of the operating room for fear of sudden airway occlusion. The standard therapy is
short-term endotracheal intubation performed in the operating room with general anesthesia. The inflammatory process
resolves rapidly with intravenous antibiotics and intubation is seldom required beyond 3 days. In the past, tracheostomy was
the standard therapy, but comparative reviews demonstrate that short-term endotracheal intubation is associated with less
morbidity and fewer complications.

26. Which of the following statements regarding congenital diaphragmatic hernia are true?

a. The incidence of right and left-sided lesions is equal


b. Malrotation is to be expected
c. Left-to-right shunting via a patent ductus arteriosus is a serious but expected physiologic consequence of
pulmonary hypoplasia
d. Survival rates of 75% are reported in several contemporary series
e. Congenital heart disease is present in approximately 20% of these infants
Answer: b, d, e

During organogenesis closure of the right hemidiaphragm normally precedes that of the left. This asynchronous closure and
the presence of the liver on the right account for the finding that 85% to 90% of congenital diaphragmatic hernias involve
the left hemidiaphragm. Malrotation is an expected finding with diaphragmatic hernia because intestinal herniation into the
thorax normally precedes the fixation of the gut to the posterior body wall. Pulmonary hypertension is a major feature of
congenital diaphragmatic hernia and right-to-left shunting via a patent ductus arteriosus is present in virtually all of these
children. Fifteen to 25% of infants with a diaphragmatic hernia have an associated anomaly, the most important being
cardiovascular abnormalities. Although ventricular septal defects and aortic coarctations are most common, virtually all
cardiac and great vessel anomalies have been reported. Cardiac ECHO screening examinations are therefore routine.
Survival rates as high as 75% to 90% in selected high-risk congenital diaphragmatic hernia patients have been reported in
several large clinical series over the last five to ten years. This is an apparent improvement from the historic range of 50%.

27. Of the following cystic malformations of the tracheobronchial tree, which is most likely to be asymptomatic when
discovered?
a. Intralobar pulmonary sequestration
b. Extralobar pulmonary sequestration
c. Congenital cystic adenomatoid malformation
d. Congenital lobar emphysema
Answer: b

Intralobar pulmonary sequestration and cystic adenomatoid malformations typically present with either neonatal respiratory
distress or infection related to inadequate clearance of secretions. Given enough time, nearly all of these lesions will become
infected. Congenital lobar emphysema is characterized by air-trapping within an otherwise normal lung. This typically
presents with respiratory distress which ranges from mild to life-threatening. Hemodynamic instability requiring emergency
thoracotomy is occasionally present.
Extralobar sequestration is typically a mass of disorganized pulmonary parenchymal tissue within its own investing pleura
and outside of the normal lung parenchyma. This does not communicate with the normal tracheobronchial tree. Infection is
rare and although hemorrhage, arterial venous shunting, mediastinal compression and occasional malignancy may occur,
these lesions are typically asymptomatic and indeed often discovered via prenatal ultrasound. Excision is recommended for
each of these lesions, typically involving a lobectomy.

28. Infants with a double aortic arch most commonly present with which of the following problems?

a. Dysphagia
b. High output cardiac failure related to a patent ductus arteriosus
c. Positional hyperemia and edema of the right upper extremity
d. Symptomatic tracheal compression
Answer: d

The double aortic arch represents the most common type of complete vascular ring. It arises from the ascending aorta and
bifurcates, with one arch passing on either side of the trachea and the esophagus. The symptoms of complete vascular rings
are due to compression of the trachea, the esophagus or both. The child with a double aortic arch is generally the most
symptomatic and most patients have symptoms in infancy. The typical clinical picture is one of symptomatic tracheal
compression and includes inspiratory wheezing, coughing, noisy breathing, shortness of breath, stridor and frequent bouts of
pneumonia. Feeding problems may become apparent when solid foods are started but this is less common than tracheal
compression. The most important screening test is the barium swallow, and CT or MRI are definitive. Angiography and
endoscopy are not usually helpful. Any patient who is symptomatic from a vascular ring should be treated surgically.

29. Which of the following is most common after primary esophagostomy for esophageal atresia with a distal
tracheoesophageal fistula?

a. Anastomotic leak
b. Esophageal stricture
c. Recurrent tracheoesophageal fistula
d. Gastroesophageal reflux
e. Tracheomalacia requiring aortopexy
Answer: d

The preferred approach for esophageal atresia with a distal tracheoesophageal fistula in a patient without other problems is
an extrapleural right thoracotomy with division of the tracheoesophageal fistula and primary esophagostomy. No
gastrostomy is ordinarily used. The results with this approach are better than those with a staged approach. Three major
complications are related to the esophageal anastomosis: leak, stricture and recurrent fistula. The incidence of leak varies
from 10% to 20% depending on the type of anastomosis done and the degree of tension. A distinct advantage of an
extrapleural anastomosis is the predictable resolution of these leaks if adequately drained. Similarly, the stricture rate varies
between 10% and 25%, again depending on the type of anastomosis done. Many infants require one or two dilations, but
few have significant long-term problems. The incidence of recurrent esophageal fistula is difficult to determine since few
authors emphasize this technical problem, but it appears to be about 10% in most reports.
Gastroesophageal reflux secondary to dysmotility of the distal esophagus is a significant problem and occurs to some degree
in virtually all of these patients. A significant number of these infants, 25% to 30% or more, are refractory to medical
therapy and require surgical fundoplication. Tracheomalacia is a complication of the malformation, not of the repair. This
appears to result from inadequate cartilagenous tracheal rings at the level of the fistula. The reported incidence is up to 25%
in some recent series. Most infants with tracheomalacia improve with growth and time, but a small percentage develop
severe respiratory difficulty which requires surgical aortopexy.

30. Which of the following is the most common primary lung tumor in infants and children?

a. Pulmonary blastoma
b. Squamous cell carcinoma
c. Endobronchial carcinoid
d. Leiomyoma
e. Metastatic osteogenic sarcoma
Answer: c

Bronchial adenomas are the most common primary lung tumors in childhood. Nevertheless, they are quite rare. Typically
these are low-grade adenocarcinomas that include endobronchial carcinoids, cylindromas, mucoepidermoid tumors, and
bronchomucous gland adenomas. Carcinoid tumors are the most common bronchial adenomas and represent over 80% of
the total.
Although common in adults, bronchogenic carcinoma of the lung is extremely rare in childhood. A review of the world’s
literature in 1983 revealed only 47 such patients. Pulmonary blastoma is a rare malignant lung tumor composed of cells that
resemble fetal lung. Its incidence is actually highest in adults, although it is reported in children as well. Benign tumors of
the lung are also rare in childhood. The most common of these are pulmonary hamartomas or chondromas. Others include
leiomyoma, leiomyoblastoma and mucus gland adenoma. Metastatic osteogenic sarcoma is more common than any primary
lung tumor, but is by definition a secondary metastatic lesion and therefore not the correct answer to the question posed
here.

31. Which of the following statements regarding congenital chest wall deformities are true?

a. Children with pectus excavatum deformities typically have physiologically insignificant limitation of exercise
tolerance
b. The rate of recurrence after operative repair of a pectus excavatum deformity is between 5% and 10%
c. Pectus carinatum is the most common congenital chest wall defect
d. The most common indication for operative repair of congenital chest wall deformities is cosmesis
Answer: a, d

The most common congenital chest wall deformity is pectus excavatum, representing approximately 90% of the total.
Approximately 5% to 7% of the lesions are pectus carinatum and a variety comprise the remainder. Most children with
pectus excavatum are asymptomatic. There have been numerous efforts to document associated cardiac and pulmonary
abnormalities. Objective data show that although there are minor demonstrable cardiopulmonary abnormalities
demonstrable, these do not appear to be significantly improved by surgery and they are generally insignificant
physiologically. As a result, the indications for repair of chest wall deformities are essentially cosmetic and psychological.
The importance of these however should not be minimized in a largely adolescent population. The repair is technically
straightforward but involves moderate morbidity. The long-term results are excellent. Recurrence is rare (approximately
1%) in most institutions with large experiences.

32. The definitive evaluation of a child with a suspected congenital cystic abnormality of the tracheobronchial tree is best
done using which of the following?

a. Rigid bronchoscopy
b. Computerized tomography or magnetic resonance imaging
c. Chest x-ray
d. Angiography
e. Barium esophagogram
Answer: b

Plain film radiography is the first imaging study performed and remains a cornerstone for the diagnosis and follow-up of
this group of lesions. The use of additional imaging provides definitive diagnosis and allows planning for the surgical
approach as well. Computed tomography (CT) and magnetic resonance imaging (MRI) can separate cystic from solid
components in a radiopaque lung mass. These are the most definitive diagnostic studies available. The MRI has
reconstructive capabilities that obviate the need for angiography. Intravenous contrast with CT scan provides similar
anatomic information. Ultrasonography is less costly, more readily performed and in select cases may be as sensitive.
Angiography is not employed regularly because these alternative imaging strategies provide similar information at lower
cost with less morbidity. A barium esophagogram is helpful in the diagnosis of children with dysphagia but that is a rare
presentation for these lesions. Bronchoscopy is rarely helpful for these lesions and in these infants and small children carries
the risk of general anesthesia and positive pressure ventilation. In children with congenital lobar emphysema and cystic
adenomatoid malformation, hyperinflation following positive pressure may induce mediastinal compression and create a
surgical emergency. For these reasons, CT or MRI are considered the best definitive diagnostic imaging choices after the
initial chest x-ray.

33. A newborn infant develops coughing, choking and cyanosis with his first feeding. He is noted to have excessive
drooling. What are the important associated anomalies that must be screened for prior to surgical intervention?

a. Right-sided aortic arch


b. Hydrocephalus
c. Genitourinary obstruction
d. Congenital heart disease
Answer: c

This child has a classical history for esophageal atresia and has a very high (90% or more) probability of a distal
tracheoesophageal fistula. The simplest way to establish the diagnosis is to attempt to pass a catheter through the mouth or
nose into the stomach. If the tube encounters obstruction, a plain radiograph should document the atresia.
Patients with esophageal atresia and tracheoesophageal fistula frequently have associated anomalies. This incidence is about
30% to 50% in most reports. Anomalies vary from minor skeletal deformities to uncorrectable cardiac defects. The most
common associated anomalies are cardiac and gastrointestinal, especially imperforate anus (10%). Vertebral, genitourinary
and limb anomalies are also seen. Importantly, approximately 5% of patients with esophageal atresia have a right-sided
aortic arch. This is an important technical issue as the normal approach is via a right thoracotomy and this should be
changed to a left thoracotomy in the presence of this finding. There is no association with hydrocephalus.

34. Which of the following ventilation strategies is the best initial approach for a neonate with a left congenital
diaphragmatic hernia and the following post ductal arterial blood gases: PaO2 50 mm Hg, PaCO2 60 mm Hg, pH 7.35?

a. High-frequency jet ventilation


b. Permissive hypercapnia with convential pressure controlled ventilation
c. Extracorporeal membrane oxygenation (ECMO)
d. Induced respiratory alkalosis
e. Inhaled nitric oxide with conventional volume controlled ventilation
Answer: b

Contemporary congenital diaphragmatic hernia management emphasizes permissive hypercapnia using any necessary mode
of respiratory support. Ordinarily, pressure controlled ventilation is the initial mode of support. The purpose is to reduce the
iatrogenic lung injury associated with high-pressure mechanical ventilation. It is this latter problem, barotrauma-induced
lung injury, which has reduced enthusiasm for induced respiratory alkalosis. Although alkalosis can sometimes be achieved,
the price is often prohibitive mean and peak airway pressures. High-frequency jet ventilation, ECMO and inhaled nitric
oxide are all evolving strategies which must be considered developmental and are reserved for use after the initial strategy is
unsuccessful.

35. There is an emerging consensus that the surgical repair for congenital diaphragmatic hernia is best done:

a. Emergently at the bedside, eliminating the risks of transporting an unstable neonate


b. While on extracorporeal membrane oxygenation
c. When the infant is potentially extubatable
d. Within the first 48 to 72 hours of life
Answer: c
Infants with diaphragmatic hernias have considerable variation in the degree of respiratory distress, and the degree of
distress dictates timing of repair. Traditionally, infants were rushed emergently to the operating room for reduction of the
herniated viscera and diaphragmatic closure. Because effective preoperative decompression of the intestine is usually
possible and because it has become clear that the underlying pulmonary hypoplasia is not reversed by emergency operation,
this sense of urgency is no longer accepted. Most surgeons now commit themselves to a period of preoperative stabilization
which is used to confirm the diagnosis and optimize medical care. The current recommendation is that operative repair be
undertaken in a stable patient who is nearing extubatable levels of ventilatory support, regardless of the means of support
which have been employed. This means that repair is done at or after the end of cardiopulmonary bypass in infants where
that is necessary. The results with a delayed approach appear at least equivalent and in several series better than with
emergent or early repair. The concept is that the deferral of the iatrogenic operative injury to a time when the neonatal
pulmonary vascular bed is less vulnerable to vasospasm is desirable.

36. Meckel’s diverticulum may present with which of the following signs or symptoms?

a. Hemorrhage
b. Intussusception
c. Volvulus
d. Patent omphalomesenteric duct
e. Right lower quadrant peritoneal findings
Answer: a, b, c, d, e

The most frequent congenital anomaly of the GI tract is a Meckel’s diverticulum. The incidence is about 2% of the general
population, many of whom remain asymptomatic throughout life. Estimations of the frequency with which symptoms
develop among people with Meckel’s diverticula vary from 4% to 30%, but is clear that the risk diminishes substantially
with age. About half of those who become symptomatic are under the age of 2. Hemorrhage, acute diverticulitis,
perforation, and small bowel obstruction or intussusception are all classic presenting scenarios for a child with a Meckel
diverticulum. (Table) The approximate frequency of these presentations is shown above. Below are illustrations of the
various anatomic abnormalities with their associated clinical presentations. (Figure) These presentations include Meckel’s
diverticulitis which is virtually indistinguishable from acute appendicitis.
SIGNS AND SYMPTOMS OF MECKEL’S DIVERTICULUM
Clinical Presentation Approximate Frequency (%)
Hemorrhage 30–35
Small bowel obstruction 30–35
Diverticulitis 20–25
Umbilical fistula 10
Other Uncommon

Normal embryologic relations of the embryonic yolk sac, yolk stalk, and gut.

37. A 3-week old infant has a barium upper gastrointestinal series to evaluate vomiting. The duodenojejunal flexure is
found to be to the right of the midline as well as more caudal and anterior than a normal ligament of Treitz. The child is
seen to reflux barium spontaneously to the level of the mid-thoracic esophagus. You would recommend which of the
following?

a. Barium enema
b. Emergency laparotomy
c. A trial of H2, blockade and cisapride therapy
d. Upper gastrointestinal endoscopy
e. Overnight pH probe analysis
Answer: b

This child has malrotation and is at risk for midgut volvulus based on the imaging findings. Typically, malrotation produces
an incomplete obstruction of the duodenum with a corkscrew or coiled appearance in the third or fourth portions of the
duodenum. Malposition of the duodenojejunal junction is diagnostic. In particular, this includes a location to the right of the
midline. Additionally, failure to achieve normal posterior and cephalad fixation is typical. The small bowel resides in the
right abdomen and the colon and cecum are on the left. Attempts to radiographically differentiate malrotation with or
without volvulus are unreliable and therefore hazardous. This child has gastroesophageal reflux which is likely secondary to
the partial duodenal obstruction from malrotation. None of the alternatives other than emergency laparotomy are
appropriate. There is no role for nonoperative management of malrotation in the neonate. Assessment, resuscitation and
preoperative preparation should be conducted concurrently as the child is prepared for laparotomy. This urgency is because
a delay measured in hours may represent the difference between a viable or infarcted midgut at laparotomy. Fifty to 75% of
malrotations are discovered in the first month of life and about 90% occur in children less than one year of age.

38. Which of the following statements regarding duodenal atresia are true?

a. 20% to 40% of these infants have Trisomy 21


b. When associated with an annular pancreas, division of the pancreas at the site of obstruction is curative
c. Bilious vomiting is typical because the obstruction is usually distal to the ampulla of Vater
d. Reconstruction is best achieved with Roux-en-Y duodenojejunostomy
Answer: a, c

Twenty to 40% of infants with congenital duodenal obstruction have Trisomy 21. Because this syndrome is not always
apparent during the physical examination, a routine karyotype should be obtained. A preoperative ECHO ultrasound
examination is also appropriate to evaluate the possibility of associated congenital heart disease. Feeding intolerance and
bilious vomiting in the first 24 to 48 hours of life are characteristic. The malformations are typically distal to the ampulla of
Vater and thus the infants present with bilious vomiting and proximal duodenal and gastric distention. Malformations
proximal to the ampulla of Vater result in nonbilious vomiting and this possibility must not be ignored.
Generally, bypass of the obstructing lesion is the best approach regardless of whether an atresia or annular pancreas is
responsible. Division of an annular pancreatic band is inappropriate for two reasons: 1) The duodenum is virtually always
atretic in addition. 2) Division of this pancreatic tissue necessarily divides the accessory pancreatic duct within it creating
the real possibility of a pancreatic fistula. Generally, the construction of a duodenostomy that minimizes the length of
defunctionalized duodenum is preferred. The procedure generally employed is a diamond-shaped duodenostomy. Simple
duodenojejunostomy without a Roux-en-Y is occasionally necessary for lesions in the distal duodenum.

Diamond-shaped duodenoduodenostomy.

39. A 1500-gram, 30-week gestation neonate is fed at 2 weeks of age. He develops abdominal distention, bilious
vomiting and guaiac-positive stool. A plain film of the abdomen demonstrates pneumotosis intestinalis. Which of the
following related statements are true?

a. An emergency barium upper GI series should be done to rule out malrotation


b. The child should have a nasogastric tube placed, broad spectrum intravenous antibiotics begun, and sequential
abdominal films obtained.
c. The likelihood of intestinal perforation is in excess of 50%
d. The expected survival rate is in the range of 70%
Answer: b, d

This clinical history is classical for neonatal necrotizing enterocolitis (NEC), an idiopathic clinical condition characterized
by mucosal intestinal injury that may progress to transmural bowel necrosis. Typically it occurs in critically ill, preterm
infants and is characterized by abdominal distention, bilious vomiting and either occult or gross blood in the stool. In this
setting, pneumotosis intestinalis is pathognomonic. When the diagnosis is suspected on clinical grounds and the plain film,
no contrast imaging should be done as this may complicate or contribute to the problems of perforation. In particular, the
child presented here has an unequivocal history and an upper GI series would be inappropriate.
Half of all infants with NEC have birth weights less than 1500 g, and 80% of these infants weigh less than 2500 g at birth.
The incidence is approximately 1 to 2 in 1000 live births. It is the most common surgical emergency in neonates in North
America. The initial management of infants with NEC consists of nasogastric decompression; broad-spectrum antibiotic
administration; and correction of hypoxemia, hypotension, acidosis, fluid or electrolyte disorders, and other reversible
medical problems. Up to 90% of infants with NEC can be managed successfully nonoperatively, but this is widely variable
among institutions because of differences in referral and practice patterns. Intestinal perforation is characterized by
pneumoperitoneum and is an indication for operation. Although the incidence of perforation is variable, it is generally less
than 20% to 40%.
The overall survival rate for neonates with NEC is about 60% to 70% for both operative and nonoperative management
groups. This represents a substantial improvement from the 20% to 30% survival probability when the entity was first
recognized 30 to 40 years ago.

40. The most common cause of pyogenic liver abscess in children today is which of the following?

a. Perforated appendicitis
b. Blunt liver injury
c. Immunocompromised host
d. Percutaneous liver biopsy
e. Omphalitis
Answer: c

In the preantibiotic era, pyogenic hepatic abscesses occurred most frequently after perforated appendicitis. This
complication is rarely seen today. Chronic granulomatous disease of childhood is a principle condition associated with
hepatic abscess. This disease is the result of deficient oxidant-mediated bacterial killing by circulating granulocytes. In the
pediatric age group, 40% of pyogenic liver abscesses occur in children with chronic granulomatous disease, and another
30% occur in children with other immunodeficiencies, most commonly leukemia. Other rare causes of liver abscesses in
pediatric patients are umbilical vein catheter-induced infection, omphalitis and other biliary disease. Pyogenic liver
abscesses following blunt liver injury or percutaneous liver biopsy are distinctly rare events.

41. Which of the followings statements regarding an infant with meconium ileus are true?

a. The probability is 100% that he will have cystic fibrosis


b. Nonoperative therapy resolves this problem in approximately two-thirds of patients
c. The average life expectancy is approximately 26 to 28 years for this infant at present
d. The finding illustrated below on plain film is an absolute operative indication (Figure 103-23A)
Answer: a, b, c, d

Meconium ileus refers to the characteristic obstruction of the small intestine in neonates with cystic fibrosis (CF). Some
10% to 20% of infants with CF present initially with meconium ileus. All infants with meconium ileus have cystic fibrosis.
CF is characterized by a transport defect of epithelium that results in impermeability of the chloride ion and therefore water.
Inspissated secretions in the pancreas and gut lead to obturator obstruction of the terminal ileum from meconium in the
neonate. Approximately two-thirds of these infants have simple meconium ileus, the remainder have complications such as
proximal volvulus, perforation or atresia. These latter problems may be associated with the development of a meconium
cyst. In this instance, speckled calcifications on plain radiograph or ultrasound are diagnostic. The film above is that of a
meconium pseudocyst consistent with intraperitoneal spillage of meconium from intestinal perforation. This finding requires
surgical exploration.
Sixty to 70% of infants with simple meconium ileus can be treated by enema installation of one several irrigation solutions
into the obstructed terminal ileum. Saline, hyperosmolar contrast agents, dilute N-acetylcysteine and a variety of other
solutions have been used successfully. Following resolution of the obstruction, most institutions now report survival rates as
high as 70% to 100%.
The average life expectancy for CF patients is now well into the third decade of life. It is primarily determined by the course
of the pulmonary disease rather than GI problems. A number of important medical advances, including the realistic prospect
of gene therapy are foreseeable for these infants.

42. You are asked to recommend therapy for an asymptomatic 2 year old who swallowed a small alkaline watch
battery 4 hours ago. A plain film shows the intact battery in the intestine beyond the stomach. The best course of therapy
is?

a. Immediate laparotomy, enterotomy and removal of the battery


b. Enteroscopy with extraction
c. Laparoscopy with ultrasound localization and extraction
d. Cathartics and a follow-up plain film in 48 hours if the child remains asymptomatic
Answer: d

Passage of an ingested foreign body beyond the gastroesophageal junction is associated with a 95% probability of
uneventful distal transit. The character of the foreign body is largely irrelevant but batteries, particularly alkaline disk
batteries, present a potentially serious hazard to children and may require an aggressive approach. A number of reports have
documented the unique risk of intestinal perforation resulting from disruption of the battery casing and associated spillage
of the toxic contents. Although some have advocated routine immediate removal, this does not appear necessary. If the
battery is endoscopically accessible in the esophagus, it should be removed when recognized. Cathartics and enemas may
help expedite passage of batteries discovered when already in the small or large bowel. Delay for more than a few days,
casing rupture on plain radiograph, or symptoms of any kind require extraction. Despite the risks, most batteries pass
without these sequelae.

43. A jaundiced 6 week old infant has biliary atresia. Which of the following statements are true?

a. Portoenterostomy is the initial procedure of choice


b. Primary liver transplantation using either a reduced sized cadaveric graft or a living related graft is now the
procedure of choice
c. Approximately two-thirds of patients managed with portoenterostomy will develop chronic liver disease sufficient
to indicate liver transplantation
d. Because biliary atresia has pathogenic components of acute and chronic inflammation, antiinflammatory therapy is
known to delay onset of liver failure
Answer: a, c

Biliary atresia is an idiopathic process in which the extrahepatic biliary ducts are replaced in whole or in part with dense
fibrous tissue containing evidence of both acute and chronic inflammation. There is an intrahepatic component as well.
Although antiinflammatory therapy is of some theoretical interest, there are no data to suggest that antiinflammatory
pharmocologic therapy will influence the natural history of liver disease associated with biliary atresia.
The approach for the usual infant in whom biliary atresia is discovered within the first 90 days of life is to confirm the
suspected diagnosis by operative cholangiogram at laparotomy and then proceed with portoenterostomy. In general terms,
one-third of these infants do well on a long-term basis, one-third have prompt failure, and the remainder have chronic liver
disease that becomes problematic more slowly. Therefore, approximately two-thirds of these patients develop chronic liver
disease for which liver transplantation is a reasonable alternative. Hepatic transplantation is best considered a necessary and
complementary approach to portoenterostomy for infants with biliary atresia. Data support its use in infants with failed
portoenterostomy or in older infants with established cirrhosis at the time of presentation. Growth failure, hepatic synthetic
failure and sequelae of portal hypertension are indications to proceed with transplantation.

44. Of the following, which is the most likely cause of hemodynamically significant lower gastrointestinal bleeding in
a 6 month old male child?

a. Meckel diverticulum
b. Henoch-Schonlein purpura
c. Intussusception
d. Crohn’s colitis
e. Hemolytic uremic syndrome
Answer: a, c

All of the choices are possible causes of lower GI bleeding in a 6 month old; intussusception and a bleeding Meckel
diverticulum are the most common. Upper GI hemorrhage with passage of blood distal should also be considered, but is not
discussed further here as sampling nasogastric aspirate is a relatively easy and reliable means of differentiating these
problems. The magnitude of blood loss associated with intussusception is usually minor, but the associated vomiting and
bowel obstruction may lead to significant volume depletion with hemoconcentration. The magnitude of the hemorrhage is
usually more significant with a Meckel diverticulum.
Infectious diarrheas also occur in this age group. Typically, signs and symptoms include fever and ileus with bloody
diarrhea. The diagnosis is confirmed with stool examination for leukocytes and cultures for specific pathogens.

45. Which of the followings statement regarding Hirschprung’s disease are true?

a. Suction rectal biopsy is virtually always diagnostic if the specimen includes submucosa
b. Hirschprung’s disease is the result of a sex linked dominant gene
c. The endorectal pullthrough is demonstrably superior to other forms of surgical construction
d. Ninety percent or more of patients have an excellent or good functional result following reconstructive surgery
e. The important cause of mortality in contemporary practice is enterocolitis
Answer: a, d, e

The incidence of Hirschprung’s disease is about 1 per 5000 live births, with no racial predilection, but with a marked male-
to-female (4:1) preponderance. Most cases are sporadic, but long-segment or total colonic aganglionosis and female gender
are strongly associated with familial disease. Recent data suggest an association with the RET protooncogene. The genetic
basis of Hirschprung’s disease is under active investigation and it appears that several genes including those located on
chromosomes 10, 13, 22 and possibly others are involved. It is neither sex linked nor dominant. There is a rare association
with the MEN syndromes, particularly medullary carcinoma of the thyroid.
The accuracy of suction rectal biopsy is 100% with a correctly done biopsy that includes submucosa and experienced
pediatric pathology in several large series. This requires both a search for ganglion cells and evaluation of the axons of the
myenteric neurons using either conventional staining techniques or histochemical staining for acetylcholinesterase.
Definitive operations for congenital aganglionosis all depend on resection or bypass of the distal aganglionic rectum with a
low rectal anastomosis to normally innervated pulled-through proximal intestine. Selection among the several described
operations depends more on a surgeon’s individual training and preference rather than upon demonstrable differences in
outcome. Although the endorectal pull-through is one of the widely practiced and popular procedures, it is not demonstrably
superior to the procedures described by Duhamel or Swenson (see text). Eighty to 90% or more of patients have excellent or
normal bowel function following reconstructive surgery for Hirschprung’s disease when evaluated after 5 years, regardless
of the procedure employed.
The primary remaining cause of mortality directly attributable to Hirschprung’s disease itself is enterocolitis. When it
occurs, this is typically found in infants or neonates for whom the diagnosis has been delayed. Postoperative enterocolitis
does occur, but it tends to be substantially less virulent. Undiagnosed neonatal Hirschprung’s enterocolitis can lead to death
in 12 to 24 hours from overwhelming sepsis.

46. The operative procedure of choice for managing the most common type of choledochal cyst is which of the
following?

a. Cyst gastrostomy
b. Cyst jejunostomy
c. Excision with Roux-en-Y hepaticojejunostomy
d. Transduodenal marsupialization
e. Endoscopic sphincterotomy
Answer: c

The most common choledochal cyst is a type I cyst; 80% to 90% of the total in most reports. These are characterized by
fusiform dilation of the choledochus itself. These cysts typically involve the entire common bile duct with only mild dilation
of the common hepatic duct and a normal intrahepatic system. The treatment of this lesion is always surgical. Internal
drainage procedures without cyst resection (e.g., cyst duodenostomy, cyst gastrostomy and cyst jejunostomy) were routinely
performed for type I choledochal cysts until the 1970s. The rate of failure (e.g., stricture, recurrent cholangitis, stone
formation, pancreatitis) ranged from 30% to 75%, depending on the length of follow-up and the type of procedure. As these
late complication rates became apparent, the risk of bile duct adenocarcinoma in the residual cyst also became widely
recognized. Therefore, the preferred operative treatment of a type I choledochal cyst is total transmural excision with Roux-
en-Y hepaticojejunostomy. Occasionally, adults with severe inflammation and fibrosis may require intramural cyst
dissection, leaving the posteriomedial (outer) wall of the cyst in situ to protect the adjacent portal vein and hepatic artery.

47. Which of the following is the most common liver tumor of childhood?

a. Hemangioma and hemangioendothelioma


b. Hepatoblastoma
c. Hepatocellular carcinoma
d. Mesenchymal hamaratoma
Answer: b

Primary liver tumors are uncommon in children. Of these, about one-third are benign and two-thirds are malignant. The
most common presenting feature for a liver tumor is an asymptomatic abdominal mass. The diagnostic evaluation is
generally an ultrasound initially followed by either computed tomography or magnetic resonance imaging, for definitive
diagnosis. The relative incidence of liver tumors is illustrated in the table below.
Hepatoblastoma is the most common liver tumor of childhood. Most hepatoblastomas are discovered within the first two
years of life. Although these are large and may require primary chemotherapy, a 65%–75% survival is achievable with
resection that yields histologically clear resection margins.

48. The risk of biliary tract adenocarcinoma developing in a patient with a choledochal cyst left in situ is
approximately which of the following?

a. Less than 1%
b. 3% to 5%
c. 10% to 15%
d. Greater than 25%
Answer: b

Adenocarcinoma of the biliary tract develops in 3% to 5% of patients who have choledochal cysts. Although this represent a
small number of patients, the total number reported exceeds 50 and the incidence is about 1000 times that of the normal
population. In addition, the carcinoma may develop as early as the adolescent years, a marked contrast to the normal
population in which a fifth or sixth decade of life presentation is typical. Neoplastic transformation of the dysplastic biliary
cyst epithelium may result from chronic inflammation. Consideration of this potential problem has contributed significantly
to the current emphasis on surgical excision of these cysts.

49. The most common cause of acute pancreatitis in childhood is which of the following?

a. Pancreas divisum
b. Cholelithiasis
c. Trauma
d. Valproic acid
e. Annular pancreas
Answer: c

Although adult pancreatitis is usually related to cholelithiasis or alcohol ingestion, pediatric causes are considerably more
varied in etiology. Fifty to 70% of childhood acute pancreatitis is either idiopathic or posttraumatic in origin. Trauma is the
single most common cause of acute pancreatitis in childhood. Cholelithiasis is an important etiology in the adolescent
population and in children with hemotologic disorders. Annular pancreas normally is associated with duodenal atresia and
produces neonatal duodenal obstruction, but not acute pancreatitis. Pancreas divisum is an anatomic variation present in
10% to 15% of normal children that is occasionally the cause of acute pancreatitis. Valproic acid is an important anti-
convulsant used in pediatric neurology. One of its possible complications is the development of acute pancreatitis or
necrotizing pancreatitis. Fortunately, this is rare as it is often lethal.

50. Which of the following statements regarding gastroschisis are true?

a. Primary fascial closure can be achieved in only about 25% of these infants
b. These infants have an incidence of approximately 40% to 50% of associated anomalies
c. Overall survival is approximately 80% to 90%
d. When the diagnosis is known prenatally, planned cesarean section is the safest method of delivery
Answer: c

Gastroschisis is a full-thickness defect of the abdominal wall with herniation of a variable amount of uncovered intestine.
Prenatal diagnosis has enabled the diagnosis of gastroschisis to be made prior to delivery. Prospective evaluation comparing
vaginal delivery with elective cesarean section has demonstrated no difference in outcome. Therefore, careful vaginal
delivery generally remains the birthing method of choice. Unlike omphalocele which is commonly associated (50%) with
other anomalies, other structural anomalies are associated with gastroschisis in approximately 10% of patients.
Primary fascial closure after reduction of the herniated viscera is the best surgical option and this is possible in 60% to 70%
of infants. Care must be taken not to generate excessive intraabdominal pressure when performing a primary abdominal
wall closure. Generally, an intraabdominal pressure less than 20 cm H2O is well tolerated. If the herniated viscera cannot be
reduced primarily, a silastic pouch constructed to temporarily contain the extra abdominal bowel and a series of partial
reductions are begun. This approach combined with adequate nutritional support by total parenteral nutrition yields survival
rates of at least 80% to 90% in most contemporary series of gastroschisis.

51. Which of the following are typical causes of neonatal intestinal obstruction?

a. Intussusception
b. Meconium ileus
c. Hirschprung’s disease
d. Meckel’s diverticulum
e. Incarcerated hernia
Answer: b, c, e

Neonatal bowel obstruction is defined as intestinal obstruction developing in the first 30 days of life. The cardinal
manifestation is bilious vomiting often in conjunction with abdominal distention. Meconium ileus and Hirschprung’s
disease are classical causes of neonatal intestinal obstruction outlined in the table. Incarcerated inguinal hernias are the most
common cause of neonatal intestinal obstruction. Hernias usually do not present a diagnostic dilemma, as simple inspection
of the groin yields an obvious diagnosis. Intussusception, while a cause of distal bowel obstruction in infants, does not
usually become a consideration until at least 3 to 6 months of age. Intestinal obstruction related to Meckel’s diverticulum is
generally related to either intussusception or volvulus associated with a band from the Meckel’s diverticulum to the
abdominal wall. Both of these events tend to occur later than the neonatal period.

52. A 6-week-old child presents with generalized seizures, a serum glucose of 30 mg/dL and concurrent
hyperinsulinemia. This child’s first priority is which of the following?

a. Permanent central venous access and glucose infusion


b. Administration of cortisone and adrenocorticotropic hormone
c. Computerized tomographic scan of the abdomen to look for an islet cell adenoma
d. Urgent pancreatic resection
Answer: a

Most cases of hyperinsulinemia in the first 2 years of life are due to nesidioblastosis, a condition associated with excessive
and diffuse formation of neoislets from primitive pancreatic ductal cells. Hyperinsulinemia secondary to islet cell adenoma
or carcinoma or islet cell hyperplasia is more common in the older child. Infants with nesidioblastosis, such as the one
described here, typically present with symptomatic hypoglycemia, seizures and hyperinsulinemia. An insulin-to-glucose
ratio (insulin in IU/ml divided by glucose in mg/dL) that is greater than 0.5 with fasting is highly suggestive.
Infants with nesidioblastosis are managed initially medically with maintenance of blood glucose levels above 40 mg/dL.
This is best carried out by the infusion of hypertonic glucose solutions through a permanent central venous catheter. In
addition, diazoxide, cortisone and adrenocorticotropic hormone and streptozocin have been used to treat the hypoglycemia.
Definitive management of nesidioblastosis may require pancreatic resection. This requires a pancreatectomy usually
estimated at approximately 95% to 99% with splenic and duodenal preservation. Following 90% to 95% pancreatectomy,
over 90% of infants with nesidioblastosis are rendered permanently euglycemic. A CT scan of the abdomen to search for an
adenoma is an appropriate diagnostic maneuver, but it is not the first priority.

53. At what age is surgical orchiopexy recommended for a child with a unilateral undescended testis?

a. Promptly upon discovery, regardless of age


b. 1 year
c. 5 to 6 years
d. Any time prior to puberty
Answer: b

Undescended testis occurs in 30% of premature boys, 3.4% of full-term boys, 0.8% of 1-year olds, and 0.8% of adults. The
implication is that if spontaneous testicular descent has not occurred by 1 year of age, it is unlikely to occur subsequently.
Therefore, orchiopexy is deferred until the patient reaches 1 year of age. Testes that remain undescended during childhood
have a significant reduction in the number of germ cells and have little function after puberty. Demonstrable histologic
changes are present by the age of 2 years. For this reason, delay beyond 1 year is not recommended.

54. An infant is noted to have a left flank mass shortly after birth and an ultrasound examination demonstrates left
hydronephrosis. The most common cause of this finding is which of the following?

a. Neonatal Wilm’s tumor


b. Congenital ureteropelvic junction obstruction
c. Multicystic dysplastic kidney
d. Vesicoureteral reflux
Answer: b

The most common cause of neonatal hydronephrosis is congenital ureteropelvic junction obstruction. It is important to rule
out bilateral ureteropelvic junction obstruction when this is found as this can present in 20% to 30% of neonatal cases. The
management requires establishment of dependent drainage of the renal pelvis, and this is usually accomplished by a
dismembered pyeloplasty with resection of the obstructing segment. A successful outcome is expected in over 90% of
patients.
Neonatal Wilm’s tumor is exceedingly rare and does not ordinarily present with obstruction of a functioning kidney, but
rather as a solid mass. Multicystic dysplastic kidney occurs in about 1 in 4000 births and is usually unilateral. It is the most
common form of renal cystic disease in infants, but these cysts do not communicate with a functional renal pelvis and there
is little or no functional renal cortex. Vesicoureteral reflux, while common, does not present in the newborn period with
hydronephrosis and a palpable kidney.

55. The medical indications for circumcision include which of the following?

a. Infants with a history of urinary tract infection


b. Hypospadias
c. Phimosis
d. Enuresis
e. Vesicoureteral reflux
Answer: a, c, e

The American Academy of Pediatrics has published guidelines on the indications for circumcision. Circumcision should be
encouraged in infants with a history of urinary tract infection or vesicoureteral reflux to decrease the chances of ascending
infection. On the contrary, in an infant with hypospadias, chordee, significant penoscrotal webbing, or other congenital
problems, circumcision should be discouraged to preserve the foreskin for use in later reconstruction. In normal infants,
circumcision is a matter of family choice and not an important issue for medical debate. Phimosis is a fibrotic contraction of
the preputial aperture so that retraction is impossible. Circumcision or dorsal slit are the most effective solutions to
phimosis. Enuresis is unrelated to the preputial skin and has no bearing on the decision for circumcision.

56. Which of the following statements regarding neuroblastoma are true?

a. Neuroblastoma is the most common abdominal malignancy of childhood


b. Approximately 80% of neuroblastoma patients are diagnosed prior to age 4 years
c. N-myc oncogene copy number in neuroblastoma tissue is inversely related to survival probability
d. Trk proto-oncogene expression in neuroblastoma tissue is inversely related to survival probability
e. All of the above
Answer: a, b, c

Neuroblastoma is the most common extracranial solid tumor and the most common abdominal malignancy of childhood.
The incidence is approximately 8 to 10 per million children under the age of 15 years. The incidence is uniform throughout
the world. This results in approximately 500 new cases reported each year in the United States. The median age at diagnosis
is about 2 years and 80% of children are less than four years of age at diagnosis. The N-myc oncogene, whose function and
mechanism of action remain the subject of investigation, was empirically shown to be a useful predictor of survival and risk.
It was found that patients with increased copies of the N-myc gene had a much worsened prognosis. Currently, most
authorities consider a copy number of more than ten to be significant. The trk proto-oncogene is a component of the high-
affinity nerve growth factor receptor and is expressed in human neuroblastoma tissue. Trk-A expression is inversely
correlated with N-myc amplification and is associated with lower stage at diagnosis and improved prognosis.

57. Which of the following are considered low risk features for neuroblastoma patients?

a. Age less than one year


b. Stage 2A and 2B disease (International Staging Criteria)
c. Stage 4S disease (International Staging Criteria)
d. Neuron specific enolase plasma level less than 100 ng/ml
e. None of the above
Answer: a, b, c, d

Risk status is determined in neuroblastoma patients by a number of characteristics which have been elaborated over the last
twenty years.

58. A one month old female infant is brought to you for evaluation of afriable polypoid mass prolapsing through the
vaginal introitus. Your presumptive diagnosis is which of the following?

a. Ectopic ureterocele
b. Rectal prolapse
c. Congenital adrenal hyperplasia with ambiguous genitalia
d. Embryonal rhabdomyosarcoma
Answer: d

Vaginal and cervical primary rhabdomyosarcomas often prolapse through the vaginal orifice as a friable polypoid mass and
may hemorrhage. Botyroid tumors are really of the embryonal subtype but grow into a hollow space such as the vagina or
bladder so they undertake a characteristic “grape-like” appearance. Other patterns of rhabdomyosarcoma include alveolar
and pleomorphic morphology. The incidence of rhabdomysarcoma is biphasic with one peak in infancy followed by the
second in the adolescent years. The nature of presentation is site dependent. This patient has a classical presentation for a
vaginal botyroid rhabdomyosarcoma. The emphasis in evaluation is to perform a thorough pretreatment workup that defines
completely local tumor extent and evaluates the regional and distant sites of metastases.

59. Which of the following approaches is considered standard care for most Wilms’ tumor patients in the United States
today?

a. Adriamycin and vincristine therapy followed by surgical resection


b. Needle biopsy followed by either chemotherapy or resection depending upon the histology
c. Primary surgical resection followed by chemotherapy
d. Radiation therapy if judged unresectable on CT or MRI imaging
Answer: c

The standard of care for Wilms’ tumor patients in the United States is initial surgical resection. Exceptions to this rule
include extensive intracaval tumors which require cardiopulmonary bypass for extraction, obviously unresectable tumors
with documented invasion of contiguous structures, and possibly bilateral tumors, especially if it is unclear which side is
most heavily involved. All resectable Wilms’ tumor patients receive postoperative chemotherapy with the possible exception
of Stage 1 favorable histology patients who are younger than 24 months of age at diagnosis and have tumors less than 250
grams in weight at resection. Chemotherapy followed by surgical resection is practiced in Europe with roughly equal
outcomes to those in the United States but this approach has the disadvantage of changing the surgical and pathologic
staging which are the basis for the National Wilms’ Tumor Studies and the cornerstone of treatment in the United States.
Needle biopsy has a very limited role for unusual presentations of Wilms’ tumor as the diagnosis is generally apparent with
modern imaging techniques. Radiation therapy is not a primary mode of therapy for Wilms’ tumor under contemporary
National Wilms’ Tumor treatment protocols.

60. Which of the following statements regarding rhabdomyosarcoma are true?

a. Surgical resection of the primary tumor results in cure of approximately 80 to 90% of all patients
b. Currently recommended therapy includes complete resection of primary tumors prior to chemotherapy for small
noninvasive lesions, or after documented response with more formidable primary tumors
c. Alveolar histology is a favorable prognostic finding
d. Overall survival of all patients is now approximately 50%
Answer: b, d

Surgical resection of the primary tumor was the mainstay of treatment 30 years ago for rhabdomyosarcoma but only resulted
in overall survival rates in the range of 20%. This improved to approximately 50% with the addition of chemotherapy.
Survival is stage dependent and if all cases (both high and low risk) are included, the overall survival from
rhabdomyosarcoma is now approximately 50%. Presently it is recommended that complete resection of primary tumors
should be undertaken either before chemotherapy for small noninvasive lesions or after documented response with more
formidable primary tumors. In certain situations where chemotherapy results in a complete or very good tumor regression,
external beam radiation may be employed as a primary means of local control. Debilitating or disfiguring surgery is only
performed if residual tumor is present after both chemotherapy and therapeutic irradiation. Alveolar histology is associated
with a particularly poor prognosis for rhabdomyosarcoma.

61. Patients with Wilms’ tumors most frequently present with which of the following?

a. Bilateral metachronous lesions


b. Bilateral synchronous lesions
c. An extrarenal primary
d. A multicentric primary lesion
e. A unifocal, unilateral lesion
Answer: e

It is currently hypothesized that Wilms’ tumor arises from primitive metanephric blastema and individual tumors often
contain not only primitive metanephric cells but also cartilage, skeletal muscle and squamous epithelium. Most tumors arise
unifocally within the kidney but approximately seven percent of unilateral Wilms’ tumors are multicentric. The proportion
of synchronous bilateral tumors among all nephroblastoma patients ranges from 4.4 to 7.0% while that of metachronous
tumors is 1.0–1.9%. Wilms’ tumors are equally distributed with regard to the left and right side and may occur with no
apparent connection to the kidneys. Usually, extrarenal Wilms’ tumor occurs in the retroperitoneal area but other reported
sites include pelvis, scrotum and inguinal region.

62. Hepatoblastomas are childhood liver tumors characterized by which of the following features?

a. Multicentricity
b. Cirrhosis in the uninvolved liver
c. Unresectable tumors subjected to cytoreductive chemotherapy may be resected with long-term survival
d. Jaundice
Answer: c

Children with hepatoblastoma most commonly present with an abdominal mass or diffuse abdominal swelling. The child is
typically in good health and the lesion may be observed by an observant parent or clinician on routine examination. Weight
loss and other symptoms are unusual. Liver function tests are usually normal or nonspecifically altered. Jaundice is
uncommon. The most useful tumor marker is the serum (a-fetoprotein (AFP)) level which is elevated in approximately 90%
of the cases.
Hepatoblastoma usually presents as a single, pseudo-encapsulated lesion often reaching large proportions before becoming
apparent. The umbilical fissure is generally not breached. Multicentricity occurs in less than 20% and cirrhosis of the
surrounding liver is unusual. Multicentricity and associated cirrhosis are typical of hepatocellular carcinoma. Complete
surgical resection remains the major objective of therapy for hepatoblastoma. At presentation approximately 60% of patients
with hepatoblastoma have resectable tumors. Chemotherapy is the major treatment option available for unresectable tumors.
Over the last decade it has become apparent that some of these patients may be rendered resectable by preoperative
chemotherapy.

63. Common sites of neuroblastoma metastasis are which of the following?

a. Lung
b. Regional lymph nodes
c. Bone marrow
d. Cortical bone
Liver
Answer: b, c, d

Neuroblastoma metastasizes to both regional lymph nodes and distant sites, most frequently bone marrow and/or cortical
bone. The liver and lungs are rarely the site of metastatic spread. Cortical bone involvement as manifested by a positive
bone scan is a particularly poor prognostic indicator. The majority of patients present with locally advanced disease at the
time of diagnosis.

64. Which of the following statements regarding renal tumors of childhood and adolescence are true?

a. Clear cell sarcoma is presently considered a variant of Wilms’ tumor with a poor prognosis
b. Clear cell sarcoma of the kidney has a high rate of metastasis to bone
c. Rhabdoid tumors may arise in the kidney, mediastinum or brain
d. Childhood rhabdoid tumors of the kidney carry an excellent prognosis
Answer: b, c

Clear cell sarcoma of the kidney is presently considered a distinct histopathologic and clinical entity from Wilms’ tumor. It
has a similar age distribution as that observed in Wilms’ tumor, but a markedly worsened prognosis. It is characterized by a
proclivity to metastasize to bones and indeed has been called the bone metastasizing renal tumor of childhood. Relapse and
death occur in 75% of patients with over half dying within one year of diagnosis. Aggressive systemic chemotherapy is
recommended for all stages of the disease. Likewise, postoperative radiation to the tumor bed is recommended regardless of
stage.
Rhabdoid tumors are rare malignancies that most commonly involve the kidney in childhood but may also occur primarily
in the mediastinum or brain. Outcome is particularly poor and there is no proven chemotherapy regimen. Rhabdoid tumors
of the kidney occur in infancy with a median age at presentation of 13 months. Survival rates are almost zero and even
Stage I patients fare poorly. Aggressive therapy is warranted including surgical resection, local radiation therapy and
systemic chemotherapy.

65. Which of the following syndromes are associated with the development of Wilms’ tumor?

a. Beckwith-Wiedemann Syndrome (hemi-hypertrophy, macroglossia, aniridia)


b. Neurofibromatosis
c. Denys-Drash syndrome (pseudohermaphroditism, glomerulopathy)
d. Gonadal dysgenesis
e. Hemolytic uremic syndrome
Answer: a, b, c, d

A number of syndromes are associated with the development of Wilms’ tumors and when present, require routine periodic
screening. There is no known association of the hemolytic uremic syndrome with the development of Wilms’ tumor.

1. The most ominous sign or symptom of urinary system disease is:


A. Urinary frequency.
B. Pyuria.
C. Pneumaturia.
D. Dysuria.
E. Hematuria.
Answer: E

DISCUSSION: While urinary frequency (voiding more than three to five times daily) or dysuria (painful voiding) may be a
sign of malignant disease, they are more commonly associated with nonmalignant inflammatory disease, neurologic disease,
or calculous disease of the urinary tract. Pyuria (pus in the urine) is most commonly associated with infection and not
malignancy. Pneumaturia (air or gas in the urine) indicates a fistula between bowel and the urinary tract or infection by
fermination in diabetic urine. Hematuria (blood in the urine) is most worrisome. While this may be produced by infection or
by calculous disease, it is most commonly associated with malignant disease in the absence of associated signs or symptoms
such as pyuria, frequency, and dysuria. Thus, of the ones mentioned, hematuria is the most ominous single sign or symptom.

2. A patient with acute urinary tract infection (UTI) usually presents with:
A. Chills and fever.
B. Flank pain.
C. Nausea and vomiting.
D. 5 to 10 white blood cells per high-power field (hpf) in the uncentrifuged urine specimen.
E. Painful urination.
Answer: E

DISCUSSION: Cystitis or infection of the bladder is the most common UTI. Lower UTI, or cystitis, is an infection in the
bladder. Painful urination and frequency are the most common presenting complaints. Hematuria may occur, but is
associated with painful urination and frequency. Flank pain, fever, chills, nausea, and vomiting usually occur only when the
infection involves the kidney. An acute UTI is identified in unspun urine only when there are more than 10 leukocytes per
hpf in the unspun urine. The normal urine may have as many as 10 WBC/per hpf without being infected.

3. Renal adenocarcinomas:
A. Are of transitional cell origin.
B. Usually are associated with anemia.
C. Are difficult to diagnose.
D. Are extremely radiosensitive.
E. Frequently are signaled by gross hematuria.
Answer: E

DISCUSSION: Renal adenocarcinomas arise from the renal tubular cells and not from the transitional cells that line the
collecting system of the kidney. Although one fifth of all patients with renal cancer may present with anemia, the most
common presenting symptom is hematuria, either gross or microscopic. Ultrasonography may confirm that a renal lesion is
either cystic or solid but computed tomography (CT) is probably the most accurate imaging study for diagnosing the
disease. Renal adenocarcinoma is little sensitive to current chemotherapeutic agents. Radiotherapy plays almost no role in
the management of the primary tumor. Operation is the treatment of choice when the disease is confined to the kidney itself
or when it has extended just outside the renal capsule. An operation has little effect once the disease is extended to adjacent
structures or to regional lymph nodes.
4. Ureteral obstruction:
A. Is associated with hematuria.
B. Is associated with deterioration of renal function and rising blood urea nitrogen (BUN) and creatinine values.
C. Is commonly caused by a urinary tract calculus.
D. Usually requires open surgical relief of the obstruction.
E. Is usually associated with infection behind the obstruction.
Answer: C

DISCUSSION: Ureteral obstruction produces loss of renal function when there is only one renal unit and the ureter is
obstructed or when obstruction is bilateral. Ureteral obstruction often is best identified by either intravenous pyelography
(IVP) or retrograde pyelography, which allows one to identify the specific site of obstruction. Calculous disease is the most
common cause of ureteral obstruction. Ureteral obstruction is not a surgical emergency that requires open surgical
intervention, but it may be relieved by retrograde or antegrade passage of a double-J stent to bypass the obstruction,
permitting orderly nonemergent identification of the cause of obstruction and selection of a treatment process.

5. Stress urinary incontinence:


A. Is principally a disease of young females.
B. Occurs only in males.
C. Is associated with urinary frequency and urgency.
D. May be corrected by surgically increasing the volume of the bladder.
E. Is a disease of aging produced by shortening of the urethra.
Answer: E

DISCUSSION: Stress urinary incontinence is seen principally in older females and is produced by pelvic floor relaxation
with shortening of urethral length. The symptom of stress urinary incontinence is urinary leakage produced by an increase in
intra-abdominal pressure, as with straining to lift or to laugh. Urgency and frequency are symptoms of urge incontinence,
not stress incontinence. Stress incontinence classically is not seen either in males or in young females who have good pelvic
floor support.

6. Which of the following is/are true of blunt renal trauma?


A. Blunt renal trauma and penetrating renal injuries are managed similarly.
B. Blunt renal trauma with urinary extravasation always requires surgical exploration.
C. Blunt renal trauma must be evaluated by contrast studies using either IVP or CT.
D. Blunt renal trauma requires exploration only when the patient exhibits hemodynamic instability.
E. Any kidney fractured by blunt renal trauma must be explored.
Answer: D

DISCUSSION: Blunt renal trauma should be explored. Only those who have gross hematuria need undergo contrast studies.
Microscopic hematuria is no longer an indication for contrast evaluation. Patients who have blunt renal trauma need to
undergo exploration only if they are hemodynamically unstable. Conservative management in the absence of hemodynamic
instability is the current trend. All penetrating injuries should undergo exploration.

7. Carcinoma of the bladder:


A. Is primarily of squamous cell origin.
B. Is preferentially treated by radiation.
C. May be treated conservatively by use of intravesical agents even if it invades the bladder muscle.
D. May mimic an acute UTI with irritability and hematuria.
E. Is preferentially treated by partial cystectomy.
Answer: D

DISCUSSION: Carcinoma of the bladder is primarily of transitional cell origin, arising from the transitional epithelium that
lines the bladder. It may be confused with an acute UTI by producing urgency, frequency, and hematuria. Bladder carcinoma
may be treated conservatively using intravesical agents if the tumor is intraepithelial in origin and does not invade through
the basement membrane. Neither radiation nor chemotherapy is the treatment of choice for disease that invades the muscle
of the bladder. Partial cystectomy may be chosen only when the disease is focal and there are no mucosal changes in other
parts of the bladder.

8. The major blood supply to the testes comes through the:


A. Hypogastric arteries.
B. Pudendal arteries.
C. External spermatic arteries.
D. Internal spermatic arteries.
Answer: D

DISCUSSION: Testes arise from portions of the wolffian bodies on the genital ridge close to the kidneys; therefore, the
major blood vessels from the testes arises from the aorta just below the renal arteries and are termed the internal spermatic
arteries. Secondary blood supply to the testes comes from the artery of the vas deferens, and a small branch from the
epigastric artery termed the external spermatic artery forms during descent of the testes from the abdomen to the scrotum.
The surgical importance of this phenomenon is that operations involving the region of the renal arteries may sacrifice the
internal spermatic artery. If the two other arteries are intact, the testes will survive; however, if the patient has had a
vasectomy and the artery of the vas has been sacrificed, there is a possibility of testicular atrophy, since the testicle will have
to be totally dependent on the arterial supply derived from the small external spermatic artery.

9. Patients who have undergone operations for benign prostatic hypertrophy or hyperplasia:
A. Require routine rectal examinations to detect the development of carcinoma of the prostate.
B. Do not need routine prostate examinations.
C. Have a lesser incidence of carcinoma of the prostate.
D. Have a greater incidence of carcinoma of the prostate.
Answer: A

DISCUSSION: Patients who have undergone operations for benign prostatic hyperplasia or hypertrophy have had only the
inner portion of the prostate removed, which consists of the periurethral glandular structures that give rise to hyperplasia
and hypertrophy. The posterior segment of the prostate, which is compressed by the anterior (inner) portion, comprises the
surgical capsule and is left behind. The posterior portion of the prostate gland is the most frequent site of origin of prostate
cancer. There is no difference in the incidence of carcinoma of the prostate in patients with benign prostatic hypertrophy and
those without benign prostatic hypertrophy or those who have and have not undergone operation for prostatic hypertrophy.
Since prostate carcinoma can develop at any time in a patient's life, routine examinations and prostate-specific antigen assay
are the most efficient methods of detecting this disease.

10. The male contribution to a couple's infertility is approximately:


A. 10%.
B. 25%.
C. 50%.
D. 75%.
Answer: C

DISCUSSION: In the United States of America it has been estimated that approximately 15% of couples have difficulty
with conception. Adequate evaluation of the marital unit for infertility demands assessment of the male partner since
infertile status may be attributed to the male as much as 50% of the time. A full evaluation of the male partner is important
to avoid extended fruitless evaluation and management of the female partner when the male is infertile.

11. To maximize fertility potential, orchidopexy for cryptorchidism should be done before:
A. Age 15 years.
B. Age 12 years.
C. Marriage.
D. Age 2 years.
Answer: D
DISCUSSION: The testes are exquisitely sensitive to temperature; therefore there is progressive deterioration of testes that
are not within the scrotum. Cryptorchid testes, whether they be in the inguinal canal, in an intra-abdominal position, or in an
ectopic position, will undergo progressive spermatogenic failure, although adequate amounts of androgens may be produced
and secreted. The timing of orchidopexy has been moved progressively backward, and now the recommendation is that
orchidopexy should be accomplished before age 2 years, to maximize the possibility of production of spermatozoa of
normal quantity and quality. In cases of unilateral cryptorchidism the matter of surgical exploration is less critical; however,
to provide maximum potential for both testes, the earlier cryptorchidism is surgically corrected the better are the chances for
normal spermatogenesis.

12. Within the age group 10 to 35 years, the incidence of carcinoma of the testis in males with intra-abdominal testes is:
A. Equal to that in the general population.
B. Five times greater than that in the general population.
C. Ten times greater than that in the general population.
D. Twenty times greater than that in the general population.
Answer: D

DISCUSSION: The incidence of carcinoma of the testis is greater in patients who have cryptorchidism, whether corrected
or not; because of this, routine self-examination by patients who have undergone operation for cryptorchidism is important.
For patients who have uncorrected intra-abdominal testes it is estimated that the incidence of the development of carcinoma
of the testis in the age group 10 to 35 years is approximately 20 times greater than that for the general population. If
cryptorchidism is diagnosed after the age of 10 to 12 years, orchiectomy may be the preferred treatment, since such testes
rarely exhibit normal function, despite adequate scrotal placement, and put the patient at great risk for an intra-abdominal
neoplasm that will be difficult to diagnose.

13. The appropriate surgical treatment for suspected carcinoma of the testis is:
A. Transscrotal percutaneous biopsy.
B. Transscrotal open biopsy.
C. Repeated examinations.
D. Inguinal exploration, control of the spermatic cord, biopsy, and radical orchectomy if tumor is confirmed.
Answer: D

DISCUSSION: If, after physical examination, and even scrotal ultrasound, a tumor of the testicle is still suspected, the
appropriate surgical treatment is high inguinal exploration with control of the cord, delivery of the testicle onto a protected
field, biopsy if necessary, and then orchiectomy at the level of the internal ring if tumor is confirmed. Transscrotal
manipulations, whether they be percutaneous or open, are to be condemned because of the possibility of tumor spillage with
the ultimate necessity for hemiscrotectomy to control local recurrence. Certainly, repeated examinations over a very short
period of time are appropriate, but no time should be lost if there is true suspicion of a testicular tumor. Before the high
inguinal exploration it is helpful to obtain serum levels of the beta subunit of human chorionic gonadotropin and alpha-
fetoprotein, which are important tumor markers. Surgical exploration should not be delayed until the actual laboratory
values are determined, as they are important to the longitudinal course of the patient and not necessarily to the diagnosis.

14. If torsion of the testicle is suspected, surgical exploration:


A. Can be delayed 24 hours and limited to the affected side.
B. Can be delayed but should include the asymptomatic side.
C. Should be immediate and limited to the affected side.
D. Should be immediate and include the asymptomatic side.
Answer: D

DISCUSSION: Torsion of the testicle should be corrected as soon as possible after the diagnosis is entertained. Incomplete
torsion can cause partial strangulation, the effects of which may be overcome if surgical intervention is accomplished within
12 hours, whereas severe torsion with complete compromise of the blood supply results in loss of the testis unless surgical
intervention occurs within approximately 4 hours. The contralateral scrotum should also be explored at the time of the
operation, since the primary anatomic defect—insufficient attachment of the testicle to the scrotal sidewall—most often is a
bilateral phenomenon. If the contralateral scrotum is not explored, the patient runs a very high risk of undergoing torsion on
the other side and the possible complication of loss of both testes.
15. Epididymitis, either unilateral or bilateral, in a prepubertal male:
A. Is a frequent diagnosis.
B. Can be dealt with on an outpatient basis.
C. Is a major scrotal problem in this age group.
D. Is a rare phenomenon.
Answer: D

DISCUSSION: Epididymitis can occur in prepubescent males, but it is a rare phenomenon and usually occurs only in
patients with chronic UTI, obstructed urethra, or very high voiding pressure. The diagnosis of epididymitis in the
prepubertal male should be reviewed with suspicion because one of the more common causes of the clinical situation that
presents as epididymitis is torsion of the testicle. If there is any concern about the validity of the diagnosis, the patient
should undergo scrotal exploration. Epididymitis will not be compromised by surgical exploration, but delay in surgical
exploration leads to loss of the testicle if the problem is torsion.

16. Patients with prostatitis, especially acute suppurative prostatitis:


A. Should have residual urine measured by intermittent catheterization.
B. Should have bladder decompression by urethral catheter.
C. Should have repeated prostatic massage.
D. Should have no transurethral instrumentation if possible.
Answer: D

DISCUSSION: Acute suppurative prostatitis should be treated with vigorous antibiotic therapy with broad-spectrum agents
initiated immediately and changed in response to results of culture and sensitivity studies. Urethral instrumentation and
repeated prostate examination should not be done, if at all possible, since sepsis is not unusual after either diagnostic
examination or urethral catheterization. If the patient does need to have the bladder decompressed, it is beneficial to use a
suprapubic catheter rather than a urethral catheter.

17. Benign prostatic hypertrophy with bladder neck obstruction:


A. Is always accompanied by significant symptoms.
B. Is best diagnosed by endoscopy and urodynamic studies.
C. Is easily diagnosed by the symptoms of frequency, hesitancy, and nocturia.
D. Is always accompanied by residual urine volume greater than 100 ml.
Answer: B

DISCUSSION: Benign prostatic hypertrophy with bladder neck obstruction is difficult, in some patients, to diagnose as they
are totally asymptomatic, even if they have residual urines of greater than 1000 ml. or renal compromise consisting of the
syndrome of so-called “silent prostatism.”

18. Which of the following statements are true concerning male infertility?

a. Although 15% of couples in the United States are affected by infertility, the male rarely contributes to the problem
b. A varicocele can be associated with diminished sperm motility and abnormal sperm morphology
c. Complete testicular failure will usually respond to systemic testosterone administration
d. Anti-sperm antibodies are an important cause of infertility which may be treated successfully with corticosteroid
administration
Answer: b, d

Infertility is defined as the inability to conceive a pregnancy within one year of unprotected intercourse. About 15% of
couples in the United States are affected, and in about 25%-50% of infertility cases, the male contributes to the problem.
The cornerstone of male fertility evaluation is the semen analysis. Oligospermia, or a low sperm count, is an incomplete
form of testicular failure due to a number of causes. A varicocele is found in about 15% of the general male population, but
40% of infertile men have this finding. Men with a varicocele can exhibit low sperm counts but more often have diminished
sperm motility and abnormal morphology. Surgical ligation or angiographic embolization of the internal spermatic vein
improves the semen parameters in 50%-70% of these men and gives subsequent pregnancy rates of 25%-50%. Complete
testicular failure is diagnosed by a testis biopsy showing no sperm production or by a markedly elevated serum FSH level,
indicating the absence of negative feedback inhibition induced by spermatogenesis. Complete testicular failure is not
remedial by treatment. Anti-sperm antibodies are found frequently in infertile men and represent an important cause of
infertility. Corticosteroid administration may be helpful if antibodies are present, but the toxicity of these medications
cannot be ignored.

19. A 65-year-old male is diagnosed as having prostatic cancer based on transrectal biopsy of a 1 cm palpable nodule.
Which of the following statement(s) are true concerning his management?

a. If the tumor is confined within the prostatic capsule (stage A or B), radical prostatectomy is an appropriate option
b. If positive lymph nodes are detected on laparoscopic pelvic lymph node dissection (stage Dl), radical
prostatectomy is indicated
c. Radical prostatectomy is invariably associated with impotence
d. External beam radiation is an appropriate treatment if the tumor is confined to the prostate
e. There is currently no role for orchiectomy in the management of prostatic cancer
Answer: a, d

The treatment of prostatic cancer depends on whether the disease is localized to the prostate or advanced beyond the gland.
Because prostate cancer advances slowly, the morbidity of therapy may exceed the therapeutic benefit in the elderly and
debilitated. Patients who have a limited life expectancy and low stage disease are frequently treated with observation only.
If the tumor is confined within the prostatic capsule (Stage A or B), options include radical prostatectomy, external beam
radiation therapy, and radioactive implants. Radical prostatectomy is usually carried out through the retropubic approach.
Through this approach a node dissection can be done for further staging, and the procedure abandoned if the nodes contain
tumor. In patients with a high index of suspicion for positive nodes, a laparoscopic pelvic node dissection can be performed
to decrease postoperative morbidity. The use of the nerve-sparing prostatectomy can be used to preserve penile erection in
those patients who are potent. In this approach, the nerves concerned with penile erection are excluded from the dissection.
The incidence of impotence following traditional radical prostatectomy is l00% but can be cut in half with the nerve-sparing
approach. Hormonal ablation is the initial treatment of choice for advanced prostatic cancer. Most prostatic cancers are
androgen-responsive. Androgen ablation will cause improvement in 80-90% of patients with regression of tumor in about
40%. The testis is the primary source of androgen and orchiectomy remains the gold standard and treatment of choice for
advanced prostatic cancer. Estrogen will produce castrate levels of testosterone, but the side effects of fluid retention and
increased incidence of thromboembolic diseases such as heart attacks and strokes make this hormone a poor choice in this
high risk age group.

20. Extracorporeal shock wave lithotripsy (ESWL) has had a dramatic effect on the management of urinary stones. Which of
the following statement(s) are true concerning shock wave lithotripsy of urinary stones?

a. The basic principle of lithotripsy involves the generation of shock waves which are focused fluoroscopically on the
calculus and are delivered to the patient who is submersed in a water bath
b. The most common complication after lithotripsy is ureteral obstruction secondary to stone fragments
c. ESWL can be associated with stone-free rates ranging between 60%-95% at six months for renal and proximal
ureteral stones
d. The combination of ESWL with percutaneous nephrolithotripsy improves the results for stone clearance in patients
with large or branched stones such as staghorn calculi
Answer: a, b, c, d

The introduction of ESWL has virtually eliminated open surgery for renal and ureteral lithiasis. The basic principles of all
lithotriptors include shock wave generation, focusing of the sound wave, and imaging of the stone. All lithotriptors produce
shock waves by a spark gap electrode or by a piezoelectric or electromagnetic element. The wave is then focused towards
the stone which is localized either employing fluoroscopy or ultrasonography. The patients are either submersed in a water
bath or “coupled” by a water cushion. The acoustic density of water and body tissues is essentially the same. Therefore,
there is little or no impedance of the shock wave at the water-body interface. Upon striking the stone, which is of different
acoustical density, the shock wave undergoes reflection and refraction, resulting in compressive and tensile forces which
fragment the stone.
Complications of ESWL are rare. The most common complication after ESWL is ureteral obstruction secondary to stone
fragments requiring either additional ESWL, urethroscopic stone retrieval or stent placement. ESWL is the treatment choice
for the vast majority of renal and proximal ureteral stones with stone-free rates ranging from 60%–95% at six months.
Stones larger than 3 cm and branch stones such as staghorn calculi are best treated with percutaneous nephrolithotripsy
alone or in combination with ESWL. The combination of extracorporeal and percutaneous techniques can result in average
dome clearance rates in excess of 80%.

21. Which of the following statement(s) are true concerning bladder carcinoma?

a. Epidemiologic studies have implicated cigarette smoking as a risk factor


b. If cystoscopy demonstrates a bladder carcinoma as the cause of painless hematuria, no further evaluation is
necessary
c. Multi-focal and recurrent bladder tumors are usually treated with transurethral resection and intravesical
chemotherapy
d. The results of treatment for locally advanced bladder tumors are similar with either radical cystectomy or radiation
therapy
Answer: a, c

A wealth of basic research and clinical data testify to a variety of chemical carcinogens inducing bladder cancer.
Occupational exposure to beta-naphthylamine and para-aminophenyl results in an increased incidence of bladder cancer.
Epidemiologic studies have also indicated cigarette smoke as a risk factor. Bladder cancer has a strong male prevalence and
is almost three times more common in men than women. The hallmark of bladder cancer is painless, total gross hematuria.
The usual diagnostic tests employed are excretory urography (IVP) and cystoscopy. The former is important because the
upper tracts (renal pelvises and ureters) are also at risk for the development of urothelial neoplasia. Cystoscopy is not only
diagnostic but also therapeutic because superficial tumors are easily excised or fulgurated through endoscopic instruments.
Approximately 70% of patients with bladder cancer will present with local disease. This is associated with five year
adjusted survival rate of 88%. Close vigilance is important because the recurrence rate exceeds 50%. Ten to 50% of
superficial tumors will progress to invasive disease. Multifocal and recurrent tumors are usually treated with intravesical
chemotherapy in addition to transurethral resection. Agents commonly employed include thiotepa, doxorubicin, and
mitomycin C. Alternatively intravesical immunotherapy has been successfully performed with installation of BCG (Bacillus
Calmette-Guerin). Locally advanced tumors are usually treated with radical cystectomy and urinary diversion. Radiation
therapy has been employed but is associated with a high rate of local recurrence.

22. The most common malignant neoplasm of the kidney is the hypernephroma or renal cell carcinoma. Which of the
following statement(s) are true concerning renal neoplasms?

a. Renal cell carcinomas can produce a variety of hormone or hormone-like substances


b. Bilateral multifocal renal cell cancers can be associated with the multiple endocrine neoplasia syndrome
c. A “tumor deformity” on IVP is diagnostic of a renal cell carcinoma
d. Early control of the renal pedicle is an important aspect of surgical management of renal cell carcinoma
e. Patients with renal cell carcinoma in a solitary kidney will inevitably require total nephrectomy and long-term
dialysis for the resultant renal failure
Answer: a, d

Renal cell carcinoma or hypernephroma account for approximately 2% of all cancers diagnosed annually. It is most
common after the fifth decade of life and has a male to female ratio of approximately 2:1. No definite etiology has been
identified, but a frequent genetic abnormality detected in renal cell cancer is the loss of heterozygosity of chromosome 3p.
Multifocal bilateral tumors are associated with von Hippel-Lindau disease. Renal carcinomas can produce a variety of
hormone or hormone-like substances (e.g., erythropoietin, renin, and parathormone) and may present with a variety of
symptoms including anemia, hypertension, fever and erythrocytosis. Excretory uroraphy (IVP) provides a good renal image
with superior detail of the collecting system. Renal masses such as benign cysts or renal cell carcinomas will both appear as
“tumor deformities”, distorting the renal outline or the collecting system. Renal cysts are far more common than renal cell
carcinoma and the diagnosis can be confirmed by renal ultrasound. Surgical excision remains the primary mode of treatment
for renal cell carcinoma. Although the need for radical nephrectomy has recently been questioned, this procedure remains a
gold standard against which less radical procedures must be judged. Radical nephrectomy is performed through an
abdominal or a thoracoabdominal approach and involves early control of the renal artery and vein. The tumor, together with
the kidney and the perirenal fat is excised within Gerota’s fascia which is not opened. Less radical approaches have been
suggested for the treatment of smaller tumors, including partial nephrectomy. This approach is especially valuable for
bilateral tumors or in patients with a solitary kidney or poor overall renal function.
23. A 28-year-old white male presents with asymptomatic testicular enlargement. Which of the following statement(s)
is/are true concerning his diagnosis and management?

a. Tumor markers, b-fetoprotein (AFP) and ‫ك‬-human chorionic gonadotropin (HCG) will both be of value in the
patient regardless of his ultimate tissue type
b. Orchiectomy should be performed via scrotal approach
c. The diagnosis of seminoma should be followed by postoperative radiation therapy
d. With current adjuvant chemotherapy regimens, retroperitoneal lymphadenectomy is no longer indicated for non-
seminomatous testicular tumors
Answer: c

Testis cancer is most common between the ages of 25 and 34 and is rare in blacks. The most common malignant neoplasm
of the testis arise from the germ cells and can represent a variety of histologic manifestations, e.g, choriocarcinoma,
embryonal cell carcinoma, seminoma, and teratoma. For therapeutic purposes, the tumors can be divided into seminomas
and nonseminomas. The usual presenting symptom is testicular enlargement that may be associated with mild discomfort.
Any solid testicular mass should be considered suspicious for testis carcinoma. The diagnostic and therapeutic approach for
any suspected testis carcinoma is inguinal exploration with orchiectomy if the operative findings confirm the presence of a
testicular mass. The inguinal approach is employed to perform high ligation of the cord at the inguinal ring and to eliminate
potential involvement of the inguinal lymph nodes which are the primary area of drainage for the scrotum. The tumor
markers, a-fetoprotein (AFP) and the b-human chorionic gonadotropin (HCG) can contribute to both diagnosis and follow-
up of testis cancer. Tumor markers are helpful when obtained prior to and following orchiectomy to help in assessing the
stage of the tumor. Pure seminoma does not cause elevated AFP but can produce a moderate rise in HCG in 10% of patients.
Seminomas are very responsive to radiation. Patients with minimal to moderate tumor burden (Stage I or II) are usually
treated with radiotherapy. The field of treatment encompasses the para-aortic and para-caval areas below the diaphragm and
ipsilateral inguinal and pelvic areas. When bulky retroperitoneal and/or distant metastases are present, cisplatin-based
combination chemotherapy is the preferred treatment. The treatment of non-seminomatous tumors is more controversial.
Stage I tumors are effectively treated with retroperitoneal lymphadenectomy. If bulky stage II and stage III non-
seminomatous tumors are present, initial treatment includes cisplatin-based chemotherapy. Evidence for residual disease
with normalization of tumor markers is usually an indication for surgical exploration.

24. Which of the following statement(s) is/are true concerning benign prostatic hypertrophy (BPH)?

a. Prostatic size has no consistent relationship to urethral obstruction


b. Renal failure secondary to obstructive uropathy occurs as bladder pressure rises and is eventually transmitted
proximally to the renal pelvis
c. Hormonal treatment for BPH involves treatment with a 5 a-reductase inhibitor which blocks the conversion of
testosterone to the dihydrotestosterone
d. Intermittent catheterization, although a temporizing measure, is not an effective treatment for relief of symptoms of
BPH
Answer: a, b, c

The prototypic bladder outlet obstruction is prostatic hyperplasia, which urologists once visualized as a progressive
encroachment on the urethral lumen related to prostatic growth. It is now clear that prostatic size has no consistent
relationship to obstruction and the diagnosis of obstructive uropathy cannot be made by endoscopic inspection or by
determination of prostatic size or appearance. Obstruction results in progressive increases in bladder pressure and decreased
urine flow rates. If bladder pressures are high enough and sustained long enough, the ureteral pump mechanism is
overcome, the ureter dilates, and by a hydraulic mechanism, intervesicular pressure is transmitted to the renal pelvis. At a
pressure of 42–50 cm H2O, glomerular filtration ceases. These relatively simple sequential events lead to renal failure.
Prostatic enlargement clearly has an endocrine basis since treatment with a 5 a-reductase inhibitor, which blocks conversion
of testosterone to dihydrotestosterone (the active male hormone in the prostate) can induce a 30% to 50% regression in
prostatic size. Although surgery or hormone therapy may be effective in initiating reversal of changes associated with
obstructive uropathy, this does not occur invariably. Removal of the hyperplastic glandular tissue is the most effective
treatment in terms of relief of symptoms. Patients who cannot be subjected to operation, however, show the same response
to intermittent catheterization and periodic bladder emptying in terms of symptoms as well as bladder wall and pressure
changes.
25. A 55-year-old male presents with severe flank pain radiating to the groin associated with nausea and vomiting.
Urinalysis reveals hematuria. A plain abdominal film reveals a radiopaque 5 mm stone in the area of the ureterovesical
junction. Which of the following statement(s) is/are true concerning this patient’s diagnosis and management?

a. A likely stone composition for this patient would be uric acid


b. The stone will likely pass spontaneously with the aid of increased hydration
c. Stone analysis is of relatively little importance
d. Patients with a calcium oxalate stone and a normal serum calcium level should undergo further extensive metabolic
evaluation
Answer: b

It is estimated that 12% of the U.S. population will develop calculus disease during their lifetime. Males have more than
twice the rate of stone formation than females. Caucasians have between a two to tenfold higher incidence of renal stone
disease than Blacks or Asians. The peak incidence of lithiasis appears to be between the ages of 45 and 64 years. Almost 3/4
of stones are composed of calcium oxalate in combination with calcium phosphate. Magnesium ammonium phosphate
(struvite) or infection stones make up approximately 12% whereas pure calcium phosphate and uric acid stones each
compromise 7%. The diagnosis of renal stones is made with appropriate history and performance of urinalysis and a non-
contrast abdominal radiograph. Urinalysis of a patient with a urinary stone will have evidence of either gross or microscopic
hematuria in 85%-95% of patients. Eighty-five to 90% of urinary stones are radio-opaque. Uric acid stones are typically not
radio-opaque.
The majority of stones will pass spontaneously with aid of increased hydration and appropriate analgesics. All stones passed
should be retrieved for subsequent analysis. Patients passing their first stone should have serum calcium and creatinine
levels and a urinalysis in addition to stone analysis. If the stone is calcium oxalate and the serum calcium level is normal, no
further evaluation is necessary other than encouraging the patient to increase fluid intake. Any patient with stones composed
of uric acid, pure calcium phosphate, cystine, or struvite are at high risk for continued stone formation and should undergo
more extensive metabolic evaluation. In addition, those patients with recurrent or enlarging stones, including those patients
with known calcium oxalate stones, should undergo a metabolic evaluation.

26. Which of the following statements are true concerning male impotence?

a. Psychologic factors account for less than half the cases of male impotence
b. Vascular testing for vasculogenic impotence may include Doppler determination of penile systolic blood pressure
and super selective pelvic arteriography
c. Penile implants are the first line treatment for patients with impotence due to diabetes or vascular dysfunction
d. Impotence associated with abdominal perineal resection is due to direct trauma to pelvic nerves and may be
improved with papaverine injection
Answer: a, b, d

Erectile dysfunction is a common condition that affects 10 million American men. The incidence increases with age. By age
55 about 8% of men are affected. By the age of 80 years, the incidence is 75%. Impotence ensues from interference with the
normal vascular, neurologic, psychological, endothelial, and hormonal mediators of erection. In many cases, the causes are
multi-factorial. Psychological factors can inhibit as well as stimulate erection and account for less than half of the cases of
impotence. Although a number of systemic diseases can cause impotence, diabetes is the most common. Impotence may
also result from systemic neurologic diseases such as multiple sclerosis. Direct trauma to the pelvic nerves by pelvic
fractures of radical pelvic surgery (radical prostatectomy, abdominal perineal resection) may also be associated with
impotence.
The determination of the effect of vascular disease on impotence can be determined through a number of techniques. An
estimate of penile blood flow can be made through Doppler determination of penile systolic blood pressure using a penile
cuff. Direct corporal injection with papaverine, a smooth muscle relaxant, bypasses psychogenic and neurologic factors and
produces an erection if the blood flow to the penis is normal. If arterial disease is suspected on the basis of poor response,
superselective pelvic arteriography with injection of vasoactive agents is necessary to document the nature of the disease.
The treatment of impotence depends on both the cause and the patient’s willingness to pursue various therapeutic
approaches. Patients with neurogenic impotence, such as following pelvic nerve injury, can experience dramatic results with
papaverine injection. Penile implants can be used to treat any type of intractable impotence, but they are usually reserved for
patients with diabetes or vascular neurologic dysfunction who do not respond to conservative measures.

27. Which of the following statement(s) are true concerning the detection and diagnosis of prostatic cancer?
a. An elevation of prostate specific antigen (PSA) is highly sensitive and specific for prostatic carcinoma
b. American blacks have an increased risk of prostatic carcinoma
c. Autopsy series would suggest that 10% of men in their 50’s will have small latent prostatic cancers
d. Transrectal prostatic biopsy is indicated for a palpable 1 cm prostate nodule
e. Serum prostatic acid phosphatase remains the most useful tumor marker for prostatic carcinoma
Answer: b, c, d

Adenocarcinoma of the prostate is the most common non-cutaneous malignant tumor in men, accounting for 20% of all
male cancers and is the second highest cause of cancer deaths in males. It is primarily a disease of older men. At autopsy,
about 10% of men in their 50’s can be shown to have small latent tumors, and with this number increasing to 70% of men in
their 80’s. However, it is estimated that only 10% of men over 65 will develop clinically significant prostate cancer. An
increased incidence in American blacks has been reported.
Early prostate cancer has few symptoms. Therefore, early diagnosis requires detection of small tumors within the prostate
gland. Three modalities are used in the early detection of prostate cancer. These include digital rectal examination, serum
prostate specific antigen (PSA), and transrectal ultrasound of the prostate. Prostate tumors usually arise in the posterior lobe
of the prostate an area readily palpable on digital rectal examination. Early prostatic cancer frequently presents as a small
firm nodule within or at the periphery of the gland. If a 1 cm nodule is detected, it is cancer about 50% of the time. Prostatic
biopsy is readily performed with little morbidity and is often required to confirm the diagnosis. Transrectal ultrasound of the
prostate may also detect prostate cancer often as a smaller more subtle lesion not easily discernable on rectal examination.
However, digital examination will also disclose some cancers that are not visualized with ultrasound. Serum PSA is used to
aid in the early detection of prostate cancer. PSA is elevated in 68% of men with cancer but 33% of men with benign
enlargement of the gland also have an enlarged PSA. Serum prostatic acid phosphatase is not specific for prostatic cancer
although a significant elevation is usually associated with metastatic disease. Serum acid phosphatase however has been
generally replaced as a tumor marker by the immunoassay for PSA. PSA is also an extremely sensitive tumor marker for
recurrences after surgery because serum levels should be undetectable if patients are tumor-free.

Anda mungkin juga menyukai